0% found this document useful (0 votes)
435 views528 pages

DDSEP 9 MCQs

The document discusses various aspects of esophageal disorders, particularly focusing on gastroesophageal reflux disease (GERD) and its mechanisms, including transient lower esophageal sphincter relaxations (TLESRs) and the role of gastric distension. It presents clinical scenarios and questions related to the diagnosis and management of esophageal conditions, highlighting the importance of factors such as obesity, anxiety, and the presence of an acid pocket in reflux episodes. The document also emphasizes the significance of biopsy-proven intestinal metaplasia as a definitive indicator of GERD.

Uploaded by

dr.shimaaibrahim
Copyright
© © All Rights Reserved
We take content rights seriously. If you suspect this is your content, claim it here.
Available Formats
Download as PDF, TXT or read online on Scribd
0% found this document useful (0 votes)
435 views528 pages

DDSEP 9 MCQs

The document discusses various aspects of esophageal disorders, particularly focusing on gastroesophageal reflux disease (GERD) and its mechanisms, including transient lower esophageal sphincter relaxations (TLESRs) and the role of gastric distension. It presents clinical scenarios and questions related to the diagnosis and management of esophageal conditions, highlighting the importance of factors such as obesity, anxiety, and the presence of an acid pocket in reflux episodes. The document also emphasizes the significance of biopsy-proven intestinal metaplasia as a definitive indicator of GERD.

Uploaded by

dr.shimaaibrahim
Copyright
© © All Rights Reserved
We take content rights seriously. If you suspect this is your content, claim it here.
Available Formats
Download as PDF, TXT or read online on Scribd
You are on page 1/ 528

Answers & critiques

CHAPTER 1

Esophageal disorders
C. Prakash Gyawali, MD, MRCP, AGAF and
Vani Konda, MD

Question 1 al. Mechanism of gastroesophageal reflux in


Which of the following settings is associated with patients with obstructive sleep apnea syndrome.
an increase in the frequency of transient lower Neurogastroenterol Motil. 2010 Jun;22(6):
esophageal sphincter relaxations (TLESRs)? 611-e172.
Hershcovici T, Mashimo H, Fass R. The
A. Baclofen administration lower esophageal sphincter. Neurogastroenterol
B. Esophageal outflow obstruction Motil. 2011 Sep;23(9):819-30.
C. Gastric acid hypersecretion
D. Lean body mass
E. Obstructive sleep apnea Question 2
A 55-year-old woman complains of daily
CORRECT ANSWER: E retrosternal burning mainly after the evening
meal, and incompletely resolved with once a
RATIONALE day PPI therapy. Medical comorbidities include
Transient lower esophageal sphincter relaxation hypertension treated with hydrochlorothiazide,
(TLESR) is a physiologic phenomenon that al- bipolar disorder treated with lamotrigine, and
lows venting of swallowed air from the stomach, glaucoma treated with pilocarpine. Past surger-
in response to distension of the proximal stom- ies include cholecystectomy and partial thyroid-
ach. Patients with GERD typically reflux gastric ectomy. An upper endoscopy is normal without
content through a compliant esophagogastric erosive esophagitis or hiatus hernia. A wireless
junction into the esophagus during a TLESR; ambulatory pH study performed off acid sup-
the frequency of TLESRs may also be higher in pression demonstrates acid exposure time of 6.5
patients with GERD. TLESRs are suppressed percent, with multiple reflux episodes clustered
during deep sleep, and are less frequent when around postprandial periods, and no nocturnal
LES relaxation is abnormal (e.g. esophageal out- reflux episodes.
flow obstruction). Baclofen, a GABA B receptor
agonist, can reduce TLESR frequency, and can Which of the following pathophysiologic factors
reduce reflux episodes in patients with reflux. directly contributes to her elevated esophageal
Obese patients and those with obstructive sleep acid exposure?
apnea can have increased frequency of TLESRs.
The frequency of TLESR is not related to degree A. Bipolar disorder
of gastric acid secretion in the stomach. B. Increased gastric acid production
C. Increased saliva production
REFERENCES D. Post cholecystectomy state
Kuribayashi S, Kusano M, Kawamura O, et E. Postprandial gastric distension

1
2 Digestive Diseases Self-Education Program®

CORRECT ANSWER: E some instances. Therefore, when an acid pocket


exists, the volume of acidic reflux is increased dur-
RATIONALE ing a transient LES relaxation (TLESR). An acid
Proximal gastric distension is a prime trigger for pocket does not increase gastric acid production
transient lower esophageal sphincter relaxation or increase hiatus hernia size. Gastric emptying
(TLESR), the most frequent mechanism for GERD. is independent of the presence of an acid pocket.
Since her symptoms occur following the eve- Finally, an acid pocket does not influence TLESR
ning meal, and the pH monitoring demonstrates frequency, but a hiatus hernia and gastric fundic
clustering of reflux episodes around postpran- distension can increase TLESR frequency.
dial periods, it is likely that postprandial gastric
distension causing increased TLESR frequency is REFERENCES
an important mechanism for her persisting reflux Herregods TV, Bredenoord AJ, Smout AJ. Patho-
symptoms. Bipolar disorder does not result in physiology of gastroesophageal reflux disease: new
elevated acid exposure time. Gastric acid produc- understanding in a new era. Neurogastroenterol
tion is not increased in typical GERD. A decrease Motil. 2015 Sep;27(9):1202-13.
in saliva production can increase esophageal acid Beaumont H, Bennink RJ, de Jong J, Boeck-
dwell time, since saliva neutralizes refluxed acid. xstaens GE. The position of the acid pocket as
There is no direct correlation between post chole- a major risk factor for acidic reflux in healthy
cystectomy state and reflux disease. subjects and patients with GORD. Gut. 2010
Apr;59(4):441-51.
REFERENCE:
Gyawali CP, Roman S, Bredenoord AJ, et al. Clas-
sification of esophageal motor findings in gastro- Question 4
esophageal reflux disease: Conclusions from an Which of the following conditions is associated
international consensus group. Neurogastroen- with increased perception of reflux episodes?
terol Motil. 2017 Dec;29(12).
A. Anxiety disorder
B. Barrett’s esophagus
Question 3 C. Diabetes mellitus
How does an acid pocket increase reflux potential? D. Older age
E. Sleep
A. By delaying gastric emptying
B. By increasing acid reflux during TLESR CORRECT ANSWER: A
C. By increasing gastric acid production
D. By increasing hiatus hernia size RATIONALE
E. By increasing TLESR frequency Esophageal perception is enhanced in patients
with affective disorders including anxiety, sleep
CORRECT ANSWER: B deprivation, stressful states and hypervigilance.
In contrast, the elderly have reduced esophageal
RATIONALE perception, and can have advanced esophageal
In the post prandial state, gastric acid produced erosive disease with limited symptoms. Barrett’s
by the stomach layers on top of the ingested meal. esophagus and diabetes mellitus are also associ-
This pool of acidic gastric content, termed an acid ated with reduced esophageal perception.
pocket, is in close proximity to the esophagogas-
tric junction, and can extend into a hiatus hernia REFERENCES
or even the distal most part of the esophagus in Weijenborg PW, Smout A, Krishnadath KK, et
Chapter 1 — Esophageal disorders 3

al. Esophageal sensitivity to acid in patients with ence of reflux or motor disorders as a mechanism
Barrett’s esophagus is not related to preserved for symptoms. A barium esophagram does not add
esophageal mucosal integrity. Neurogastroenterol much to the evaluation of chest pain in the ab-
Motil 2017;29. sence of dysphagia.
Fass R, Naliboff BD, Fass SS, et al. The effect
of auditory stress on perception of intraesophageal REFERENCES
acid in patients with gastroesophageal reflux dis- Roman S, Gyawali CP, Savarino E, et al. Ambula-
ease. Gastroenterology. 2008 Mar;134(3):696-705. tory reflux monitoring for diagnosis of gastro-
esophageal reflux disease: Update of the Porto
consensus and recommendations from an interna-
Question 5 tional consensus group. Neurogastroenterol Motil.
A 50-year-old obese man presents for further 2017 Oct;29(10):1-15.
evaluation of a three month history of retrosternal Fass R, Achem SR. Noncardiac chest pain:
chest pain. He reports a heavy, pressure-like pain diagnostic evaluation. Dis Esophagus. 2012
in the chest that sometimes improves with belch- Feb;25(2):89-101.
ing or with an oral antacid. He has not noticed
food triggers, reports it is not temporally related
to meals, and has no nocturnal pain. He has no Question 6
regurgitation. He has tried an H2 receptor antago- Which one of the following medications can in-
nist without benefit. Prior to referral, he under- crease likelihood of symptomatic gastroesophageal
went an upper endoscopy with biopsy which was reflux?
normal. Physical examination is normal.
A. Alendronate
Which of the following is the most appropriate B. Baclofen
next step? C. Metoprolol
D. Naproxen
A. Initiate PPI trial with omeprazole 40 mg bid E. Theophylline
B. Perform 96 hour wireless pH monitoring
C. Perform a barium esophagram CORRECT ANSWER: E
D. Perform esophageal manometry
E. Refer to a cardiologist RATIONALE
Theophylline can reduce basal LES pressure,
CORRECT ANSWER: E and can promote symptomatic gastroesophageal
reflux. Baclofen, on the other hand, reduces tran-
RATIONALE sient LES relaxations, and reduces reflux episodes.
The patient is male, middle aged, and obese, which Beta blockers do not impact reflux, but beta adren-
are all risk factors for cardiac disease. Symptoms ergic agonists can increase reflux. Naproxen and
are atypical for reflux disease, and typical triggers alendronate are irritants to the esophageal mucosa
for reflux are not reported. In individuals at risk if retained in the esophagus, but do not increase
for heart disease, cardiac evaluation takes pre- reflux tendency.
cedence over evaluation for reflux disease. A PPI
trial would delay cardiac evaluation and therefore REFERENCE
would not be appropriate as an initial approach. If Mungan Z, Pınarbaşı Şimşek B. Which drugs are
cardiac disease were to be excluded, prolonged pH risk factors for the development of gastroesopha-
monitoring off PPI and esophageal manometry are geal reflux disease? Turk J Gastroenterol. 2017
appropriate tests to establish or refute the pres- Dec;28(Suppl 1):S38-S43.
4 Digestive Diseases Self-Education Program®

Question 7 D. Obesity
Which of the following is a consequence of re- E. Older age
duced saliva production following radiation for
head and neck cancer? CORRECT ANSWER: A

A. Barrett’s esophagus RATIONALE


B. Erosive esophagitis The presence of Helicobacter pylori infection is as-
C. Esophageal stricture sociated with reduced gastric acid production, and
D. Increased gastric acid volume long term infection with atrophic gastritis can be
E. Ineffective esophageal motility associated with achlorhydria. The time to recur-
rence of reflux symptoms following PPI therapy is
CORRECT ANSWER: B longer in Helicobacter pylori positive patients, and
the risk of Barrett’s esophagus is lower. Eradica-
RATIONALE tion of Helicobacter pylori infection is reported to
Salivary bicarbonate is necessary for neutraliza- increase reflux symptoms, but has not been dem-
tion of mucosal acidification following volume onstrated to increase reflux complications. With
clearance of refluxate. If salivary production is increasing age, esophageal perception decreases,
decreased, the risk for erosive esophagitis increas- and advanced grade esophagitis can exist without
es. There is no evidence for increased prevalence significant symptoms. The presence of a hiatus
of Barrett’s esophagus in patients with decreased hernia can increase esophageal acid exposure
salivary flow. Esophageal strictures and ineffec- time, especially supine acid burden, which can
tive esophageal motility are not related to saliva increase risk for complicated reflux disease. Male
production. Finally, saliva does not play a role in gender and obesity are additional risk factors for
altering gastric acid volume. Barrett’s esophagus and reflux esophagitis, while
women and African American ethnicity are associ-
REFERENCES ated with lower rates of complicated reflux disease.
Manabe N, Haruma K, Kusunoki H, et al. Dif-
ferences in salivary secretory function between REFERENCES
patients with erosive esophagitis and those with Wang Z, Shaheen NJ, Whiteman DC, et al. Helico-
nonerosive reflux disease. J Gastroenterol Hepa- bacter pylori infection is associated with reduced
tol. 2018 Apr;33(4):807-813. risk of Barrett’s esophagus: an analysis of the Bar-
Korsten MA, Rosman AS, Fishbein S, et al. rett’s and esophageal adenocarcinoma consortium.
Chronic xerostomia increases esophageal acid Am J Gastroenterol 2018 doi: 10.1038/s41395-
exposure and is associated with esophageal injury. 018-0070-3.
Am J Med. 1991 Jun;90(6):701-6. Schwizer W, Menne D, Schütze K, et al. The
effect of Helicobacter pylori infection and eradi-
cation in patients with gastro-oesophageal reflux
Question 8 disease: A parallel-group, double-blind, placebo-
In which of the following settings is the likelihood controlled multicentre study. United European
of future complications of reflux disease, including Gastroenterol J. 2013 Aug;1(4):226-35.
advanced grade erosive esophagitis and Barrett’s
esophagus the lowest? Question 9
A 50-year-old gentleman with chest pain and
A. Helicobacter pylori coronary artery disease continues to have symp-
B. Hiatus hernia toms despite optimal management of his coronary
C. Male gender disease. His cardiologist suspects an overlap of
Chapter 1 — Esophageal disorders 5

non-cardiac chest pain, and requests that he be ern diagnosis of GERD: the Lyon Consensus. Gut.
evaluated for the presence of GERD. He reports 2018 Jul;67(7):1351-1362.
postprandial heartburn three to five times a week.
He has no other esophageal symptoms, and no
alarm features. Upper endoscopy demonstrates Question 10
a 3 cm segment with salmon colored mucosa, A 25-year-old gentleman has heartburn and acid
and biopsies demonstrate intestinal metaplasia regurgitation several times a week. An upper
without dysplasia. There is a three cm axial hiatus endoscopy is completely normal, and no hiatus
hernia. An esophageal high resolution manometry hernia is seen. Esophageal biopsies from the distal
confirms the presence of the hiatus hernia, and esophagus reveal papillary elongation and basal
also demonstrates ineffective esophageal motil- cell hyperplasia. A prolonged wireless pH study
ity, without contraction reserve. A pool of yellow- demonstrates acid exposure time of six to eight
ish fluid is noted in the gastric fundus, which the percent on each day of the four day study. A bar-
endoscopist reports as the presence of bile. ium swallow demonstrates ‘reflux to the thoracic
inlet’. He is concerned about taking medications,
Which of the following features is most suggestive and wants to know if he absolutely needs to start
of a diagnosis of GERD? PPI therapy.

A. Bile in the gastric fundus Among patients with non-erosive reflux disease,
B. Hiatus hernia which of the following factors predicts symptom
C. Ineffective esophageal manometry improvement with proton pump inhibitor
D. Intestinal metaplasia therapy?
E. Post-prandial heartburn
A. Abnormal esophageal biopsy
CORRECT ANSWER: D B. Abnormal pH study
C. Absence of hiatus hernia
RATIONALE D. Presence of acid regurgitation
Biopsy proven intestinal metaplasia on biopsies E. Reflux seen on barium swallow
taken from the distal esophagus is conclusive for a
diagnosis of GERD, and correlates well with distal CORRECT ANSWER: B
esophageal acid exposure time. While hiatus her-
nia and ineffective esophageal motility may both RATIONALE
predict a higher esophageal acid burden, these are Abnormal acid exposure on pH monitoring inde-
not pathognomonic for GERD, and can be seen pendently predicts response to antireflux therapy,
without significant GERD as well as in healthy including PPI therapy. Microscopic esophagitis
asymptomatic controls. Heartburn as a symptom on routine esophageal biopsies is not predictive
has only modest sensitivity and specificity for of PPI response in nonerosive reflux disease,
GERD. Bile is seen normally in the gastric fundus, as there is overlap with reflux hypersensitivity,
and does not establish the presence of GERD. which is a functional disorder. The presence of
While each of these features (other than intestinal a hiatus hernia can be associated with higher
metaplasia) by itself does not establish GERD, esophageal acid exposure times. Regurgitation as
these features in combination increase confidence a symptom does not improve as well as heartburn
in a GERD diagnosis. with acid suppression. Barium swallow finding
of reflux does not correlate with symptoms, acid
REFERENCE burden on pH testing, or symptom response to
Gyawali CP, Kahrilas PJ, Savarino E, et al. Mod- anti-reflux therapy.
6 Digestive Diseases Self-Education Program®

REFERENCES tip can dip into the stomach during esophageal


Patel A, Sayuk GS, Gyawali CP. Parameters on shortening. Post-nasal drip is not thought to im-
esophageal pH-impedance monitoring that predict pact esophageal acid exposure time.
outcomes in patients with gastroesophageal reflux
disease. Clin Gastroenterol Hepatol 2015;13:884-91. REFERENCE
Savarino E, Bredenoord AJ, Fox M, et al. Ad- Roman S, Gyawali CP, Savarino E, et al. Ambula-
vances in the physiologic assessment and diagno- tory reflux monitoring for diagnosis of gastro-
sis of gastroesophageal reflux disease: A consensus esophageal reflux disease: Update of the Porto
statement. Nature Reviews Gastroenterol Hepatol consensus and recommendations from an interna-
2017;14(11):665-676. tional consensus group. Neurogastroenterol Motil.
2017 Oct;29(10):1-15.

Question 11
Which of the following can falsely elevate acid ex- Question 12
posure time with wireless pH monitoring studies? A 33-year-old lady presents with retrosternal
burning for the past five years. She has been taking
A. Acidic food/beverage swallows a PPI once a day before breakfast for two years,
B. Alkaline reflux but continues to have burning two to three times
C. Antacid use a day. Taking an antacid does not provide benefit.
D. Esophageal shortening She has no alarm symptoms. She underwent an
E. Post nasal drip upper endoscopy while on PPI a year ago, which
was normal, including biopsies.
CORRECT ANSWER: A
Which of the following is the most appropriate
RATIONALE next step?
Esophageal pH monitoring is often used to clarify
esophageal reflux burden, especially when esoph- A. Barium esophagram with a solid barium bolus
ageal symptoms persist despite acid suppression. B. Dual phase gastric emptying study
Wireless pH monitoring has a single recording C. High resolution esophageal impedance
site on a pH capsule placed in the esophagus manometry
six cm proximal to the endoscopically identified D. PH-impedance monitoring on her current
squamocolumnar junction. Since there is a single PPI dose
site of pH recording, acidic swallows cannot be E. Repeat endoscopy with wireless pH
differentiated from acid reflux. A careful diary is monitoring off PPI
kept by the patient, which is reconciled with the
pH recording. An oral intake is excluded from CORRECT ANSWER: E
analysis to avoid confounding of the acid expo-
sure time from acidic swallows. Alkaline reflux RATIONALE
is not detected with wireless pH monitoring, and The patient has retrosternal burning that has not
would not increase acid exposure time. Antacids responded to daily PPI therapy. She has no prior
neutralize gastric acid, and can potentially reduce conclusive evidence of GERD, and therefore has
acid exposure time. Esophageal shortening does unproven GERD. This is an indication for pH
not impact pH assessment using a wireless cap- monitoring performed off PPI therapy, to estab-
sule, since the capsule is attached to the esopha- lish or refute the presence of GERD. Among the
geal wall, but can impact catheter based pH options provided, a repeat endoscopy with pro-
recordings, where an improperly placed catheter longed wireless pH monitoring off PPI is the most
Chapter 1 — Esophageal disorders 7

appropriate option, where abnormal acid exposure five. In contrast, the number needed to treat for a
time will reinforce the need for PPI therapy, and heartburn response is less than two. Regurgitation
physiologic acid exposure time will suggest an al- does not respond well to acid suppression, and can
ternate mechanism for symptoms. pH-impedance persist despite adequate heartburn response in
monitoring on PPI therapy is indicated in patients patients with typical reflux symptoms. Less than
with proven GERD (prior esophagitis, peptic stric- 20 percent of patients with cough and hoarseness
ture or Barrett’s esophagus on endoscopy, prior report response to PPI therapy, and response is
abnormal pH study), where the intent of reflux even lower with globus sensation. Consequently,
monitoring is to establish mechanism of ongoing the more atypical the symptom, the higher the
symptoms. Barium radiography with solid barium need for ambulatory reflux monitoring off PPI to
bolus is useful in the evaluation of esophageal rule in or rule out reflux disease, since positive pH
dysphagia. High resolution impedance manom- testing can predict treatment response to PPI.
etry is performed for unexplained dysphagia, and
sometimes for persisting esophageal symptoms, REFERENCE
but an ambulatory pH study will have higher yield Gyawali CP, Fass R. Management of Gastroesoph-
in the evaluation of heartburn. A gastric emptying ageal Reflux Disease. Gastroenterology. 2018
study will not help initial evaluation of retrosternal Jan;154(2):302-318.
burning.

REFERENCE Question 14
Savarino E, Bredenoord AJ, Fox M, et al. Advanc- A 45-year-old man presents with post prandial
es in the physiologic assessment and diagnosis regurgitation. Initially, he had heartburn and
of gastroesophageal reflux disease: A consensus regurgitation, and an endoscopy performed prior
statement. Nature Reviews Gastroenterol Hepatol to PPI initiation demonstrated erosive esophagitis
2017;14(11):665-676. (LA grade C). He was given PPI twice a day in ad-
dition to lifestyle modification, which improved his
heartburn, but regurgitation persists. He has no
Question 13 dysphagia. Repeat endoscopy three months later
Among the following symptoms, which one demonstrates resolution of his esophagitis, and no
is most likely to respond to a trial of PPI evidence of Barrett’s esophagus. He has a three cm
therapy? intermittent sliding hiatus hernia.

A. Chronic unexplained cough What is the next best step in evaluation? ?


B. Hoarseness of voice
C. Idiopathic globus sensation A. Barium swallow
D. Non-cardiac chest pain B. Endoscopic ultrasound
E. Post prandial regurgitation C. Esophageal manometry
D. Gastric emptying study
CORRECT ANSWER: D E. pH-impedance study on PPI

RATIONALE CORRECT ANSWER: E


Noncardiac chest pain is the atypical reflux
symptom most likely to respond to a trial of PPI RATIONALE
therapy, especially if ambulatory pH testing dem- This is a patient with proven GERD, with prior
onstrates abnormal reflux parameters, when the evidence of LA grade C reflux esophagitis that has
number needed to treat for a response is four to healed with PPI therapy. However, symptoms
8 Digestive Diseases Self-Education Program®

persist despite PPI therapy. Ambulatory reflux with a suboptimal response of reflux symptoms
monitoring is indicated to evaluate the mechanism to standard reflux therapy. Affective disorders
of persisting regurgitation, and is performed on including anxiety and depression are also associ-
maximal PPI therapy. If regurgitation correlates ated with an incomplete response to PPI therapy,
with reflux episodes, if a high number of reflux possibly related to visceral hypersensitivity and
episodes are identified, or if acid exposure time is hypervigilance. In contrast, high esophageal acid
elevated, escalation of management can be consid- exposure times, and presence of erosive esophagi-
ered, and the patient can be offered antireflux sur- tis are associated with better symptom responses
gery, especially since he also has a hiatus hernia. compared to lack of conclusive evidence for reflux
Behavioral therapy (e.g. diaphragmatic breathing) on esophageal testing. PPIs work best when ad-
can be offered if rumination is suspected. Esopha- ministered 30-45 min before the first meal of the
geal manometry may be performed to assess peri- day. The S-isomer of omeprazole (esomeprazole)
staltic performance prior to antireflux surgery, and has better bioavailability because of slower elimi-
barium swallow can define anatomic relationships nation, and may provide better symptom response
at the esophagogastric junction, but these tests in equivalent doses.
would not evaluate whether his current symp-
toms are due to incomplete response to therapy. REFERENCES
A gastric emptying study can determine if gastro- Weijenborg PW, Cremonini F, Smout AJ, Brede-
paresis could be contributing to regurgitation if noord AJ. PPI therapy is equally effective in well-
pH impedance testing is negative. An endoscopic defined non-erosive reflux disease and in reflux
ultrasound is not useful in this patient. esophagitis: a meta-analysis. Neurogastroenterol
Motil. 2012 Aug;24(8):747-57, e350.
REFERENCE Wang AJ, Wang H, Xu L, et al. Predictors of
Roman S, Gyawali CP, Savarino E, et al. Ambula- clinical response of acid suppression in Chinese
tory reflux monitoring for diagnosis of gastro- patients with gastroesophageal reflux disease. Dig
esophageal reflux disease: Update of the Porto Liver Dis. 2013 Apr;45(4):296-300.
consensus and recommendations from an inter-
national consensus group. Neurogastroenterol
Motil. 2017 Oct;29(10):1-15. Question 16
Which of the following is a known consequence of
successful antireflux surgery?
Question 15
Which of the following is a predictor of suboptimal A. Delayed solid phase gastric emptying
response of heartburn to PPI therapy? B. Belching and regurgitation
C. Dysphagia
A. Administration of PPI before breakfast D. Esophageal hypomotility
B. Administration of S-isomer of omeprazole E. Small bowel bacterial overgrowth
C. High acid exposure time on pH testing
D. Presence of irritable bowel syndrome CORRECT ANSWER: C
E. Presence of LA Grade B reflux esophagitis
RATIONALE
CORRECT ANSWER: D Dysphagia is a recognized consequence of anti-
reflux surgery. Early postoperative dysphagia is
RATIONALE common, likely related to postoperative edema,
The presence of functional bowel disorders, and typically improves without need for interven-
including irritable bowel syndrome, is associated tion. Late postoperative dysphagia persisting
Chapter 1 — Esophageal disorders 9

beyond or occurring after three months after sur- A. Diaphragmatic breathing


gery occurs in six to 25 percent, and can be due B. Esophageal manometry
to wrap dysfunction, esophageal hypomotility, or C. pH-impedance monitoring
recurrent complicated reflux, among other causes. D. Upper endoscopy
Preoperative dysphagia, and absence of contrac- E. CT scan of the chest
tion reserve on preoperative manometry are risk
factors for late post-fundoplication dysphagia. CORRECT ANSWER: D
Unless there is vagal injury, solid phase gastric
emptying is commonly accelerated after fundo- RATIONALE
plication, due to altered gastric accommodation. Inspection of the fundoplication during endoscopy
Belching and regurgitation improve following can determine if there is wrap dysfunction. In
antireflux surgery, and recurrence may be a sign particular, a slipped fundoplication with a recur-
of wrap dysfunction. Esophageal hypomotility can rent hiatus hernia, especially a paraesophageal
improve following antireflux surgery, especially if hernia, can be associated with dysphagia and pain
pre-operative manometry demonstrates contrac- during swallowing. Endoscopy can also determine
tion reserve. Small bowel bacterial overgrowth is if erosive esophagitis or an esophageal stricture
not associated with antireflux surgery. could be contributing to symptoms. A barium
radiograph can complement endoscopic inspection
REFERENCES of the fundoplication by establishing the relation-
Kellokumpu I, Voutilainen M, Haglund C, et al. ship of the fundoplication to the esophagogastric
Quality of life following laparoscopic Nissen fun- junction. Manometry is useful in evaluating for a
doplication: assessing short-term and long-term motor mechanism for esophageal symptoms, and
outcomes. World J Gastroenterol 2013;19:3810-8. would be warranted after a negative endoscopy.
Tsuboi K, Lee TH, Legner A, et al. Identifica- While a CT scan can demonstrate a hiatus hernia,
tion of risk factors for postoperative dysphagia this is not the most efficient method of establish-
after primary anti-reflux surgery. Surg Endosc ing wrap dysfunction. Diaphragmatic breathing is
2011;25:923-9. useful in treating supragastric belching and rumi-
van Rijn S, Rinsma NF, van Herwaarden- nation, but not wrap dysfunction.
Lindeboom MYA, et al. Effect of vagus nerve
injury on short and long-term efficacy of antireflux REFERENCE
surgery. AM J Gastroenterol 2016 ;111 :508-515. Yadlapati R, Hungness ES, Pandolfino JE. Com-
plications of Antireflux Surgery. Am J Gastroen-
terol. 2018 Aug;113(8):1137-1147.
Question 17
A 45-year-old gentleman with postprandial
heartburn and acid regurgitation with a hiatus Question 18
hernia underwent laparoscopic antireflux sur- A 45-year-old gentleman undergoes upper
gery. Initial symptomatic response was excellent, endoscopy for new onset dyspeptic symptoms
and proton pump inhibitors were discontinued. without heartburn. A three cm circumferential
Two years later, the patient developed retroster- segment of salmon colored mucosa is identified
nal discomfort during and following swallowing, in the distal esophagus, and the remainder of the
localized to the lower chest. He has no regurgita- esophagus is normal. Biopsies are obtained, and
tion or heartburn, and has not lost weight. He histopathology reveals intestinal metaplasia with
has tried proton pump inhibitors without benefit. low-grade dysplasia. In addition to initiation of
Which of the following is the most appropriate PPI therapy, which of the following is an appro-
next step? priate next step?
10 Digestive Diseases Self-Education Program®

A. Perform endoscopic mucosal resection A. Barium esophagram with a solid bolus


B. Refer patient for distal esophagectomy B. Endoscopy with biopsy
C. Refer patient for fundoplication C. Esophageal manometry
D. Repeat biopsies after PPI therapy D. pH-impedance monitoring off PPI
E. Repeat biopsies in three years E. PPI trial with omeprazole twice a day

CORRECT ANSWER: D CORRECT ANSWER: B

RATIONALE RATIONALE
Untreated reflux disease can influence interpreta- Young males with solid food dysphagia need to be
tion of biopsies obtained from Barrett’s esopha- evaluated for eosinophilic esophagitis, and this is
gus, and increases diagnosis of low-grade dyspla- best performed with an endoscopy with proximal
sia or indefinite dysplasia. Over-diagnosis of low and distal esophageal biopsies. The histologi-
grade dysplasia can significantly impact health cal hallmark consists of demonstration of more
care costs. In these instances, repeating biopsies than 15 eosinophils/high power field. Since PPI
following initiation of PPI therapy can provide therapy can normalize eosinophil counts, endos-
a more representative interpretation. Esopha- copy with biopsy should be performed prior to
geal mucosal resection is performed for nodules initiation of PPI therapy when this diagnosis is
identified within the Barrett’s segment. Distal suspected. Although barium radiography pro-
esophagectomy is an option for limited esopha- vides complementary information, and has a
geal cancer without metastases and high-grade high sensitivity for identification of subtle stric-
dysplasia, although endoscopic ablation is the tures, endoscopy with biopsy is preferred for
preferred management option. A fundoplication initial diagnosis when eosinophilic esophagitis is
is an option for management of reflux disease, but suspected. Esophageal manometry is performed
would not be offered to an asymptomatic patient if endoscopy and/or barium radiography do not
without esophagitis. If repeat biopsies demon- establish a mechanism for dysphagia. pH-imped-
strate non-dysplastic Barrett’s esophagus, a three ance monitoring is performed off PPI therapy if
to five-year surveillance interval is appropriate. esophageal acid burden needs to be quantified,
especially in patients with reflux symptoms where
REFERENCES a PPI trial does not improve symptoms.
Wani S, Rubenstein JH, Vieth M, Bergman
J. Diagnosis and Management of Low-Grade REFERENCE
Dysplasia in Barrett’s Esophagus: Expert Review Furuta GT, Katzka DA. Eosinophilic Esophagitis.
From the Clinical Practice Updates Committee N Engl J Med. 2015 Oct 22;373(17):1640-8.
of the American Gastroenterological Association.
Gastroenterology. 2016 Nov;151(5):822-835.
Question 20
An 18-year-old college freshman is referred for
Question 19 endoscopy with a 12 month history of intermit-
A 22-year-old male comes to see you with a two tent retrosternal pain, and one episode of food
year history of heartburn responsive to antacids, impaction that spontaneously resolved. Endos-
and intermittent solid food dysphagia, particu- copy demonstrates normal esophageal mucosa,
larly with bread and meat. He has no regurgi- and no stricture. Distal esophageal biopsies
tation or weight loss. Physical examination is demonstrate 20 eosinophils per high power field.
normal. Which of the following is an appropriate Which of the following is the most appropriate
investigative approach? next step?
Chapter 1 — Esophageal disorders 11

A. Cromolyn sodium once a day Question 21


B. Empiric esophageal dilation Which of the following is a direct risk factor for
C. Montelukast once a day esophageal candidiasis?
D. Proton pump inhibitor twice a day
E. Swallowed budesonide twice a day A. Diabetes mellitus
B. Sinusitis
CORRECT ANSWER: D C. Barrett’s esophagus
D. Heterotopic gastric mucosa
RATIONALE E. Allergic rhinitis
Despite the absence of esophageal findings on
endoscopy, the presence of more than 15 eosino- CORRECT ANSWER: A
phils per high power field is suggestive of eosino-
philic esophagitis (EoE), especially with a history RATIONALE
of esophageal symptoms including food impac- Diabetes mellitus is an established risk factor
tion. However, these findings can overlap with for esophageal candidiasis. Other risk fac-
reflux disease. In many instances, proton pump tors include immunodeficiency states includ-
inhibitor (PPI) therapy resolves eosinophilia on ing HIV-AIDS, immunosuppressive therapy,
histopathology, and improves symptoms. Up to antibiotic use and corticosteroid use. Therefore,
50 percent of EoE patients will have normaliza- while sinusitis and allergic rhinitis are not risk
tion of biopsies on PPI. Endoscopy is repeated factors, treatment with antibiotics and topical
after eight weeks of bid proton pump inhibitor fluticasone respectively can be associated with
therapy to evaluate for persisting eosinophilia; esophageal candidiasis. Motor disorders such as
if present, swallowed budesonide or fluticasone, achalasia can manifest with esophageal can-
or an elimination diet are options. Cromolyn didiasis. It is extremely rare in asymptomatic
sodium and montelukast have not been dem- healthy individuals. GERD and Barrett’s esoph-
onstrated to consistently improve esophageal agus are not risk factors for esophageal candi-
eosinophilia or esophageal symptoms. Empiric diasis, but esophageal cancer is a risk factor.
esophageal dilation is not recommended in eo- Heterotopic gastric mucosa can be seen in the
sinophilic esophagitis, but dominant esophageal proximal esophagus, also termed inlet patch.
strictures with persisting dysphagia may benefit This can be a risk factor for Barrett’s esophagus
from targeted dilation. and Helicobacter pylori infection, but is not as-
sociated with esophageal candidiasis.
REFERENCES
Dellon ES, Gonsalves N, Hirano I, et al. ACG REFERENCES
clinical guideline: Evidenced based approach to Lee SP, Sung IK, Kim JH, et al. The clini-
the diagnosis and management of esophageal cal course of asymptomatic esophageal can-
eosinophilia and eosinophilic esophagitis (EoE). didiasis incidentally diagnosed in general
Am J Gastroenterol. 2013 May;108(5):679-92. health inspection. Scand J Gastroenterol.
Hirano Ikuo. How to Approach a Patient 2015;50(12):1444-50.
with Eosinophilic Esophagitis. Gastroenterology Asayama N, Nagata N, Shimbo T, et al.
2018;155:601-606. Relationship between clinical factors and
Dellon ES, Liacouras CA, Molina-Infante J et severity of esophageal candidiasis according
al. Updated International Consensus Diagnostic to Kodsi’s classification. Dis Esophagus. 2014
Criteria for Eosinophilic Esophagitis: Proceed- Apr;27(3):214-9.
ings of the AGREE Conference. Gastroenterology
2018; 155:1022–1033.
12 Digestive Diseases Self-Education Program®

Question 22 REFERENCE
A 65-year-old lady underwent endoscopy for Kim SH, Jeong JB, Kim JW, et al. Clinical and
a three week history of retrosternal chest pain endoscopic characteristics of drug-induced
during swallowing. She has hypertension and esophagitis. World J Gastroenterol. 2014 Aug
diabetes, treated with amlodipine and metformin 21;20(31):10994-9.
respectively. She has rosacea, for which she takes
doxycycline. She has celiac disease, treated with a
gluten free diet. She smokes a packet of cigarettes Question 23
a day. Physical examination is normal. A 22-year-old male with schizoaffective disorder is
Endoscopy was performed, and revealed brought to the emergency room at 1a.m. after hav-
an erosion in the proximal esophagus. Biopsies ing swallowed a button battery at 11p.m.. He has a
revealedan acute neutrophilic infiltrate, but no history of GERD, and takes a daily PPI with con-
viral inclusions. . Distal esophageal biopsies were trol of his heartburn. He reports infrequent dys-
normal. She was started on a daily PPI, but reports phagia to solids. He currently has no symptoms,
no improvement. and is able to handle secretions without problems.
He last ate at 7p.m. the previous evening.
Which of the following is a likely mechanism for Chest x-ray shows a radio-opaque foreign body
her condition? below the carina but above the diaphragm.

A. Celiac disease Which of the following is the most appropriate


B. Cigarette smoking next step?
C. Diabetes mellitus
D. Doxycycline use A. Administer glucagon intravenously
E. Metformin use B. Perform a CT of the chest and abdomen
C. Perform a gastrografin swallow
CORRECT ANSWER: D D. Perform endoscopy right away
E. Perform endoscopy within 12 hours
RATIONALE
The proximal esophagus at the level of the aortic CORRECT ANSWER: D
arch is one of the sites where swallowed pills can
hold up. This area also corresponds to the transi- RATIONALE
tion zone between skeletal and smooth muscle in Button and disc batteries represent some of the
the esophageal wall, and may be associated with most toxic ingestions possible. Leaked contents
lower contractile pressures. Doxycycline can cause of button batteries can begin to damage mucosa
an intense local reaction if it comes under pro- in a hollow viscus within 15 min, and damage
longed contact with esophageal mucosa, and can can extend to the muscular layers within 30 min.
lead to erosion and ulceration. Retrosternal pain, While pediatric ingestions are more common,
odynophagia and dysphagia are common present- adult ingestions have also been reported. There-
ing symptoms of such pill-induced esophagitis. fore, ingested button and disc batteries need to be
Metformin is not associated with pill esophagitis. retrieved emergently, especially in a patient with a
Cigarette smoking can be associated with reflux history of intermittent dysphagia. A gastrografin
disease, which can cause distal esophageal ero- swallow is not indicated for foreign body inges-
sions and esophagitis. Diabetes mellitus can be tions unless a leak or perforation is suspected,
associated with esophageal candidiasis, but not in which instance a CT with oral contrast could
esophageal erosions. Celiac disease is not associ- provide more detail. Non-emergent endoscopy is
ated with esophageal erosions. appropriate for ingested objects that are not sharp,
Chapter 1 — Esophageal disorders 13

especially when there is no clinical evidence of Question 25


complete esophageal obstruction. Which of the following potential consequences of
long-term PPI therapy is supported by the stron-
REFERENCE gest and most consistent evidence?
Ikenberry SO, Jue TL, Anderson MA, et al.
Management of ingested foreign bodies and A. Chronic kidney injury
food impactions. Gastrointest Endosc. 2011 B. Fundic gland polyps
Jun;73(6):1085-91. C. Hip fracture
D. Iron deficiency anemia
E. Dementia
Question 24
Which one of the following management options CORRECT ANSWER: B
for achalasia carries the highest risk for reflux
symptoms and esophagitis? RATIONALE
Fundic gland gastric polyps are common, and
A. Botulinum toxin injection are consistently seen in long-term PPI users
B. Laparoscopic myotomy with an odds ratio of 2.2. There is strong and
C. Oral nifedipine three times a day consistent evidence that fundic gland polyps are
D. Per oral endoscopic myotomy caused by long-term PPI use, even though they
E. Pneumatic dilation to 35 mm are benign and do not lead to sinister conse-
quences. Chronic kidney injury has an odds ra-
CORRECT ANSWER: D tio of 1.7, and is thought to occur from repeated
episodes of idiosyncratic interstitial nephritis.
RATIONALE Bone fracture has an even lower odds ratio of
Of the options outlined above, the highest risk for 1.4, and a recent randomized study did not dem-
GERD and reflux esophagitis is seen following per onstrate an increased risk of hip fracture with
oral endoscopic myotomy. As many as 19 percent chronic PPI use. Iron metrics can be abnormal,
develop reflux symptoms, 29 percent have reflux but clinical iron deficiency is infrequent. Al-
esophagitis and 39 percent have abnormal pH test- though dementia was reported to be associated
ing after per oral endoscopic myotomy, compared with long-term PPI use, this has recently been
to nine percent, eight percent and 17 percent respec- refuted in a new report.
tively after laparoscopic myotomy on meta-analysis.
Therefore, patients undergoing per oral endoscopic REFERENCES
myotomy need to be counseled of the increased risk Freedberg DE, Kim LS, Yang Y-X. The risks and
for GERD when planning management for achalasia. benefits of long-term use of proton pump inhibi-
While reflux can occur following pneumatic dila- tors: expert review and best practice advice from
tion, rates are lower. Reflux symptoms may occur the American Gastroenterological Association.
infrequently following botulinum toxin injection and Gastroenterology 2017;152:706-15.
medical therapy with calcium blockers. Vaezi MF, Yang YX, Howden CW. Complica-
tions of Proton Pump Inhibitor Therapy. Gastro-
REFERENCE enterology 2017; 153(1):35-48.
Repici A, Fuccio L, Maselli R, et al. GERD after
per-oral endoscopic myotomy as compared with
Heller’s myotomy with fundoplication: a systemat- Question 26
ic review with meta-analysis. Gastrointest Endosc. A 55-year-old male was found to have worsen-
2018 Apr;87(4):934-943. ing cough over past year. Despite evaluation by
14 Digestive Diseases Self-Education Program®

pulmonologist, allergist, and trials of inhalers and REFERENCES


nasal steroids, the patient has continued cough. Vaezi MF, Katzka D, Zerbib F. Extraesophageal
The patient denies pyrosis and has not been on Symptoms and Diseases Attributed to GERD:
any acid suppression medications. Where is the Pendulum Swinging Now? Clin
Gastroenterol Hepatol. 2018 Jul;16(7):1018-
What is the best step to determine if the cough is 1029.
GERD related? Kahrilas PJ, Altman KW, Chang AB et
al. Chronic Cough Due to Gastroesophageal
A. Upper GI series Reflux in Adults: CHEST Guideline and Expert
B. Wireless ph Monitoring, over 96 hours Panel Report. Chest. 2016 Dec;150(6):1341-
C. Trial of proton pump inhibitor therapy and 1360.
lifestyle modification
D. Antireflux surgery
Question 27
CORRECT ANSWER: B A 45-year-old businessman comes to you with con-
cerns that his medications are not working for his
RATIONALE heartburn. Typically, his heartburn is an upward
Ninety-six hour wireless pH monitoring is a good burning sensation in the chest. He takes omepra-
test to rule out GERD when suspicion for it is zole every day 30 minutes prior to breakfast. This
low. Cough can be due to many potential etiolo- typically controls his symptoms, especially when
gies, including  environmental or occupational he has his normal routine at home. However, he
irritants, primary or secondary smoking, use of travels frequently and often has dinners with cli-
angiotensin-converting-enzyme (ACE) inhibitors, ents. While traveling, he cannot sleep well due to
abnormal chest radiographic findings, asthma, regurgitation and coughing at night.
upper airway cough syndrome due to a variety of
rhinosinus conditions (eg postnasal drip), non- What would be the most helpful suggestion for
asthmatic eosinophilic bronchitis, and suppura- this patient?
tive lung disease. Often, more than one etiology is
a contributing factor. Other etiologies should be A. Avoid late and heavy dinner and sleep at
evaluated first, as in this case. If no cause is found, incline
the next best step is EGD with wireless pH moni- B. EGD with wireless pH testing, over 96 hours,
toring off of PPI. An empiric trial of proton pump off therapy
inhibitor (PPI) therapy with lifestyle modification C. Nissen fundoplication
(Answer C) is a cost effective way to see if there D. Start BID PPI every day
is a response to symptoms, however, it has a low E. Barium swallow
positive predictive value (mostly due to placebo
effect). In addition, response to treatment may be CORRECT ANSWER: A
variable for potential extraintestinal manifesta-
tions of GERD, so diagnostic testing to evaluate for RATIONALE
the presence of pathologic reflux is a better choice This patient has nocturnal GERD. Avoiding late
here. The data for anti-reflux surgery for treat- night meals and head of the bed elevation is ef-
ment of extraesophageal symptoms is weak and fective in treating nocturnal GERD (Answer A).
proper patient selection is challenging. An Upper The patient has classic symptoms of GERD and
GI series (Answer A) represents a brief period of is generally well controlled making wireless pH
time, and has poor sensitivity and specificity in testing (Answer B) unnecessary. A barium swal-
evaluation of GERD. low (Answer E) would be of limited utility in the
Chapter 1 — Esophageal disorders 15

diagnosis of GERD or in the management of his an antireflux surgery. The optimal timing of
intermittent breakthrough symptoms. While some PPI therapy is 30 minutes prior to meals on an
patients do require escalation to BID PPI therapy empty stomach in order for the best absorption
or surgery (Answer C, D), he may be better served and activation of drug. Taking the medication
to focus on directed lifestyle modifications for his after meals (Answer B) would reduce absorp-
intermittent symptoms that are related to larger tion of drug and make it less effective. She does
dinner time meals. not have dysphagia or stricture and a dilation
(Answer D) is not needed. She does not meet
REFERENCE criteria for eosinophilic esophagitis and topi-
Ness-Jensen E, Hveem K, El-Serag H, Lagergren cal steroids (Answer E) are not indicated. A CT
J. Lifestyle Intervention inGastroesophageal scan (Answer C) is not part of the standard anti-
Reflux Disease. Clin Gastroenterol Hepatol. 2016 reflux procedure work up.
Feb;14(2):175-82.
REFERENCES
Bello B, Zoccali M, Gullo R, Allaix ME, Herbella
Question 28 FA, Gasparaitis A, Patti MG. Gastroesophageal
A 35-year-old female has had heartburn for reflux disease and antireflux surgery-what is the
seven years and has had to escalate PPI therapy proper preoperative work-up? J Gastrointest Surg.
from QD to BID prior to meals, and recently 2013 Jan;17(1):14-20.
added H2 blockers at bedtime. She denies any Jobe BA, Richter JE, Hoppo T, et al. Pre-
dysphagia. She has to maintain a very strict diet operative diagnostic work-up prior to antireflux
in order to be without symptoms. She is inter- surgery: an “evidence and experience-based”
ested in surgery to be able to liberalize her diet consensus of the Esophageal Diagnostic Advisory
and spare the amount of medication she needs Panel. J Am Coll Surg 2013;217(4):586-97.
to take. Her EGD demonstrates a small hiatal
hernia and biopsies had 3 eosinophils per hpf
in the distal esophagus. A wireless pH study Question 29
performed at the time off therapy demonstrates A 56-year-old female with a BMI 42, diabetes, and
percent time with pH less than four at seven hyperlipidemia presents with a 5 cm hiatal hernia.
percent and DeMeester score of 22. She has symptoms of heartburn during the day
and significant nocturnal regurgitation such that
What should be done prior to a Nissen she is sleeping in a recliner at night.
fundoplication in this patient?
What surgical option would be most effective in
A. High resolution manometry this patient?
B. Change the time of her PPI to after meals
C. CT scan of the chest A. Hiatal hernia repair with Nissen
D. EGD with empiric dilation fundoplication
E. Trial of swallowed fluticasone B. Hiatal hernia repair with Dor fundoplication
C. Magnetic sphincter augmentation
CORRECT ANSWER: A D. Hiatal hernia repair with gastric bypass
E. Gastric sleeve
RATIONALE
A high resolution manometry study (Answer CORRECT ANSWER: D
A) would be needed to exclude the presence
of a major motor disorder prior to pursuing
16 Digestive Diseases Self-Education Program®

RATIONALE CORRECT ANSWER: D


The severe reflux may be due to the hiatal hernia
and worsened by the obesity. This patient has RATIONALE
medically complicated obesity and thus bariatric Vagal nerve injury (Answer D) is a known adverse
surgery is an option. A gastric bypass in this situ- event after anti-reflux surgery, especially after
ation offers the best anti-reflux procedure for this redo procedures. While SMA syndrome and malig-
patient. A fundoplication in the setting of obesity nancy (Answers B and C) may cause delayed gas-
has a higher rate of recurrence of symptoms (An- tric emptying due to partial gastric outlet obstruc-
swers A, B). While a gastric sleeve is an option for tion, the clinical scenario and upper GI findings
the obesity, a gastric sleeve (Answer E) may cause are not highly suggestive of these options. Chronic,
de novo reflux or worsen pre-existing symptoms. long standing diabetes, not new onset diabetes
Magnetic sphincter augmentation (Answer C) has (Answer A), is associated with gastroparesis. H.
demonstrated promising results in patients with Pylori infection (Answer E) may cause nausea and
a BMI less than 35 and hiatal hernia less than vomiting, but not typically associated with delayed
3 cm. Data are not available for patients with gastric emptying.
higher BMI.
REFERENCE
REFERENCES van Rijn S, Rinsma NF, van Herwaarden-Linde-
Abdelrahman T, Latif A, Chan DS, Jones H, Farag boom MYA, et al. Effect of vagus nerve integrity on
M, Lewis WG, Havard T, EscofetX. Outcomes after short and long-term efficacy of antireflux surgery.
laparoscopic anti-reflux surgery related to obesity: Am J Gastroenterol 2016;111:508-15.
A systematic review and meta-analysis. Int J Surg.
2018 Mar;51:76-82.
Stenard F, Iannelli A. Laparoscopic sleeve Question 31
gastrectomy and gastroesophageal reflux. World J A 40-year-old female has long standing GERD,
Gastroenterol. 2015 Sep 28;21(36):10348-57. well controlled on daily proton pump inhibitor
(PPI) therapy. She changed the way she eats to a
healthier diet and smaller portion sizes. Hearing
Question 30 concerns about the risks of PPI therapy, she tried
A 45-year-old female underwent a hiatal hernia to stop her omeprazole. However, two days later
repair and Nissen fundoplication five years ago. she had constant heartburn worse than she recalls
She had recurrence of the hernia last year and is having before starting the medication.
recovering from a redo operation earlier this year.
She presents with nausea and vomiting. An upper What would be the best option to counsel this
GI series demonstrates delayed gastric emptying, patient?
but no narrowing or strictures.
A. Increase to BID PPI for four weeks and
What is the likely cause of the delayed gastric then step back down to QD PPI therapy
emptying? indefinitely
B. Add Metoclopramide
A. New onset diabetes C. Start maintenance H2 blocker therapy and
B. SMA syndrome trial step down off PPI therapy again
C. Malignant partial obstruction D. Add SSRIs
D. Vagal nerve injury E. Add baclofen
E. H. Pylori infection
CORRECT ANSWER: C
Chapter 1 — Esophageal disorders 17

RATIONALE RATIONALE
This patient is potentially experiencing rebound The patient had an isolated trigger of repeated
acid hypersecretion and may require a tapered emesis that caused a Mallory Weiss tear.. This
step down approach (Answer C) to H2 blocker patient no longer has an indication for continued
maintenance or on-demand PPI therapy. Symp- PPI therapy and it should be discontinued (An-
toms upon cessation of PPI therapy are not an swers A, B). Unlike erosive esophagitis or gastric
indication for indefinite therapy (Answers A). ulcers which would prompt a repeat EGD to
SSRIs (Answer D) would be indicated in esopha- check for healing (Answer C), a healing Mallory
geal hypersensitivity, and Baclofen to inhibit Weiss tear does not require endoscopic monitor-
TLESRs during the postprandial period. Metoclo- ing. He does not have symptoms that would war-
pramide (Answer B) is not recommended in the rant additional work up (Answers D and E).
treatment of GERD, due to adverse effects
of tardive dyskinesia. REFERENCE
Heidelbaugh JJ, Kim AH, Chang R, Walker PC.
REFERENCES Overutilization of proton-pump inhibitors: what
Hunfeld NG, Geus WP, Kuipers EJ. Systematic the clinician needs to know. Therap Adv Gastroen-
review: Rebound acid hypersecretion after therapy terol. 2012 Jul;5(4):219-32.
with proton pump inhibitors. Aliment Pharmacol
Ther. 2007 Jan 1;25(1):39-46.
Kim J, Blackett JW, Jodorkovsky D. Strate- Question 33
gies for Effective Discontinuation of Proton Pump A 60-year-old male had voice hoarseness and
Inhibitors. Curr Gastroenterol Rep. 2018 May presented to ENT. A direct laryngoscopy demon-
16;20(6):27. strated redness consistent with laryngopharyngeal
reflux, and he was started on BID PPI. He denies
heartburn,dysphagia or any symptoms related to
Question 32 meals. He has worsening voice hoarseness and
A 60-year-old male was discharged from the hospi- otherwise no change in any symptoms after three
tal three weeks ago after an Upper GI bleed. He had months of PPI therapy. EGD was negative for
eaten a seafood salad and had repeated episodes of esophageal pathology.
vomiting, the last of which contained coffee ground
material. His hemoglobin and hemodynamic were What is the next best step?
stable during the whole hospitalization. An EGD
showed no active bleeding and demonstrated a A. Continue PPI BID
healing Mallory-Weiss tear. He was discharged B. Refer for antireflux surgery
home on Omeprazole. He is asymptomatic. C. Wireless pH monitoring, over 96 hours,
on therapy
What is the next best step? D. Wireless pH monitoring, over 96 hours,
off therapy
A. Discontinue PPI E. Repeat EGD with esophageal
B. Repeat EGD to check for healing biopsies
C. Check for H. Pylori infection
D. Barium swallow CORRECT ANSWER: D

CORRECT ANSWER: A RATIONALE


An empiric trial of PPI therapy is a cost effective
way to see if there is a response to symptoms as an
18 Digestive Diseases Self-Education Program®

initial step to assess for GERD. Response to treat- RATIONALE


ment may be variable. On therapy testing (Answer This patient has globus sensation with a negative
C) is appropriate when the diagnosis of GERD is ENT evaluation and vague symptoms of dysphagia
not in question and the goal is to gauge adequacy with food slow to go down. High resolution ma-
of therapy. Off therapy monitoring (Answer D) nometry (Answer C) should be performed to rule
is appropriate when the diagnosis of GERD is out any major motor disorder and may frequently
uncertain, as in this case. The data for anti-reflux identify a non specific motility disorder. GERD is
surgery for treatment of extresophageal symptoms commonly associated with globus, but the patient
is weak and proper patient selection is challeng- is not clinically responding to BID PPI therapy.
ing. In this case, the diagnosis of GERD is not yet pH monitoring can establish if GERD is present;
established, thus referral for antireflux surgery however it is limited in assessing if it is the cause
is inappropriate. Given the lack of esophageal of symptoms. Addition of H2 blocker at night is
symptoms and the unremarkable endoscopy, the unlikely to provide additional benefit given the
esophageal biopsies (Answer E) are unnecessary. lack of any response with PPI therapy (Answer D).
This would be more appropriate to exclude eosino- Antireflux surgery is not indicated in this patient
philic esophagitis if suspected. without clear presence of GERD(Answer E). In
small studies, ablation with either argon plasma
REFERENCE coagulation or radiofrequency ablation (RFA) of
Vaezi MF, Katzka D, Zerbib F. Extraesophageal the cervical gastric inlet patch has been associated
Symptoms and Diseases Attributed to GERD: with improvement of globus symptoms in PPI
Where is the Pendulum Swinging Now? Clin Gas- non-responsive patients, but there is no role for
troenterol Hepatol. 2018 Jul;16(7):1018-1029. RFA of the distal esophagus in this case (Answer
A). All organic causes should be ruled out prior
to entertaining globus as a functional disorder
Question 34 (Answer B).
A 35-year-old female has a constant sensation
that something is in her throat. She denies trouble REFERENCE
swallowing, but states that food is slow to go Tawil J, Fass R. Globus: Current Concepts and
down. An ENT evaluation was unremarkable and Dilemmas. J Clin Gastroenterol.  2018 Nov/
she was started on BID proton pump inhibitor Dec;52(10):845-852
therapy. An EGD had an irregular Z line, but biop-
sies were normal. She denies any heartburn and
has not noticed any difference on proton pump Question 35
inhibitor therapy. 32-year-old male with a history of eosinophilic
esophagitis has presented to the emergency depart-
What is the next best step? ment three times over the past year for a food im-
paction. He comes to you for a second opinion, and
A. EGD with Radiofrequency ablation reports intermittent dysphagia, but no pyrosis. He
of the Z line is taking swallowed fluticasone 440mcg twice daily
B. Referral to a health psychologist and pantoprazole 40mg daily.
C. High resolution manometry His last endoscopy two months ago showed a
D. Add H2 blockers at night stricture at 25cm, which would not allow passage of
E. Anti reflux surgery the scope. The remainder of his esophagus revealed
mild linear furrowing, but no other dominant stric-
CORRECT ANSWER: C tures or fixed rings. Biopsies of the stricture and
proximal esophagus did not have any eosinophils.
Chapter 1 — Esophageal disorders 19

What is the next best step in the management of Question 36


this patient? You are performing an upper endoscopy in a pa-
tient with long-standing GERD symptoms. You see
A. Increase fluticasone to 880mcg swallowed the diaphragmatic impression at 40 cm, the top of
twice daily the gastric folds at 37 cm, circumferential salmon
B. Change pantoprazole to omeprazole colored mucosa extending proximally to 35 cm,
C. Dilate the stricture using a 8-10mm through two tongues of salmon colored mucosa from 35 cm
the scope balloon to 32 cm and a salmon colored island at 31 cm.
D. Dilate the stricture using a 54 French Savary How would you document the segment length ac-
dilator cording the to Prague classification?
E. Dilate the stricture using a 18-20mm through
the scope balloon A. C2M3I1
B. C2M5
CORRECT ANSWER: C C. C5M8
D. C5M9
RATIONALE E. H3C2M3
This patient has EoE with a fixed ring/stricture,
that is causing recurrent food impactions. There is CORRECT ANSWER: B
no evidence of active inflammation or eosinophilic
infiltration, thus there is no indication to alter his RATIONALE
medical management. Dilation therapy does not al- The Prague Classification is a validated way to
ter the underlying inflammatory process, but is rec- document measurements in a Barrett’s segment.
ommended for dominant strictures/rings, as long According to the AGA consensus recommen-
as the inflammation is decently controlled. If care is dations on Barrett’s esophagus, this is a qual-
taken to avoid excessive dilation in any one treat- ity indicator and should be documented in all
ment session, the risk of perforation is less than 1 endoscopy reports with Barrett’s esophagus is
percent. The passage of single large-caliber (54–60 suspected. The Barrett’s segment is measured
French) bougies to treat a seemingly isolated distal from the top of the gastric folds to the circumfer-
ring in EoE can be hazardous, because endoscopi- ential extent and designated with the letter C and
cally unrecognized stenoses may coexist proximal- then to the maximal contiguous extent desig-
ly; dilation with graduated balloon catheters is safer nated with the letter M. Therefore in this ex-
in this instance. Since this patient’s esophageal ample, the circumferential extent is the distance
diameter is less than 9mm (the approximate outer between 37cm and 35cm is 2cm which would
diameter of standard upper endoscope), it is rea- be designated as C2 and maximal extent is the
sonable to begin with an 8mm balloon. Given the distance between 37 and 32 is 5 cm which would
fibrotic nature of the esophageal mucosa in EoE, be designated as M5 (Answer B).
excessive dilation is not recommended, and can
lead to perforation or large rents causing significant REFERENCE
chest pain; therefore choice E is incorrect. Sharma P, Katzka DA, Gupta N, et al. Qual-
ity indicators for the management of Barrett’s
REFERENCE esophagus, dysplasia, and esophageal adenocar-
Dellon ES, Gonsalves N, Hirano I et al. ACG Clini- cinoma: international consensus recommenda-
cal Guideline: Evidenced Based Approach tions from the American Gastroenterological
to the Diagnosis and Management of Esophageal Association Symposium. Gastroenterology
Eosinophilia and Eosinophilic Esophagitis (EoE). 2015;149:1599-606.
Am J Gastroenterol 2013; 108:679–692.
20 Digestive Diseases Self-Education Program®

Question 37 Patients with Barrett’s esophagus and persistent


A 63-year-old male underwent a surveillance endos- low-grade dysplasia have an increased risk for
copy and was found to have low-grade dysplasia on high-grade dysplasia and cancer. Clin Gastroen-
biopsies of a 4 cm segment of Barrett’s esophagus. terol Hepatol 2016;14:956-962.
What should you do next? Vennalaganti P, Kanakadandi V, Goldblum
JR, et al. Discordance among pathologists in the
A. Arrange for radiofrequency ablation united states and europe in diagnosis of low-grade
B. Refer for EUS dysplasia for patients with Barrett’s esophagus.
C. Confirm dysplasia with an expert Gastroenterology 2017;152:564-570.
gastrointestinal pathologist Shaheen NJ, Falk GW, Iyer PG, Gerson
D. Refer for antireflux surgery LB; American College of Gastroenterology.ACG
E. Start PPI therapy and repeat endoscopy in Clinical Guideline: Diagnosis and Management
three years of Barrett’s Esophagus. Am J Gastroenterol. 2016
Jan;111(1):30-50.
CORRECT ANSWER: C

RATIONALE Question 38
There is high interobserver variability among A 60-year-old male has a 3 cm segment of Bar-
pathologists in the diagnosis of dysplasia in Bar- rett’s esophagus with a visible lesion.
rett’s esophagus. Therefore, a pathologist with
gastrointestinal expertise should confirm the Which of these lesions is the least likely to be
diagnosis of dysplasia (Answer C). Confirmed low cured by endoscopic therapy?
grade dysplasia may be managed with either en-
doscopic therapy such as radiofrequency ablation A. A raised 1.5 cm area with a regular mucosal
(Answer A) or with close surveillance. A surveil- pattern and increased vascular pattern
lance interval of three to five years is appropriate B. A 1 cm ulcerated area with loss of mucosal
for non-dysplastic Barrett’s, but not in the setting pattern and surrounding irregular vascular
of current low grade dysplasia (Answer E). If the pattern.
patient does not undergo endoscopic eradica- C. A 1 cm flat area with irregular mucosal and
tion therapy, surveillance endoscopy should be vascular pattern
performed every 6 months for one year and then D. An 8 mm polypoid lesion with a villous
annually until there is reversion to non-dysplastic mucosal pattern
Barrett’s. EUS is useful for evaluation of lymph E. A 2 cm slightly elevated area with a
node metastasis, but not in the staging of superfi- cerebriform mucosal pattern
cial metaplasia (Answer B). The decision to pursue
anti-reflux surgery should be based on symptoms, CORRECT ANSWER: B
and the dysplasia would need to be addressed first
(Answer D). RATIONALE
A depressed lesion has the highest risk of sub-
REFERENCES mucosal invasion, which is associated with a 20
Duits L, van der Wel M, Cotton C, et al. Patients percent or greater risk of lymph node metastasis
with Barrett’s esophagus and confirmed persis- and may not be amenable to endoscopic therapy
tent low-grade dysplasia are at increased risk (Answer B). Additionally, irregular mucosal pat-
for progression to neoplasia. Gastroenterology terns and irregular vascular patterns are more
2017;152:993-1001. consistent with neoplasia then those that are
Kestens C, Offerhaus GJ, van Baal JW, et al. round, tubular, villous, or cerebriform.
Chapter 1 — Esophageal disorders 21

REFERENCES months for one year, and then yearly (Answer B).
Peters FP, Brakenhoff KP, Curvers WL, Rosmolen EUS is not warranted in confirmed high-grade dys-
WD, Fockens P, ten Kate FJ, Krishnadath KK, plasia without any evidence of cancer (Answer E).
Bergman JJ. Histologic evaluation of resection
specimens obtained at 293 endoscopic resections REFERENCE
in Barrett’s esophagus. Gastrointest Endosc. 2008 Desai M, Saligram S, Gupta N, Vennalaganti P,
Apr;67(4):604-9. Bansal A, Choudhary A, Vennelaganti S, He J, Titi
Sharma P, Bergman JJ, Goda K, Kato M, M, Maselli R, Qumseya B, Olyaee M, Waxman I,
Messmann H, Alsop BR, Gupta N, Vennalaganti P, Repici A, Hassan C, Sharma P. Efficacy and safety
Hall M, Konda V, Koons A, Penner O, Goldblum outcomes of multimodal endoscopic eradication
JR, Waxman I. Development and Validation of therapy in Barrett’s esophagus-related neoplasia:
a Classification System to Identify High-Grade a systematic review and pooled analysis. Gastroin-
Dysplasia and Esophageal Adenocarcinoma in test Endosc. 2017 Mar;85(3):482-495.e4.
Barrett’s Esophagus Using Narrow-Band Imaging.
Gastroenterology. 2016 Mar;150(3):591-8. doi:
10.1053/j.gastro.2015.11.037. Epub 2015 Nov 25. Question 40
PubMed PMID: 26627609. A 70-year-old male was treated for a nodule in the
setting of a 5 cm segment of Barrett’s esophagus.
The nodule was high grade dysplasia and resected
Question 39 in entirety with negative margins. The remainder
A 57-year-old male had a 4 cm segment of Bar- of the segment underwent radiofrequency ablation
rett’s with a nodule identified and resected by for a total of five sessions, with still 50 percent of
endoscopic mucosal resection. The pathology was the Barrett’s segment present. Biopsies of the pre-
consistent with high-grade dysplasia and margins viously treated areas were negative for dysplasia,
were negative. There was no evidence of cancer. but confirmed persistent metaplasia.

What is the next step? What therapeutic options for the Barrett’s mucosa
may be considered at this point?
A. Continue PPI BID indefinitely
B. Repeat EGD and biopsies yearly A. Completion endoscopic submucosal dissection
C. Biopsy the rest of the segment and treat if B. Photodynamic therapy
any residual dysplasia C. Repeat radiofrequency ablation regimen
D. Radiofrequency ablation D. Esophagectomy
E. Endoscopic ultrasound E. Cryotherapy

CORRECT ANSWER: D CORRECT ANSWER: E

RATIONALE RATIONALE
Due to the risk of metachronous and synchronous After five sessions, the patient is a radiofrequency
lesions, the standard approach is to resect any ablation (RFA) failure, thusrepeated RFA is un-
visible lesion in the setting of dysplasia and then likely to eradicate the Barrett’s segment (Answer
treat the remainder of the at-risk epithelium with C). Cryotherapy has been reported as a salvage
ablative modes such as radiofrequency ablation treatment after RFA failures (Answer E) for the
(Answer D). After therapy, surveillance is typically goal of total Barrett’s eradication. It does not have
more rigorous for high-grade dysplasia with some the long-term data as present with RFA and does
proposed intervals of three months for one year, six carry a more favorable risk profile than radical en-
22 Digestive Diseases Self-Education Program®

doscopic mucosal resection or endoscopic submu- yearly (Answer A) and not yet yearly or as needed
cosal dissection (answer A). Photodynamic therapy (Answers C, E). EUS in confirmed high grade dys-
(Answer E) has largely fallen out of favor due to its plasia without any evidence of cancer is of limited
adverse event profile. Given the high grade dysplai- yield given the zero rate of lymph node metastasis
sa is cleared and there is no presence of neoplasia, in high grade dysplasia (Answer D). PET/CT (An-
esophagectomy (Answer D) is not indicated. swer B) is useful for detection of metastatic disease
for staging and monitoring cancer.
REFERENCE
Sengupta N, Ketwaroo GA, Bak DM, Kedar V, REFERENCES
Chuttani R, Berzin TM, Sawhney MS,Pleskow DK. Krishnamoorthi R, Singh S, Ragunathan K, A
Salvage cryotherapy after failed radiofrequency Katzka D, K Wang K, G Iyer P. Risk of recur-
ablation for Barrett’s esophagus-related dysplasia rence of Barrett’s esophagus after successful
is safe and effective. Gastrointest Endosc. 2015 endoscopic therapy. Gastrointest Endosc. 2016
Sep;82(3):443-8. doi: 10.1016/j.gie.2015.01.033. Jun;83(6):1090-1106.e3.
Epub 2015 Apr 14. Stier MW, Konda VJ, Hart J, Waxman I.
Post-ablation surveillance in Barrett’s esophagus:
A review of the literature. World J Gastroenterol.
Question 41 2016 May 7;22(17):4297-306.
A 65-year-old male with a history of Barrett’s Shaheen NJ, Falk GW, Iyer PG, Gerson
esophagus with a nodule of high grade dysplasia LB; American College of Gastroenterology.ACG
was treated successfully by endoscopic mucosal Clinical Guideline: Diagnosis and Management
resection and subsequent radiofrequency ablation. of Barrett’s Esophagus. Am J Gastroenterol. 2016
Surveillance endoscopy reveals complete eradica- Jan;111(1):30-50.
tion of intestinal metaplasia.

What is the recommended surveillance strategy Question 42


for this patient? Which patient would benefit most from endoscop-
ic screening for Barrett’s esophagus?
A. Repeat endoscopy every three months for a
year and then every six months and then A. 50-year-old Asian female with history of
yearly intestinal metaplasia on biopsies of GEJ
B. PET / CT yearly junction and H. Pylori infection
C. Upper endoscopy in one year B. 60-year-old Caucasian male with BMI 40 and
D. Upper endoscopy in three years history of smoking who has denies any
E. No further surveillance necessary heartburn symptoms and has been on
omeprazole for eight years
CORRECT ANSWER: A C. 55-year-old African American male with daily
alcohol history
RATIONALE D. 70-year-old female on daily PPI for mild to
The risk of recurrence after endotherapy is pres- moderate GERD
ent. Monitoring with endoscopy would allow E. 65-year-old Hispanic male with chronic GERD
for detection and any therapy for ideally early, with symptoms three times a week for 10 years
superficial recurrence. After Barrett’s endotherapy and negative endoscopy three years ago during
for HGD and early cancer, surveillance is typically a work up for anemia.
rigorous with proposed intervals of three months
for one year, six months for one year, and then CORRECT ANSWER: B
Chapter 1 — Esophageal disorders 23

RATIONALE CORRECT ANSWER: E


Patients with multiple risk factors for esophageal
cancer such as male gender, 50 years or older, RATIONALE
elevated BMI / central obesity, hiatal hernia, and Patients with multiple risk factors for esophageal
cigarette smoking who have chronic reflux either cancer such as male gender, age 50 and greater,
by symptoms or by routine PPI therapy benefit elevated BMI / central obesity, hiatal hernia, and
the most from screening (Answer B). Females are cigarette smoking who have chronic reflux either
much less likely to develop Barrett’s esophagus or by symptoms or by routine PPI therapy benefit
esophageal cancer and their risk should be deter- the most from screening. Progression in Bar-
mined on a case by case basis, based on their risk rett’s Esophagus may be assessed with the PIB
factors (Answer D). Patients of Asian ancestry score and takes into account Male sex, cigarette
may be more likely to require attention to risk for smoking, Barrett’s mucosa length, and confirmed
gastric cancer and intestinal metaplasia of biopsies low-grade dysplasia (Answer E). Short segments
at GEJ may actually reflect gastric pathology and are less likely to progress compared to long seg-
not risk of esophageal cancer (Answer A). African ments (Answer A). Patients who have had multiple
– American race and alcohol are associated with negative surveillance endoscopies are less likely to
squamous cell carcinoma and not Barrett’s esoph- progress than those recently diagnosed (Answer
agus or esophageal adenocarcinoma (Answer C). C). African American race and alcohol history are
Additional endoscopic screening is not required associated with squamous cell carcinoma (Answer
after a negative endoscopy (Answer E). D). One should avoid performing biopsies in the
setting of inflammation, which could lead to an
REFERENCE overstaged diagnosis of dysplasia. Acid suppres-
Shaheen NJ, Falk GW, Iyer PG, Gerson LB. sion should be given for eight weeks and repeat
American College of Gastroenterology. ACG EGD with biopsies should be performed after the
Clinical Guideline: Diagnosis and Management esophagitis is healed to assess for true low-grade
of Barrett’s Esophagus. Am J Gastroenterol. 2016 dysplasia (Answer B).
Jan;111(1):30-50.
REFERENCE
Parasa S, Vennalaganti S, Gaddam S, et al. Gastro-
Question 43 enterology 2018;154:1282-9.
Which patient has the highest risk for progression
to esophageal adenocarcinoma?
Question 44
A. 75-year-old female with ultrashort Barrett’s A 60-year-old female presented in cardiogenic
esophagus with no visible lesions shock and had a prolonged ICU course. Her previ-
B. 60-year-old male with 1 cm Barrett’s esopha- ous medical history and medications were not
gus with LA grade D erosive esophagitis and known at the time of presentation. On day seven
biopsies showing low-grade dysplasia in the ICU she had coffee ground material coming
C. 70-year-old female with 4 cm Barrett’s out of NG tube. An endoscopy demonstrated cir-
esophagus in surveillance for 12 years cumferentially ulcerated mucosa in the esophagus,
D. 55-year-old African American male with with patchy areas of black colored tissue. Which
daily alcohol history with intermittent factor contributed most to this acute presentation?
dysphagia
E. 55-year-old Hispanic male with 4 cm A. Missed dose of PPI
Barrett’s esophagus with low-grade B. Hypotension
dysplasia, confirmed C. Unknown history of eosinophilic esophagitis
24 Digestive Diseases Self-Education Program®

D. Mechanical ventilation Heller approach or peroral endoscopic myotomy


E. Binge drinking prior to presentation would be appropriate in a surgically fit patient
(Answer A) and botulinum toxin may be consid-
CORRECT ANSWER: B ered in a poor surgical candidate. Medications
such as calcium channel blockers and nitrates
RATIONALE (Answer C) are not definitive treatment options
This is acute esophageal necrosis,otherwise known for achalasia and not warranted in malignancy.
as black esophagus, which occurs with reflux, Additional information isneeded on the diagnosis
compromise of mucosa, and ischemia likely due to and prognosis prior to committing to a G tube
low flow states. (Answer E).

REFERENCE REFERENCE
Gurvits GE, Cherian K, Shami MN, Korabathina Zaninotto G, Bennett C, Boeckxstaens G, et al. The
R, El-Nader EM, Rayapudi K,Gandolfo FJ, Alsh- 2018 ISDE achalasia guidelines. Dis Esophagus.
umrany M, Patel H, Chowdhury DN, Tsiakos A. 2018 Sep 1;31(9).
Black esophagus: new insights and multicenter
international experience in 2014. Dig Dis Sci. 2015
Feb;60(2):444-53. Question 46
A 67-year-old female has had long standing dys-
phagia. She is on proton pump inhibitor therapy.
Question 45 She has several rings in her esophagus and narrow
A 70-year-old male presents with progressive dys- caliber throughout her esophagus. Biopsies do
phagia over the past four months and 30 pound not have any eosinophils and have intraepithelial
weight loss. A barium swallow demonstrates a lymphocytes around the papilla (up to 30 per hpf)
dilated esophagus with a bird’s beak appearance. with spongiosis.
What is the next best step?
What treatment do you recommend?
A. Referral for per-oral endoscopic myotomy
B. high resolution esophageal manometry A. Topical steroids and serial dilation
C. Calcium channel blocker B. Pneumatic balloon dilation
D. EGD C. Fluconazole
E. Placement of G tube D. Ganciclovir
E. Calcium Channel blockers
CORRECT ANSWER: D
CORRECT ANSWER: A
RATIONALE
RATIONALE
Achalasia and pseudoachalasia are on the dif- This patient has lymphocytic esophagitis charac-
ferential. Given the advanced age, progressive terized by peripapillary lymphocytosis and spon-
course, and significant weight loss, an endoscopy giosis. The exact definition has not been standard-
with careful attention to GEJ should be performed ized in terms of number of lymphocytes per hpf.
to rule out malignancy causing a pseudoachalasia Treatment is with proton pump inhibitor, steroids,
presentation (Answer D). Manometry should be and dilation when there are structural lesions (An-
done after the endoscopy to confirm and subtype swer A). The other choices are for primary motility
the achalasia. If achalasia is confirmed and malig- disorders (Answer B and E) or infectious etiologies
nancy is rule out, myotomy either with a modified (Answer C and D).
Chapter 1 — Esophageal disorders 25

REFERENCE REFERENCES
Rouphael C, Gordon IO, Thota PN. Lymphocytic Jobe BA, Kahrilas PJ, Vernon AH, et al. Endo-
esophagitis: Still an enigma a decade later. World scopic appraisal of the gastroesophageal valve
J Gastroenterol. 2017 Feb 14;23(6):949-956. after antireflux surgery. Am J Gastroenterol.
2004;99(2):233–43.
Abdelmoaty WF, Swanstrom LL. Endoscopic
Question 47 Evaluation of Post-Fundoplication Anatomy. Curr
A 47-year-old male patient had long standing Gastroenterol Rep. 2017 Aug 24;19(10):51.
GERD and is status post Nissen fundoplication Juhasz A, Sundaram A, Hoshino M, Lee TH,
five years ago. He was doing well until three Filipi CJ, Mittal SK. Endoscopic assessment of
months ago when he had frequent regurgitation failed fundoplication: a case for standardization.
and chest pain. An EGD demonstrates the top of Surg Endosc. 2011 Dec;25(12):3761-6.
the gastric folds at 35 cm and the diaphragmatic
impression at 40 cm. The scope traversed easily
into the stomach. An initial retroflexed view does Question 48
not show a visible wrap at the hiatus and closer A 57-year-old female with Raynaud’s syndrome
inspection demonstrates a wrap proximal to and scleroderma complains of dysphagia. Barium
the hiatus. swallow demonstrates a dilated, fluid-filled
esophagus. A high-resolution manometry is
What would you describe this as in your ordered.
endoscopy report?
What are the most likely findings on this study?
A. Slipped Nissen
B. Herniated fundoplication A. High IRP, Absent contractility
C. Tight Wrap B. High IRP, Premature contractions
D. Malpostioned wrap C. Normal IRP, Premature contractions
E. Complete disruption D. Normal IRP, Hypercontractile contractions
E. Low IRP, Absent contractility
CORRECT ANSWER: B
CORRECT ANSWER: E
RATIONALE
An endoscopy and barium imaging are useful in RATIONALE
identifying fundoplication failures. This is a case Patients with scleroderma are likely to have major
where the wrap has migrated above the hiatus and motility disorders of the esophagus. They can
caused a trans-diaphragmatic wrap herniation have a patulous lower esophageal sphincter and
(Answer B). Given the patient is symptomatic, this dilated, fluid-illed esophagus in later stages of the
may warrant redo fundoplication. Other failures disease. The high-resolution manometry findings
include slipped Nissen or malpositioned wrap, in in scleroderma esophagus include absent contrac-
which the gastric folds are seen proximal to the tility with a low IRP sphincter (Answer E). Acha-
wrap impression and the wrap is seen around lasia subtypes would have a high IRP and absent
the stomach rather than the GEJ (Answer A, D), contractility (Answer A, classic Type I subtype) or
a tight wrap, in which passage of the scope may premature contractions (Answer B, spastic type
encounter resistance, and proximal dilation may III subtype). Answer C reflects possible diffuse
be seen (Answer C), or complete loss of the folds of esophageal spasm and Answer D may reflect Jack-
the fundoplication (Answer E). hammer esophagus.
26 Digestive Diseases Self-Education Program®

REFERENCES definitive treatment of their achalasia, if they are


Kahrilas PJ, Bredenoord AJ, Fox M, Gyawali CP, surgical candidates. Botox injection is temporary,
Roman S, Smout AJ, Pandolfino JE; International and should only be offered if the patient is not a
High Resolution Manometry Working Group. surgical candidate. Botox does cause scarring, so
The Chicago Classification of esophageal motility multiple injections could make subsequent surgery
disorders, v3.0. Neurogastroenterol Motil. 2015 more difficult. Nitrates are unlikely to help in this
Feb;27(2):160-74. situation (AnswerE)
Denaxas K, Ladas SD, Karamanolis GP. Evalu-
ation and management of esophagealmanifesta- REFERENCE
tions in systemic sclerosis. Ann Gastroenterol. Zaninotto G, Bennett C, Boeckxstaens G, et al. The
2018 Mar-Apr;31(2):165-170. 2018 ISDE achalasia guidelines. Dis Esophagus.
2018 Sep 1;31(9).

Question 49
A 64-year-old female patient with mild COPD Question 50
underwent a Heller myotomy 20 years ago for the A 35-year-old female has been having daily dyspha-
treatment of achalasia. She did well until three gia, regurgitation, and chest pain for six months.
years ago, when she had daily dysphagia, regurgi- She had no previous heartburn. She lost five
tation and drooling at night. An endoscopy shows pounds. She had no response to daily proton pump
a fluid-filled, tortuous esophagus with mild resis- inhibitor therapy. She was being referred for work
tance to scope passage at the gastroesophageal up to consider a Nissen fundoplication. EGD with
junction. She had no improvement with a balloon wireless pH monitoring off therapy demonstrates
dilation to 20 mm. A barium swallow demon- no obvious strictures or masses. The LES was not
strates a severely dilated esophagus and manome- patulous, and there was no hiatal hernia. Biopsies
try demonstrates an above normal IRP with absent of distal and mid esophagus were normal. The time
contractility. spent with pH 4 or less was seven percent and the
Demeester score was 25. There was no correlation
What is the next best step? between reflux and symptoms. Manometry findings
demonstrated an Integrated relaxation pressure
A. Botox injection (IRP) of 22 (above normal), distal contractile in-
B. Pneumatic dilation tegral (DCI) with normal vigor, and no premature
C. Esophagectomy contractions. There were abnormal appearing swal-
D. Redo Heller myotomy lows with repetitive contractions and compartmen-
E. Nitrates talized pressurization with all swallows. A barium
swallow showed no masses or strictures and a
CORRECT ANSWER: B barium tablet hung up at the GEJ.

RATIONALE What is the likely diagnosis in this case?


This patient has recurrence of symptoms after
achalasia treatments and unfortunately has a late- A. Refractory GERD with failure to respond to
stage achalasia. Less invasive treatment (Pneu- PPI therapy
matic balloon dilation) (Answer B) should be pur- B. EGJ outflow obstruction as a primary motility
sued prior to more invasive treatments (per-oral disorder
endoscopic myotomy, Heller myotomy, esopah- C. Eosinophilic esophagitis
gectomy) (Answers C, D). Some patients may fail D. Reflux hypersensitivity
pneumatic dilation and may go on to require more E. Functional heartburn
Chapter 1 — Esophageal disorders 27

CORRECT ANSWER: B

RATIONALE

This patient has EGJ outflow obstruction. Achala-


sia subtypes and variants may sometimes present
early and not have classic findings of a dilated
esophagus, rosette appearance of lower esophageal
sphincter, or bird’s beak appearance of barium
study. There may be an elevated IRP with pre-
served peristalsis, which may represent an evolv-
ing achalasia. The elevated acid exposure noted
on pH monitoring more likely reflects stasis in the
setting of the distal esophageal obstruction rather
than true GERD (Answer A). This is not the clini-
cal presentation for EoE (Answer C) or esophageal
hypersensitivity (Answers D and E)

REFERENCE
Kahrilas PJ, Bredenoord AJ, Fox M, Gyawali CP,
Roman S, Smout AJ, Pandolfino JE; International
High Resolution Manometry Working Group.
The Chicago Classification of esophageal motility
disorders, v3.0. Neurogastroenterol Motil. 2015
Feb;27(2):160-74.
28 Digestive Diseases Self-Education Program®
Answers & critiques

CHAPTER 2

Acid diseases of the stomach


David A. Leiman, MD, MSHP, Amir Masoud, MD
and Sushila R. Dalal, MD

Question 1 reveals microcytic anemia and he undergoes a


The proton pump inhibitor with the lowest diagnostic colonoscopy, which is normal. He
potency is: subsequently has an upper endoscopy which
is notable for significant green bilious fluid in
A. Omeprazole the stomach, which is suctioned completely to
B. Pantoprazole reveal a large, hyperemic pedunculated antral
C. Rabeprazole polyp with eroded surface epithelium. The lesion
D. Lansoprazole is removed with hot snare polypectomy and
pathology confirms a hyperplastic polyp. These
CORRECT ANSWER: B polyps are proliferations of:

RATIONALE A. Gastric foveolar cells


Within class switching of proton pump B. Gastric parietal cells
inhibitors (PPIs) for patients with incomplete C. Gastric enterochromaffin-like cells
control of symptoms is frequently done in D. Gastric chief cells
clinical practice. For the management of
gastroesophageal reflux disease (GERD), this CORRECT ANSWER: A
practice can be “considered” according to
guidelines. More recent data suggest varying RATIONALE
potencies of PPI that might be responsible The patient has evidence of a hyperplastic
for some patient’s incomplete response. polyp, which can result from inflammatory
When measured as omeprazole equivalents, proliferation of mucous producing foveolar cells
pantoprazole has the lowest potency, followed by often in the setting of chronic bile exposure. This
lansoprazole, omeprazole and rabeprazole. finding frequently is associated with mucosal
atrophy, but may be seen in the absence of
REFERENCE autoimmune gastritis or H. pylori infection.
Graham, DY and Tansel, A. “Interchangeable Iron deficiency anemia and intermittent gastric
use of proton pump inhibitors based on outlet obstruction due to prolapsing of large
relative potency.” Clin Gastroenterol Hepatol. hyperplastic polyps has been described. Parietal,
2018;16:800-808. enterochromaffin-like (ECL) and chief cells
secrete gastric acid, histamine and pepsinogen,
respectively.
Question 2
A 76-year-old white man complains of dyspnea, REFERENCE
early satiety and abdominal pain. Blood work Shaib YH, Rugge M, et al. “Management of

29
30 Digestive Diseases Self-Education Program®

gastric polyps: an endoscopy-based approach.” REFERENCE


Clin Gastroenterol Hepatol. 2013;11:1374-1384. Stabler, S. “Vitamin B12 deficiency.” N Eng J Med.
2013;268:149-160.

Question 3
A 64-year-old white woman with hypothyroidism Question 4
and insulin-dependent diabetes is referred to A 77-year-old man with a history of peptic ulcer
gastroenterology for megaloblastic, normocytic disease underwent partial gastrectomy with a
anemia. She underwent an unremarkable gastrojejunostomy anastomosis (Billroth II)
colonoscopy. An upper endoscopy is notable for without vagotomy 30 years ago for refractory ulcer
pallorous mucosa and biopsies from the gastric disease. He has no other significant past medical
body revealed a paucity of oxyntic mucosa with history. Several years after surgery he developed
pseudopyloric metaplasia. There are no H. pylori dyspepsia, which was successfully treated with
organisms seen with routine or special stains. over-the-counter antacids. Given persistence
Serologic evaluation is expected to reveal B12 of symptoms he was placed on 20 mg daily
deficiency and the most sensitive test to confirm omeprazole. He continued to have intermittent
pernicious anemia is: episodes of severe epigastric abdominal pain
with occasional admissions for gastrointestinal
A. Schilling test bleeding therefore was placed on total cumulative
B. Anti-parietal cell antibody dose of 80 mg omeprazole daily. He undergoes
C. Anti-intrinsic factor antibody an upper endoscopy, revealing anastomotic and
D. Serum homocysteine levels jejunal ulcerations. A fasting serum gastrin level
was 195 pg/ml. The best next step in evaluation is:
CORRECT ANSWER: B
A. Secretin stimulation test
RATIONALE B. Octreotide nuclear medicine scan
Autoimmune metaplastic atrophic gastritis C. Empirically increase omeprazole to three
(AMAG) can occur in the context of times daily dosing
polyglandular autoimmune syndrome. When D. Exploratory laparotomy
present, AMAG can progress to pernicious
anemia that is characterized by low levels of CORRECT ANSWER: A
vitamin B12. Parietal cell dropout
and resultant loss of intrinsic factor (IF) is RATIONALE
required for B12absorption. As a result, both This patient most likely has persistent peptic
anti-parietal cell and anti-intrinsic factor ulcers due to retained antrum as a result of
antibodies are used to determine the cause of B12 his prior Billroth II operation. In this scenario,
deficiency. However, the sensitivity for the former a portion of the antrum is inadvertently left
is 80 percent compared to 50 percent for the behind after surgery and, chronically bathed in
latter. The Schilling test can provide insight into alkaline fluid in the excluded portion, leads to
the etiology of B12 deficiency, including problems unopposed gastrin hypersecretion and gastric
with IF production versus malabsorption; acid production. A secretin stimulation test can
however, it is rarely available and therefore is differentiate between hypergastrinemia from
not the ideal test. Serum homocysteine levels gastrinoma or retained antrum. If an octreotide
are highly sensitive to detect B12 deficiency but nuclear medicine scan were performed and
cannot identify a cause and may be affected by the negative, it would neither confirm the etiology nor
presence of renal insufficiency. exclude other diagnoses within the differential.
Chapter 2 — Acid diseases of the stomach 31

There is no evidence of perforation so surgery at REFERENCES


this time is premature and Kelly, JP, Kaufmann, DW et al. “Risk of aspirin-
increasing his PPI further does not treat the associated major upper-gastrointestinal bleeding
underlying predisposing cause. with enteric-coated or buffered product.” Lancet
1996;348:1413-1416.
REFERENCE Laine L and Jensen D. “Management of
Gibril F, Lindeman RJ, et al. “Retained gastric patients with ulcer bleeding. Am J Gastroenterol.
antrum syndrome: a forgotten, treatable, cause 2012;107:345-360.
of refractory peptic ulcer disease.” Dig Dis Sci
2001;46:610-617.
Question 6
In this patient described in the Question above,
Question 5 what is the most cost-effective management
A 73-year-old man with coronary artery disease option to reduce the risk of recurrent bleeding?
requiring coronary artery bypass grafting and
daily low dose plain aspirin is hospitalized with A. Switching to bolus intravenous PPI twice daily
acute anemia and melena. His aspirin is held and B. Continuous intravenous PPI infusion for 72 hours
he is placed empirically on intravenous proton C. Place on oral twice daily PPI
pump with continuous infusion. He undergoes D. Start histamine-2 receptor antagonist
upper endoscopy, which reveals a single eight
millimeter ulcer in the duodenal bulb with CORRECT ANSWER: C
a visible vessel. After successful endoscopic
therapy with epinephrine injection and the use of RATIONALE
hemoclips, he remains stable. Prior to discharge, Numerous studies have evaluated the superiority
he is recommended to resume aspirin therapy. of PPI over H2RA in raising intragastric pH
and maintaining a more basic environment.
Which intervention is most likely to reduce the More recent cost-effectiveness analyses have
risk of recurrent bleeding? shown that, once adequate hemostasis has been
achieved, switching to oral PPI does not diminish
A. Switching to low dose enteric coated aspirin effectiveness and is substantially less costly than
B. Maintenance on low dose aspirin with the intravenous administration.
addition of daily PPI
C. Switching to low dose buffered aspirin REFERENCES
D. Switching to low dose plain aspirin Sachar H, Vaidya K and Laine L. “Intermittent vs
continuous proton pump inhibitor therapy for high-
CORRECT ANSWER: B risk bleeding ulcers: a systematic review and meta-
analysis.” JAMA Int Med. 2014;174:1755-1762.
RATIONALE Laine L, Shah A and Bemanian M.
In patients 70 years or older with a history of “Intragastric pH with oral vs intravenous bolus
gastrointestinal bleeding and on chronic NSAIDs, plus infusion proton-pump inhibitor therapy in
the use of PPI can reduce the risk of recurrent patients with bleeding ulcers.” Gastroenterology.
bleeding. In the setting of an acute bleeding 2008;134:1836-1841.
episode, aspirin should resume within seven days Rodriguez EA, Donnath E et al. “Value of oral
of adequate hemostasis. However, there are no proton pump inhibitors n acute, non-variceal
advantages of enteric coated or buffered aspirin in upper gastrointestinal bleeding: a network meta-
reducing the risk of recurrent bleeding. analysis.” J Clin Gastro. 2017;51:707-719.
32 Digestive Diseases Self-Education Program®

Question 7 undergoes upper endoscopy. There is a single,


A 45-year-old woman with rheumatoid arthritis 2.5cm ulcerated lesion noted in the gastric body
complicated by chronic arthralgia is placed on but the remainder of the exam is otherwise
non-steroid anti-inflammatory drugs (NSAIDs) for normal. Biopsies are obtained from the lesion
symptom control. Which formulation is associated and reveal features consistent with a gastric
with the highest risk of peptic ulcer disease neuroendocrine tumors. Which type of gastric
complications? carcinoids have the highest malignancy potential?

A. Ibuprofen A. Type 1
B. Naproxen B. Type 2
C. Meloxicam C. Type 3
D. Diclofenac D. Type 4

CORRECT ANSWER: B CORRECT ANSWER: C

RATIONALE RATIONALE
The use of NSAIDs is independently associated Gastric carcinoids arise in enterochromaffin-
with an increased risk of gastrointestinal like (ECL) cells. Type 1 gastric carcinoids are the
ulceration and bleeding. The risk of ibuprofen is most common type of gastric carcinoids and are
lowest whereas the risk of naproxen is highest most commonly associated with chronic atrophic
among this group. gastritis. They generally have the lowest metastatic
risk at less than five percent. Type 2 gastric
REFERENCES carcinoids are the least common type of gastric
García-Rodríguez LA and Hernandez-Diaz carcinoids and associated with Zollinger-Ellison
S. “Relative risk of upper gastrointestinal syndrome and MEN-1 syndrome. They have
complications among users of acetaminophen a metastatic risk of approximately 10 percent.
and nonsteroidal anti-inflammatory drugs.” Sporadic, or type 3, gastric carcinoids have the
Epidemiology. 2001;12:570-576. highest malignant potential with a metastatic
Hernández-Díaz S and Rodríguez L. risk greater than 50 percent. There are no type 4
“Association between nonsteroidal anti- gastric carcinoids.
inflammatory drugs and upper gastrointestinal
bleeding/perforation: an overview of REFERENCES
epidemiologic studies published in the 1990s.” Delle Fave G, O’Toole, D, et al. “ENETS
Arch Intern Med. 2000;160:2093-2099. consensus guidelines update for gastroduodenal
de Abajo FJ and García-Rodríguez LA. neuroendocrine neoplasms.” Neuroendocrinology
“Risk of upper gastrointestinal bleeding and 2016;103:119–124.
perforation associated with low-dose aspirin as Basuroy R, Srirajaskanthan R et al. “Review
plan and enteric-coated formulations.” BMC Clin article: the investigation and management
Pharmacol. 2001;1:1. of gastric neuroendocrine tumors.” Aliment
Pharmacol Ther. 2014;39:1071-1084.

Question 8
A 63-year-old man without a significant past Question 9
medical history is admitted to the hospital A 35-year-old man without a significant past
with melena and acute anemia. He is placed on medical history is admitted to the trauma
intravenous proton pump inhibitor therapy and intensive care unit after sustaining multiple
Chapter 2 — Acid diseases of the stomach 33

injuries as a result of a motor vehicle crash. RATIONALE


Due to maxillofacial injuries he is placed on There are multiple effects of NSAIDs on the
mechanical ventilation and initially receives total gastrointestinal tract. Although they can lead to
parenteral nutrition. He is placed on stress-dose direct epithelial injury, it is primarily through COX-
steroids on admission. He developed mild renal mediated processes that NSAIDs can affect many of
insufficiency and later is found to have a new these gastroprotective effects, including decreasing
upper extremity deep venous thrombosis, for prostaglandin synthesis. Also, they upregulate
which he is placed on therapeutic anticoagulation. gastric acid production, reduce production of the
Which factor most contributes to his risk of gastric mucous layer and reduce gastric blood flow.
developing gastrointestinal bleeding during his
hospitalization? REFERENCE
Bjarnason I, Scarpignato C et al. “Mechanisms
A. Renal failure of damage to the gastrointestinal tract from
B. Use of corticosteroids nonsteroidal anti-inflammatory drugs.”
C. Mechanical ventilation Gastroenterology. 2018;154:500-514.
D. Use of anticoagulation
E. Use of parenteral nutrition
Question 11
CORRECT ANSWER: C Which site within the stomach is most likely to be
unbuffered by the ingestion of food in the post-
RATIONALE prandial state?
Use of mechanical ventilation for greater than
48 hours is an independent risk factor for A. Antrum
gastrointestinal bleeding in the context of critical B. Body
illness and warrants prophylaxis. Neither renal C. Pylorus
failure or the use of corticosteroids, anticoagulants D. Cardia
or parenteral nutrition is an independent risk
factor for developing gastrointestinal bleeding. CORRECT ANSWER: D

REFERENCE RATIONALE
Cook, DJ, Fuller, HD, et al. “Risk factors for Gastric acid production is stimulated by multiple
gastrointestinal bleeding in critically ill patients.” mechanisms, including via vagal inputs even at the
N Eng J Med 1994;330:377-381. thought of ingesting a meal. This acid production
is buffered during meal consumption. However, a
layer of unbuffered contents referred to as the acid
Question 10 pocket sits within the proximal stomach in the
Which of the following gastroprotective area of the cardia and fundus. Buffering of, and
mechanisms is affected by the use of non-steroid potentially physically inhibiting, this layer with
anti-inflammatory drugs (NSAIDs)? the use of alginates can reduce the symptoms of
gastroesophageal reflux after meals.
A. Decreased prostaglandin synthesis
B. Down-regulation of gastric acid production REFERENCE
C. Increased production of gastric mucous layer Rohof WO, Bennink, RJ et al. “An alginate-antacid
D. Improved gastric blood flow formulation localizes to the acid pocket to reduce acid
reflux in patients with gastroesophageal reflux disease.”
CORRECT ANSWER: A Clin Gastroenterol Hepatol. 2013;11:1585-1591.
34 Digestive Diseases Self-Education Program®

Question 12 Question 13
A 44-year-old woman with a family history A 30-year-old woman with a history of anxiety
of MEN-1 reports significant abdominal pain disorder presents to the gastroenterology clinic
and heartburn. She reports a 45 pound weight with epigastric abdominal bloating discomfort and
loss over the prior six months. She is tried on early satiety. Her pain has been present on most
standard dose proton pump inhibitor without days for the previous year, during which time
any substantive benefit and therefore undergoes she has been working to complete a PhD thesis
upper endoscopy. She is found to have erosive in astrophysics. There is mild nausea without
esophagitis, several cratered gastric and multiple vomiting, no relationship with food or bowel
shallow ulcerations in the distal duodenum movements and she has no other chronic medical
beyond the ampulla. She is diagnosed with problems. Her primary care provider treated her
Zollinger-Ellison syndrome (ZES) and is referred with proton pump inhibitors without success
for further management. and she has had a normal metabolic profile and
complete blood count. She was then referred for
Which of the following is a cause of weight gain open-access upper endoscopy and biopsies of
frequently seen after medical gastric acid control the small bowel and stomach were normal. She
in the management of ZES? now presents for a follow-up appointment. What
is the mechanism by which buspirone reduces
A. Appropriate fat and nutrient absorption symptoms in functional dyspepsia?
B. Direct cytoreductive properties of PPI
C. Increase in bowel movement frequency A. Reduction in gastric emptying time
D. Improvement in dietary intake B. Fundic relaxation and accommodation
C. Direct appetite stimulation
CORRECT ANSWER: A D. Direct anxiolytic properties

RATIONALE CORRECT ANSWER: B


The frequently observed weight loss in patients
with ZES is likely multifactorial. Direct catabolic RATIONALE
tumor effects contribute, particularly in patients Buspirone is a 5-HT1A agonist that has clinical
with metastatic disease. Importantly, the gastric efficacy for patients with anxiety and has been
acid hypersecretion and associated diarrhea can shown to be superior to placebo in alleviating
lead to fat and other nutrient malabsorption. Once symptoms of functional dyspepsia (FD).
patients with ZES are appropriately controlled The symptoms most improved include post-
with medical acid inhibition, there is a reduction prandial fullness, bloating and early satiety,
in bowel movement frequency. Abdominal pain rather than abdominal pain. There may be
does not always lead to a reduction in oral intake, several mechanisms by which the therapeutic
with many patients able to maintain their weight effects occur, and are likely independent of
leading up to a diagnosis. Although PPIs are the anxiolytic properties alone based on the
indicated and effective at controlling secondary lack of correlation between anxiety scores and
effects of ZES, they do not have primary anti- symptom improvement. Patients with FD showed
neoplastic effects. improvement in symptoms, which were associated
with a delay in liquid gastric emptying rate and an
REFERENCE increase in meal-induced gastric accommodation.
Riff, BP, Leiman DA, et al. “Weight gain
in Zollinger-Ellison syndrome after acid REFERENCES
suppression.” Pancreas. 2016;45:193-197. Tack, J, Janssen, P, et al. Efficacy of buspirone,
Chapter 2 — Acid diseases of the stomach 35

a fundus relaxing drug, in patients with dismiss these without discussion would not be
functional dyspepsia. Clin Gastroenterol Hepatol. appropriate.
2012;10:1239-1245.
Miwa H, Nagahara A, Tominaga K, et al. REFERENCE
Efficacy of the 5-HT1A agonist tandospirone Freedberg DE, Kim LS, Yang YX. “The risks
citrate in improving symptoms of patients with and benefits of long-term use of proton pump
functional dyspepsia: a randomized controlled inhibitors: expert review and best practice
trial. Am J Gastroenterol 2009;104:2779 –2787. advice from the American Gastroenterological
Association.” Gastroenterology. 2017;152:706-715.

Question 14
A 69-year-old woman with a history of Question 15
hypothyroidism, treated breast cancer and In patients with bleeding from a peptic ulcer,
gastroesophageal reflux disease presents to your which is the best management approach?
clinic. Her heartburn occurred most days of
the week and previously responded to 40mg of A. Epinephrine injection only to clean based
omeprazole, which she has been using daily for ulcer
the last 10 years. Her mother died of a myocardial B. Bipolar cautery only to pigmented spot
infarction in her 70s and she reports seeing a news C. Application of hemostatic clips only to
story implicating proton pump inhibitors with adherent clot
the development of cardiovascular disease. Your D. Combination therapy of epinephrine and
initial recommendation should be: hemostatic clips to non-bleeding visible vessel

A. Hold PPI now and start histamine-2 receptor CORRECT ANSWER: D


antagonists
B. Recommend upper endoscopy to exclude RATIONALE
alternative cause of reflux symptoms Stigmata of recent hemorrhage help guide
C. Reassure her that there are no associations management for bleeding peptic ulcers. The risk of
between cardiovascular disease and PPI use rebleeding is greatest for active bleeding (Forrest
D. Reduce dose to 20mg IA and IB). The risk is lower, in descending order,
for non-bleeding visible vessel (Forrest IIA),
CORRECT ANSWER: D adherent clot (Forrest IIB), flat pigmented spot
(Forrest IIC) and clean-based ulcers (Forrest
RATIONALE III). Given the high risk of rebleeding, patients
In this patient, the presence of typical esophageal with Forrest IA, IB and IIA ulcers should receive
symptoms (heartburn) that responded to PPI endoscopy therapy. Combination therapy is
is sufficient to make the diagnosis of GERD. significantly superior to epinephrine injection
Her symptom frequency is greater than two alone but combination therapy is not superior to
days per week, suggesting the use of chronic cautery or mechanical monotherapy.
acid suppression with PPI is appropriate but
this should be at the lowest dose necessary to REFERENCES
manage symptoms. Therefore, a trial at 20mg Marmo R, Rotondano G, et al. “Dual Therapy
daily is correct. Although there are equivocal Versus Monotherapy in the Endoscopic Treatment
data regarding PPI use and associated risks that of High-Risk Bleeding Ulcers: A Meta-Analysis
include cardiovascular disease, several studies of Controlled Trials.” Am J Gastroenterol.
have reported such associations and to completely 2007;102:279-289.
36 Digestive Diseases Self-Education Program®

Laine, L and Jensen, DM. “Management of presentation: a systematic review. Obes Surg
patients with ulcer bleeding.” Am J Gastroenterol. 2014;24:299–309.
2012;107:345-360.

Question 17
Question 16 In the above patient, ulcer healing time can be
A 55-year-old man with type 2 diabetes, accelerated by which of the following?
hypertension and osteoarthritis underwent Roux-
en-Y gastric bypass for morbid obesity five months A. Re-do operation now with attempt and
ago. He reports losing 100 pounds from his altering pouch size
maximal weight but recently developed abdominal B. Ingestion of opened PPI capsules
pain and fatigue. A CT abdomen pelvis with C. Use of high dose oral PPI
contrast reveals expected post-surgical anatomy D. Use of sucralfate
but no other significant findings. He subsequently
undergoes an upper endoscopy revealing a linear CORRECT ANSWER: B
ulcer along the margin of the gastroenterostomy.
RATIONALE
Which of the following is a risk factor for the The rate of marginal ulcer after Roux-en-Y gastric
development of marginal ulcers? bypass can be as high as 16 percent. The risk
of developing these ulcers at the gastrojejunal
A. Weight loss of greater than 50 pounds after anastomosis is likely multifactorial, including
surgery size and location of the pouch, the presence of
B. Hypertension a gastrogastric fistula, the presence of a foreign
C. Long gastric pouch (suture) material or ischemia. There is also
D. PPI use evidence for acidity playing a role, with the
jejunum receiving acid chyme directly from the
CORRECT ANSWER: C stomach. Even when controlling for the use of
sucralfate and PPI dose, the ingestion of opened
RATIONALE PPI capsules was associated with a reduction
Relative ischemia to the gastroenterostomy is in healing time compared to standard oral
a primary risk factor for the development of PPI. Surgery to revise this patient’s bypass is
marginal ulcer in patients with a history of Roux- premature.
en-Y gastric bypass. This risk is increased in
patients who smoke, use NSAIDs or have diabetes. REFERENCE:
A short gastric pouch is associated with a reduced Schulman AR, Chan WW, et al. “Opened proton
risk of marginal ulcer. pump inhibitor capsules reduce time to healing
compared with intact capsules for marginal
REFERENCES ulceration follow Roux-en-Y gastric bypass.” Clin
Azagury DE, Aby Dayyeh BK, Greenwalt, IT, Gastroenterol Hepatol. 2017;15:494-500.
Thompson CC. “Marginal ulceration after Roux-
en-Y gastric bypass surgery: characteristics, risk
factors, treatment, and outcomes.” Endoscopy. Question 18
2011;43:950-954. A 60-year-old man with long-standing heartburn
Coblijn UK, Goucham AB, Lagarde SM, et al. undergoes upper endoscopy to screen for Barrett’s
Development of ulcer disease after Roux-en-Y esophagus. His symptoms have been well-
gastric bypass, incidence, risk factors, and patient controlled on proton pump inhibitors (PPIs) for 15
Chapter 2 — Acid diseases of the stomach 37

years. On examination of the stomach, there are have a clear relationship with eating or bowel
multiple pearly white polyps with a lacy reticular movements. It has not changed in character over
vascular pattern in varying sizes from three this time. An abdominal ultrasound was negative.
millimeters to two centimeters. She denies any heartburn or regurgitation but
reports frequent nausea. She had a colonoscopy
Which is a risk factor in the development of and upper endoscopy for iron deficiency three
dysplasia in fundic gland polyps? years ago, but these exams were normal without
any biopsies being taken.
A. Hereditary nonpolyposis colorectal cancer
(Lynch) syndrome Which testing modality would you recommend to
B. Small fundic gland polyp size assess for Helicobacter pylori infection?
C. The presence of antral gastritis
D. Barrett’s esophagus A. Urease breath test
B. Stool antigen
CORRECT ANSWER: C C. Serum antibody
D. Upper endoscopy with biopsies
RATIONALE
Fundic gland polyps are commonly encountered CORRECT ANSWER: A
during upper endoscopy, especially among
users of proton pump inhibitors. These polyps RATIONALE
are generally considered to have low risk for Histology is the gold standard for documenting
malignant transformation, which may be on the H. pylori infection but this patient already had
basis of chemopreventive properties of PPIs. an upper endoscopy and it would not be the most
However, dysplasia is associated with increasing cost-effective approach. Serum antibodies are
size of the largest fundic gland polyps, the non-specific for active infection. Urease breath test
presence of antral gastritis and in patients has marginally better performance characteristics
with familial adenomatous polyposis (FAP) (95 percent sensitivity and 95 percent specificity)
syndrome. Barrett’s esophagus is not associated compared to stool antigen (93 percent sensitivity
with the development of dysplastic fundic and 90 percent specificity). Although the
gland polyps. likelihood of celiac disease in this patient is low,
given the presence of iron deficiency it could be
REFERENCES concurrently evaluated; use of serologic screening
Bianchi LK, Burke CA, et al. “Fundic gland polyp would be more cost effective than repeating upper
dysplasia is common in familial adenomatous endoscopy with duodenal biopsies.
polyposis.” Clin Gastroenterol Hepatol.
2008;6:180-185. REFERENCES
Svoboda A, Tillisch-Svoboda S. “Barrett’s Gisbert JP and Pajares JM. “Review article:
patients developing gastric polyps during acid 13C-urea breath test in the diagnosis of
suppression are at greater risk of esophageal Helicobacter pylori infection—a critical review.”
dysplasia.” Endoscopy. 1996;28:S20. Aliment Pharmacol Ther 2004;20:1001-17.
Wright CL and Kelly JK. “The use of
routine special stains for upper gastrointestinal
Question 19 biopsies.” Am J Surg Pathol. 2006;30:357-61.
A 53-year-old African American woman presents Best LM, Takwoingi Y, et al. “Non-invasive
with epigastric abdominal pain. Her pain has diagnostic tests for Helicobacter pylori infection.”
been present for the last 12 months but does not Cochrane Database Syst Rev. 2018;3:Cd012080.
38 Digestive Diseases Self-Education Program®

Question 20 Question 21
A 63-year-old female patient has a urease A 73-year-old man with a previously placed
breath test that confirms active Helicobacter cardiac stent is on dual antiplatelet therapy
pylori infection. She reports a recent urinary with aspirin and ticagrelor. He recently strained
tract infection treated with ciprofloxacin and was his back playing tennis and has been using
prescribed azithromycin for bronchitis last year ibuprofen for pain relief. He presents to the
but reports no drug allergies. emergency department after the acute onset
of hematochezia and weakness but denies
What regimen would you recommend? any passage of blood clots, hematemesis or
abdominal pain. What is the best justification for
A. PPI + amoxicillin + clarithromycin nasogastric lavage?
B. PPI + bismuth + metronidazole +
tetracycline A. It is associated with reduced mortality
C. PPI + levofloxacin + amoxicillin B. It can reduce the need for additional
D. PPI + amoxicillin + rifabutin endoscopy
C. It is a well-tolerated test
CORRECT ANSWER: B D. It is the most effective way to discriminate
upper versus lower source of bleeding
RATIONALE
There are several potential reasons for treatment CORRECT ANSWER: B
failure in patients H. pylori, with antibiotic
resistance an increasingly important factor. RATIONALE
Amoxicillin resistance is estimated to be less The placement of nasogastric (NG) tubes is
than one percent, but this patient has multiple uncomfortable but if used in the appropriate
recent antibiotic drug exposures reducing the context can provide benefit. In patients with
likelihood of success with triple therapy. An suspected large volume upper GI bleeding, it
optimal up-front strategy would include a 14- can be used to clear the gastric fundus of blood
day regimen of quadruple therapy and saving and improve diagnostic visualization. Studies
a regimen with rifabutin in the setting of evaluating the performance characteristics of
treatment failure or salvage. NG lavage reveal a significant false negative rate
that can be up to 15 percent, limiting its role as
REFERENCES an optimal discriminator between upper and
Fallone CA, Chiba N, et al. “The Toronto lower GI bleeding. The use of NG lavage has not
Consensus for the Treatment of Helicobacter been shown to reduce mortality in the setting of
pylori Infection in Adults. Gastroenterology. GI bleeding.
2016;151:51-69 e14.
Malfertheiner P, Megraud F et al. REFERENCES
“Management of Helicobacter pylori infection- Scott, L and Kearney, D. “A Randomized
the Maastricht V/Florence Consensus Report.” Controlled Trial of Gastric Lavage Prior
Gut. 2017;66:6-30. to Endoscopy for Acute Upper Gastrointestinal
Chey WD, Leontiadis GI, Howden CW, Bleeding.” J Clin Gastro. 2004;38:861-865.
Moss SF. “ACG Clinical Guideline: Treatment Huang ES, Karsan S et al. “Impact of
of Helicobacter pylori Infection.” Am J nasogastric lavage on outcomes in acute GI
Gastroenterol. 2017;112:212-39. bleeding.” Gastrointest Endosc. 2011;74:971-980.
Pallin DJ and Saltzman JR. “Is nasogastric
tube lavage in patients with acute upper GI
Chapter 2 — Acid diseases of the stomach 39

bleeding indicated or antiquated?” Gastrointest CORRECT ANSWER: A


Endosc. 2011;74:981-984.
RATIONALE
Helicobacter pylori is the most common
Question 22 infection worldwide and a frequent cause of
What is the single most important reason non-ulcer dyspepsia, though its incidence
for proton pump inhibitor failure in and prevalence are not uniformly distributed.
gastroesophageal reflux disease? Although the United States as a whole has
relatively lower prevalence compared to many
A. Helicobacter pylori infection other countries, this finding is experienced
B. Rapid metabolism primarily within higher socioeconomic
C. Poor compliance with inappropriate subgroups. African Americans and people of
administration Latino ethnic background have documented
D. Delayed gastric emptying prevalence approaching those of developing
countries. Those with first degree relatives
CORRECT ANSWER: C with H. pylori are at increased risk and, given
the association between H. pylori and gastric
RATIONALE cancer, people with a family history of gastric
There are many potential reasons for PPI failure cancer would be expected to benefit from
in patients with symptoms of gastroesophageal testing. Immigrants from Germany would not be
reflux. However, the single most important reason expected to have a high pretest probability for H.
is inappropriate drug administration. Patients pylori infection and therefore screening in this
should be counseled to take their medication 30- group would not be appropriate.
60 minutes prior to meals for optimal physiologic
gastric acid inhibition, with the morning meal REFERENCE
favored over the evening meal due to relative more El-Serag HB, Kao, JY, et al. “Houston consensus
gastric acid production at this time. conference on testing for Helicobacter
pylori infection in the United States.” Clin
REFERENCE Gastroenterol Hepatol. 2018;16:992-1002.
Fass R, Shapiro M, Dekel R and Sewell J.
“Systematic review: proton-pump inhibitor
failure in gastro-oesophageal reflux disease Question 24
– where next?” Aliment Pharmacol Ther. A 76-year-old man with a 35-year smoking
2005;22:79-94. history, coronary artery disease, hypertension,
hyperlipidemia and obesity presents to the
emergency room with two days of intermittent
Question 23 dark black stools. On presentation he is
Which factor warrants testing for Helicobacter hypotensive with a blood pressure of 90/60
pylori infection? mm Hg and a pulse of 105 bpm, both of
which respond to IV fluid resuscitation. His
A. Family history of gastric cancer hemoglobin is 9.6g/dl from a prior baseline of
B. First generation immigrant from Germany 13g/dl and rectal exam is notable for melena.
C. Maternal uncle with a history of H. pylori He undergoes urgent endoscopy and there is
infection large volume blood and clot noted in the fundus
D. Previous upper endoscopy for dyspepsia with resultant limited views. After lavage and
with negative gastric antral and body biopsies suctioning to mobilize the clot, there is brisk
40 Digestive Diseases Self-Education Program®

immediate hemorrhage noted from a large attributes to food poisoning. She noted bright red
mass in the gastric cardia. What is the best next blood in her emesis over the last several episodes
endoscopic approach? of vomiting, which prompted presentation.
Routine vital signs and basic lab work is ordered.
A. Use of argon plasma coagulation (APC) Which scoring system will provide the single
B. Placement of hemoclips to assist best estimate of whether she would benefit from
interventional radiology in locating bleeding hospital admission and early endoscopy (within
source 24 hours)?
C. Use of hemostatic powder (TC-325)
D. Multimodal therapy with epinephrine A. The Glasgow-Blatchford score
injection and bipolar cautery B. The Rockall risk score
C. The Albumin, INR, mental status, systolic
CORRECT ANSWER: C blood pressure, and older age (AIMS-65) score
D. The Baylor bleeding score
RATIONALE
This patient has multiple risk factors for esophageal CORRECT ANSWER: A
cancer and it is reasonable to expect the observed
mass in the gastric cardia is malignant. This likely RATIONALE
represents a hemodynamically significant brisk Multiple scoring tools to stratify patients with
bleed and transferring the patient at this time upper GI bleeding exist. Unlike the Rockall
would be suboptimal. Instead, endoscopic wide score, which is better for assessing mortality risk,
area treatment would be ideal. Hemostatic powder the Glasgow-Blatchford score utilizes routinely
was recently approved in the United States and has obtained laboratory parameters and clinical history.
been shown to be effective in the management of It has been shown to have the best performance
bleeding from gastrointestinal tumors. There are no characteristics (99 percent negative predictive
trials comparing the effectiveness of argon plasma value) to exclude the need for early endoscopy in
coagulation (APC), which has been used to control patients found to have low risk scores.
tumor bleeding, with hemostatic spray. However,
the latter modality would likely be associated with REFERENCE
a smaller effective treatment area. Focal therapy, Laursen SB, Hansen JM, Schaffalitzky de
including dual modality treatment, is unlikely to Muckadell OB. “The Glasgow Blatchford Score
provide effective control of the bleeding. is the most accurate assessment of patients
with upper gastrointestinal hemorrhage.” Clin
REFERENCE Gastroenterol Hepatol. 2012;10:1130-1135.
Haddara S., Jacques J, et al. “A novel hemostatic
powder for upper gastrointestinal bleeding (the
‘GRAPHE’ registry).” Endoscopy. 2016:48:1084- Question 26
1095. A patient with pernicious anemia will be expected
to have the following:

Question 25 A. High gastrin, high gastric pH, increased


A 55-year-old woman with a history of migraines, parietal cell mass
hypertension and uterine fibroids presents to the B. Low gastrin, low gastric pH, positive secretin
emergency department with hematemesis. She stimulation test
reports multiple episodes of retching and dry C. High gastrin, low gastric pH, increased
heaves over the preceding 12 hours, which she parietal cell mass
Chapter 2 — Acid diseases of the stomach 41

D. High gastrin, high gastric pH, negative RATIONALE


secretin stimulation test People from southeast Asia have been shown
to have corpus predominant H pylori gastritis.
CORRECT ANSWER: D Corpus gastritis is accompanied by gastric atrophy
and consequently a high pH and high gastrin level.
RATIONALE Of note, intestinal metaplasia is often seen.
In pernicious anemia, there is a destruction of
parietal cells which leads to atrophic gastritis. This REFERENCES
will lead to a high gastric pH and consequently Roy PK, Venzon DJ, Shojamanesh H, et
high gastrin level. The secretin stimulation test al. Zollinger- Ellison syndrome. Medicine
will of course be negative in this setting. 2000;79:379-411.
Murugesan SV, Varro A, Pritchard DM.
REFERENCES Review article: Strategies to determine whether
Roy PK, Venzon DJ, Shojamanesh H, et hypergastrinaemia is due to Zollinger-Ellison
al. Zollinger- Ellison syndrome. Medicine syndrome rather than a more common benign
2000;79:379-411. cause. Aliment Pharmacol Ther. 2009;29:1055–68.
Murugesan SV, Varro A, Pritchard DM. Schubert ML, Kaunitz JD. Gastric Secretion.
Review article: Strategies to determine whether In: Sleisinger & Fordtran’s Gastrointestinal & Liver
hypergastrinaemia is due to Zollinger-Ellison Disease: Pathophysiology/Diagnosis/Management,
syndrome rather than a more common benign cause. 10th ed. Feldman M, Friedman LS, Brandt LJ, Eds.
Aliment Phar- macol Ther. 2009;29:1055–68. Philadelphia, PA: Elsevier Saunders, 2015.
Schubert ML, Kaunitz JD. Gastric Secretion.
In: Sleisinger & Fordtran’s Gastrointestinal &
Liver Disease: Pathophysiology/Diagnosis/ Question 28
Management, 10th ed. Feldman M, Friedman A 62-year-old retired lawyer presents with three
LS, Brandt LJ, Eds. Philadelphia, PA: Elsevier months of “chest burning” which is worse after
Saunders, 2015. heavy meals. He denies any dysphagia, weight
loss, early satiety or other alarm features. Your
next step in management is:
Question 27
A 35-year-old man from Southeast Asia presents A. Referral for cardiac workup
with chronic abdominal pain and nausea. B. Trial of six to eight weeks of daily omeprazole
Suspecting H pylori, you order a breath test which C. As needed ranitidine
is positive. Based on this information, you would D. Oral antacids (TUMS)
expect the following:
CORRECT ANSWER: A
A. High gastrin, high gastric pH, increased
parietal cell mass RATIONALE
B. Low gastrin, low gastric pH, positive secretin Anyone with chest pain should first undergo a
stimulation test cardiac workup before moving on to evaluation for
C. High gastrin, low gastric pH, increased GERD or GI disorders.
parietal cell mass
D. High gastrin, high gastric pH, decreased REFERENCE
parietal cell mass Noncardiac chest pain: diagnosis and
management. Current Opinion in
CORRECT ANSWER: D Gastroenterology 2017; 33(4): 293–300.
42 Digestive Diseases Self-Education Program®

Question 29 A. Order a CT scan


A 35-year-old gentleman who recently immigrated B. Stop PPI and recheck gastrin
from China presents with six months of abdominal C. Stop NSAIDS and continue PPI
bloating. You order an H pylori breath test D. Order a basal acid output (BAO)
which is positive. Based on this you prescribe the
following: CORRECT ANSWER: C

A. Clarithromycin, amoxicillin, metronidazole, RATIONALE


and omeprazole The patient most likely has NSAID induced
B. Amoxicillin, metronidazole, omeprazole, gastritis and ulcers. While the gastrin level is
bismuth elevated, it was post prandial and done on PPI—
C. Levofloxacin, metronidazole, omeprazole both of which can cause higher levels. A suspicion
D. Rifabutin, clarithromycin, omeprazole of ZES (imaging or BAO) is warranted but gastrin
levels are usually greater than 1000pg/ml and the
CORRECT ANSWER: A ulcers post bulbar. H pylori is a possibility and
should be tested but the first step here would be to
RATIONALE stop the NSAIDs.
A is the only correct combination of approved
regimens. It is important to note that antibiotic REFERENCES
resistance is on the rise and that triple therapy Roy PK, Venzon DJ, Shojamanesh H, et al.
is no longer effective in a large subset of patients Zollinger- Ellison syndrome: clinical presentation
including those from the far east. Current in 261 patients. Medicine 2000;79:379-411.
recommendations are for starting quadruple Murugesan SV, Varro A, Pritchard DM.
therapy in these patients. This is not one of Review article: Strategies to determine whether
the choices but would be the correct answer if hypergastri- naemia is due to Zollinger-Ellison
available. Of the choices above, A is an approved syndrome rather than a more common benign
regimen. Clarithromycin and metronidazole are cause. Aliment Pharmacol Ther. 2009;29:
the most prone to resistance. 1055-68.

REFERENCE
Chey WD, Leontiadis GI, Howden CW, Moss Question 31
SF. ACG Clinical Guideline: Treatment of A 40-year-old obese lady presents with six weeks
Helicobacter pylori Infection. Am J Gastroenterol. of severe post prandial abdominal pain. She
2017;112:212-239. underwent a gastric bypass two years ago and had
lost 60 pounds before starting to gain it back.
She is now back to her preoperative weight.
Question 30
A 62-year-old tennis instructor presents with You perform a detailed workup and find:
chronic post prandial abdominal pain. He had
been on PPI for two weeks with minimal relief. A. H. pylori gastritis
You perform an upper endoscopy and see multiple B. Atrophic gastritis
antral ulcers and erosions. A post prandial serum C. Elevated methyl-malonic acid levels
gastrin level is ordered and found to be 210pg/ml. D. Low folate

Based on this your next step is: CORRECT ANSWER: C


Chapter 2 — Acid diseases of the stomach 43

RATIONALE RATIONALE
H pylori may very well be seen as this may be It is important to note that enteric coated
encountered in asymptomatic patients but this is preparations of NSAIDs have not been shown to
not specific to bypass. It is important to note that reduce PUD. An important mechanism behind
after roux-en y bypass, vitamin B12 deficiency is this disease is the inhibition of prostaglandin
common (given the important role of the stomach synthesis (through inhibition of COX-1) which
in its absorption—see DDSEP study material). In play an important role in maintaining the
fact, it is often difficult to manage, not responding protective mucosal barrier. This effect is systemic
to standard multivitamin regimens. Folate and therefore enteric preparations would not help.
deficiency is not seen. While the co-incidence of H pylori and NSAID
intake has been shown to increase the risk of
REFERENCES PUD, the incidence of this infection is decreasing,
Blume CA, Boni CC, Casagrande DS, et al. not increasing. The patient does not have alarm
Nutritional profile of patients before and after features and is young which would make an
Roux-en-Y gastric bypass: 3-year follow-up. Obes “urgent” upper endoscopy premature and not
Surg 2012;22:1676–85. indicated.
von Drygalski A, Andris DA. Anemia after
bariatric surgery: more than just iron deficiency. REFERENCES
Nutr Clin Pract 2009;24:217–26. Talley NJ, Vakil NB, Moayyedi P. American
Gasteyger C, Suter M, Gaillard RC, et al. Gastroenterological Association technical review
Nutritional deficiencies after Roux-en-Y gastric on the evaluation of dyspepsia. Gastroenterol.
bypass for morbid obesity often cannot be prevented 2005;129:1756-80.
by standard multivitamin supplementation. Am J Kemmerly, T, Kaunitz, JD. Gastroduodenal
Clin Nutr 2008;87:1128–33. mucosal defense. Curr Opin Gastroenterol 2014.
Malinowski SS. Nutritional and metabolic Laine L, Jensen DM. Management of patients
complications of bariatric surgery. Am J Med Sci with ulcer bleeding. Am J Gastroenterol 2012;
2006;331:219–25. 107:345-360.

Question 32 Question 33
A 32-year-old accountant presents with two A 79-year-old gentleman on apixaban for atrial
months of gnawing abdominal pain. He has a fibrillation presents to the emergency department
history of migraines for which he takes ibuprofen with one day of light headedness and dark stools.
with good effect. Suspecting NSAID induced His initial vital signs include a supine blood
gastritis, erosions or ulcers you tell him: pressure of 110/60 mm Hg, heart rate of 98 bpm,
and respiratory rate of 12 per minute. His labs
A. He should discuss with his primary care show a BUN of 30 mg/dL and a hgb of 9g/dl. After
a switch to a different (non-NSAID) agent to obtaining adequate IV access your next step is to:
manage his migraines
B. He should switch to an enteric coated NSAID A. Transfuse platelets
C. The incidence of H pylori is increasing so he B. Transfuse prbc to a goal of 10g/dl
should be tested C. Start IV fluids
D. He needs an urgent upper endoscopy D. Transfuse FFP to reverse the anticoagulation

CORRECT ANSWER: A CORRECT ANSWER: C


44 Digestive Diseases Self-Education Program®

RATIONALE be to confirm the diagnosis. The physical exam


The patient has sufficient evidence of a GI bleed shows distension and a succession splash which go
and orthostatic vitals. The hemoglobin threshold along with GOO. An urgent surgical consultation
for transfusion would be under eight and therefore is probably premature and the next step would be
would not yet be indicated. IV fluid resuscitation imaging with a CT scan.
should always be undertaken in patient with GI
bleed. FFP or platelets would not reverse this agent. REFERENCE
Gibson JB, Behrman SW, Fabian TC, Britt LG.
REFERENCES Gastric outlet obstruction resulting from peptic
Villanueva C, Colomo A, Bosch A et al, ulcer disease requiring surgical intervention is
Transfusion strategies for acute upper infrequently associated with Helicobacter pylori
gastrointestinal bleeding. New Engl J Med 2013. infection. J Am Coll Surg. 2000 Jul. 191(1):32-7
368:1; 11-21.
Barkun AN, Bardou M, Kuipers EJ et al.
International consensus recommendations on the Question 35
management of patients with non-variceal upper An 82-year-old Chinese immigrant presents with
gastrointestinal bleeding. Ann Intern Med. 2010. severe abdominal pain, vomiting, and intermittent
152: 101-113. dark stool. His vital signs reveal hypotension and
Laine L, Jensen DM. Management of patients tachycardia and his CBC shows a hemoglobin of 8.5g/
with ulcer bleeding. Am J Gastroenterol 2012; dl. You perform an upper endoscopy and encounter
107:345-360. a dilated, fluid filled stomach, with significant pyloric
deformity secondary to a large cratered 3.5cm ulcer
without any signs of active bleeding (Forrest III). In
Question 34 this patient you would expect the following:
A 45-year-old who was recently admitted for an
upper GI bleed secondary to a giant pre-pyloric A. Elevated gastrin level, low gastric pH
ulcer is readmitted with abdominal discomfort, B. Low gastrin level, low gastric pH
nausea, and vomiting. Your exam reveals a C. Normal gastrin level, high gastric pH
distended abdomen with mild tenderness and a D. Low gastrin level, high gastric pH
“sloshing” sound. An abdominal x-ray (KUB) was
obtained in the emergency room showing no free CORRECT ANSWER: A
air. Of the following, the next best step is to:
RATIONALE
A. Urgent surgical consultation for suspected Patients with gastric outlet obstruction have
perforation elevated gastrin levels. This occurs due to the
B. Start IV PPI antral distension which leads to acetyl choline
C. Obtain a CT scan release. This stimulates the parietal cells to secret
D. Repeat EGD with possible dilation acid and interacts with the G cells to release
gastrin. You will therefore have a low pH and a
CORRECT ANSWER: C high (or normal?) gastrin level.

RATIONALE REFERENCES
The patient likely has a gastric outlet obstruction Fisher JC, Masiakos PT, Oviedo J, et al. Gastric
(GOO) secondary to the large ulcer which may outlet obstruction as a consequence of a duodenal
have caused edema and associated deformity. web masquerading as gastrinoma in an adult. Curr
While PPI should be started the next step would Surg 2003;60:602–6.
Chapter 2 — Acid diseases of the stomach 45

Malfertheiner P, Bayerdorffer E, Diete U, Gil stool.” You decide not to correct him and instead
J, Lind T, Misiuna P, et al. The GU-MACH study: inquire about the patient.
the effect of 1-week omeprazole triple therapy
on Helicobacter pylori infection in patients with Which piece of information will confirm your
gastric ulcer. Aliment Pharmacol Ther. 1999;13: suspicion of a stress ulcer?
703–712.
A. The patient was on a heparin drip for a DVT
B. She was intubated and placed on a ventilator
Question 36 last night for “respiratory failure”
A 59-year-old gentleman presents with one month C. This evening, she was started on
of abdominal pain. He notes a “few pounds” of hydrocortisone 50mg IV Q8H for suspected
weight loss but is “trying to eat better.” Your next adrenal insufficiency
step is: D. Her platelets are 21,000

A. H pylori testing CORRECT ANSWER: D


B. Trial of four to eight weeks of PPI
C. Schedule for an upper endoscopy RATIONALE
D. Obtain a serum gastrin level The only established independent risk factors
for stress ulcers were mechanical ventilation for
CORRECT ANSWER: B 48 hours or longer (patient was ~24 hours) and
coagulopathy (plt less than 50k or INR greater
RATIONALE than 1.5). Secondary risk factors include high dose
Current guidelines suggest performing an upper steroids--more than 250mg of hydrocortisone
endoscopy in patients 60 years of age or older (patient received one or two doses at most of
in the presence of dyspepsia and/or alarm 50mg), sepsis, large surface area burns, and acute
features. While the patient is close to that age, renal or liver failure.
he has no real alarm features and a short course
of PPI therapy is a reasonable option (also REFERENCE
potentially both diagnostic and therapeutic). Cook DJ, Fuller HD, Guyatt GH et al. Risk factors
If the patient has no improvement with PPI, for gastrointestinal bleeding in critically ill
HP testing should be obtained and treated if patients. N Engl J Med 1994;330:377–381.
positive. Serum gastrin level would not aid in
diagnosis or management.
Question 38
REFERENCE A 35-year-old obese lady underwent roux-en-y
Talley NJ, Vakil NB, Moayyedi P. American gastric bypass nine months ago. She lost 70
Gastroenterological Association technical review pounds already and was feeling great until about
on the evaluation of dyspepsia. Gastroenterol. four months ago. She complained of severe post
2005;129:1756-80. prandial abdominal pain that was worse with
meals. An upper endoscopy revealed a marginal
ulcer at that time. She was started on high
Question 37 dose PPI therapy for two months without any
You are called by the ICU attending in the middle improvement. You perform an upper endoscopy
of the night for assistance with a 45-year-old which again demonstrates the ulcer.
critically ill female patient who started having
“a GI bleed with copious amounts of melanotic Your next step is:
46 Digestive Diseases Self-Education Program®

A. Obtain a CT scan to evaluate for other causes Which of her medications puts her at the highest
of her pain. risk for PUD?
B. Surgical referral
C. Add on Carafate A. Iron
D. Start a neuromodulator B. Sumatriptan
C. Alendronate
CORRECT ANSWER: B D. Tamoxifen

RATIONALE CORRECT ANSWER: C


A patient with a marginal ulcer not responding
to PPI therapy should be seen by the surgeon RATIONALE
for possible intervention. Her pain is consistent Oral iron, and not infusions, are associated with
with the finding and therefore more imaging PUD. Sumatriptan alone, or tamoxifen, are not
or testing may be unnecessary (though, if an known to cause ulcers.
option, HP testing should be selected). Carafate
has not been shown to enhance healing of these REFERENCE
ulcers and there is no role for a neuromodulator Miyake K, Kusunoki M, Shinji Y, Shindo T,
in this case. Kawagoe T, Futagami S, Gudis K, Tsukui T,
Nakajima A, Sakamoto C. Bisphosphonate
REFERENCES increases risk of gastroduodenal ulcer in
ASGE standards of practice committee. Role rheumatoid arthritis patients on long-term
of endoscopy in the bariatric surgery patient. nonsteroidal antiinflammatory drug therapy. J
Gastrointest Endosc 2008;68:1–10. Gastroenterol. 2009;44(2):113.
Wilson J, Romagnuolo J, Byrne T, et al.
Predictors of endoscopic findings after
Roux-en Y gastric bypass. Am J Gastroenterol Question 40
2006;101:2194–2199. A 67-year-old gentleman on dual antiplatelet
Allison R. Schulman, Walter W. Chan, therapy presents with hematochezia and
Aiofe Devery, Michele B. Ryan, Christopher hemodynamic instability after “passing out” at
C. Thompson. Clinical Gastroenterology and home. His blood pressure is 80/50 and his blood
Hepatology 2017, Vol. 15. work reveals a hemoglobin of 6 g/dl (down from
13). You perform an immediate bedside upper
endoscopy which reveals a large gastric ulcer
Question 39 with a visible vessel and active oozing. You inject
A 63-year-old lady is admitted with abdominal with epinephrine and perform cautery with
pain and iron deficiency anemia. She reports good effect.
long standing anemia and a negative workup
in the past year including an upper endoscopy, Which of the following would have been most
colonoscopy, and video capsule endoscopy. She helpful in improving hemostasis?
was started on iron infusions with a modest
improvement in her anemia. Her other medical A. PPI drip prior to endoscopy
history includes osteoporosis, osteoarthritis for B. Use of clips instead of cautery
which she takes over the counter NSAIDs, breast C. CT angiogram
cancer (20 years ago treated with lumpectomy D. Urgent surgery
and local radiation), and migraines for which
she takes sumatriptan once or twice a month. CORRECT ANSWER: A
Chapter 2 — Acid diseases of the stomach 47

RATIONALE above 7g/dl. PPI drip should be started and has


PPI administration prior to therapy has been been shown to reduce the need for endoscopic
shown to decrease the need for endoscopic intervention which should only be undertaken
intervention (at the time of endoscopy). Clipping, after adequate resuscitation and, ideally, after PPI
cautery, APC, or injected therapies alone have is started. There are some cases were IR is needed,
not been shown to be superior to combination however an upper endoscopy can be undertaken
therapy. Surgery is not indicated and CTA is first in most cases.
not needed as the site was easily identified and
bleeding controlled. REFERENCES
Villaneuva C, Colomo A, Bosch A, et al.
REFERENCES Transfusion strategies for acute upper
Park CH, Lee SJ, Park JH, et al. Optimal injection gastrointestinal bleeding. N Engl J Med.
volume of epinephrine for endoscopic prevention 2013;368(1):11-21.
of recurrent peptic ulcer bleeding. Gastrointest Lau JY et al. Omeprazole before endoscopy in
Endosc. 2004;60:875–80. patients with gastrointestinal bleeding. N Engl J
Saltzman JR, Strate LL, Di Sena V, Med 2007 Apr 19; 356:1631-40.
et al. Prospective trial of endoscopic clips Ramaswamy RS, Choi HW, Mouser HC,
versus combination therapy in upper GI et al. Role of interventional radiology in
bleeding (PROTECCT–UGI bleeding) Am J the management of acute gastrointestinal
Gastroenterol. 2005;100:1503–8. bleeding. World Journal of Radiology.
Lau JY et al. Omeprazole before endoscopy in 2014;6(4):82-92.
patients with gastrointestinal bleeding. N Engl J
Med 2007 Apr 19; 356:1631-40.
Question 42
You see a 78-year-old male patient originally from
Question 41 Ecuador in hospital follow up. He was admitted
A 49-year-old gentleman is brought to the overnight with melena and acute anemia. The
emergency room by his wife after she “found him on-call gastroenterologist performed an upper
on the floor in a pool of black stool.” Initial blood endoscopy and found a large, non-bleeding
pressure is 70/30 mm Hg and HR 122 bpm. His (Forrest III), three centimeter cratered ulcer in
hemoglobin on labs is 6.6g/dl and BUN 64 mg/ the gastric antrum. No biopsies were obtained.
dL. Only one IV is secured. Your next step in In addition to PPI therapy what would you
management is: recommend?

A. Start PPI drip immediately A. An upper endoscopic ultrasound


B. Transfuse two units of packed red cells B. Serum gastrin level
C. Perform an urgent upper endoscopy C. Basal acid output (BAO)
D. Send directly to IR as “he is too unstable D. Repeat upper endoscopy in 12 weeks
to scope”
CORRECT ANSWER: D
CORRECT ANSWER: B
RATIONALE
RATIONALE A large ulcer (greater than 2.5cm—giant) is
Ideally the patient should have two large bore IVs concerning for cancer. While an EUS may be
but, in this setting, resuscitation is paramount. He needed eventually, PPI therapy followed by a
should be transfused to bring his hemoglobin to repeat upper endoscopy in 12 weeks should be
48 Digestive Diseases Self-Education Program®

undertaken to confirm healing (non-healing is the side effects of these medications and is asking
suspicious for malignancy). Ulcers in ZES are you your opinion: “will I go crazy or have a heart
typically post bulbar and serum gastrin level, at attack?!” You tell her:
this point, will not yet inform management.
A. There are some evidence that PPI therapy may
REFERENCE lead to dementia in certain populations
Kim HU. Diagnostic and treatment approaches B. Long term PPI therapy has been shown to
for refractory peptic ulcers. Clin Endosc. 2015;48: mildly increase the risk of heart disease
285-90. C. PPI therapy can cause C difficile infections in
critically ill patients (not her)
D. She may have some benign polyps in her
Question 43 stomach but nothing else
A 39-year-old lady presents with four months of E. There are potential risks with any long term
epigastric abdominal pain that are not responding medication and we will taper you down to the
to daily PPI, prescribed by her primary care. She lowest effective dose
undergoes an upper endoscopy which shows a
normal esophagus, stomach, and duodenum (to CORRECT ANSWER: E
the second part). What is the next best step?
RATIONALE
A. Refer for pH (reflux) testing PPI therapy has not been shown to be causative
B. increase PPI to twice daily for heart disease, dementia, pneumonia, C
C. Add on a night time H2 blocker difficile, or various other disorders and diseases
D. Start amitriptyline mentioned in the media. While there may be an
association in some cases the evidence is weak
CORRECT ANSWER: D to moderate at best. In general, these medicines
should only be used when necessary (this patient
RATIONALE has erosive esophagitis) and tapered to the lowest
This patient likely has non ulcer dyspepsia as effective (and necessary) dose.
demonstrated by her lack of response to PPI and
normal appearing endoscopy. It is important to note REFERENCE
that these patients typically do not respond to PPI (as Complications of Proton Pump Inhibitor Therapy.
compared to those with erosive esophagitis or PUD). Vaezi M.F., Yang Y.-X., Howden C.W. (2017)
Therefore, a neuromodulator should be considered. Gastroenterology, 153 (1), pp. 35-48.

REFERENCE
Savarino E, Pohl D, Zentilin P, et al. Functional Question 45
heartburn has more in common with functional A 32-year-old, otherwise healthy, gentleman
dyspepsia than with non-erosive reflux disease. presents with abdominal pain and diarrhea.
Gut. 2009;58:1185-91. His primary care provider orders stool studies
and an H pylori breath test, both of which are
negative. He undergoes an upper endoscopy
Question 44 which shows severe erosive esophagitis and
You see a 45-year-old lady in follow up. She has multiple duodenal ulcers. He is started on PPI
been on 40mg of omeprazole daily for six months therapy for six weeks and undergoes a repeat
for reflux complicated by erosive esophagitis (seen upper endoscopy when he sees no improvement.
on upper endoscopy). She is very nervous about Your next best test is:
Chapter 2 — Acid diseases of the stomach 49

A. gastric emptying test B. start triple therapy, substituting amoxicillin


B. serum gastrin level for metronidazole.
C. CT scan C. refer for allergy testing
D. (Upper) Endoscopic ultrasound D. manage his symptoms with Carafate and
follow with EGD for cancer screening
CORRECT ANSWER: B
CORRECT ANSWER: C
RATIONALE
This presentation raises the concern for ZES. RATIONALE
While gastroparesis may present with abdominal It is important to note that most cases of
pain and recalcitrant GERD, post bulbar ulcers penicillin allergy are not true allergies. Therefore,
are not seen. The lack of response to PPI suggests in patients who have failed one or two course of
an underlying disorder and further investigation HP therapy, referral for allergy testing should
should be undertaken, starting with a gastrin level be considered (given the efficacy of amoxicillin).
and basal gastric output and peak acid output Alternative strategies include Levofloxacin based
(following pentagastrin administration). It is therapies. There are no recommendations for
important to note that PPI therapy may elevate routine EGD in patients with H pylori infections
gastrin levels but not to the levels seen in ZES and a lack of supporting history (suggestive of
(typically greater than 1000). Ideally these tests increased cancer risk).
should be done off PPI but withdrawal should be
careful to prevent complications. REFERENCE
Chey WD, Leontiadis GI, Howden CW, Moss
REFERENCES SF. ACG Clinical Guideline: Treatment of
Roy PK, Venzon DJ, Shojamanesh H, et al. Helicobacter pylori Infection. Am J Gastroenterol.
Zollinger- Ellison syndrome: clinical presentation in 2017;112:212-239.
261 patients. Medicine 2000;79:379-411.
Murugesan SV, Varro A, Pritchard DM. Review
article: Strategies to determine whether hypergastri- Question 47
naemia is due to Zollinger-Ellison syndrome rather A 23-year-old female who is status post orthotopic
than a more common benign cause. Aliment Phar- heart transplant is admitted with abdominal pain,
macol Ther. 2009;29:1055-68. anorexia, nausea, and difficulty swallowing. She is
on mycophenolate and tacrolimus. An abdominal
ultrasound shows small gall stones but no ductal
Question 46 dilation or other clear abnormalities.
A 35-year-old gentleman from Thailand presents
with abdominal pain and found to have H pylori You perform an upper endoscopy and expect to
by breath test. He states that, as a child, he had find the following:
a rash to penicillin. You start him on 10 days of
quadruple therapy with bismuth, tetracycline, A. A large gastric ulcer, likely ischemic
metronidazole, and omeprazole. His symptoms B. Multiple shallow ulcers, possible viral
persist and a repeat breath test, four weeks after C. Erythema, erosions, and superficial ulcers,
completion of the regimen, is still positive. At this possibly NSAID induced
point you opt to: E. Normal upper endoscopy (her pain is related
to sickle cell disease)
A. Repeat the same regimen but increase the
duration to 14 days. CORRECT ANSWER: B
50 Digestive Diseases Self-Education Program®

RATIONALE on the evaluation of dyspepsia. Gastroenterol.


In immunocompromised patients and those who 2005;129:1756-80.
received multiple transfusions, the consideration
of viral infection, including CMV, is essential.
While the most common causes of ulcer disease Question 49
are H pylori and NSAIDs, additional testing Essential components of vitamin B12 absorption
should be ordered on these patients as these are include:
not routinely done on biopsy specimens (including
PCR and CMV immunohistochemistry). A. acid secretion by the chief cells to activate
intrinsic factor
REFERENCE B. release of cobalamin by bicarbonate and
Cytomegalovirus in upper gastrointestinal pepsin
ulcers. Murray RN, Parker A, Kadakia SC, Ayala C. secretion of intrinsic factor by the parietal
E, Martinez EM. J Clin Gastroenterol. 1994 cells before binding to b12 in the stomach
Oct;19(3):198-201. D. binding of cobalamin to intrinsic factor in the
duodenum under the effect of haptocorrin

Question 48 CORRECT ANSWER: D


A 44-year-old lady presents with two months
of abdominal pain that is worse with meals and RATIONALE
20 pound weight loss which she attributes to Acid is secreted from the parietal cells as is intrinsic
the hot weather and more walking. Your next factor. Cobalamin is released from protein by the
step is: effect of acid and pepsin. Intrinsic factor eventually
binds to B12 in the small intestine, not stomach. D
A. Start PPI therapy for eight weeks, for is the only correct answer.
dyspepsia
B. Consider starting a neuromodulator, for REFERENCE
functional abdominal pain Stabler SP. Clinical practice. Vitamin B12
C. Perform an upper endoscopy for further deficiency. N Engl J Med 2013;368:149-60.
evaluation
D. Order an H pylori breath test
Question 50
CORRECT ANSWER: C A 26-year-old presents with abdominal pain
diarrhea and headaches. Vital signs show a
RATIONALE blood pressure of 150/90 mm Hg and labs show
The patient likely has non reflux dyspepsia a calcium of 12mg/dl, creatinine of 1.6 mg/dL,
however the weight loss—even with her bicarb of 20 mEg/L, and hemoglobin of 15g/dl.
explanation—is concerning. She should undergo A CT scan with oral contrast is overall normal
an upper endoscopy. If negative, then you can but shows a suggestion of thickened gastric
weigh the option of empirically trying PPI vs wall. You perform an upper endoscopy which
H pylori testing. A neuromodulator would be shows thickened folds and multiple ulcers in the
indicated if all your workup is negative. duodenum to the second part. The patient has a
condition that is:
REFERENCE
Talley NJ, Vakil NB, Moayyedi P. American A. Autosomal recessive
Gastroen- terological Association technical review B. Likely related to tumors of the adrenal glands
Chapter 2 — Acid diseases of the stomach 51

C. May be associated with H pylori RATIONALE


D. May be associated with gynecomastia Women have shorted length of esophagus
regardless of their height, and have lower rates
CORRECT ANSWER: D of erosive esophagitis and Barrett’s esophagus
than men. While some have proposed different
RATIONALE standard for pH testing for reflux in men and
The patient has ZES that is part of MEN I. This is women, this has not yet been done. The LESP is
autosomal dominant (a is incorrect) and includes the same in women and men.
pituitary tumors (hence the gynecomastia from
hyperprolactinemia) and hyperparathyroidism REFERENCES
(explaining the elevated Ca). H pylori would not Liu JJ, Di Sena V, Ookubo R, Carr-Locke
explain the patient’s presentation. DL, Saltzman JR. Endoscopic treatment of
gastroesophageal reflux disease: effect of gender
REFERENCES on clinical outcome. Scand J Gastroenterol. 2006;
Gibson JB, Behrman SW, Fabian TC, Britt LG. 41:144-148.
Gastric outlet obstruction resulting from peptic Li q, Castell JA, Castell DO. Manometric
ulcer disease requiring surgical intervention is determination of esophageal length. Am J
infrequently associated with Helicobacter pylori Gastroenterol. 1994; 94: 1468-1471.
infection. J Am Coll Surg. 2000 Jul. 191(1):32-7.
Roy PK, Venzon DJ, Shojamanesh H, et
al. Zollinger- Ellison syndrome. Medicine
2000;79:379-411.

Question 51
A 52-year-old female comes to GI clinic reporting
a sour acid taste in her mouth and burning
sensation that occurs after eating. She has
noted some dysphagia to solids. but not liquids.
No history of food impaction. She undergoes
EGD and is found to have mild esophagitis. The
following is true regarding sex differences related
to GERD for men and women:

A. Women have higher rates of Barrett’s


esophagus
B. Women had lower rate of erosive esophagitis
than men
C. Women and Men have the same length
esophagus
D. The standard for pH testing in reflux are
different for men and women
E. Lower esophageal sphincter pressure (LESP)
is lower in women

CORRECT ANSWER: B
52 Digestive Diseases Self-Education Program®
Answers & critiques

CHAPTER 3

Pancreatic physiology & disease


Darwin L. Conwell, MD, Danielle Marino, MD
and Shivangi Kothari, MD

Question 1 to 40 percent by age 70. BRCA1 mutations have


A 25-year-old male presents to the emergency been associated with familial pancreas cancer
department with severe epigastric pain and mild families. SPINK mutations have been associated
elevations in lipase (3 x ULN) diagnostic of acute with chronic tropical pancreatitis. Delta F508
pancreatitis. The patient describes multiple is the most common mutation in cystic fibrosis
episodes of pain and associated pancreas enzyme that leads to pancreas insufficiency in childhood.
elevations since early childhood that generally The clinical scenario is classic for hereditary
respond to brief hospitalizations and conserva- pancreatitis.
tive treatment including intravenous fluids and
IV analgesics. CT imaging reveals parenchymal REFERENCE
calcifications seen throughout the pancreas. Fur- Shelton CA, Umapathy C, Stello K, Yadav D,
ther history discloses two relatives with similar Whitcomb DC. Hereditary Pancreatitis in the
pain attacks. United States: Survival and Rates of Pancre-
atic Cancer. Am J Gastroenterol. 2018 Sep;
Which of the following gene mutations is most 113(9):1376-1384.
likely to be associated with the cause of the re-
current pancreatitis in this patient?
Question 2
A. BRCA1 A 52-year-old male with a remote history of
B. PRSS1 recurrent pancreatitis presents with chronic
C. SPINK1 abdominal pain. There is a history of moderate
D. Delta F508 smoking and alcohol consumption. The patient
E. PRSS2 is suspected of having chronic pancreatitis. but
CT imaging is negative. A pancreas function test
CORRECT ANSWER: B (secretin stimulation test) is ordered.

RATIONALE Which of the following cells are stimulated by


The PRSS1 mutation has been shown to be the secretin to increase bicarbonate concentration in
causative genetic factor in hereditary pancre- the pancreas fluid?
atitis. Hereditary Pancreatitis is an autosomal
dominant gene mutation with 80 percent pen- A. Acinar cells
etrance. Symptoms start in childhood with acute B. Duct Cells
recurrent pancreatitis and progress to chronic C. Stellate cells
pancreatitis, diabetes and exocrine insufficiency. D. Paneth cells
The incidence of pancreatic cancer is increased E. Parietal cells

53
54 Digestive Diseases Self-Education Program®

CORRECT ANSWER: B the etiology is unclear and/or who are having com-
plications from acute pancreatitis.
RATIONALE
Secretin is a hormone secreted into the blood REFERENCE
stream by the S cells of the small intestine in Tenner S, Baillie J, DeWitt J, Vege SS. American
response to duodenal pH. Secretin binds to the College of Gastroentrology Guideline: Manage-
basolateral membrane of the duct cells, which ment of Acute Pancreatitis. Am J Gastroenterol.
increases intracellular cAMP and calcium resulting 2013; 108: 1400-15.
in a bicarbonate rich fluid secretion into the pan-
creas ducts. Measurement of peak bicarbonate
concentration (Normal: greater than 80 meq/L) Question 4
in pancreas fluid is used to assess for pancreas Cholecystokinin is a hormone secreted from the en-
function. terocytes of the small intestine in response to a meal.

REFERENCE Which one of the pancreas cells secrete zymogen


Pandol, Stephen J. Sleisinger and Fordtran’s Gas- proteins in pancreas fluid in response to CCK?
trointestinal and Liver Disease, 10th ed. Chapter
56, 934-943. A. Acinar cells
B. Duct cells
C. Paneth cells
Question 3 D. G cells
A 25-year-old college student has been diagnosed E. Stellate cells
with acute pancreatitis. His vital signs are stable
and he is not toxic. He is unable to tolerate oral CORRECT ANSWER: A
intake due to nausea and vomiting. He denies
alcohol use. There is a family history of gallstone RATIONALE
disease in his mother and sisters. Cholecystokinin is a hormone secreted by the Ito
cells of the small intestine into the blood stream
What is the best diagnostic test to elucidate the and bind to the basolateral membrane of the
etiology of pancreatitis in this patient? acinar cells. Through stimulus secretin coupling
events within the acinar cell, zymogen granules at
A. CT Abdomen the apices of the cells containing inactive proteins
B. Endoscopic Ultrasound are secreted into the pancreas ducts.
C. Transabdominal Ultrasound
D. MRI/MRCP REFERENCE
E. HIDA Scan Pandol, Stephen J. Sleisinger and Fordtran’s Gas-
trointestinal and Liver Disease, 10th ed. Chapter
CORRECT ANSWER: C 56, 934-943.

RATIONALE
In the search for etiology, the American College of Question 5
Gastroenterology guidelines recommend that trans- There are three major phases of pancreas
abdominal ultrasound be performed in all patients secretion: cephalic, gastric and intestinal.
with acute pancreatitis (strong recommendation, The intestinal phase of pancreas secretion
low quality of evidence). It is recommended that CT associated with a meal accounts for what degree
and MRI imaging be reserved for patients in whom of pancreas secretion?
Chapter 3 — Pancreatic physiology & disease 55

A. 10 percent RATIONALE
B. 20 percent In a RCT, Lactated Ringer’s has been shown to de-
C. 80 percent crease systemic inflammation in acute pancreatitis.
D. 100 percent Lactated ringers has been shown to decrease blood
E. 30 percent CRP levels when given as a resuscitation fluid in
acute pancreatitis. The mechanism involves altera-
CORRECT ANSWER: C tions in the GPR81-receptor. The American College
of Gastroenterology, as well as American Gastroen-
RATIONALE terological Association Guidelines, recommend goal
The cephalic phase (10 percent of total pancreas directed therapy with lactated ringers as first line
secretion) occurs due to the sight, smell or taste treatment for acute pancreatitis.
of food. The gastric phase (~20 to 25 percent of
total pancreas secretion) occurs as food enters REFERENCES
and distends the stomach. The intestinal phase, Wu, B et al., Lactated Ringer’s solution reduces
which accounts for 70 to 90 percent of total secre- systemic inflammation compared with saline in
tion results when food particles and acidic gastric patients with acute pancreatitis. Clin Gastroen-
contents are delivered into the small intestine. terol Hepatol. 2011 Aug;9(8):710-717.
This stimulates Secretin and CCK release from the Lerch MM, et al., The anti-inflammasome
small bowel enterocytes resulting in a bicarbonate effect of lactate and the lactate GPR81-receptor in
and protein rich pancreas fluid. pancreatic and liver inflammation. Gastroenterol-
ogy. 2014 Jun;146(7):1602-5.
REFERENCE
Pandol, Stephen J. Sleisinger and Fordtran’s Gas-
trointestinal and Liver Disease, 10th ed. Chapter Question 7
56, 934-943. A 25-year-old female with abdominal pain under-
goes a CT scan in the emergency room. An inciden-
tal 2.5 cm pancreas cyst is seen. There is no history
Question 6 of pancreatitis and no family history of pancreas
A 53-year-old male presents with epigastric pain cancer. EUS reveals a “central stellate scar” and
radiating to his back, nausea and vomiting for 12 multiple small cysts in a honeycomb appearance.
hours. His BP is 100/60 mm Hg, heart rate 110 Aspiration of the cyst is negative for malignancy
bpm. Labs reveal a WBC of 12,000 (normal 4,000- with low CEA level and high glycogen levels.
10,000), Hemoglobin 17 g/dL (normal 13-17 g/
dL), normal hepatic profile, lipase 3,000 U/L What is the next best step in management?
(normal 0-150 U/L).
A. Observation
Which of the following resuscitation fluids should B. Surgical resection
be initiated? C. Endoscopic ablation
D. ERCP with Spyglass
A. Lactated Ringers E. Repeat EUS in one year
B. Normal Saline
C. Albumin bolus CORRECT ANSWER: A
D. Five percent dextrose in half normal saline
E. Hydroxyethyl starch RATIONALE
This is an asymptomatic cyst with EUS features
CORRECT ANSWER: A of a “classic” serous cystadenoma. These are NOT
56 Digestive Diseases Self-Education Program®

premalignant and do not require surveillance. a systematic review and meta-analysis. Clin
They also have been termed glycogen rich Gastroenterol Hepatol. 2014 Feb;12(2):219-28.
adenomas. Observation is indicated and manage-
ment will be dictated by symptoms associated
with cyst enlargement. Question 9
Secretin release by duodenal S-cells is required to
REFERENCE stimulate pancreatic bicarbonate secretion. Which
Elta, GH et al., ACG Clinical Guideline: Diagnosis of the following is the most important regulatory
and Management of Pancreatic Cysts. The American mechanism of duodenal secretin release?
Journal of Gastroenterology 113, 464-479 (2018).
A. PH less than 4.5
B. Short chain fatty acids
Question 8 C. Amino acids
Which of the following is frequently found in D. Disaccharides
patients with chronic pancreatitis? E. Cholecystokinin

A. Hypochlorhydria CORRECT ANSWER: A


B. Osteoporosis
C. Vitamin E deficiency RATIONALE
D. Hypertension Secretin is secreted by S-cells located in the
E. Chronic constipation proximal duodenal mucosa. This is released when
duodenal pH falls below 4.5. Secretin activates the
CORRECT ANSWER: B cAMP-dependent chloride channel CFTR, which
leads to increased chloride output. The other
RATIONALE choices do not stimulate secretin release.
A recent systematic review documented osteopo-
rosis in one in four patients with chronic pancre- REFERENCE
atitis. Osteoporosis is likely multifactorial and Chey WY, Chang TM. Neural control of the release
involves decreased vitamin D levels, enhanced and action of secretin. J Physiol Pharmacol 2003;
bone reabsorption, and the concomitant effects 54 (4): 105-112.
of cigarette smoking. While steatorrhea due to
pancreatic exocrine insufficiency can lead to fat
soluble vitamin deficiencies (A, D, E, K), these are Question 10
not as frequently seen clinically. Hypochlorhydria, Which of the following is not a major category of
hypertension, and chronic constipation are not digestive enzymes seen in pancreas juice?
related to chronic pancreatitis. In fact, many pa-
tients with chronic pancreatitis will have chronic A. Proteases
diarrhea due to steatorrhea. B. Amylase
C. Pepsinogen
D. Lipase
REFERENCES E. Carboxypeptidase
Duggan SN, O’Sulllivan M et al. Patients with
chronic pancreatitis are at increased risk for osteo- CORRECT ANSWER: C
porosis. Pancreas. 2012 Oct;41(7):1119-24.
Duggan SN, Smyth et al. High prevalence of RATIONALE
osteoporosis in patients with chronic pancreatitis: The four major categories of digestive enzymes are
Chapter 3 — Pancreatic physiology & disease 57

proteases, amylase, lipases and nucleases. REFERENCE


Most of the proteins are digestive enzymes or Crockett, S. D., Wani, S., Gardner, T. B., Falck-
co-factors, which include 20 isozymes of 12 Ytter, Y., Barkun, A. N., & American Gastroentero-
different enzymes. logical Association Institute Clinical Guidelines
Committee. (2018). American gastroenterological
REFERENCE association institute guideline on initial manage-
Pandol, Stephen J. Sleisinger and Fordtran’s ment of acute pancreatitis. Gastroenterology,
Gastrointestinal and Liver Disease, 10th ed. 154(4), 1096-1101.
Chapter 56, 934-943.

Question 12
Question 11 Pancreatic acinar cell agonists stimulate digestive
A 21-year-old female underwent ERCP for enzyme secretion via several receptor-mediated
choledocholithiasis. Biliary cannulation was mechanisms.
difficult and a needle-knife sphincterotomy
was performed. Stones were removed and Which of the following intracellular events result
occlusion cholangiogram did not reveal any f from pancreatic acinar cell stimulation?
illing defects. The next morning, the patient
presents with severe epigastric abdominal A. GRP decreases cyclic AMP
pain, nausea and vomiting. Her lipase is B. CCK increases cyclic AMP
4000 IU/L (normal 0-60 IU/L). C. Secretin increases cyclic AMP
D. VIP increases calcium
What is the next best step in management? E. Acetylcholine decreases calcium

A. CT scan of abdomen CORRECT ANSWER: C


B. 300cc/hr of lactated ringers intravenously
C. Repeat ERCP with pancreatic duct stent RATIONALE
placement Pancreatic acinar cell agonists stimulate digestive
D. Ultrasound of the abdomen enzyme secretion through two separate path-
E. Intravenous steroids to decrease ampullary ways. In one pathway, agonists such as gastrin-
inflammation releasing peptide, cholecystokinin, and acetyl-
choline mediate secretion through increases in
CORRECT ANSWER: B intra-cellular calcium. In the other pathway,
agonists such as vasoactive intestinal polypeptide
RATIONALE and secretin mediate secretion through increases
This patient presents with post-ERCP in cyclic AMP.
pancreatitis. Regardless of the cause of acute
pancreatitis, the immediate treatment is aggres- REFERENCES
sive, goal-directed intravenous fluids with lactat- Pandol, Stephen J. Sleisinger and Fordtran’s Gas-
ed ringers. Lactated Ringers has been shown trointestinal and Liver Disease, 10th ed. Chapter
to decrease SIRS response and CRP levels in pa- 56, 934-943.
tients with acute pancreatitis. There is no indica- From Pancreatic Physiology and Pancreatitis
tion for abdominal imaging given the diagnosis GastroSlide 64, AGA Institute
is clear based on clinical criteria (history and
elevated lipase levels). Corticosteroids have not
been shown to decrease the risk of pancreatitis.
58 Digestive Diseases Self-Education Program®

Question 13 Question 14
A 54-year-old alcoholic male with a history of recur- A 64-year-old male with a recent history of acute
rent acute pancreatitis undergoes a CT scan for pancreatitis has a dilated main pancreatic duct
abdominal pain that reveals pancreatic parenchymal with prominent side branch lesions seen on CT
calcifications and a dilated main pancreatic duct of scan. Endoscopic evaluation reveals mucus extrud-
11 mm. There is no evidence of a pancreatic mass or ing from a dilated ampulla.
evidence of duodenal obstructive symptoms.
What is the next best step in management of this
What is the best long term management of pain in patient?
this patient with chronic pancreatitis?
A. EUS +/- FNA
A. Pancreaticoduodenectomy B. ERCP with pancreatic stent
(Whipple procedure) C. Surgical resection
B. Lateral pancreatico-jejunostomy D. Repeat MRI in three years
(Peustow) procedure E. ERCP with Spyglass
C. Total pancreatectomy with islet
autotransplantation (TPIAT) CORRECT ANSWER: C
D. Distal pancreatectomy
E. ERCP with pancreatic duct stent placement RATIONALE
This patient has a main duct IPMN, which has
CORRECT ANSWER: B a high potential for malignant transformation
and should be resected if possible. Resection is
RATIONALE also indicated for branch-duct IPMN’s which are
Surgical resection / drainage is the best long- symptomatic (e.g. pancreatitis), associated with
term option for symptomatic “large duct” chronic obstructive jaundice or main duct involvement,
pancreatitis. Patients with a diffusely dilated duct have a solid component within the cyst, or have
(as above), are the best candidates for a drain- concerning features on EUS-FNA.
age procedure, such as the lateral pancreatico-
jejunostomy. A randomized controlled trial REFERENCE
compared lateral pancreatico-jejunostomy to Elta, GH et al, ACG Clinical Guideline: Diagnosis
ERCP with pancreatic stenting (+/- lithotripsy), and Management of Pancreatic Cysts. The Ameri-
and found that surgical treatment led to improved can Journal of Gastroenterology 113, 464-479
pain control at 24 months follow-up. Patients (2018).
with focal ductal disease and pain can be managed
by surgical resection of the diseased ductal seg-
ment. (Whipple, distal pancreatectomy). Chronic Question 15
pancreatitis patients with “small duct” disease are A 23-year-old female has a childhood history of
often times considered for TPIAT. recurrent acute pancreatitis and family history
of chronic pancreatitis in several relatives. The
REFERENCES patient does not have any evidence of diabetes
Cahen, DL et al. Endoscoic versus surgical drain- and has chronic abdominal pain. Genetic test-
age of the pancreatic duct in chronic pancreatitis. ing reveals a PRSS1 gene mutation. Radiologic
N Engl J Med 2007; 356(7):676-84. imaging is consistent with small duct chronic
Trikudanathan G, et al. Modern treatment of pancreatitis.
patients with chronic pancreatitis. Gastroenterol
Clin North Am. 2012; Mar;41(1):63-76. What is the next best step in management?
Chapter 3 — Pancreatic physiology & disease 59

A. Pancreatico duodenectomy CORRECT ANSWER: D


(Whipple procedure)
B. Lateral pancreatico jejunostomy (Peustow) RATIONALE
procedure The patient has sequelae of necrotizing pancre-
C. Total pancreatectomy with islet atitis which has resulted in a pancreatic duct leak
autotransplantation (TPIAT) fistulizing into the pleural space, causing a pleural
D. Distal pancreatectomy effusion (pancreaticopleural fistula). The high
E. ERCP with pancreatic duct stent amylase confirms the pancreatic duct disruption.
The next best step in management is an attempt at
CORRECT ANSWER: C ERCP with endoscopic placement of a pancreatic
stent to attempt to bridge the pancreas duct dis-
RATIONALE ruption. If endoscopic therapy is not successful,
Total pancreatectomy with autologous islet trans- surgical options could be considered.
plantation is emerging as the best treatment strategy
for children with recurrent or chronic pancreatitis REFERENCES
from PRSS1 Gene mutations. TPIAT preserves beta Telford JJ, et al. Pancreatic Stent Placement for
cell mass and mitigates diabetic complications. In Duct Disruption. Gastrointest Endosc 2002 Jul;
addition, patients have reported up to a 92 percent 56(1): 18-24.
relief of pancreatic pain at one year post TPIAT. Ali T, et al. Pancreaticopleural fistula. Pan-
creas Jan; 38 (1):26-31.
REFERENCES
Bellin MD, Gelrud A, et al. Total Pancreatectomy
With Islet Auto-Transplantation (TPIAT): Sum- Question 17
mary of an NIDDK Workshop. Pancreas. 2014 A 53-year-old female with remote history of nec-
November; 43(8); 1163-1171. rotizing pancreatitis presents with massive UGI
Bellin MD, Forlenza G, et al. Total pancreatec- bleeding requiring intravenous fluids and blood
tomy with islet autotransplantation resolves pain transfusions. After stabilization in the emergency
in young children with severe chronic pancreatitis. department and transfer to the ICU, EGD reveals
Pancreatology, 2018 Apr; 18(3): 286-290. large gastric varices with a red wale sign, without
esophageal varices.

Question 16 Which of the following is the definitive manage-


A 45-year-old male with a history of necrotizing ment for this patient?
acute pancreatitis eight months earlier presents
to the emergency department with shortness of A. Surgical consultation for splenectomy
breath. Chest x-ray reveals a large left pleural B. Banding of gastric varices
effusion. Pleural fluid reveals a very high amylase C. Distal pancreatectomy
greater than 1,000 U/L. D. Interventional radiology consultation for
visceral angiogram and embolization
What is the next best step in management? E. CT Angiogram

A. Surgical resection of pancreas CORRECT ANSWER: A


B. EUS Rendezvous procedure
C. Pleural drainage with catheter RATIONALE
D. ERCP with stent placement The patient has a history of necrotizing pancreati-
E. Bronchoscopy with bronchoalveolar lavage tis that has resulted in obstruction of the splenic
60 Digestive Diseases Self-Education Program®

vein with resultant isolated bleeding gastric Question 19


varices. Splenectomy is the definitive treatment of A 57-year-old female presents with intermittent
choice. Gastric varices may be treated with injec- abdominal pain, heart burn and nausea. The
tion of cyanoacrylate glue, but rebleeding is com- patient also reports diarrhea, up to three to four
mon and this is not widely available in the United loose bowel movements per day. An upper en-
States. Banding can be done for gastric varices doscopy reveals esophagitis, antral erythema with
close to the GEJ (GOV-1), but generally is not erosions and multiple small ulcerations in the
effective for other gastric varices. Visceral angio- duodenum. Small bowel biopsies show inflam-
gram with embolization is not a treatment option mation with mild villous atrophy. Celiac sprue
for gastric varices. Distal pancreatectomy would serology and gastric biopsies are unremarkable.
not treat splenic vein thrombosis. The above clinical symptoms are most consis-
tent with which of the following neuroendocrine
REFERENCE tumors?
Butler JR et al, Natural history of pancreatitis-
induced splenic vein thrombosis: a systematic A. Gastrinoma
review and meta-analysis of its incidence and B. Glucagonoma
rate of gastrointestinal bleeding HBP (Oxford) C. Insulinoma
2011;13:839-45. D. VIPoma
E. Somatostatinoma

Question 18 CORRECT ANSWER: A


Which of the following is the most common func-
tioning pancreatic endocrine tumor? RATIONALE
Gastrinoma (Zollinger-Ellison syndrome), produc-
A. Gastrinoma es massive hypersection of gastric acid resulting
B. Glucagonoma in severe peptic ulcer disease and chronic diar-
C. Insulinoma rhea. The majority (approximately 80 percent) of
D. VIPoma these tumors can be localized in the “gastrinoma
E. Somatostatinoma triangle” in the region of the duodenum. Secretin
testing of patients with elevated gastrin levels can
CORRECT ANSWER: C assist in the differentiation between gastrinoma
and atrophic gastritis.
RATIONALE
Insulinoma is the most common pancreatic endo- REFERENCE
crine tumor (one to two per million people/year), Jensen, Robert T et al, Sleisinger and Fordtran’s
producing symptoms from low glucose levels. Gastrointestinal and Liver Disease, 10th ed.
Most are benign, solitary lesions easily found on Chapter 33, 501-541.
EUS.

REFERENCES Question 20
Jensen, Robert T et al, Sleisinger and Fordtran’s A 62-year-old female presents to the emergency
Gastrointestinal and Liver Disease, 10th ed. Chap- department for left lower quadrant pain and
ter 33, 501-541. fever. A CT scan reveals sigmoid diverticulitis,
Metz DC, Jensent RT. Gastrointestinal neuro- and a cyst in the body of the pancreas. The cyst
endocrine tumors: pancreatic endocrine tumors. measures 2.5cm and the pancreatic duct is dilated
Gastroenterology 2008;135(5):1469–92. ` to 7mm in the body and tail.
Chapter 3 — Pancreatic physiology & disease 61

What do you recommend for this patient? Question 21


A 59-year-old male with abdominal pain is found
A. MRI now and in one year to have acute pancreatitis in the emergency depart-
B. MRI in six months ment. He is febrile with a temperature of 40 C.
C. MRI in one year Laboratory studies reveal WBC 16,000 (normal
D. Surgical evaluation 4,000-10,000), Hemoglobin 18 g/dL (normal 13-17
E. EUS-FNA g/dL), lipase 2,000 U/L (normal 0-150 U/L), AST
45 U/L (normal 10-35), ALT 88 U/L (normal 10-
CORRECT ANSWER: E 35), total bilirubin 2.1 mg/dL (normal 0-1.2mg/dL),
ALK 198 IU/L (44-150 IU/L). He is hypotensive
RATIONALE initially, but responds to aggressive fluid hydra-
Pancreatic cystic lesions less than 3cm can be fol- tion with lactated ringers. Intravenous antibiotics
lowed with MRI in one year, then every two years are started. Abdominal ultrasound in ED revealed
for five years. Lesions that are greater than 3cm or dilated common bile duct to 12 mm and multiple
have concerning features such as mural nodule, small stones in the gallbladder. He is stabilized and
solid component, dilated pancreatic duct, should admitted to the hospital ward for further manage-
undergo EUS-FNA to evaluate for a mucinous cystic ment. What is the next best step in management?
neoplasm. Mucinous cystadenomas almost always
occur in females and are usually solitary, located in A. Cholecystectomy
the body or tail of the pancreas, and do not commu- B. ERCP
nicate with the pancreatic duct. These lesions carry C. MRCP
a moderate risk of malignant transformation. D. Percutaneous cholecystostomy tube placement
E. Repeat Abdominal Ultrasound
REFERENCES
Grace H. Elta, Brintha K. Enestvedt, Bryan CORRECT ANSWER: B
G. Sauer and Anne Marie Lennon. ACG Clini-
cal Guideline: Diagnosis and Management of RATIONALE
Pancreatic Cysts. Am J Gastroenterol. 2018 The patient requires aggressive IVFs and antibiot-
Apr;113(4):464-479. ics. In addition, there is evidence of ascending
Brugge William. Diagnosis and management cholangitis (jaundice, RUQ pain and fever) and
of cystic lesions of the pancreas J Gastrointest he requires ERCP to decompress the bile duct.
Oncol. 2015 Aug; 6(4): 375–388. According to ACG Guidelines for acute pancre-
atitis, patients presenting with acute pancreatitis
CORRECT ANSWER: D and concurrent cholangitis should undergo ERCP
within the first 24 hours of admission. There is
RATIONALE no need for MRCP given the US findings strongly
The patient’s clinical scenario is strongly sugges- suggesting bile duct stones. Surgical consultation
tive of autoimmune pancreatitis (AIP). Initiation for consideration of cholecystectomy during the
of corticosteroid treatment and follow-up imaging hospital admission is indicated, but the more im-
and monitoring of response to therapy is indicated. mediate need is further treatment with ERCP and
Standard treatment is 40-60 mg of Prednisone per biliary decompression of bile duct stones.
day for four weeks followed by tapering of dose.
REFERENCE
REFERENCE Tenner S, et al., American College of Gastroenter-
Hart PA, Recent advances in autoimmune ology Guideline: Management of Acute Pancreati-
pancreatitis. Gastroenterology, 2015; 149:39-51. tis. Am J Gastroenterol 2013.
62 Digestive Diseases Self-Education Program®

Question 22 obtained three days before his visit reveals a six


A 33-year-old male with acute abdominal pain is cm pancreatic fluid collection with a mature wall.
diagnosed with acute pancreatitis. An abdomi-
nal CT scan during his index admission reveals How would you characterize this fluid collection?
a six cm fluid collection without evidence of ne-
crosis. The patient responds well to conservative A. Acute Fluid Collection
management during his hospitalization. B. Acute Necrotic Collection
C. Walled off pancreatic necrosis
How would you characterize this fluid collection? D. Pancreas pseudocyst
E. Cystic Neoplasm
A. Acute Fluid Collection
B. Acute Necrotic Collection CORRECT ANSWER: C
C. Walled off pancreas necrosis
D. Pancreatic pseudocyst RATIONALE
E. Cystic Neoplasm The patient has a remote history of acute necro-
tizing pancreatitis and a large fluid collection. CT
CORRECT ANSWER: A scan four months after initial attack of necrotiz-
ing pancreatitis reveals a six cm collection with a
RATIONALE mature wall. The classification of fluid collections
The patient has an acute bout of interstitial pan- in pancreatitis is based on the duration of the
creatitis with a fluid collection. The classification pancreatitis (greater than or less than four weeks)
of acute fluid collections in pancreatitis are based and presence or absence of necrosis. This patient
on the duration of the pancreatitis (greater than has a fluid collection greater than four weeks in
or less than four weeks) and presence or absence the setting of necrosis. According to the Revised
of necrosis. This patient has a fluid collection Atlanta Criteria, this is classified as Walled Off
less than four weeks without any evidence of ne- Pancreatic Necrosis. Symptomatic (abdominal
crosis. According to the Revised Atlanta Criteria, pain, fever, jaundice, bowel obstruction or gastric
this is classified as an Acute Fluid Collection. outlet obstruction) collections require drainage
with a minimally invasive approach (surgical,
REFERENCE endoscopic) based on local expertise.
Banks PA, et al., Classification of Acute pancre-
atitis – 2012: revision of the Atlanta classifica- REFERENCE
tion and definitions by international consensus. Banks PA, et al., Classification of Acute pancre-
Gut 2013; 62:102-111. atitis – 2012: revision of the Atlanta classifica-
tion and definitions by international consensus.
Gut 2013; 62:102-111.
Question 23
A 47-year-old male with history of severe necro-
tizing gallstone pancreatitis four months earlier Question 24
is seen in the office for follow-up. Review of his A 70-year-old male is referred for evaluation of
records reveal severe pancreatitis with compli- a pancreatic body mass seen on CT scan. The
cations of renal failure and ICU admission for patient has been seen in dermatology for evalu-
ventilatory support for five weeks. He was in a ation of multiple subcutaneous masses in the
skilled nursing facility for two months. Current- lower extremities. Skin biopsy reveals pannicu-
ly, he is doing well and tolerating a low-fat diet litis. Upon further questioning, the patient also
on pancreatic enzyme replacement. A CT scan describes stiffness and pain in multiple joints.
Chapter 3 — Pancreatic physiology & disease 63

The clinical features are most characteristic of RATIONALE


which of the following pancreas masses? In a patient with chronic pancreatitis and a
pancreatic mass, the most likely etiology is
A. Pancreatic lymphoma adenocarcinoma. This patient has radiologically
B. Solid Pseudopapillary Epithelial Neoplasm resectable pancreas cancer. There is no evidence
(SPEN) of lymphadenopathy or vascular invasion. Per-
C. Acinar Cell Carcinoma forming an ERCP with stent placement to relieve
D. Main-duct IPMN biliary obstruction has not been shown to be of
E. Cystadenocarcinoma benefit in patients with a resectable pancreatic
mass. In fact, surgical outcomes are worse if a
CORRECT ANSWER: C stent is placed in the bile duct. Surgical consul-
tation should be obtained and the patient should
RATIONALE undergo pancreaticoduodenectomy. EUS is
The patient has findings characteristic of a sometimes done, but most cases of resectable
rare pancreatic tumor known as an acinar cell disease should go straight to surgery.
carcinoma. Acinar cell carcinomas make up
less than one percent of all pancreas cancers. REFERENCE
These tumors can occur in the elderly and can Ghaneh P, et al., Biology and Management of
be associated with polyarthritis, panniculitis and Pancreatic Cancer. GUT 2007; 56(8)1134-52.
subcutaneous nodules.

REFERENCE Question 26
Glazer ES, et al., Sytematic Review and Case A 31-year-old female with history of alcohol
Series Report of Acinar Cell Carcinoma of the abuse presented at an outside hospital two
Pancreas. Cancer control 2016; 23 (4): 446-454. months ago with acute pancreatitis. Her hospital
course was complicated by necrotizing pancre-
atitis and questionable pancreatic duct leak. She
Question 25 now presents to see you for persistent abdominal
A 56-year-old male with known chronic pancre- pain, early satiety and nausea, vomiting. Labs
atitis presents with progressive abdominal pain, reveal WBC 9,000 cells/mm3, AST 52 U/L, ALT
weight loss and obstructive jaundice and a bili- 30 U/L, alk phos 90 U/L, TB 1.5 mg/dl
rubin of eight. A CT scan with contrast reveals
a four cm mass in the pancreas head. There is You get a repeat CT with contrast as below.
no lymphadenopathy and vascular architecture
is maintained. What is the next best step in
management?

A. Pancreaticoduodenectomy
(Whipple procedure)
B. Lateral pancreaticojejunostomy procedure
(Peustow) procedure
C. EUS +/- FNA
D. MRI/MRCP
E. ERCP with bile duct brushing and stent

CORRECT ANSWER: A
64 Digestive Diseases Self-Education Program®

What is collection noted on the CT? CT:

A. Pancreatic ascites
B. Pancreatic pseudocyst
C. Walled off pancreatic necrosis
D. Acute pancreatic fluid collection
E. Acute pancreatic necrosis

CORRECT ANSWER: C

RATIONALE
The revised Atlanta classification delineates the
pancreatic cyst collections into a) acute peripan-
creatic fluid collection seen in the first four weeks,
non-encapsulated peripancreatic fluid collections MRI:
b) Pseudocysts develop after four weeks and well
encapsulated c) acute necrotic collection occurs
less than four weeks from the acute pancreatitis d)
walled off necrosis is after four weeks, encapsulated
necrosis as in this patient.

REFERENCE
Banks PA, Bollen TL, Dervenis C, et al. Classifica-
tion of acute pancreatitis - 2012: revisions of the
Atlanta classification and definitions by interna-
tional consensus. Gut 2013; 62:102.

Question 27 A. Pancreatic pseudocyst


24-year-old female undergoes CT for abdominal B. Serous cystadenoma
pain and nausea. CT, as below, revealed a 3.5 cm C. Mucinous cystadenoma
pancreatic tail cystic lesion with peripheral calcifi- D. Side branch IPMN
cation, though further evaluation is recommended E. Pancreatic pseudopapillary tumor
with MRI. She denies any history of pancreatitis or
family history of pancreatic cancer. She denies any CORRECT ANSWER: E
weight loss. MRI as below revealed the 3.5 cms
hyperintense lobulated lesion with peripheral cal- RATIONALE
cifications and possibly debris or solid component She has a pancreatic pseudopapillary tumor. This
within. Labs reveal a normal CBC and comprehen- is commonly seen in young female in their 20s or
sive metabolic panel. Amylase and lipase are nor- 30s. They can occur anywhere in the pancreas. On
mal. CA 19-9 is 6 U/mL (normal 0-35 U/mL). EUS CT, a well encapsulated mass is seen with periph-
is performed for further evaluation and it reveals eral calcifications in up to 30 percent of patients.
a solid hypoechoic mass with some calcifications MRI reveals a well-defined lesion with high and
without any obvious cystic areas. low signal density on T1 and T2 weighted images,
as seen in this patient. On EUS the lesions can be
What is the most likely diagnosis? solid, cystic or mixed and fluid CEA is low. EUS
Chapter 3 — Pancreatic physiology & disease 65

FNA can have a 75 to 80 percent accuracy in diag- D. Pancreatic pseudocyst


nosing pancreatic pseudopapillary tumor. Surgical E. Pancreatic adenocarcinoma
resection is the treatment of choice.
CORRECT ANSWER: A
REFERENCES
Lanke G, Ali FS, Lee JH. Clinical update on the RATIONALE
management of pseudopapillary tumor of pan- This patient most likely has a serous cystadenoma.
creas. World J Gastrointest Endosc. 2018 Sep They are common in females and usually seen in
16;10(9):145-155. their late 50s or 60s. CT and MRI can have a clas-
Brugge W. Diagnosis and management of cys- sic microcystic appearance with a central fibrous
tic lesions of the pancreas. J Gastrointest Oncol. scar that can be seen in 30 percent of serous cysts
2015 Aug; 6(4): 375–388. and is considered pathognomonic. These are
very vascular cysts and EUS FNA sample can be
bloody, thus yield of cytology is low. Cyst fluid
Question 28 CEA concentration is usually low. Despite their
A 65-year-old female with no significant past benign nature, they can grow in size and cause ab-
medical history presents to the emergency depart- dominal pain, jaundice or weight loss. Therefore,
ment after a fall. CT abdomen reveals a pancre- surgery can be considered for the symptomatic
atic neck three cm grape cluster-like cystic mass serous cysts.
with a central scar. She is asymptomatic and has
no abdominal pain, jaundice, weight loss or family REFERENCES
history of pancreatic cancer. MRI is performed, Brugge William. Diagnosis and management of
that reveals a microcystic mass as below without cystic lesions of the pancreas. J Gastrointest On-
any solid component. CBD and PD are normal in col. 2015 Aug; 6(4): 375–388.
caliber. EUS is performed and fluid CEA is less Brugge WR, Lauwers GY, SAhani D et al.
than 5 ng/mL. Cystic neoplasms of the pancreas. N Engl J Med
2004;351:1218-26.

Question 29
A 60-year-old healthy male undergoes a non-
contrast CT for further evaluation of flank pain.
CT confirms nephrolithiasis, but also reveals
multiple possible cysts in the pancreas and MRI
is recommended for further evaluation. MRI as
below reveals multiple subcentimeter cysts in the
pancreatic body, with a 3.5 cm cyst in the head of
the pancreas, with a 1.5 cm solid mural nodule in
the cyst. The PD is prominent in the head of the
pancreas and measured four mm in diameter.
(See figure on the following page.)
What is the most likely diagnosis?

A. Serous cystadenoma
B. Mucinous cystic neoplasm
C. Intraductal papillary mucinous neoplasm
66 Digestive Diseases Self-Education Program®

Brugge William. Diagnosis and management


of cystic lesions of the pancreas J Gastrointest
Oncol. 2015 Aug; 6(4): 375–388.

Question 30
A 60-year-old female undergoes a CT for further
evaluation of hematuria. She is incidentally found
to have a 15 mm cyst in the tail of the pancreas.

What is the next best step in the evaluation


of this patient?

A. EUS FNA
B. ERCP
C. Surveillance MRI in one year
D. Surgery
E. Surveillance MRI in three months

CORRECT ANSWER: A
She denies any history of pancreatitis or family
RATIONALE history of pancreas cancer. She is a non-smoker
Size greater than three cm, mural nodule and dilated and is otherwise healthy without any comorbidi-
pancreatic duct are all high risk characteristics for ties. She denies any weight loss and the CBD and
mucinous cyst. Thus, this patient should be referred PD on the CT scan are normal. The cyst does not
to a multidisciplinary group and undergo EUS-FNA have a solid component on the scan. Which is the
for further characterization of the cyst, with analysis most appropriate next step?
of the cyst fluid cytology and chemistry (fluid CEA
and amylase levels). Cyst internal septations, pres- A. EUS FNA now
ence of debris, papillary projections and mural nod- B. ERCP now
ules can be visualized effectively with EUS. Presence C. MRI in one year
of vascular invasion, pancreatic duct communication D. CT in one year
and lymph node metastases can be evaluated. EUS E. Surgery referral
also allows for FNA of the high risk cystic lesions
for biochemical, cytological and DNA analysis that CORRECT ANSWER: C
might further aid diagnosis, characterization and
differentiation of the pancreatic cysts. RATIONALE
Per the 2018 ACG guidelines, incidentally found
REFERENCES one to two cm cysts without a solid component,
Grace H. Elta, Brintha K. Enestvedt, Bryan G. Sauer non-dilated PD, less than three cm can be sur-
and Anne Marie Lennon. ACG Clinical Guideline: veyed with MRI in one year. If no change, then
Diagnosis and Management of Pancreatic Cysts. MRI every one years for a total of three years is
Am J Gastroenterol. 2018 Apr;113(4):464-479. recommended. If cyst remains stable, then MRI
Chapter 3 — Pancreatic physiology & disease 67

every two years to four years is recommended. If D. ERCP now


cyst still stable, then lengthening of surveillance E. Refer to pain management
internal can be considered. MRI is preferred over
CT to reduce radiation, and MRI is more sensi- CORRECT ANSWER: B
tive to evaluate the PD, and also delineate the cyst
communication with the PD. RATIONALE
He has locally advanced pancreatic cancer and
REFERENCES thus is not currently a candidate for surgical
Grace H. Elta, Brintha K. Enestvedt, Bryan G. Sauer resection. According to the 2018 NCCN guide-
and Anne Marie Lennon. ACG Clinical Guideline: lines he should receive chemotherapy. Induc-
Diagnosis and Management of Pancreatic Cysts. tion chemotherapy can be given for four to six
Am J Gastroenterol. 2018 Apr;113(4):464-479. months, followed by chemoradiation or SBRT if
he does not develop systemic metastases. If his
performance status is good and he does have
Question 31 disease response to the chemotherapy, then cura-
A 75-year-old male with no significant past medical tive resection can be considered. His Bilirubin is
history presents with dyspepsia and 30 lbs uninten- normal, thus he currently does not require ERCP.
tional weight loss. Labs reveal a normal CBC and He currently does not have any significant cancer
comprehensive metabolic panel (including hepatic related pain and thus does not need evaluation by
function panel). EGD shows mild chronic inactive pain management.
gastritis without any ulcers or concerning lesions.
CT abdomen as below reveals a three cm pancreatic REFERENCE
body mass that is encasing the celiac artery bifurca- Tempero MA, et al. Pancreatic adenocarcinoma
tion. No metastasis is noted on CT. CA 19-9 is 500 NCCN Guidelines. version 2.2018.
U/mL (normal 0-35 U/mL) . Endoscopic ultra-
sound is performed and FNA of the mass reveals
pancreatic adenocarcinoma. Question 32
A 70-year-old male presents with painless jaun-
dice with AST 350 U/L, ALT 400 U/L, total biliru-
bin 14.5 mg/dL, direct bilirubin 12 mg/dL, alkaline
phosphatase 600 U/L. He also reports 15 lb weight
loss over the past six weeks. He has a 30 pack
year smoking history. He undergoes a CT scan as
below that shows a dilated CBD and intrahepatic
bile ducts, with a distal CBD stricture and a dilated
pancreatic duct in the body and tail with tapering
at the head of the pancreas. CT does not show an
obvious pancreatic mass. ERCP is performed with
plastic CBD stent placement and distal CBD stric-
ture brushings do not reveal any malignancy.
What is the next best step in the management of (See figures on the following page.)
this patient?
What is the next best step in the management of
A. Surgery now this patient?
B. Neoadjuvant chemotherapy
C. Ablation of tumor with RFA
68 Digestive Diseases Self-Education Program®

sectional imaging, small tumors (less than three


cm) can still be missed on CT and MRI. Endoscop-
ic Ultrasound (EUS) is a sensitive imaging modal-
ity that allows for high resolution imaging of the
pancreas, as well as fine-needle aspiration (FNA),
to assist in the diagnosis of pancreatic mass le-
sions. EUS-FNA has a high yield (85 percent
sensitivity) for diagnosis of pancreatic malignancy
in patients with suspected cancer with a non-diag-
nostic CT. In this patient, despite negative CT, the
clinical suspicion for malignancy is high. EUS can
help evaluate for any small mass lesions. Before
referring patient to surgery or oncology, other
benign etiologies for the stricture should be ruled
out, such as inflammatory stricture from stone
passage, autoimmune pancreatitis or cholangi-
opathy, primary sclerosing cholangitis or chronic
pancreatitis.

REFERENCE
Krishna SG, Rao BB, Ugbarugba E, et al. Diag-
nostic performance of endoscopic ultrasound for
detection of pancreatic malignancy following an
indeterminate multidetector CT scan: a systemic
review and meta-analysis. Surg Endosc. 2017
Nov;31(11):4558-4567.

Question 33
60-year-old male with known history of alcohol in-
A. Refer to surgery for duced chronic pancreatitis with chronic abdominal
pancreaticoduodenectomy pain well controlled on pancreatic enzyme supple-
(Whipple’s surgery) ment presents for follow up visit. He has been
B. Check CA 19-9 abstinent from drinking for past 10 years. He was
C. EUS FNA well until three months ago, when he started to
D. MRI lose weight and now reports a 20 lb unintentional
E. Refer to oncology weight loss. He reports a 20 pack-year smoking
history. He denies any abdominal pain, but reports
CORRECT ANSWER: C polyuria. His labs reveal normal CBC and basic
chemistry, except a fasting blood sugar of 180 mg/
RATIONALE dL. He has a past medical history of HTN.
Pancreatic cancer is the fourth leading cause of
cancer-related death in the United States, with a What is the next best step?
dismal five-year relative survival rate of approxi-
mately eight percent. This is mostly due to delay A. Increase pancreatic enzymes dose
in diagnosis. Despite improvement in the cross- B. Add PPI to enzymes
Chapter 3 — Pancreatic physiology & disease 69

C. Check CA 19-9 Question 34


D. Pancreas protocol CT A 45-year-old female is admitted with severe epi-
E. Refer to surgery gastric pain radiating to the back, associated with
nausea and vomiting. On admission her pulse is 90
CORRECT ANSWER: D beats per minute and BP 110/70 mm Hg. On physi-
cal exam she has significant epigastric tenderness
RATIONALE with mild guarding, but no rebound tenderness.
Patients with chronic pancreatitis are at an Labs reveal lipase greater than 10,000 U/L, AST
increased risk of developing pancreatic cancer. 350 U/L, ALT 450 U/L, TB 6.5 mg/dL, alkaline
Five years after diagnosis, chronic pancreatitis phosphatase 400 U/L, WBC 20,000 cells/mm3.
patients have a nearly eight-fold increased risk Ultrasound showed multiple gallstones in the gall-
of pancreatic cancer. Thus, a patient with known bladder, with dilated common bile duct measuring
chronic pancreatitis that develops alarm symp- 1.5 cm in diameter. She is started on aggressive IV
toms of unintentional weight loss, change in fluids with lactated ringers and broad spectrum an-
frequency or intensity of pain, new onset diabe- tibiotics. Three hours after admission she develops
tes or hyperglycemia, jaundice should undergo fever with temperature of 101.5 degrees F with HR
further evaluation for malignancy. CT as an early of 100/min and BP 90/60 mm Hg.
initial imaging test in chronic pancreatitis can
help evaluate for a mass lesion, check for any What is the next best step?
other chronic pancreatitis complications such as
pseudocysts, portosplenic venous thrombosis, A. CT abdomen with IV contrast
collaterals and arterial pseudoaneurysms etc, and B. MRCP
also evaluate for any other etiology of abdominal C. Urgent ERCP
pain and weight loss. CA 19-9 is not specific and D. Surgery consult for interval
is also dependent on tumor size. With the alarm- cholecystectomy
ing symptoms in this patient, increasing pan- E. Recheck labs in six hours
creatic enzymes or adding PPI would only delay
evaluation of a more serious process. CORRECT ANSWER: C

REFERENCES RATIONALE
Kirkegård J, Mortensen FV, Cronin-Fenton D. Patients with gallstone pancreatitis with clini-
Chronic Pancreatitis and Pancreatic Cancer Risk: cal deterioration and clinical picture suggestive
A Systematic Review and Meta-analysis. Am J of ascending cholangitis should undergo urgent
Gastroenterol. 2017 Sep;112(9):1366-1372. ERCP. Patients with mild biliary pancreatitis with
Raimondi S1, Lowenfels AB, Morselli-Labate no signs of biliary obstruction should undergo
AM, Maisonneuve P, Pezzilli R. Pancreatic cancer interval cholecystectomy as soon as possible
in chronic pancreatitis; aetiology, incidence, and once patient improves, ideally during the same
early detection. Best Pract Res Clin Gastroenterol. hospitalization. ERCP in patients with biliary
2010 Jun;24(3):349-58. pancreatitis without evidence of biliary obstruc-
Conwell DL1, Lee LS, Yadav D, Longnecker tion or cholangitis is not necessary. This patient
DS, Miller FH, Mortele KJ, Levy MJ, Kwon R, Lieb has developed ascending cholangitis and thus will
JG, Stevens T, Toskes PP, Gardner TB, Gelrud A, benefit from an urgent ERCP.
Wu BU, Forsmark CE, Vege SS. American Pan-
creatic Association Practice Guidelines in Chronic REFERENCES
Pancreatitis: evidence-based report on diagnostic Folsch UR, Nitsche R, Ludtke R, Hilgers RA,
guidelines. Pancreas. 2014 Nov;43(8):1143-62. Creutzfeldt W. Early ERCP and papillotomy com-
70 Digestive Diseases Self-Education Program®

pared with conservative treatment for acute biliary A. Endoscopic cystgastrostomy


pancreatitis. The German Study Group on Acute B. Surgical cystgastrostomy
Biliary Pancreatitis. N Engl J Med. 1997;336: C. Percutaneous drainage by IR
237–242. D. Expectant management with repeat
Fan ST, Lai EC, Mok FP, Lo CM, Zheng SS, imaging in six weeks
Wong J. Early treatment of acute biliary pancre- E. Start broad-spectrum antibiotics
atitis by endoscopic papillotomy. N Engl J Med.
1993;328: 228–232. CORRECT ANSWER: D
Lyu YX, Cheng YX, Jin HF, Jin X, Cheng B, Lu
D. Same-admission versus delayed cholecystecto- RATIONALE
my for mild acute biliary pancreatitis: a systematic Since this patient is asymptomatic and has no
review and meta-analysis. BMC Surg. 2018 Nov signs of infection, it is reasonable to follow the
29;18(1):111. patient clinically and with radiological surveil-
lance of the cyst for improvement/resolution.
Pseudocyst drainage is recommended for pa-
Question 35 tients that are symptomatic, have rapidly enlarg-
A 45-year-old female was recently admitted with ing pseudocyst or infected pseudocysts. If the
gallstone pancreatitis and was hospitalized for one pseudocyst is amenable to endoscopic drainage,
week. She is now five weeks out from her admis- then endoscopic drainage is preferred over surgi-
sion and is tolerating a low fat diet well. Today at cal drainage.
her follow up appointment she denies any abdomi-
nal pain, nausea, vomiting. CT performed two days REFERENCES
ago revealed a nine cm pancreatic fluid collection Cui ML, Kim KH, Kim HG, Han J, Kim H, Cho KB,
suggestive of a pseudocyst, as seen below. Her Jung MK, Cho CM, Kim TN. Incidence, risk factors
WBC is 6,000 cells/mm3 and hepatic function and clinical course of pancreatic fluid collections
panel is within normal limits. Her amylase and li- in acute pancreatitis. Dig Dis Sci. 2014;59(5):1055.
pase are within normal limits. She denies any early Muthusamy VR, Chandrasekhara V, Acosta
satiety or fever. RD, et al. ASGE Standards of Practice Committee,
The role of endoscopy in the diagnosis and treat-
ment of inflammatory pancreatic fluid collections.
Gastrointest Endosc. 2016;83(3):481.

Question 36
A 29-year-old male presents with severe epigastric
pain, nausea and vomiting for the past eight hours.
His heart rate is 110 beats/min, BP 105/60 mm
Hg. Labs reveal lipase 4,000 U/L, AST 80 U/L,
ALT 1000 U/L, TB 0.6 mg/dL, alkaline phospha-
tase 90 U/L, WBC 12,000 cells/mm3, Hemoglobin
16.6 mg/dL, Hematocrit 50 percent, BUN 55 mg/
dL and creatinine 1.2 mg/dL. Ultrasound showed
multiple gallstones in the gallbladder, with a non-
dilated common bile duct.
What is the next best step in the management of
the patient’s pseudocyst? What is the best next step?
Chapter 3 — Pancreatic physiology & disease 71

A. Lactated Ringers, 20mL/kg bolus followed by REFERENCES


3mL/kg/hr Buxbaum JL, Quezada M, Da B, et al. Early ag-
B. Normal Saline, 40mL/kg bolus followed by gressive hydration hastens clinical improvement
5mL/kg/hr in mild acute pancreatitis. American Journal of
C. CT abdomen and pelvis with IV contrast Gastroenterology. 2017;112(5):797-803.
D. Broad spectrum IV Antibiotics Crockett, S. D., Wani, S., Gardner, T. B., Falck-
E. Urgent ERCP Ytter, Y., Barkun, A. N., & American Gastroentero-
logical Association Institute Clinical Guidelines
CORRECT ANSWER: A Committee. (2018). American gastroenterological
association institute guideline on initial manage-
RATIONALE ment of acute pancreatitis. Gastroenterology,
The patient presents with acute pancreatitis, likely 2018; 154(4), 1096-1101.
gallstone related. The critical first step in the man- Wu, B. U., Hwang, J. Q., Gardner, T. H.,
agement of acute pancreatitis is to initiate aggres- Repas, K., Delee, R., Yu, S., et al. (2011). Lactated
sive IV fluid hydration with lactated ringers. In a ringer’s solution reduces systemic inflammation
prospective RCT by Buxbaum et al., aggressive, compared with saline in patients with acute pan-
goal directed hydration with LR increased both the creatitis. Clinical Gastroenterology and Hepatol-
proportion of patients achieving clinical improve- ogy; 9(8), 710-717.e1.
ment at 36 hours and increased the rate of clinical
improvement. This study provides a model for
goal-directed fluid therapy for the first 24hrs from Question 37
presentation: LR 20mL/kg bolus followed by 3mL/ A 40-year-old male presents to the hospital for the
kg/hr with repeat bolus (20mL/kg followed by 3mL/ fifth time in the past two years with acute pancre-
Kg/hr) at 12hrs into therapy if increases in HCT, atitis. His hepatic profile has always been normal,
BUN, or Cr are seen. Lactated ringers is preferred imaging has revealed non-dilated bile ducts, and
over normal saline, as there was a RCT by Wu et al. he does not consume any alcohol. He is on no
comparing the two solutions, which found there was medications and has no other medical problems.
a significant reduction in SIRS after 24 hours among He did undergo ERCP with biliary sphincterotomy
subjects resuscitated with lactated Ringer’s solution, during one of the episodes, which was unremark-
compared with normal saline (84 percent reduction able; there was no dilation and no debris removed
vs 0 percent, respectively; p = .035); administration from the bile duct with balloon sweeps.
of lactated Ringer’s solution also reduced levels of
CRP, compared with normal saline (51.5 vs 104 mg/ Which of the following gene mutations does this
dL, respectively; p = .02). patient most likely have?
CT abdomen is not necessary to make the
diagnosis of acute pancreatitis if the patient has A. PTEN
classic symptoms and an elevated lipase. Broad B. STK11
spectrum antibiotics are not recommended pro- C. SPINK1
phylactically, but may be used in those patients D. MSH6
with concern for ascending cholangitis or infected E. MLH1
pancreatic fluid collections. Urgent ERCP is rec-
ommended in patients with cholangitis, but is not CORRECT ANSWER: C
necessary for all patients with gallstone pancre-
atitis. Those with concern for choledocholithiasis RATIONALE
should undergo ERCP, but it should not be done This patient has recurrent idiopathic pancreatitis.
urgently, unless there is evidence of cholangitis. Recent studies have suggested that the majority of
72 Digestive Diseases Self-Education Program®

patients with idiopathic chronic pancreatitis have CORRECT ANSWER: A


a variety of mutations in susceptibility genes such
as CFTR, CTRC, CASR, CEL, PRSS1, SPINK1, and RATIONALE
others, as well as disease modifying genes such as This patient has presented with acute pancreatitis,
PRSS1-PRSS2 promoter variants, CLDN2, GGT1, likely gallstone related (given multiple stones seen
and others. These variants are associated with on ultrasound and elevated aminotransferases).
multiple pathologic processes, typically linked to a She has no evidence of biliary obstruction by labs
trypsin activation pathway, or an unfolded protein or imaging, and thus does not require ERCP for
response in the rough endoplasmic reticulum. biliary decompression. Urgent/emergent ERCP is
The remainder of the choices are genes that are required in cases in which patients with gallstone
not associated with recurrent pancreatitis. PTEN is pancreatitis have concomitant ascending chol-
the mutation in Cowden’s syndrome; STK11 is associ- angitis, but is not necessary otherwise. If there
ated with Peutz-Jeghers; MSH6 and MLH1 are mis- is concern for retained bile duct stone, MRCP or
match repair genes associated with Lynch Syndrome. EUS can be used to evaluate first, as these do not
carry the potential risks of ERCP (pancreatitis,
REFERENCES perforation, bleeding etc.)
Jalaly NY, Moran RA, Fargahi F et al. An Evalua-
tion of Factors Associated With Pathogenic PRSS1, REFERENCES
SPINK1, CTFR, and/or CTRC Genetic Variants in Crockett, S. D., Wani, S., Gardner, T. B., Falck-
Patients With Idiopathic Pancreatitis. Am J Gas- Ytter, Y., Barkun, A. N., & American Gastroentero-
troenterol. 2017 Aug;112(8):1320-1329. logical Association Institute Clinical Guidelines
Committee. (2018). American gastroenterological
association institute guideline on initial manage-
Question 38 ment of acute pancreatitis. Gastroenterology,
A 40-year-old female with a history of obesity 154(4), 1096-1101.
and type 2 diabetes mellitus presents with severe
epigastric pain, nausea and vomiting. She is on
metformin. She denies alcohol use. Her vital signs Question 39
are normal. Her labs are: lipase 1,100 U/L, AST 50 A 72-year-old male with a history of recently diag-
U/L, ALT 60 U/L, TB 0.6 mg/dL, alkaline phos- nosed type 2 diabetes, HTN and long-standing to-
phatase 90 U/L, WBC 11,000 cells/mm3, Hct 40 bacco use presents with painless jaundice. He is found
percent, BUN 25 mg/dL and creatinine 1.1 mg/ to have a three by four cm solid mass in the head of
dL. Ultrasound shows multiple gallstones in the the pancreas, with biliary and pancreatic ductal dila-
gallbladder, with a non-dilated common bile duct. tion. There are multiple small lesions in the liver. His
CA 19-9 is 780 U/mL (normal 0-35 U/mL).
Which statement is true regarding need for ERCP
in this patient? Which is the best-established environmental risk
factor for this patient’s diagnosis?
A. It is not necessary
B. It should be done emergently A. Alcohol use
C. It should be done during the admission, B. Cigarette smoking
but not urgently C. Type 2 diabetes mellitus
D. The risk of worsening pancreatitis after ERCP D. Diet high in red and processed meats
is 15 percent in this patient E. Living near power lines
E. It should not be done; percutaneous biliary
drainage is preferred in this setting CORRECT ANSWER: B
Chapter 3 — Pancreatic physiology & disease 73

RATIONALE post-ERCP pancreatitis, presumably by preventing


This patient presents with a pancreatic adenocar- pancreatic duct obstruction related to edema from
cinoma in the head of the pancreas, with likely manipulation, and allowing pancreatic juices to
liver metastases. The most well established envi- flow normally into the duodenum. Choudhary et
ronmental risk factor for this is cigarette smoking. al. conducted a meta-analysis of eight RCTs (656
The other choices (except for living near power subjects) and 10 nonrandomized studies (4904
lines) are also associated with increased risk of subjects). Their meta-analysis of the RCTs showed
pancreatic cancer, but the association is not as that prophylactic PD stents decreased the odds
strong and the relative risk not as high. of post-ERCP pancreatitis (OR, 0.22; 95 percent
CI, 0.12–0.38; P less than .01). The absolute risk
REFERENCES reduction was 13.3 percent (95 percent CI, 8.8
Hassan MM, Bondy ML, Wolff RA, et al. Risk fac- percent–17.8 percent). The NNT was only eight.
tors for pancreatic cancer: case control study. Am Rectal indomethacin has also been shown to
J Gastroenterol 2007;102(12):2696–707. decrease the incidence of post-ERCP pancreatitis,
Hidalgo, M. Pancreatic cancer. N Engl J Med via an anti-inflammatory pathway. Oral NSAIDs
2010; 362(17):1605–17. are not used for this purpose. One study evaluat-
ing the use of oral diclofenac to decrease the risk of
post-ERCP pancreatitis was negative. While aggres-
Question 40 sive intravenous fluids should be given in patients
A 21-year-old woman (45 kg) undergoes ERCP for with high risk of post-ERCP pancreatitis, lactated
choledocholithiasis. Biliary cannulation is difficult, ringers is preferred, and a bolus of 4,000mL for a
and the wire enters the pancreatic duct three times small woman is too aggressive; the usual bolus for
before finally entering the bile duct. Three stones treatment of acute pancreatitis is 20mL/kg (900mL
are removed with balloon sweeps. What should be for this patient). Pancreatic sphincterotomy is a risk
done to decrease the risk of post-ERCP pancreati- factor for post-ERCP pancreatitis.
tis in this patient?
REFERENCES
A. Administer ibuprofen 800mg orally Buxbaum J, Yan A, Yeh K, Lane C, Nguyen N,
B. Administer a bolus of 4000mL of Laine L. Aggressive hydration with lactated
normal saline Ringer’s solution reduces pancreatitis after endo-
C. Place a pancreatic duct stent scopic retrograde cholangiopancreatography. Clin
D. Place a metal stent in the bile duct Gastroenterol Hepatol. 2014;12(2):303–307.e1.
E. Perform pancreatic sphincterotomy Choudhary, A., Bechtold, M.L., Arif, M. et al.
Pancreatic stents for prophylaxis against post-
CORRECT ANSWER: C ERCP pancreatitis: a meta-analysis and systematic
review. Gastrointest Endosc. 2011; 73: 275–282.
RATIONALE Cheon, Y.K., Cho, K.B., Watkins, J.L. et al. Ef-
The incidence of post–endoscopic retrograde ficacy of diclofenac in the prevention of post-ERCP
cholangiopancreatography (ERCP) pancreati- pancreatitis in predominantly high-risk patients:
tis has been reported to be approximately three a randomized double-blind prospective trial. Gas-
percent to 10 percent in systematic reviews. This trointest Endosc. 2007; 66: 1126–1132.
patient has some high risk features for post-ERCP Wang AY, Strand DS, Shami VM. Preven-
pancreatitis, including female gender, young age, tion of Post-Endoscopic Retrograde Cholangio-
difficult cannulation, and repeated pancreatic pancreatography Pancreatitis: Medications and
duct cannulation. Pancreatic duct stents have Techniques. Clin Gastroenterol Hepatol. 2016
been shown to decrease the risk of and severity of Nov;14(11):1521-1532.
74 Digestive Diseases Self-Education Program®

Question 41 logical Association Institute Clinical Guidelines


A 67-year-old woman presents with severe acute Committee. (2018). American gastroenterological
pancreatitis. She was admitted four days ago to association institute guideline on initial manage-
the ICU, but has responded well to aggressive in- ment of acute pancreatitis. Gastroenterology,
travenous fluids. Her vital signs have normalized, 154(4), 1096-1101.
she has no evidence of infection or heart failure,
and her abdominal pain is improving. However,
each time the medical team tries to start her on a Question 42
low fat diet, she has some worsening abdominal A 53-year-old woman with an admission six
pain and nausea. She is not able to take in more months ago for severe necrotizing pancreatitis
than a few bites per day due to these symptoms. presents to the hospital with epigastric pain and
Her labs are normal except for an albumin of 3g/ melena. She denies any alcohol or NSAID use and
dL and prealbumin of 9 mg/dL. What should you is on no medications. A CT scan done three weeks
recommend to manage the patient’s nutritional ago reveals that her known necrotic fluid collec-
needs? tion is decreasing in size, and was only two cm at
that time. Her vital signs reveal a heart rate of 115
A. Place a percutaneous endoscopic gastrostomy beats per minute, BP 100/50 mm Hg. Labs: lipase
tube for feeding 400 U/L (normal 0-60 U/L), hepatic profile is
B. Initiate total parenteral nutrition normal, Hemoglobin 7.8 g/dL (baseline is 10). You
C. Begin tube feeds through a naso-jejunal tube perform an urgent upper endoscopy, which reveals
D. Keep patient NPO and trial oral diet again in small gastric varices without stigmata of bleed-
one week ing, no blood in stomach, no ulcers throughout
E. Start a multivitamin and albumin infusions the exam, but some blood is seen coming from the
major ampulla.
CORRECT ANSWER: C
What is the next best step?
RATIONALE
Nutrition is a very important component of therapy A. Interventional radiology consult for
for acute pancreatitis. Enteral feeding may de- angio-embolization
crease septic complications by helping to maintain B. Treatment with cyanoacrylate injection
the integrity of the intestinal mucosa, reducing bac- C. Nuclear medicine tagged red blood cell scan
terial translocation and seeding, and maintaining D. Octreotide drip
glycemic control. Meta-analyses have demonstrat- E. Surgical consultation for pancreatic resection
ed that enteral feeding (compared to TPN), reduces
infectious complications, multi-organ failure and CORRECT ANSWER: A
mortality. Clinical trials suggest that gastric feed-
ing via nasogastric tube might be as well tolerated RATIONALE
as jejunal feeding, with similar outcomes; however This patient has hemosuccus pancreaticus, which
placing a PEG in a patient that will likely be able to is bleeding from the main pancreatic duct. This
tolerate an oral diet soon would be inappropriate. can be due to a pseudoaneurysm in the peripan-
Keeping the patient nil per os for another week is creatic arterial circulation, or bleeding from small
not an appropriate option. vessels within a pseudocyst or fluid collection
that communicates with the pancreatic duct. The
REFERENCE patient is hemodynamically unstable, and the di-
Crockett, S. D., Wani, S., Gardner, T. B., Falck- agnosis has been confirmed on endoscopy, so the
Ytter, Y., Barkun, A. N., & American Gastroentero- next step is angiogram and possible embolization.
Chapter 3 — Pancreatic physiology & disease 75

Treatment with cyanoacrylate injection is Atrophic gastritis). In order for intrinsic factor
a treatment option for bleeding gastric varices; to bind vitamin B12, B12 must first be released
while this patient does have small gastric varices, from binding with the R-protein, which occurs via
there are no stigmata and another source was pancreatic protease breakdown of the R-protein.
found, so it would not be appropriate to treat Patients with chronic pancreatitis are not able to
them. Nuclear medicine tagged red blood cell break down the R-protein as efficiently, and thus
scan has no role when the site of bleeding is found can develop vitamin B12 deficiency.
endoscopically.
REFERENCES
REFERENCE Green R. Vitamin B12 deficiency from the perspec-
Yu, P and Gong, J. Hemosuccus pancreaticus: A tive of a practicing hematologist. Blood. 2017; May;
mini-review. Ann Med Surg (Lond). 2018 Apr; 28: 129 (19):2603-2611.
45–48. Gueant GL at al. Malabsorption of vitamin B12
in pancreatic insufficiency of the adult and of the
child. Pancreas 1990 Sep;5(5):559-67.
Question 43
A 65-year-old male with chronic pancreatitis
related to long-standing alcohol use comes to see Question 44
you for a second opinion. He has been abstinent Which of the following conditions carries the
from alcohol for twenty years. He reports a one greatest risk for the development of pancreatic
year history of six loose, oily stools per day, but adenocarcinoma?
minimal abdominal pain. He was recently found
to have vitamin B12 deficiency by his PCP. What is A. Peutz-Jeghers syndrome
the likely mechanism for this patient’s vitamin B12 B. BRCA-2 mutation
deficiency? C. Lynch syndrome
D. PRSS1 mutation
A. Antibodies to intrinsic factor E. Cystic fibrosis
B. Atrophy of gastric lining due to chronic
alcohol use CORRECT ANSWER: D
C. Reduced breakdown of the R-protein by
pancreatic proteases RATIONALE
D. Decreased absorption of vitamin B12 in the The PRSS1 gene encodes cationic trypsinogen.
ileum due to chronic diarrhea A mutation in this gene causes hereditary pan-
E. Loss of vitamin B12 in stool, due to steatorrhea creatitis, which carries an approximately 50-fold
increased risk of developing pancreatic adeno-
CORRECT ANSWER: C carcinoma (approximately 50 percent chance).
Peutz-Jeghers patients have an 11 to 36 percent
RATIONALE risk of developing pancreatic adenocarcinoma in
Vitamin B12 absorption requires intrinsic fac- their lifetime. BRCA-2 carriers have about a five
tor to bind to B12 to facilitate absorption in the percent risk of pancreatic cancer. Lynch Syndrome
terminal ileum. Any interruption of terminal ileal patients have an approximately two to 18 percent
absorptive capacity can thus lead to vitamin B12 lifetime risk of pancreatic cancer, depending on
deficiency (eg. Crohn’s disease, ileal resection). which mismatch repair gene is mutated. Patients
Intrinsic factor is produced by parietal cells, so any with cystic fibrosis have a very high likelihood (80
condition that leads to decreased parietal cell mass to 85 percent) of developing pancreatic insuf-
or function can lead to vitamin B12 deficiency (eg. ficiency, and may develop overt pancreatitis, but
76 Digestive Diseases Self-Education Program®

there is no specific association with pancreatic limited data regarding the use of steroid-sparing
adenocarcinoma. agents in patients with Type 2 AIP.

REFERENCES REFERENCES
Klein AP. Genetic susceptibility to pancreatic can- Sah RP,Chari ST, Pannala R et al. Differences in
cer. Mol carcinog 2012;51: 14. clinical profile and relapse rate of type 1 versus
Lowenfels AB, Maisonneuve P, DiMagno EP et type 2 autoimmune pancreatitis. Gastroenterol-
al. Hereditary pancreatitis and the risk of pancre- ogy. 2010;139 (1): 140.
atic cancer. International Hereditary Pancreatitis O’Reilly DA, Malde DJ, Duncan T, et al. Re-
Study Group. J Natl Cancer Inst. 1997;89 (6):442. view of the diagnosis, classification and manage-
ment of autoimmune pancreatitis. World J Gastro-
intest Pathophysiol. 2014 May 15; 5(2): 71–81.
Question 45
A 24-year-old man with ulcerative colitis in remis-
sion on vedolizumab presents with acute pancre- Question 46
atitis. Labs reveal a lipase 875 U/mL (normal 0-60 A 27-year-old man with Crohn’s ileocolitis on inf-
U/L), AST 35 U/L, ALT 45 U/L, total bilirubin 2.5 liximab and azathioprine for the past two months
mg/dL, alkaline phosphatase 200 U/L, WBC 9,000 presents with severe epigastric pain radiating to
cells/mm3. CT scan reveals a mass in the pancre- his back and nausea. He does not smoke or drink
atic head with mild biliary and pancreatic ductal any alcohol. He has no family history of pancreati-
dilation. EUS-FNA reveals a granulocytic epithelial tis or inflammatory bowel disease. His vital signs
lesion without any evidence of malignancy. What is are normal. Lipase is 3,000 U/L and hepatic panel
the next step in the management of this patient? is normal. Ultrasound shows a normal gallbladder
without stones and no biliary dilation. What is the
A. Whipple procedure likely etiology of his acute pancreatitis?
B. PET-CT scan
C. Laparoscopy A. Alcohol
D. Initiation of glucocorticoids B. Gallstones
E. Switch to infliximab C. Azathioprine
D. Duodenal Crohn’s disease
CORRECT ANSWER: D E. SPINK1 mutation

RATIONALE CORRECT ANSWER: C


This patient has Type 2 autoimmune pancreatitis
(AIP). Characteristic features include: association RATIONALE
with inflammatory bowel disease, mass-like lesion Drugs account for one to two percent of all cases
or other focal features on imaging, biopsy showing of acute pancreatitis, and azathioprine is a com-
granulocyte-epithelial lesions with absent or very mon culprit. The pancreatitis caused by azathio-
mildly positive IgG4 staining. Unlike type 1 AIP, prine is usually idiosyncratic, but if it occurs, the
type 2 involves only the pancreas. Type 2 AIP pa- medication should be stopped, and there should
tients usually present with acute pancreatitis and/ be no re-challenge with it or with the related mer-
or obstructive jaundice due to mass-like lesion in captopurine. While the other etiologies are pos-
the pancreatic head. Type 2 AIP, like type 1 AIP, is sible, there is no mention of upper GI symptoms
very responsive to treatment with glucocorticoids. suggestive of active Crohn’s, no family history of
Surgical resection is not indicated given the high pancreatitis, and no evidence of biliary disease; in
likelihood of response to medical therapy. There is addition he denies alcohol use.
Chapter 3 — Pancreatic physiology & disease 77

REFERENCES REFERENCE
Badalov N, Baradarian R, Kadirawel I, et al. Drug- Modlin IM, Oberg K et al. Gastroenteropancreatic
induced acute pancreatitis: an evidence-based neuroendocrine tumours. Lancet Oncology 2008;
review. Clin Gastroenterol Hepatol 2007;5: 648. 9: 61-72.

Question 47 Question 48
A 63-year-old man with recently diagnosed A 36-year-old man presents with severe acute
diabetes presents with diarrhea and a new rash. pancreatitis. His initial vital signs are: tempera-
The rash is located around his mouth and on his ture 37 degrees C, heart rate of 115 beats/minute
buttocks and groin. The rash appears erythema- and blood pressure of 89/50 mm Hg. Labs reveal
tous and there are multiple vesicles; some fresh a white blood cell count of 18,000 cells/mm3,
and others crusting over. He has lost 26 pounds in Hct of 50 percent, lipase 3000 U/L, creatinine
the past two months unintentionally, so a CT scan of 1.5 mg/dL, normal hepatic function panel. A
was done, which reveals a three cm solid lesion in CT scan reveals significant fluid and stranding
the body of the pancreas. What is the most likely around the pancreas, with an area in the pancre-
diagnosis? atic body that is not enhancing. The gallbladder
is normal and there is no biliary dilation. He is
A. Insulinoma started on aggressive IV fluid hydration with
B. Pancreatic adenocarcinoma lactated ringers.
C. Glucagonoma
D. VIPoma What is the next best step?
E. Pancreatic Lymphoma
A. Initiate broad spectrum antibiotics and
CORRECT ANSWER: C aggressive intravenous fluid hydration
B. Start TPN via central venous access
RATIONALE C. Keep NPO, monitor hematocrit and BUN/
Glucagonoma is a rare neuroendocrine tumor aris- creatinine and adjust fluid accordingly
ing from pancreatic alpha-2 cells of the islets. This D. Perform ERCP
tumor secretes high levels of glucagon, which lead E. Perform endoscopic ultrasound followed by
to insulin resistance, diarrhea, and weight loss. ERCP if stone is confirmed in bile duct
Necrolytic migratory erythema is the characteris-
tic rash located on the limbs, around the lips and CORRECT ANSWER: C
groin/perineum/buttocks, which is seen in 70 to
80 percent of patients with this particular tumor. RATIONALE
NME is described as a crusting, erythematous, ve- This patient has evidence of acute necrotizing pan-
sicular rash, and is often the presenting symptom. creatitis. Antibiotics are not recommended unless
Insulinoma causes hypoglycemia and can there is evidence of acute cholangitis or infection
often present with seizures or other sequelae of elsewhere. This patient does meet SIRS criteria
hypoglycemia. It is the most common pancreatic due to the severity of his pancreatitis, he is afebrile
endocrine tumor. VIPoma causes significant secre- and there is no evidence of cholangitis, so antibi-
tory diarrhea, often leading to dehydration and otics are unnecessary.
hypokalemia. Pancreatic lymphoma can often be The mainstay of therapy for acute pancre-
confused for adenocarcinoma, but does not cause atitis is goal-directed fluid therapy, with close
any symptoms of hormonal hypersecretion and monitoring of hemoglobin/hematocrit and BUN/
would not present as this patient did. Creatinine to ensure adequate fluid hydration and
78 Digestive Diseases Self-Education Program®

avoidance of hemoconcentration (which can lead RATIONALE


to end-organ damage). TPN would not be ap- This patient is presenting with gallstone pancre-
propriate, as enteral feeding is always preferred. atitis and ascending cholangitis, as evidenced by
There is no evidence of biliary obstruction or cho- Charcot’s triad of RUQ pain, fever and jaundice.
ledocholithiasis, thus ERCP and EUS would not be Cholangitis is the primary indication to perform
warranted at this time. ERCP in the setting of acute gallstone pancreatitis.
Given the clinical presentation, labs and imaging,
REFERENCE there is a very high probability of a common bile
Crockett, S. D., Wani, S., Gardner, T. B., Falck- duct stone, therefore an MRCP is unnecessary. Ur-
Ytter, Y., Barkun, A. N., & American Gastroentero- sodeoxycholic acid is not indicated in patients with
logical Association Institute Clinical Guidelines cholangitis. Cholecsytectomy should be done after
Committee. (2018). American gastroenterological the biliary obstruction is relieved with ERCP.
association institute guideline on initial manage-
ment of acute pancreatitis. Gastroenterology, REFERENCE
2018; 154(4), 1096-1101. Crockett, S. D., Wani, S., Gardner, T. B., Falck-
Ytter, Y., Barkun, A. N., & American Gastroentero-
logical Association Institute Clinical Guidelines
Question 49 Committee. (2018). American gastroenterological
A 40-year-old woman with obesity presents to the association institute guideline on initial manage-
emergency department with severe epigastric pain ment of acute pancreatitis. Gastroenterology,
for 10 hours. She is febrile with a temperature of 2018; 154(4), 1096-1101.
39.4o C , HR 112 bpm and blood pressure 100/70
mm Hg. Abdominal exam reveals right upper
quadrant tenderness on palpation. White blood Question 50
cell count is 22,000 cells/L, lipase 3000 U/L, AST Which of the following statements is true regard-
222 U/L, ALT 400 U/L, alkaline phosphatase 300 ing hereditary pancreatitis?
U/L and bilirubin 4.1 mg/dL. CT scan of the abdo-
men reveals mild stranding around the head of the A. It is inherited in an autosomal recessive
pancreas, multiple small gallstones in the gallblad- fashion
der and a common bile duct measuring nine mm. B. It is due to a mutation in cationic trypsinogen
She is started on aggressive IV fluids and broad C. It is associated with a five-fold increased risk
spectrum antibiotics. of pancreatic cancer
D. It rarely leads to chronic pancreatitis
What is the next best step in the management of E. Approximately 85 percent of patients will
this patient? develop type 1 diabetes by the time they are 20
years old
A. Continued supportive care with aggressive
IV fluids and antibiotics CORRECT ANSWER: B
B. Urgent ERCP
C. MRCP RATIONALE
D. Cholecystectomy Hereditary pancreatitis, caused by mutations in
E. Ursodeoxycholic acid the cationic trypsinogen gene (PRSS1), carries a
40- to 60-fold increased lifetime risk for develop-
CORRECT ANSWER: B ing pancreatic cancer. It is an autosomal dominant
condition with incomplete penetrance, and often
leads to chronic pancreatitis due to recurrent epi-
Chapter 3 — Pancreatic physiology & disease 79

sodes of acute pancreatitis. Due to chronic inflam-


mation and fibrosis of the pancreatic parenchyma,
up to a quarter of patients can develop type 1
diabetes by mid-adulthood.

REFERENCES
Teich N, Rosendahl J, Toth M, et al. Mutations of
human cationic trypsinogen (PRSS1) and chronic
pancreatitis. Hum Mutat 2006;27(8):721-730.
NIH Genetics Home REFERENCE-https://
ghr.nlm.nih.gov/condition/hereditary-pancreatitis
80 Digestive Diseases Self-Education Program®
Answers & critiques

CHAPTER 4

Diseases of the biliary tract


Joshua Paul Spaete, MD, Moira B. Hilscher, MD,
John E. Eaton, MD, and Christina Ha, MD

Question 1 RATIONALE
A 45-year-old man undergoes an upper endos- The patient was found to have an incidental
copy for further evaluation of reflux. At the time sporadic ampullary adenoma. These adenomas
of his EGD, an abnormality is seen at his am- can progress to adenocarcinoma and should be
pulla. No additional lesions are seen within the resected. Historically pancreaticoduodenectomy
examined duodenum. Further evaluation with a or a trans duodenal ampullectomy were the
side viewer reveals the abnormality noted in the treatment of choice. However, these procedures
image. Biopsies are obtained and return positive are associated with significant morbidity and
for adenoma without high grade dysplasia. Labs mortality, and endoscopic removal for non-in-
reveal a total bilirubin of 1.0 mg/dL, alkaline vasive ampullary adenomas is preferred. In this
phosphatase of 67 IU/L, AST 15 IU/L, ALT 18 scenario, the adenoma is greater than or equal
IU/L. An EUS examination confirms a 15 mm to one cm but not large (less than 4cm) and is
ampullary lesion without invasion of the mus- without ductal involvement, endoscopic resec-
cularis propria, common bile duct or pancreas tion should be considered as first line therapy.
duct. The next best step in management is: If, at the time of attempted endoscopic resec-
tion, the lesion does not lift, surgical resection
could be considered. Given that the adenoma
could harbor underlying carcinoma, which may
not have been diagnosed by pinch biopsies,
and the relatively young age and health of the
patient, upfront resection would be favored over
surveillance.

REFERENCES
Ardengh, J.C., et al., Endoscopic papillectomy:
The limits of the indication, technique and re-
sults. World J Gastrointest Endosc, 2015. 7(10):
A. Surveillance with side viewing exam in p. 987-94.
one year Committee, A.S.o.P., et al., The role of en-
B. Transduodenal ampullectomy doscopy in ampullary and duodenal adenomas.
C. Endoscopic resection Gastrointest Endosc, 2015. 82(5): p. 773-81.
D. No further intervention is required El, H., II and G.A. Cote, Endoscopic diag-
E. Pancreaticoduodenectomy nosis and management of ampullary lesions.
Gastrointest Endosc Clin N Am, 2013. 23(1): p.
CORRECT ANSWER: C 95-109.

81
82 Digestive Diseases Self-Education Program®

Question 2 NETs. While endoscopic resection for ampullary


A 52-year-old man undergoes an upper endos- neuroendocrine tumors has been described, given
copy for further evaluation of dysphagia. At the the high rate of metastases and an inability to as-
time of his EGD, an abnormality is seen at his sess lymph nodes with endoscopic resection, this
ampulla. Further evaluation with a side viewing approach should be reserved for patients who are
endocsope reveals the abnormality noted in the not surgical candidates. As noted previously, even
image. Biopsies are obtained and return posi- small neuroendocrine tumors are at high risk for
tive for a neuroendocrine tumor. Labs reveal a metastases and surveillance is not recommended.
total bilirubin of 1.0 mg/dL, alkaline phospha-
tase of 67 IU/L, AST 15 IU/L, ALT 18 IU/L. An REFERENCES
EUS examination confirms an 11 mm ampullary Baptiste, G.G., et al., Is there an Optimal Surgi-
lesion without enlarged lymph nodes, invasion cal Approach to Neuroendocrine Tumors of the
of the common bile duct or pancreas duct. Cross Ampulla? A Single Institution Experience Over 15
sectional imaging of the liver does not reveal evi- Years. Am Surg, 2016. 82(7): p. 637-43.
dence of metastatic disease or enlarged LN. The Carter, J.T., et al., Neuroendocrine tumors of the
next best step in management is: ampulla of Vater: biological behavior and surgical
management. Arch Surg, 2009. 144(6): p. 527-31.
Dogeas, E., et al., Duodenal and Ampullary
Carcinoid Tumors: Size Predicts Necessity for
Lymphadenectomy. J Gastrointest Surg, 2017.
21(8): p. 1262-1269.
Jayant, M., et al., Neuroendocrine tumors of
the ampulla of vater: presentation, pathology and
prognosis. JOP, 2012. 13(3): p. 263-7.

Question 3
A. Surveillance with side viewing exam in A 54-year-old woman presents to your clinic for
one year further nausea, vomiting and jaundice. On exam,
B. Transduodenal (surgical) ampullectomy her abdomen is soft, non-tender and not distend-
C. Endoscopic resection ed. Her sclerae are anicteric. Laboratory studies
D. No further intervention is required are obtained and
E. Pancreaticoduodenectomy reveal a total
bilirubin of 1.5
CORRECT ANSWER: E mg/dL, alkaline
phosphatase of
RATIONALE 24 IU/L, AST
Given the rarity of ampullary neuroendocrine 18 IU/L, ALT 19
tumors limited data exists regarding the best ap- IU/L. Her MRCP
proach to this type of tumor. Pancreaticoduode- image is shown
nectomy is recommended for all ampullary neuro- here.
endocrine tumors given the high rate of metastases
seen even with small lesions. Transduodenal For definitive
ampullectomy is felt to offer an inadequate onco- management,
logical resection and generally has not been recom- you recommend:
mended as the first line approach to ampullary
Chapter 4 — Diseases of the biliary tract 83

A. No further therapy is needed ously resolved within 15 to 30 minutes. Labs reveal


B. ERCP a total bilirubin of 4 mg/dL, alkaline phosphatase
C. EUS of 250 IU/L, AST 652 IU/L, ALT 530 IU/L, lipase
D. Surgical Resection 50 U/L (ULN 51) , INR 3.5, WBC 25.1 10^9/L. A
E. Repeat MRI in 6 months RUQ ultrasound is obtained and shows a 12 mm
CBD with cholelithiasis. His GB wall is not thick-
CORRECT ANSWER: D ened and no pericholecystic fluid is seen. Soon after
your evaluation in the ED the patient is noted to be
RATIONALE hypotensive and confused. He is placed on inotro-
This patient has a type I choledochal cyst. The risk pes intubated and transferred to the ICU. In addi-
of malignancy increases with age approaching 28 tion to antibiotics and IV fluids you recommend:
percent in one series. Most cancers are seen within
the cyst or gallbladder. She should undergo cyst A. MRCP
resection, cholecystectomy with hepaticoenteros- B. EUS
tomy if possible. While ERCP can help to better C. Urgent ERCP
delineate her anatomy and may be requested D. Continued supportive care
prior to surgical intervention, it will not provide E. ERCP in 5 days once clopidogrel has been
definitive management. EUS is unlikely to add any eliminated
significant information and is not necessary. Given F. Percutaneous cholecystostomy tube
the malignant potential of this cyst, it should
be resected, so further imaging or no additional CORRECT ANSWER: C
therapy would not be appropriate in this scenario.
RATIONALE
REFERENCES This patient has severe cholangitis and fulfills
Martin, R.F., Biliary cysts: a review and sim- criteria for Reynolds pentad (fever, jaundice, ab-
plified classification scheme. Surg Clin North Am, dominal pain, altered mental status and hypoten-
2014. 94(2): p. 219-32. sion). He should undergo urgent (within 24hrs)
Soares, K.C., et al., Choledochal cysts: presen- ERCP. Delayed ERCP has been associated with an
tation, clinical differentiation, and management. J increased risk of death, organ failure and length
Am Coll Surg, 2014. 219(6): p. 1167-80. of ICU stay. ERCP can safely be performed in pa-
Soares, K.C., et al., Presentation and Clini- tients on anticoagulants. In the case outline above
cal Outcomes of Choledochal Cysts in Children sphincterotomy, which is high risk for bleeding,
and Adults: A Multi-institutional Analysis. JAMA should be avoided and a stent or balloon dilation
Surg, 2015. 150(6): p. 577-84. could be considered as preferred alternatives for
Soreide, K., et al., Bile duct cysts in adults. Br decompression and stone extraction, respectively.
J Surg, 2004. 91(12): p. 1538-48. The pre-test probability for choledocholithiasis is
high enough that additional imaging is not neces-
sary, and neither EUS nor MRCP is warranted.
Question 4 Percutaneous drain placement could be consid-
An 84-year-old man with a past medical history ered if ERCP fails or is not available.
notable for CAD with drug eluting stent placement
three months ago and atrial fibrillation on an 81 REFERENCES
mg aspirin and clopidogrel presents to the hospital Hou, L.A., et al., Optimal Timing of Endoscopic
with right upper quadrant pain for the last 24 hrs. Retrograde Cholangiopancreatography in Acute
He states he has had several similar episodes over Cholangitis. J Clin Gastroenterol, 2017. 51(6): p.
the last 3 months; however, the episodes had previ- 534-538.
84 Digestive Diseases Self-Education Program®

Khashab, M.A., et al., Delayed and unsuc- for confirmation of the diagnosis, if the fluid is
cessful endoscopic retrograde cholangiopancrea- analyzed for bile, as well as source control. While
tography are associated with worse outcomes in ERCP has been shown to be highly effective for
patients with acute cholangitis. Clin Gastroenterol treatment of bile leaks, it will do little to hasten an
Hepatol, 2012. 10(10): p. 1157-61. already formed symptomatic fluid collection that
Kiriyama, S., et al., Tokyo Guidelines 2018: may or may not be communicating with the duct.
diagnostic criteria and severity grading of acute Surgical revision should be considered if percu-
cholangitis (with videos). J Hepatobiliary Pancreat taneous drainage and or ERCP fail to resolve the
Sci, 2018. 25(1): p. 17-30. leak, or if a transection injury occurs, as this is
unlikely to be amenable to endoscopic therapy.

Question 5 REFERENCES
A 35-year-old woman who underwent cholecystec- Ahmad, F., et al., An algorithm for the manage-
tomy three days previously presents to the ED for ment of bile leak following laparoscopic cholecys-
further evaluation of RUQ abdominal pain and fe- tectomy. Ann R Coll Surg Engl, 2007. 89(1):
vers. On physical exam her abdomen is tender in the p. 51-6.
RUQ with guarding and rebound. Labs reveal a total Baillie, J., Endoscopic approach to the patient
bilirubin of 2 mg/dL, alkaline phosphatase of 150 with bile duct injury. Gastrointest Endosc Clin N
IU/L, AST 60 IU/L, ALT 82 IU/L, lipase 50 U/L (50 Am, 2013. 23(2): p. 461-72.
ULN), WBC 16.4 10^9/L. Cross sectional imaging is Nunez, D., Jr., J.L. Becerra, and L.C. Martin,
performed and reveals this finding seen here. Subhepatic collections complicating laparoscopic
cholecystectomy: percutaneous management
Abdom Imaging, 1994. 19(3): p. 248-50.
Saraswat, V.A., et al., Endoscopic manage-
ment of postoperative bile leak. J Gastroenterol
Hepatol, 1996. 11(2): p. 148-51.

Question 6
A 53-year-old woman presents to the ED for further
management of persistent right upper quadrant
pain. She is normotensive and afebrile. On exam,
The next best step in management is: she is tender to palpation in the RUQ with guarding
but no rebound. Labs reveal a total bilirubin of 1.7
A. ERCP mg/dL, alkaline phosphatase of 80 IU/L, AST 35
B. Percutaneous drainage tube placement IU/L, ALT of 36 IU/L, lipase of 15 U (ULN 50) and
C. Surgical revision WBC of 11.2 10^9/L. A RUQ ultrasound is obtained
D. No further intervention is needed and reveals a CBD of 5 mm, a thickened gallbladder
E. EUS wall, pericholecystic fluid and cholelithiasis. The
next best step in management is:
CORRECT ANSWER: B
A. ERCP
RATIONALE B. EUS
This patient has a symptomatic bile leak post cho- C. MRCP
lecystectomy and percutaneous drainage should D. No further management necessary
be performed. Percutaneous drainage will allow E. Surgical referral for cholecystectomy
Chapter 4 — Diseases of the biliary tract 85

CORRECT ANSWER: E and lipase of 10 U (ULN 50 U). A prior episode


six months ago revealed a similar elevation in her
RATIONALE AST and ALT. MRCP was performed and revealed
This patient has cholecystitis and should undergo a CBD which measured five millimeters without
cholecystectomy. While she does have a mild eleva- evidence of a filling defect. EUS is performed and
tion in her bilirubin, it is below the 1.8-4 mg/dL is unrevealing.
which is considered a strong predictor of choledo-
cholithiasis. She has no other very strong, strong, You recommend:
or moderate predictors for choledocholithiasis, per
the ASGE 2010 proposed guidelines, and should A. Initiation of a fiber supplement
undergo cholecystectomy. Of note, recent studies B. Initiation of dicyclomine
evaluating the ASGE high risk criteria for choledo- C. ERCP with manometry
cholithiasis have consistently demonstrated 50 per- D. Initiation of narcotic pain medications
cent to 70 percent accuracy in predicting choledo- E. No further therapy necessary
cholithiasis in patients in whom high risk criteria
are met. The ASGE guidelines are thus limited and CORRECT ANSWER: C
may result in unnecessary diagnostic ERCP. These
guidelines should be used in conjunction with clini- RATIONALE
cal judgement. This patient fulfills criteria for functional biliary
sphincter of Oddi disorder (type II SOD). She
REFERENCES has biliary type pain and has had elevated liver
Committee, A.S.o.P., et al., The role of endoscopy enzymes with episodes of pain. Structural etiolo-
in the evaluation of suspected choledocholithiasis. gies for biliary have been excluded. In this patient,
Gastrointest Endosc, 2010. 71(1): p. 1-9. ERCP with biliary manometry is a reasonable ap-
He, H., et al., Accuracy of ASGE high-risk proach. If manometry is positive, a biliary sphinc-
criteria in evaluation of patients with suspected terotomy should be performed. ERCP in patients
common bile duct stones. Gastrointest Endosc, with functional biliary sphincter of Oddi disorder
2017. 86(3): p. 525-532. carries a very high risk of post ERCP pancreatitis.
Narvaez Rivera, R.M., et al., Accuracy of ASGE In those who fulfill criteria, have positive manom-
criteria for the prediction of choledocholithiasis. etry and are properly selected, long term improve-
Rev Esp Enferm Dig, 2016. 108(6): p. 309-14. ment with sphincterotomy has been shown in up
to 70-90 percent of patients. Fiber supplements,
dicyclomine and narcotic pain medications have
Question 7 not been shown to be effective for biliary sphincter
A 40-year-old woman presents to your clinic for of Oddi disorder and are unlikely to improve the
further evaluation of RUQ abdominal pain. The patient’s underlying pain.
patient reports intermittent RUQ pain which
occurs every two to three months. It typically REFERENCES
will last 30 minutes to one hour and resolve. In Cotton, P.B., et al., Effect of endoscopic sphincter-
between episode she feels well. It is not improved otomy for suspected sphincter of Oddi dysfunction
with acid suppression or bowel movements. She on pain-related disability following cholecystecto-
underwent cholecystectomy for cholecystitis 10 my: the EPISOD randomized clinical trial. JAMA,
years prior. LFTs have been obtained during these 2014. 311(20): p. 2101-9.
episodes. Labs following her most recent episode Cotton, P.B., et al., Rome IV. Gallbladder and
reveal a total bilirubin of 1 mg/dL, alkaline phos- Sphincter of Oddi Disorders. Gastroenterology,
phatase of 50 IU/L, AST 103 IU/L, ALT 110 IU/L, 2016.
86 Digestive Diseases Self-Education Program®

Freeman, M.L., Complications of endoscopic biliary sphincter of Oddi disorder. Her pain is
retrograde cholangiopancreatography: avoidance consistent with biliary type pain however she does
and management. Gastrointest Endosc Clin N Am, not have abnormal liver function tests or biliary
2012. 22(3): p. 567-86. ductal dilation. This patient should not undergo
Geenen, J.E., et al., The efficacy of endoscopic an ERCP for suspected sphincter of Oddi disorder.
sphincterotomy after cholecystectomy in patients The EPISOD trial demonstrated a 10-15 percent
with sphincter-of-Oddi dysfunction. N Engl J rate of post ERCP pancreatitis in expert hands.
Med, 1989. 320(2): p. 82-7. Prior studies have suggested rates of post ERCP
Toouli, J., et al., Manometry based randomised pancreatitis as high as 30 percent in those with
trial of endoscopic sphincterotomy for sphincter of suspected sphincter of Oddi dysfunction.
Oddi dysfunction. Gut, 2000. 46(1): p. 98-102.
REFERENCES
Cotton, P.B., et al., Effect of endoscopic sphincter-
Question 8 otomy for suspected sphincter of Oddi dysfunction
A 40-year-old woman presents to your clinic for on pain-related disability following cholecystecto-
further evaluation of RUQ abdominal pain. The my: the EPISOD randomized clinical trial. JAMA,
patient reports intermittent RUQ pain occurring 2014. 311(20): p. 2101-9.
several times per month. At its most severe, it is Cotton, P.B., et al., Rome IV. Gallbladder and
a 10/10, and typically lasts two to three hours. Sphincter of Oddi Disorders. Gastroenterology,
She has not noted an improvement with bowel 2016.
movements or acid suppression. She underwent Freeman, M.L., Complications of endoscopic
cholecystectomy for similar pain one year prior retrograde cholangiopancreatography: avoidance
without improvement. LFTs have been obtained and management. Gastrointest Endosc Clin N Am,
during her pain episodes and reveal a total biliru- 2012. 22(3): p. 567-86.
bin of 1 mg/dL, alkaline phosphatase of 50 IU/L, Yaghoobi, M., et al., Incidence and predic-
AST 15 IU/L, ALT 25 IU/L, and lipase of 10 U tors of post-ERCP pancreatitis in patients with
(ULN 50 U). A CT scan was previously obtained suspected sphincter of Oddi dysfunction undergo-
and revealed a CBD which measured six mm. ing biliary or dual sphincterotomy: results from
EUS was performed and was unremarkable. The the EPISOD prospective multicenter randomized
patient reports she has researched online and she sham-controlled study. Endoscopy, 2015. 47(10):
is sure she has sphincter of Oddi dysfunction. She p. 884-90.
is adamant that you perform an ERCP. You inform
her that she does not fulfill criteria for sphincter of
Oddi dysfunction and that her risk of pancreatitis Question 9
following an ERCP is approximately: A 25-year-old man with a history of migraines
presents for further evaluation of intermittent
A. 0-five percent RUQ abdominal pain associated with meals for
B. 10-30 percent several years. He had undergone an abdominal
C. 45-60 percent ultrasound, CT scan of his abdomen and upper
D. 70-85 percent endoscopy without a clear etiology. Given concern
E. 90-100 percent for possible acalculous cholecystitis, he under-
went a cholecystectomy several years ago. His
CORRECT ANSWER: B AST and ALT have been mildly elevated in the
past but are currently normal. alkaline phospha-
RATIONALE tase is elevated at 134 IU/L. IgG4, AMA and im-
This patient does not fulfill criteria for functional munoglobulins are not elevated. HIV is negative.
Chapter 4 — Diseases of the biliary tract 87

He currently is asymptomatic. He underwent an light endoscopy, biopsies are recommended at the


MRI and subsequently an ERCP. His cholangio- time of initial colonoscopy. In an asymptomatic
gram is shown here. patient without a mass lesion, EUS is unlikely to
be of significant benefit and would not be recom-
mended. The patient does not have a clear indi-
cation for an upper endoscopy with or without
biopsies and this is not recommended.

REFERENCES
European Association for the Study of the, L.,
EASL Clinical Practice Guidelines: management of
cholestatic liver diseases. J Hepatol, 2009. 51(2):
p. 237-67.
Lindor, K.D., et al., ACG Clinical Guideline:
Primary Sclerosing Cholangitis. Am J Gastroen-
terol, 2015. 110(5): p. 646-59; quiz 660.
Palmela, C., et al., Inflammatory Bowel Dis-
ease and Primary Sclerosing Cholangitis: A Review
of the Phenotype and Associated Specific Features.
Gut Liver, 2018. 12(1): p. 17-29.
Razumilava, N., G.J. Gores, and K.D. Lin-
He should undergo what additional test as part of dor, Cancer surveillance in patients with primary
his work up of this condition? sclerosing cholangitis. Hepatology, 2011. 54(5): p.
1842-52.
A. Colonoscopy with biopsies Rossi, R.E., D. Conte, and S. Massironi,
B. Colonoscopy without biopsies Primary sclerosing cholangitis associated with
C. EUS inflammatory bowel disease: an update. Eur J
D. Upper endoscopy with biopsies Gastroenterol Hepatol, 2016. 28(2): p. 123-31.
E. Upper endoscopy without biopsies

CORRECT ANSWER: A Question 10


A 77-year-old woman underwent a cholecystec-
RATIONALE tomy four days ago for acute cholecystitis. She did
The cholangiogram is consistent with PSC and this well postoperatively and was discharged POD 1.
patient should undergo a full colonoscopy with She now presents to the emergency department
biopsies regardless of symptoms. The prevalence with right upper quadrant pain and fever. Labs
of IBD in in patients with PSC is approximately 60- reveal a total bilirubin of 2.5 mg/dL, alkaline
80 percent. These patients may be asymptomatic phosphatase of 75 IU/L, AST 60 IU/L, ALT 75
as they are more likely to have quiescent disease. IU/L, amylase 60 U (ULN 50 U), and WBC of 18.3
In patients with PSC and IBD a threefold risk for 10^9/L. A CT scan is obtained while the patient is
colorectal dysplasia and colorectal cancer has been in the ED and demonstrates an eight centimeter
reported in comparison to patients with IBD alone, fluid collection around the duodenum within the
thus warranting yearly surveillance colonoscopy. In retroperitoneal space. A percutaneous drain is
patients who do not have IBD in addition to their placed. The fluid is sent for analysis and the bili-
PSC, surveillance every five years is recommended. rubin level returns elevated at 13 mg/dL, and the
Given that disease may not be seen with white amylase level is 50,000 U.
88 Digestive Diseases Self-Education Program®

You recommend: similar pain in the past which were not persistent.
On exam, she is tender to palpation in the RUQ
A. ERCP without rebound or guarding. Her sclerae are
B. Upper GI series icteric. Labs reveal a total bilirubin of 3.5 mg/dL,
C. EUS guided cystgastrostomy alkaline phosphatase of 200 IU/L, AST 200 IU/L,
D. Upper endoscopy ALT 190 IU/L, and lipase of 41 U (ULN 50 U) and
E. Supportive care with antibiotics and fluids WBC of 13.5 10^9/L. Her medical history is other-
wise notable for hypertension. You recommend:
CORRECT ANSWER: B
A. Right upper quadrant ultrasound
RATIONALE B. MRCP
This patient has a duodenal perforation and C. EUS
should undergo an upper GI series with gastro- D. ERCP
grafin to localize the site of the leak. Duodenal E. Cholecystectomy
perforation is a rare complication of cholecystec-
tomy occurring in 0.07 percent to 0.09 percent CORRECT ANSWER: A
of cholecystectomy cases. While rare, failure to
recognize this complication is associated with RATIONALE
significant mortality. The location of the fluid col- The differential for a patient with right up-
lection within the retroperitoneal space, as well per quadrant pain and abnormal liver function
as markedly elevated amylase level and elevated tests is broad and includes cholecystitis, biliary
bilirubin level raise concern for a possible perfo- obstruction, acute hepatitis, and viral gastro-
ration. Perforation is a relative contraindication enteritis. In this patient with prior symptoms
to endoscopic evaluation and additional imaging suggestive of biliary colic, cholecystitis or cho-
with an upper GI series should be performed prior ledocholithiasis are a concern. The next best test,
to an endoscopic evaluation. As this is a perfora- in a patient with right upper quadrant pain and
tion and not a bile leak, ERCP is of no benefit. elevated aminotransferase levels, bilirubin and
EUS guided cystgastrostomy would be appropriate alkaline phosphatase, would be a right upper
in the setting of a matured pseudocyst or WON quadrant ultrasound. While the sensitivity for
but, in the setting of a recent perforation, drainage detection of choledocholithiasis is approximately
into the stomach would not be of benefit. 50 percent, ultrasound has a high sensitivity
for cholelithiasis, bile duct dilation and GB wall
REFERENCES thickness. In addition, if the biliary tree appears
Croce, E., et al., Duodenal perforations after normal, doppler studies can be performed to
laparoscopic cholecystectomy. Surg Endosc, 1999. evaluate for possible underlying vascular etiolo-
13(5): p. 523-5. gies of pain and abnormal aminotransferases,
Machado, N.O., Duodenal injury post laparo- ALP or bilirubin. MRCP and EUS have a much
scopic cholecystectomy: Incidence, mechanism, higher sensitivity for choledocholithiasis, approx-
management and outcome. World J Gastrointest imately 92 percent and 95 percent, respectively,
Surg, 2016. 8(4): p. 335-44. than US or CT scan. These should be considered
in those patients who are at an intermediate risk
for choledocholithiasis rather than as a first line
Question 11 test for right upper quadrant pain with abnor-
A 46-year-old woman presents to the ED for fur- mal liver tests. ERCP and cholecystectomy may
ther evaluation of persistent right upper quadrant be necessary in this patient, but the underlying
pain. The patient reports intermittent episodes of etiology should first be determined.
Chapter 4 — Diseases of the biliary tract 89

REFERENCES in the Opisthorchis and Clonorchis genus. While


Committee, A.S.o.P., et al., The role of endoscopy Fasciola does infect the liver, an association has
in the evaluation of suspected choledocholithiasis. not been reported with cholangiocarcinoma.
Gastrointest Endosc, 2010. 71(1): p. 1-9.
Cronan, J.J., US diagnosis of choledocholi- REFERENCES
thiasis: a reappraisal. Radiology, 1986. 161(1): p. Fevery, J., et al., Malignancies and mortality in
133-4. 200 patients with primary sclerosering cholan-
Giljaca, V., et al., Endoscopic ultrasound gitis: a long-term single-centre study. Liver Int,
versus magnetic resonance cholangiopancrea- 2012. 32(2): p. 214-22.
tography for common bile duct stones. Cochrane Razumilava, N., G.J. Gores, and K.D. Lin-
Database Syst Rev, 2015(2): p. CD011549. dor, Cancer surveillance in patients with primary
sclerosing cholangitis. Hepatology, 2011. 54(5): p.
1842-52.
Question 12 Williamson, K.D. and R.W. Chapman, Primary
You are evaluating a 77-year-old man for obstruc- sclerosing cholangitis: a clinical update. Br Med
tive jaundice and weight loss. The patient reports Bull, 2015. 114(1): p. 53-64.
an approximate 25-pound weight loss over the
last one month. He denies abdominal pain. Labs
reveal a total bilirubin of 17.5 mg/dL, alkaline Question 13
phosphatase of 441 IU/L, AST 60 IU/L, ALT 70 A 42-year-old woman with a history of a prior
IU/L, and lipase of 41 U (ULN 50 U) and WBC cholecystectomy presents to your clinic for further
of 8 10^9/L. A right upper quadrant ultrasound evaluation of episodic RUQ abdominal pain which
is obtained and shows intra- and extra- hepatic lasts 30 min to one hour and occurs every month.
biliary dilation up to two centimeters. Subsequent She has also had an intermittent rise in her AST
pancreas protocol CT is notable for narrowing of and ALT two times the upper limit of normal on
the mid bile duct with a normal downstream CBD. two separate occasions over the last six months.
A mass is not visualized within the pancreas. CA Imaging including ultrasound and MRCP has been
19-9 is elevated to 1900 U/mL and CEA is 8 ng/ performed and has shown a four mm bile duct
mL. You are concerned for a possible extrahepatic without a filling defect. Her symptoms are felt to
cholangiocarcinoma. Which of these is not a risk be consistent with functional biliary sphincter of
factor for cholangiocarcinoma? Oddi disorder (type II sphincter of Oddi dysfunc-
tion). She undergoes an ERCP with manometry.
A. PSC Her manometry does not show basal biliary
B. Type IV choledochal cyst sphincter pressure measurements greater than 40
C. Opisthorchis viverrini infection mm Hg (the upper limit of normal for basal biliary
D. Obesity sphincter pressure is 40 mm Hg). Based upon the
E. Cirrhosis manometry results you should:
F. Fasciola hepatica infection
A. Perform a biliary sphincterotomy
CORRECT ANSWER: F B. Perform a pancreatic sphincterotomy
C. Perform both a biliary and a pancreatic
RATIONALE sphincterotomy
There are a number of known risk factors for D. Place a biliary stent without sphincterotomy
cholangiocarcinoma including PSC, choledochal E. Not perform a sphincterotomy
cysts, obesity, chronic liver disease, toxins such as
Thorotrast as well as liver flukes including those CORRECT ANSWER: E
90 Digestive Diseases Self-Education Program®

RATIONALE factors for gallstone formation except:


This patient does fulfill criteria for functional
biliary sphincter of Oddi disorder (type II SOD) A. Female sex
and ERCP with manometry is a reasonable ap- B. TPN use
proach. Her manometry is not consistent with a C. Oral contraceptive use
sphincter of Oddi dysfunction and sphincteroto- D. Crohn’s disease
my should not be performed. Two prior random- E. Infliximab use
ized trials have shown benefit with sphincteroto-
my in patients who fulfill criteria for type II SOD CORRECT ANSWER: E
and have positive manometry, but this benefit is
not seen in patients who fulfill criteria but do not RATIONALE
have positive manometry. There is additional When biliary cholesterol homeostasis is dis-
risk associated with sphincterotomy, namely rupted, gallstones are formed. This can occur
bleeding and perforation and, in this scenario, it through a number of mechanisms including an
should not be pursued. increase in hepatic cholesterol secretion, hypo-
motility of the gallbladder, decreased bile salt
REFERENCES secretion, and increased cholesterol absorption.
Cotton, P.B., et al., Effect of endoscopic sphinc- In the patient above, several possible mecha-
terotomy for suspected sphincter of Oddi nisms are at play. An increase in estrogen leads
dysfunction on pain-related disability following to an increase in hepatic synthesis and secre-
cholecystectomy: the EPISOD randomized clini- tion of cholesterol as well as a decrease in bile
cal trial. JAMA, 2014. 311(20): p. 2101-9. salt synthesis. Increased progesterone leads to
Cotton, P.B., et al., Rome IV. Gallbladder gallbladder hypomotility and a longer time in
and Sphincter of Oddi Disorders. Gastroenterol- which the supersaturated bile will remain in the
ogy, 2016. gallbladder, promoting crystallization. Crohn’s
Freeman, M.L., Complications of endoscopic disease and short gut impairs the enterohepatic
retrograde cholangiopancreatography: avoid- circulation of bile salts, leading to a decrease in
ance and management. Gastrointest Endosc Clin bile salt secretion. TPN is believed to contribute
N Am, 2012. 22(3): p. 567-86. to gallstone formation primarily through bili-
Yaghoobi, M., et al., Incidence and predic- ary stasis due to a decrease in enteral stimula-
tors of post-ERCP pancreatitis in patients with tion. While several medications, including oral
suspected sphincter of Oddi dysfunction under- contraceptives, hormone replacement therapy,
going biliary or dual sphincterotomy: results ceftriaxone, fibrates, calcineurin inhibitors and
from the EPISOD prospective multicenter octreotide have been shown to promote gallstone
randomized sham-controlled study. Endoscopy, formation, infliximab is not one of them and
2015. 47(10): p. 884-90. does not place the patient at an increased risk.

Question 14 REFERENCES
A 28-year-old woman with short gut syndrome Lammert, F., et al., Gallstones. Nat Rev Dis
due to multiple small bowel resections for Primers, 2016. 2: p. 16024.
Crohn’s disease is found to have cholelithiasis Quigley, E.M., et al., Hepatobiliary compli-
on a right upper quadrant ultrasound performed cations of total parenteral nutrition. Gastroen-
for abdominal pain. She is TPN dependent due terology, 1993. 104(1): p. 286-301.
to her short gut syndrome. She receives inflix- Stokes, C.S., M. Krawczyk, and F. Lammert,
imab for her Crohn’s disease. The patient also Gallstones: environment, lifestyle and genes. Dig
takes an oral contraceptive. All of these are risk Dis, 2011. 29(2): p. 191-201.
Chapter 4 — Diseases of the biliary tract 91

Question 15 Question 16
A 36-year-old man with sickle cell disease A 40-year-old woman presents to the emergency
presented to the emergency department with department for further evaluation of right upper
abdominal pain. He previously underwent a cho- quadrant pain. She is noted to be febrile to 39C,
lecystectomy. On RUQ US, he is noted to have hypotensive with a BP of 80/40 mmHg, and
a dilated bile duct to 12 mm. His total bilirubin tachycardic with a HR of 120 bpm. Initial labs
is six mg/dL (indirect 4 mg/dL), ALP is 200 reveal a bilirubin of six mg/dL, ALP 250 IU/L,
IU/L, AST 84 IU/L, ALT 90 IU/L. He undergoes AST 500 IU/L, ALT 430 IU/L and lipase of 1500
ERCP. At the time of his ERCP, he is found to U (ULN 50). A right upper quadrant shows a
have choledocholithiasis. A single black stone is dilated bile duct to 11 mm with an eight millime-
removed. This stone is most likely composed of ter hyperechoic focus within the CBD as well as
what percentage of cholesterol: several hyperechoic foci within the gallbladder.
She is stabilized and undergoes ERCP with stone
A. 90-100 percent extraction. She does well post procedure. What
B. 60-80 percent is her risk of a recurrent biliary event if she does
C. 40-50 percent not undergo cholecystectomy?
D. 20-30 percent
A. 0-10 percent
CORRECT ANSWER: D B. 20-40 percent
C. 50-70 percent
RATIONALE D. 80-100 percent
This patient has formed a pigmented stone most
likely secondary to hemolysis due to his underly- CORRECT ANSWER: B
ing sickle cell disease. In this scenario hemolysis
leads to an excess of systemic bilirubin. Uncon- RATIONALE
jugated bilirubin is not water soluble and com- Currently no clear guidelines exist regarding
bines with calcium to form calcium bilirubinate optimal timing of cholecystectomy for choledo-
within the mucin glycoproteins that are secreted cholithiasis. In patients who have gallstone
by the gallbladder mucosa. Gallstones are clas- pancreatitis or choledocholithiasis, recurrent
sified into cholesterol gallstones and pigmented biliary events (biliary colic, recurrent gallstone
gallstones. Cholesterol gallstones contain more pancreatitis, choledocholithiasis) have been
than 50 percent cholesterol by weight. Pig- observed to occur in 18-40 percent patients if
mented gallstones are further subdivided into cholecystectomy is delayed beyond the initial
black and brown stones and contain less than 30 hospitalization. Interestingly, a recent retrospec-
percent cholesterol by weight. tive cohort study found that just over 50 percent
of patients with choledocholithiasis requiring
REFERENCES ERCP underwent cholecystectomy. Given the
Cariati, A., Gallstone Classification in Western clear risk and possibility of loss of follow up,
Countries. Indian J Surg, 2015. 77(Suppl 2): p. early cholecystectomy in surgically fit candidates
376-80. (prior to hospital discharge or within six weeks
Lammert, F., et al., Gallstones. Nat Rev Dis of discharge) should be recommended in this
Primers, 2016. 2: p. 16024. patient population.
Vitek, L. and M.C. Carey, New pathophysi-
ological concepts underlying pathogenesis of REFERENCES
pigment gallstones. Clin Res Hepatol Gastroen- Hernandez, V., et al., Recurrence of acute gall-
terol, 2012. 36(2): p. 122-9. stone pancreatitis and relationship with chole-
92 Digestive Diseases Self-Education Program®

cystectomy or endoscopic sphincterotomy. Am J small as three mm. In patients with polyps that
Gastroenterol, 2004. 99(12): p. 2417-23. are greater than or equal to 10 mm, cholecys-
Huang RJ, Barakat MT, Girotra M, Banerjee tectomy is recommended. If the polyp is smaller
S. Practice Patterns for Cholecystectomy After En- than 10 mm and the patient is felt to have symp-
doscopic Retrograde Cholangiopancreatography toms attributable to the polyp, cholecystectomy
for Patients With Choledocholithiasis. Gastroen- is suggested. If polyps are smaller than 10 mm
terology 2017;153:762-771.e2. without symptoms, ultrasound surveillance is
Reinders, J.S., et al., Early laparoscopic cho- recommended. In this patient, the polyp is larger
lecystectomy improves outcomes after endoscopic than 10 mm and he should undergo a cholecys-
sphincterotomy for choledochocystolithiasis. tectomy. Given the size, surveillance with US is
Gastroenterology, 2010. 138(7): p. 2315-20. not recommended. MRCP or CT is likely to be of
van Baal, M.C., et al., Timing of cholecystec- limited utility in this scenario. The patient does
tomy after mild biliary pancreatitis: a systematic have a gallbladder in place so ERCP for function-
review. Ann Surg, 2012. 255(5): p. 860-6. al biliary sphincter of Oddi disorder should not
be performed, as the EPISOD trial showed that
there is no justification to perform manometry
Question 17 or sphincterotomy in patients with normal labs
A 36-year-old man with recurrent RUQ pain as- and imaging (prior SOD type III patients), which
sociated with meals is seeing you in clinic. Ami- as of Rome IV are considered to have “functional
notransferase levels have been measured with biliary-type pain.”
these episodes and have been normal. He under-
went a RUQ US which did not reveal cholelithia- REFERENCES
sis; however, an 11-mm non-mobile echogenic Chou, S.C., et al., Polypoid lesions of the gallblad-
lesion was noted within the gallbladder wall. You der: analysis of 1204 patients with long-term
recommend: follow-up. Surg Endosc, 2017. 31(7): p. 2776-2782.
McCain, R.S., et al., Current practices and fu-
A. Cholecystectomy ture prospects for the management of gallbladder
B. ERCP with manometry for functional polyps: A topical review. World J Gastroenterol,
biliary sphincter of Oddi disorder 2018. 24(26): p. 2844-2852.
C. CT scan Wiles, R., et al., Management and follow-up of
D. MR/MRCP gallbladder polyps : Joint guidelines between the
E. A repeat US in one year European Society of Gastrointestinal and Abdomi-
nal Radiology (ESGAR), European Association
CORRECT ANSWER: A for Endoscopic Surgery and other Interventional
Techniques (EAES), International Society of Di-
RATIONALE gestive Surgery - European Federation (EFISDS)
Polypoid gallbladder lesions are seen in up to and European Society of Gastrointestinal Endos-
five percent of adults in Western populations. copy (ESGE). Eur Radiol, 2017. 27(9): p. 3856-
Polypoid lesions can be pseudopolyps including 3866.
cholesterol polyps, inflammatory polyps, adeno- Zielinski, M.D., et al., Comparison of surgi-
myoma which are benign or true polyps includ- cally resected polypoid lesions of the gallbladder
ing adenocarcinomas as well as adenomas (which to their pre-operative ultrasound characteristics. J
have malignant potential). A prior series noted Gastrointest Surg, 2009. 13(1): p. 19-25.
that only polyps six mm or larger in size were Cotton P.B., et al. Gallbladder and Sphinc-
found to have malignancy however in a more ter of Oddi Disorders. Gastroenterology
recent series malignancy was noted in polyps as 2016;150:1420–1429.
Chapter 4 — Diseases of the biliary tract 93

Question 18 19(1): p. 2.
A 52-year-old man with known PSC is found to Esnaola, N.F., et al., Evaluation and manage-
have rising aminotransferase levels. He undergoes ment of intrahepatic and extrahepatic cholangio-
an MR/MRCP and is found to have a new irregu- carcinoma. Cancer, 2016. 122(9): p. 1349-69.
lar hilar mass and intrahepatic ductal dilation. His Njei, B., et al., Systematic review with meta-
bilirubin is eight mg/dL, ALP 450 U/L, AST 200 analysis: endoscopic retrograde cholangiopancre-
U/L, ALT 230 IU/L. CA 19-9 is 900 U/mL, CEA atography-based modalities for the diagnosis of
is 8 ng/mL and AFP is four ng/mL. There is no cholangiocarcinoma in primary sclerosing cholan-
evidence of metastatic disease on cross sectional gitis. Aliment Pharmacol Ther, 2016. 44(11-12): p.
imaging of his chest abdomen and pelvis. Which 1139-1151.
of these should be avoided in this patient?

A. Percutaneous biopsy of the hilar mass Question 19


B. EUS for fine needle biopsy of any abnormal A 28-year-old female refugee from Swaziland
appearing lymph nodes presents to your clinic for further evaluation of
C. Surgical laparoscopy abdominal pain and loose stools. The patient
D. ERCP with cholangioscopy reports recurrent epigastric pain as well as three
E. Referral to a tertiary care center for to four daily non-bloody loose stools. On exam she
consideration of transplantation is cachectic, and her abdomen is not tender to pal-
pation. You note several flat painless dark brown
CORRECT ANSWER: A lesions on her skin. She reports she has only noted
the skin lesions over the last one month. A RUQ
RATIONALE ultrasound is obtained and intra and extrahepatic
While either percutaneous or EUS guided FNB of ductal dilation is noted. No cholelithiasis is seen
the mass are likely to yield a tissue diagnosis, they and flow appears to be normal within the liver.
do carry with them a risk of seeding, and would Labs studies are notable for an ALP of 743 IU/L, a
preclude possible liver transplantation should the bilirubin of 1.0 mg/dL, AST 68 IU/L, and ALT 78
mass lesion be found to be a cholangiocarcinoma. IU/L. An ERCP is obtained and papillary stenosis
While a mass lesion should not be sampled, if an is noted with no irregularity or stricture within the
abnormal lymph node is visualized fine needle remainder of the bile duct or intrahepatic ducts. A
biopsy is not contraindicated. In this scenario, the sphincterotomy is performed with prompt drain-
concern for cholangiocarcinoma is high and surgi- age of contrast and no evidence of a distal stric-
cal laparoscopy for staging and possible resection ture. Brushings are obtained and are unrevealing.
is not unreasonable, provided the anatomy is felt CT of the abdomen is performed and is notable
to be conducive to surgical intervention and the only for a mild dilation of her bile duct to 11 mm
patient is a good surgical candidate. ERCP with without a pancreas head mass. What is the best
cholangioscopy and stenting is an alternative to al- test to send in order to determine the etiology for
low for a tissue diagnosis as well as biliary decom- the patient’s papillary stenosis?
pression. Referral to a tertiary care center that is
performing liver transplantation for cholangiocar- A. HIV testing
cinoma is also a reasonable option. B. IgG4 levels
C. AMA
REFERENCES D. CEA
Doherty, B., V.E. Nambudiri, and W.C. Palmer, E. CA 19-9
Update on the Diagnosis and Treatment of Chol-
angiocarcinoma. Curr Gastroenterol Rep, 2017. CORRECT ANSWER: A
94 Digestive Diseases Self-Education Program®

RATIONALE
This patient presentation is most consistent with
HIV cholangiopathy. This condition has become
rare in the United States following initiation of
HARRT therapy. Approximately 80 percent of
cases are in patients with advanced disease (CD4
less than 100). Almost all patients have a marked
increase in ALP with only mild increase in AST,
ALT and bilirubin. The most common presenta-
tion is with papillary stenosis. In the case of our
patient, she is a refugee from a country with a rate
of HIV infection of approximately 20-30 percent,
placing her at an increased risk for underlying
HIV/AIDS. The new rash consistent with Kaposi
sarcoma provides further support for underlying A. Cholangiocarcinoma
AIDS. The patient does not have a pancreas head B. HCC
mass and it is unlikely given the findings on her C. Pancreatic adenocarcinoma
ERCP that there is a primary cholangiocarcinoma, D. Gallbladder cancer
making CEA and CA 19-9 unlikely to point to an E. Colon Cancer
underlying malignancy. AMA is associated with
PBC; however, this presentation is not consistent CORRECT ANSWER: D
with PBC. Autoimmune cholangiopathy is a con-
sideration as well but, given the patient’s young RATIONALE
age and female sex and isolated papillary stenosis, The MRCP shows a pancreaticobiliary maljunc-
it is unlikely that this is an underlying autoim- tion. These patients are at an increased risk of
mune condition. developing gallbladder cancer, with one series
report cancer arising in 15 percent of patients with
REFERENCES pancreaticobiliary maljunction and congenital bili-
Devarbhavi, H., et al., HIV/AIDS cholangiopathy: ary dilation and in 36 percent without bile duct
clinical spectrum, cholangiographic features and dilation. There is also an increased risk of devel-
outcome in 30 patients. J Gastroenterol Hepatol, oping cholangiocarcinoma, in seven percent with
2010. 25(10): p. 1656-60. congenital biliary dilation and 3.1 percent without
Tonolini, M. and R. Bianco, HIV-related/AIDS bile duct dilation. More recent studies report a
cholangiopathy: pictorial review with emphasis nine percent rate of pancreatic cancer in patients
on MRCP findings and differential diagnosis. Clin with maljunction without bile duct dilation. It is
Imaging, 2013. 37(2): p. 219-26. believed that reflux of pancreatic juice into the
biliary tree is responsible for the increased risk for
gallbladder and bile duct cancers. In patients with
Question 20 congenital biliary dilation, resection of the extra-
A 21-year-old woman presents for further evalua- hepatic duct and gallbladder is recommended. The
tion of intermittent abdominal pain. On exam, she management of patients without bile duct dilation
is tender to palpation without rebound or guard- is more controversial. While cholecystectomy is
ing. AST is 78 IU/L, ALT 90 IU/L, ALP 158 IU/L, recommended in these patients, there is disagree-
Bilirubin 1.5 mg/dL. An MRCP is performed and is ment regarding the need for extrahepatic ductal
shown here. This patient is most likely to develop resection. These patients are not at an increased
which malignancy? risk for colon cancer or HCC.
Chapter 4 — Diseases of the biliary tract 95

REFERENCES A. Cholangiocarcinoma
Funabiki, T., et al., Pancreaticobiliary maljunction B. Pancreatic adenocarcinoma
and carcinogenesis to biliary and pancreatic ma- C. An ampullary adenoma
lignancy. Langenbecks Arch Surg, 2009. 394(1): p. D. IgG4 disease
159-69. E. PSC
Kamisawa, T., et al., Diagnostic criteria for F. Chronic pancreatitis
pancreaticobiliary maljunction 2013. J Hepatobili-
ary Pancreat Sci, 2014. 21(3): p. 159-61. CORRECT ANSWER: D
Kamisawa, T., et al., Biliary carcinogenesis in
pancreaticobiliary maljunction. J Gastroenterol, RATIONALE
2017. 52(2): p. 158-163. This is a young woman with recurrent pancreatitis
Ohuchida, J., et al., Long-term results of treat- of unclear etiology and a new distal biliary stricture.
ment for pancreaticobiliary maljunction without The MR shows findings that are consistent with au-
bile duct dilatation. Arch Surg, 2006. 141(11): p. toimmune pancreatitis (a sausage shaped pancreas
1066-70. with capsular enhancement). In this setting, the
Rungsakulkij, N. and P. Boonsakan, Synchro- stricture is likely related to underlying IgG4 disease.
nous gallbladder and pancreatic cancer associ- It is unclear in this imaging if the stricture is caused
ated with pancreaticobiliary maljunction. World J by pancreatic head swelling from her pancreatitis
Gastroenterol, 2014. 20(39): p. 14500-4. or if IgG4 cholangiopathy is most likely responsible.
In a young patient, PSC and HIV cholangiopathy
are other considerations. While PSC can present as
Question 21 isolated extra hepatic duct stricturing, it is uncom-
An 18-year-old woman presents for further mon and, in the setting of imaging suggesting
evaluation of jaundice. Her past medical his- autoimmune pancreatitis, it is less likely. She has no
tory is notable for 2 prior episodes of idiopathic risk factors for cholangiocarcinoma and, given her
pancreatitis. Her AST is 50, ALT 60, ALP 140, age, this is not felt to be a likely etiology. The same
bilirubin 4 and lipase is 150. On exam her sclerae applies for pancreatic adenocarcinoma and, ad-
are icteric. She is not tender to palpation over ditionally, no pancreas head mass is seen. Changes
her abdomen. A RUQ ultrasound is obtained and of chronic pancreatitis would be unlikely after two
extra and intrahepatic ductal dilation are noted. episodes of pancreatitis, and chronic pancreatitis is
A MR/MRCP is pursued and shows abrupt taper- not felt to be the most likely etiology.
ing of the CBD within the pancreas head, as well
as delayed pancreatic enhancement with a thin REFERENCES
rim of peripheral enhancement as seen here. Her Geary, K., et al., IgG4-Related Cholangiopathy and
CBD stricture is most likely secondary to which Its Mimickers: A Case Report and Review High-
underlying condition? lighting the Importance of Early Diagnosis. Int J
Surg Pathol, 2018. 26(2): p. 165-173.
Joshi, D. and G.J. Webster, Biliary and hepat-
ic involvement in IgG4-related disease. Aliment
Pharmacol Ther, 2014. 40(11-12): p. 1251-61.
Okazaki, K., et al., IgG4 cholangiopathy:
current concept, diagnosis, and pathogenesis. J
Hepatol, 2014. 61(3): p. 690-5.
Watanabe, T., et al., Mechanisms of lower bile
duct stricture in autoimmune pancreatitis. Pan-
creas, 2014. 43(2): p. 255-60.
96 Digestive Diseases Self-Education Program®

Question 22 pain medications for this indication would not be


A 65-year-old man with known pancreatic head the correct answer. Endoscopic ultrasound is an
adenocarcinoma presents with abdominal pain invasive procedure and is no more likely deter-
following stent placement for obstructive jaun- mine the etiology of the patient’s symptoms when
dice two days prior. Prior to the procedure, the compared to US. Surgical consultation is prema-
patient was pain free. On exam, he is tender ture without a clear diagnosis.
to palpation in the RUQ and epigastrium with
guarding but no rebound. Laboratory studies REFERENCES
show a bilirubin of five mg/dL down from 10 mg/ Ho, H., et al., Management of complications asso-
dL, AST 88 IU/L, ALT 90 IU/L, and ALP 180 ciated with partially covered biliary metal stents.
IU/L, lipase of 50 U, and WBC 14.1 10^9/L. An Dig Dis Sci, 2010. 55(2): p. 516-22.
abdominal x-ray shows the stent in position as Isayama, H., et al., Cholecystitis after metallic
well as an air cholangiogram. stent placement in patients with malignant distal
biliary obstruction. Clin Gastroenterol Hepatol,
Your next step in management of this patient is: 2006. 4(9): p. 1148-53.
Jaganmohan, S. and J.H. Lee, Self-expandable
A. ERCP metal stents in malignant biliary obstruction.
B. IVFs and pain medications for post ERCP Expert Rev Gastroenterol Hepatol, 2012. 6(1): p.
pancreatitis 105-14.
C. Right upper quadrant US Nakai, Y., et al., Metallic stent with high axial
D. EUS force as a risk factor for cholecystitis in distal
E. Surgical consultation malignant biliary obstruction. J Gastroenterol
Hepatol, 2014. 29(7): p. 1557-62.
CORRECT ANSWER: C Suk, K.T., et al., Risk factors for cholecystitis
after metal stent placement in malignant biliary
RATIONALE obstruction. Gastrointest Endosc, 2006. 64(4): p.
This patient likely has cholecystitis following 522-9.
metal stent placement. In several series, this Yang, M.J., et al., Partially covered versus
has been shown to occur in three to 10 percent uncovered self-expandable nitinol stents with
of patients, which is higher than the rate of post anti-migration properties for the palliation of
ERCP pancreatitis, which is around one to two malignant distal biliary obstruction: A random-
percent. Tumor involvement of the cystic orifice ized controlled trial. Scand J Gastroenterol, 2015.
is associated with the development of chole- 50(12): p. 1490-9.
cystitis post metal stent placement. There does
not appear to be a clear association between the
type of metal stent and cholecystitis. The correct Question 23
answer is further imaging, in this case right up- A 45-year-old woman with no significant
per quadrant ultrasound, which will likely show past medical history presents to your office
pericholecystic fluid and a thickened gallblad- for further management of right upper quad-
der wall. Cross sectional imaging with a CT scan rant pain over the last one year. The patient
could be considered as an alternative to an US. reports intermittent episodes of severe 9/10
There is no indication that the stent has migrated RUQ pain every two to three weeks. The pain
or that it is obstructed based upon his labora- typically lasts one to two hours and resolves
tory studies and x- ray, so ERCP at this time is spontaneously. She has no improvement with
premature. Based upon his lipase of 50, he does defecation. On exam her abdomen is soft not
not have post ERCP pancreatitis and IVFs and tender or distended. Labs obtained during one
Chapter 4 — Diseases of the biliary tract 97

of her pain episodes show an AST of 14 IU/L, REFERENCES


ALT 24 IU/L, Bili 0.8 mg/dL, ALP 92 IU/L, Behar, J., et al., Functional gallbladder and
and lipase of 32 U. She has undergone RUQ sphincter of oddi disorders. Gastroenterology,
ultrasound without stones or sludge. There 2006. 130(5): p. 1498-509.
is no pericholecystic fluid or wall thickening. Cotton, P.B., et al., Rome IV. Gallbladder and
Endoscopic ultrasound was performed, and Sphincter of Oddi Disorders. Gastroenterology, 2016.
a four millimeter CBD was visualized with no Gurusamy, K.S., et al., Cholecystectomy for
microlithiasis within the CBD or gallbladder. suspected gallbladder dyskinesia. Cochrane Data-
The ampulla was normal in appearance as well base Syst Rev, 2009(1): p. CD007086.
as the remainder of her upper GI tract. CCK-
stimulated cholescintigraphy is obtained and
demonstrates a GBEF of 22 percent. Question 24
An 89-year-old woman presents with black stools
This patient most likely has: as well as RUQ pain radiating to her shoulder as
well as nausea and vomiting. She denies any prior
A. Functional Biliary Sphincter Disorder episodes of abdominal pain. On exam, her abdo-
B. Functional Pancreatic Sphincter Disorder men is distended and tender to palpation. Labs
C. Functional Gallbladder Disorder are notable for a WBC of 18.2, Hgb 10 gm/dl,
D. Acalculous cholecystitis previously 12, ALT 147 IU/L, AST 376 IU/L, Tbili
E. Functional dyspepsia 2.8 mg/dl. CT imaging reveals a markedly dilated
stomach without small bowel dilation. The gall-
CORRECT ANSWER: C bladder is heterogenous in appearance with sev-
eral calcified stones. The GB wall is thickened and
RATIONALE appears inseparable from the adjacent duodenum.
This patient fulfills criteria for a functional gall- Air is seen within the gallbladder lumen. The CBD
bladder disorder. She has biliary type pain along is not dilated. She has no prior history of biliary
with an absence of a structural etiology for her instrumentation. The next best step in manage-
pain. The diagnosis is further supported by low ment of this condition is:
EF seen on cholescintigraphy as well as normal
LFTs and lipase. Functional gallbladder disorder A. Surgical consultation with consideration of
is a common indication for cholecystectomy oc- therapeutic upper endoscopy
curring in approximately 10-20 percent of adults B. ERCP
who undergo cholecystectomy. The role of biliary C. EUS
and/or pancreatic sphincter dysfunction in pa- D. Percutaneous drain placement
tients with an intact gallbladder is controversial, E. NG tube placement
as the intact GB is felt to serve a pressure offset.
Current criteria require a prior cholecystectomy CORRECT ANSWER: A
to diagnosis a functional biliary or pancreatic
disorder. In addition, the patient does not have RATIONALE
abnormalities in her LFTs, lipase or ductal dila- This patient has gastric outlet obstruction second-
tion. The patient does not have sludge, perichole- ary to a gallstone that has formed a cholecystodu-
cystic fluid or gallbladder wall thickening, elimi- odenal fistula or Bouveret’s syndrome. Bouveret’s
nating acalculous cholecystitis. The patient does is a rare cause of obstruction of the GI tract by
not report postprandial fullness early satiety or gallstones representing approximately two percent
epigastric burning or pain to suggest dyspepsia as of all gallstone ileus cases. It should be suspected
a likely etiology. in patients with gastric outlet obstruction and
98 Digestive Diseases Self-Education Program®

unexplained pneumobilia. The therapeutic ap-


proach should start with an endoscopy; however,
this is only successful approximately nine percent
of the time. Surgery is often required for defini-
tive management and it is reasonable to have
them involved upfront given the low success rate
endoscopically. It should be noted that surgical
intervention carries an approximately 12 percent
mortality rate, likely due to the advanced age
and the medical comorbidities of these patients.
In this scenario, the stone has eroded into the
duodenal bulb and neither EUS or ERCP will be of
benefit. NG tube placement will allow for decom-
pression of the stomach, but it will not resolve the
obstruction. Percutaneous drain placement would
not be of benefit in this scenario.

REFERENCES
Baharith, H. and K. Khan, Bouveret syndrome:
When there are no options. Can J Gastroenterol
Hepatol, 2015. 29(1): p. 17-8. A. Cholangitis
Cappell, M.S. and M. Davis, Characterization B. Pancreatitis
of Bouveret’s syndrome: a comprehensive review C. Obstructive jaundice
of 128 cases. Am J Gastroenterol, 2006. 101(9): p. D. Diarrhea
2139-46. E. GI bleeding
Haddad, F.G., W. Mansour, and L. Deeb,
Bouveret’s Syndrome: Literature Review. Cureus, CORRECT ANSWER: B
2018. 10(3): p. e2299.
Nickel, F., et al., Bouveret’s syndrome: pre- RATIONALE
sentation of two cases with review of the literature This patient has a choledochocele. While many
and development of a surgical treatment strategy. are asymptomatic, the most common clinical
BMC Surg, 2013. 13: p. 33. presentation is pancreatitis, which is seen in
38-70 percent of patients with a choledochocele.
Jaundice is seen in 11-25 percent of patients and
Question 25 cholangitis is seen in 0-10 percent of patients.
A 68-year-old man presents to your clinic for fur- The most common presentation of choledochal
ther evaluation of abdominal pain. On exam, the cysts type I, II, IV, V is cholangitis, followed by
patient’s abdomen is soft, not tender and not dis- obstructive jaundice, then pancreatitis. Diarrhea
tended. You note icteric sclerae. Labs are obtained and GI bleeding are not typical presentations of
and demonstrate an AST of 92 IU/L, ALT 87 IU/L, choledochal cysts. This is the only type of chole-
Bili 3 mg/dL, ALP 180 IU/L, and lipase is 23 U/L. dochal cyst for which ERCP can offer definitive
A RUQ ultrasound demonstrates ductal dilation management. The risk of malignancy is very low,
but no cholelithiasis or sludge. MRCP is subse- especially when compared to other choledochal
quently performed and is shown here. ERCP is cysts, and the need for surveillance following en-
performed with the image shown above. The most doscopic management (snare resection or sphinc-
common clinical presentation of this condition is: terotomy) remains controversial.
Chapter 4 — Diseases of the biliary tract 99

REFERENCES (CCA). The sensitivity of conventional cytology


Culetto, A., et al., Endoscopic treatment of a choled- in diagnosis of malignant biliary strictures ranges
ochocele. Endoscopy, 2016. 48 Suppl 1: p. E286. from 15-38 percent. Supplementation with FISH
Law, R. and M. Topazian, Diagnosis and studies increases the sensitivity of routine cytology
treatment of choledochoceles. Clin Gastroenterol in the diagnosis of malignancy. Detection of FISH
Hepatol, 2014. 12(2): p. 196-203. polysomy without other definitive evidence of
malignancy requires further evaluation with ERCP
in three to four months. Polysomy on serial exams
Question 26 in patients with primary sclerosing cholangitis
A 54-year-old woman is referred for evaluation a (PSC) is associated with an increased risk of CCA
biliary stricture. Her medical history is significant compared with patients with PSC who do not have
for hypertension, irritable bowel syndrome, and serial polysomy (69 vs. 18 percent).
primary sclerosing cholangitis. Her laboratory
evaluation is as follows: Percutaneous or endoscopic sampling of a suspi-
cious biliary stricture is not recommended due to
AST 239 U/L risk of peritoneal seeding. While MRCP is a useful
ALT 473 U/L tool in surveillance for hepatobiliary cancers in
Alkaline phosphatase 760 U/L patients with PSC, the presence of FISH poly-
Total bilirubin 9.0 mg/dL somy requires further cytologic assessment of the
CA 19-9 22 U/mL dominant stricture. In patients with PSC, CA 19-9
(with a cutoff value of 129 U/mL) has a sensitivity
MRCP reveals a 1.4 cm regenerative nodule in the and specificity of 79 and 98 percent in the diagno-
right lobe of the liver and a hilar stricture. ERCP sis of CCA. Approximately one third of patients
is performed with dilation and brushing of the with sustained elevation in CA 19-9 do not de-
stricture. Cytology reveals atypical cells, and FISH velop CCA however. In addition, approximately 7
studies are consistent with polysomy. percent of the population are unable to express CA
19-9 due to lack of activity of fucosyltransferase 2
Which of the following is the next best step in this and 3. In the absence of a mass, ultrasound may
patient’s management? show intrahepatic ductal dilation but lacks sen-
sitivity and specificity in diagnosing CCA. Liver
A. Referral for surgical resection. transplantation preceded by neoadjuvant chemo-
B. Repeat MRI/MRCP in 6 months. radiation in selective candidates is the recom-
C. EUS-guided biopsy of stricture mended approach towards curative therapy rather
D. ERCP with biliary brushings and than surgical resection.
cytology in three months.
E. Ultrasound and CA 19-9 in three months. REFERENCES
Rizvi S, Gores GJ. Current diagnostic and man-
agement options in perihilar cholangiocarcinoma.
CORRECT ANSWER: D Digestion. 2014;89:216-224.
Barr Fritcher EG, Kipp BR, Slezak JK, et al.
RATIONALE Correlating routine cytology, quantitiative nuclear
This patient’s evaluation reveals a stricture of the morphometry by digital image analygis, and
common bile duct and polysomy on FISH stud- genetic alterations by fluorescence in situ hybrid-
ies. Although cytology is negative, polysomy on ization to assess the sensitivity of cytology for
FISH studies in the setting of a dominant stricture detecting pancreatobiliary tract malignancy. Am J
is concerning for perihilar cholangiocarcinoma Clin Pathol 2007;128:272-279.
100 Digestive Diseases Self-Education Program®

Barr Fritcher EG, Kipp BR, Voss JS, et al. CORRECT ANSWER: B
Primary sclerosing cholangitis patients with serial
polysomy flouresence in situ hybridization results RATIONALE
are at increased risk of cholangiocarcinoma. Am J This patient has an early stage intrahepatic CCA
Gastroenterol 2011;106:2023-2028. (iCCA) arising in the setting of cirrhosis second-
ary to nonalcoholic steatohepatitis. While surgical
resection offers potentially curative therapy for
Question 27 iCCA, the presence of cirrhosis and portal hyper-
A 48-year-old man presents for routine evalu- tension make the patient high risk for hepatic
ation of cirrhosis secondary to nonalcoholic resection. While iCCA was conventionally con-
steatohepatitis. He has a history of non-bleeding sidered a contraindication for liver transplanta-
varices and medically controlled ascites. His tion due to high risk of recurrence, recent stud-
medical history is otherwise significant for obe- ies demonstrate good survival in patients with
sity with a BMI of 31. His laboratory evaluation cirrhosis and highly selective patients with very
is as follows: early iCCA. A retrospective, multi-center study
revealed a low five-year recurrence of iCCA among
AST 148 U/L patients with lesions smaller than two centime-
ALT 213 U/L ters (12 percent compared to 77 percent among
Alkaline phosphatase 391 U/L patients with lesions larger than two centimeters
Total bilirubin 3.3 mg/dL or multifocal disease). Overall survival among
CA 19-9 444 U/mL patients with lesions smaller than two centime-
Alpha feto-protein 2.8 ng/mL ters was 65 percent. Thus, patients with cirrhosis
Hemoglobin 12.5 g/dL and iCCA who otherwise qualify as candidates for
Platelets 91 x 109/L liver transplantation can be considered for trans-
Sodium 140 mmol/L plantation at specialized transplant centers if they
INR 1.3 have a single lesion less than or equal to three
Albumin 3.0 g/dL centimeters in diameter, no evidence of vascular
Creatinine 1.1 mg/dL invasion or extrahepatic disease, and no history of
prior attempted resection which might violate the
Ultrasound reveals a nodule in the left lobe of tumor plane.
the liver. CT scan is done to further evaluate this Locoregional therapies such as TACE and
lesion and reveals a 1.8 cm heterogeneous mass radiofrequency ablation are a reasonable approach
with portal venous enhancement in segment two in patients with more advanced iCCA who are
without vascular invasion. Ultrasound guided not candidates for resection or transplantation.
biopsy confirms well differentiated adenocarci- While waiting for transplantation, local regional
noma. Chest CT and endoscopic ultrasound with therapies may be employed as a bridge to trans-
perihilar lymph node sampling is negative for plantation. Gemcitabine and cisplatin constitute
malignancy. What is the next best step in manage- first line chemotherapy in patients with advanced
ment of this patient? stage CCA who are not candidates for locoregional
therapy or surgery.
A. Refer to oncology for gemcitabine and
cisplatin. REFERENCES
B. Refer to liver transplantation center Rizvi S, Khan SA, Hallemeier CL, et al. Chol-
C. Refer for surgical resection angiocarcinoma- evolving concepts and thera-
D. Refer for transarterial chemoembolization peutic strategies. Nat Rev Clin Oncol. 2018
E. Refer for radiofrequency ablation. Feb;15(2):95-111.
Chapter 4 — Diseases of the biliary tract 101

Li YY, Li H, Lv P, et al. Prognostic value of cir- over time have been identified as risk factors for
rhosis for intrahepatic cholangiocarcinoma after malignancy. Although this patient is asymptom-
surgical treatment. J Gastrointest Surg. 2011 Apr; atic from her gallbladder polyp and gallstones,
15(4): 608-613. laparoscopy cholecystectomy is recommended for
Sapisochin G, Rodriguez de Lope C, Gastaca polyps larger than one centimeter. Ultrasound
M, et al. “Very early” intrahepatic cholangiocar- surveillance is recommended for gallbladder pol-
cinoma in cirrhotic patients: should liver trans- yps without high-risk features.
plantation be reconsidered in these patients? Am J Oral dissolution therapy with ursodeoxycholic
Transplant. 2014 Mar;14(3):660-7. acid can be used in a select subset of patients with
Sapisochin G, Rodriguez de Lope C, Gastaca cholelithiasis. Studies suggest that this is most
M, et al. Intrahepatic cholangiocarcinoma or effective in patients with gallstones < 1cm in diam-
mixed hepatocellular-cholangiocarcinoma in pa- eter however.
tients undergoing liver transplantation: a Spanish
matched cohort multicenter study. Ann Surg. 2014 REFERENCES
May;259(5):944-52. Gallahan WC, Conway JD. Diagnosis and manage-
ment of gallbladder polyps. Gastroenterol Clin
North Am. 2010;39:359-367.
Question 28 Terzi C, Sokmen S, Seckin S, et al. Polypoid
A 58-year-old woman presents for evaluation of lesions of the gallbladder: report of 100 cases with
a gallbladder lesion which was noted incidentally special reference to operative indications. Surgery.
on a CT scan done to evaluate kidney stones. An 2000 Jun;127(6):622-7.
ultrasound is recommended and reveals choleli- Myers RP, Shaffer EA, Beck PL. Gallbladder
thiasis and a hyperechoic polyp which measures polyps: epidemiology, natural history and man-
12 mm in diameter. Her bile ducts appear nor- agement. Can J Gastroenterol. 2002,16:187-94.
mal. She is asymptomatic and denies abdominal Cairns V, Neal CP, Dennison AR, et al. Risk
discomfort. What is the next best step in man- and cost-effectiveness of surveillance followed by
agement? cholecystectomy for gallbladder polyps. Arch Surg.
2012:1-6.
A. Recommend laparoscopic cholecystectomy. Tomida S, Abei M, Yamaguchi T, et al. Long-
B. No follow up needed. term ursodeoxycholic acid therapy is associated
C. Recommend laparoscopic cholecystectomy if with reduced risk of biliary pain and acute cho-
she develops symptoms suggestive of biliary lecystitis in patients with gallbladder stones: a
colic. cohort analysis. Hepatology. 1999;30(1):6.
D. Ultrasound in six months.
E. Ursodeoxycholic acid 13-15 mg/kg/day.
Question 29
A 35 year-old-woman presents to the clinic for
CORRECT ANSWER: A
evaluation of elevated liver tests. Her liver tests
were checked as part of her insurance policy one
RATIONALE
year ago and they have subsequently remained
Gallbladder polyps are common and can be benign abnormal. Her medical history is otherwise sig-
or neoplastic. Polyp size is an important factor in nificant for ulcerative colitis which was diagnosed
determining risk of malignancy. Age greater than five years ago. Her last colonoscopy was done one
60, coexisting gallstones, lower polyp number, year ago and revealed mild chronic colitis. Biop-
polyp size more than one centimeter, primary scle- sies had no evidence of dysplasia. Her laboratory
rosing cholangitis and evidence of polyp growth tests are as follows:
102 Digestive Diseases Self-Education Program®

Total bilirubin 1.2 mg/dL is a concern for an overlap syndrome with autoim-
Alkaline phosphatase 360 U/L mune hepatitis. Anti-mitochondrial antibodies are
ALT 45 U/L useful to establish a diagnosis of primary biliary
AST 62 U/L cholangitis. While PSC patients are at an increased
risk of gallbladder cancer, a prophylactic cholecys-
MRCP is performed and the results are shown
tectomy would not be indicated.
below:

REFERENCES
Thackeray EW, Charatcharoenwitthaya P, Elfaki
D, et al. Colon neoplasms develop early in the
course of inflammatory bowel disease and primary
sclerosing cholangitis. Clin Gastroenterol Hepatol.
2011;9:52-56.
Chapman R, Fevery J, Kalloo A, et al. Diagno-
sis and management of primary sclerosing cholan-
gitis. Hepatology 2010;51:660-678.

Question 30
A 40-year-old man presents to the emergency
What is the next best step in management?
room with abdominal pain, recurrent fevers, and
worsening jaundice. He has a past medical history
A. Liver biopsy
significant for recurrent episodes of cholangitis,
B. Check anti-mitochondrial antibody titers
ascites, and esophageal varices. A CT scan (below)
C. Cholecystectomy
is performed in the emergency room. MRCP per-
D. Colonoscopy with surveillance biopsies in
formed several weeks earlier is shown below:
4 years
E. Colonoscopy with surveillance biopsies now

CORRECT ANSWER: E

RATIONALE
The above patient has asymptomatic large-duct
primary sclerosing cholangitis (PSC). The diag-
nosis is secured with the cholangiogram, which
shows typical features of PSC, consisting of mul-
tifocal segmental biliary strictures and dilations.
Up to 80 percent of PSC patients have underlying
inflammatory bowel disease, particularly ulcer-
ative colitis (UC). Individuals with PSC-UC are
four times more likely to develop colorectal cancer
compared to those with UC alone. Therefore,
annual colonoscopy with surveillance biopsies is
recommended for patients with PSC-UC.
A liver biopsy is not necessary to confirm a
diagnosis of PSC unless the cholangiogram is
normal and small-duct PSC is suspected, or there
Chapter 4 — Diseases of the biliary tract 103

Which of the following is true regarding this While Caroli’s syndrome predisposes to cholangi-
patient’s diagnosis? tis, the pathogenesis of this disorder is thought to
be due to genetic mutation in ciliary proteins and
A. Autosomal dominant polycystic kidney not due to parasitic infection.
disease is the most common renal
manifestation. REFERENCES
B. Cyst aspiration and sclerotherapy are Ozlem Y, Bayraktar Y. Clinical characteristics
indicated. of Caroli’s syndrome. World J Gastroenterol.
C. This patient is at increased risk for 2007;13:1934-1937.
cholangiocarcinoma.
D. Liver biopsy is required to establish the
diagnosis. Question 31
E. This patient’s liver disease is secondary A 49-year-old man presents to transplant clinic for
to parasitic infection. evaluation of fevers and jaundice. He underwent
an orthotopic liver transplant two months ago for
CORRECT ANSWER: C decompensated cirrhosis secondary to primary
sclerosing cholangitis (PSC). His post-operative
RATIONALE course was notable for mild elevations in his al-
This patient has Caroli’s syndrome which is a kaline phosphatase, AST, and ALT. A liver biopsy
congenital disorder characterized by multifocal, done one month post-transplant did not show
segmental dilation of large intrahepatic bile ducts rejection. Laboratory tests reveal the following:
associated with congenital hepatic fibrosis. The
diagnosis of Caroli’s syndrome is established by WBC count 14,000/L
imaging studies. This patient’s cholangiogram Total bilirubin 6.2 mg/dL
illustrates characteristic features of Caroli’s syn- Direct bilirubin 5.2 mg/dL
drome including diffuse dilation of the biliary tree Alkaline phosphatase 450 U/L
with extensive fusiform and saccular dilation of the AST 100 U/L
intrahepatic bile ducts. Other characteristic radio- ALT 108 U/L
graphic features (indicated by arrowheads on CT ERCP reveals multiple intrahepatic strictures.
scan) include the “central dot sign” where intralu-
What is the next best step in management of this
minal portal vein radicals appear as foci of contrast
patient?
enhancement within the dilated bile ducts.
Biliary cysts in Caroli’s syndrome are irregu-
A. Refer for retransplantation.
larly shaped and communicate with the bile ducts.
B. Increase immunosuppression.
In contrast, cysts in polycystic liver disease are
C. Hepatitis C viral load.
more regularly shaped and do not communicate
D. Repeat liver biopsy to rule out PSC.
with bile ducts.
E. Doppler ultrasound of the hepatic arteries.
Renal manifestations occur in up to 60
percent of patients with Caroli’s syndrome and
include medullary sponge kidney, cortical cysts, CORRECT ANSWER: E
autosomal-recessive polycystic kidney disease, or
very rarely, autosomal-dominant polycystic kidney RATIONALE
disease. Patients with Caroli’s syndrome are at Biliary tract complications are among the most
increased risk for cholangiocarcinoma which is common post-transplant complications and can
thought to be due to bile stasis and high concen- occur in five to 15 percent of patients following a
trations of unconjugated secondary bile salts. deceased-donor transplant and up to 32 percent in
104 Digestive Diseases Self-Education Program®

living-donor transplantation. Post-transplant bili- significant for well-controlled hypothyroidism and


ary strictures can be categorized as either anasto- hyperlipidemia. Her only medications are levothy-
motic or non-anastomotic strictures. roxine and simvastatin. Her physical exam reveals
scattered excoriations on her extremities and
Non-anastomotic strictures typically develop at chest. Her laboratory tests are as follows:
the hilum and/or the intrahepatic ducts. This can
be secondary to ischemic injury (examples, he- Total bilirubin 1.8 mg/dL
patic artery thrombosis or stenosis, or prolonged Direct bilirubin 0.9 mg/dL
ischemia time) or immune-mediated processes Alkaline phosphatase 520 U/L
(examples, ABO-incompatible graft or chronic ALT 74 U/L
ductopenic rejection). Hepatic artery complica- AST 62 U/L
tions, such as stenosis or thrombosis, constitute Albumin 4.5 g/dL
the most common etiology of non-anastomotic INR 1.0
biliary strictures. Hepatic artery thrombosis can IgA tissue transglutaminase 4.0 U/mL
lead to biliary complications and graft failure. IgG4 12.0 mg/dL
Therefore, early detection and prevention of he- Antinuclear antibodies 1:128
patic artery thrombosis is important. Ultrasound Hepatitis B surface antibodies Positive
with Dopplers constitutes an effective means to Hepatitis B core antibody Positive
detect hepatic artery stenosis and thrombosis. Hepatitis B surface antigen Negative
This patient does not currently harbor signs of Hepatitis C antibodies Negative
graft failure which might necessitate referral for HIV Negative
retransplantation. Antimitochondrial antibodies Negative
Smooth muscle antibodies Negative
The development of biliary strictures within two Serum protein electrophoresis Normal
months of the transplant is too early for recurrent
PSC. Cholangiographic or histologic evidence of The patient undergoes an MRCP, which is normal.
PSC at least three months following transplant is A liver biopsy is performed and shown below
required to establish a diagnosis of recurrent PSC
and exclusion of other secondary causes of biliary
strictures.

REFERENCES
Kochhar G, Parungao JM, Hanouneh IA, et al.
Biliary complications following liver transplanta-
tion. World J Gastroenterol. 2013;19:2841-2846.
Fosby B, Karlsen TH, Melum E. Recurrence
and rejection in liver transplantation for primary
sclerosing cholangitis. World J Gastroenterol.
Which of the following is true regarding her liver
2012;18:1-15.
disease?

A. She is at higher risk for death from


Question 32 atherosclerosis.
A 46-year-old woman presents to the clinic with B. Her prognosis is better because she has
a one-year history of progressive fatigue and negative AMA titers.
pruritus. She has a two-year history of an elevated C. Ursodeoxycholic acid 13-15 mg/kg/day is
alkaline phosphatase. Her past medical history is recommended.
Chapter 4 — Diseases of the biliary tract 105

D. Ursodeoxycholic acid 23-25 mg/kg/day Question 33


is recommended. An 18-year-old woman is referred for pruritus,
E. Obeticholic acid is first-line therapy. jaundice and elevated liver tests. Two years ago,
her liver tests were found to be elevated during an
CORRECT ANSWER: C evaluation for abdominal pain. One year ago, she
began to note pruritus, scleral icterus and jaun-
RATIONALE dice. She has no other health issues. Her physical
This patient has chronic cholestatic liver test exam is notable for scleral icterus. Her liver tests
abnormalities and a liver biopsy that is consis- are shown below:
tent with primary biliary cholangitis (PBC). This
liver biopsy illustrates the classic features of Total bilirubin 4.2 mg/dL
PBC, with florid duct lesions (one at the bottom Direct bilirubin 2.0 mg/dL
center and one at the top) and a mononuclear Alkaline phosphatase 398 U/L
inflammatory infiltrate surrounding bile ducts. GGT 601 U/L
A positive antimitochondrial antibody (AMA) ALT 106 U/L
titer in the setting of an elevated alkaline phos- AST 98 U/L
phatase is sufficient to establish this diagnosis.
However, when PBC is suspected in a patient Serologic testing for other causes of chronic liver
with a negative AMA, a liver biopsy is needed disease and a MRCP are unrevealing. A liver bi-
to secure the diagnosis. Fewer than five per- opsy shows portal inflammation, ductular prolifer-
cent of all patients with PBC are AMA negative. ation, and stage 3 fibrosis. Genetic testing results
However, patients with AMA-negative PBC have confirm a likely diagnosis of progressive familial
the same prognosis and response to ursodeoxy- intrahepatic cholestasis type III.
cholic acid when compared to individuals with
AMA-positive PBC. Mutation of which of the following genes is likely
The treatment of choice for PBC is ursodeoxy- to be identified in this patient?
cholic acid 13-15 mg/kg/day. Studies comparing
dosing of UDCA reveal that a dose of 13-15 mg/ A. ABCB4
kg/day is superior to 5-7 mg/kg/day and to 23-25 B. ABCB11
mg/kg/day in cost and biochemical response. C. ATP8B1
Obeticholic acid may be used in patients D. JAG1
with PBC and an inadequate response to UDCA. E. ATP7B
Recent studies reveal an improvement in alkaline
phosphatase and total bilirubin levels in patients CORRECT ANSWER: A
with PBC treated with obeticholic acid compared
with placebo. The primary and limiting side ef- RATIONALE
fect of obeticholic acid is pruritis. Progressive familial intrahepatic cholestasis
(PFIC) is a group of disorders inherited in an
REFERENCES autosomal recessive manner that results in symp-
Nevens F, Andreone P, Mazzella G, et al. A Place- tomatic cholestasis. Cirrhosis and complications
bo-Controlled Trial of Obeticholic Acid in Primary from portal hypertension can develop early in life.
Biliary Cholangitis. N Engl J Med. 2016 Aug PFIC type III has several key features that can
18;375(7):631-43. enable providers to distinguish it from the other
Lindor KD, Gershwin ME, Poupon R, et al. PFIC subtypes. First, a subset of patients with
Primary biliary cirrhosis. Hepatology. 2009 PFIC type III will present when they are young
Jul;50(1):291-308. adults. In contrast, PFIC I & II typically present
106 Digestive Diseases Self-Education Program®

within the first few months of life. Second, GGT is AST 42 U/L
elevated in PFIC type III, and normal in the other ALT 31 U/L
subtypes. Third, ductular proliferation can be seen Alkaline phosphatase 118 U/L
in liver biopsy specimens of patients with PFIC Total bilirubin 1.8 mg/dL
type III, and this finding is absent in PFIC type I Hemoglobin A1c 6.5 percent
& II. Last, PFIC III is associated with mutations in
the ABCB4 gene, which affects a protein involved Abdominal ultrasound reveals cholelithiasis.
in biliary phospholipid excretion (MDR3). Dimin-
Which of the following is the most likely cause of
ished biliary phospholipid excretion leads to an
impaired gallbladder motility in this patient?
increase in bile lithogenicity. PFIC II is associated
with mutation of the ACBC11 gene which encodes
A. Estrogen
an ATP-dependent canalicular bile salt export
B. Progesterone
pump (BSEP). PFIC I is associated with mutation
C. Elevated triglycerides
of ATP8B1 which encodes the canalicular mem-
D. Clofibrate
brane protein FIC1.
E. Ileal resection
Ursodeoxycholic acid is the initial medical
treatment of choice for all PFIC subtypes. CORRECT ANSWER: B
Mutations in the JAG1 gene are associated with
Alagille Syndrome. This presents with ductope- RATIONALE
nia, cholestasis, and liver failure and is associated There are multiple factors which can predispose
with multiple extrahepatic features including to gallstone formation including gallbladder
dysmorphic facies and cardiac and renal abnor- dysmotility leading to bile stasis, secretion of bile
malities. Mutations in the ATP7B gene on chro- supersaturated with cholesterol, and cholesterol
mosome 13 impact a copper transport protein and nucleation leading to crystal formation. Gallblad-
are associated with Wilson’s disease. der motility is stimulated by the fat content in
ingested meals, vagal (cholinergic) nerve stimu-
REFERENCES lation, cholecystokinin (CCK) and motilin. In
Davit-Spraul A, Gonzales E, Baussan C, et al. contrast, the physiologic increases in progesterone
Progressive familial intrahepatic cholestasis. Or- during pregnancy decrease gallbladder motility.
phanet J Rare Dis. 2009;4:1750-1172. Clofibrate is a lipid-lowering drug that re-
Erlinger S, Arias IM, Dhumeaux D. Inherited duces the activity of cholesterol 7α-hydroxylase
disorders of bilirubin transport and conjugation: (the rate limiting enzyme in bile salt synthesis in
new insights into molecular mechanisms and con- the classical pathway). This promotes cholesterol
sequences. Gastroenterology. 2014;46:1625-1638. supersaturation in the bile and decreases bile salt
concentrations but is not known to impact gall-
bladder motility.
Question 34 Estrogen, obesity, and hyperlipidemia predis-
A 35-year-old woman, G2P1, who is 25 weeks pose to gallstone formation by promoting choles-
pregnant presents for evaluation of right upper terol supersaturation. In addition to its impact on
quadrant pain. Her medical history is significant gallbladder motility, progesterone can also promote
for obesity, hyperlipidemia which was previ- cholesterol saturation by decreasing bile salts.
ously treated with clofibrate, and ileal resection Ileal Crohn’s disease and ileal resection pre-
for Crohn’s disease. Her pregnancy thus far has dispose to gallstone formation by interrupting
been complicated by gestational hypertension re- the enterohepatic circulation of bile acids. This
quiring treatment with labetolol. Her lab evalua- results in reduced hepatic bile acid secretion and
tion is as follows: supersaturation of bile with cholesterol.
Chapter 4 — Diseases of the biliary tract 107

REFERENCES CORRECT ANSWER: A


O’Connel K, Brasel K. Bile metabolism and litho-
genesis. Surg Clin North Am. 2014;94:361-375. RATIONALE
Shaffer EA. Review article: control of gallblad- In the United States, pigmented stones (black
der motor function. Aliment Pharmacol Ther. and brown) are less common than cholesterol
2000;14:2-8. gallstones. Both types of pigmented stones
Nayak A, Slivka A. Ceftriaxone-induced gall- contain an excess of unconjugated bilirubin and
stones: case report and literature review. ACG are composed of calcium hydrogen bilirubinate,
Case Rep J.2014;1:170-172. which is oxidized and polymerized in the hard
black stones but unpolymerized in softer brown
stones. Black pigmented gallstones are frequently
Question 35 radiopaque and form in sterile bile. Risk factors
A 56-year-old woman presents for evaluation of for black pigmented stones include hemolysis
right upper quadrant pain. Her medical history (example, sickle cell disease), cirrhosis, cystic fi-
is remarkable for obesity with a BMI of 31, hyper- brosis, and diseases affecting the ileum (example,
lipidemia, diabetes mellitus, NASH cirrhosis, and Crohn’s disease). In contrast, brown stones are
a recent admission for melena. During her prior more likely to occur in the bile ducts, are radiolu-
admission, she was treated with a proton pump cent, and form secondary to biliary stasis (ex-
inhibitor and octreotide. EGD revealed a gas- ample, biliary stricture) and infection (example,
tric ulcer with signs of recent bleeding and small Clonorchis sinensis).
esophageal varices without red wale signs. Obesity, female gender, hyperlipidemia are
risk factors for cholesterol gallstone formation.
Her lab evaluation is as follows: Octreotide decreases gallbladder motility and
long term use can increase the risk of choleli-
AST 69 U/L
thiasis.
ALT 35 U/L
Total bilirubin 1.6 mg/dL
REFERENCES
Alkaline phosphatase 121 U/L
Stinton LM, Myers RP, Shaffer EA. Epidemiol-
Leukocytes 7,500/L
ogy of gallstones. Gastroenterol Clin N Am.
An abdominal ultrasound is notable for a positive 2010;39:157-169.
sonographic Murphy’s sign, cholelithiasis, an eight Vitek L, Carey MC. New pathophysiologi-
millimeter gallbladder wall, normal appearing bile cal concepts underlying pathogenesis of pig-
ducts, and a cirrhotic appearing liver with spleno- ment gallstones. Clin Res Hepatol Gastroenterol.
megaly. She undergoes cholecystectomy. Exami- 2012;36:122-129.
nation of the gallbladder reveals numerous hard
gallstones, which are predominately composed of
calcium bilirubinate. Question 36
A 48-year-old woman presents for evaluation of
Which of the following is the most likely risk chronic abdominal pain. Her medical history is
factor for gallstones in this patient? significant for obesity, hyperlipidemia, hyperten-
sion, and migraine headaches. Abdominal ultra-
A. Cirrhosis sound reveals a shadowing defect in the common
B. Obesity bile duct concerning for choledocholithiasis, and
C. Recent octreotide use ERCP is recommended.
D. Gender
E. Hyperlipidemia Her laboratory evaluation is as follows:
108 Digestive Diseases Self-Education Program®

AST 32 U/L biliary tract. Patients may present clinically with


ALT 29 U/L abdominal pain, jaundice, and pancreatitis.
Alkaline phosphatase 112 U/L
Total bilirubin 1.2 mg/dL In a study of 388 patients with PBM who did not
have ductal dilation, 38 percent were diagnosed
with cancer. Of these cancers, the majority were
gallbladder (93 percent) and the remaining were
bile duct malignancies. The rate of malignancy in
the biliary tract was less common in those with
ductal dilation (11 percent) and nearly two-thirds
of these cases were due to gallbladder cancer,
while the remaining cases represented extrahepat-
ic bile duct malignancies. Consequently, a cho-
lecystectomy is indicated once PBM is detected.
Excision of the common bile duct in addition to a
cholecystectomy has been advocated for individu-
als with PBM plus ductal dilation.
What is the next best step in management of
this patient? REFERENCES
Tashiro S, Imaizumi T, Ohkawa H, et al. Pancreat-
A. Sphincterotomy icobiliary maljunction: retrospective and nation-
B. Recommend HIDA scan for suspected wide survey in Japan. J Hepatobiliary Pancreat
Sphincter of Oddi dysfunction Surg. 2003;10:345-351.
C. Obtain biliary brushings for cytology and Miyazaki M, Takada T, Miyaka S, et al. Risk
FISH factors for biliary tract and Ampullary carcinomas
D. Cholecystectomy and prophylactic surgery for these factors. J Hepa-
E. Stenting of the pancreatic duct tobiliary Pancreat Surg. 2008;15:15-24.
Kamisawa T, Kaneko K, Itoi T, et al. Pancre-
CORRECT ANSWER: D aticobiliary maljunction and congenital biliary
dilatation. Lancet Gastroenterol Hepatol. 2017
RATIONALE Aug;2(8):610-618.
This patient’s ERCP reveals pancreaticobiliary
maljunction (PBM) in a patient with a type III
choledochal cyst (also known as a choledocho- Question 37
cele). PBM is a congenital anomaly in which the An 18-year-old man is referred for a liver trans-
bile duct and pancreatic duct join outside the plantation evaluation after a recent hospitalization
duodenal wall, which results in formation of a long for variceal hemorrhage. He has also struggled
common channel. As a result of this anatomic with intractable pruritus for years. Since he was a
variant, the Sphincter of Oddi does not regulate baby, he was noted to have jaundice and liver test
the function of the pancreatobiliary junction. This abnormalities with a conjugated hyperbilirubi-
allows for unregulated bidirectional pancreati- nemia. In addition, he has a history of peripheral
cobiliary reflux. PBM can occur with or without pulmonary stenosis, butterfly-shaped thoracic ver-
congenital dilation of the biliary duct, although tebrae, triangular-shaped face with deep-set eyes
this patient’s ERCP does not show evidence of and a prominent forehead. A prior ophthalmologic
biliary dilation. Reflux of pancreatic juice in the exam was notable for a posterior embryotoxon,
biliary tree can predispose to malignancies of the and ductopenia was present on a liver biopsy.
Chapter 4 — Diseases of the biliary tract 109

Which of the following is the most likely and acquired, which result in cholestasis due
diagnosis? to progressive destruction of intrahepatic bile
ducts. Caroli’s syndrome is a congenital disorder
A. Vanishing bile duct syndrome characterized by multifocal, segmental dilation
B. Caroli’s syndrome of large intrahepatic bile ducts associated with
C. Alagille syndrome congenital hepatic fibrosis. Biliary atresia refers
D. Progressive familial intrahepatic to a fibro-obliterative disease of the extrahepatic
cholestasis type III bile ducts which presents with biliary obstruction
E. Biliary atresia in the neonatal period. Although biliary atresia
usually presents in isolation, it can occur with
CORRECT ANSWER: C laterality malformation or other congenital mal-
formations such as intestinal atresia and cardiac
RATIONALE abnormalities.
Alagille syndrome is an autosomal dominant
condition that can present as neonatal jaundice, REFERENCES
characterized by a conjugated hyperbilirubinemia Hartley JL, Gissen P, Kelly DA. Alagille syndrome
and ductopenia. Cirrhosis and complications sec- and other hereditary causes of cholestasis. Clin
ondary to portal hypertension can develop in a Liver Dis. 2013;17:279-300.
subset of patients later in life. Alagille syndrome Ziol M, Barbu V, Rosmorduc O, et al. ABCB4
has several key extrahepatic manifestations that heterozygous gene mutations associated with
are illustrated in this case. Cardiac defects are fibrosing cholestatic liver disease in adults. Gas-
present in 90 percent of patients, with peripheral troenterology. 2008;135:131-141.
pulmonary stenosis being the most common. Haber BA, Russo P. Biliary atresia. Gastroen-
Skeletal features, classically abnormal fusion terol Clin North Am. 2003;32(3):891.
of the spine leading to a butterfly shape of the Woolf GM, Vierling JM. Disappearing
thoracic vertebrae, are also common. A posterior intrahepatic bile ducts: the syndromes and
embryotoxon (a corneal abnormality notable on a their mechanisms. Semin Liver Dis. 1993
slit-lamp examination) can be seen in 90 percent Aug;13(3):261-75.
of patients with Alagille syndrome, in addition
to 15 percent of the normal population. Lastly,
facial features characterized by a triangular face Question 38
with deep-set eyes and a pronounced forehead A 48-year-old man presents to clinic for evalu-
are typical. Genetic testing plays an important ation of abdominal pain, fever, and jaundice.
role in confirming the diagnosis. Mutations in the His medical history is significant for gallstone
JAG1 gene (encodes the NOTCH signaling path- pancreatitis for which he underwent laparoscopic
way ligand Jagged-1) are found in 95 percent of cholecystectomy one month prior. He is mildly
cases, while defects in the NOTCH2 gene can be tachycardic with a heart rate of 103 beats per
detected in the small subset of patients without minute. Temperature is 37.6° C. Physical exami-
JAG1 mutations. nation is remarkable for scleral icterus and right
Progressive familial intrahepatic cholesta- upper quadrant pain on palpation. (See imaging
sis type III (PFIC III) is a cholestatic disease on the following page.)
caused by mutations in the ABCB4 gene. This
gene encodes the MDR3 protein which partici- Total bilirubin 4.4 mg/dL
pates in phosphatidycholine translocation in the Alkaline phosphatase 381 U/L
bile canaliculus. Vanishing bile duct syndrome ALT 106 U/L
describes a group of disorders, both congenital AST 98 U/L
110 Digestive Diseases Self-Education Program®

Which of the This patient’s imaging is not consistent with


following is the pancreaticobiliary maljunction, which is a congenital
most likely anomaly in which the bile duct and pancreatic duct
diagnosis? join outside the duodenal wall, which results in for-
mation of a long common channel. Postcholecys-
A. Postcholecystec- tectomy syndrome describes a group of symptoms,
tomy syndrome. including abdominal pain and dyspepsia, which
B. Duct of recur or do not remit following cholecystectomy.
Luschka leak.
C. Right REFERENCES
posterior. Schnelldorfer T, Sarr MG, Adams DB. What is the
duct stricture. duct of luschka?-a systematic review. J Gastroin-
D. Pancreatobiliary test Surg. 2012;16:656-662.
malformation. ASGE Standards of Practice Committee,
E. Type IVA biliary Chathadi KV, Chandrasekhara V, et al. The role
cyst. of ERCP in benign diseases of the biliary tract.
Gastrointest Endosc. 2015;81:795-803.
CORRECT Sicklick JK, Camp MS, Lillemoe KD, et al. Sur-
ANSWER: C gical management of bile duct injuries sustained
during laparoscopic cholecystectomy: periop-
RATIONALE erative results in 200 patients. Ann Surg. 2005
Laparoscopy cholescystectomy is one of the most May;241(5):786-92.
commonly performed abdominal operations in the Girometti R, Brondani G, Cereser L, et al. Post-
United States. The rate of major biliary injury is cholecystectomy syndrome: a spectrum of biliary
two to seven times higher with a laparoscopic as findings at magnetic resonance cholangiopancrea-
opposed to open approach. This patient’s imaging tography. Br J Radiol. 2010;83(989):351.
demonstrates lack of filling of the right posterior
duct which is suggestive of stricture formation. A
right posterior duct stricture is a complication of Question 39
cholecystectomy which can induce cholestasis, A 58-year-old man presents for evaluation of
cholangitis, and atrophy of the right hepatic lobe right upper quadrant pain which started eight
over time. The management of a right posterior hours prior to presentation. He is hemodynami-
duct stricture is generally surgical. cally stable and afebrile, and his lab evaluation is
Leaks from a subvesical bile duct (also referred as follows:
to as duct of Luschka) are among the more common
complications of cholecystectomy. These ducts pass AST 93 U/L
through the gallbladder fossa and empty into the ALT 77 U/L
right hepatic or common hepatic duct. Subvesical Alkaline phosphatase 132 U/L
ducts are small and difficult to detect and as a result Total bilirubin 4.8 mg/dL
are prone to injury during a cholecystectomy. The Leukocytes 5,600/L
mainstay of treatment for this type of leak involves Abdominal ultrasound is done and reveals
an ERCP with either temporary stent placement or cholelithiasis and a common bile duct which
sphincterotomy. This enables the site to heal by de- measures nine millimeters.
creasing the pressures in the proximal biliary tree,
thereby allowing bile to preferentially flow across What is the next best step in management of
the papilla rather than the site of the leak. this patient?
Chapter 4 — Diseases of the biliary tract 111

A. Cholecystectomy Question 40
B. MRCP A 52-year-old man is referred in order to evaluate
C. ERCP with sphincterotomy an incidental finding detected on a CT scan. Re-
D. ERCP with sphincterotomy followed cently, he underwent a CT of his abdomen during
by cholecystectomy an evaluation for hematuria. This showed a cystic
E. Cholecystectomy with intra-operative structure on the medial wall of the duodenum.
cholangiogram You perform an ERCP (shown below), which re-
veals a focal dilation of the intra-duodenal portion
CORRECT ANSWER: D of the bile duct.

RATIONALE
The above patient is presenting with symp-
tomatic gallstone disease. His bilirubin
greater than 4.0 mg/dL, combined with
the presence of a dilated common bile duct
(CBD), are concerning for choledocholithiasis.
Risk stratification for choledocholithiasis in
patients presenting with symptomatic cho-
lelithiasis should be performed. Very strong
predictors of a CBD stone include a bilirubin
greater than 4 mg/dL, ascending cholangitis,
and a CBD stone visualized on an ultrasound.
It is recommended that a preoperative ERCP Which of the following is the correct diagnosis?
be performed if any of these very strong
predictors are present. Strong predictors of a A. Type I biliary cyst
CBD stone include a dilated CBD (greater than B. Type II biliary cyst
six millimeters with intact gallbladder) and a C. Type III biliary cyst
modest elevation in bilirubin (1.8-4 mg/dL). D. Type IVA biliary cyst
If both of the strong predictors are present, a E. Type IVB biliary cyst
pre-operative ERCP is suggested. Moderate
CORRECT ANSWER: C
predictors of a CBD stone include abnormal
liver tests other than bilirubin, age greater RATIONALE
than 55 years and gallstone pancreatitis. If The cholangiogram illustrates a choledochocele
none of these predictors are present then (also referred to as a Type III choledochal cyst),
patients should proceed directly to a cholecys- which is a cystic dilation of the intra-duodenal
tectomy. However, for the remaining patients portion of the common bile duct. The character-
at an intermediate-risk for a CBD stone, one istic features of the other congenital biliary cyst
of the following is suggested: preoperative subtypes are as follows: Type I=dilation of extra-
MRCP, preoperative endoscopic ultrasound or hepatic bile duct with normal intrahepatic duct
intraoperative cholangiogram. (most common subtype); Type II=diverticulum
of the extrahepatic bile duct; Type IVa multiple
REFERENCE dilation of the intra and extrahepatic biliary tree;
ASGE Standards of Practice Committee, Maple JT, Type IVb=multiple dilations of the extrahepatic
Ben-Menachem T, et al. The role of endoscopy in bile duct; Type V (Caroli’s disease)=multiple cystic
the evaluation of suspected choledocholithiasis. dilations of the intrahepatic bile ducts with normal
Gastrointest Endosc. 2010;71:1-9. extrahepatic duct.
112 Digestive Diseases Self-Education Program®

While choledochoceles can be asymptomatic, one year ago with nonspecific findings on patholo-
abdominal pain is present in 90 percent of symp- gy. He is up to date with cancer screening and had
tomatic cases, and jaundice develops in a smaller an unremarkable colonoscopy two years ago. His
subset of patients (~20 percent). There are several physical examination is remarkable for jaundice.
key features that distinguish choledochoceles from His laboratory evaluation is as follows:
other choledochal cysts. First, pancreatitis is the
most common complication in choledochoceles, Hemoglobin 13.4 g/dL
and it can occur in 70 percent of individuals. Pan- Leukocytes 7,500/L
creatitis is less common among other subtypes of ESR 62
biliary cysts. Second, choledochoceles (in addition AST 35 U/L
to Type II cysts) have a very low malignant poten- ALT 39 U/L
tial (~two percent), particularly when they present ALP 555 U/L
without an anomalous pancreatobiliary junction, Total bilirubin 5.4 mg/dL
with only several cases of cholangiocarcinoma or CA 19-9 18 U/L
ampullary cancer being reported. This stands in
stark contrast to the other cyst subtypes (Types I, You recommend MRCP which reveals stricturing
IV and V), which carry a high risk for biliary tract of the extrahepatic common bile duct, a prominent
cancers. Lastly, symptomatic choledochoceles distal bile duct stricture, and enlargement of the
can be treated less invasively through endoscopic pancreas. What is the next best step in evaluation
cyst excision or sphincterotomy. Conversely, the of this patient?
treatment of Type I and IV cysts involves complete
extrahepatic bile duct cyst excision (Roux-en- A. Check serum IgG4 level
Y hepaticojejunostomy), Type II is treated by a B. ERCP with biliary brushings for cytology
diverticulotomy followed by a primary common and FISH
bile duct closure, and Type V (Caroli’s disease) can C. Endoscopic ultrasound to assess pancreatic
be treated with either a hepatic resection or liver head and distal common bile duct
transplantation (when indicated). D. PET scan
E. Refer for hepaticojejunostomy
REFERENCES
CORRECT ANSWER: A
Law R, Topazian M. Diagnosis and treatment
of choledochoceles. Clin Gastroenterol Hepatol.
RATIONALE
2014;12:196-203.
This patient presents with jaundice and imaging
Soares KC, Arnaoutakis DJ, Kamel I, et al.
features suggestive of sclerosing cholangitis. The
Choledochal cysts: presentation, clinical dif-
presence of a prominent pancreas on imaging and
ferentiation, and management. J Am Coll Surg.
history of lymphadenopathy is suggestive of IgG4-
2014;219:1167-1180.
related disease. IgG4 related sclerosing disease
is a systemic disease characterized by extensive
Question 41 tissue infiltration by IgG4 positive plasma cells
A 62-year-old man is referred for a second opin- and T cells, which cause fibrotic lesions associated
ion regarding primary sclerosing cholangitis. He with obliterative phlebitis. IgG4-related disease
presented with jaundice and vague abdominal can affect the pancreas, bile duct, gallbladder,
discomfort three months prior, and PSC was diag- salivary glands, kidney, lung, prostate, and retro-
nosed after characteristic features were noted on peritoneum. Lymphadenopathy is also common.
ERCP. His medical history is otherwise remark- The disease occurs more often in older men and is
able for coronary artery disease and excisional often characterized by elevated IgG4 levels.
biopsy of an enlarged supraclavicular lymph node The patient’s history of lymphadenopathy with
Chapter 4 — Diseases of the biliary tract 113

imaging features of sclerosing cholangitis and has a layer of cuboidal cells with underlying ovari-
enlargement of the pancreas make IgG4 related an-type stroma. What is the most likely diagnosis?
sclerosing disease the most likely diagnosis.
A. Intrahepatic cholangiocarcinoma
REFERENCES B. Echinococcal cyst
Ghazale A, Chari ST, Zhang L, et al. Immuno- C. Type I biliary cyst
globulin G4-associated cholangitis: clinical profile D. Biliary cystadenoma
and response to therapy. Gastroenterology. E. Type II biliary cyst
2008;134:706-715.
Hart PA, Topazian MD, Witzig TE, et al. Treat- CORRECT ANSWER: D
ment of relapsing autoimmune pancreatitis with
immunomodulators and rituximab: the Mayo RATIONALE
Clinic experience. Gut. 2013;62:1605-1615. Biliary cystadenomas are irregular cystic lesions
that are slow-growing tumors originating from the
intrahepatic biliary tree (rarely the extrahepatic
Question 42 biliary tree). These lesions are found in middle-age
A 72-year-old woman presents to establish care after women (90 percent). Typical imaging features are
recently moving from Central America. She does shown above: a multiloculated cyst with internal
note abdominal fullness and early satiety. She is oth- septations and a thickened irregular wall. Calci-
erwise healthy. On physical examination, her abdo- fications and papillary wall nodules may also be
men is distended with a palpable mass in her upper present. While not a universal finding, it is classic
abdomen. Her laboratory evaluation is as follows: for these cysts to have an ovarian-like stroma that
expresses receptors for estrogen and progesterone.
Total bilirubin 1.1 mg/dL While the true natural history of biliary cystadeno-
Direct bilirubin 0.4 mg/dL mas is unclear, in some reports, up to 20 percent
Alkaline phosphatase 120 U/L undergo malignant transformation into biliary cyst-
ALT 41 U/L adenocarcinomas. An elevated CA 19-9 can be seen
AST 33 U/L in either condition biliary cystadenoma or cystad-
CA 19-9 315 U/mL enocarcinoma. Consequently, complete surgical
removal is generally recommended once detected.
You recommend MRI of the abdomen which is The rate of recurrence after surgery is low.
shown below:
It would be atypical for a cholangiocarcinoma
She subsequently undergoes a surgical resection of to have the above radiographic features. Echino-
the lesion. The pathology shows that the structure coccal cysts can present as a complex cyst. While
the patient has risk factors for
exposure to Echinococcus, the
absence of daughter cysts and
the histologic features in this
case make this option less likely.
Type I biliary cysts involve the
extrahepatic duct and are asso-
ciated with an increased risk of
biliary tract malignancies. Type
II biliary cysts are diverticula
connected to the extrahepatic
duct through a narrow stalk.
114 Digestive Diseases Self-Education Program®

REFERENCE lain gallbladder is associated with an increased


Soares KC, Arnaoutakis DJ, Kamel I, et al. risk of gallbladder adenocarcinoma. This risk
Cystic neoplasms of the liver: biliary cystad- appears to be higher in patients with selective mu-
enoma and cystadenocarcinoma. J Am Coll Surg. cosal calcification. Porcelain gallbladder is often
2014;218:119:128. diagnosed incidentally on imaging as patients are
frequently asymptomatic. An incidental find-
ing of calcium in the gallbladder requires further
Question 43 evaluation to confirm porcelain gallbladder as
A 68-year-old woman is referred to you for further other entities including gallstones, renal cysts, and
evaluation of abdominal imaging findings. She cystic lesions of the pancreas can create a similar
presented to her primary care physician with ab- appearance on plain films. Abdominal CT has a
dominal discomfort and constipation. An abdomi- high sensitivity for identification of calcification
nal x-ray was obtained and revealed normal bowel or cancer within the gallbladder and is therefore
caliber and a rim of calcification in the gallbladder a good diagnostic modality. The management
wall. Her lab evaluation is as follows: of porcelain gallbladder is controversial, but it is
generally considered an indication for cholecystec-
Leukocytes 8,900/L tomy given associated cancer risk.
Hemoglobin 12.6 g/dL
AST 32 U/L REFERENCES
ALT 29 U/L Shimizu M, Miura J, Tanaka T, et al. Porcelain
Alkaline phosphatase 98 U/L gallbladder: relation between its type by ultra-
Total bilirubin 1.9 mg/dL sound and incidence of cancer. J Clin Gastroen-
Direct bilirubin 0.4 mg/dL terol. 1989;11:471.
Her abdominal discomfort improved after initia- Khan ZS, Livingston EH, Huerta S. Reassess-
tion of a bowel regimen, and her abdomen is soft ing the need for prophylactic surgery in patients
and non-tender on exam.Which of the following is with porcelain gallbladder: case series and sys-
the next best step in management? tematic review of the literature. Arch Surg. 2011
Oct;146(10):1143-7.
A. No further studies needed as she is
asymptomatic
B. Repeat ultrasound in six months
Question 44
A 54-year-old woman presents to your clinic for
C. Laparoscopic cholecystectomy
a second opinion regarding primary biliary chol-
D. Exploratory laparotomy
angitis. She was diagnosed with PBC three years
E. CT abdomen/pelvis with IV contrast
prior after characteristic findings were seen on liver
biopsy which was done to evaluate chronic eleva-
CORRECT ANSWER: E
tion in her alkaline phosphatase. She has been on
ursodeoxycholic acid at a dose of 15 mg/kg/day for
RATIONALE
two years. Her laboratory evaluation is as follows:
The incidental finding of calcification of the
gallbladder wall is concerning for porcelain AST 54 U/L
gallbladder. Porcelain gallbladder is categorized ALT 61 U/L
according to the extent of calcification. Complete Alkaline phosphatase 396 U/L
intramural calcification describes a continuous Total bilirubin 2.1 mg/dL
band of calcium infiltrates in the gallbladder wall. Platelets 190x109/L
In contrast, selective mucosal calcification occurs Creatinine 1.1mg/dL
with more segmental deposits of calcium. Porce- INR 1.1
Chapter 4 — Diseases of the biliary tract 115

She endorses moderate pruritis and fatigue. She addition of fenofibrate to UDCA in patients with
is not interested in participating in clinical trials. PBC who had an inadequate biochemical response
Which of the following is the next best step in to UDCA resulted in decreased risk of liver decom-
management of this patient’s PBC? pensation, mortality, and liver transplantation.
Studies of methotrexate in PBC have yielded
A. Increase ursodeoxycholic acid dose to conflicting results and thus it is not routinely
18-23 mg/kg/day recommended. Advanced liver disease secondary
B. Discontinue UDCA and start obeticholic to PBC is an indication for liver transplantation,
acid 10 mg per day although this patient does not demonstrate signs
C. Start methotrexate 0.25 mg/kg/week of decompensated cirrhosis at this time.
D. Start fenofibrate 145 mg daily
E. Refer for liver transplantation REFERENCES
Carey EJ, Ali AH, Lindor KD. Primary biliary cir-
CORRECT ANSWER: D rhosis. Lancet. 2015 Oct 17;386(10003):1565-75.
Pares A, Caballeria L, Rodes J. Excellent long-
RATIONALE term survival in patients with primary biliary
This patient has demonstrated an inadequate cirrhosis and biochemical response to ursodeoxy-
response to ursodeoxycholic acid (UDCA), which cholic acid. Gastroenterology 2006;130:715-20.
is generally defined as persistent elevation in Corpechot C, Chazouilleres O, Rousseau A, et
alkaline phosphatase to greater than 1.67 times al. A placebo-controlled trial of bezafibrate in pri-
the upper limit of normal after one year of UDCA mary biliary cholangitis. N Engl J Med. 2018 Jun
therapy. Biochemical response to UDCA predicts 7;378(23):2171-2181.
prognosis in patients with PBC. Approximately Nevens F, Andreone P, Mazzella G, et al. A
30 percent of patients demonstrate suboptimal placebo-controlled trial of obeticholic acid in pri-
response to UDCA and are at risk for more rapid mary biliary cholangitis. N Engl J Med.2016 Aug
progression of disease when compared with pa- 18:375(7):631-43.
tients who normalize alkaline phosphatase after
initiation of UDCA. As a result, adjuvant or alter-
native therapy is recommended in patients who do Question 45
A 31-year-old man is seen for a one-week history
not respond to UDCA.
of epigastric pain and scleral icterus. One month
Obeticholic acid is a farsenoid X receptor
agonist which has been shown to decrease alkaline earlier, he developed diarrhea and fatigue, which
has persisted. He denies any prior medical prob-
phosphatase and total bilirubin levels when used
lems, though he admits he has not seen a doctor in
alone or in combination with UDCA. Worsening
years. He is currently visiting family in the United
pruritus is a common, dose related side effect of
States, but he resides in South Africa. His labora-
obeticholic acid that is more common at 10 mg
tory tests are shown below:
than five mg. Hence, among those without ad-
vanced liver disease, starting obeticholic acid at Total bilirubin 3.5 mg/dL
five mg per day is recommended. Direct bilirubin 2.9 mg/dL
Bezafibrate is an agonist of the peroxisome pro- Alkaline phosphatase 720 U/L
liferator activated receptor which has been shown ALT 105 U/L
to improve biochemical response in patients who AST 117 U/L
had an inadequate response to UDCA. Although Albumin 2.1 g/dL
bezafibrate is not available in the United States, INR 1.2
fenofibrate is approved for management of hy- HIV viral load 450,000 copies/mL
perlipidemia. A retrospective study revealed that CD4 count 25 cells/mm3
116 Digestive Diseases Self-Education Program®

An abdominal ultrasound shows intra and REFERENCES


extrahepatic ductal dilation and an ERCP Tonolini M, Bianco R. HIV-related?AIDS chol-
shows strictured intrahepatic ducts with angiopathy: pictorial review with emphasis on
papillary stenosis. MRCP findings and differential diagnosis. Clin
Imaging. 2013;37:219-226.
Hashmey R, Smith NH, Cron S, et al. Cryp-
Which of the following statements is true regard-
tosporidiosis in Houston, Texas. A report of 95
ing this patient’s diagnosis?
cases. Medicine (Baltimore). 1997;76:118-139.
Forbes A, Blanshard C, Gazzard B. Natural
A. Cryptosporidium parvum is the most com
history of AIDS related sclerosing cholangitis: a
monly implicated organism
study of 20 cases. Gut. 1993;34:116-121.
B. Symptoms and cholangiographic abnormalities
Castiella A, Iribarren JA, Lopez P, et al.
are likely to improve with treatment of the
Ursodeoxycholic acid in the treatment of AIDS-
causative organism
associated cholangiopathy. Am J Med. 1997
C. This is frequently seen in patients with a CD4
Aug;103(2):170-1.
count less than 250 cells/mm3
D. Ursodeoxycholic acid is contraindicated.
E. Cryptococcus neoformans may lead to Question 46
these findings A 26-year-old man is admitted to the hospital
for recurrent fever, rigors and jaundice. He is a
CORRECT ANSWER: A recent immigrant from Vietnam. His physical
exam is significant for a temperature of 39°C,
RATIONALE scleral icterus and right-upper quadrant tender-
AIDS cholangiopathy is uncommon in the West- ness. His laboratory tests are shown below:
ern world, but it can be encountered among Total bilirubin 4.9 mg/dL
patients with untreated HIV who are severely Direct bilirubin 4.1 mg/dL
immunosuppressed, typically with a CD4 count Alkaline phosphatase 428 U/L
less than 100/mm3. The presence of papillary ALT 95 U/L
stenosis and sclerosing cholangitis is unique to AST 106 U/L
AIDS cholangiopathy and is the most common
cholangiographic manifestation. Cryptospo- Intravenous antibiotics are initiated. An abdomi-
ridium parvum is the most commonly isolated nal ultrasound shows dilated intra- and extrahe-
pathogen associated with AIDS cholangiopa- patic ducts. An ERCP is performed (cholangio-
thy. Other implicated pathogens include CMV, gram below), and multiple stones are extracted.
Microsporidium, and Cyclospora. Despite the
pivotal role of infection in the pathogenesis of
AIDS cholangiopathy, antimicrobial therapy
directed at the underlying pathogen does not
improve the disease course or cholangiographic
abnormalities. Treatment of AIDS cholangi-
opathy consists of highly active anti-retroviral
therapy (HAART) and endoscopic measures to
relieve biliary obstruction if present. Studies of
ursodeoxycholic acid in AIDS cholangiopathy
demonstrate mild improvement in serum bio-
chemical parameters and symptoms.
Chapter 4 — Diseases of the biliary tract 117

Which of the following is the most likely Total bilirubin 4.9 mg/dL
causative organism? Direct bilirubin 4.1 mg/dL
Alkaline phosphatase 428 U/L
A. Entamoeba histolytica ALT 95 U/L
B. Echinococcus granulosis AST 106 U/L
C. Cryptosporidium parvum
Intravenous antibiotics are initiated. An abdomi-
D. Clonorchius sinensis
nal ultrasound shows dilated intra- and extrahe-
E. Schistosoma japonicum
patic ducts. An ERCP is performed (cholangio-
gram below), and multiple stones are extracted.
CORRECT ANSWER: D

RATIONALE
This patient presents with a history consistent
with recurrent pyogenic cholangitis. This is clas-
sically described in patients of East Asian descent
who present with recurrent episodes of cholangitis
with intra- and extrahepatic ductal dilation and
choledocholithiasis. This disease can lead to liver
abscesses and constitutes a risk for cholangiocar-
The patient is most likely to have which of the
cinoma. While the exact etiology of the disease is
following?
not known, the most quoted theory links biliary
tract infection with parasites. The most com-
A. Brown pigmented stones
monly implicated parasites include Clonorchis
B. Black pigmented stones
sinensis, Opisthorchis species, Fasciola hepatica,
C. Cholesterol stones
and A. Lumbricoides.
D. Primary sclerosing cholangitis
Entamoeba histolyticum is a parasite
E. Gangrenous cholecystitis
which causes infection in the developing world.
Liver abscess is the most common extraintestinal
CORRECT ANSWER: A
manifestation of amebic infection. Echinococcus
can also cause multiseptated, cystic lesions
RATIONALE
in the liver. Cryptosporidium parvum is the
Recurrent pyogenic cholangitis (oriental cholangi-
most frequently implicated organism in AIDS
tis) is characterized by the development of biliary
cholangiopathy. Schistosomiasis typically
pigmented stones and strictures which results in
presents with portal fibrosis and granulomas
recurrent episodes of cholangitis. It is found among
of the liver with evidence of portal hypertension
individuals who have lived in Southeast Asia. The
and eosinophillia.
precise etiology is unknown, although parasitic
and/or bacterial infections have been postulated as
Question 47 contributing factors, particularly since this disorder
A 26-year-old man is admitted to the hospital for is limited to an area where biliary parasites are en-
recurrent fever, rigors and jaundice. He is a recent demic. The presence of bile duct stones is frequent-
immigrant from Vietnam. His physical exam is ly associated with intrahepatic ductal dilation (left
significant for a temperature of 39°C, scleral ic- hepatic ducts are more often involved than those
terus and right-upper quadrant tenderness. on the right) and downstream strictures. Brown
pigment stones are commonly associated with this
His laboratory tests are shown as follows: condition because they form as a result of stasis and
118 Digestive Diseases Self-Education Program®

infection. These stones are soft and radiolucent. D. Transfuse packed red blood cells and fresh
Often, colonies of bacteria can be found growing frozen plasma
on brown pigmented stones. Black pigmented and E. Upper endoscopy
cholesterol stones are generally formed in sterile
bile within the gallbladder. Patients with primary CORRECT ANSWER: B
sclerosing cholangitis have cholangiographic ap-
pearance of “beads on a string”, and while such RATIONALE
patients can develop hepatolithiasis, this would not Hemobilia is a rare cause of gastrointestinal bleeding.
be a typical presentation. The triad of bleeding, jaundice and abdominal pain
has been described with this condition. However, only
REFERENCE a minority (approximately one-third) of patients will
Tsui W, Chan Y, Wong C, et al. Hepatolithiasis and present with all of these features. An important clue
the syndrome of recurrent pyogenic cholangitis: that should raise the index of suspicion of hemobilia is
clinical, radiologic, and pathologic features. Semin a recent history of biliary tract or hepatic parenchymal
Liver Dis. 2011;31:33-48. instrumentation. Trauma can also be a precipitating
factor. Hemobilia secondary to cholelithiasis, choledo-
cholithiasis, or a ruptured hepatic artery aneurysm
Question 48 can occur, but are less common than iatrogenic or
A 63-year-old man is admitted to the hospital for traumatic causes. Angiography can identify the source
abdominal pain and melena. He has a history of of bleeding and achieve hemostasis through emboli-
hepatitis C and is currently enrolled in a hepatitis zation. An ERCP can be employed to clear the biliary
C therapeutic trial. As part of that clinical study, he tree in a subset of patients with ascending cholangitis
underwent a percutaneous liver biopsy and labora- secondary to a thrombus, but it usually does not play
tory tests which showed AST of 105 U/L and ALT a significant role in hemostasis.
of 115 U/L, and the remainder of his liver tests were
normal. Two days later, he developed mild right REFERENCE
upper quadrant pain. Over the next 24 hours, his Chin WM, Enns R. Hemobilia. Curr Gastroenterol
pain intensified and he had recurrent episodes of Rep. 2010;12:121-129.
melena. He subsequently presented to the emer-
gency room and is admitted. Aside from sinus
tachycardia, his vital signs are normal. His labora- Question 49
tory studies on admission are shown below: A 24-year-old woman is referred to your practice
for a second opinion regarding abnormal imaging
Hemoglobin 6.9 g/dL
findings and recurrent low grade fevers. Ultra-
WBC 10.5/L
sound done to evaluate right upper quadrant pain
Total bilirubin 3.4 mg/dL
revealed dilation of the common bile duct. ERCP
Direct bilirubin 2.6 mg/dL
is recommended and revealed multiple dilations of
Alkaline phosphatase 160 U/L
the extra-hepatic biliary tree as shown below.
ALT 190 U/L
AST 185 U/L

Which of the following is the next best step in


management?

A. Octreotide, pantoprazole, and antibiotics


B. Interventional radiology for embolization
C. Surgery consult for hepatectomy
Chapter 4 — Diseases of the biliary tract 119

Her medical history is otherwise significant for Question 50


hypothyroidism. Liver enzymes were assessed A 43-year-old woman presents to the office after
and are normal. Roux-en-Y surgery for weight loss. She has a
strong family history of gallstones, and asks about
Which of the following is the next best step in measures to prevent gallstone formation after her
management? surgery.

A. Colonoscopy with biopsies Which of the following agents has potential effica-
B. Liver biopsy cy to reduce gallstone formation for this patient?
C. Surgical referral for cholecystectomy and
hepaticojejunostomy A. Conjugated estrogens
D. ERCP with biliary brushings and stenting B. Ursodeoxycholic acid
of the common bile duct C. Fenofibrate
E. Referral to liver transplant center D. Simvastatin
E. Cholestyramine
CORRECT ANSWER: C
CORRECT ANSWER: B
RATIONALE
This patient’s imaging findings, with multiple RATIONALE
dilations of the extrahepatic biliary tree, are Risk factors for gallstone formation include
characteristic of type IVB biliary cysts. Due to increased age, female gender, pregnancy, dyslip-
high malignant potential, type IVB biliary cysts idemia, diabetes, obesity and rapid weight loss
require cholecystectomy and cyst excision with – especially after gastric bypass surgery. Medica-
Roux-en-Y hepaticojejunostomy. If intrahe- tions such as hormone replacement therapies/
patic ducts are involved, hepatic lobe resection oral contraceptive agents, fibrates, somatostatin
is also recommended. analogues also increase gallstone risk. Currently,
This patient does not have evidence of pri- there is evidence suggesting potential benefit of
mary sclerosing cholangitis and therefore does prophylactic cholecystectomy during Roux-en-Y
not require screening colonoscopy with biop- gastric bypass, given the potential risk of gallstone
sies to assess for IBD. Due to the malignant formation with rapid weight loss following sur-
potential of her cysts, ERCP with stenting is not gery. However, there is also data from random-
sufficient. ized controlled trials that the use of ursodeoxycho-
lic acid following surgery may help reduce risk of
REFERENCES
gallstone formation for this group of patients.
Ronnekleiv-Kelly SM, Soares KC, Ejaz A, et al.
Management of choledochal cysts. Curr Opin Gas-
REFERENCE
troenterol. 2016 May;32(3):255-31.
Stokes et al. Ursodeoxycholic Acid and Diets
Law R, Topazian M. Diagnosis and treatment
Higher in Fat Prevent Gallbladder Stones Dur-
of choledochoceles. Clin Gastroenterol Hepatol.
ing Weight Loss: A Meta-analysis of Randomized
2014;12:196-203.
Controlled Trials. 2014. Clin Gastroenterol Hepa-
Soares KC, Arnaoutakis DJ, Kamel I, et al.
tol. 2014;12:1090-1100.
Choledochal cysts: presentation, clinical dif-
ferentiation, and management. J Am Coll Surg.
2014;219:1167-1180.
120 Digestive Diseases Self-Education Program®
Answers & critiques

CHAPTER 5

Viral hepatitis
Alfred Sidney Barritt, IV, MD & Andrew S. deLemos, MD

Question 1 Bourliere M, Gordon SC, Flamm SL, Cooper CL,


A 56-year-old woman with genotype 1a HCV Ramji A, Tong M, et al. Sofosbuvir, Velpatasvir,
returns to clinic. She was a non-responder to and Voxilaprevir for Previously Treated HCV In-
peg-interferon/ribavirin based therapy and a fection. The New England Journal of Medicine.
relapser to sofosbuvir/ledipasvir based therapy. 2017;376(22):2134 - 2146.
She has F3 disease and would like to be retreat-
ed. What is the best course of therapy? Question 2
A 66-year-old man has been diagnosed with
A. Sofosbuvir/Ledipasvir plus ribavirin genotype 3 chronic hepatitis C. He has Child’s
for 12 weeks A cirrhosis with a MELD score of 9. He is naïve
B. Sofosbuvir/Ledipasvir for 24 weeks to therapy and has no resistance mutations. He
C. Sofosbuvir/ Velpatasvir for 12 weeks asks if he should be treated. Which of the fol-
D. Sofosbuvir/ Velpatasvir/ Voxilaprevir lowing is the best course of action?
for 12 weeks
E. Treatment not possible at this time A. Since he has cirrhosis and genotype 3 HCV,
success rates are low and he should wait for
CORRECT ANSWER: D liver transplantation
B. Daily fixed dose combination Sofosbuvir/
RATIONALE Ledipasvir for 24 weeks
While relapse to a sofosbuvir/ledipasvir combina- C. Daily fixed dose combination Sofosbuvir/
tion is relatively rare, it can happen either due to Ledipasvir plus ribavirin for 12 weeks
poor adherence or due to viral resistance to the D. Daily fixed dose combination Sofosbuvir/
NS5A/NS5B treatment combination. The POLAR- Velpatasvir for 12 weeks
IS-1 trial, a placebo controlled phase 3 clinical trial E. Daily fixed dose combination Sofosbuvir/
studying a 12-week daily fixed dose combination of Daclatasvir for 12 weeks
Sofosbuvir/ Velpatasvir/ Voxilaprevir for 12 weeks
in NS5A treatment experienced patients had an CORRECT ANSWER: D
overall cure rate of 97 percent. Baseline RASs and
cirrhosis were not predictive of treatment failure. RATIONALE
Extended therapy with sofosbuvir/ledipasvir or The ASTRAL-3 trial compared sofosbuvir plus
the addition of ribavirin has not been shown to Velpatasvir to sofosbuvir plus ribavirin in more
increase cure rates after relapse. than 500 patients. Among those with compen-
sated cirrhosis, cure rates were 93 percent in the
REFERENCES sofosbuvir/Velpatasvir arm compared to 73 per-
https://www.hcvguidelines.org/treatment- cent in the sofosbuvir ribavirin arm. Sofosbuvir/
experienced/gt1/ns5a Ledipasvir combinations are only approved

121
122 Digestive Diseases Self-Education Program®

for genotypes 1 and 4. Sofosbuvir/Daclatasvir REFERENCES


combinations are second line therapy for genotype https://www.hcvguidelines.org/unique-popula-
3 HCV with cirrhosis, but it must be dosed for 24 tions/renal-impairment
weeks. As this patient has well compensated cir- Roth D, Nelson DR, Bruchfeld A, et al. Grazo-
rhosis, treatment and cure now would reduce the previr plus elbasvir in treatment-naive and treat-
risk of decompensated liver disease in the future. ment experienced patients with hepatitis C virus
genotype 1 infection and stage 4-5 chronic kidney
REFERENCES disease (the C-SURFER study): a combination
https://www.hcvguidelines.org/treatment-naive/ phase 3 study. Lancet. 2015;386(10003):1537-45.
gt3/compensated-cirrhosis
Foster GR, Afdhal NH, Roberts SK. Sofosbuvir
and Velpatasvir for HCV Genotype 2 and 3 Infec- Question 4
tion. N Engl J Med. 2015;373(27):2608-17. A 44-year-old male with known chronic hepatitis
B, HBeAg +, on entecavir, develops jaundice and is
admitted to the hospital. He has recently returned
Question 3 from a two-month trip to Italy, Serbia and Turkey.
A 47-year-old man with stage 5 chronic kidney dis- He takes hydrochlorothiazide chronically for mild
ease on hemodialysis is referred to your clinic. He hypertension. His total bilirubin is 7.0 mg/dl,
has genotype 1a HCV and F2 fibrosis. He wants to AST 2045 U/L, ALT 3010 U/L, INR 1.2. Over the
discuss treatment options. What is the best advice? first 24 hours, the following labs return:

A. He is not eligible for HCV therapy due to HB surface Ag (+)


hemodialysis HB envelope eAg
B. Sofosbuvir/Ledipasvir combination therapy HB envelope eAb
dosed after dialysis for 12 weeks HBV DNA detected, less than 10 IU/mL
C. Sofosbuvir/Ledipasvir combination therapy HAV IgM (-)
for 12 weeks HAV total (+)
D. Elbasvir/Grazoprevir combination therapy HCV Ab (-)
for 12 weeks HCV RNA undetected
E. Daily fixed dose combination Sofosbuvir/ Anti-nuclear antibody (-)
Velpatasvir for 12 weeks
What is the most likely cause of his acute icteric
CORRECT ANSWER: D illness?

RATIONALE A. Acute on chronic flare of hepatitis B


Elbasvir and grazoprevir are hepaticaly metabo- B. Acute hepatitis A infection
lized and undergo minimal renal elimination C. Superimposed Hepatitis Delta infection
making them safe for use in patients with end D. Medication non compliance
stage renal disease. The C-Surfer trial evaluated E. Drug induced liver injury
elbasvir and grazoprevir in genotype 1 patients
with advanced renal disease inclusive of patients CORRECT ANSWER: C
on hemodialysis. Cure rates in this trial were 94-
99 percent. Sofosbuvir containing regimen are RATIONALE
not approved for patient with CKD stage 4-5 or Hepatitis D or hepatitis delta only occurs in
those on hemodialysis, even when given in a dose patients with chronic hepatitis B. The infection
reduced or post-dialysis fashion. with the delta virus can occur at the time of HBV
Chapter 5 — Viral hepatitis 123

infection, known as co-infection or afterwards, indicated as the patient has decompensated liver
known as superinfection. Hepatitis D is trans- disease. Supportive care is appropriate, although
mitted parenterally and sexually and is of higher there are emerging data for the use of N-acetyl cys-
prevalence in eastern and southern Europe as well teine for cases of non-acetaminophen liver failure,
as parts of Africa and South America. Patients but such supportive care should be in a transplant
with superinfection with hepatitis delta will often center. Hepatitis B immune globulin and vaccina-
have suppressed HBV replication and are at high tion have no role in acute HDV infection.
risk for a fulminant course. This vignette is not
an acute on chronic flare of hepatitis B as there is REFERENCES
no evidence of significant HBV replication. The Hughes SA, Wedemeyer H, Harrison PM. Hepati-
patient is immune to HAV. Since his HBV DNA is tis delta virus. Lancet 2011;378:73-85.
relatively suppressed, it appears that he is taking
the entecavir. Hydrochlorothiazide is exceedingly
unlikely to cause a drug induced liver injury. Question 6
A 28-year-old woman, originally from Taiwan,
REFERENCE presents to labor and delivery late in the third
Hughes SA, Wedemeyer H, Harrison PM. trimester of her pregnancy and is ready to deliver.
Hepatitis delta virus. Lancet 2011;378:73-85. She is found to have chronic hepatitis B and nor-
mal liver tests. She received no prenatal care. She
is asymptomatic from her liver disease and has
Question 5 never had any decompensation events.
Over the following 24 hours, the patient in the
above vignette becomes encephalopathic, the INR What is the appropriate course of action?
rises to 2.9 and the total bilirubin climbs to 11.5
mg/dl. A. Hepatitis B immune globulin for the mother
B. Prophylactic lamivudine for the infant
What is the most appropriate course of action? C. Hepatitis B immune globulin and HBV
vaccination for the infant
A. Switch to tenofovir alafenamide fumarate D. HBV vaccination only for the infant
(TAF) 25mg daily E. No intervention/Supportive care for the infant
B. Start interferon alfa 2b 180mcg weekly
C. Close monitoring in the intensive care unit CORRECT ANSWER: C
with supportive care
D. Emergent transfer to a liver transplant center RATIONALE
E. Administer Hepatitis B immune globulin In order to reduce the possibility of vertically
and Hepatitis B vaccine transmitted HBV, both hepatitis B immune globu-
lin and vaccination should be given to the infant.
CORRECT ANSWER: D Vaccination alone is insufficient as immunity to
vaccination is not immediate. While antiviral
RATIONALE therapy in the third trimester in the pregnant
The patient has become coagulopathic and en- mother may be appropriate in certain cases, there
cephalopathic, from suspected superinfection with is no role for prophylactic antiviral therapy for
hepatitis delta, and should be evaluated for possible infants nor is lamivudine approved for use in this
liver transplantation emergently. Therapies aimed population.
at treating hepatitis B will be ineffective as the HBV
DNA is already suppressed. Interferon is contra-
124 Digestive Diseases Self-Education Program®

REFERENCES progress to fulminant liver failure compared to


Pan CQ, Duan ZP, Bhamidimarri KR, Zou HB, ~1 percent of acute HBV infections.
Liang XF, Li J, Tong MJ. An algorithm for risk
assessment and intervention of mother to child REFERENCE
transmission of hepatitis B virus. Clin Gastroen- Ryder SD, Beckingham IJ. ABC of diseases of
terol Hepatol 2012;10:452-9. liver, pancreas, and biliary system: Acute hepati-
tis. BMJ 2001;322:151-3.

Question 7
A 44-year-old African American male has an Question 8
otherwise asymptomatic icteric illness three A 45-year-old African American female with
weeks after consuming raw shellfish. He takes no genotype 1a HCV is considering retreatment of
prescription or over the counter medications. His her HCV with Sofosbuvir/Velpatasvir. She is a
ultrasound shows no ductal dilatation, no stones relapser to prior therapy with peginterferon and
and no pancreatic masses. He is found by his pri- ribavirin. By fibrosure testing and imaging she is
mary care physician to have a positive HAV IgM. suspected to have well compensated cirrhosis, but
has never had a biopsy. Her MELD score is eight.
Which of the following is true regarding
Hepatitis A infection? Which of the following is true ?

A. Most Americans have been exposed to HAV A. If the patient’s HCV RNA is negative at week
by age five four, she is eligible for 8 weeks of therapy
B. HAV can be especially severe in pregnant B. Due to her cirrhosis, she should receive a total
women of 12 weeks of therapy
C. HAV has a human reservoir and thus spreads C. Because of her cirrhosis, her cure rate is
rapidly in overcrowded living situations estimated at less than 90 percent
D. The risk of fulminant liver disease is higher D. Therapy at this point is inappropriate and
with acute HAV infection compared to acute the patient should be evaluated for liver
HBV infection transplantation
E. HAV does not cause chronic liver injury and
is not associated with chronic disease CORRECT ANSWER: B

CORRECT ANSWER: A RATIONALE


Sofosbuvir/Velpatasvir combination therapy has
RATIONALE a cure rate of 99 percent in treatment experienced
Hepatitis A is spread via the fecal oral route and genotype 1 patients with well compensated cir-
gains access to the host by ingestion of contami- rhosis when given for 12 weeks as shown by the
nated food or water. In countries with poor ASTRAL-1 trial. Shorter duration of therapy is
sanitation, most children are exposed to HAV reserved for patients without cirrhosis and treat-
by age five. In contrast only about 10 percent of ment duration is not impacted by HCV RNA moni-
children and 33 percent of adults in the United toring. The only regimen approved for shorter
States have been exposed to HAV. HAV has no duration of therapy in treatment experienced
human host and does not cause chronic disease. patients is daily fixed-dose combination of gleca-
Hepatitis E is spread by similar contamination previr/pibrentasvir. As she has well compensated
and is known to be severe in pregnant women. cirrhosis with a low MELD score, cure may reduce
Approximately 0.1 percent of HAV infections the need for future transplant.
Chapter 5 — Viral hepatitis 125

REFERENCES and thus reduce risk of transmission to her baby.


https://www.hcvguidelines.org/treatment-experi- Tenofovir is appropriate selection as viral kinetics
enced/gt1a/p-r/compensated-cirrhosis will rapidly reduce the HBV DNA and the tenofo-
Feld JJ, Jacobson IM, Hézode C, Asselah T, vir compounds are pregnancy class B. Entecavir
Ruane PJ, Gruener N, et al.. Sofosbuvir and also has rapid viral kinetics and is pregnancy class
Velpatasvir for HCV Genotype 1, 2, 4, 5, and 6 C. Interferon therapy is not recommended in preg-
Infection. The New England Journal of Medicine. nant women. Hepatitis B Immune Globulin and
2015;373(27):2599 - 2607. HBV vaccination is the correct intervention for the
baby once delivered, but will have no impact on the
mother or fetus at this point. Screening programs
Question 9 for HCC are not recommended until age 50 for
A 24-year-old woman is 34 weeks pregnant and non-cirrhotic Asian females with chronic HBV.
seeking prenatal care. She has one child with chron-
ic HBV. This is her second pregnancy and it has been REFERENCE
uneventful thus far. She emigrated from mainland Pan CQ, Duan ZP, Bhamidimarri KR, Zou HB,
China 1 year ago. Her blood work is below. Liang XF, Li J, Tong MJ. An algorithm for risk
assessment and intervention of mother to child
HBsAg (+) transmission of hepatitis B virus. Clin Gastroen-
HBsAb (-) terol Hepatol 2012;10:452-9.
HBeAg (+)
HBeAb(-)
HBV DNA 300,000,000 copies Question 10
AST 19 U/L A 50-year-old Caucasian male is sent to you after
ALT 29 U/L blood work from an insurance physical reveals ab-
Total bilirubin 1.0 mg/dl normal LFT’s. He has diabetes, hypertension and
INR 1.0 has a BMI of 34. He does not have any symptoms
Albumin 3.5 g/dl of liver disease and does not drink alcohol or use
any illicit drugs. He takes metformin, hydrochlo-
What is the next appropriate step? rothiazide and metoprolol.

A. Initiate tenofovir therapy Total Bilirubin 1.0 mg/dL


B. Initiate pegylated interferon alfa 2b therapy AST 44 U/L
C. Watchful waiting, monitor liver enzymes ALT 85 U/L
D. Hepatitis B Immune Globulin and Alkaline Phosphatase 120 IU/L
HBV vaccination for mother INR 1.0
E. Screen for hepatocellular carcinoma HCV Ab (-)
HBs Ag (-)
CORRECT ANSWER: A HBsAb (+)
HBc Total (+)
RATIONALE HBeAg (-)
This woman has chronic hepatitis B likely ac- HBeAb (+)
quired at birth from vertical transmission as she
is from an endemic area of the world. She has a Which of the following is true?
high viral load and is in the third trimester of her
pregnancy, thus it is appropriate to initiate an A. The patient has surreptitious alcohol abuse
oral nucleoside/tide drug to reduce the viral load B. The lab abnormalities are from chronic hepatitis B
126 Digestive Diseases Self-Education Program®

C. The patient has been vaccinated against HBV HAV total (+)
D. A liver biopsy would show interface hepatitis HCV Ab (-)
and plasma cells Anti nuclear antibody (-)
E. A liver biopsy would show ballooning Anti Actin (smooth muscle)antibody (-)
hepatocytes and steatosis Fasting bile salts normal

CORRECT ANSWER: E Which of the following is true?

RATIONALE A. This illness is unlikely to have a severe course


The description of the above patient is consistent in pregnant women
with non-alcoholic fatty liver. The patient’s age B. Hepatitis E IgM will likely be positive
and comorbidities increase the likelihood that he C. This patient will like suffer a relapsing-
has NASH whose histologic description is given remitting course prior to clearing the virus
in answer E. The HBV serologies are consistent D. This illness was preventable with proper
with prior exposure and natural immunity rather vaccination
than vaccination and should not be confused with E. This illness is spread by exposure to
chronic disease. Plasma cell interface hepatitis is infected blood products
more consistent with autoimmune hepatitis and
less likely in this case. The ALT over AST pattern CORRECT ANSWER: B
of the LFTs is more consistent with a non-alcohol-
ic fatty liver. RATIONALE
Hepatitis E (HEV) is an acute self limited viral
REFERENCES hepatitis that is similar in presentation to hepatitis
Ryder SD, Beckingham IJ. ABC of diseases of A. The virus is endemic to India and Mexico and
liver, pancreas, and biliary system: Chronic viral is spread by the fecal-oral route usually through
hepatitis. BMJ 2001;322:219-21 contaminated food or water supplies. HEV is
more likely to have a severe or even fulminant
presentation in pregnant women and this risk
Question 11 increases with each trimester. While protective
A 25-year-old American woman notices the onset antibodies form after an infection, there is cur-
of fatigue, diarrhea and scleral icterus. She is in rently no commercially available vaccine in the
the second trimester of her first pregnancy and United States. Relapsing cholestatic hepatitis is
has no significant past medical history. She takes a rare variant of Hepatitis A, but is not seen with
prenatal vitamins only and has received routine Hepatitis E.
prenatal care. Four weeks ago, she returned from
a trip to India. She knows of no sick contacts. Her REFERENCE
initial blood work is below. Kamar N, Bendall R, Legrand-Abravanel F, Xia
NS, Ijaz S, Izopet J, Dalton HR. Hepatitis E. Lan-
Total bilirubin 6.2 mg/dL cet 2012;379:2477-88.
AST 2300 U/L
ALT 3400 U/L
INR 1.8 Question 12
HBsAg (-) A 21-year-old Caucasian male presents to the
HBsAb (+) emergency room due to abdominal pain, fatigue
HBc IgM (-) and loss of appetite. He drinks one or two beers
HAV IgM (-) per day but also abuses prescription opiates and
Chapter 5 — Viral hepatitis 127

intravenous heroin. His last needle use was seven tis. BMJ 2001;322:151-3.
days ago. On physical exam, he has mild sclera
icterus, tender hepatomegaly and needle tracks in
both antecubital fossa. His blood work is below. Question 13
A 19-year-old college student has had a 10 day
Total Bilirubin 5.1 mg/dL history of nausea, vomiting, diarrhea and fatigue.
AST 950 U/L She recently returned from a two week trip to
ALT 1300 U/L Haiti for volunteer work. On exam, she has a low
Alkaline phosphatase 115 IU/L grade fever (380C) tender hepatomegaly and mild
Albumin 3.4 scleral icterus. Two of her classmates have had
HBsAg (-) similar symptoms. Her blood work is below.
HBsAb (+)
HBc IgM (-) Total Bilirubin 4.8 mg/dl
HAV IgM (-) AST 1250 U/L
HCV Ab (-) ALT 1800 U/L
Alkaline phosphatase 99 IU/L
What lab test is most likely to make a diagnosis? INR 0.9

A. Antimitochondrial and anti smooth Which of the following laboratory tests is most
muscle antibodies likely to reveal the diagnosis?
B. HCV RNA
C. HCV RIBA A. HAV IgM
D. HBc IgG B. HAV total
E. HAV total C. CMV stool PCR
D. HBsAb
CORRECT ANSWER: B E. HCV Ab

RATIONALE CORRECT ANSWER: A


This patient has the signs, symptoms and serum
chemistries consistent with an acute hepatitis. The RATIONALE
initial serologic evaluation shows negative viral This patient has the presentation of an acute
serologies and immunity to HBV, but in light of hepatitis and has recently traveled to an endemic
the active injection drug use, acute HCV must be region for HAV where sanitation remains poor.
considered as the HCV Ab may take up to six weeks The HAV total will only reveal prior infection and
to develop. HCV RIBA is used to confirm a positive immunity to HAV. Stool CMV PCR is unlikely as
HCV Ab if there is concern for a false positive test this patient has no obvious history of immunosup-
or prior exposure with spontaneous viral clearance; pression. HbsAb would describe HBV immunity
this test is also no longer recommended by the CDC rather than acute hepatitis and given her age, she
and due to a shortage of reagents is no longer avail- was likely vaccinated at birth. The patient has no
able in many laboratories. HAV total and HBc IgG clear risk factors for acute HCV exposure such as
would signal prior exposure to HAV and HBV re- needlestick injury or injection drug use.
spectively and are not consistent with acute disease.
REFERENCES
REFERENCES Ryder SD, Beckingham IJ. ABC of diseases of
Ryder SD, Beckingham IJ. ABC of diseases of liver, pancreas, and biliary system: Acute hepati-
liver, pancreas, and biliary system: Acute hepati- tis. BMJ 2001;322:151-3.
128 Digestive Diseases Self-Education Program®

Question 14 is referred from student health for hepatitis B.


A 63-year-old woman undergoes cancer chemo- She has never had any symptoms of her hepatitis
therapy for lymphoma that includes rituximab and B and denies jaundice, abdominal pain, ascites
prednisone. Four weeks after her initial round of and GI bleeding. Her physical exam is normal.
treatment, she develops jaundice, encephalopathy Below is her laboratory evaluation.
and coagulopathy and ultimately dies.
HBsAg (+)
Pretreatment screening and prophylaxis for which HBsAb (-)
medical problem may have prevented this compli- HBeAg (+)
cation? HBeAb (-)
HBV DNA 180,000 IU/mL
A. Prior HAV exposure ALT 15 U/L
B. Prior HBV exposure Total Bilirubin 0.9 mg/dl
C. Prior HCV exposure
D. UGT1A1 genotype What is the best description of this woman’s
E. History of prior Drug Induced Liver Injury hepatitis B status?

CORRECT ANSWER: B A. Immune tolerant phase


B. Immune clearance/HBeAg (+) chronic
RATIONALE hepatitis B
All patients who receive immunosuppressant C. Inactive carrier state
medications should be screened for chronic D. pre-core mutant chronic hepatitis
hepatitis B. The combination of rituximab and E. Natural immunity via past exposure
prednisone is particularly high risk for induc-
ing a HBV flare or even reverse seroconversion, CORRECT ANSWER: A
among those who have been exposed in the past,
due to the combination of both B and T cell sup- RATIONALE
pression. The fact that this patient was from This young patient likely acquired her hepati-
an endemic area for HBV should only reinforce tis B via vertical transmission at birth as she is
the need for screening. Prior HAV exposure from an endemic area. Often these patients will
would render immunity. Immunosuppression in have a high HBV DNA for the first 20-30 years
the setting of chronic HCV may cause the HCV of life with normal LFTs in what is described
RNA to increase, but fulminant flares are not as the immune tolerant phase. There is active
described. The UGT1A1 genotype is the genetic viral replication occurring and thus they have a
test for Gilbert’s syndrome and is important for positive e antigen as well. During the immune
patients who receive iranotecan based chemo- clearance phase, there is often a sharp spike in
therapy. Drug induced liver injury is thought to the ALT and the HBV DNA may decline. There
be idiopathic and prophylaxis is not available. can be a seroconversion from HBeAg to HBeAb.
The inactive carrier state occurs after the sero-
REFERENCES conversion to HBeAb and is notable for low DNA
Hoofnagle JH. Reactivation of hepatitis B. Hepa- levels and relatively normal LFT’s. HBeAg (-)
tology 2009;49:S156-65. chronic hepatitis, AKA, pre-core mutant hepatitis
occurs usually in older patients when the virus
is no longer able to produce HBeAg, however,
Question 15 these patients may also have more advanced liver
A 22-year-old female graduate student from Korea disease as well.
Chapter 5 — Viral hepatitis 129

REFERENCE HBsAg (+)


Yim HJ, Lok AS. Natural history of chronic HBeAg(+)
hepatitis B virus infection: what we knew in HBV DNA 10,000 IU/mL
1981 and what we know in 2005. Hepatology HBV genotype: A
2006;43:S173-81. ALT 340 U/L
AST 277 U/L
Total bilirubin 1.0 mg/dl
Question 16 INR 1.0
The student described in the preceeding vingette Albumin 3.9 g/dl
inquires about the long term management of her
disease. What is the next appropriate step in Which treatment option offers the best chance for
managing this patient’s disease? HBsAb seroconversion?

A. Begin entecavir 0.5 mg daily A. Lamivudine


B. Begin peginterferaon alfa 2b 180mcg weekly B. Entecavir
C. Monitoring of LFT’s and HBV DNA q six months C. Lamivudine + Adefovir
D. Liver Biopsy D. peginterferon alfa 2a
E. Initiate HCC screening E. Tenofovir

CORRECT ANSWER: C CORRECT ANSWER: D

RATIONALE RATIONALE
This patient has an HBV DNA over 20,000 IU/ Oral nucleoside/tide agents are very effective for
mL but normal LFT’s and thus qulaifies for moni- viral suppression, but have a low rate of HBeAg
toring only. If her ALT was abnormal or if the seroconversion. Interferon given over the course
HBV DNA were several logs higher, a liver biopsy of a year gives the best opportunity for HBeAg
would be a reasonable intervention, although seroconversion and HBsAg loss in this patient
monitoring is still appropriate. She will need HCC who has genotype A infection, high ALT and low
surveillence at age 50 since she is from Asia even viral loads.
in the absence of cirrhosis.
REFERENCE
REFERENCE Yim HJ, Lok AS. Natural history of chronic hepatitis
Terrault NA, Lok SF, McMahon BJ, et al. B virus infection: what we knew in 1981 and what we
Update on preveention, diagnosis, and treatment know in 2005. Hepatology 2006;43:S173-81.
of chronic hepatitis B: AASLD 2018 hepatitis B
guidance. Hepatology 2018;67:1560-1599.
Question 18
A patient with biopsy proven NASH returns to
Question 17 clinic for routine care. She has normal synthetic
A 43-year-old caucasian male with chronic function and is gradually improving her weight
hepatitis B presents to the office. He is and diabetes control. She has never been exposed
asymptommatic from his disease and he has to viral hepatitis.
not had any decompensation events including
jaundice, encephalopathy or variceal bleed- Which of the following viruses cannot currently be
ing. He is naïve to HBV therapy as well. Here prevented by vaccination?
are his labs:
130 Digestive Diseases Self-Education Program®

A. Hepatitis A gies are at increased risk to develop acute liver


B. Hepatitis B failure if they develop acute HAV infection.
C. Hepatitis C It is therefore recommended that all patients
D. Hepatitis D with chronic HCV be vaccinated against HAV.
E. Hepatitis E Pregnant women are at higher risk of fulminant
disease from hepatitis E infection, but not A.
CORRECT ANSWER: C Children are routinely exposed to HAV and are
not at greater risk of a fulminant course.
RATIONALE
Hepatitis A and Hepatitis B have long been REFERENCE
preventable via vaccination and are frequently Pham H, Geraci SA, Burton MJ. Adult immuni-
offered as a combination vaccine. Hepatitis D is zations: update on recommendations. Am J Med
preventable through vaccination against hepatitis 2011;124:698-701.
B. There is a vaccine for hepatitis E that is safe
and efficacious, but it is not available in the United
States. Hepatitis C remains the lone hepatotrophic Question 20
virus that is not preventable through vaccination. One of your patients, a 25-year-old male gradu-
ate student ate at the suspicious restaurant from
REFERENCES the previous question. He develops mild icterus,
Hagan LM, Schinazi RF. Best strategies for global nausea, vomiting and diarrhea two weeks after his
HCV eradication. Liver Int 2013;33 Suppl 1:68-79. exposure there. On laboratory testing, his HAV
Ryder SD, Beckingham IJ. ABC of diseases of IgM is positive. His three roommates, none of
liver, pancreas, and biliary system: Chronic viral whom ate at the restaurant, are all asymptomatic.
hepatitis. BMJ 2001;322:219-21. After reporting the findings to the health depart-
ment, what other interventions are appropriate?

Question 19 A. Administer HAV immune globulin to any


The county health department reports an out- unvaccinated contact
break of hepatitis A traceable to a local restau- B. Offer ribavirin prophylaxis to roommates
rant with a past record of health code violations. C. HAV vaccination to all roommates
Several patients report to your office concerned D. Quarantine the index patient
about exposure. Which patients are at increased E. No other intervention necessary
risk to develop fulminant hepatic failure follow-
ing acute hepatitis A infection? CORRECT ANSWER: A

A. Patients with chronic hepatitis C virus infection RATIONALE


B. Patients with HBsAb (+) and HB core total (+) Hepatitis A immune globulin should be con-
C. Children exposed to hepatitis A sidered for household contacts of patients with
D. Patients with chronic renal failure acute hepatitis A. When administered within
E. Pregnant women two weeks, immune globulin is 80-90 percent
effective in preventing spread of HAV infection
CORRECT ANSWER: A to close contacts. If administered later, im-
mune globulinin may not prevent HAV, but can
RATIONALE attenuate the symptoms. Vaccination is appro-
Patients with chronic HCV and likely also pa- priate in the long term, but will not prevent the
tients with chronic liver disease of other etiolo- spread of this current outbreak.
Chapter 5 — Viral hepatitis 131

REFERENCES HBsAg (+)


Centers for Disease Control and Prevention. HBeAg (-)
Prevention of hepatitis A through active or pas- HBeAb (+)
sive immunization. Recommendations of the HBV DNA 2000 IU/mL
Advisory Committee on Immunization Prac- Albumin 2.9 g/dl
tices (ACIP). MMWR Morb Mortal Wkly Rep INR 1.5
2006;55:1–23. Total Bilirubin 5.1 mg/dl
HBV genotype B

Question 21 Which of the following interventions is


A 55-year-old Caucasian male presents to his not recommended?
primary care physician for an annual physical.
He is in good health, but is found to have an ALT A. EGD to screen for varices
of 90 U/L and a skin rash diagnosed by his der- B. Ultrasound to screen for hepatocellular
matologist as porphyria cutanea tarda. The PCP carcinoma
calls you for advice about the elevated ALT. C. Start peginterferon alfa
Which of the following blood tests is likely to D. Start oral nucleoside (entecavir or tenofovir)
explain these findings? E. Screen relatives for HBV

A. Hepatitis A IgM CORRECT ANSWER: C


B. Hepatitis B surface antibody
C. Hepatitis C antibody RATIONALE
D. Hepatitis Delta Antibody The patient presented has hepatitis B induced
E. Hepatitis E antibody cirrhosis and appears to be decompensated
with Child’s B cirrhosis by his exam and labora-
CORRECT ANSWER: C tory values. As he has detectable HBV DNA, he
requires therapy to prevent a flare of his disease
RATIONALE and further transplant evaluation, but the use
Hepatitis C can be associated with a variety of of interferon is contraindicated in this setting,
dermatologic findings including PCT. In addi- thus he should receive an oral agent. EGD and
tion to phlebotomy, HCV therapy may result in ultrasound screening are recommended inter-
the resolution of the skin finding. ventions for patients with cirrhosis as is screen-
ing family members for HBV and vaccinating as
REFERENCES appropriate.
Berk DR, Mallory SB, Keeffe EB, Ahmed A.
Dermatologic disorders associated with chronic REFERENCE
hepatitis C: effect of interferon therapy. Clin Yim HJ, Lok AS. Natural history of chronic
Gastroenterol Hepatol 2007;5:142-51. hepatitis B virus infection: what we knew in
1981 and what we know in 2005. Hepatology
2006;43:S173-81.
Question 22
A 55-year-old Asian male presents to your office
with the complaint for abdominal distention. Question 23
On exam he has mild scleral icterus, spider an- A 60-year-old patient with Crohn’s disease is in
giomata across the chest and a fluid wave across need of biologic therapy with infliximab. Pre-
the abdomen. Here are his labs: infusion testing shows the following labs:
132 Digestive Diseases Self-Education Program®

ALT 19 U/L Question 24


HAV total Ab positive A 60-year-old patient with Crohn’s disease is
HBs Ab positive in need of biologic therapy with infliximab.
HBs Ag negative Pre infusion testing shows the following labs:
Hbc total positive
HCV Ab negative ALT 19 U/L
HAV total Ab positive
The patient is concerned about her prior HBV HBs Ab negative
exposure. What is the most appropriate action? HBs Ag positive
HBe Ag negative
A. Prophylaxis against reverse seroconversion HBe Ab positive
with entecavir 0.5mg daily Hbc total positive
B. Treatment of chronic hepatitis B with entecavir HBV DNA 400 IU
0.5mg daily HCV Ab negative
C. No action required
D. Booster vaccination against HBV The patient is concerned about her prior HBV
E. Check and follow HBV DNA levels exposure. What is the most appropriate action?

CORRECT ANSWER: C A. Prophylaxis against reverse seroconversion


with entecavir 0.5mg daily
RATIONALE B. Treatment of chronic hepatitis B with entecavir
This patient has been exposed to HBV in the past 0.5mg daily
and like 95 percent of adults has cleared the virus. C. No action required due to inactive carrier state
She does not have chronic HBV as demonstrated by (HBe antibody positive, normal ALT, low viral
a negative surface antigen. Under bimodal B and T load)
cell immunosuppression, patients who have been D. Booster vaccination against HBV
exposed to HBV are at risk of reverse seroconver- E. Check and follow HBV DNA levels
sion and do require prophylaxis. TNF-a drugs, like
infliximab, do not meet this threshold and can be CORRECT ANSWER: B
used safely in a patient who has been previously ex-
posed to HBV. The risk of HBV reverse seroconver- RATIONALE
sion among patients like this with anti TNF therapy This patient has chronic hepatitis B. While they
is low and was initially thought to be ~ 1.7 percent. are in the inactive carrier state (as demonstrated
More recent data suggests that the risk is even by the positive envelope antibody, low viral load
lower. A 2018 study showed 0/178 patients with and normal ALT), the addition of a biologic medi-
reverse seroconversion when on anti TNF drugs. cation, like infliximab, an anti TNF-α drug, puts
the patient at risk of HBV reactivation. Nearly 40
REFERENCES percent of patient who revived long term TNF-α
Pauly MP, Tucker LY, Szpakowski JL, Ready antagonist had a reactivation of previously quies-
JB, Baer D, Hwang J, Lok AS. Incidence of cent HBV.
Hepatitis B Virus Reactivation and Hepatotox-
icity in Patients Receiving Long-term Treat- REFERENCE
ment With Tumor Necrosis Factor Antagonists., Pauly MP, Tucker LY, Szpakowski JL, Ready
Clin Gastroenterol Hepatol. 2018 Apr 24. JB, Baer D, Hwang J, Lok AS. Incidence of
pii: S1542-3565(18)30394-X. doi: 10.1016/j. Hepatitis B Virus Reactivation and Hepatotox-
cgh.2018.04.033. icity in Patients Receiving Long-term Treat-
Chapter 5 — Viral hepatitis 133

ment With Tumor Necrosis Factor Antagonists., have HBV ccc DNA within their hepatocytes that
Clin Gastroenterol Hepatol. 2018 Apr 24. is dormant, but under extreme combined B and
pii: S1542-3565(18)30394-X. doi: 10.1016/j. T cell immunosuppression, the patients are at
cgh.2018.04.033. risk for reverse seroconversion where they can
lose HBV surface Ab and manifest HBV surface
antigen and present as an acute HBV infection.
Question 25 Prophylaxis is required during therapy and for at
A 45-year-old man has recently been diagnosed least 12-18 months after therapy due to the long
with leukemia. The chemotherapeutic regimen lasting effects of anti-B cell monoclonal antibod-
will include rituximab and high dose steroids. ies like rituximab. Reactivation of HCV in HCV
He is a former IV drug user but has been sober Ab positive, RNA negative patients has not been
for 20 years. His lab work is below. reported.

ALT 25 U/l REFERENCE


HAV total antibody positive Pauly MP, Tucker LY, Szpakowski JL, Ready
HBs antibody positive JB, Baer D, Hwang J, Lok AS. Incidence of
HBs antigen negative Hepatitis B Virus Reactivation and Hepatotox-
HBc total positive icity in Patients Receiving Long-term Treat-
HCV Antibody positive ment With Tumor Necrosis Factor Antagonists.,
HCV RNA undetected Clin Gastroenterol Hepatol. 2018 Apr 24.
pii: S1542-3565(18)30394-X. doi: 10.1016/j.
Which of the following is true? cgh.2018.04.033.

A. The patient is at risk of HCV reactivation


due to prior exposure Question 26
B. The patient should receive prophylaxis A 35-year-old woman with treatment naïve chron-
against HBV reverse seroconversion through ic hepatitis C genotype 1a infection is seen in your
his chemotherapy office for treatment of her hepatitis C infection.
C. The patient should receive prophylaxis against She has a remote history of IVDU. Her physical
HBV reverse seroconversion through his exam is unremarkable. Laboratories demonstrate
chemotherapy and for 12-18 months afterwards a normal CBC, ALT 112 U/L, AST 75 U/L, total
D. The HBV reactivation is not a concern in bilirubin 1.0 mg/dL, creatinine 0.6 mg/dL. Her
this patient HIV Ab is negative, HBsAg is positive, HBcAb is
E. A watchful waiting approach is reasonable positive, HBsAb is negative. Her hepatitis C viral
with q 3 month monitoring of the ALT and load is 600,000 IU/ml. A transient elastography
HBV DNA. is notable for stage 1 fibrosis.

CORRECT ANSWER: C What is the most appropriate next step in the


management of this patient?
RATIONALE
This patient has been exposed to HBV in the past A. Sofosbuvir/velpatasvir
and has cleared the virus. The HBVcore total Ab B. Sofosbuvir/ledipasvir
is indicative of prior exposure while the HBV sur- C. HBV DNA
face Ab is detectable and gives immunity against D. Glecaprevir/pibrentasvir
re-infection under most routine clinical scenarios.
Patients who have been exposed to HBV still CORRECT ANSWER: C
134 Digestive Diseases Self-Education Program®

RATIONALE Question 27
Prior to initiating therapy for chronic hepatitis C, A 52-year-old male is seen in your clinic for evalu-
the patient needs to be fully evaluated for chronic ation of newly diagnosed cirrhosis in the setting of
hepatitis B infection. The patient has a positive alcohol abuse and hepatitis C genotype 1b infec-
surface antigen and core antibody with a negative tion. His PMH is notable for a hiatal hernia and
surface antibody which is consistent with chronic persistent gastroesophageal reflux despite abstain-
hepatitis B infection. An HBV DNA level is neces- ing from alcohol over last three months. He has
sary to determine whether the patient has chronic mild esophageal dysphagia and is currently taking
active hepatitis B infection or inactive chronic high dose PPI therapy twice a day.
hepatitis B. HBV/HCV coinfected patients can
develop reactivation or a flare of chronic hepatitis The optimal treatment strategy for this patient’s
B during or after treatment with DAA therapy. If hepatitis C infection at this time would be which of
the HBV DNA level is greater than 20,000 IU/mL, the following regimens?
the patient requires antiviral therapy for hepatitis
B concurrently with DAA therapy through SVR12. A. Sofosbuvir/ledipasvir
Patients with low levels of HBV DNA or an unde- B. Sofosbuvir/velpatasvir
tectable level can either receive prophylactic treat- C. Grazoprevir/elbasvir
ment for HBV or be followed at regular intervals D. Sofosbuvir/ribavirin
during DAA therapy for HBV reactivation. Treat-
ment should be initiated if the HBV DNA levels CORRECT ANSWER: C
increase greater than 10-fold or over 1000 IU/mL
from an undetectable level at baseline. There is RATIONALE
insufficient evidence to recommend HBV antiviral The solubility of the NS5A inhibitors, velpatasvir
therapy in patients with a negative surface anti- and ledipasvir decreases as pH increases, so PPI
gen, but positive core antibody. therapy is expected to decrease the concentra-
The other three choices would be acceptable tion of both drugs and can impact the efficacy of
treatment options for this patient’s chronic therapy with those regimens. Coadministration
hepatitis C genotype 1a infection but withholding of velpatasvir with proton pump inhibitors is not
treatment until the hepatitis B is completely recommended.
evaluated is obligatory. Sofosbuvir/ledipasvir can be taken simultane-
ously under fasted conditions with any PPI at a
REFERENCES dose comparable to omeprazole 20 mg or lower.
https://www.hcvguidelines.org/ Antacids should be avoided within four hours of
Chen G, Wang C, Chen J, Ji D, Wang Y, Wu V, sofosbuvir/ledipasviror sofosbuvir/velpatasvir
et al. Hepatitis B reactivation in hepatitis B and C administration. H2 receptor blockers should be
coinfected patients treated with antiviral agents: A taken simultaneously or 12 hours apart.
systematic review and meta-analysis. Hepatology. Since this patient has significant acid reflux
2017;66(1):13-26. symptoms despite high dose PPI therapy, grazo-
Wang C, Ji D, Chen J, Shao Q, Li B, Liu J, et previr/elbasvir, which does not have a PPI interac-
al. Hepatitis due to Reactivation of Hepatitis B tion would be an ideal option for this patient with
Virus in Endemic Areas Among Patients With compensated cirrhosis. NS5A RAV testing would
Hepatitis C Treated With Direct-acting Antiviral be required for genotype 1a patients, but not
Agents. Clinical gastroenterology and hepatol- genotype 1b patients treated with this regimen.
ogy : the official clinical practice journal of the The SVR12 rate was 97 percent in the subgroup of
American Gastroenterological Association. patients with genotype 1b and stage 4 fibrosis in
2017;15(1):132-6. the C-EDGE trial.
Chapter 5 — Viral hepatitis 135

REFERENCES C with prior treatment failure after DAA therapy


American Association for the Study of Liver based on the POLARIS-1 trial. The SVR12 in
Diseases & Infectious Diseases Society of America genotype 1 patients treated with this combination
(2018). HCV Guidance: Recommendations for was 97 percent. Compensated cirrhotic patients
Testing, Managing, and Treating Hepatitis C. Re- can be treated with this regimen as well. Baseline
trieved from http://hcvguidelines.org/ NS5A RAV testing prior to initiating therapy is not
Zeuzem S, Ghalib R, Reddy KR, Pockros PJ, required.
Ben Ari Z, Zhao Y, et al. Grazoprevir-Elbasvir
Combination Therapy for Treatment-Naive Cir- REFERENCE
rhotic and Noncirrhotic Patients With Chronic Bourliere M, Gordon SC, Flamm SL, Cooper CL,
Hepatitis C Virus Genotype 1, 4, or 6 Infection: A Ramji A, Tong M, et al. Sofosbuvir, Velpatasvir,
Randomized Trial. Annals of internal medicine. and Voxilaprevir for Previously Treated HCV
2015;163(1):1-13. Infection. The New England journal of medicine.
Package inserts for Sofosbuvir/ledipasvir and 2017;376(22):2134-46.
Sofosbuvir/velpatasvir

Question 29
Question 28 A 42-year-old woman with pulmonary sarcoidosis
A 45-year-old woman with genotype 1a HCV is hospitalized for shortness of breath and a flare
presents for re-evaluation. She is treatment expe- of her sarcoidosis. She has been started on 60 mg
rienced and did not achieve an SVR after complet- of prednisone. An astute medical resident taking
ing eight weeks of sofosbuvir/ledipasvir therapy care of the patient sees prior hepatitis serologies
12 months ago. A transient elastography in the notable for HBsAg negative, anti-HBc positive.
office today suggests stage 3 fibrosis. The resident sends an HBV DNA level which is un-
detectable. The pulmonologist anticipates at least
What is the best next step? six weeks of prednisone therapy with a dose 20 mg
or more daily. As the consulting gastroenterolo-
A. Sofosbuvir/velpatasvir/voxilaprevir gist, what is the best next step in management?
B. NS5A RAV testing
C. Sofosbuvir/daclatasvir A. Re-check HBV DNA in three months
D. Sofosbuvir/ledipasvir for 24 weeks B. Start tenofovir
C. Observation
CORRECT ANSWER: A D. Hepatitis B vaccination

RATIONALE CORRECT ANSWER: B


This patient has prior exposure to DAA therapy,
namely an NS5B (sofosbuvir) and an NS5A RATIONALE
(ledipasvir) combination. While the emergence The American Gastroenterology Association
of NS5A RAVs may have resulted in treatment guidelines state that patients who have HBsAg-
failure with sofosbuvir/ledipasvir, the virus negative/anti-HBc+ serologic profile who are being
reverts to wild type after therapy is discontinued treated with prednisone more than 20 mg daily
and re-challenging this patients with another (or equivalent) for four weeks or more or are on
NS5A containing regimen is recommended with chronic moderate dose prednisone therapy (10-20
the addition of an NS3B/4A protease inhibitor. 12 mg daily) are at moderate risk of reactivation one
weeks of sofosbuvir/velpatasvir/voxilaprevir was to 10 percent and require hepatitis B prophylaxis
approved for patients with genotypes 1-6 hepatitis during treatment and for at least six months after
136 Digestive Diseases Self-Education Program®

stopping immunosuppression. Patients who are A. Alpha fetoprotein every six months
HBsAg- negative/anti-HBc+ are likely to have a B. Liver ultrasound and AFP every six months
lower reactivation rate than their counterparts C. No surveillance
with HBsAg+/anti-HBc+ serostatus but given the D. Liver CT every 12 months
paucity of data in this patient group, prophylaxis
is recommended. This approach is preferred over CORRECT ANSWER: B
monitoring except in situations where the patient
“places a higher value on avoiding long- term use RATIONALE
of antiviral therapy and the cost associated with its The annual incidence of hepatocellular carcinoma
use and a lower value on avoiding the small risk of in patients with hepatitis C and cirrhosis is be-
reactivation (particularly in those who are HBsAg tween one to five percent. Hepatocellular carci-
negative) may reasonably select no prophylaxis noma surveillance is recommended for patients
over antiviral prophylaxis.” with hepatitis C and cirrhosis. Although this
Patients who are HBsAg+ and are treated with patient does not have clinical symptoms of cirrho-
moderate (10-20 mg) or high (over 20 mg) of pred- sis, the stage 4 fibrosis as estimated by transient
nisone for four weeks or more are at high risk for elastography in addition to the mildly reduced
reactivation (over 10 percent) and require HBV an- platelet count make HCC screening prudent for
tiviral prophylaxis with continuation of therapy for this patient. The AGA recommends a VCTE cut-
at least six months after discontinuation of therapy. off of 12.5 kPa to detect cirrhosis in patients with
chronic hepatitis C.
REFERENCE The primary modality for screening is liver ul-
Reddy KR, Beavers KL, Hammond SP, et al. trasound at an interval of every six months. Alpha
American Gastroenterological Association Insti- fetoprotein testing alone is not recommended for
tute Guideline on the Prevention and Treatment screening by the AASLD due to lack of sensitiv-
of Hepatitis B Virus Reactivation During Immu- ity and specificity. However, the updated AASLD
nosuppressive Drug Therapy. Gastroenterology HCC guidelines provide a recommendation that
2015;148(1):215-19. u/s with or without the addition of AFP testing
may be used for screening owing to recent data
that AFP may increase the sensitivity of screening
Question 30 when combined with u/s.
A 62-year-old man presents for follow-up of The risk of HCC is considerably reduced in pa-
chronic hepatitis C infection. Prior to initiating tients achieving SVR after DAA therapy. Among
therapy with sofosbuvir/ledipasvir, a transient 22,500 patients treated with DAA (19,518 with
elastography demonstrated stage 4 fibrosis with SVR; 2982 without SVR), without SVR, those
a score of 14.0 kPa. 12 weeks after completing with SVR had a significantly reduced risk of HCC
treatment, he now has an undetectable HCV RNA. (0.90 vs 3.45 HCC/100 person-years; adjusted
He has no symptoms of cirrhosis and continues hazard ratio, 0.28, 95 percent CI=0.22-0.36). The
to work full-time. His physical examination is risk of HCC, though attenuated, does persist in
normal. His ultrasound shows a heterogeneous patients with cirrhosis after their hepatitis C has
appearing liver with top-normal spleen size. been eradicated. Ongoing HCC surveillance in this
Labs are as follows: population with liver ultrasound every six months
is the best answer.
PLT 130,000, bilirubin 1.0 mg/dL, INR 1.0
creatinine 0.9 mg/dL, AST: 35 U/L, ALT: 30 U/L. REFERENCES
Which of the following is the best approach for Van der Meer AJ, Veldt BJ, Feld JJ, et al. Associa-
monitoring this patient in the future? tion between sustained virological response and
Chapter 5 — Viral hepatitis 137

all-cause mortality among patients with chronic the diagnosis of mixed cryoglobulinemia, which
hepatitis C and advanced hepatic fibrosis. JAMA. will typically present with purpura, arthralgias,
2012;308:2584-2593. and weakness along with active sediment on the
Lim JK, Flamm SL, Singh S, Falck-Ytter YT, urinalysis and renal insufficiency. Mixed cryo-
Clinical Guidelines Committee of the American globulinemia can be cured by eradicating hepatitis
Gastroenterological A. American Gastroenterolog- C in most individuals. A minority of patients can
ical Association Institute Guideline on the Role of have persistent mixed cryoglobulinemia despite
Elastography in the Evaluation of Liver Fibrosis. eradication of HCV, due to the development of
Gastroenterology. 2017;152(6):1536-43. monoclonal B-cell populations. In the absence
Kanwal F, Kramer J, Asch SM, Chayanupat- of positive cryoglobulins or findings to support
kul M, Cao Y, El-Serag HB. Risk of Hepatocel- a diagnosis of MPGN, further evaluation with
lular Cancer in HCV Patients Treated With rheumatoid factor and complement testing is not
Direct-Acting Antiviral Agents. Gastroenterology. necessary.
2017;153(4):996-1005 e1. Chronic hepatitis C is independently associated
with the development of chronic kidney disease,
with a 51 percent increase in the risk of protein-
Question 31 uria and 43 percent increase in the incidence of
A 55-year-old woman with treatment naïve CKD. This patient’s renal dysfunction could be at-
chronic hepatitis C genotype 2 infection is seen tributed to chronic hepatitis C infection and war-
for consultation. Her blood pressure is 140/90 rants further evaluation by nephrology. However,
and her physical exam is notable for pre-tibial IV rituximab would be an appropriate option only
edema, but there are no skin findings. Labora- for a patient with severe mixed cryoglobulinemia
tories demonstrate a normal CBC, ALT 50 U/L, with significant end-organ damage, such as acute
AST 47 U/L, total bilirubin 1.0 mg/dL. Creatinine renal failure requiring hemodialysis.
has been stable at 1.5 mg/dL over the last year. A The most appropriate next step in the man-
urinalysis is negative for RBCs, but notable for 2+ agement of this patient is treatment of the
albuminuria. Serum cryoglobulins are negative. hepatitis C. Glecaprevir/pibrentasvir treatment
A transient elastography demonstrates stage 2 for eight weeks would be indicated for genotype 2
fibrosis. and stage 2 fibrosis. Sofosbuvir/ledipasvir is not
indicated for the treatment of genotype 2 chronic
What is the most appropriate next step in the hepatitis C infection.
management of this patient?
REFERENCES
A. Check rheumatoid factor, C3, C4 Dammacco F, Sansonno D. Therapy for hepatitis C
B. Sofosbuvir/ledipasvir treatment for virus-related cryoglobulinemic vasculitis. N Engl J
12 weeks Med. 2013;369(11):1035-45.
C. Referral to nephrology for initiation of Fabrizi F, Verdesca S, Messa P, Martin P.
IV rituximab Hepatitis C virus infection increases the risk
D. Glecaprevir/pibrentasvir treatment of developing chronic kidney disease: a sys-
for eight weeks tematic review and meta-analysis. Dig Dis Sci.
2015;60(12):3801-3813.
CORRECT ANSWER: D Emery JS, Kuczynski M, La D, Almarzooqi
S, Kowgier M, Shah H, et al. Efficacy and Safety
RATIONALE of Direct Acting Antivirals for the Treatment of
The patient has chronic hepatitis C without Mixed Cryoglobulinemia. The American journal of
physical exam or laboratory findings to support gastroenterology. 2017;112(8):1298-308.
138 Digestive Diseases Self-Education Program®

Question 32 with normal renal function.


A 55-year-old African American man with a histo- Ledipasvir pharmacokinetics are not significantly
ry of hypertension, diabetes, and stage IV chronic affected by renal impairment. Ob- servation would
kidney disease (eGFR=25 mL/min) is referred not be appropriate for this patient given the stage
to you by his primary care provider for evalua- 2 disease.
tion and management of newly diagnosed chronic Grazoprevir/elbasvir is an equally satisfactory
hepatitis C infection, genotype 3. A transient treatment in chronic hepatitis C patients with
elastography demonstrates stage 2 fibrosis. CKD stage 4 or 5 and on hemodialysis. However,
this regimen is only effective in genotypes 1a, 1b,
What is the most appropriate treatment choice and 4. In the C-SURFER study, this regimen
for this patient? resulted in a 99 percent sustained virologic re-
sponse rate in patients with eGFR less than 30ml/
A. Sofosbuvir/ledipasvir min/1.73m2, including those on dialysis.
B. Grazoprevir/elbasvir
C. Glecaprevir/pibrentasvir REFERENCES
D. Observation American Association for the Study of Liver
Diseases & Infectious Diseases Society of America
CORRECT ANSWER: C (2018). HCV Guidance: Recommendations for
Testing, Managing, and Treating Hepatitis C. Re-
RATIONALE trieved from http://hcvguidelines.org/
The most appropriate treatment choice for this Gane E, Lawitz E, Pugatch D, Papatheodoridis
patient is glecaprevir/pibrentasvir. This com- G, Brau N, Brown A, et al. Glecaprevir and Pibren-
bination is approved for use in patients with tasvir in Patients with HCV and Severe Renal Im-
CKD stage 4 or 5 (eGFR less than 30 mL/min pairment. The New England journal of medicine.
or ESRD). In the EXPEDITION-4 trial, the 2017;377(15):1448-55.
pangenotypic NS3/NS4A protease inhibitor Roth D, Nelson DR, Bruchfeld A et al.
glecaprevir and the pangenotypic NS5A inhibitor Grazoprevir plus elbasvir in treatment-naïve
pibrentasvir was studied for 12 weeks in patients and treatment- experienced patients with
with advanced CKD (stage 4 or 5) and included hepatitis C virus genotype 1 infection and stage
patients on hemodialysis. 19 percent patients 4-5 chronic kidney disease (the C_SURFER
had compensated cirrhosis and 40 percent were study): a combination phase 3 study. Lancet
treatment experienced. The SVR12 in the modi- 2015;386(10003):1537-45.
fied intention to treat analysis was 100 percent.
The duration of therapy of treatment with gle-
caprevir/pibrentasvir is the same as that recom- Question 33
mended for patients without CKD. Thus, the A 58-year-old woman is admitted to your hospital
patient in the case vignette should receive eight with acute variceal hemorrhage and a new diag-
weeks of therapy. nosis of decompensated cirrhosis. She underwent
Sofosbuvir should not be used in patients with successful band ligation therapy and has been
eGFR less than 30 mL/min/1.73m2 due to lack transferred out of the intensive care unit to the floor.
of safety data in this population. The primary Evaluation for the etiology of her cirrhosis is notable
metabolite of sofosbuvir, namely GS-331007, is for genotype 1b chronic hepatitis C infection. On the
primarily eliminated via the kidney. In patients floor, she has persistent large volume ascites, despite
with eGFR less than 30 mL/min/1.73m2 and a therapeutic paracentesis which was negative
for those on dialysis, the metabolite GS 331007 for SBP. Her MELD-Na score is 28 with a serum
increases significantly when compared to patients creatinine which has increased from 0.8 mg/dL on
Chapter 5 — Viral hepatitis 139

admission to 1.8 mg/dL today in spite of 1 g/kg of treatment in decompensated patients since gleca-
albumin administration over the last 48 hrs. previr is a protease inhibitor. This antiviral drug
Which of the following would be the best manage- class is poorly tolerated in advanced cirrhosis due
ment strategy for this patient at this time? to risk of decompensation.

A. Sofosbuvir/velpatasvir REFERENCES
B. Evaluation for liver transplant Charlton M, Everson GT, Flamm SL, et al. Ledi-
C. Glecaprevir/pibrentasvir pasvir and Sofosbuvir Plus Ribavirin for Treatment
D. Sofosbuvir/ledipasvir and ribavirin of HCV Infection in Patients with Advanced Liver
Disease. Gastroenterology 2015;149(3):649-59.
CORRECT ANSWER: B Curry MP, O'Leary JG, Bzowej N, Muir AJ, Ko-
renblat KM, Fenkel JM, et al. Sofosbuvir and Vel-
RATIONALE patasvir for HCV in Patients with Decompensated
The best management strategy for this patient Cirrhosis. The New England journal of medicine.
would be evaluation for liver transplant. In a 2015;373(27):2618-28.
patient with a MELD score of 28 and concern for Chhatwal J, Samur S, Kues B, Ayer T, Roberts
evolving hepatorenal syndrome with a more than MS, Kanwal F, et al. Optimal timing of hepatitis C
doubling of the creatinine since admission, a liver treatment for patients on the liver transplant wait-
transplant evaluation is indicated. While treat- ing list. Hepatology. 2017;65(3):777-88.
ment with sofosbuvir/velpatasvir or sofosbuvir/
ledipasvir and ribavirin for 12 weeks are approved
therapies for decompensated cirrhosis based on Question 34
data from ASTRAL-4 and SOLAR-1 and SO- A 40-year-old Caucasian female with a history of
LAR-2, immediate hepatitis C treatment for this allogeneic stem cell transplant for AML is referred
patient will likely be insufficient to avoid liver-re- to clinic for evaluation of possible hepatitis B
lated death or the need for liver transplantation. infection. She was recently hospitalized for treat-
The SOLAR-1 and SOLAR-2 trial demonstrated ment of pneumonia. As part of her evaluation in
improvement in MELD scores in the majority of the hospital, she was found to be hypogammaglob-
patients with genotype 1 or 4 HCV and cirrhosis ulinemic, and was treated for four days with 1 g/kg
(Child Pugh B and C) who underwent treatment intravenous immunoglobulin. Prior to discharge,
with 12 or 24 weeks of sofosbuvir/ledipasvir and she inadvertently has hepatitis B serologies
ribavirin. The sustained virologic response rates checked which reveal HBsAg-, HBsAb+, HBcAb+.
in SOLAR-1 and SOLAR-2 for Child-Turcotte- The pt. has not traveled outside the U.S. and
Pugh Class B patients was 85-87 percent after 12 does not have any remote or recent risk factors for
weeks of treatment with sofosbuvir/ledipasvir hepatitis B exposure. Prior to undergoing chemo-
and ribavirin. The ASTRAL-4 trial demonstrated therapy for her stem cell transplant, her hepatitis
the efficacy of sofosbuvir/velpatasvir treatment in B serologies were appropriately checked and at
genotype 1, 2, 3, 4, or 6 and decompensated cir- that time she was HBsAg-, HBsAb+, and HBcAb-.
rhosis with SVR rates of 83-86 percent in the 12 You send an HBV DNA which is undetectable.
or 24-week arms. Treatment of decompensated ALT is 15 U/L. Which of the following is the best
hepatitis C cirrhosis is recommended to be car- treatment recommendation for this patient?
ried out at a liver transplant center and patients
may improve enough to avoid liver transplan- A. Tenofovir
tation, though more data is needed to further B. Entecavir
understand this possibility. C. Hepatitis B vaccination
Glecaprevir/pibrentasvir is not approved for D. Reassurance
140 Digestive Diseases Self-Education Program®

CORRECT ANSWER: D baby, what is the most appropriate management


strategy?
RATIONALE
The patient was appropriately screened for hepa- A. Initiate ursodeoxycholic acid
titis B prior to beginning chemotherapy and had B. Start tenofovir now
evidence of immunity to hepatitis B with a positive C. Measure HBV DNA prior to the third trimester
sAb and a negative cAb. The current hepatitis D. Measure smooth muscle Ab
B AASLD guidance is that all patients undergo-
ing immunosuppressive and cytotoxic therapy CORRECT ANSWER: C
be screened for hepatitis B infection prior to the
initiation of therapy. RATIONALE
Treatment with IVIG is associated with passive The most appropriate management for this
exposure of hepatitis B core antibody from pooled patient is repeating her HBV DNA prior to the
plasma and the cAb would be expected to disap- third trimester. This patient is likely an inactive
pear from the patient’s serum over time. There is no carrier of chronic hepatitis B, given her ethnic
indication for antiviral therapy or vaccination in this heritage, normal LFTs and serum HBV DNA less
scenario. than 2,000 IU/mL. However, repeating her HBV
DNA prior to beginning the third trimester makes
REFERENCE sense to help establish that she is an inactive
Ramsay I, Gorton RL, Patel M, Workman S, carrier and does not meet criteria for antiviral
Symes A, Haque T, et al. Transmission of Hepati- therapy to prevent vertical transmission.
tis B Core Antibody and Galactomannan Enzyme Patients with high hepatitis B viral loads are at
Immunoassay Positivity via Immunoglobulin risk of vertical transmission of hepatitis B to the
Products: A Comprehensive Analysis. Clini- baby. In one study, the rate of vertical transmis-
cal infectious diseases : an official publication sion was 0 percent for mothers with an HBV DNA
of the Infectious Diseases Society of America. level less than 106 copies/mL (approximately
2016;63(1):57-63. 200,000 IU/mL), as compared to 7.6 percent
for levels over 108 copies/mL (over 2 X 107 IU/
mL). Therefore, it is recommended that mothers
Question 35 with an HBV DNA over 200,000 IU/ml at week
A 24-year-old woman originally from Ghana is 28, should be placed on oral antiviral therapy
admitted to the ob/gyn floor for management of to reduce viral load, and in turn, reduce the risk
hyperemesis gravidarum. She is pregnant with of vertical transmission. The available antiviral
her second child and the estimated gestational treatments with acceptable risk profiles for the
age of the fetus is eight weeks. She is receiving IV fetus are telbivudine (FDA pregnancy Class B),
fluids. Her exam is unremarkable. and tenofovir (FDA pregnancy Class B).
You are asked to see her for mildly elevated Ursodeoxycholic acid would be indicated for a
transaminases on admission, with an ALT of 60 pregnant woman with intrahepatic cholestasis of
U/L, AST of 40 U/L. The alkaline phosphatase pregnancy which presents in the second trimes-
and total bilirubin are normal. HCV Ab is nega- ter. This patient has normal canalicular enzymes.
tive, HBsAg+, HBsAb-, HBcAb+, HBV DNA is
500 IU/mL. REFERENCE
Her AST and ALT normalize the day following Zou H, Chen Y, Duan Z, et al. Virologic factors
admission following fluid resuscitation. associated with failure to passive-active immu-
In addition to immunization and provision of noprophylaxis in infants born to HBsAg-positive
hepatitis B immunoglobulin after delivery of the mothers. J Viral Hepat. 2012;19(2):18-25.
Chapter 5 — Viral hepatitis 141

Terrault NA, Lok ASF, McMahon BJ, Chang KM, C. Ledipasvir + sofosbuvir
Hwang JP, Jonas MM, et al. Update on preven- D. Elbasvir/grazoprevir
tion, diagnosis, and treatment of chronic hepatitis E. Glecaprevir/pibrentasvir
B: AASLD 2018 hepatitis B guidance. Hepatology.
2018;67(4):1560-99. CORRECT ANSWER: A
Wang L, Kourtis AP, Ellington S, et al. Safety
of tenofovir during pregnancy for the mother RATIONALE
and fetus: a systematic review. Clin Infect Dis. This patient has evidence of recent decompen-
2013;57(12):1773-81. sated cirrhosis. Treatment of chronic hepatitis C
has been shown to reduce morbidity and mortal-
ity, including in patients with decompensated
Question 36 cirrhosis. Current AASLD guidelines recommend
A 62-year-old male presents to your office for treatment with ledipasvir (90 mg)/sofosbuvir
follow-up of a recent hospitalization for hepatitis (400 mg) with ribavirin (600 mg, increased as
C cirrhosis complicated by new onset ascites. He tolerated) for 12 weeks. This recommendation is
underwent a paracentesis that was negative for based on data from the SOLAR-2 study, which
SBP and had 3L removed. A screening upper was comprised of patients with Child-Pugh class
endoscopy reveals small varices without high risk B and C cirrhosis, hemoglobin over 10.0 g/dL
features. On exam, he has mild muscle wasting, and creatinine clearance over 40 mL/min where
persistent ascites, but no asterixis. His family SVR rates were 87 percent and 86 percent in CTP
members have noted intermittent episodes of class B and CTP class C respectively. Ledipasvir
confusion recently, however. On further ques- and sofosbuvir alone would not be recommended
tioning, he states that he was treated with pe- in this decompensated patient, nor would it be
gylated-interferon and ribavirin many years ago, the correct therapy for a CTP class A, compen-
but his treatment was discontinued by his prior sated cirrhotic treatment experienced patient in
physician because of lack of reduction in his viral whom RBV is also recommended.
load on treatment.
Withholding therapy is not advisable for this re-
Laboratory findings: cently decompensated patient. Protease contain-
Hemoglobin 11.5 g/dL ing regimens: elbasvir/grazoprevir or glecaprevir/
Leukocyte count 6100/uL pibrentasvir are not recommended for patients
Platelets 85,000/uL with decompensated cirrhosis.
Creatinine 1.0 mg/dL
AST 75 U/L REFERENCES
ALT 78 U/L www.hcvguidelines.org/
Alkaline phosphatase 80 U/L Belli LS, Berenguer M, Cortesi PA, Strazzabos-
Total bilirubin 1.4 mg/dL co M, Rockenschaub SR, Martini S, et al. Delisting
INR: 1.3 of liver transplant candidates with chronic hepa-
titis C after viral eradication: A European study.
His HCV RNA is found to be 2,360,000 IU/mL, Journal of hepatology. 2016;65(3):524-31.
with an HCV genotype of 1A. Manns M, Samuel D, Gane EJ, Mutimer D, Mc-
Caughan G, Buti M, et al. Ledipasvir and sofosbuvir
Which is the best treatment option? plus ribavirin in patients with genotype 1 or 4 hepa-
titis C virus infection and advanced liver disease: a
A. Ledipasvir + sofosbuvir + ribavirin multicentre, open-label, randomised, phase 2 trial.
B. No therapy Lancet Infect Dis. 2016;16(6):685-97.
142 Digestive Diseases Self-Education Program®

Cheung MCM, Walker AJ, Hudson BE, Verma S, There is no compelling reason to switch to tenofovir
McLauchlan J, Mutimer DJ, et al. Outcomes after as the patient has excellent viral suppression. In
successful direct-acting antiviral therapy for pa- the absence of a hepatitis flare with increase in
tients with chronic hepatitis C and decompensated ALT, checking for HDV infection is not needed.
cirrhosis. Journal of hepatology. 2016;65(4):741-7.
REFERENCE
Terrault NA, Lok ASF, McMahon BJ, Chang KM,
Question 37 Hwang JP, Jonas MM, et al. Update on preven-
A 48-year-old Vietnamese man who has known tion, diagnosis, and treatment of chronic hepatitis
chronic hepatitis B returns for a follow-up visit. B: AASLD 2018 hepatitis B guidance. Hepatology.
He initiated entecavir therapy 3 years ago and at 2018;67(4):1560-99.
that time was sAg+ and eAg+ with an HBV DNA of
30,000 IU/ml and an ALT of 120 U/L.
Today, he feels well and has a normal physical Question 38
examination. His liver ultrasound shows a mildly A 25-year-old man presents to the emergency room
coarsened liver echotexture without focal liver with two weeks of abdominal pain, nausea, and
lesion and his transient elastography is consistent anorexia. His physical exam is notable for scleral
with stage 4 fibrosis. Laboratory studies today are icterus and mild tenderness to palpation in the right
notable for a normal CBC, ALT 25 U/L, AST 20 upper quadrant. He reports experimenting with IV
U/L, total bilirubin normal, sAg+, eAg-, eAb+, and drugs about one month ago but denies alcohol use.
undetectable HBV DNA. Abdominal ultrasound shows a normal contour of
the liver, no dilation of the bile ducts, and normal
What is the most appropriate treatment strategy? Doppler evaluation of the hepatic and portal vessels.

A. Discontinue entecavir Laboratory studies were as follows:


B. Switch entecavir to tenofovir
C. Check HDV Ab Hemoglobin 11.2 g/dL
D. Continue entecavir WBC count 6100/uL
Platelet count 202,000/uL
CORRECT ANSWER: D Creatinine 1.0 mg/dL
AST 335 U/L
RATIONALE ALT 405 U/L
This patient was started on therapy and at the time Alkaline phosphatase 80 U/L
had immune active e antigen positive chronic hepa- Total bilirubin 7.5 mg/dL
titis B. Treatment is recommended for patients who INR: 1.2
are e-antigen positive with an ALT greater than two
times the upper limit of normal and HBV DNA over Studies for viral hepatitis show a negative HBsAg, anti-
20,000 IU/ml. The patient now has excellent viral HBs, anti-HBc, anti-HAV and anti-HCV (by EIA).
suppression with normal ALT and has seroconver-
ted from eAg+ to eAb+. Patients who do not have What is the best next step in management?
cirrhosis who seroconvert should receive consolida-
tion therapy for at least 12 months prior to discon- A. Liver biopsy
tinuing antiviral therapy. Discontinuing entecavir B. Abdominal CT scan
therefore is incorrect for two reasons: 1) the patient C. Hepatitis C viral RNA
just seroconverted and 2) the patient has compen- D. EBV polymerase chain reaction
sated cirrhosis based on ultrasound and VCTE. E. Hepatitis BeAg and eAb testing
Chapter 5 — Viral hepatitis 143

CORRECT ANSWER: C CORRECT ANSWER: C

RATIONALE RATIONALE
This patient is at risk for acute hepatitis C due to This correct answer is C, liver ultrasound. Hepatitis
his recent initiation of IV drug abuse. In patients B is unique as a risk factor for hepatocellular carci-
with exposure less than six months, the hepatitis noma (HCC) in the absence of cirrhosis. The AASLD
C antibody may be negative and hepatitis C viral recommends HCC surveillance for black or Asian
RNA should be obtained. Although acute hepatitis men over 40 years, and Asian women over 50 years
might be due to atypical infections, such as EBV, with chronic hepatitis B regardless of fibrosis status.
the AST and ALT elevation in this patient would A liver ultrasound should be performed every six
be more consistent with acute hepatitis C, whereas months in these populations in addition to those
in acute EBV, one would expect the AST and ALT with cirrhosis secondary to hepatitis B. Alpha-
elevations to be lower and be preceded by a viral fetoprotein every six months is recommended for
prodrome with fever and sore throat. Evaluation patients where ultrasound is not readily available.
for the hepatitis BeAg and eAb is not indicated This patient is an inactive carrier and therefore
with negative surface antigen and core antibody does not require treatment.
testing. A liver biopsy and CT scan are reason-
able tests to investigate liver test abnormalities REFERENCES
but given this patient’s history of recent IV drug Terrault NA, Lok ASF, McMahon BJ, Chang KM,
abuse, acute hepatitis C should be ruled out first. Hwang JP, Jonas MM, et al. Update on preven-
tion, diagnosis, and treatment of chronic hepatitis
REFERENCES B: AASLD 2018 hepatitis B guidance. Hepatology.
Centers for Disease Control and Prevention 2018;67(4):1560-99.
(CDC). Testing for HCV infection: an update of Heimbach JK, Kulik LM, Finn RS, Sirlin CB,
guidance for clinicians and laboratorians. MMWR Abecassis MM, Roberts LR, et al. AASLD guide-
Morb Mortal Wkly Rep. 2013;62(18):362-5. lines for the treatment of hepatocellular carcino-
Blackard JT, Shata MT, Shire NJ, et al. Acute ma. Hepatology. 2018;67(1):358-80.
hepatitis C virus infection: a chronic problem.
Hepatology. 2008 Jan;47(1):321-31.
Question 40
A 40-year-old Chinese male presents to your office
Question 39 for evaluation of chronic hepatitis B. He acquired
A 44-year-old Cambodian male was diagnosed by his HBV from vertical transmission and has never been
primary care provider with chronic hepatitis B. He treated. During his last visit, a transient elastography
is referred for further evaluation and management. was performed which suggested stage 1 fibrosis. He
Examination is normal. Labs demonstrate a normal is asymptomatic, and his physical exam is normal.
complete blood count and aminotransferases. Hepa- He is hepatitis B eAg- and hepatitis B eAb+.
titis B tests are as follows: HBsAg positive, HBeAg The last two sets of laboratories are shown below.
negative, Anti-HBe positive, and HBV DNA 50 IU/ml.
What is the next best step in the management of Lab value 3 months prior Current
this patient? ALT 120 U/L 92 U/L
AST 75 U/L 105 U/L
A. Pegylated Interferon HBV DNA 2050 IU/mL 2725 IU/mL
B. Oral nuceleos(t)ide
C. Liver ultrasound What is the best next step in the management
D. Observation of this patient?
144 Digestive Diseases Self-Education Program®

A. Lamivudine tion, diagnosis, and treatment of chronic hepatitis


B. Tenofovir B: AASLD 2018 hepatitis B guidance. Hepatology.
C. Tenofovir/emtricitabine 2018;67(4):1560-99.
D. Observation

CORRECT ANSWER: B Question 41


A 32-year-old woman with chronic immune active
RATIONALE eAg+ chronic hepatitis B presents to your office.
This patient is in the HBeAg-negative Immune She was exposed through vertical transmission.
Reactivation Phase, also referred to as HBeAg- She has never required treatment for hepatitis B
negative chronic hepatitis. Approximately 20 before. She has no history of autoimmune or psy-
percent of patients who seroconvert from HBeAg chiatric disease. She works as a molecular biologist
to anti-HBe positive and up to 20 percent of inac- and is aware that her hepatitis B genotype is A. She
tive chronic hepatitis B patients enter this phase. is interested in pursuing the therapy that optimizes
Patients in the immune active chronic hepatitis B her chance to seroconvert from sAg+ to sAg-.
with ALT over 2 times the upper normal limit and
HBV DNA greater than 2000 IU/mL (e-antigen What is your treatment recommendation?
negative) or over 20,000 IU/mL (e-antigen posi-
tive) should be placed on anti-viral treatment. The A. Tenofovir
evidence for the ALT threshold more than two B. Adefovir
times upper normal limit is weak. Consideration C. Tenofovir and pegylated interferon
of the age of patient (over 40 years), degree of D. Lamivudine
fibrosis, and family history of HCC should factor
into the decision making for introduction of treat- CORRECT ANSWER: C
ment in patients with ALT values less than two
times upper normal limit. This patient certainly RATIONALE
meets treatment criteria and of the oral nucleos(t) Loss of Hepatitis B surface antigen is rarely
ides, tenofovir (or entecavir) are the best choices achieved with nucleot(s)ide therapy alone. How-
due to potency and minimal resistance. Oral ever, the addition of PEG-IFN to tenofovir therapy
antiviral treatment is continued indefinitely for has been studied with recent notable outcomes.
those in the HBeAg- negative Immune Reactiva- 48 weeks of TDF plus PEG-IFN was associated
tion Phase if the treatment is well tolerated. Loss with loss of HBsAg in 10.4 percent of patients
of HBsAg is a rare event in HBeAg- CHB and after 120 weeks of follow-up. In comparison, no
currently there is insufficient evidence to definitely patients seroconverted on TDF alone after 120
guide this scenario, though treatment discontinu- weeks of therapy. 10.4 percent is still a small
ation can be considered assuming close ongoing number, and recent AASLD guidelines do not sug-
follow-up if surface antigen clearance occurs. gest combination therapy. Pegylated interferon
monotherapy could also be considered, particu-
Lamivudine is a poor choice due to high resistance larly in genotype A and B HBV disease, but is not
potential. The combination of tenofovir/emtric- offered as a choice in this question. The decision
itabine would be recommended in a patient with to start a patient on PEG-IFN should be reserved
HBV/HIV coinfection. for physicians comfortable using this drug and
familiar with the side effects. It should be avoided
REFERENCES in patients with uncontrolled psychiatric disease,
Terrault NA, Lok ASF, McMahon BJ, Chang KM, a history of autoimmunity, cytopenias, cardiac dz,
Hwang JP, Jonas MM, et al. Update on preven- and decompensated cirrhosis. HBV genotypes A
Chapter 5 — Viral hepatitis 145

and B have a more favorable response to PEG-IFN What is the next best step in the management
in contrast to genotypes C and D. of this patient?

REFERENCES A. IV solumedrol
Ahn SH, Marcellin P, Ma X, Caruntu FA, B. IV acyclovir
Tak WY, Elkhashab M, et al. Hepatitis B C. IV N-acetylcysteine
Surface Antigen Loss with Tenofovir Disoproxil D. Ribavirin
Fumarate Plus Peginterferon Alfa-2a: Week
120 Analysis. Digestive diseases and sciences. CORRECT ANSWER: B
2018;63;3487-3497.
Marcellin P, Ahn SH, Ma X, Caruntu FA, Tak RATIONALE
WY, Elkashab M, et al. Combination of Tenofovir The next best step in the management of this
Disoproxil Fumarate and Peginterferon alpha-2a patient is IV acyclovir. The presentation of fe-
Increases Loss of Hepatitis B Surface Antigen in ver, altered mental status, right upper quadrant
Patients With Chronic Hepatitis B. Gastroenterol- pain with hepatomegaly, vital signs consistent
ogy. 2016;150(1):134-44 e10. with systemic inflammatory response syndrome,
leukopenia, high aminotransferases and coagu-
lopathy, collectively should raise concern for
Question 42 herpes hepatitis. A high index of suspicion is
A 22-year-old woman is admitted to the ICU with needed for this condition. In two studies analyz-
fevers, malaise, and mental status changes. Her ing case reports and cases series, the diagnosis
recent history is notable for delivering a healthy of herpes hepatitis was made post-mortem in 57
child two week prior to presentation. She has not percent and 77 percent of the cases, respectively.
taken any acetaminophen or new medications. The case fatality rate exceeds 80 percent in un-
She has no recent travel history. treated patients with progression to acute liver
Laboratory analysis demonstrates a white failure. A recent review of 20 confirmed/prob-
blood cell count of 2,200, platelet count 120,000, able cases by the ALFSG also report 60 percent
ALT 4000 U/L, AST 6500U/L, total bilirubin mortality at 21 days.
5mg/dL, direct bilirubin, 3.9 mg/dL, INR 2.2, and Intravenous acyclovir does improve survival,
creatinine 1.5 mg/dL. Ultrasound demonstrates and therefore, institution of IV acyclovir should
mild hepatomegaly, no features to suggest cirrho- not be delayed in situations where there is clinical
sis, and spleen size of 13 cm. A transjugular liver suspicion for herpes hepatitis. Patients at risk for
biopsy is performed as shown below. this condition are usually immunocompromised
or peripartum. Aminotransferase levels in the
1000s with disproportionately lower bilirubin,
leukopenia, and a septic-like presentation are
typical presenting features. Mucocutaneous or
genital lesions are present in the minority of cases.
The diagnosis can be confirmed with liver biopsy
demonstrating intranuclear nuclear inclusions,
multinucleated giant cells, and coagulative necro-
sis with minimal inflammation.
IV Solu-Medrol is a treatment for severe acute
autoimmune hepatitis. This case does not provide
information about auto-antibodies or gammaglob-
ulin levels, though acute severe presentation of au-
146 Digestive Diseases Self-Education Program®

toimmune hepatitis is frequently accompanied by On exam, he has a low-grade fever, icteric sclera
a lack of autoantibodies and normal gammaglobu- and is mildly tender to palpation in the RUQ. An
lin levels. The liver biopsy shown does not show u/s shows a heterogeneous liver echotexture with-
characteristic features of autoimmune hepatitis. out biliary dilation or gallstones.
Thus, IV solume- drol is incorrect. Laboratories studies are as follows.: hepatitis A
IV N-acetylcysteine is a treatment for acet- IgM negative, hepatitis B DNA negative, hepatitis
aminophen overdose which this patient does not C RNA negative, smooth muscle antibody nega-
have. A recent AGA guideline statement suggests tive, anti-nuclear antibody negative. What is the
that IV N-acetyl cysteine be used in non-acetamin- best treatment for this patient?
ophen ALF only in the context of a clinical trial.
In cases with an indeterminate cause in which A. Observation
acetaminophen could be involved, use of NAC B. Ribavirin
should be strongly considered. C. MRI/MRCP
Ribavirin is the treatment for patients with D. HIDA scan
chronic HEV infection which can occur in
immunosuppressed patients. HEV can cause CORRECT ANSWER: A
ALF in pregnant patients in endemic parts of
the world. RATIONALE
This patient has acute hepatitis E, genotype 3. This
REFERENCES is a zoonotic infection acquired in developed coun-
Norvell JP, Blei AT, Jovanovic BD, et al. Herpes try through exposure to infected animals such pigs,
simplex virus hepatitis: an analysis of the pub- boars, and deer. The presentation is that of an acute
lished literature and institutional cases. Liver viral hepatitis, and the treatment in an immunocom-
Transpl. 2007;13(10)1428-34. petent patient is supportive care. Men over 40 years
Flamm SL, Yang YX, Singh S, Falck-Ytter YT, old seem to be a group at higher risk for infection.
Committee AGAICG. American Gastroenterologi- In immunocompetent individuals, hepatitis E is
cal Association Institute Guidelines for the Di- generally a self-limited condition, but in solid-organ
agnosis and Management of Acute Liver Failure. transplant recipients, chronic infection can ensue.
Gastroenterology. 2017;152(3):644-7. Approximately 60 percent of transplant recipients
Little L, Rule J, Peng L, Gottfried M, Lee WM, who are infected with hepatitis E develop chronic
Acute Liver Failure Study G. Herpes Simplex infections. The best treatment for chronic hepatitis
Virus-Associated Acute Liver Failure Often Goes E in solid organ transplant recipients is ribavirin.
Unrecognized. Hepatology. 2018. In one study, the sustained virologic response rate
was 78 percent after a course of approximately three
Question 43 months of ribavirin.
A 64-year-old man is referred to your GI clinic for HEV RNA can be identified in serum or stool
evaluation of abnormal liver transaminases and for diagnosis of hepatitis E. However, HEV RNA
jaundice. He feels unwell and hasn’t been able to PCR is not readily available outside of research
keep food down for a week. His PMH is notable settings and therefore the CDC states that the
for diet-controlled diabetes and hypertension. He diagnosis can be confirmed only by testing for the
drinks a few beers on Sunday while watching foot- presence of antibody against HEV or HEV RNA.
ball but denies a history of chronic alcohol abuse. Providers must be aware of the possibility of false
He recently returned from a trip to Texas to a positive and negatives for HEV serologies.
gaming ranch where he and his friends hunted Although the pt. has RUQ pain and anorexia,
deer and wild boar. There are no recent antibiotic his u/s does not show any biliary dilation or gall-
exposures. stones. Further work-up for obstructive jaundice
Chapter 5 — Viral hepatitis 147

with MRCP or evaluation for biliary dyskinesia at toms of acute viral hepatitis with negative testing
this time is not warranted. for hepatitis B and hepatitis C and positive hepati-
tis A IgM Ab. There has been a recent increase in
REFERENCES hepatitis A infections. Infections in the U.S. occur
Kamar N, Izopet J, Tripon S, et al. Ribavirin for in patients who have traveled to another country
chronic hepatitis E virus infection in transplant where hepatitis A virus transmission is common
recipients. N Engl J Med. 2014;370(12):1111-20. and in sporadic outbreaks associated with con-
Dao Thi VL, Debing Y, Wu X, et al. Sofos- taminated uncooked foods. An important risk
buvir Inhibits Hepatitis E Virus Replication In group is MSM and injection drug users. A recent
Vitro and Results in an Additive Effect When outbreak in Tennessee has been associated with
Combined With Ribavirin. Gastroenterology. this demographic.
2016;150(1):82-5. Hepatitis A generally resolves over the course
https://www.cdc.gov/hepatitis/hev/hevfaq.htm of weeks and does not evolve into a chronic hepa-
titis. However, in approximately 10 percent of pa-
tients or less, a relapsing course can occur. About
Question 44 one-fifth of these individuals will have more than
A 25-year-old male presents to his local emergency one relapse. The relapse events are associated with
room with malaise, nausea, vomiting, abdominal similar symptoms that were present at the time of
pain, and yellowish discoloration of the skin. The the initial presentation but tend to be milder. The
patient endorses having same sex relationships for hepatitis A IgM remains detectable in those with
money so that he can afford to buy drugs. He has a relapsing course and the hepatitis A virus can be
been intermittently homeless over the last year. detected in the stool indicating the possibility for
Vital signs are within normal limits. On exami- infection transmission. Nearly all patients recover
nation, he has mild jaundice and no findings to completely over a six to 12-month period.
suggest chronic liver disease. Liver and spleen are Autoimmune hepatitis is less likely with this
not enlarged. presentation and the serum IgG is normal with
Lab results are as follows: ALT 850 U/L, AST only weakly positive smooth muscle Ab. Hepati-
700 U/L, total bilirubin 5mg/dL, direct biliru- tis E infection would be associated with zoonotic
bin 3.0 mg/dL, alkaline phosphatase 137 U/L. exposure or travel to an endemic area. While
Hepatitis A IgM positive, HBsAg negative, anti- EBV could be a consideration, the IgM viral
HBc IgM negative, hepatitis C antibody negative, capsid is negative and the pronounced transami-
cytomegalovirus IgM and IgG negative, EBV viral nitis is more than what one would see with EBV
capsid IgM negative, smooth muscle antibody+ hepatitis.
1:40, normal serum IgG.
REFERENCES
Which of the following is the most likely diagnosis? Bornstein JD, Byrd DE, Trotter JF. Relapsing
Hepatitis A: a case report and review of the litera-
A. Autoimmune hepatitis ture. J Clin Gastroenterol. 1999;28(4):355-6.
B. EBV https://www.tn.gov/health/cedep/tennessee-
C. Hepatitis E hepatitis-a-outbreak.html
D. Hepatitis A
Question 45
CORRECT ANSWER: D Which of the following is the most appropriate
management strategy for a healthy 50-year-old
RATIONALE woman whose son recently returned home from
Hepatitis A is responsible for this patient’s symp- Mexico with acute hepatitis A infection?
148 Digestive Diseases Self-Education Program®

A. Hepatitis A immune globulin 87(28/29):261–76.


B. Observation Fiore AE, Wasley A, Bell BP. Prevention of
C. Hepatitis A IgM Hepatitis A Through Active or Passive Immuniza-
D. Hepatitis A vaccine tion. Recommendations of the Advisory Commit-
tee on Immunization Practices (ACIP). MMWR.
CORRECT ANSWER: A 2006; 55(RR07);1-23.

RATIONALE
The current guidelines for post exposure pro- Question 46
phylaxis for hepatitis A infection vary by age and A 53-year-old woman status-post kidney trans-
health status. For healthy people over 12 months plant for FSGS 3 months ago is admitted to the
to 40 years old, hepatitis A vaccine is preferred hospital with anorexia and recent bouts of diar-
over IG due to likelihood of response, long-term rhea. She underwent induction with Campath
protection, and equivalent efficacy to IG. IG is and is currently taking tacrolimus and mycophe-
recommended within two weeks of exposure for nolate mofetil. You are asked to see her as the
persons over 40 years old due to lack of knowledge consulting gastroenterologist and arrange for her
on vaccine performance and possibility of more to have an EGD/flex sig and biopsies the following
severe presentation in patients with older age. IG day. CMV serostatus is donor positive and recipi-
is particularly important to give to immunosup- ent positive.
pressed patients exposed to hepatitis A given the
risk of fatal hepatitis A infection and reduced Laboratory findings:
response to vaccination in this population.
Hepatitis A vaccine is currently recommended Hemoglobin 9.0 g/dL
for all children at age one year. Patients with WBC count 2500/uL
chronic liver disease are at increased risk of mor- Platelet count 120,000/uL
bidity and mortality if they acquire acute hepatitis Creatinine 1.3 mg/dL
A infection. Thus, pre-exposure prophylaxis has AST 175 U/L
been recommended for patients with chronic liver ALT 122 U/L
disease who are susceptible to HAV. Alkaline phosphatase 150 U/L
Other high-risk groups include chronically Total bilirubin 0.9 mg/dL
immunosuppressed patients, those who travel to
endemic areas, working in occupations for which You suspect CMV disease with GI and liver
there is a high likelihood for exposure, family involvement. Which of the following treatments
members of infected patients, men who have sex would you recommend?
with men, HIV-positive patients, those with clotting
factor disorders, and those who use injection and A. Oral ganciclovir
non-injection illicit drugs. Hepatitis A is transmit- B. Intravenous ganciclovir
ted via the fecaloral route and parenteral transmis- C. Oral valacyclovir
sion is rare; thus, those on chronic hemodialysis are D. Intravenous acyclovir
not considered an especially high-risk group. E. Oral acyclovir

REFERENCES CORRECT ANSWER: B


https://www.cdc.gov/hepatitis/hav/havfaq.
htm#general RATIONALE
WHO position paper on hepatitis A vac- The above case demonstrates a post-transplant
cines– June 2012. Wkly Epidemiol Rec. 2012; immunosuppressed patient with CMV hepatitis
Chapter 5 — Viral hepatitis 149

with concurrent GI involvement. Liver biopsies of In addition to recommending that he abstain from
CMV would be notable for the presence of inclu- alcohol, what is the best treatment option?
sion bodies, resembling an owl’s eye, and a small
foci of mononuclear cell infiltrates and microab- A. Observation
scesses. B. Pegylated Interferon
Intravenous ganciclovir, at 5 mg/kg twice daily, C. Telbivudine
or oral valganciclovir, 900 mg orally twice daily, D. Entecavir
are the treatment of choice for CMV hepatitis and/
or GI CMV. Both valganciclovir and intravenous CORRECT ANSWER: D
ganciclovir have been demonstrated to be effica-
cious for the treatment of mild to moderate CMV RATIONALE
disease in solid organ transplant recipients. Oral The AASLD recommends that adult patients with
ganciclovir has poor oral availability (7 percent) compensated cirrhosis and low-level viremia (less
compared to oral valgancyclovir (70 percent), than 2,000 IU/mL) be treated with antiviral therapy
and therefore is not recommended. Acyclovir and to reduce the risk of decompensation, regardless of
vala- cyclovir have weak activity against CMV ALT level. The preferred treatment regimens are
and should not be used for the treatment of CMV entecavir or tenofovir. Optimal duration of therapy
disease. is not currently known. In a large open label trial
where patients were treated for 240 weeks with te-
REFERENCES nofovir, at the end of the study 71 of the 96 patients
Asberg A, Humar A, Rollag H, et al. Oral valganci- with cirrhosis (74 percent) had regression of cirrho-
clovir is noninferior to intravenous ganciclovir for sis. Therefore, some compensated patients may not
the treatment of cytomegalovirus disease in solid only be protected from flares of active hepatitis but
organ transplant recipients. Am J Transplant. may also experience improvement in their histol-
2007;7:2106–13. ogy and therefore observation is not appropriate.
Kotton CN, Kumar D, Caliendo AM, et al.. Inter- Pegylated interferon should generally be avoided in
national consensus guidelines on the management cirrhosis due to risk of decompensation. Telbivudine
of cytomegalovirus in solid organ transplantation. has an approximately 20 percent risk of resistance at
Transplantation. 2010;89:779–795. two years and therefore is not the best oral agent.

REFERENCES
Question 47 Marcellin P, Gane R, Buti M, et al. Regres-
A 48-year-old male establishes care in your clinic sion of cirrhosis during treatment with teno-
for management of chronic hepatitis B. He is fovir disoproxil fumarate for chronic hepatitis
treatment naïve, only recently obtained insur- B: a 5-year open-label follow-up study. Lancet
ance, but now wants to be sure he has appropriate 2013;381(9865):468-75.
follow-up. He has hypertension and drinks two Terrault NA, Lok ASF, McMahon BJ, Chang
to three beers each night after work. His physical KM, Hwang JP, Jonas MM, et al. Update on
exam is unremarkable. He undergoes a transient prevention, diagnosis, and treatment of chronic
elastography in the office demonstrating stiffness hepatitis B: AASLD 2018 hepatitis B guidance.
values that would be consistent with cirrhosis. Hepatology. 2018;67(4):1560-99.
Notable lab abnormalities include platelet
count 108,000, ALT 40 U/L, AST 32 U/L, T Bili
1.0 mg/dL, creatinine 0.8 mg/dL, HBsAg posi- Question 48
tive, HBeAg negative, Anti-HBe positive, HBV A 35-year-old woman is one month post-op from
DNA 1000 IU/ml. a mastectomy for breast cancer. She received
150 Digestive Diseases Self-Education Program®

neoadjuvant chemotherapy with docetaxel, doxo- A palliative care consult is therefore inappropriate.
rubicin, cyclophosphamide, and trastuzumab. She A liver biopsy with her coagulopathy and obvious
is admitted to her local hospital with new onset of HBV diagnosis is incorrect. HBV antiviral therapy
jaundice, anorexia, and abdominal pain. could be started but is not associated with a mor-
Notable lab abnormalities include ALT 1200 tality benefit in patients with ALF due to HBV.
U/L, AST 1100 U/L, T Bili 25.0 mg/dL, creatinine
1.4 mg/dL, INR: 2.2, HBcIgM positive, HBsAg REFERENCES
positive, HBV DNA over 160,000,000. Karvellas CJ, Cardoso FS, Gottfried M, Reddy
You are the on-call gastroenterologist at a ter- KR, Hanje AJ, Ganger D, et al. HBV-Associated
tiary care facility 60 miles from the local hospital Acute Liver Failure After Immunosuppression
and you are called by the admitting hospitalist for and Risk of Death. Clinical gastroenterology and
your recommendation. He says she is oriented x hepatology : the official clinical practice journal
3, but she seems to be a little bit sleepy. of the American Gastroenterological Association.
2017;15(1):113-22.
What is your advice? Reddy KR, Beavers KL, Hammond SP, Lim
JK, Falck-Ytter YT, American Gastroenterological
A. Palliative care consult Association I. American Gastroenterological As-
B. Liver biopsy sociation Institute guideline on the prevention and
C. Tenofovir treatment of hepatitis B virus reactivation during
D. Transfer for liver transplant evaluation immunosuppressive drug therapy. Gastroenterol-
ogy. 2015;148(1):215-9; quiz e16-7
CORRECT ANSWER: D

RATIONALE Question 49
This patient is presenting with acute reactivation A 65-year-old male with genotype 1a HCV pres-
hepatitis B infection and acute liver failure. She re- ents for evaluation. He is treatment naïve and a
ceived high dose chemotherapy which would have recent transient elastography reveals stage 2 fibro-
placed her at high risk (more than 10 percent) for sis. He had a stroke six months ago in the context
reactivation of hepatitis B infection from previ- of atrial fibrillation, but fortunately has regained
ously undiagnosed chronic hepatitis B. Prior to most of his strength back and is independent with
receiving chemotherapy, this patient should have all ADLs. Medications include amiodarone, lisino-
been screened for occult hepatitis B infection, and pril, and atorvastatin. Which regimen should be
provided immunoprophylaxis if found to be sAg+ used to treat this patient’s hepatitis C?
or even if previously exposed to hepatitis B (sAg-/
cAb+/sAb+). The coagulopathy and signs of grade A. Glecaprevir/pibrentasvir
I hepatic encephalopathy are consistent with acute B. Sofosbuvir/ledipasvir
liver failure. Patients with ALF from HBV infec- C. Sofosbuvir/velpatasvir
tion who are immunosuppressed compared to pa- D. Sofosbuvir/velpatasvir
tients who are not had worse overall survival at 21 E. No treatment since the patient has had a stroke
days, in a retrospective review of 156 cases of HBV
ALF from the ALFSG (43 percent vs. 63 percent). CORRECT ANSWER: A
Patient survival from breast cancer is excellent and
this patient should absolutely be considered for liv- RATIONALE
er transplant even with a recent cancer diagnosis, Glecaprevir/ pibrentasvir should be used for this
while additional data regarding her cancer staging patient due to the interaction between sofosbuvir
and risk of recurrence of malignancy are assessed. containing regimens and amiodarone.
Chapter 5 — Viral hepatitis 151

The FDA issued a post-marketing warning tient’s hepatitis C?


about potential for interaction between sofosbuvir
and amiodarone. Nine patients taking sofosbu- A. Glecaprevir/ pibrentasvir
vir (with other antiviral agents) and amiodarone B. Sofosbuvir/ledipasvir
developed significant bradycardia. Seven patients C. Sofosbuvir/velpatasvir
were on concomitant beta-blockade. One patient D. Elbasvir/grazoprevir
died of a cardiac arrest whilst three others re-
quired pacemaker placement. Two-thirds of the CORRECT ANSWER: C
events occurred within 24 hours of co-administra-
tion while the other third occurred within 12 days. RATIONALE
Three patients had recurrence of bradycardia The patient has a history of CAD. The co-admin-
with rechallenge of sofosbuvir treatment while on istration of HMG-CoA Reductase inhibitors with
amiodarone. The mechanism of bradycardia is not glecaprevir/ pibrentasvir is not recommended
fully understood. Amiodarone is considered an ab- due to an increased risk of rhabdomyolysis from
solute contraindication to the use of a sofobsuvir- increased statin concentrations. It is not rec-
containing regimen. ommended that a patient with CAD stop statin
There is no reason to withhold therapy because therapy, particularly when other treatments are
of this patient’s prior stroke as he has recovered available. Any decision to stop statin therapy
significantly. On the other hand, if the pt. had should be undertaken with the prescribing physi-
persistent hemiparesis and aphasia with a high cian. Sofosbuvir/velpatasvir would be an ap-
risk of mortality in one year due to complications propriate choice for this patient with genotype 3
from his stroke than it is certainly reasonable to chronic hepatitis C. Monitoring of the patient’s
not treat this patient. HR on atenololol is prudent to ensure bradycardia
doesn’t develop.
REFERENCES
U.S. Food and Drug Administration. FDA Drug REFERENCE
Safety Communication: FDA warns of serious slow- Glecaprevir/ pibrentasvir package insert
ing of the heart rate when anti-arrhythmic drug
amiodarone is used with hepatitis C treatments
containing sofosbuvir (Harvoni) or Sovaldi in com-
bination with another Direct Acting Antiviral drug.
U.S. Food and Drug Administration 2015.
Fontaine H, Lazarus A, Pol S, et al. Bradyar-
rhythmias associated with Sofobuvir Treatment. N
Eng J Med. 2015; 5: 373 (19): 1886-8

Question 50
A 48-year-old male with genotype 3 HCV presents
for evaluation. He is treatment naïve. A recent
transient elastography reveals stage 3 fibrosis. He
is a heavy smoker and had an acute myocardial
infarction one year ago with drug-eluting stent
placement. Medications include atenolol, aspirin,
lisinopril, and atorvastatin.
Which regimen should be used to treat this pa-
152 Digestive Diseases Self-Education Program®
Answers & critiques

CHAPTER 6

Metabolic, hereditary,
inflammatory and vascular
diseases of the liver
Kiran Bambha, MD, MSc and Robert C. Lowe, MD

Question 1 B. Lobular inflammation, steatosis,


A 48-year-old woman is referred to you for evalu- centrilobular hepatocyte necrosis
ation of a two-year history of unexplained elevated C. Hepatocyte ballooning, lobular
liver enzymes. Her AST is 67 U/L and ALT is 83 inflammation, steatosis
U/L. Total bilirubin is 0.8 mg/dL, alkaline phos- D. Steatosis, Mallory hyaline, portal
phatase is 99 U/L and serum albumin is 3.9 g/dL. lymphocytic inflammation
Her CBC is normal. Further testing reveals the fol-
lowing: HBsAg negative, HBsAb positive, HBcAb CORRECT ANSWER: C
total negative, HCV Ab negative, anti-nuclear Ab
negative, anti-smooth muscle Ab negative, anti- RATIONALE
mitochondrial Ab negative. Her iron studies are The definitive diagnosis of nonalcoholic steatohepa-
normal. She is alpha-one-antitrypsin phenotype titis (NASH) remains a histologic diagnosis. The key
MM. Ultrasound of the liver reveals an enlarged histologic features for a diagnosis of NASH include:
and echogenic liver with patent portal and hepatic hepatic steatosis, lobular inflammation and bal-
veins, and no radiographic signs of portal hyper- looned hepatocytes. Portal inflammation may some-
tension or cirrhosis. Her physical exam is notable times be seen, particularly in young patients with
for a well-appearing woman with abdominal obe- NASH. Although Mallory hyaline may be seen in
sity and acanthosis nigricans of the posterior neck. NASH, it is not necessary for the histologic diagnosis.
Her BMI is 37 kg/m2. Her past medical history
includes the following diagnoses: type 2 diabetes REFERENCES
mellitus, hypertension, polycystic ovaries and Chalasani N, Younossi Z, Lavine JE, et al. The
obstructive sleep apnea. Her medications include Diagnosis and Management of Nonalcoholic
insulin and amlodipine. She has no past or current Fatty Liver Disease: Practice Guidance from
history of alcohol use. You discuss with the patient the American Association for the Study of Liver
your recommendations for proceeding with a liver Disease. Hepatology 2018;67:328-57.
biopsy for diagnostic purposes. Which of the fol- Yeh MM, Brunt EM. Pathological features of
lowing are histologic diagnostic features of nonal- fatty liver disease. Gastroenterology 2014;147:754-64.
coholic steatohepatitis (NASH) in adults? Kleiner DE, Brunt EM, Van Natta M, et al.
Design and validation of a histological scor-
A. Portal lymphocytic infiltrate, steatosis, ing system for nonalcoholic fatty liver disease.
endotheliitis Hepatology 2005;41:1313-21.

153
154 Digestive Diseases Self-Education Program®

Question 2 steatohepatitis. New England Journal of Medicine


You are evaluating a 55-year-old woman with a 2010;362:1675-85.
long-standing history of elevated liver enzymes
(ALT 57 U/L and AST 46 U/L). An extensive
serologic workup was negative and imaging was Question 3
notable for an echogenic and enlarged liver with You recently diagnosed a 66-year-old man with cir-
patent hepatic and portal veins. The patient does rhosis due nonalcoholic steatohepatitis. The patient
not have a history of alcohol excess. Over the presents to your clinic now inquiring about his long-
course of the evaluation you ordered a liver biopsy term prognosis. Which of the following is the most
that revealed histologic features typical of nonal- common cause of long-term mortality among patients
coholic steatohepatitis, with stage 1 fibrosis. The with nonalcoholic steatohepatitis (NASH) cirrhosis?
patient presents now for her biopsy results. She
would like to know what treatments are available A. Colon cancer
for her NASH. Which of the following statements B. Cardiovascular disease
is true regarding the management of nonalcoholic C. Hepatic failure
steatohepatitis (NASH)? D. Infection

A. Vitamin E has not been demonstrated to im CORRECT ANSWER: B


prove fibrosis in NASH.
B. Metformin improves inflammation and bal RATIONALE
looning in NASH. The leading cause of death in patients with NASH
C. Ursodeoxycholic acid improves steatosis and is cardiovascular disease. Death from liver-related
inflammation in NASH. causes is much more common in NASH than in
D. Iron depletion through phlebotomy improves the general population, but is not the leading
inflammation and fibrosis in NASH. cause of death. Cancer-related death is among the
top three causes of death in patients with NASH,
CORRECT ANSWER: A but is not the most common.

RATIONALE REFERENCES
In a large randomized clinical trial of adults with Adams LA, Lymp JF, St Sauver J, et al. The natural
biopsy-proven nonalcoholic steatohepatitis (NASH), history of nonalcoholic fatty liver disease: a population-
vitamin E demonstrated efficacy in improving ste- based cohort study. Gastroenterology 2005;129:113-21.
atosis and inflammation, but was not associated with Chalasani N, Younossi Z, Lavine JE, et al. The
improvements in hepatic fibrosis. Metformin, ursode- Diagnosis and Management of Nonalcoholic Fatty
oxycholic acid and phlebotomy have all been inves- Liver Disease: Practice Guidance from the Ameri-
tigated as therapeutic agents in NASH and have not can Association for the Study of Liver Disease.
demonstrated efficacy in improving NASH histology. Hepatology 2018;67:328-57.

REFERENCES
Chalasani N, Younossi Z, Lavine JE, et al. The Question 4
Diagnosis and Management of Nonalcoholic Fatty Which of the following conditions is associated
Liver Disease: Practice Guidance from the Ameri- with increased risk for nonalcoholic fatty liver
can Association for the Study of Liver Disease. disease (NAFLD)?
Hepatology 2018;67:328-57.
Sanyal AJ, Chalasani N, Kowdley KV, et al. Pi- A. Alopecia areata
oglitazone, vitamin E, or placebo for nonalcoholic B. Psoriasis
Chapter 6 — Metabolic, hereditary, inflammatory and vascular diseases of the liver 155

C. Sarcoidosis with autoimmune-type features. Drugs that may be


D. Lupus associated with microvesicular steatosis include:
aspirin, valproate, nucleoside reverse transcriptase
CORRECT ANSWER: B inhibitors, cocaine and tetracycline. Acute fatty liver
of pregnancy, not intrahepatic cholestasis of preg-
RATIONALE nancy, is associated with microvesicular steatosis.
Psoriasis has been demonstrated in epidemiologic
studies to be associated with NAFLD and may be REFERENCES
associated with increased risk for hepatic fibrosis. Bernstein DL, Hülkova H, Bialer MG, Desnick RJ.
Alopecia areata, sarcoidosis and lupus have not Cholesteryl ester storage disease: review of the find-
been associated with NAFLD. ings in 135 reported patients with an underdiagnosed
disease. Journal of Hepatology 2013;58:1230-43.
REFERENCES Robert J. Fontana. Pathogenesis of Idiosyn-
Chalasani N, Younossi Z, Lavine JE, et al. The cratic Drug-Induced Liver Injury and Clinical
Diagnosis and Management of Nonalcoholic Fatty Perspectives. Gastroenterology 2014; 146:914–28.
Liver Disease: Practice Guidance from the Ameri-
can Association for the Study of Liver Disease.
Hepatology 2018;67:328-57. Question 6
Miele L, Vallone S, Cefalo C, et al. Prevalence, A healthy 26-year-old man is referred to you for
characteristics and severity of nonalcoholic fatty evaluation of an incidental finding of an elevated
liver disease in patients with chronic plaque pso- total bilirubin on routine physical exam. He has
riasis. Journal of Hepatology 2009; 51(4):778-86. no medical problems and takes no medications or
herbal supplements. He feels well and his physical
exam is normal. His lab testing reveals the follow-
Question 5 ing: AST 22 U/L, ALT 23 U/L, alkaline phos-
Which of the following is associated with microve- phatase 88 U/L, total bilirubin 1.8 mg/dL, direct
sicular steatosis on liver biopsy? bilirubin 0.3 mg/dL, albumin 4.3 g/dL, INR 0.8,
Hematocrit 46 and platelet count of 237. Which of
A. Celiac disease the following statements is true in the context of
B. Cholesteryl ester storage disease the most likely diagnosis in this man?
C. Macrodantin
D. Intrahepatic cholestasis of pregnancy A. He is at increased risk for alcohol toxicity and
should avoid alcohol use.
CORRECT ANSWER: B B. He requires no further evaluation or treatment.
C. He has a genetic variation in the ABCB11 gene.
RATIONALE D. He is at increased risk for developing cirrhosis.
Cholesteryl ester storage disease is caused by defi-
cient lysosomal acid lipase activity, predominantly CORRECT ANSWER: B
resulting in cholesteryl ester accumulation, par-
ticularly in the liver, spleen and macrophages. The RATIONALE
disease is characterized by microvesicular steatosis This young, healthy, asymptomatic man has a
leading to liver failure, accelerated atherosclerosis mild indirect hyperbilirubinemia in the setting of
and premature death. Celiac disease may be as- normal hepatic function and normal hematocrit.
sociated with macrovesicular steatosis, but is not He most likely has Gilbert’s syndrome and no
typically associated with microvesicular steatosis. further evaluation or treatment is needed. This is a
Macrodantin may cause a drug-induced liver injury benign disorder that is due to genetic variations in
156 Digestive Diseases Self-Education Program®

the UGT1A1 gene that results in decreased activity Question 8


of the bilirubin uridine diphosphate glucuronosyl Which of the following statements is true regard-
transferase enzyme. Gilbert‘s syndrome is not as- ing the hyperbilirubinemia syndromes?
sociated with risk for progressive liver disease.
A. Rotor syndrome causes an increase in uncon-
REFERENCE jugated bilirubin levels.
Erlinger S, Arias IM, Dhumeaux D. Inherited dis- B. Dubin Johnson syndrome causes an increase
orders of bilirubin transport and conjugation: new in unconjugated bilirubin levels.
insights into molecular mechanisms and conse- C. Crigler-Najjar Type 1 causes an increase in
quences. Gastroenterology 2014:146:1625-38. unconjugated bilirubin levels.
D. Crigler-Najjar Type 2 results in a more severe
clinical phenotype than Criger-Najjar Type 1.
Question 7
Which of the following statements is true regard- CORRECT ANSWER: C
ing patients with Dubin Johnson syndrome?
RATIONALE
A. Dubin Johnson syndrome is associated with Crigler-Najjar is an autosomal recessive disorder
increased unconjugated bilirubin levels. resulting in mutations in the UGT1A1 gene leading to
B. Dubin Johnson syndrome is associated with defects in bilirubin glucuronidation. Crigler-Najjar
increased risk of hepatocellular carcinoma at a is characterized clinically by marked increases in
young age. unconjugated bilirubin levels from birth, putting the
C. Dubin Johnson syndrome is associated with infant at risk for kernicterus. There are two types of
increased risk for liver failure and need for Crigler-Najjar, Type 1 and Type 2. Type 1 is charac-
liver transplant. terized by the complete absence of the functional
D. Dubin Johnson syndrome is associated with enzyme, resulting in the most severe phenotype of
normal liver enzymes and serum bile acid levels. the disorder. Criger-Najjar, Type 1 is often fatal if un-
treated. Crigler-Najjar, Type 2 is the milder variant
CORRECT ANSWER: D with reduced, but not absent, enzyme activity. Rotor
syndrome and Dubin Johnson syndrome cause
RATIONALE elevations in conjugated bilirubin levels.
Dubin Johnson syndrome is caused by a genetic
variation in the ABCC2 gene, resulting in a defect REFERENCE
in the canalicular transport of conjugated bilirubin Erlinger S, Arias IM, Dhumeaux D. Inherited dis-
into the bile. This results in marked conjugated orders of bilirubin transport and conjugation: new
hyperbilirubinemia at birth. The liver appears black insights into molecular mechanisms and conse-
on gross examination due to the defects in canalicu- quences. Gastroenterology 2014:146:1625-38.
lar transport. Liver enzymes and serum bile acid
levels are normal and individuals with Dubin John-
son syndrome do not develop liver failure, nor are Question 9
they at increased risk for hepatocellular carcinoma. Which of the following statements is true regard-
ing the hereditary cholestasis disorders?
REFERENCE
Erlinger S, Arias IM, Dhumeaux D. Inherited dis- A. Serum gamma glutamyltransferase (GGT)
orders of bilirubin transport and conjugation: new level is undetectable in PFIC type 2.
insights into molecular mechanisms and conse- B. Serum gamma glutamyltransferase (GGT)
quences. Gastroenterology 2014:146:1625-38. level is increased in PFIC type 1.
Chapter 6 — Metabolic, hereditary, inflammatory and vascular diseases of the liver 157

C. PFIC type 2 is associated with increased CORRECT ANSWER: B


risk for hepatocellular carcinoma.
D. PFIC type 3 is associated with a normal RATIONALE
GGT level. The patient’s elevated liver enzymes in the context
of elevated iron saturation and elevated ferritin
CORRECT ANSWER: C and homozygosity for the C282Y mutation point
to a diagnosis of hereditary hemochromatosis. An
RATIONALE elevated serum ferritin level greater than 1000 is
Progressive familial intrahepatic cholestasis (PFIC) associated with increased risk for this patient hav-
encompasses a group of inherited disorders char- ing advanced hepatic fibrosis or cirrhosis and he
acterized by impaired bile formation leading to gen- warrants a liver biopsy. Although phlebotomy is
eralized cholestasis. PFIC type1 and PFIC type 2 are indicated for this patient, completing his evaluation
characterized by normal GGT levels. PFIC type 3 is with liver biopsy is the appropriate next step. This
associated with an elevated GGT level. PFIC type 2 patient has elevated liver enzymes and a high se-
is associated with an increased risk for hepatocel- rum ferritin in the context of homozygosity for the
lular carcinoma (HCC), warranting screening for C282Y mutation and ultimately warrants treatment
HCC beginning at a young age. with phlebotomy once his liver disease has been
accurately staged. Iron chelators are generally not
REFERENCE required unless phlebotomy cannot be tolerated.
Harris MJ, Le Couteur DG, Arias IM. Progressive
familial intrahepatic cholestasis: genetic disorders REFERENCES
of biliary transporters. J Gastroenterol Hepatol. Bacon BR; Adams PC, Kowdley KV, et al. Diag-
2005;20:807-17. nosis and Management of Hemochromatosis:
2011 Practice Guideline by the American Associa-
tion for the Study of Liver Disease. Hepatology
Question 10 2011;54:328-43.
A 55-year-old man is referred to you for evalua- Pietrangelo A. Hereditary Hemochromatosis:
tion of elevated liver enzymes and abnormal iron Pathogenesis, Diagnosis, and Treatment. Gastro-
studies. His AST is 70 U/L and ALT 85 U/L. His enterology 2010;139:393-408.
iron saturation is 88 percent and serum ferritin
is 1600. His CBC is normal. The patient feels well
and is asymptomatic. His physical examination Question 11
is normal with BMI 24 kg/m2 and normal blood You are evaluating a chronically ill 58-year-old
pressure. He is taking no medications or supple- man a five-year history of elevated liver enzymes
ments. He does not drink alcohol. His serologic (ALT 96 U/L and AST 87 U/L). His iron satura-
testing is negative for viral hepatitis, autoimmune tion is 97 percent and serum ferritin is 1131. He
hepatitis and alpha 1 antitrypsin deficiency. He is is C282Y homozygous. His past medical history
homozygous for the C282Y mutation. His liver ul- is notable for diabetes mellitus and congestive
trasound with doppler is normal. What is the next heart failure. His main clinical complaints are
appropriate step in the evaluation of this patient? fatigue, joint pains in his hands and loss of libido.
Over the course of an extensive evaluation you
A. Commence phlebotomy order a liver biopsy that reveals 4+ intrahepatic
B. Proceed with liver biopsy iron staining, with stage 2-3 fibrosis. The patient
C. Begin iron chelation therapy presents now for recommendations regarding his
D. Continue close monitoring and treat when hereditary hemochromatosis and what to expect
symptoms develop with treatment. Which of the following state-
158 Digestive Diseases Self-Education Program®

ments regarding treatment of hereditary hemo- A. Hemochromatosis due to mutations in the


chromatosis is true? Hemojuvulin gene is associated with normal
hepcidin levels.
A. Cardiomyopathy and diabetes usually improve B. Hemochromatosis due to mutations in the
with iron depleting therapy. HFE gene is associated with elevated hepcidin
B. Arthropathy and fatigue usually do not levels.
improve with iron depleting therapy. C. Hemochromatosis due to mutations in the
C. Testicular atrophy and cardiomyopathy usually ferroportin gene is associated with normal
improve with iron depleting therapy. hepcidin levels.
D. Arthropathy and testicular atrophy usually D. Hemochromatosis due to mutations in the
improve with iron depleting therapy. Transferrin Receptor 2 gene is associated with
elevated hepcidin levels.
CORRECT ANSWER: A
CORRECT ANSWER: C
RATIONALE
Among the array of systemic complications associ- RATIONALE
ated with hereditary hemochromatosis, arthropa- Hepcidin deficiency is a key feature of most known
thy and hypogonadism are much less responsive forms of hemochromatosis. Hepcidin is secreted
to iron depleting therapy. Additionally, established by the liver and regulates iron release from mac-
cirrhosis usually does not regress with iron deplet- rophages and duodenal enterocytes by interacting
ing therapy. Iron depleting therapy, such as phle- with ferroportin. HFE, Transferrin Receptor 2
botomy, may improve a patient’s sense of malaise (TfR2), and Hemojuvulin regulate the expression
and fatigue, improve abnormal skin pigmentation, of hepcidin in response to the body’s iron needs.
reduce insulin needs and improve cardiac function. Dysfunction of any of these regulators results in
unrestricted flow of iron into the plasma. Muta-
REFERENCES tions in HFE, TfR2 and Hemojuvulin are all asso-
Bacon BR; Adams PC, Kowdley KV, et al. Diag- ciated with reduced hepcidin levels. The exception
nosis and Management of Hemochromatosis: is ferroportin disease in which the mutation in the
2011 Practice Guideline by the American Associa- ferroportin gene renders ferroportin unresponsive
tion for the Study of Liver Disease. Hepatology to the inhibitory action of hepcidin on iron trans-
2011;54:328-43. port, resulting in unregulated release of iron into
Pietrangelo A. Hereditary Hemochromatosis: the plasma despite normal hepcidin levels.
Pathogenesis, Diagnosis, and Treatment. Gastro-
enterology 2010;139:393-408. REFERENCES
Bacon BR; Adams PC, Kowdley KV, et al. Diag-
nosis and Management of Hemochromatosis:
Question 12 2011 Practice Guideline by the American Associa-
A 43-year-old man presents to you for evaluation tion for the Study of Liver Disease. Hepatology
of elevated liver enzymes (ALT 72 U/L and AST 2011;54:328-43.
68 U/L). Over the course of his clinical work-up, Pietrangelo A. Hereditary Hemochromatosis:
you find that his iron saturation is 91 percent and Pathogenesis, Diagnosis, and Treatment. Gastro-
serum ferritin is 868. You suspect a diagnosis of enterology 2010;139:393-408.
hereditary hemochromatosis and plan to proceed
with genetic testing. Which of the following state-
ments is true regarding hepcidin levels in heredi- Question 13
tary hemochromatosis? A 43-year-old woman with an elevated alkaline
Chapter 6 — Metabolic, hereditary, inflammatory and vascular diseases of the liver 159

phosphatase level of 454 U/L, positive anti-mito- localized within the inner mitochondrial matrix. In
chondrial antibody [AMA] (titer, 1:1280) and el- approximately 95 percent of patients with PBC, AMA
evated serum IgM level comes to see you for initial are directed towards PDC-E2.
consultation and recommendations regarding her
chronic liver disease. Her past medical history is REFERENCE
notable only for hypercholesterolemia with a total Lindor KD, Gershwin E, Poupon R, et al. Primary
cholesterol level of 290. She has no cardiovascular Biliary Cirrhosis. Hepatology 2009;50:291-308.
disease. Her physical exam is normal with a BMI
of 22 kg/m2. Which of the following statements is
true regarding her liver disease? Question 14
You have evaluated a 51-year-old woman with an
A. Her elevated total cholesterol level that is related elevated serum alkaline phosphatase level of 379
to her liver disease places her at increased risk U/L , total bilirubin level of 2.1 mg/dL and positive
for a cardiovascular event despite her young age. anti-mitochondrial antibody (AMA) with a titer of
B. Her first-degree relatives are at the same risk 1:1280. Fibroscan is suggestive of minimal fibro-
for developing PBC as the general population. sis. You have given her a new diagnosis of primary
C. She is at risk for developing varices in the biliary cholangitis (PBC). She presents to your
absence of cirrhosis. clinic with questions about the natural history of
D. The AMA is directed against the bilirubin her PBC and her prognosis. Which of the following
uridine diphosphate glucuronosyl transferase statements is true regarding the natural history and
enzymes found primarily in hepatocytes. epidemiology of primary biliary cholangitis (PBC)?

CORRECT ANSWER: C A. Total bilirubin level is the best predictor of


survival in PBC.
RATIONALE B. PBC typically progresses rapidly to cirrhosis.
This patient’s clinical presentation with an elevated C. Patients with PBC who test negative for the
alkaline phosphatase, elevated AMA and elevated anti-mitochondrial antibody (AMA-negative
IgM, particularly in the context of her female gender PBC) have more severe liver disease than
and age, are diagnostic of primary biliary cholan- patients with AMA-positive PBC.
gitis (PBC). Although the total cholesterol may be D. PBC affects women and men equally.
elevated in PBC, this has not been associated with
increased risk for cardiovascular events in these CORRECT ANSWER: A
patients (in the absence of other cardiovascular
predisposition). The patient’s first-degree relatives RATIONALE
are approximately 100 times more likely to develop Although several clinical and biochemical features
PBC than the general population. PBC can be associ- have prognostic value in PBC, total bilirubin level
ated with the development of pre-sinusoidal portal is the best predictor of survival. PBC tends to be
hypertension in the absence of cirrhosis, resulting a more slowly progressive chronic liver disease
in risk for developing varices. The autoantigens of compared to other chronic liver disease etiologies.
AMA have been identified as the E2 subunits of the Patients with AMA-negative PBC have a similar
2-oxo-acid dehydrogenase complexes, including the natural history as patients with AMA-positive PBC.
E2 subunits of the pyruvate dehydrogenase complex PBC is much more common in women than men.
(PDC-E2), branched chain 2-oxo acid dehydroge-
nase complex, 2-oxo-glutarate dehydrogenase com- REFERENCE
plex and the E3 binding protein of dihydrolipoamide Lindor KD, Gershwin E, Poupon R, et al. Primary
dehydrogenase. The AMA target antigens are all Biliary Cirrhosis. Hepatology 2009;50:291-308.
160 Digestive Diseases Self-Education Program®

Question 15 A. Florid duct lesion


You recently diagnosed a 41-year-old woman with B. Periductal concentric fibrosis
primary biliary cholangitis (PBC) with stage 2 fi- C. Hepatocyte rosettes with dense plasma cell
brosis. Her alkaline phosphatase is elevated to 289 infiltrate
U/L and total bilirubin is 1.2 mg/dL. The patient D. Endotheliitis
would like to discuss her PBC treatment options
with you. Which of the following statements is CORRECT ANSWER: B
true regarding the use of ursodeoxycholic acid in
the management of PBC? RATIONALE
The characteristic histopathologic feature of
A. Ursodeoxycholic acid is associated with a PSC is periductal concentric fibrosis (sometimes
reduction in HDL levels in PBC. referred to as “onion-skinning”). The florid duct
B. Ursodeoxycholic acid reduces the risk of lesion is seen in primary biliary cholangitis. Hepa-
variceal bleeding in PBC. tocyte rosettes with plasma cell infiltrate may be
C. Ursodeoxycholic acid improves fatigue in PBC. seen in autoimmune hepatitis. Endotheliitis is a
D. Ursodeoxycholic acid reduces the risk of histologic feature that is associated with acute cel-
osteoporosis in PBC. lular rejection after liver transplantation.

CORRECT ANSWER: B REFERENCE


Chapman R, Fevery J, Kalloo A, et al. Diagnosis
RATIONALE and Management of Primary Sclerosing Cholangi-
Ursodeoxycholic acid in the management of PBC tis. Hepatology 2010;51:660-78.
has been shown to reduce LDL levels, reduce the
risk of variceal bleeding, slow the progression of
PBC and reduce the risk of needing a liver trans- Question 17
plant. Ursodeoxycholic acid has not been shown to A 27-year-old man with newly diagnosed well-
improve fatigue or to reduce the risk of osteoporo- compensated cirrhosis due to primary sclerosing
sis in PBC. cholangitis (PSC) presents for initial consultation.
His past medical history is notable for ulcerative
REFERENCE colitis that is well-controlled with mesalamine. He
Lindor KD, Gershwin E, Poupon R, et al. Primary feels well and has no specific complaints. His AST,
Biliary Cirrhosis. Hepatology 2009;50:291-308. ALT, total bilirubin, albumin, INR and platelet
count are all normal. His alkaline phosphatase is
elevated to 347 U/L. Which of the following do
Question 16 you recommend for further management of this
A 26-year-old man with an elevated alkaline patient?
phosphatase level of 399 U/L and total bilirubin
level of 1.0 mg/dL presents to you for evaluation A. Liver biopsy to grade and stage his PSC.
of his elevated liver enzymes. His past medical B. ERCP to assess for progression of his
history is notable for ulcerative colitis. Magnetic biliary strictures.
resonance cholangiography reveals multiple stric- C. Annual ultrasound evaluation of the
tures and dilations in his intra-hepatic and extra- gallbladder.
hepatic biliary ducts consistent with a diagnosis D. Initiation of ursodeoxycholic acid at 30 mg/
of primary sclerosing cholangitis (PSC). Which kg/day for colon cancer prevention.
of the following histopathologic findings on liver
biopsy is characteristic of PSC? CORRECT ANSWER: C
Chapter 6 — Metabolic, hereditary, inflammatory and vascular diseases of the liver 161

RATIONALE cent of patients. IgG4-related sclerosing cholangitis


The patient is at risk for gallbladder cancer and is clinically distinct from primary sclerosing chol-
warrants annual screening with gallbladder ultra- angitis (PSC). Distinctions between PSC and IgG4-
sound. He is also at risk for cholangiocarcinoma related sclerosing cholangitis are crucial because of
and colon cancer. Since it is already known that the the notably different prognoses in these conditions.
patient has cirrhosis, liver biopsy in this scenario IgG-4 related sclerosing cholangitis characteristically
will not provide additional information to help demonstrates a robust response to corticosteroids.
guide management. ERCP is a useful procedure in However, there is no proven medical treatment for
the setting of concerns for cholangitis or cholangio- PSC. PSC is more likely to be associated with an
carcinoma, but ERCP in the setting of stable PSC elevated pANCA. IgG-4 related sclerosing cholangitis
without specific clinical concerns is not warranted. more commonly occurs in older men.
Ursodeoxycholic acid has not been shown to impact
the natural history of PSC. High dose ursodeoxy- REFERENCES
cholic acid (28-30 mg/kg/day) may be harmful in Ghazale A, Chari ST, Zhang L, et al. Immuno-
the setting of PSC and should be avoided. globulin G4-associated cholangitis: clinical profile
and response to therapy. Gastroenterology
REFERENCE 2008;134:706-15.
Chapman R, Fevery J, Kalloo A, et al. Diagnosis Chapman R, Fevery J, Kalloo A, et al. Diag-
and Management of Primary Sclerosing Cholangi- nosis and Management of Primary Sclerosing
tis. Hepatology 2010;51:660-78. Cholangitis. Hepatology 2010;51:660-78.

Question 18 Question 19
Which of the following statements is true regard- A 34-year-old African-American woman presents
ing IgG4-related sclerosing cholangitis? to you for evaluation of persistently elevated liver
enzymes. Her alkaline phosphatase is 256 U/L,
A. It is commonly associated with elevated AST 42 U/L and ALT 24U/L. Total bilirubin level
pANCA levels. is normal. She has no other medical diagnoses.
B. The typical age of onset is 20-25 years old. She does not drink alcohol or smoke tobacco. She
C. Corticosteroids are first-line treatment. complains of fatigue and a chronic dry cough.
D. It affects women more commonly than men. Serologic testing for viral hepatitis, autoimmune
liver disease, iron overload and alpha-one-anti-
CORRECT ANSWER: C trypsin deficiency is negative. Abdominal ultra-
sound reveals an enlarged and heterogeneous-ap-
RATIONALE pearing liver with patent hepatic and portal veins.
IgG4-related sclerosing cholangitis is one manifes- Chest x-ray reveals bilateral hilar adenopathy. You
tation of the disease entity known as IgG4-related recommend a liver biopsy, which reveals multiple
disease (IgG4-RD). IgG4-RD can affect many organs small granulomas. Her serum angiotensin con-
in the body and is characterized by dense lympho- verting enzyme level is elevated to 165 U/L (ULN
plasmacytic infiltrations with a predominance of 8-53 U/L). Which of the following statements is
IgG4-positive plasma cells, often associated with true regarding hepatic sarcoidosis?
some degree of fibrosis. Patients may present with a
mass in the affected organ or diffuse enlargement of A. A diagnosis of hepatic sarcoid mandates
an organ (such as the pancreas). Multiple organs are initiation of corticosteroids.
affected in the majority of patients with IgG4-RD. B. Hepatic sarcoid is characterized by caseating
Serum IgG4 levels may be elevated in up to 66 per- granulomas in the liver.
162 Digestive Diseases Self-Education Program®

C. An elevated serum angiotensin converting medications besides his tacrolimus. He takes no


enzyme level is diagnostic of hepatic sarcoid. supplements. He is not drinking alcohol. Ultra-
D. An elevated serum alkaline phosphatase with sound with doppler of the liver is normal. Serum
normal or only mildly elevated aminotrans- CMV PCR is undetectable. You suspect acute
ferases is typical of hepatic sarcoid. cellular rejection given his low tacrolimus level.
You obtain a liver biopsy. Which of the following
CORRECT ANSWER: D features would be expected on his liver biopsy?

RATIONALE A. Ductopenia and cholestasis


Hepatic sarcoid is characterized by non-caseat- B. Intranuclear inclusions with prominent
ing granulomas in the liver. Although corticoste- mononuclear infiltrate
roids are useful in the management of hepatic C. Ductulitis and endotheliitis
sarcoid, not all patients with hepatic sarcoid D. Multinucleate hepatocytes
warrant treatment. Specifically, in asymptom-
atic patients it is unclear if steroids prevent liver CORRECT ANSWER: C
damage or halt the progression of the disease.
Thus, a diagnosis of hepatic sarcoid alone does RATIONALE
not compel the initiation of corticosteroids. Al- The characteristic histologic features of acute
though the serum level of angiotensin converting cellular rejection early after liver transplant
enzyme may be elevated in the setting of hepatic include ductulitis and endotheliitis. Recurrent
sarcoid, an elevated serum angiotensin convert- episodes of acute cellular rejection, or chronic
ing enzyme level may occur with other diseases under-immunosuppression, can result in chronic
and is, therefore, not diagnostic of hepatic rejection, which may be characterized on liver
sarcoid. The typical biochemical pattern found biopsy by ductopenia and cholestasis. Multi-
in patients with hepatic sarcoid is an elevated nucleate hepatocytes and intranuclear inclusions
serum alkaline phosphatase level with or without may be seen in viral infection after liver trans-
mild elevations in the serum aminotransferases. plant (for example, HSV and CMV).

REFERENCE REFERENCE
Deutsch-Link S, Fortuna D, Weinberg EM. A Demetris AJ, Batts KP, Dhillon AP, et al. Banff
Comprehensive Review of Hepatic Sarcoid. Schema for Grading Liver Allograft Rejection:
Seminars in Liver Disease 2018;38:284-97. An International Consensus Document. Hepa-
tology 1997;25:658-63.

Question 20
A 63-year-old man underwent deceased donor Question 21
liver transplant eight months ago for alcohol- A 60-year-old man who underwent deceased do-
induced cirrhosis. His liver enzymes normalized nor liver transplant two months ago for hepatitis
soon after transplant and remained normal. His C (HCV) cirrhosis presents for urgent clinical
liver enzymes have been repeatedly elevated evaluation. His HCV was treated and cured
over the past week with AST 180 U/L, ALT 212 pre-transplant. His immediate post liver trans-
U/L, alkaline phosphatase of 188 U/L, normal plant course was uneventful and he received
total bilirubin and normal INR. His tacrolimus four doses of anti-thymocyte globulin (ATG)
level most recently was 2.0 ng/mL (target level induction. His liver enzymes and liver function
5-7 ng/mL). He is feeling reasonably well, but normalized soon post-transplant, as anticipated.
does note some mild fatigue. He takes no other He presents now with a three-day history of
Chapter 6 — Metabolic, hereditary, inflammatory and vascular diseases of the liver 163

fevers, persistent diarrhea and a maculopapular and an Approach to Management. Transplanta-


rash on his chest, back, hands, arms and legs. tion 2016; 100:2661-70.
His current medications include tacrolimus and
mycophenolate mofetil for immunosuppression.
He also takes sulfamethoxazole-trimethoprim, Question 22
valganciclovir and fluconazole for routine infec- A 55-year-old woman underwent liver transplan-
tion prophylaxis post-liver transplant. He states tation two days ago for cirrhosis due to primary
he is feeling very poorly and he appears clinically biliary cholangitis. She is obtunded, intubated
unwell on exam with blood pressure of 93/58 and on dual pressor support in the Surgical ICU.
mmHg. Lab data reveals the following: AST 19 Her lab studies reveal rising liver enzymes and
U/L, ALT 22 U/L, alkaline phosphatase 101 U/L, INR: AST 3768 U/L, ALT 3216 U/L and INR 3.7.
total bilirubin 0.7 mg/dL, albumin 3.6 g/dL and Her arterial pH is 7.25. Emergent ultrasound
INR 0.9. His tacrolimus level is 7 ng/mL (target with doppler of the liver demonstrates no flow in
level: 5-7 ng/mL). He has developed new leuko- the hepatic artery. What is the next appropriate
penia and anemia with a white blood cell count step in her management?
of 1.0 and hematocrit of 19. What is the most
likely diagnosis? A. Emergent CT angiography
B. Emergent angiography with Interventional
A. Acute cellular rejection Radiology
B. CMV infection C. List for re-transplant as Status 1A
C. Small for size syndrome (highest priority)
D. Graft versus Host Disease D. Initiate IV heparin

CORRECT ANSWER: D CORRECT ANSWER: C

RATIONALE RATIONALE
This patient’s presentation is characteristic of This patient meets all the diagnostic criteria for
graft versus host disease (GVHD) post-liver early hepatic artery thrombosis after liver trans-
transplant with fevers, leukopenia, anemia, se- plant, with her deterioration in her clinical status
cretory diarrhea and maculopapular rash. Liver characterized by severe elevations in her AST and
enzymes are normal is the setting of GVHD. ALT with rising INR and acidosis. Her hepatic
Rash, diarrhea and cytopenia are not character- artery could not be visualized on ultrasound with
istics of acute cellular rejection. Additionally, the doppler. Hepatic artery thrombosis early after liver
normal liver enzymes would argue against acute transplant is a surgical emergency. This patient is
cellular rejection. CMV infection is unlikely as unstable with her altered mental status and hypo-
the patient is still on CMV prophylaxis with val- tension requiring dual-pressor support. She needs
ganciclovir. Small for size syndrome is charac- re-transplant and meets criteria for listing as Status
terized by post-operative coagulopathy and liver 1A, connoting highest urgency. CT angiography will
dysfunction due to insufficient functional liver not alter the diagnosis or change management. This
mass, which is not consistent with the patient’s patient is too unstable to undergo interventions
presentation here. on her hepatic artery by Interventional Radiology.
Initiation of IV heparin would be dangerous in the
REFERENCE setting of this patient’s immediate post-transplant
Murali AR, Chandra S, Stewart Z, et al. Graft status and, in the context of her acutely failing
versus Host Disease After Liver Transplantation allograft due to her hepatic artery thrombosis, anti-
in Adults: A Case series, Review of Literature, coagulation should be avoided.
164 Digestive Diseases Self-Education Program®

REFERENCE cal history. Her only medication is oral contraceptive


https://optn.transplant.hrsa.gov/media/1200/ pills. She takes no supplements. She does not smoke
optn_policies.pdf#nameddest=Policy_09. Ac- or drink alcohol. Her labs reveal: AST 256 U/L,
cessed 11/18/2018. ALT 211 U/L, alkaline phosphatase is 214 U/L, total
bilirubin 1.3 mg/dL and INR 1.0. Serologic testing is
negative for viral and autoimmune hepatitis. Preg-
Question 23 nancy test is negative. On physical exam her abdomen
Which of the following statements is true regard- is distended with a discernable fluid wave. She has 1+
ing the wedged hepatic venous pressure (WHVP), bilateral lower extremity edema. She is alert and ori-
the free hepatic venous pressure (FHVP) and the ented x 4 with no asterixis. Which of the following is
hepatic venous pressure gradient (HVPG) mea- the next most appropriate test to make the diagnosis?
surements in an asymptomatic patient with portal
hypertension due to schistosomiasis? A. Liver biopsy
B. Hepatic ultrasound with doppler
A. WHVP is high; FHVP is normal; HVPG is high C. Hepatic arteriogram
B. WHVP is low; FHVP is normal; HVPG is high D. MRCP
C. WHVP is normal; FHVP is normal; HVPG is normal
D. WHVP is high; FHVP is high; HVPG is normal Correct answer: B

CORRECT ANSWER: C RATIONALE


This patient has signs and symptoms most consis-
RATIONALE tent with acute Budd Chiari syndrome with new
The HVPG is calculated by subtracting the FHVP onset ascites and elevated liver enzymes. Hepatic
from the WHVP. Schistosomiasis causes intrahe- ultrasound with doppler is the next appropriate test
patic, pre-sinusoidal portal hypertension. This is to assess the patency of the hepatic veins and IVC.
characterized by normal WHVP, normal FHVP Budd Chiari is a hepatic venous outflow derange-
and normal HVPG. Answer “a” is characteristic ment; therefore, hepatic arteriogram would not be
of sinusoidal portal hypertension (such as cir- a reasonable next test. MRCP assesses the biliary
rhosis) with an elevated WHVP, normal FHVP tree and would not be the next best test. Liver bi-
and elevated HVPG. Answer “d” is characteristic opsy is invasive and is rarely indicated for the initial
of post-hepatic portal hypertension (such as right diagnosis of symptomatic Budd Chiari syndrome.
heart failure), with an elevated WHVP, elevated
FHVP and resultant normal HVPG. REFERENCES
DeLeve L, Valla DC, Garcia-Tsao G. Vascular Dis-
REFERENCE orders of the Liver. Hepatology 2009;49:1729-64.
Garcia-Tsao G, Abraldes JG, Berzigott A, Bosch J. Menon KVN, Shah V, Kamath PS. The Budd-
Portal Hypertensive Bleeding in Cirrhosis: Risk Strati- Chiari Syndrome. New England Journal of Medi-
fication, Diagnosis, and Management: 2016 Practice cine 2004; 350:578-585.
Guidance by the American Association for the Study
of Liver Diseases. Hepatology 2017;65:310-35.
Question 25
Which of the following is a characteristic feature
Question 24 of Budd Chiari Syndrome?
A 23-year-old previously healthy woman presents
with new onset abdominal pain, abdominal distention A. Absence of hepatic venous collaterals on
and lower extremity edema. She has no past medi- cross-sectional imaging.
Chapter 6 — Metabolic, hereditary, inflammatory and vascular diseases of the liver 165

B. Homogeneous portal perfusion of the liver on 1.6. RUQ ultrasound revealed a normal GB with-
contrast-enhanced CT scan. out gallstones, pericholecystic fluid, and GB wall
C. Hepatic imaging demonstrating hepatic thickening. CBD was three millimeters, liver was
caudate lobe hypertrophy. enlarged, and no liver masses were seen. Doppler
D. Serum-Ascites Albumin Gradient (SAAG) less than study showed normal flow in the hepatic and portal
1.1, and ascites total protein greater than 3 mg/dL. veins. Spleen was mildly enlarged. Upon question-
ing, his family reports that he drinks one to two
CORRECT ANSWER: C pints of hard liquor daily. Which of the following
treatments is the best option for this patient?
RATIONALE
Caudate lobe hypertrophy is found in a majority of A. Corticosteroids
patients with Budd Chiari syndrome, and is due to B. Ursodeoxycholic acid
the separate venous drainage of the caudate into C. Pentoxifylline
the inferior vena cava allowing for sparing of the D. Infliximab
outflow, resulting in compensatory hypertrophy. E. Azathioprine
The characteristic pattern of parenchymal perfu-
sion of a Budd Chiari liver on contrast-enhanced CORRECT ANSWER: A
cross sectional imaging includes early homogeneous
central enhancement with delayed patchy enhance- RATIONALE
ment of the peripheral liver and prolonged retention In a patient with a history of heavy alcohol use, the
of contrast in the hepatic periphery, resulting in diagnosis of acute alcoholic hepatitis (AH) is sug-
inhomogeneous portal perfusion. A characteristic gested by the onset or sudden worsening of jaundice
feature of Budd Chiari syndrome on hepatic imaging with or without other signs of hepatic dysfunction.
is its association with intrahepatic or subcapsular Physical exam in AH may reveal stigmata of liver
hepatic venous collaterals, which are identified in up cirrhosis, as well as SIRS criteria, due to the signifi-
to 80 percent of patients. The SAAG in Budd Chiari cant inflammatory response in the liver. Laboratory
is characteristically elevated 1.1 or more, along with findings include an elevated bilirubin, and an AST/
an elevated ascites total protein level. ALT ratio greater than 1.5 with a total AST or ALT
less than 400 IU/L. Biopsy is not typically per-
REFERENCE formed, but will show macrovesicular steatosis, a
DeLeve L, Valla DC, Garcia-Tsao G. Vascular Dis- neutrophilic lobulitis, the presence of Mallory-Denk
orders of the Liver. Hepatology 2009;49:1729-64. bodies, and pericellular fibrosis. Unfortunately, these
findings are also seen in NASH, so an alcohol history
is a crucial part of the workup. Current guidelines
Question 26 define “Definite AH” by the biopsy findings, but allow
A 57-year-old man presents to the ED with a a diagnosis of “Probable AH” if the following criteria
complaint of RUQ, fever, chills and jaundice for are met: 1) Heavy alcohol use for greater than five
five days. He reports no dysphagia or odynophagia, years; 2) active alcohol use until four weeks prior
no diarrhea or constipation, His VS are as fol- to presentation; 3) sudden onset or worsening of
lows: Pulse 100 bpm and regular, BP 145/70 mm jaundice; 4) AST/ALT greater than 1.5 with levels
Hg, RR 14 per min, O2 saturation 98 percent on less than 400 IU/L; and 5) absence of other causes
room air. His exam is remarkable for moderately of liver disease. In patients with AH, prognosis can
tender hepatomegaly. His CBC: WBC 16.1K cells/ be estimated using the MELD score or the Maddrey
mm3, Hgb 12.1 mg/dL, PLT 147K cells/mm3. Liver Discriminant Function; a DF greater than 32 pre-
function tests: AST 143 U/L, ALT 88 U/L, ALP 110 dicts a 20-50 percent 30-day mortality and patients
U/L, T. bilirubin 9.1 mg/dL, Albumin 3.0 g/L, INR meeting this threshold should be treated. Current US
166 Digestive Diseases Self-Education Program®

guidelines call for the use of corticosteroids (specifi- suggested by the onset or sudden worsening of
cally prednisolone) in severe AH. Response to ste- jaundice with or without other signs of hepatic dys-
roid therapy is assessed at day seven using the Lille function. Physical exam in AH may reveal stigmata
score, with therapy continued for a total of 28 days in of liver cirrhosis, as well as SIRS criteria, due to
initial responders. Pentoxifylline had shown possible the significant inflammatory response in the liver.
benefit in earlier studies, but more recent data calls Laboratory findings include an elevated bilirubin,
this into question. Infliximab had been studied in AH and an AST/ALT ratio greater than 1.5 with a total
and was found to have severe infectious complica- AST or ALT less than 400 IU/L. Biopsy is not
tions, precluding its use. Azathioprine is used in the typically performed, but will show macrovesicular
treatment of autoimmune hepatitis but has no role steatosis, a neutrophilic lobulitis, the presence of
in AH and ursodeoxycholic acid is utilized in Primary Mallory-Denk bodies, and pericellular fibrosis. Un-
Biliary Cholangitis. fortunately, these findings are also seen in NASH,
so an alcohol history is a crucial part of the workup.

Question 27
A 57-year-old man with a history of alcohol abuse Question 28
presents to the ED with a complaint of RUQ, f/c, A 50-year-old man presents to clinic with a com-
and jaundice for five days. He reports no dysphagia plaint of fatigue. He drinks approximately one pint
or odynophagia, no diarrhea or constipation. His of whiskey daily, and has done so for more than 20
VS are as follows: Pulse 92 bpm and regular, BP years. His exam is remarkable for hepatomegaly,
145/70 mm Hg, RR 14, O2 saturation 98 percent on palmar erythema, and spider angiomata of his neck
room air. His exam is remarkable for moderately and chest, but no asterixis or altered mental func-
tender hepatomegaly. His CBC: WBC 16.1K cells/ tioning. Liver function tests: AST 110 U/L, ALT 60
mm3, Hgb 12.1 mg/dL, PLT 147K cells/mm3. Liver U/L, ALP 110 U/L, T. bilirubin 1.8 mg/dL, Albu-
function tests: AST 143 U/L, ALT 88 U/L, ALP 110 min 3.0 g/L, INR 1.2. RUQ ultrasound revealed a
U/L, T. bilirubin 11.2 mg/dL, Albumin 3.0 g/L, INR normal GB without gallstones, pericholecystic fluid,
1.6. RUQ ultrasound revealed a normal GB without and GB wall thickening. CBD was three millime-
gallstones, pericholecystic fluid, or GB wall thicken- ters, liver was small and nodular, no liver masses
ing. CBD was three millimeters, liver was enlarged, were seen, and Doppler study showed normal flow
and no liver masses were seen. Doppler study in the hepatic and portal veins. Spleen was mildly
showed normal flow in the hepatic and portal veins. enlarged. Which of the following will lead to the
Spleen was mildly enlarged. A liver biopsy would greatest improvement in his survival?
most likely show which of the following?
A. Corticosteroids
A. PAS positive, diastase resistant globules in B. Ursodeoxycholic acid
hepatocytes C. Pentoxifylline
B. Ballooning degeneration and Mallory’s hyaline D. Infliximab
C. Lymphoplasmacytic infiltration with rosettes E. Abstinence
D. Extensive fibrosis with minimal inflammation
E. Granulomatous inflammation CORRECT ANSWER: E

CORRECT ANSWER: B RATIONALE


This patient has stigmata of cirrhosis on exam
RATIONALE and laboratory tests indicative of mild synthetic
In a patient with a history of heavy alcohol use, dysfunction. His history of alcohol use and his
the diagnosis of acute alcoholic hepatitis (AH) is transaminases (with an AST/ALT ratio greater 1.5)
Chapter 6 — Metabolic, hereditary, inflammatory and vascular diseases of the liver 167

are suggestive of alcoholic liver disease (ALD). He RATIONALE


does not have acute alcoholic hepatitis, so corti- Alpha-1 antitrypsin deficiency is a common metabolic
costeroids are not indicated. Pentoxifylline is not cause of liver disease in children and adults, affect-
currently recommended for alcoholic hepatitis, ing approximately 1:3500 live births. Production of
and TNF alpha inhibitors have been shown to a misfolded A1 AT protein leads to low A1AT levels
cause high rates of infection in acute AH and are in the serum (less than 15 percent of normal), which
not used. Ursodeoxycholic acid has no benefit in leads to lung damage via the unopposed action of
alcoholic liver disease. Abstinence from alcohol has pulmonary neutrophil elastase. In the liver, however,
been shown to correlate with outcomes in ALD, and damage is caused by accumulation of the abnormal
complete abstinence is the most important fact of protein in the endoplasmic reticulum of hepatocytes,
management. In patients with alcoholic cirrhosis, causing oxidative damage and eventual fibrosis and
baclofen has been shown to be safe and to increase cirrhosis. On biopsy, the misfolded protein appears
abstinence rates, while other abstinence-inducing as PAS positive globules within hepatocyte that are
agents such as acamprosate and naltrexone have resistant to digestion with diastase. Biopsy, however,
possible hepatotoxicity in patients with ALD. is not needed to make the diagnosis. A1 AT should be
suspected in any patient with unexplained cirrhosis
REFERENCE or unexplained mild elevations of AST and ALT. A
Singal AK, Bataller R et al. ACG Clinical Guide- clinical clue to A1 AT deficiency is a personal or fam-
line: Alcoholic Liver Disease. Am J Gastro ily history of early COPD (before age 50). Although a
2018;113:175-194. low A1 AT level in the serum can suggest the diagno-
sis, this enzyme is an acute phase reactant and the
A1AT level may be falsely elevated in the setting of in-
Question 29 flammation. The gold standard of diagnosis is A1 AT
A 49-year-old man with an eight-year history of phenotyping using isoelectric focusing. This testing
COPD is found to have the following liver en- allows identification of the abnormally folded protein
zymes: AST 92 U/L, ALT 105 U/L, ALP 105 U/L, (the S or Z forms) instead of the normal protein (the
T. bilirubin 0.9 mg/dL, Albumin 3.0 g/L, INR 1.2. M form). Patients with the PiZZ phenotype express
CBC is remarkable only for a PLT 99K cells/mm3. 10-20 percent of the normal A1AT and tend to have
RUQ ultrasound revealed a normal GB with no the most severe disease.
gallstones, CBD was three millimeters, liver was
of small size with a nodular contour, no liver REFERENCE
masses were seen, and Doppler study showed Mitchell EL, Khan Z. Liver Disease in Alpha-1 Anti-
normal flow in the hepatic and portal veins. trypsin Deficiency: Current Approaches and Future
Moderate splenomegaly was present. Hepatitis Directions. Curr Pathobiol Rep 2017;5:243-252.
serologies, ANA, anti-smooth muscle antibod-
ies, iron studies, and serum ceruloplasmin are all
negative. Which of the following tests would you Question 30
order next? A 51-year-old man with an eight-year history of
alpha-1 antitrypsin deficiency is found to have
A. Urinary copper excretion the following liver enzymes: AST 92 U/L, ALT
B. C282Y mutation analysis 105 U/L, ALP 105 U/L, T. bilirubin 0.9 mg/dL,
C. Anti liver-kidney-microsomal antibody albumin 3.0 g/L, INR 1.2. CBC is remarkable
D. Alpha 1 antitrypsin phenotyping only for a PLT of 102K cells/mm3. Liver biopsy
E. Liver biopsy shows established cirrhosis and PAS (+), diastase
resistant globules in the hepatocytes. What is the
CORRECT ANSWER: D mechanism of liver injury in this disorder?
168 Digestive Diseases Self-Education Program®

A. Deficient alpha-1 AT leads to neutrophilic trypsin Deficiency: Current Approaches and Future
infiltration of the liver and hepatocyte necrosis Directions. Curr Pathobiol Rep 2017;5:243-252.
B. Excess misfolded alpha-1 AT accumulates in Townsend SA, Edgar RG et al. Systematic
the liver, leading to oxidative damage to review: the natural history of alpha-1 antitrypsin
hepatocytes. deficiency, and associated liver disease. Aliment
C. Excess misfolded alpha-1 AT accumulates in Pharmacol Ther 2018;47:877-885.
the liver, leading to lymphocytic infiltration of
the liver and subsequent fibrosis.
D. Deficient alpha-1 AT leads to hepatocyte Question 31
damage by enzymes that would normally be A 32-year-old woman with a history of hypothy-
deactivated by this enzyme. roidism presents to the ED with a complaint of
fatigue and jaundice. Her exam is remarkable
CORRECT ANSWER: B for moderate RUQ tenderness on palpation. Her
CBC: WBC 11.1K cells/mm3, Hgb 11.1 mg/dL,
RATIONALE PLT 100K cells/mm3. Liver function tests: AST
Alpha-1 antitrypsin deficiency is a common meta- 131 U/L, ALT 155 U/L, ALP 109 U/L, T. bilirubin
bolic cause of liver disease in children and adults, 4.1 mg/dL, Albumin 3.0 g/L, TP 8.9 g/L, INR
affecting approximately 1:3500 live births. Produc- 1.2. RUQ ultrasound revealed a normal GB with
tion of a misfolded A1 AT protein leads to low A1AT no gallstones, pericholecystic fluid, or GB wall
levels in the serum (less than 15 percent of normal), thickening. CBD was three millimeters, liver was
which leads to lung damage via the unopposed ac- of small size with a nodular contour, and no liver
tion of pulmonary neutrophil elastase. In the liver, masses were seen. Doppler study showed normal
however, damage is caused by accumulation of the flow in the hepatic and portal veins. Moder-
abnormal protein in the endoplasmic reticulum of ate splenomegaly was present. ANA was 1:2560.
hepatocytes, causing oxidative damage and eventu- Which of the following therapies would be a com-
al fibrosis and cirrhosis. On biopsy, the misfolded ponent of the initial treatment of this patient?
protein appears as PAS positive globules within
hepatocyte that are resistant to digestion with A. Azathioprine
diastase. A1 AT should be suspected in any patient B. Cyclosporin
with unexplained cirrhosis or unexplained mild C. Ursodeoxycholic acid
elevations of AST and ALT. A clinical clue to A1 AT D. Tenofovir alefenamide
deficiency is a personal or family history of early E. Cyclophosphamide
COPD (before age 50). Although a low A1 AT level
in the serum can suggest the diagnosis, this enzyme CORRECT ANSWER: A
is an acute phase reactant and the A1AT level may
be falsely elevated in the setting of inflammation. RATIONALE
The gold standard of diagnosis is A1 AT phenotyp- Autoimmune hepatitis (AIH) is a chronic inflamma-
ing using isoelectric focusing. This testing allows tory disorder of the liver characterized by elevated
identification of the abnormally folded protein (the transaminases, positive autoimmune serologies,
S or Z forms) instead of the normal protein (the M and elevated immunoglobulin levels, with vary-
form). Patients with the PiZZ phenotype express ing degrees of liver damage. Clues to the diagnosis
10-20 percent of the normal A1AT and tend to have include personal or family history of autoimmune
the most severe disease. conditions, and female gender, though the sex ratio
is only 3.6:1. The presentation can vary from asymp-
REFERENCES tomatic disease with abnormal liver enzymes, to
Mitchell EL, Khan Z. Liver Disease in Alpha-1 Anti- decompensated cirrhosis, to an acute hepatitis with
Chapter 6 — Metabolic, hereditary, inflammatory and vascular diseases of the liver 169

markedly elevated transaminases and the possibility A. Antimitochondrial antibody


of acute liver failure. In this question, the patient ap- B. Anti-Smith antibody
pears to have cirrhosis, which is present at diagnosis C. Anti – ribonuclear protein (RNP)
in approximately one-third of adults and up to half D. Anti smooth-muscle antibody
of children with AIH. A liver biopsy is indicated to E. Antiphospholipid antibody
assess the degree of active inflammatory disease, and
if inflammation is present, the patient should begin CORRECT ANSWER: D
therapy. Two standard treatment regimens for AIH
include corticosteroids (prednisone or prednisolone) RATIONALE
alone, or corticosteroids combined with azathio- Autoimmune hepatitis (AIH) is a chronic inflam-
prine. The combination regimen allows for a lesser matory disorder of the liver characterized by
dose of steroids and a lower incidence of side effects elevated transaminases, positive autoimmune
with the same therapeutic efficacy. In patients who serologies, and elevated immunoglobulin levels,
fail to respond, or who are intolerant of azathioprine, with varying degrees of liver damage. Clues to
mycophenolate mofetil and calcineurin inhibitors the diagnosis include personal or family history
(such as cyclosporin) have been used with success. of autoimmune conditions, and female gender,
There is data supporting the use of budesonide in though the sex ratio is only 3.6:1. This 27-year-
place of prednisone, but this regimen is not as effec- old woman with a prior history of autoimmune
tive in patients with cirrhosis or advanced fibrosis. thyroid disease presents with apparent cirrhosis
(low platelets, splenomegaly, hypoalbuminemia
REFERENCES and a small nodular liver) and a moderate transa-
Manns MP, Czaja AJ, Gorham JD, et al. Diagnosis minitis, which can be the initial presentation in
and management of autoimmune hepatitis. Hepa- up to one-third of adults and half of children with
tol 2010;51:1-31. AIH. The diagnosis of autoimmune hepatitis can
European Association for the Study of the be made based on the Revised Original Scor-
Liver. EASL Practice Guidelines: Autoimmune ing System, which includes a number of clinical,
hepatitis. J Hepatol 2015;63:971-1004. serologic, and histologic markers of AIH. Sero-
logic testing is particularly useful in diagnosing
AIH. Type 1 AIH is characterized by the presence
Question 32 of ANA, anti-smooth muscle antibodies (ASMA),
A 27-year-old woman with a history of hypothy- and anti-SLA (soluble liver antigen). Type 2 AIH
roidism presents to the ED with a complaint of (seen more often in children) is characterized by
fatigue and jaundice. Her exam is remarkable the presence of anti-LKM (liver-kidney-micro-
for moderate RUQ tenderness on palpation. Her somal) or anti-LC-1 antibodies. Standard screen-
CBC: WBC 11.1K cells/mm3, Hgb 11.1 mg/dL, ing in suspected cases of AIH includes ANA,
PLT 120K cells/mm3. Liver function tests: AST ASMA, and anti-LKM, with other antibody testing
103 U/L, ALT 110 U/L, ALP 95 U/L, T. bilirubin done in equivocal cases.
3.8 mg/dL, Albumin 3.0 g/L, TP = 8.1 g/L, INR
1.1. RUQ ultrasound revealed a normal GB with REFERENCES
no gallstones, pericholecystic fluid, or GB wall Manns MP, Czaja AJ, Gorham JD, et al. Diagno-
thickening. CBD was three millimeters, liver was sis and management of autoimmune hepatitis.
of small size with a nodular contour, and no liver Hepatol 2010;51:1-31.
masses were seen. Doppler study showed normal European Association for the Study of the
flow in the hepatic and portal veins. Moderate Liver. EASL Practice Guidelines: Autoimmune
splenomegaly was present. Which of the following hepatitis. J Hepatol 2015;63:971-1004.
antibodies would likely be positive in this patient?
170 Digestive Diseases Self-Education Program®

Question 33 Manns MP, Czaja AJ, et al. Diagnosis and


A 21-year-old woman is diagnosed with autoim- Management of Autoimmune Hepatitis. Hepatol-
mune hepatitis and is started on prednisone and ogy 2010;51: 1-31.
azathioprine. Within a week, she develops mid-
abdominal pain, radiating to the back, and her
lipase level is 537 U/L. What alternative therapy Question 34
may be useful in this patient? A 13-year-old boy with cystic fibrosis is found to
have significant hepatosplenomegaly on physical
A. Cyclophosphamide exam. His CBC: WBC 9.1K cells/mm3, Hgb 12.1
B. Anakinra mg/dL, PLT 130K cells/mm3. Liver function tests:
C. Mycophenelate mofetil AST 97 U/L, ALT 110 U/L, ALP 105 U/L, T. bili-
D. Infliximab rubin 2.1 mg/dL, Albumin 3.7 g/L, GGT 375 U/L,
E. Natalizumab INR 0.9. RUQ ultrasound revealed a normal GB,
CBD 3 mm, and an enlarged echogenic liver. Dop-
CORRECT ANSWER: C pler study showed normal flow in the hepatic and
portal veins. Which of the following medications
RATIONALE may slow the progression of his liver injury?
The two standard treatment regimens for AIH
include corticosteroids (prednisone or predniso- A. Corticosteroids
lone) alone, or corticosteroids combined with B. Ursodeoxycholic acid
azathioprine. The combination regimen allows C. Pancreatic enzyme replacement
for a lower dose of steroids and a lower inci- D. Rifaximin
dence of side effects with the same therapeutic E. Budesonide
efficacy. This patient appears to have developed
azathioprine-induced pancreatitis, which is a rare CORRECT ANSWER: B
complication, more often seen in patients with
Crohn’s disease who are treated with azathioprine. RATIONALE
In patients who are intolerant of azathioprine (or Patients with CF frequently experience hepatobiliary
who fail to respond to therapy), mycophenolate complications, ranging from asymptomatic LFT el-
mofetil and calcineurin inhibitors (such as cy- evations (primarily AST, ALT, and GGT) to cirrhosis
closporin) have been used with success. There is and portal hypertension, which occurs in approxi-
data supporting the use of budesonide in place of mately five percent of cases. A small number of chil-
prednisone, but this regimen is not as effective in dren with CF experience neonatal cholestasis, with
patients with cirrhosis or advanced fibrosis, so it significant conjugated hyperbilirubinemia, while
is reserved for patients with lesser degrees of liver others experience a gradual progression to cirrhosis
fibrosis. The TNF alpha inhibitors are not used to by late childhood. The pathogenesis of liver disease
treat AIH, nor is the IL-1 inhibitor anakinra. is thought to be due to the production of thick biliary
secretions, similar to the thick mucus and pancreatic
REFERENCES secretions caused by the CFTR mutation, which ob-
Czaja AJ. Diagnosis and Management of Autoim- struct the biliary tree and induce inflammation and
mune Hepatitis: Current Status and Future Direc- fibrosis. The diagnostic criteria for CF induced liver
tions. Gut and Liver 2016;10:177-203. disease (CFLD) include hepatomegaly and or spleno-
European Association for the Study of the megaly, abnormal AST/ALT/GGT, ultrasound evi-
Liver. EASL Clinical Practice Guidelines: Au- dence of cirrhosis, or biopsy proven biliary cirrhosis:
toimmune Hepatitis. Journal of Hepatology if any two if these are present, the diagnosis of CFLD
2015:63:971-1004. is made. Ursodiol may ameliorate CFLD, though the
Chapter 6 — Metabolic, hereditary, inflammatory and vascular diseases of the liver 171

data is not conclusive, and it is most often used in inflammation. Progression to cirrhosis and ESLD is
children with significant cholestatic liver disease. rare, but patients with GD have an increased risk of
HCC (as well as hematologic malignancies). Treat-
REFERENCE ment of GD with drugs such as eliglustat will lead to
Sathe MN, Freeman AJ. Gastrointestinal, a marked decrease in liver and spleen size.
Pancreatic, and Hepatobiliary Manifesta-
tions of Cystic Fibrosis. Pediatr Clin North Am REFERENCE
2016;63:679-698. Nagral A. Gaucher Disease. J Clin Exp Hepatol
2014;4:37-50.

Question 35
A 13-year-old boy of Eastern European descent, Question 36
with a history of humeral and femoral fractures in A four-month-old child is evaluated for lethargy
the past, is found to have significant hepatospleno- and seizures that occur between feedings. Exam is
megaly on physical exam. His CBC: WBC 9.1K remarkable for hepatomegaly and skin xanthomas.
cells/mm3, Hgb 9.1 mg/dL, PLT 95K cells/mm3. He is found to be profoundly hypoglycemic between
Liver function tests: AST 31 U/L, ALT 33 U/L, ALP feedings, and responds to frequent feedings. Even
95 U/L, T. bilirubin 0.8 mg/dL, Albumin 4.0 g/L, with control of his metabolic derangement, which
INR 0.9. RUQ ultrasound revealed a normal GB, long-term sequela may be expected in this child?
CBD three millimeters, markedly enlarged liver and
spleen with no liver masses noted. Doppler study A. Cirrhosis
showed normal flow in the hepatic and portal veins. B. Hepatocellular carcinoma
Which of the following enzyme deficiencies may be C. Insulinoma
responsible for this clinical syndrome? D. Episodic hyperbilirubinemia
E. Movement disorders
A. Glycogen phosphorylase
B. Glucose 6-phosphate dehydrogenase CORRECT ANSWER: B
C. Glucocerebrosidase
D. Hexosaminidase A RATIONALE
E. Sphingomyelinase Glycogen storage disease type I (von Gierke’s Dis-
ease) is an inborn error of metabolism occurring in
CORRECT ANSWER: C 1/100,000 births. A defect in glucose-6-phosphatase
makes it impossible for the child to break down gly-
RATIONALE cogen into glucose. The glycogen-rich liver becomes
Gaucher’s Disease (GD) is a lysosomal storage enlarged, while the lack of glucose production leads
disease caused by mutations in the beta-glucocer- to profound hypoglycemia and lactic acidosis be-
ebrosidase gene, which leads to accumulation of tween feedings, manifesting as lethargy and seizures.
glucocerebroside in macrophages, with the liver, Patients also have significant hyperlipidemia and may
spleen, and bone marrow being the major sites of develop xanthomata. Dietary management (frequent
accumulation. This is an autosomal recessive disor- feedings either orally or via gastrostomy tube) can
der that has a high prevalence in Ashkenazi Jew- manage the hypoglycemia, but patients remain at risk
ish populations. Clinical manifestations typically for the development of hepatic adenomas, which may
include hepatosplenomegaly, and liver enzymes undergo malignant transformation in up to 10 percent
may be normal or only mildly elevated. On liver of cases. Liver transplantation may be considered
biopsy, Gaucher’s cells (lipid-laden macrophages) when HCC is identified, and leads to correction of
are prominent, with variable amounts of associated much of the metabolic defect in this condition.
172 Digestive Diseases Self-Education Program®

REFERENCE REFERENCE
Rake JP, Visser G et al. Glycogen storage disease Rake JP, Visser G et al. Glycogen storage disease
type I: diagnosis, management, clinical course type I: diagnosis, management, clinical course
and outcome. Results of the European Study on and outcome. Results of the European Study on
Glycogen Storage Disease Type I (ESGSD I). Eur J Glycogen Storage Disease Type I (ESGSD I). Eur J
Pediatrics 2002;161 Supp1:S20. Pediatrics 2002;161 Supp1:S20.

Question 37 Question 38
A 14-week-old child presents to his pediatrician A 72-year-old man with a history of CAD and PVD
with lethargy and seizures that occur between feed- is day five post-op from aorto-bifemoral bypass
ings. Exam is remarkable for hepatomegaly and surgery, and he has developed jaundice. His VS
skin xanthomas on the buttocks. He is found to be are as follows: Pulse 100 bpm and regular, BP
profoundly hypoglycemic between feedings, with 125/70 mm Hg, RR 14, O2 saturation 98 percent
evidence of lactic acidosis on lab evaluation. Liver on room air. His exam is remarkable for jaundice
biopsy will show which of the following findings? and mild RUQ tenderness on palpation. His CBC:
WBC 9.1K cells/mm3, Hgb 12.5 mg/dL, PLT 320K
A. PAS positive/diastase resistant granules in cells/mm3. Liver function tests: AST 1920 U/L,
hepatocytes ALT 2130 U/L, ALP 103 U/L, T. bilirubin 6.1 mg/
B. Microvesicular steatosis within hepatocytes dL, albumin 3.9 g/L, INR 1.0. Serum LDH was
C. Lymphocytic infiltrate with piecemeal necrosis 2600 U/L. RUQ ultrasound revealed a normal
of hepatocytes GB with no gallstones, CBD was three millime-
D. Glycogenosis of hepatocytes ters, liver was of normal size and contour, no liver
E. Normal Histology masses were seen, Doppler study showed normal
flow in the hepatic and portal veins. Which of the
CORRECT ANSWER: D following treatments should be administered?

RATIONALE A. Ursodeoxycholic acid


Glycogen storage disease type I (von Gierke’s Dis- B. Intravenous heparin
ease) is an inborn error of metabolism occurring in C. Tenofovir dipivoxil
1/100,000 births. A defect in glucose-6-phosphatase D. Intravenous methylprednisolone
makes it impossible for the child to break down gly- E. Supportive care only
cogen into glucose. The glycogen-rich liver becomes
enlarged, while the lack of glucose production leads to CORRECT ANSWER: E
profound hypoglycemia and lactic acidosis between
feedings, manifesting as lethargy and seizures. Patients RATIONALE
also have significant hyperlipidemia and may develop Ischemic hepatitis, due to an acute lowering of
xanthomata. Dietary management (frequent feed- blood flow to the liver, is an uncommon condi-
ings either orally or via gastrostomy tube) can manage tion, accounting for one to three percent of ICU
the hypoglycemia, but patients remain at risk for the admissions. It is often called “shock liver,” but only
development of hepatic adenomas, which may undergo 50 percent of patients have overt shock, so there
malignant transformation in up to 10 percent of cases. should be a high index of suspicion in patients
PAS positive/diastase resistant granules in hepatocytes with a less obvious decrease in hepatic perfusion.
arefound in A1 AT disease. Lymphocytic infiltrate with Patients may experience symptoms of nausea, RUQ
piecemeal necrosis of hepatocytes would be character- pain, and malaise, and the liver enzyme pattern is
istic of chronic hepatitis as with HCV. characterized by markedly elevated transaminases
Chapter 6 — Metabolic, hereditary, inflammatory and vascular diseases of the liver 173

and an elevated serum LDH. Alkaline phospha- so there should be a high index of suspicion in
tase levels are only mildly elevated. Transaminases patients with a less obvious decrease in hepatic
usually decrease rapidly and normalize within two perfusion. Patients may experience symptoms
weeks, but elevations in bilirubin may persist after of nausea, RUQ pain, and malaise, and the liver
the transaminases improve. Treatment consists of enzyme pattern is characterized by markedly
supportive care and restoration of cardiac output elevated transaminases and an elevated serum
and liver perfusion – no specific drug therapy is LDH. Alkaline phosphatase levels are only mildly
recommended in ischemic hepatitis. elevated. Transaminases usually decrease rapidly
and normalize within two weeks, but elevations
REFERENCES in bilirubin may persist after the transaminases
Lightsey JM and Rockey DC. Current Concepts improve. Liver biopsy typically reveals hepatocyte
in Ischemic Hepatitis. Curr Opin Gastroenterol necrosis in Zone 3 of the acinus with few inflam-
2017;33:158-163. matory cells. Treatment consists of supportive
Tapper EB, Sengupta N, Bonder A. The In- care and restoration of cardiac output and liver
cidence and Outcomes of Ischemic Hepatitis: A perfusion – no specific drug therapy is recom-
Systematic Review with Meta-analysis. Am J Med mended in ischemic hepatitis.
2015;128:1314-1321.
REFERENCES
Lightsey JM and Rockey DC. Current Concepts
Question 39 in Ischemic Hepatitis. Curr Opin Gastroenterol
A 64-year-old man with a history of CAD and PVD 2017;33:158-163.
is day five post-op from a three-vessel CABG. His Tapper EB, Sengupta N, Bonder A. The In-
VS are as follows: Pulse 100 bpm and regular, BP cidence and Outcomes of Ischemic Hepatitis: A
125/70 mm Hg, RR 14, O2 saturation 98 percent Systematic Review with Meta-analysis. Am J Med
on room air. His exam is remarkable for scleral ic- 2015;128:1314-1321.
terus and mild RUQ tenderness on palpation. His
liver function tests: AST 1538 U/L, ALT 1684 U/L,
ALP 112 U/L, T. bilirubin 4.1 mg/dL, albumin 3.9 Question 40
g/L, INR 1.0. Serum LDH is 2000 U/L. Which of Which of the following is a risk factor for ischemic
the following pathologic lesions would be seen on cholangiopathy?
liver biopsy?
A. Primary biliary cholangitis
A. Periportal lymphocytic infiltrates B. Liver transplantation
B. Steatohepatitis C. Budd-Chiari Syndrome
C. Sinusoidal dilation D. Autoimmune hepatitis
D. Zone 3 hepatocyte necrosis E. Acetaminophen toxicity
E. PAS positive granules within hepatocytes
CORRECT ANSWER: B
CORRECT ANSWER: D
RATIONALE
RATIONALE Ischemic cholangiopathy is a rare condition of
Ischemic hepatitis, due to an acute lowering of bile duct injury due to impaired blood supply. It
blood flow to the liver, is an uncommon condi- is most commonly associated with liver trans-
tion, accounting for one to three percent of ICU plantation and with the administration of hepatic
admissions. It is often called “shock liver,” but intra-arterial chemotherapy, but it is also seen in
only 50 percent of patients have overt shock, AIDS cholangiopathy, polyarteritis nodosa, and
174 Digestive Diseases Self-Education Program®

hereditary hemorrhagic telangiectasia. Clinical CORRECT ANSWER: B


features can range from asymptomatic liver en-
zyme abnormalities (mainly alkaline phosphatase RATIONALE
and GGT) to progressive cholestasis with jaundice. Portal vein thrombosis (PVT) is a relatively com-
Ischemic stenosis of the CBD can also present mon occurrence in patients with hepatic cirrhosis,
as acute obstructive jaundice or cholangitis. The occurring in eight to 25 percent of patients with
finding of diffuse biliary structuring following decompensated liver disease. When a cirrhotic
liver transplantation should prompt evaluation of patient has evidence of PVT, the possibility of
hepatic artery flow. Ischemic cholangiopathy is HCC-related tumor thrombus should be enter-
most often seen following transplantation using tained, and an AFP level and further imaging
donation after circulatory death organs. should be obtained. Tumor thrombus should
suspected if endoluminal material enhances
REFERENCE during the arterial phase of imaging or if there
Deltenre P, Valla DC. Ischemic Cholangiopathy. J is evidence of arterial pulsatile flow is seen on
Hepatol, 2017;44:806-817. Doppler ultrasound. PVT can occur despite a
patient’s thrombocytopenia and/or an elevated
INR. The INR in cirrhotic patients may not truly
Question 41 reflect the degree of anticoagulation, as liver
A 51-year-old woman with a history of HCV cirrho- dysfunction decreases levels of natural anticoagu-
sis presents to the ED with a complaint of two days lant factors such as proteins C and S in addition
of RUQ pain. She has no h/o ascites, but has Grade to the levels of clotting factors. Thus, though the
2 varices on EGD one year ago, currently being INR accurately reflects liver synthetic function,
treated with nadolol. Her VS are as follows: Pulse the degree of hypo- or hypercoagulability cannot
67 bpm and regular, BP 125/70 mm Hg, RR 14, be estimated from this value alone, and patients
O2 saturation 98 percent on room air. Her exam with elevated INR may in fact develop PVT. Sys-
is remarkable for moderate RUQ tenderness on temic anticoagulation has been shown to improve
palpation. Her CBC: WBC 11.1K cells/mm3, Hgb outcomes in cirrhotic PVT, but anticoagulation
11.1 mg/dL, PLT 102K cells/mm3. Liver function should be delayed until variceal bleeding prophy-
tests: AST 95 U/L, ALT 130 U/L, ALP 102 U/L, laxis has been implemented with band ligation.
T. bilirubin 1.2 mg/dL, Albumin 2.9 g/L, INR 1.3. Options for anticoagulation include LMWH and
RUQ ultrasound revealed a normal GB, CBD three warfarin; there is too little data on NOACs to rec-
millimeters, liver was of small size with a nodular ommend their use at this time. With anticoagula-
contour, and no liver masses were seen. Doppler tion, rates of recanalization of the PV range from
study showed normal flow in the hepatic veins and 55-75 percent in a mean interval of 6 months.
a thrombus in the main PV, with no evidence of Therapy should be given for at least six months,
collateralization, and no arterial pulsations within and recanalization should be sought with periodic
the clot. A serum AFP level is normal. What is the ultrasound.
best next step in the management of this patient?
REFERENCES
A. Begin therapy with a NOAC immediately. DeLeve LD, Valla D-C, Garcia-Tsao G. Vas-
B. Perform ligation of her varices, then cular Disorders of the Liver. Hepatology
anticoagulate 2009;49:1729-1763.
C. Continue to observe her and recheck a European Association for the Study of the
Doppler study in three months Liver. EASL Clinical Practice Guidelines: Vascu-
D. Refer her for urgent liver transplant lar Diseases of the Liver. Journal of Hepatology
E. Begin DAA therapy for her HCV infection 2016;64:179-202.
Chapter 6 — Metabolic, hereditary, inflammatory and vascular diseases of the liver 175

Question 42 percent of cases. Other common causes include


A 40-year-old woman with no significant past Protein C and S deficiency, antithrombin deficien-
history presents to the ED with a complaint of two cy, and Factor V Leiden mutation. Antiphospholip-
days of RUQ pain. She takes no medications and id antibody syndrome and PNH are both possible
is generally healthy. Her VS are as follows: Pulse causes of PVT, but are significantly less likely.
97 bpm and regular, BP 135/70 mm Hg, RR 14,
O2 saturation 98 percent on room air. Her exam REFERENCES
is remarkable for moderate RUQ tenderness on DeLeve LD, Valla D-C, Garcia-Tsao G. Vas-
palpation, mild hepatomegaly, and moderate sple- cular Disorders of the Liver. Hepatology
nomegaly. Her CBC: WBC 9.1K cells/mm3, Hgb 2009;49:1729-1763.
14.1 mg/dL, PLT 480K cells/mm3. Liver function European Association for the Study of the
tests: AST 95 U/L, ALT 112 U/L, ALP 102 U/L, T. Liver. EASL Clinical Practice Guidelines: Vascu-
bilirubin 0.9 mg/dL, Albumin 4.0 g/L, INR 1.0. lar Diseases of the Liver. Journal of Hepatology
RUQ ultrasound revealed a normal GB, CBD 3 2016;64:179-202.
mm, liver was of normal size and contour, and no
liver masses were seen. Moderate splenomegaly
was present. Doppler study showed normal flow in Question 43
the hepatic veins and a thrombus in the main PV, A patient with no h/o liver disease presents with
with no evidence of collateralization. What is the two weeks of abdominal pain and is found to
most likely etiology of this patient’s PVT? have evidence of portal vein thrombosis (PVT) on
Doppler ultrasound. A contrast CT scan shows
A. A myeloproliferative disorder clot restricted to the main portal vein, no evidence
B. Hepatocellular carcinoma of collateralization, and no evidence of cirrhosis
C. Antiphospholipid antibody syndrome or HCC. A hypercoagulable workup is negative,
D. Paroxysmal Nocturnal Hemoglobinuria and anticoagulation is begun (LMWH followed by
E. Polyarteritis nodosa warfarin). A six-month course of anticoagulation
is expected to lead to complete recanalization of
CORRECT ANSWER: A the PV in what percentage of patients?

RATIONALE A. 10 percent
Portal vein thrombosis (PVT) typically presents B. 25 percent
with abdominal pain, and may be accompanied by C. 50 percent
fever or other inflammatory markers. If the clot is D. 75 percent
confined to the portal vein, symptoms are mild-to- E. 90 percent
moderate, but if there is extension into the SMV,
intestinal ischemia or infarction may occur. Dop- CORRECT ANSWER: C
pler ultrasound is used to make the diagnosis of
PVT, with an absence or decreased flow in the PV RATIONALE
lumen. If Doppler flow is seen within the obstruct- Acute portal vein thrombosis in patients without
ing thrombus, or if there is evidence of pulsatile ar- cirrhosis is an uncommon condition, typically as-
terial flow in the thrombus, the possibility of tumor sociated with a hypercoagulable state. Myelopro-
thrombus must be entertained. CT will confirm the liferative disorders are the most common etiology
diagnosis, showing a luminal filling defect without of PVT, seen in 30-40 percent of cases. Other
enhancement. In the absence of cirrhosis, a num- common causes include Protein C and S deficien-
ber of hypercoagulable states may result in PVT, cy, antithrombin deficiency, and Factor V Leiden
with myeloproliferative disorders present in 30-40 mutation. Anticoagulation therapy is indicated in
176 Digestive Diseases Self-Education Program®

acute PVT, with the goal of preventing extension E. Use of a cyclophosphamide-based


of clot into the mesenteric veins, and recanalizing myeloablative regimen
the portal system to prevent portal hypertension.
Initial treatment with LMWH, followed by oral an- CORRECT ANSWER: B
ticoagulation, leads to complete PV recanalization
in 50 percent of patients, with partial recanaliza- RATIONALE
tion in an additional 40 percent. Current guide- Sinusoidal obstruction syndrome (SOS, formerly
lines recommend at least three months of antico- Hepatic Veno-occlusive Disease) is a liver disorder
agulation, with lifelong therapy in patients with an characterized by injury to the sinusoidal vascular
identifiable prothrombotic disorder. epithelium leading to fibrinogen deposition, vascu-
lar congestion, and eventually hepatocyte necrosis
REFERENCES in a centrilobular pattern. SOS is most often seen
DeLeve LD, Valla D-C, Garcia-Tsao G. Vas- in patients undergoing hematopoietic cell trans-
cular Disorders of the Liver. Hepatology plant and is associated with specific myeloablative
2009;49:1729-1763. regimens, especially those containing cyclophos-
European Association for the Study of the phamide or high dose XRT. SOS is also seen after
Liver. EASL Clinical Practice Guidelines: Vascu- exposure to other toxins, including Jamaican “bush
lar Diseases of the Liver. Journal of Hepatology tea,” a beverage containing pyrrolizidine alkaloids.
2016;64:179-202. Clinically, SOS is characterized by the rapid onset
of ascites, edema, and weight gain, typically within
three weeks of transplantation. Tender hepato-
Question 44 megaly is common, as are elevation of AST, ALT,
A 30-year-old woman with a history of AML and bilirubin. A minority of patients develop over
underwent Hematopoietic Stem Cell Transplanta- hepatic failure with encephalopathy and coagulopa-
tion (HSCT) 15 days ago. She has now developed thy. Diagnostic criteria include the Seattle criteria
abdominal distension and weight gain, RUQ (serum bilirubin greater than two mg/dL, hepa-
pain, and jaundice. Her VS are as follows: Pulse tomegaly or RUQ pain, and weight gain of more
97 and regular, BP 110/70, RR 14, O2 saturation than two percent of body weight) and the Baltimore
98 percent on room air. Her exam is remarkable criteria (bilirubin more than two mg/dL and two of
for moderate RUQ tenderness on palpation, and the following: hepatomegaly, ascites, weight gain
abdominal distension with shifting dullness and greater than five percent). The incidence of SOS can
a fluid wave. Liver function tests show an AST 121 be reduced by choosing a less toxic myeloablative
U/L, ALT 130 U/L, ALP 109 U/L, T. bilirubin 3.8 regimen and by administering ursodeoxycholic acid
mg/dL, Albumin 3.0 g/L, INR 1.1. RUQ ultra- or low-dose heparin in the pre-transplant period.
sound revealed a normal GB and moderate ascites.
Liver was enlarged and no liver masses were seen. REFERENCES
Doppler study showed normal flow in the hepatic DeLeve LD, Valla D-C, Garcia-Tsao G. Vas-
and portal veins. This condition resolved with sup- cular Disorders of the Liver. Hepatology
portive treatment. Which of the following medica- 2009;49:1729-1763.
tions may reduce the incidence of this condition in European Association for the Study of the
patients undergoing HSCT? Liver. EASL Clinical Practice Guidelines: Vascu-
lar Diseases of the Liver. Journal of Hepatology
A. Methylprednisolone 2016;64:179-202.
B. Ursodeoxycholic acid Valla D-C, Cazals-Hatem D. Sinusoidal Obstruc-
C. Epsilon aminocaproic acid tion Syndrome. Clin Res Hepatol Gastroenterol
D. Octreotide 2016;40:378-385.
Chapter 6 — Metabolic, hereditary, inflammatory and vascular diseases of the liver 177

Question 45 gain of greater than two percent of body weight)


A 27-year-old man with a history of AML under- and the Baltimore criteria (bilirubin greater than
went Hematopoietic Stem Cell Transplant (HSCT) two mg/dL and two of the following: hepatomeg-
18 days ago. He now complains of abdominal dis- aly, ascites, weight gain more than five percent).
tension and an 11 pound weight gain, RUQ pain, The incidence of SOS can be reduced by choosing
and jaundice. His exam is remarkable for moder- a less toxic myeloablative regimen and by admin-
ate RUQ tenderness on palpation, and abdominal istering ursodeoxycholic acid or low-dose heparin
distension with shifting dullness and a fluid wave. in the pre-transplant period.
Liver function tests show an AST 135 U/L, ALT
170 U/L, ALP 120 U/L, T. bilirubin 4.8 mg/dL, Al- REFERENCES
bumin 3.0 g/L, INR 1.0. RUQ ultrasound revealed DeLeve LD, Valla D-C, Garcia-Tsao G. Vascular Dis-
significant ascites and an enlarged liver. Doppler orders of the Liver. Hepatology 2009;49:1729-1763.
study showed normal flow in the hepatic and por- European Association for the Study of the
tal veins. Which of the following is the most likely Liver. EASL Clinical Practice Guidelines: Vascu-
diagnosis in this patient? lar Diseases of the Liver. Journal of Hepatology
2016;64:179-202.
A. Acute cellular rejection Valla D-C, Cazals-Hatem D. Sinusoidal Ob-
B. Hepatosplenic candidiasis struction Syndrome. Clin Res Hepatol Gastroen-
C. Sinusoidal obstruction syndrome terol 2016;40:378-385.
D. Vanishing bile duct syndrome
E. Acute viral hepatitis
Question 46
CORRECT ANSWER: C A 50-year-old man presents to the ED with
hematemesis and melena. He reports frequent
RATIONALE episodes of abdominal pain. He drinks approxi-
Sinusoidal obstruction syndrome (SOS, formerly mately one pint of whiskey daily, and has done so
Hepatic Veno-occlusive Disease) is a liver disorder for more than 20 years. His exam is remarkable
characterized by injury to the sinusoidal vascular for moderate epigastric tenderness. Liver function
epithelium leading to fibrinogen deposition, vas- tests: AST 105 U/L, ALT 60 U/L, ALP 110 U/L, T.
cular congestion, and eventually hepatocyte necro- bilirubin 1.0 mg/dL, Albumin 3.8 g/L, INR 1.0.
sis in a centrilobular pattern. SOS is most often RUQ ultrasound revealed a normal GB without
seen in patients undergoing hematopoietic cell gallstones, pericholecystic fluid, and GB wall
transplant and is associated with specific myeloab- thickening. CBD was 3 mm, liver mildly enlarged,
lative regimens, especially those containing cyclo- no liver masses were seen. A Fibroscan reveals F2
phosphamide or high dose XRT. SOS is also seen fibrosis. EGD reveals large fundic gastric varices
after exposure to other toxins, including Jamaican with no esophageal varices. What is the most
“bush tea,” a beverage containing pyrrolizidine likely cause of his varices?
alkaloids. Clinically, SOS is characterized by the
rapid onset of ascites, edema, and weight gain, A. Portal vein thrombosis
typically within three weeks of transplantation. B. Splenic vein thrombosis
Tender hepatomegaly is common, as are elevation C. Hepatic artery thrombosis
of AST, ALT, and bilirubin. A minority of patients D. Hepatic vein thrombosis
develop over hepatic failure with encephalopathy E. Sinusoidal Obstruction Syndrome
and coagulopathy. Diagnostic criteria include the
Seattle criteria (serum bilirubin greater than two CORRECT ANSWER: B
mg/dL, hepatomegaly or RUQ pain, and weight
178 Digestive Diseases Self-Education Program®

RATIONALE A. Elevated serum succinoacetone level


Isolated gastric varices (IGV1) may be caused by B. Decreased serum alpha-1 antitrypsin level
splenic vein thrombosis, seen in chronic pancre- C. Decreased serum ceruloplasmin level
atitis or after pancreatic surgery. This patient has D. Decreased porphyrin decarboxylase activity
no evidence of cirrhosis, but episodes of abdomi- E. Decreased pyruvate kinase activity
nal pain may be indicative of recurrent acute or
chronic pancreatitis. There is limited data on the CORRECT ANSWER: A
management of these varices, as most case series
have larger numbers of GOV1 than IGV or GOV2, RATIONALE
but non-selective beta blockers may be used for In Hereditary Tyrosinemia Type I is a rare meta-
prophylaxis of gastric variceal bleeding. For acute bolic disorder, affecting one in 12,000 - 100,000
bleeding from IGV1, injection therapy with cyano- newborns of European descent, but is significant-
acrylate glue has been shown in small trials to be ly more common in French-Canadian lineages.
superior to EVL in initial hemostasis and preven- The metabolism of tyrosine leads to the produc-
tion of early rebleeding. tion of acetoacetate or fumarate, but in patients
In the setting of portal hypertension, TIPS is lacking fumaryloacetate hydrolase, a toxic inter-
recommended if endoscopic therapy fails. For the mediate accumulates. Clinical manifestations
prevention of rebleeding, Balloon-occluded Ret- include progressive liver injury and renal tubular
rograde Transvenous Obliteration (BRTO) may be dysfunction that manifests as Fanconi syndrome,
effective. This procedure is feasible only if there with RTAs, phosphate wasting, and aminoacid-
are sufficient gastro/splenorenal collaterals, which uria. Liver disease is characterized by hepato-
can be seen on CT scan. In this procedure, the megaly, hyperbilirubinemia, and an elevated AFP
left renal vein is cannulated, and the splenorenal level. This may present as a rapidly progressive
collaterals and fundic varices are obliterated using liver disease or progress slowly to cirrhosis, with
sclerosants or coils. This procedure may be paired an elevated risk of HCC. If the disease is diag-
with a TIPS to prevent increases in portal pressure nosed early (via newborn screening), it can be
after obliteration of the collateral vessels. treated with a diet low in tyrosine and phenylala-
nine, as well as the use of nitisinone, which blocks
REFERENCE early steps in the tyrosine pathway and prevents
Garcia-Pagan JC, Barrufet M et al. Manage- the formation of the toxic metabolites. Liver
ment of Gastric Varices. Clin Gastro Hepatol transplantation is indicated for patients with
2014;12:919-928. severe disease unresponsive to therapy.

REFERENCE
Question 47 Chinsky JM, Singh R et al. Diagnosis and Treat-
A two-month-old boy of French-Canadian descent ment of Tyrosinemia Type 1: A US and Canadian
presents with hypoglycemia, jaundice, and failure Consensus Group review and Recommendations.
to thrive. He is found to have significant hepato- Genet Med 20178;19:1380.
megaly on physical exam. His liver function tests:
AST 71 U/L, ALT 90 U/L, ALP 105 U/L, T. bilirubin
4.6 mg/dL, Albumin 3.7 g/L, INR 0.9. His AFP is Question 48
markedly elevated. He also has significant amino- A 27-year-old man with a history of bipolar disor-
aciduria and renal tubular acidosis. der presents to the ED with confusion and jaun-
dice. His VS are as follows: Pulse 100 bpm and
Which of the following tests will be abnormal in regular, BP 125/70 mm Hg, RR 14, O2 saturation
this child? 98 percent on room air. His exam is remarkable
Chapter 6 — Metabolic, hereditary, inflammatory and vascular diseases of the liver 179

for jaundice and moderate RUQ tenderness on REFERENCE


palpation. His CBC: WBC 9.1K cells/mm3, Hgb European Association for the Study of the Liver.
8.5 mg/dL, MCV 99, PLT 120K cells/mm3. Liver EASL Clinical Practice guidelines: Wilson’s Dis-
function tests: AST 1250 U/L, ALT 1555 U/L, ALP ease. Journal of Hepatology 2012;56:671-685.
53 U/L, T. bilirubin 4.1 mg/dL, Albumin 3.9 g/L, Roberts EA and Schilsky ML. Diagnosis and
INR 1.7. RUQ ultrasound revealed a normal GB Treatment of Wilson Disease: An Update. Hepa-
with no gallstones, CBD was three millimeters, tology 2008:47:2089-2111.
liver was of normal size and contour, no liver
masses were seen, Doppler study showed normal
flow in the hepatic and portal veins. Reticulocyte Question 49
count is 24 percent, Haptoglobin is less than six A 27-year-old man with a history of schizoaffec-
mg/dL, LDH is significantly elevated, and periph- tive disorder is found to have the following liver
eral smear shows no schistocytes, but macrocytes enzymes: AST 92 U/L, ALT 105 U/L, ALP 80 U/L,
and nucleated RBCs. T. bilirubin 0.9 mg/dL, Albumin 3.0 g/L, INR 1.2.
CBC is remarkable only for PLT of 99K cells/mm3.
This clinical scenario is most compatible with RUQ ultrasound revealed a normal GB with no
which of the following diagnoses? gallstones, CBD was three millimeters, liver was of
small size with a nodular contour, no liver masses
A. Acute HBV infection were seen, Doppler study showed normal flow in
B. Ischemic hepatitis the hepatic and portal veins. Moderate spleno-
C. Wilson’s Disease megaly was present.
D. Autoimmune hepatitis
E. Budd-Chiari Syndrome Which of the following tests would be compatible
with a diagnosis of Wilson’s Disease?
CORRECT ANSWER: C
A. Serum non-ceruloplasmin bound copper level
RATIONALE of less than 150 mcg/L
This is a case of acute Wilsonian hepatitis. Clini- B. Urine copper level of 180 mcg/24 hrs
cal clues include the marked transaminitis and C. Serum ceruloplasmin level of 60 mg/L
evidence of liver failure with a low alkaline phos- D. White matter lesions on brain MRI
phatase level, and a Coombs negative hemolytic E. Antiphospholipid antibody titers of more than
anemia due to copper-induced lysis of RBCs. The 1:2500
patient’s psychiatric history is also suggestive,
as many patients with Wilson’s disease have a CORRECT ANSWER: B
history of neuro-psychiatric disorders, includ-
ing Parkinsonian movement disorders, psychotic RATIONALE
disorders. Acute hepatitis B, ischemia, and Wilson Disease (WD) is caused by a mutation in
autoimmune hepatitis may all present with mark- the P-type ATP-ase that mediates the excretion
edly elevated transaminases, but the associated of copper into the bile. Failure to excrete copper
non-immune hemolysis and low ALP are atypical leads to deposition in the liver, eyes, and brain.
for these other disorders. Budd-Chiari syndrome Hepatic involvement can manifest as chronic
may also present with a marked transaminitis, progressive liver injury leading to cirrhosis, and
but it is typically characterized by ascites and an acute fulminant hepatitis that is treatable only
significant tender hepatomegaly, and Doppler with transplant. Neurologic symptoms are due
studies would show evidence of hepatic vein to deposition in the basal ganglia and other brain
thrombosis. structures and include Parkinsonian symptoms,
180 Digestive Diseases Self-Education Program®

dysautonomia, and behavioral changes. The CORRECT ANSWER: B


diagnosis of WD is made via several biochemi-
cal tests. Serum ceruloplasmin less than 20 mg/ RATIONALE
dL warrants further workup for Wilson’s Disease, A serum ceruloplasmin less than five mg/L and a
while a level less than five mg/dL is highly sugges- 24-hour urine copper excretion greater than 100
tive. 24-hour urine copper levels of greater than mcg/24 hrs are both highly suggestive of Wilson’s
100 mcg are highly suggestive, while levels greater Disease, a disorder of copper metabolism caused
than 40 mcg/24 hrs warrant additional testing. by a mutation in a P-type ATP-ase that mediates
Serum copper levels are often decreased due to the the excretion of copper into the bile. Treatment of
decrease in circulating ceruloplasmin, which is the Wilson’s Disease consists of copper chelation ther-
major copper-binding protein in serum. However, apy. Commonly-used therapies include D-peni-
non-ceruloplasmin bound copper will be elevated, cillamine, trientine, and zinc. Patients on therapy
typically above 250 mcg/L. Liver biopsy may also should have 24-hour urine copper determination
be used to diagnose WD, with a hepatic copper every six to 12 months. Patients on maintenance
content of greater than 250 mcg/gram highly sug- trientine or D- penicillamine should have urine
gestive of this diagnosis. Kayser-Fleischer rings, copper excretion of 200-500 mcg/24 hrs. Patients
when present, are a strong indicator of WD in a on zinc therapy should have much lower copper
patient with a low serum ceruloplasmin. How- excretion, in the range of 75 mcg/24 hrs.
ever, this finding is seen in only 44-62 percent of
cases of isolated hepatic disease, while it is almost REFERENCES
always present in patients with neurologic WD. European Association for the Study of the Liver.
EASL Clinical Practice guidelines: Wilson’s Dis-
REFERENCE ease. Journal of Hepatology 2012;56:671-685.
European Association for the Study of the Liver. Roberts EA and Schilsky ML. Diagnosis and
EASL Clinical Practice guidelines: Wilson’s Dis- Treatment of Wilson Disease: An Update.
ease. Journal of Hepatology 2012;56:671-685. Hepatology 2008:47:2089-2111.
Roberts EA and Schilsky ML. Diagnosis and
Treatment of Wilson Disease: An Update. Hepa-
tology 2008:47:2089-2111.

Question 50
A 19-year-old male with elevated liver enzymes
is found to have a serum ceruloplasmin of three
mg/L, and a urine copper excretion of 210 mcg/24
hrs. He is started on trientine to treat his ill-
ness. What level of urine copper is the goal of this
therapy?

A. No urine copper should be found if therapy


is effective
B. Urine copper should be 200-500 mcg/24 hrs
C. Urine copper should be 500-1000 mcg/24 hrs
D. Urine copper should be 1000-1500 mcg/24 hrs
E. Urine copper should be more than 1500
mcg/24 hrs
Answers & critiques

CHAPTER 7

Cirrhosis and
liver transplantation
Catherine Lucero, MD and Jamile’ Wakim-Fleming, MD

Question 1 cause of his cirrhosis is related to longstanding


A 63-year-old man with a history of hepatitis C is hepatitis C and given normal liver enzymes, so
seen in your office for evaluation of new diagnosis no further workup is needed. General cirrhosis
of cirrhosis. He has had longstanding hepatitis C care includes screening for hepatocellular carci-
for 30 years and recently achieved cure with 12 noma, variceal screening, and ensuring patient
weeks of direct acting antiviral therapy. He was is vaccinated to hepatitis A and B. Vitamin E and
referred to you after seeing a hematologist for milk thistle are not approved FDA therapies for
thrombocytopenia. He is asymptomatic. His exam cirrhosis related to hepatitis C.
is notable for spider angiomata, a non-distended
abdomen, and palmar erythema. His most recent REFERENCES
labs include a total bilirubin of 1.2 mg/dL, AST 24 Udell JA, et al. Does this Patient with Liver Disease
U/L, ALT 22 U/L, albumin of 3.6 g/dL, INR 1.0, Have Cirrhosis? JAMA. 2012; 307(8): 832-42.
and a platelet count of 145,000/µL. Garcia-Tsao G, et al. Portal hypertensive
bleeding in cirrhosis: Risk stratification, diagno-
Which of the following would be included in this sis, and management: 2016 practice guidance by
patient’s management? the American Association for the study of liver
diseases. Hepatology. 2017; 65(1):310-335.
A. Liver biopsy Heimbach JK, et al. AASLD guidelines for the
B. Abdominal ultrasound treatment of hepatocellular carcinoma. Hepatol-
C. Vitamin E ogy. 2018; 67(1):358-380.
D. Milk thistle
E. Weight loss
Question 2
CORRECT ANSWER: B A 63-year-old man with a history of hepatitis
C undergoes variceal screening and does not
RATIONALE have varices. There is no evidence of hepatocel-
A liver biopsy is not always needed to make lular carcinoma on imaging. He sees you for
the diagnosis of cirrhosis. It can be established follow-up to discuss the results of his tests. On
through clinical, laboratory and/or imaging examination he does not have any evidence of
findings. Laboratory findings of cirrhosis in this encephalopathy, ascites, or jaundice. His labs
patient include spider angiomata, palmar ery- remain unchanged with total bilirubin of 1.2 mg/
thema, and low platelet count. A platelet count dL, AST 24 U/L, ALT 22 U/L, albumin of 3.6 g/
less than 160 x 103/µL has been found to have a dL, INR 1.0, and a platelet count of 145,000/µL.
positive likelihood ratio of 6.3. The most likely The patient agrees to follow you with serial visits

181
182 Digestive Diseases Self-Education Program®

and imaging. He asks what his prognosis is if his C. Upper endoscopy


current presentation remains unchanged. What do D. Computerized tomography
you tell him is his estimated median survival? E. Liver biopsy

A. 1.5 years CORRECT ANSWER: C


B. Three years
C. Six years RATIONALE
D. Nine years Management of any patient with cirrhosis should
E. 12 years include upper endoscopy to screen for the pres-
ence and size of varices. Variceal hemorrhage
CORRECT ANSWER: E can occur at an estimated annual rate of five to 15
percent. The most important predictor of hemor-
RATIONALE rhage is variceal size, followed by decompensated
Cirrhosis can be compensated or decompensated. cirrhosis and the presence of red wale signs seen
Decompensated cirrhosis is defined by the pres- on upper endoscopy.
ence of ascites, variceal hemorrhage, encephalopa-
thy and jaundice, whereas patients with compen- REFERENCES
sated cirrhosis have none of those manifestations. The North Italian Endoscopic Club for the Study
The median survival in patients with compensated and Treatment of Esophageal Varices. Prediction
cirrhosis is around 12 years, while median survival of the first variceal hemorrhage in patients with
with decompensated cirrhosis is about 1.5 years. cirrhosis of the liver and esophageal varices. A
prospective multicenter study. New England Jour-
REFERENCE nal of Medicine 1988; 319(15):983-989.
D’Amico G, Garcia-Tsao G, Pagliaro L. Natural Garcia-Tsao G, et al. Portal hypertensive
history and prognostic indicators of survival in bleeding in cirrhosis: Risk stratification, diagno-
cirrhosis. A systematic review of 118 studies. J sis, and management: 2016 practice guidance by
Hepatol 2006; 44:217-231. the American Association for the study of liver
diseases. Hepatology. 2017; 65(1):310-335.

Question 3
A 53-year-old woman with longstanding alcohol Question 4
use presents to your office for management of new What is the most likely mechanism responsible for
diagnosis of cirrhosis. She continues to drink two beta-blockers decreasing risk of variceal bleed?
beers daily. She has no complaints of abdominal
pain and has no other significant medical history. A. Splanchnic vasodilatation
She is taking thiamine and folic acid. Her vital B. Reduction in portal pressure
signs are stable and she does not have ascites but C. Rise in cardiac output
has palmar erythema. Her labs include AST 300 D. Enhancement of bacterial translocation
U/L, ALT 150 U/L, total bilirubin of 1.2 mg/dL,
albumin of 3.0 g/dL, INR 1.2, and a platelet count CORRECT ANSWER: B
of 125,000/µL. She had a recent ultrasound that
was negative for liver cancer. What is the next best RATIONALE
step in management? Non-selective beta-blockers (propranolol, nadolol)
reduce portal pressure and are recommended for
A. Observation primary or secondary prophylaxis of variceal hem-
B. Beta-blocker orrhage. They likely improve survival in cirrhosis
Chapter 7 — Cirrhosis and liver transplantation 183

by lowering the rates of rebleeding and of other RATIONALE


complications of cirrhosis through reduction in Primary prophylaxis for a patient with varices who
splanchnic blood flow, gastroesophageal collateral has never bled is either a non-selective beta-blocker
blood flow, and portal pressures. Beta-blockers (NSBB) or variceal ligation. Choice depends on
have also been shown to reduce gut bacterial patient preferences, resources, side effects and/or
translocation. contraindications. In this patient who has relative
hypotension and asthma, beta-blockers would be
REFERENCES contraindicated and serial band ligation would
Gluud LL, Klingenberg S, Nikolova D, Gluud C. be the primary modality of treatment. A decrease
Banding ligation versus beta-blockers as primary in hepatic venous pressure gradient greater than
prophylaxis in esophageal varices: systematic 20 percent in patients treated with nonselective
review of randomized trials. Am J Gastroenterol β-blockers has been associated with a lower rate
2007; 102(12):2842-2848. of first variceal hemorrhage, ascites, and death, so
Albillos A, et al. Stratifying Risk in the Preven- observation alone would not be recommended. The
tion of Recurrent Variceal Hemorrhage: Results of combination of NSBB and ligation is not recom-
an Individual Patient Meta-Analysis. Hepatology. mended for primary prophylaxis as it is associated
2017; 66:1219-1231. with more side effects without a further reduction
Kroeger RJ, Groszmann RJ. Effect of selective in the risk of first variceal hemorrhage beyond that
blockade of beta-2 adrenergic receptors on portal achieved with either therapy alone. Transjugular
and systemic hemodynamics in a portal hypertensive intrahepatic portosystemic shunt placement (TIPS)
rat model. Gastroenterology 1985; 88:896-900. has increased risk of increased risk of hepatic
encephalopathy and death and would not be first
choice in this patient who has not yet bled.
Question 5
A 55-year-old man with NASH cirrhosis presents to REFERENCE
the endoscopy suite for variceal screening. He has Villanueva C, Aracil C, Colomo A et al. Acute hemo-
longstanding asthma controlled with inhaled cor- dynamic response to beta-blockers and prediction of
ticosteroids. Vitals include T 98.0o F, BP of 88/60 long term outcome in primary prophylaxis of varice-
mmHg, HR 70 beats per minute. Abdomen is soft al bleeding. Gastroenterology 2009; 137(1):119-128.
without ascites and he has no wheezing on auscul-
tation. Labs include AST 60 U/L, ALT 40 U/L, total
bilirubin of 2.1 mg/dL, albumin of 3.2 g/dL, INR Question 6
1.2, and a platelet count of 60,000/µL. You perform A 70-year-old woman with hepatitis B cirrhosis
an upper endoscopy and there are two columns of was admitted after a witnessed episode of he-
large varices without stigmata of recent bleed. matemesis. She is currently in the intensive care
unit and is intubated and sedated. Vitals include
What is the next best step in management? T 98.0o F, BP of 104/62 mmHg, HR 88 beats per
minute. Abdomen is soft without ascites. Labs
A. Endoscopic Band ligation include AST 60 U/L, ALT 40 U/L, total bilirubin
B. Endoscopic Band ligation and start nadolol of 2.1 mg/dL, albumin of 3.2 g/dL, Hemoglobin
C. Start nadolol of eight g/dL, INR 1.2, and a platelet count of
D. Referral for Transjugular Intrahepatic 75,000/µL. She is started on antibiotics and an
Portosystemic Shunt (TIPS) octreotide infusion. You perform an upper endos-
E. Observation copy and there is fresh clot in stomach and large
esophageal varices with red wale signs. What is
CORRECT ANSWER: A the next best step in management?
184 Digestive Diseases Self-Education Program®

A. Endoscopic Band ligation CORRECT ANSWER: E


B. Endoscopic Band ligation and start nadolol
C. Start nadolol RATIONALE
D. Referral for TIPS Bacterial infections in patients with cirrhosis who
E. Observation are admitted with an upper gastrointestinal can
occur in as much as 45 percent cases therefore
CORRECT ANSWER: B antibiotics are given to reduce risk of rebleeding
and death. Endoscopy should be performed within
RATIONALE 12 hours of presentation. Patients with cirrhosis
Non-selective beta-blockers plus variceal liga- admitted with gastrointestinal hemorrhage should
tion are recommended for secondary prophylaxis be resuscitated with avoidance of over transfu-
of variceal hemorrhage. Patients who survive an sion (goal hemoglobin ~7-8 g/dL), antibiotics,
episode of acute variceal hemorrhage have a very and infusion of a splanchnic vasoconstrictor. This
high risk of rebleeding and death. The median patient’s current hemoglobin is 8.5 g/dL and
rebleeding rate in untreated individuals is around there are no hemodynamic signs of instability
60 percent within the first one to two years. TIPS that would warrant transfusion. Overtransfusion
evaluation could be considered if the patient con- or volume overexpansion can precipitate vari-
tinued to bleed despite band ligation or if she were ceal rebleeding. A restrictive transfusion strategy
Child’s C and this were her first presentation. (transfuse once hemoglobin fell below seven g/dL)
in cirrhotic patients with gastrointestinal hemor-
REFERENCES rhage has been shown to improve survival.
Bosch J, Garcia-Pagan JC. Prevention of variceal
rebleeding. Lancet 2003; 361(9361):952-954. REFERENCES
Puente A, et al. Drugs plus ligation to prevent Garcia-Tsao G, Bosch J. Management of varices
rebleeding in cirrhosis: an updated systematic and variceal hemorrhage in cirrhosis. N Engl J
review. Liver Int 2014; 34:823- 833. Med 2010; 362(9):823-832.
Villanueva C, et al. Transfusion strategies for
acute upper gastrointestinal bleeding. N Engl J
Question 7 Med 2013; 368(1):11-21.
A 62-year-old man with newly diagnosed hepati-
tis C cirrhosis is admitted with melena. He is on
no medications. Vitals include T 98.0o F, BP of Question 8
100/52 mmHg, HR 90 beats per minute. He is The above patient underwent urgent endoscopy
alert and oriented, but his abdomen is distended and was found to have three columns of large
with large ascites. Labs include AST 60 U/L, ALT esophageal varices with red wale signs. He un-
40 U/L, total bilirubin of 3.1 mg/dL, albumin of derwent successful band ligation without further
2.7 g/dL, sodium 135 mEq/L, creatinine of 0.5 episodes of bleeding. His large ascites was tapped
mg/dL, hemoglobin of 8.5 g/dL, INR 1.5, and a and he has no signs of spontaneous bacterial
platelet count of 59,000/µL. Which of the follow- peritonitis. His mental status remains intact
ing is included in the management of this patient? without evidence of asterixis. His labs include:
AST 40 U/L, ALT 30 U/L, total bilirubin of two
A. Admission into a regular room mg/dL, albumin of 2.6 g/dL, sodium 137 mEq/L,
B. Early endoscopy within 36 hours creatinine of 0.5 mg/dL, hemoglobin of eight g/
C. Initiation of vasodilating agent dL, INR 1.7, and a platelet count of 59,000/µL.
D. Blood transfusion He had abdominal imaging that was negative for
E. Antibiotics hepatocellular carcinoma. He had a transthoracic
Chapter 7 — Cirrhosis and liver transplantation 185

echocardiogram that did not reveal any evidence gram but returned to drinking and currently drinks
of heart failure or pulmonary hypertension.What three beers daily. He has no other medical history.
is the next best step in management to prevent Vitals include T 98.7o F, BP of 122/64 mmHg, HR
further bleeding episodes and improve survival? 86 beats per minute. His abdomen is distended
with large ascites and he has 2+ pitting edema up
A. Balloon tamponade to his knees bilaterally. Labs include AST 220 U/L,
B. Sclerotherapy ALT 100 U/L, total bilirubin of 1.5 mg/dL, albumin
C. Transjugular intrahepatic portosystemic shunt of 3.7 g/dL, sodium 142mEq/L, creatinine of 0.5
(TIPS) placement mg/dL, and a platelet count of 159,000/µL. He had
D. Proton pump inhibitor a diagnostic paracentesis, which revealed ascites
E. Beta blocker polymorphonuclear count of 100/mm3, albumin
2.0 g/dL, total protein 1.6 g/dL. In addition to
CORRECT ANSWER: C alcohol cessation, what is the next best step in
management?
RATIONALE
Candidates for preemptive “early” TIPS are those A. Start furosemide and spironolactone
patients with variceal hemorrhage who are Child C B. Peritoneovenous shunt
(score 10-13) or Child Class B with active hemor- C. Placement of peritoneal catheter
rhage at endoscopy. The patient is Child Class C D. TIPS placement
(score 11). Data from a randomized clinical trial E. Referral for liver transplantation
demonstrate that early TIPS placed within 72
hours of diagnostic endoscopy following initial CORRECT ANSWER: A
vasoactive/endoscopic therapy is associated with
improved outcomes including survival rates. This RATIONALE
patient does not have contraindications to TIPS Ascites is the most common decompensating
such as advanced age, hepatocellular carcinoma, event in cirrhosis. The initial approach to treat-
heart failure, or significant encephalopathy. ing ascites involves dietary sodium restriction and
oral diuretics. Fluid restriction is not required
REFERENCES unless there is hyponatremia (sodium less 130
Monescillo A, et al. Influence of portal hyperten- mEq/L). Spironolactone is more effective than
sion and its early decompression by TIPS place- loop diuretics and should be started at a dose of
ment on the outcome of variceal bleeding. Hepa- 50–100 mg daily. Side effects of diuretics include
tology 2004; 40:793-801. encephalopathy, electrolyte abnormalities, and
Garcia-Pagan JC, et al. Early TIPS (Transjugu- painful gynecomastia. TIPS placement would be
lar Intrahepatic Portosystemic Shunt) Cooperative indicated if the patient had diuretic intolerant or
Study Group. Early use of TIPS in patients with refractory ascites. Liver transplant can be consid-
cirrhosis and variceal bleeding. N Engl J Med ered, however would not be the first choice in a
2010; 362:2370-2379. patient with alcoholic liver disease who recently
relapsed. A peritoneal catheter is not the first line
treatment and is mainly used in the palliative set-
Question 9 ting. A peritoneovenous shunt is a subcutaneously
A 50-year-old man with new onset ascites presents placed silicone tube that transfers ascites from the
to the emergency room with abdominal pain and peritoneal cavity to the systemic circulation. It can
discomfort. He has a history of heavy alcohol use be performed in patients who are not candidates
for thirty years. He was abstinent for two years for TIPS or transplant, but it carries risk of occlu-
after completing an alcohol relapse prevention pro- sion, infection, and bleeding.
186 Digestive Diseases Self-Education Program®

REFERENCES Chest tube placement for drainage should be


Runyon BA. Ascites and spontaneous bacterial avoided because it can lead to hemodynamic,
peritonitis. In: Feldman M, Friedman LS, Brandt renal, and electrolyte abnormalities. The defini-
LJ, eds. Sleisenger and Fordtran’s Gastrointestinal tive treatment for refractory ascites and hepatic
and Liver Disease. 9th ed. Philadelphia: Saunders hydrothorax is liver transplantation.
Elsevier, 2010:1517-41.
Deltenre P, et al. Transjugular intrahepatic REFERENCE
portosystemic shunt in refractory ascites: a meta- Orman ES, Lok ASF. Outcomes of patients with
analysis. Liver Int 2005; 25:349-356. chest tube insertion for hepatic hydrothorax.
Hepatol Int. 2009; 3: 582-586.

Question 10
A 42-year-old man with hepatitis B cirrhosis Question 11
with ascites was started on furosemide 40 mg and You are called while consulting on the liver service
spironolactone 100mg after recent discharge from for a newly diagnosed patient with hepatitis C
the hospital and was stable for two weeks. He now cirrhosis who was admitted for lower extremity
returns to the emergency room with acute dys- swelling. The team started oral diuretics and is
pnea. He is visibly uncomfortable. Vital signs in- asking for an outpatient appointment. He has
clude heart rate 110 beats per minute, blood pres- no complaints and feels better with initiation of
sure 110/68, and oxygen saturation is 94 percent diuretics. His abdomen is distended with large as-
on room air. His abdomen remains distended cites and he has 1+ pitting edema up to his knees
but his ascites and edema has improved. He has bilaterally. Labs include AST 50 U/L, ALT 22 U/L,
decreased breath sounds on the right side. Labs total bilirubin of 2.5 mg/dL, albumin of 3.4 g/
include AST 150 U/L, ALT 68 U/L, total bilirubin dL, sodium 144mEq/L, potassium of 3.2 mEq/L
of 1.5 mg/dL, sodium 142 mEq/L, creatinine of creatinine of 0.5 mg/dL, INR of 1.5, and a platelet
0.6 mg/dL, potassium of 4.1 mEq/L, and a platelet count of 159,000/µL. His ultrasound reveals large
count of 159,000/µL. His imaging shows a new ascites but does not show hepatocellular carci-
pleural effusion halfway up the right lung field. noma or new thrombosis.

In addition to consideration of TIPS placement, What is the next best step in management prior to
which of the following should be involved in this discharge?
patient’s management?
A. Endoscopy for variceal screening
A. Start beta-blockers for tachycardia B. Diagnostic paracentesis
B. Hold diuretics C. Liver transplant evaluation
C. Chest tube placement D. MRI abdomen
D. Liver transplant evaluation E. Prophylactic antibiotics

CORRECT ANSWER: D CORRECT ANSWER: B

RATIONALE RATIONALE
Management of hepatic hydrothorax should not be All patients with cirrhosis and ascites who are hos-
treated with a chest tube. The use of oral diuret- pitalized should undergo a diagnostic paracentesis
ics and repeated thoracentesis is only transiently to exclude spontaneous bacterial peritonitis (SBP).
effective for patients with refractory hydrothorax, Diagnosis of SBP is an ascites neutrophil count
and in these cases, TIPS should be considered. (not total WBC count) greater than 250/mm3. The
Chapter 7 — Cirrhosis and liver transplantation 187

most frequent manifestations are abdominal pain, is greater than 2.5, the ascites is related to post-
fever, and elevated white blood cell count, but up sinusoidal hypertension. The etiologies are cardiac
to one third of patients with SBP may be entirely ascites, Budd-Chiari and veno-occlusive disease.
asymptomatic. This patient does not have Budd-Chiari based on
imaging and no history of hematologic diseases,
REFERENCE which would put him at higher risk for veno-
Moore KP, et al. The management of ascites in occlusive disease. Given history of uncontrolled
cirrhosis: report on the consensus conference of hypertension and alcohol use this patient likely
the International Ascites Club. Hepatology 2003; has heart failure and resultant cardiac ascites from
38:258–266. uncontrolled disease. If this were related to alco-
holic cirrhosis, the patient would give a high SAAG
and low protein ascites. Patients with tuberculous
Question 12 peritonitis and malignant causes usually have a
A 57-year-old man from China presents to the low SAAG and high protein ascites.
emergency room with new onset ascites. He has
had decreased appetite and lost ten pounds in the REFERENCES
last two months. He has a history of heavy alcohol Runyon BA, Montano AA, Akriviadis EA, Antillon
use and hypertension. He has not seen a physi- MR, Irving MA, McHutchison JG. The serum-as-
cian in over twenty years. Vitals include T 98.8 F, cites albumin gradient is superior to the exudate-
BP of 90/60 mmHg, HR 89 beats per minute. His transudate concept in the differential diagnosis of
abdomen is distended with large ascites and he has ascites. Ann Intern Med 1992; 117: 215-220.
2+ pitting edema up to his knees bilaterally. Labs Runyon BA. Cardiac ascites: a characterization.
include AST 60 U/L, ALT 30 U/L, total bilirubin J Clin Gastroenterol. 1988; 10: 410.
of 1.2 mg/dL, albumin of 3.2 g/dL, sodium 138
mEq/L, creatinine of 0.8 mg/dL, and a platelet
count of 162,000/µL. He had an ultrasound, which Question 13
was limited, but did not show any masses or throm- A 57-year-old woman with hepatitis B cirrhosis
boses. He had a paracentesis, which revealed fluid is admitted with melena and was found to have
polymorphonuclear count of 100/mm3, albumin moderate portal hypertensive gastropathy with ac-
2.0 g/dL, and total protein 3.6 g/dL. What is the tive oozing. She underwent a diagnostic paracen-
most likely etiology of his ascites? tesis, which revealed ascites polymorphonuclear
count of 120/mm3, albumin 1.5 g/dL, total protein
A. Alcoholic cirrhosis 1.6 g/dL. Labs are AST 45 U/L, ALT 40 U/L,
B. Budd Chiari syndrome total bilirubin of 2.5 mg/dL, albumin of 3.7 g/dL,
C. Cardiac ascites sodium 142mEq/L, and a creatinine of 1.1 mg/
D. Tuberculous peritonitis dL. Which of the following is an indication to start
E. Malignant ascites prophylaxis in this patient?

CORRECT ANSWER: C A. Elevated bilirubin


B. Low sodium
RATIONALE C. Elevated creatinine
Etiology of ascites can be differentiated based on D. Gastrointestinal bleed
serum-ascites albumin gradient (SAAG) and as- E. Recent diagnosis of spontaneous bacterial
cites protein. This patient has a high SAAG more peritonitis
than 1.1 suggestive of portal hypertension. When
the SAAG is greater than 1.1 and the total protein CORRECT ANSWER: D
188 Digestive Diseases Self-Education Program®

RATIONALE A. Infection
The only clear indications for antibiotic prophy- B. Constipation
laxis in patients with cirrhosis are those admitted C. Narcotic use
with GI hemorrhage and patients who have recov- D. Volume overload
ered from an episode of SBP. Although ascites flu- E. Gastrointestinal bleeding
id total protein less than one g/dl increases risk of
development of SBP, long-term prophylaxis in this CORRECT ANSWER: D
patient population is not generally recommended
as the risk for developing antibiotic resistance RATIONALE
and C. difficile colitis outweighs the small effect Episodic hepatic encephalopathy (HE) is usually
in preventing infection. Norfloxacin prophylaxis precipitant-induced in over 80 percent of cases
has been shown to decrease risk of SBP in patients and include dehydration, infections, over diuresis,
with the following parameters: Child–Pugh score gastrointestinal bleeding, constipation, and the
greater than nine points with serum bilirubin level use of narcotics and sedatives. Key is to identify
more than three mg/dl, creatinine level greater and treat the precipitant. A diagnostic workup to
than 1.2 mg/dl, blood urea nitrogen level greater rule out other disorders that can alter brain func-
than 25 mg/ dl, or sodium level less than 130 tion and mimic HE should also be performed.
mEq/L. The patient in this case does not have any
of the above laboratory parameters. REFERENCE
Viltstrup H, et al. Hepatic encephalopathy in
REFERENCES chronic liver disease: 2014 Practice Guideline by
Fernandez J, et al. Antibiotic prophylaxis in the American Association for the Study of Liver
cirrhosis: Good and bad. Hepatology. 2016; 63: Diseases and the European Association for the
2019-2031. Study of the Liver. Hepatology. 2014; 60(2):715-35.
Fernandez J, Navasa M, Planas R et al. Pri-
mary prophylaxis of spontaneous bacterial perito-
nitis delays hepatorenal syndrome and improves Question 15
survival in cirrhosis. Gastroenterology 2007; 133: A 64-year-old woman with alcoholic cirrhosis is
818-824. admitted to the hospital with recurrent episode of
hepatic encephalopathy. She was recently started
on lactulose and has been having two to three soft,
Question 14 formed bowel movements daily. Vital signs include
A 62-year-old man with hepatitis C cirrhosis is ad- heart rate 80 BPM, blood pressure 122/67 mm Hg,
mitted with altered mental status. He had a recent and oxygen saturation is 100 percent on room air.
dental procedure and was given pain medication She is alert and oriented but has asterixis. Her abdo-
and a short course of antibiotics. He is only taking men is soft and nondistended, and she has no evi-
spironolactone 50mg for small ascites. Patient is dence of edema on exam. Labs include AST 100 U/L,
alert but not oriented to place and time. He has evi- ALT 50 U/L, total bilirubin of 2.5 mg/dL, albumin of
dence of asterixis. His mucous membranes are dry 3.2 mg/dL, and a platelet count of 122,000/µL
and he has no evidence of ascites on exam. His labs
include WBC 4.7K/mm3, AST 45 U/L, ALT 40 U/L, In addition to working up reversible causes, what
total bilirubin of 2.5 mg/dL, albumin of 3.7 g/dL, is the next best step in management?
sodium 142mEq/L, and a creatinine of 0.5 mg/dL.
A. Addition of rifaximin
What is the LEAST likely etiology of his B. Protein restriction
encephalopathy? C. Salt restriction
Chapter 7 — Cirrhosis and liver transplantation 189

D. Addition of Zinc beats per minute, blood pressure 90/52 mmHg,


E. Addition of L-ornithine L- acetate and oxygen saturation is 98 percent on room
air. He is alert and oriented without evidence of
CORRECT ANSWER: A asterixis. His abdomen is significantly distended
with fluid wave, and he has 2+ pitting edema to
RATIONALE the knees bilaterally. Labs include AST 100 U/L,
Management of recurrent or persistent hepatic ALT 120 U/L, total bilirubin of 2.4 mg/dL, albu-
encephalopathy requires combination of ri- min of three g/dL, sodium of 135 mEq/L, potas-
faximin and lactulose titrated at a dose that will sium of 3.5 mEq/L, BUN of 30 and a creatinine
produce two to three soft, formed bowel move- of 1.5mg/dL (previously 0.8 on discharge) and a
ments per day. Rifaximin has also been shown to platelet count of 100,000/µL.
be effective in maintaining remission in patients
with recurrent HE and in decreasing HE-related In addition to culture workup and urine studies,
hospitalizations. Strict protein restriction is not what is the next best step in management?
necessary and these patients often are at risk
for sarcopenia, which is a negative predictor of A. Continue diuretics
outcomes in patients with cirrhosis. Zinc can be B. Continue lactulose
used for hepatic encephalopathy if there is docu- C. Volume expand with albumin
mented deficiency but is not a first line agent. D. Start midodrine
L-ornithine L-acetate has shown to decrease am- E. Start octreotide
monia levels and improve psychometric testing in
one study, but it is not the first-line treatment for CORRECT ANSWER: C
recurrent hepatic encephalopathy. Low salt diet
is not part of the treatment algorithm for hepatic RATIONALE
encephalopathy. Acute kidney injury (AKI) occurs in approximately
20 percent of hospitalized patients with cirrhosis.
REFERENCES Management includes discontinuing diuretics and
Bass NM, Mullen KD, Sanyal A et al. Rifaximin lactulose, obtaining cultures (including diagnostic
treatment in hepatic encephalopathy. N Engl J paracentesis for SBP), and expanding intravas-
Med 2010; 362(12):1071-1081. cular volume with albumin. The most common
Kircheis G, et al. Therapeutic efficacy of L‐ cause of acute kidney injury in cirrhosis is albu-
ornithine‐L‐aspartate infusions in patients with min responsive azotemia. Acute tubular necrosis
cirrhosis and hepatic encephalopathy: results of a (ATN) and hepatorenal syndrome are also in the
placebo‐controlled, double‐blind study. Hepatol- differential diagnosis. Initiation of midodrine and
ogy 1997; 25:1351‐1360. octreotide should be considered if renal function
Montano‐Loza AJ, et al. Muscle wasting is as- does not respond to trial of albumin and other
sociated with mortality in patients with cirrhosis. etiologies (ATN) are ruled out.
Clin Gastroenterol Hepatol 2012; 10:166‐173.
REFERENCE
Salerno F, Gerbes A, Gines P, Wong F, Arroyo
Question 16 V. Diagnosis, prevention and treatment of the
A 54-year-old man with hepatitis B cirrhosis is hepatorenal syndrome in cirrhosis: a consensus
admitted to the hospital with worsening ascites. workshop of the international ascites club. Gut
He was recently discharged with an increase in 2007; 56:1310-1318.
diuretics to furosemide 40 mg and spironolactone
100mg daily. Vital signs include heart rate 102
190 Digestive Diseases Self-Education Program®

Question 17 Question 18
A 62-year-old woman with cirrhosis second- A 56-year-old man with recently diagnosed
ary to primary biliary cholangitis is admitted to hepatitis B is in your office for further evalua-
the hospital with first variceal bleed. Vital signs tion for new diagnosis of liver cancer. He has
include heart rate 88 beats per minute, blood no other past medical history and is a former
pressure 90/52 mmHg, and oxygen saturation is smoker. He recently started tenofovir. His exam
98 percent on room air. She is alert and orient- is notable for palpable hepatomegaly without
ed without evidence of asterixis. Her abdomen is evidence of jaundice, spider angiomata, or pal-
mildly distended with ascites. Labs include AST mar erythema. His current blood work includes
100 U/L, ALT 150 U/L, total bilirubin of 1.4 mg/ AST 120 U/L, ALT 150 U/L, sodium of 137
dL, albumin of 3.5 g/dL, creatinine of 0.6 mg/ mEq/L, total bilirubin of 1.8 mg/dL, albumin of
dL, INR of 1.1 and a platelet count of 100,000/ 3.7 g/dL, creatinine of one mg/dL, INR of one
µL. She underwent successful band ligation and a platelet count of 150,000/µL. You review
and her diagnostic paracentesis was negative his most recent MRI, which reveals a three
for spontaneous bacterial peritonitis. She had a centimeter segment 3 lesion and a two centime-
normal ultrasound without masses or thrombo- ter segment 7 lesion. Both lesions demonstrate
ses and is started on diuretics and a nonselective arterial enhancement, venous washout, and a
beta-blocker with good response. She is ready pseudocapsule. There is no evidence of vascu-
for discharge. lar invasion or extrahepatic disease.

What is the next step in management? What is the next best step in management?

A. Extended course antibiotics A. Ablation


B. Computerized Tomography B. Sorafenib
C. TIPS placement C. Resection
D. Liver Transplant Evaluation D. Liver transplantation evaluation
E. Observation E. Transarterial chemoembolization

CORRECT ANSWER: D CORRECT ANSWER: D

RATIONALE RATIONALE
Management of a patient with cirrhosis should Management of hepatocellular carcinoma (HCC)
include referral to a transplant center at their includes liver transplant evaluation regardless
first decompensation regardless of MELD score of decompensation. Liver transplantation is an
given median survival in decompensated liver effective option for patients with up to three
disease is as low as 1.5 years. tumors within Milan criteria: three lesions each
measuring less than three centimeters, or a
REFERENCE single lesion less than five centimeters without
Martin P, DiMartini A, Feng S, Brown R Jr, Fal- evidence of vascular invasion or extrahepatic
lon M. Evaluation for liver transplantation in disease. Surgical resection should be considered
adults: 2013 practice guideline by the American in patients with a single mass (usually less than
Association for the Study of Liver Diseases and five centimeters) in the setting of good liver
the American Society of Transplantation. Hepa- function and absence of clinically significant
tology. 2014; 59(3):1144-65. portal hypertension. Ablation with or without
transarterial chemoembolization (TACE) has
been used as a bridge to liver transplantation for
Chapter 7 — Cirrhosis and liver transplantation 191

controlling tumor burden. TACE is recommend- such as hereditary hypercholesterolemia and hy-
ed for nonsurgical candidates who have reason- peroxaluria. There are a few special conditions
able liver function and multinodular tumors that whereby patients can receive MELD exception
are beyond Milan criteria. Sorafenib is a multi- points, including HCC, hepatopulmonary syn-
kinase inhibitor with antiproliferative properties drome, portopulmonary hypertension, familial
that is reserved for those patients with advanced amyloid polyneuropathy, polycystic liver disease,
HCC who are not candidates for locoregional and primary hyperoxaluria. There are no MELD
treatment and have good liver function. exception points granted for refractory ascites.

REFERENCES REFERENCE
Mazzaferro V, Regalia E, Doci R et al. Liver Freeman RB Jr, Gish RG, Harper A, Davis GL,
transplantation for the treatment of small hepa- Vierling J, Lieblein L, et al. Model for end-stage
tocellular carcinomas in patients with cirrhosis. liver disease (MELD) exception guidelines:
N Engl J Med 1996;334(11):693-699. results and recommendations from the MELD
Llovet JM, Ricci S, Mazzaferro V et al. Exception Study Group and Conference (MES-
Sorafenib in advanced hepatocellular carcinoma. SAGE) for the approval of patients who need
N Engl J Med 2008;359(4):378-390. liver transplantation with diseases not con-
sidered by the standard MELD formula. Liver
Transpl 2006;12(12 Suppl 3):S128-S136.
Question 19
A 25-year-old man who has known primary
hyperoxaluria is seeing you for transplant evalu- Question 20
ation. He is now dialysis dependent. His current A 64-year-old man with alcoholic cirrhosis is ad-
blood work includes AST 20 U/L, ALT 25 U/L, mitted for dyspnea on exertion. He takes furose-
sodium of 137 mEq/L, total bilirubin of one mg/ mide and spironolactone for ascites. Vital signs
dL, albumin of 3.7 g/dL, creatinine of 2.6 mg/ include heart rate 90 beats per minute, blood
dL, INR of one and a platelet count of 200,000/ pressure 94/62 mmHg, and oxygen saturation is
µL. He recognizes that his Model for End Stage 88 percent on room air. He is alert and oriented
Liver Disease (MELD) score is low and he is without evidence of asterixis. His abdomen is
worried he will not get transplanted. He asks nondistended. He has evidence of clubbing in
about his access to organ transplantation. his extremities. Labs include AST 100 U/L, ALT
50 U/L, total bilirubin of 1.6 mg/dL, albumin of
Which of the following conditions do NOT 3.2 g/dL, creatinine of 0.5 mg/dL, INR of 1.1 and
qualify for MELD exception points? a platelet count of 90,000/µL. His echocardio-
gram is notable for late bubbles and he had an
A. Familial amyloid polyneuropathy arterial blood gas of 7.38/36/58/24.
B. Primary hyperoxaluria
C. Polycystic liver disease What aspect of the patient’s hepatopulmonary
D. Hepatopulmonary syndrome syndrome will qualify him for MELD exception
E. Refractory ascites points?

CORRECT ANSWER: E A. Presence of A-a gradient


B. Evidence of late bubbles on echocardiography
RATIONALE C. Dyspnea on exertion
Certain inherited and metabolic liver diseases D. Exam findings of clubbing
qualify nationally for MELD exception points E. paO2 less than 60mmHg
192 Digestive Diseases Self-Education Program®

CORRECT ANSWER: E Which of the following parameters is an absolute


contraindication for transplant?
RATIONALE
Hepatopulmonary syndrome qualifies for ex- A. Pulmonary vascular resistance (PVR) greater
ception points, when paO2 less than 60 mmHg. than or equal to 260-dynes.s.cm-5
Hepatopulmonary syndrome (HPS) is a pulmonary B. Pulmonary capillary wedge pressure less
complication that occurs in five to 10 percent of than12mmHg
patients awaiting liver transplantation. It results C. Mean pulmonary artery pressure greater than
from pulmonary vasodilatation that leads to gas 50mmHg
exchange abnormalities and hypoxemia. The clini- D. paO2 less than 60mmHg
cal presentation varies from mild dyspnea to severe
hypoxemia requiring supplemental oxygen and fin- CORRECT ANSWER: C
ger clubbing, cyanosis, and vascular spiders can be
seen on exam. Diagnosis is made by excluding car- RATIONALE
diopulmonary disease, a Pa02 less than 80 mmHg Portopulmonary hypertension (POPH) is a pul-
or an alveolar arterial oxygen gradient greater than monary complication of cirrhosis that is detected
15 mmHg on arterial blood gas, along with pres- in four to eight percent of liver transplant (LT)
ence of late bubbles on contrast echocardiography. candidates, resulting from pulmonary vasocon-
Treatment options for hepatopulmonary syndrome strictors that are released from the splanchnic
are limited, and therefore MELD exception points circulation. Diagnosis is made during right
are granted in more advanced cases. heart catheterization with the following param-
eters: pulmonary arterial pressure greater than
REFERENCES 25mmHg with a pulmonary capillary wedge pres-
Palma DT, Fallon MB. The hepatopulmonary syn- sure less than 15mmHg. Mild POPH with mean
drome. J Hepatol 2006;45(4):617-625. pulmonary arterial pressure (MPAP) less than
Rodriguez-Roisin R, Krowka MJ. Hepatopul- 35 mmHg, is not of major concern but moderate
monary syndrome—a liver-induced lung vascular (MPAP greater than or equal to 35 mmHg) and
disorder. N Engl J Med 2008;358(22):2378- severe POPH (MPAP greater than or equal to 45
2387. mmHg) are predictors of increased mortality fol-
lowing LT. In a study from the Mayo Clinic, mor-
tality was 100 percent with MPAP greater than 50
Question 21 mmHg. If MPAP can be reduced with vasodilator
A 61-year-old man with NASH cirrhosis is see- therapy to less than 35 mmHg and pulmonary
ing you for liver transplant evaluation. He has vascular resistance less than 400 dynes.s.cm-5,
history of ascites controlled with diuretics and transplant is possible with acceptable outcomes.
hepatic encephalopathy controlled with lactu-
lose. He complains of mild dyspnea. Vital signs REFERENCES
include heart rate 80 beats per minute, blood Krowka M, et al. Pulmonary hemodynamics and
pressure 102/68 mmHg, and oxygen saturation is perioperative cardiopulmonary-related mortal-
98 percent on room air. He is alert and oriented ity in patients with portopulmonary hypertension
without evidence of asterixis. His abdomen is undergoing liver transplantation. Liver Transpl
nondistended. You appreciate a right ventricu- 2000;6:443–450.
lar heave. During his pre-transplant evaluation Ashfaq M, et al. The impact of treatment of
he was found to have elevated right ventricular portopulmonary hypertension on survival follow-
systolic pressures and subsequently underwent a ing liver transplantation. Am J Transplant 2007;7:
right heart catheterization. 1258-1264.
Chapter 7 — Cirrhosis and liver transplantation 193

Question 22 REFERENCE
A 42-year-old woman who is two months post- Lucey MR, et al. Long-term management of the
transplant for cirrhosis secondary to autoimmune successful adult liver transplant: 2012 practice
hepatitis is seeing you in the office. She has no com- guideline by the American Association for the
plaints and is feeling well. She is taking tacrolimus Study of Liver Diseases and the American Society
two mg twice daily, Mycophenolate Mofetil 1000mg of Transplantation. Liver Transplantation. 2013
twice daily, and is on a prednisone taper. Her exam Jan;19(1):3-26.
is normal aside from a healing surgical incision.
Her labs include AST 250 U/L, ALT 300 U/L, total Question 23
bilirubin of 1.6 mg/dL, alkaline phosphatase 300 A 56-year-old man who was transplanted for
U/L, GGT 250 U/L. Tacrolimus level is 4.6 ng/mL. hepatitis C cirrhosis three months ago is noted to
Abdominal ultrasound with doppler shows patent have abnormal liver tests. He was viremic at the
hepatic portal vein and hepatic artery. There is no time of transplantation. He was recently treated
evidence of fluid collection or biliary dilatation. for rejection with pulse steroids in the setting of
Which of the following is the next best step in lowered immunosuppression. His exam is normal
management? aside from a healing surgical incision. His labs
include AST 250 U/L, ALT 300 U/L, total biliru-
A. Takeback to the operating room for surgical bin of 1.6 mg/dL, alkaline phosphatase 300 U/L,
repair of bile leak GGT 250 U/L, and tacrolimus level eight ng/mL.
B. Liver biopsy In addition to obtaining imaging, what is the next
C. Hepatic venogram best step in management?
D. Endoscopic cholangiopancreatography
E. Relist for liver transplantation A. Liver biopsy
B. Bolus steroids
CORRECT ANSWER: B C. Send for endoscopic retrograde
cholangiography
RATIONALE D. Start anticoagulation
Acute cellular rejection is diagnosed on liver E. Lower immunosuppression
biopsy alone. The most common early post-
transplant complications are acute cellular CORRECT ANSWER: A
rejection, hepatic artery thrombosis and biliary
stricture. Her imaging findings argue against a RATIONALE
hepatic vein stenosis, hepatic artery thrombosis, Recurrent hepatitis C is universal for individuals
bile leak, or biliary stricture. Given the timing of transplanted with a detectable viral load prior to
her transplant and young age her goal tacrolimus transplantation. Clinical course is variable but the
level should be closer to 10 ng/mL. Acute cellular progression of liver damage and fibrosis appears
rejection occurs in 20–30 percent of transplant to be accelerated. Treatment post-transplant can
recipients. There are three major histologic fea- be curative in as high as 96-98 percent of pa-
tures associated with acute cellular rejection: a tients on immunosuppression. Prior to bolusing
mixed inflammatory infiltrate in the portal triad steroids, it is important to differentiate between
(predominantly lymphocytic), destructive or non- recurrent hepatitis C and acute cellular rejection,
destructive nonsuppurative cholangitis involving which can only be assessed on liver biopsy.
interlobular bile duct epithelium (ductulitis), and
endotheliitis. First-line treatment is with high- REFERENCES
dose corticosteroids, to which greater than 90 Berenguer M. Natural history of recurrent hepati-
percent of patients respond. tis C. Liver Transpl. 2002 Oct;8(10 Suppl 1):S14-8.
194 Digestive Diseases Self-Education Program®

Reau N, et al. Glecaprevir/Pibrentasvir Treat- Question 25


ment in Liver or Kidney Transplant Patients With A 66-year-old man who was transplanted for PSC
Hepatitis C Virus Infection. Hepatology. 2018; 68: five years ago is seeing you in the office for rou-
1298-1307. tine follow-up. He is on tacrolimus monotherapy
and has had normal liver tests. He has recently
stopped smoking and remains abstinent from
Question 24 alcohol. His wife recently passed away from breast
A 60-year-old woman who was transplanted for cancer. He asks you about his risk of cancer?
hepatitis C cirrhosis four months ago is noted
to have abnormal liver tests. Her treatment for Which of the following cancers is the LEAST likely
hepatitis C was delayed due to insurance issues. to present after liver transplantation?
She was treated last month for rejection with pulse
steroids in the setting of lowered immunosup- A. Squamous cell carcinoma
pression. Her exam is normal. Her labs include B. Prostate cancer
AST 400 U/L, ALT 250 U/L, total bilirubin of 1.7 C. Oropharyngeal cancer
mg/dL, alkaline phosphatase 250 U/L, GGT 400 D. Lung cancer
U/L, and tacrolimus level three ng/mL. Her viral E. Colon cancer
load for hepatitis C is 352,000 IU/mL. She had an
ultrasound and MRCP, which did not show any CORRECT ANSWER: B
duct or vascular issues. What is the next best step
in management? RATIONALE
The cumulative incidence of de novo cancer in-
A. Start antiviral therapy creases from three percent to five percent at one
B. Bolus steroids to three years to 11 percent to 20 percent at 10
C. Endoscopic retrograde cholangiography years after liver transplantation. Skin malignan-
D. Liver Biopsy cies, notably squamous cell carcinoma and basal
E. Lower immunosuppression cell carcinoma are the most common form of ma-
lignancy in recipients of solid organ transplants.
CORRECT ANSWER: D Cigarette smokers are at increased risk of de-
veloping lung cancer and oropharyngeal cancer.
RATIONALE Colon cancer is increased in patients undergoing
Liver biopsy is the only way to differentiate transplantation for primary sclerosing cholangi-
between acute cellular rejection and hepatitis C. tis if they have concomitant inflammatory bowel
Though she has evidence of recurrent viremia, she disease. Risk of prostate cancer is not increased
also has a history recent rejection and has lower after liver transplantation.
levels of tacrolimus. It can be difficult to differenti-
ate between the two entities. Central endotheliitis REFERENCES
is more consistent with acute cellular rejection, Watt KD, et al. Long-term probability of and
where lobular hepatitis is more supportive of mortality from de novo malignancy after liver
recurrent hepatitis C. transplantation. Gastroenterology 2009;137:2010-
2017.
REFERENCE Rademacher S, et al. The 28-Year Incidence of
Moreira, RK. Recurrent hepatitis C and acute al- De Novo Malignancies After Liver Transplanta-
lograft rejection: clinicopathologic features with tion: A Single-Center Analysis of Risk Factors and
emphasis on the differential diagnosis between Mortality in 1616 Patients. Liver Transplantation.
these entities. Adv Anat Pathol. 2011;18:393-405. 2017; 23: 1404–1414.
Chapter 7 — Cirrhosis and liver transplantation 195

Question 26 cirrhotic patient home without a complete ex-


A 59-year-old woman with cirrhosis and ascites amination to look for a source of bleeding is not
presents with bright red blood per rectum. Her appropriate because the bleeding may recur if not
Hgb is 5.8 g/dL. She is transfused one unit of addressed and treated properly. NSBB is indicated
packed red blood cells. The bleeding stops and if the source of bleeding is related to portal hyper-
her vitals improve. An EGD was performed and tension.
showed small flat esophageal varices without
red wales and without evidence of blood in the REFERENCES
stomach. On rectal exam there were large internal Current Management of the Complications of Cir-
hemorrhoids with blood. Hgb is now 7.0 g/dL, rhosis and Portal Hypertension: Variceal Hemor-
INR=1.7. rhage, Ascites, and Spontaneous Bacterial Perito-
nitis. Garcia-Tsao G. Dig Dis. 2016; 34(4):382-6.
Of the choices below what is the next best action? Improving the management of gastrointestinal
bleeding in patients with cirrhosis. Michael J. Wil-
A. Discharge home since the bleeding stopped liams & Peter Hayes. Expert Review of Gastroen-
B. Start a non-selective beta-blocker NSBB terology & Hepatology .Volume 10, 2016 - Issue 4 .
C. Transfuse Fresh frozen plasma FFP Pages 505-515.
D. Perform a lower endoscopy to determine Severe Portal Hypertension-Inducted Anorec-
the source of the bleeding tal Varices Bleeding Treated via a Transjugular
E. Tell her if hemorrhoids bleed again you Intrahepatic Portosystemic Shunt.
will refer for a liver transplantation Liao YH, Hsia WT, Yung-Fang C, Tsai PP.Am J
Gastroenterol. 2016 Jan;111(1):27.
CORRECT ANSWER: D

RATIONALE Question 27
Ascites denotes the presence of portal hyperten- A 56-year-old woman has autoimmune hepatitis-
sion and decompensated cirrhosis. Patients at related cirrhosis, history of hepatic encephalopa-
this stage are at risk for gastrointestinal bleed- thy and ascites. She is brought to the emergency
ing. In this case scenario, a bleeding source from department by her daughter for being “sluggish”
anywhere in the GI tract is plausible. An upper for the past few days. Her daughter increased
endoscopy was performed but did not reveal an lactulose frequency but the patient did not im-
upper GI source. A diagnosis should be firmly prove despite having had 10 bowel movements
established in order to apply the proper treatment a day for the past two days. Her medications
plan. Thus option D is the best next option. Lower include spironolactone, furosemide and lactulose.
endoscopy may reveal diverticular bleed, rectal On exam her vital signs are stable and she is alert
variceal bleed, colon polyps, colon cancer or hem- and oriented to place and person with no asterixis.
orrhoidal bleed. Each of these possibilities dictate
a different plan of action. Thus it is important to What is the next best step in the management of
determine the bleeding source. this patient?
Hemorrhoidal bleed is not an indication for liver
transplantation and is not indicative of portal A. Admit her for more testing to rule out
hypertension. Treatment of bleeding hemorrhoids electrolyte abnormalities and infection
may require surgical ligation and local conserva- B. Add rifaximin and zinc
tive therapy. The bleeding stopped and the INR is C. Obtain an MRI of the brain
1.8, therefore transfusion of FFP is not necessary. D. Obtain blood ammonia level
Even though the bleeding stopped, discharging a E. Refer her for psychometric testing
196 Digestive Diseases Self-Education Program®

CORRECT ANSWER: A REFERENCE


Portal hypertension: A review of portosystemic
RATIONALE collateral pathways and endovascular interven-
The next best step is choice A. In a patient who tions. Pillai AK, Andring B, Patel A, Trimmer C,
has had prior episodes of overt hepatic encepha- Kalva SP. Clin Radiol 2015 Oct; 70 (10):1047-59.
lopathy OHE and no evidence of a neurologic
event, an MRI of the brain is not necessary as
a first line plan. However, an infection or elec- Question 29
trolyte abnormalities may have precipitated the A 50-year-old man is undergoing evaluation for
current encephalopathy and should be further mitral regurgitation. He is referred from cardiol-
investigated. OHE is usually associated with ogy clinic for suspicion of portal hypertension due
precipitating factors such as: gastrointestinal to a platelet count of 99x 109/L. On examination
bleeding, acute kidney injury, infection, con- he is mildly jaundice and the liver is pulsatile.
stipation, electrolyte imbalances, portal vein
thrombosis, hepatocellular carcinoma, etc. OHE What is the next best management option?
must be differentiated from other neurological
diseases such as acute cerebrovascular accidents A. Obtain a percutaneous liver biopsy to
CVA, alcohol-related issues, and other forms of determine if he has cirrhosis
metabolic encephalopathy when it occurs the first B. Obtain an ultrasound of the liver and the
time and when a neurological examination shows spleen to determine the presence of
a deficit. Blood ammonia level will not help in the splenomegaly as a cause of his thrombocytopenia
diagnosis or management of this patient and is C. Refer for a bone marrow biopsy
not a diagnostic test. D. Refer for a trans-jugular liver biopsy with
portal pressure measurements
REFERENCES E. Obtain a fibroscan
Serum ammonia level for the evaluation of hepatic
encephalopathy. Ge PS, Runyon BA. JAMA. 2014 CORRECT ANSWER: D
Aug 13; 312(6):643-4.
Covert and Overt Hepatic Encephalopathy: RATIONALE
Diagnosis and Management. Patidar KR, Ba- Thrombocytopenia is a common complication in
jaj JS. Clin Gastroenterol Hepatol. 2015 Nov; liver disease. It is usually defined as moderate for
13(12):2048-61. a platelet count less than 100 × 109/L, and severe
when platelets are less than 50 × 109/L.
A decreased platelet count can often be a diag-
Question 28 nostic clue to unsuspected cirrhosis and to the
What is the most common cause of portal presence of esophageal varices. Multiple factors,
hypertension? including splenic sequestration, reduced activ-
ity of the hematopoietic growth factor thrombo-
A. Cirrhosis poietin, cirrhotic coagulopathy or bone marrow
B. Right sided heart failure suppression by viral infections or medications can
C. Budd Chiari contribute to the development of thrombocyto-
D. Schistosomiasis penia in cirrhotic patients. A TJ liver biopsy with
E. Portal vein thrombosis portal pressure measurements will demonstrate
the presence or the lack of cirrhosis and whether
CORRECT ANSWER: A portal hypertension is of cardiac or liver origin.
Fixing the cardiac condition will most likely cor-
Chapter 7 — Cirrhosis and liver transplantation 197

rect the thrombocytopenia if the portal hyperten- REFERENCE


sion is cardiac in origin. Portal hypertensive bleeding in cirrhosis: Risk
stratification, diagnosis, and management: 2016
REFERENCES practice guidance by the American Association for
Management of thrombocytopenia due to liver cir- the study of liver diseases. Garcia-Tsao G, Abral-
rhosis: a review. Hayashi H1, Beppu T, Shirabe K, des JG, Berzigotti A, Bosch J. Hepatology. 2017
Maehara Y, Baba H. World J Gastroenterol. 2014 Jan;65(1):310-335.
Mar 14;20(10):2595-605.
Garcia-Pagán JC, Di Pascoli M, Caca K, et al:
Use of early-TIPS for high-risk variceal bleeding: Question 31
results of a post-RCT surveillance study. J Hepatol A patient is referred to you for management and
2013;58:45-50. follow-up of recently diagnosed compensated
cirrhosis and no history of GI bleed. As part of
the work up you obtain an upper endoscopy. This
Question 30 looks normal without evidence of varices.
A 66-year-old woman with a history of heart
failure has been complaining of abdominal dis- Which of the following is the next best step in
comfort. An US of the liver shows a nodular liver. management?
Portal pressure measurements show a hepatic
venous pressure gradient HVPG = 10 mmHg. A. Obtain portal pressure measurements
B. Nonselective beta blocker
Which of the following is the next best step? D. EGD in 2-3 years
E. Proton pump inhibitor
A. Perform an upper endoscopy
B. Begin a transplant evaluation CORRECT ANSWER: D
C. Refer to cardiology to improve cardiac function
D. Start NSBB RATIONALE
E. Start a proton pump inhibitor NSBB are indicated to prevent variceal hemor-
rhage in patients with cirrhosis and varices.
CORRECT ANSWER: A There is no evidence that they prevent the
formation of varices. With a normal EGD there
RATIONALE is no role for PPI or for measurements of portal
The patient in question has portal hypertension pressures. Repeating EGD in two to three years
due to elevated hepatic venous pressure gradi- is indicated as long as there is no evidence of
ent of 10 mm. The gradient is determined from decompensation of the cirrhosis at which time
the difference between the wedge pressure and an EGD is recommended when the decompensa-
the free hepatic pressure. A gradient above five tion occurs.
mm Hg indicates portal hypertension. In cardiac
hepatopathy there is portal hypertension but the REFERENCE
HVPG is normal, therefore option C in incorrect. Portal hypertensive bleeding in cirrhosis: Risk
The HVPG does not determine the indication stratification, diagnosis, and management: 2016
for transplant evaluation and option B is incor- practice guidance by the American Association for
rect. Nadolol is recommended for prophylaxis of the study of liver diseases. Garcia-Tsao G , Abral-
variceal bleed. An upper endoscopy is the best des JG, Berzigotti A Bosch ,J Hepatology. 2017
first choice. Jan;65(1):310-335.
198 Digestive Diseases Self-Education Program®

Question 32 A. Perform band ligation


What is the maximal dose of carvedilol B. Administer IV antibiotics and vasoactive drugs
recommended in the management of esophageal C. Initiate beta-blockade
varices? D. Place TIPS
E. Refer for liver transplantation
A. 6.25 mg daily
B. Carvedilol is not recommended CORRECT ANSWER: B
C. 12.5 mg daily
D. 12.5 mg twice a day RATIONALE
E. 3.25 mg daily The initial standard therapy for acute variceal hem-
orrhage is resuscitation in an intensive care unit.
CORRECT ANSWER: C Transfusion with PRBC if necessary along with cor-
rection of coagulopathy, administration of antibiot-
RATIONALE ics and vasoactive drugs. Once the patient has been
Carvedilol is non selective BB with vasodilating stabilized an upper endoscopy is indicated in order
properties that is used to decrease portal pressure to identify the source and stop the bleeding. The
and prevent first variceal hemorrhage. It has more patient in question is undergoing the first gastric
robust effect on the reduction of portal pressure variceal hemorrhage, endoscopic therapy with
than nadolol or propranolol. A safe and effective tissue adhesive in experienced hands is the recom-
dose is 12.5 mg/day. Doses higher than 12.5 mg a mended treatment over EVL because of its higher
day are associated with increased side effects and efficacy. Failure to control bleeding or in the event
hypotension in patients with impaired liver func- of a recurrent bleeding from GOV1, a TIPS proce-
tion due to alpha 1 antagonist action and excessive dure is recommended. Standard therapy for portal
first pass metabolism. hypertensive varices is shown to decrease mortality
by three-fold regardless of the presence of blood
REFERENCES on EGD and TIPS is preferred when initial therapy
Randomized controlled trial of carvedilol versus fails. Referral for liver transplantation is not a first
variceal band ligation for the prevention of the step in the management of this patient.
first variceal bleed. Tripathi D, Ferguson JW,
Kochar N, Leithead JA, Therapondos G, McAvoy REFERENCES
NC, Stanley AJ, Forrest EH, Hislop WS, Mills PR, Multicenter External Validation of Risk Stratifica-
Hayes PC. Hepatology. 2009 Sep;50(3):825-33. tion Criteria for Patients With Variceal Bleeding.
Carvedilol ( coreg) Package insert Philadelphia. Conejo I, Guardascione MA, Tandon P, Cachero
SmithKline Beecham Pharmaceuticals. May 1997. A, Castellote J, Abraldes JG, Amitrano L, Genescà
J, Augustin S. Clin Gastroenterol Hepatol. 2018
Question 33 Jan;16(1):132-139.
A 74-year-old man presents with hematemesis. Garcia pagan2010 Acute variceal bleeding:
He is known to have alcoholic cirrhosis Child risk stratification and management (including
Turcot Pugh B. He had two esophageal bands TIPS).Hernández-Gea V, Berbel C, Baiges A,
placed a year ago for primary prophylaxis. He García-Pagán JC. Hepatol Int. 2018 Feb;12(Suppl
failed to follow up until this admission. An EGD 1):81-90.
shows a gastric fundic varix GOV2 with a red Expanding consensus in portal hypertension:
wale but no active bleeding. Report of the Baveno VI Consensus Workshop:
Stratifying risk and individualizing care for portal
What is the next best step in the management of hypertension.de Franchis R1; Baveno VI Faculty. J
this patient? Hepatol. 2015 Sep;63(3):743-52.
Chapter 7 — Cirrhosis and liver transplantation 199

Question 34 ascites and hepatorenal syndrome. Hepatol Int.


A 67-year-old woman with primary biliary cholan- 2018 Feb;12(Suppl 1):122-134.
gitis and cirrhosis diagnosed 13 years ago, history
of esophageal varices, and ascites presents for a
follow up. She complains of not feeling well. Her Question 35
blood pressure is 92/45 mm Hg and her pulse is A 41-year-old man with alcohol related cirrhosis
80 bpm. On examination she is jaundiced, and has is admitted to the intensive care unit with coffee
moderate ascites. Laboratory studies drawn at ground emesis. On admission he is mildly hypo-
her primary care office a day earlier show a serum tensive but responds quickly to IV fluids. Antibi-
sodium level of 125 mEq/L and a creatinine of otics, a proton pump inhibitor and IV octreotide
2.2 mg/dL up from a baseline of 1.8 mg/dL. Her have been started. His platelet count is 15x109 /L,
medications consists of furosemide 40 mg, spi- Bilirubin 3.1 mg /dL, AST 98 U/L, ALT 79 U/L,
ronolactone 100 mg and nadolol 20 mg all taken serum creatinine is 1.6 mg/dL, Hgb is 8.0 g/dL,
once a day. The primary care doctor sent her back INR 2.0. You plan for an EGD.
to you to discuss liver transplant.
Which of the following should you recommend
What is your next best step? prior to EGD?

A. Begin liver transplant evaluation A. Transfusion of FFP


B. Obtain a renal ultrasound B. Platelet transfusion
C. Admit her for further management C. IV albumin
D. Stop diuretics and nadolol D. Nasogastric tube placement
E. Fluid restrict to 1 L/day and repeat labs E. Transfusion of one unit of PRBC
in 2 days
CORRECT ANSWER: B
CORRECT ANSWER: C
RATIONALE
RATIONALE The initial therapy for acute variceal hemorrhage
Option C is the most appropriate next step. The is resuscitation in an intensive care unit. Blood
patient in Question should be evaluated for condi- volume restitution should be initiated promptly
tions that precipitated her electrolyte abnormali- for a hgb level below 7 g/dl. A restrictive transfu-
ties such as over-diuresis, infections including sion strategy to maintain a hgb level above 7g/dl
pneumonia, urinary infection, bacteremia, spon- has been shown to significantly improve survival.
taneous bacterial peritonitis, intoxications, or Over transfusion or volume over expansion can
hepatorenal syndrome. She should be monitored precipitate variceal rebleeding. Thrombocytopenia
closely while holding her meds and starting work is a common complication in liver disease, and
up to evaluate for precipitating causes including a liver disease-related thrombocytopenia is often
renal ultrasound. Fluid restriction is only indicat- defined as moderate (less than 100 × 109/L) and
ed when other causes for hyponatremia have been severe (less than 50 × 109/L). Although clinically
ruled out and serum Na level is below 125 mEq/L. significant spontaneous bleeding does not usually
occur until the platelet count is less than 10 -20 ×
REFERENCES 109/L, severe thrombocytopenia increases the risk
John S, Thuluvath PJ. Hyponatremia in cirrhosis: of bleeding during and after invasive procedures
pathophysiology and management. World J Gas- in cirrhotic patients and platelet transfusions to
troenterol. 2015 Mar 21;21(11):3197-205. greater than 50 have shown in a systemic review
Piano S, Tonon M, Angeli P. Management of to reduce the risk of bleeding during a procedure.
200 Digestive Diseases Self-Education Program®

REFERENCES has the potential to cause bleeding, tumor seeding,


Razzaghi A1, Barkun AN. Platelet transfusion and the possibility that a negative biopsy is due to
threshold in patients with up­per gastrointestinal the failure to obtain tissue representative of the
bleeding: a systematic review. J Clin Gastroenterol nodule rather than a truly benign nodule. Referral
2012; 46: pp. 482-486. for liver transplantation or liver resection are not
Villanueva C, Colomo A, Bosch A, Concep- appropriate without a correct diagnosis.
ción M, Hernandez-Gea V, Aracil C, Graupera I,
Poca M, Alvarez-Urturi C, Gordillo J, Guarner- REFERENCE
Argente C, Santaló M, Muñiz E, Guarner C. Heimbach JK, Kulik LM, Finn RS, Sirlin CB,
Transfusion strategies for acute upper gastro- Abecassis MM, Roberts LR, Zhu AX, Murad MH,
intestinal bleeding. N Engl J Med. 2013 Jan Marrero JA. AASLD guidelines for the treatment
3;368(1):11-21. Erratum in: N Engl J Med. 2013 of hepatocel­lular carcinoma. Hepatology. 2018
Jun 13;368(24):2341. Jan; 67(1):358-380.

Question 36 Question 37
A 45-year-old man with chronic alcohol and HCV A 45-year-old man with chronic alcohol and HCV
well compensated cirrhosis presents with a liver well compensated cirrhosis presents with a liver
ultrasound report in hand that notes a 1.6 cm liver ultrasound report in hand that notes a 1.6 cm liver
lesion in the left lobe of the liver. lesion in the left lobe of the liver. The patient now
returns to discuss the result of a triphasic liver CT
Which of the following is the next appropriate step? that you ordered. Now the lesion measures 2.7 cm
and enhances on arterial phase and washes out
A. Repeat a liver ultrasound in 6 months to on delayed images and a rim of capsule is visible
assess for growth around the tumor. Platelet count is noted to be
B. Refer for liver transplantation 85K/mm3.
C. Obtain triphasic CT liver
D. Obtain a liver biopsy What is the next best step management?
E. Refer for surgical resection
A. Measure serum AFP
CORRECT ANSWER: C B. Refer for tumor biopsy
C. Refer for liver transplantation
RATIONALE D. Refer for surgical resection
The diagnosis of hepatocellular carcinoma HCC E. Refer for ablative therapy and liver
should be suspected in patients with cirrhosis who transplant evaluation
present with a new liver lesion on imaging study.
However other benign lesions may occur and should CORRECT ANSWER: E
be ruled out. The diagnosis of HCC is typically made
by dynamic radiologic imaging, CT scan or MRI RATIONALE
with contrast. The AASLD suggests several options The patient in question has a growing hepatocellu-
in patients with cirrhosis and an indeterminate lar carcinoma and is now within the Milan criteria
nodule. These include follow-up imaging, imaging T2. The radiographic characteristics meet LIRADs
with an alternative modality or alternative contrast five criteria. The recommendation is bridging
agent, or biopsy, but cannot recommend one option therapy to decrease the progression of the tumor
over the other. Biopsy may be required in selected and subsequent drop out of the transplant wait-
cases, but its routine use is not suggested. Biopsy ing list. There is no recommendation favoring one
Chapter 7 — Cirrhosis and liver transplantation 201

form or the other for the purpose of bridging to Question 39


liver transplantation. The presence of portal hy- A 35-year-old woman with cirrhosis presents
pertension, as suggested by the low platelet counts with abdominal pain for almost a week. She gets
weighs against surgical resection in this otherwise paracentesis weekly at her local hospital. Last
well-compensated patient. paracentesis was yesterday. She does not know
the results of the fluid analysis. In the ED a liver
REFERENCE ultrasound is performed that shows a new portal
Heimbach JK, Kulik LM, Finn RS, Sirlin CB, vein thrombus compared to imaging performed 10
Abecassis MM, Roberts LR, Zhu AX, Murad MH, weeks ago. Which of the following is the next best
Marrero JA. AASLD guidelines for the treatment step in management?
of hepatocel­lular carcinoma. Hepatology. 2018
Jan; 67(1):358-380. A. Begin anticoagulation
B. Perform paracentesis
C. Consult hematology
Question 38 D. No further action because the thrombus is due
The best approach to the management of a patient to cirrhosis and will resolve with transplantation
with cirrhosis, Child A, normal bilirubin level and E. MRI
no varices on EGD, who has a 1.9 cm hepatoma on
the edge of the right liver lobe is: CORRECT ANSWER: E

A. Liver transplantation RATIONALE


B. Tumor resection Portal vein thrombosis (PVT) is most commonly
C. Repeat study in 6 months found in cirrhosis and often diagnosed inciden-
D. Trans-catheter arterial tally by imaging studies. It commonly complicates
chemoembolization cirrhosis with portal hypertension, and liver
E. Radiofrequency ablation tumors. PVT may cause portal hypertension with
gastrointestinal bleeding and small bowel isch-
CORRECT ANSWER: B emia. Prevalence among cirrhotic patients ranges
between 4.4 percent-15 percent, and is responsible
RATIONALE for about five percent to 10 percent of overall cases
The patient has a 1.9 cm hepatocellular carci- of portal hypertension. The extent of clot should
noma, according to the AASLD guidelines, single be investigated and an MRI as in option E is indi-
tumors (less than2.5 cm) that are favorably cated. When thrombus involves the main portal
located may be equally well treated by either vein or is progressing into the mesenteric vein, the
resection or ablation. In this case cirrhosis is well consequence is intestinal ischemia or infarction
compensated and there is no evidence of portal and anticoagulation should be initiated. Prognosis
hypertension and the tumor is located where it is is good when PVT is recognized and treated before
easily amenable to resection. Tumor resection is the occurrence of intestinal infarction.
the preferred option.
REFERENCE
REFERENCE Ponziani FR, Zocco MA, Campanale C, Rinninella
Heimbach JK, Kulik LM, Finn RS, Sirlin CB, E, Tortora A, Di Maurizio L, Bombardieri G, De
Abecassis MM, Roberts LR, Zhu AX, Murad MH, Cristofaro R, De Gaetano AM, Landolfi R, Gas-
Marrero JA. AASLD guidelines for the treatment barrini A. Portal vein thrombosis: insight into
of hepatocel­lular carcinoma. Hepatology. 2018 physiopathol­ogy, diagnosis, and treatment. World
Jan; 67(1):358-380. J Gastroenterol. 2010 Jan 14; 16(2):143-55.
202 Digestive Diseases Self-Education Program®

Question 40 patients requiring frequent LVP (more than twice


A 50-year-old man is admitted with ascites and a month). PPI usage is associated with increased
abdominal tenderness. He has HCV cirrhosis. A risk of SBP in cirrhotic patients and should be
paracentesis is performed. Seven L of ascitic fluid used only for clear indications.
are removed. Which of the following should be
pursued next? REFERENCES
Pose E, Cardenas A. Translating Our Current
A. Albumin IV Understanding of Ascites Management into New
B. Pantoprazole IV Therapies for Patients with Cirrhosis and Fluid
C. Midodrine PO Retention. Dig Dis. 2017; 35(4):402-410.
D. TIPS Salvatore Piano, Marta Tonon, Paolo An­geli.
E. Simeprevir 30mg PO Management of ascites and hepatorenal syn­
drome. Hepatol Int (2018) 12 (Suppl 1):S122–
CORRECT ANSWER: A S134.
Gines A, Fernandez-Esparrach G, Monescillo
RATIONALE A, et al. Randomized controlled trial comparing
Ascites is the most common complication of cir- albumin, dextran-70 and polygelin in cirrhotic
rhosis, it is associated with poor quality of life, patients with ascites treated by paracentesis. Gas-
increased morbidity and mortality, and poor troenterology 1996;111:1002–1010.
long-term outcome. After a first episode of ascites,
survival is 85 percent during the first year and 56
percent at five years without liver transplantation. Question 41
Ascites formation results from increased renal A 50-year-old man is admitted with ascites and
sodium retention due to high activity of the renin- abdominal tenderness. He has HCV cirrhosis. A
angiotensin-aldosterone system resulting from paracentesis is performed. Seven L of ascitic fluid
high splanchnic vasodilation and portal hyperten- are removed. Results of the cell count in ascitic
sion. fluid show an absolute neutrophil count of 750
When patients fail diuretic therapy and dietary cells/mm3. You decide to begin therapy for spon-
salt restriction, large-volume paracentesis or TIPS taneous bacterial peritonitis.
is required. In patients with large and tense ascites
the first-line treatment should be therapeutic What is the next best treatment regimen?
paracentesis combined with the infusion of albu-
min because it has been shown to be more effec- A. IV antibiotics and IV albumin
tive than the standard diuretic. After large volume B. PO antibiotic
paracentesis there is a significant alteration of the C. Wait for results of ascitic fluid culture
systemic circulation such as an acute increase of D. Pentoxyfylline to prevent AKI
cardiac output and a reduction in the systemic E. Concomitant use of IV hydroxyethyl starch
vascular resistance and arterial blood pressure.
This can be prevented by plasma volume expan- CORRECT ANSWER: A
sion. The administration of human albumin, at the
dose of 8 g/l of ascites removed, is the most effec- RATIONALE
tive measure to prevent this circulatory dysfunc- The diagnosis of SBP is established with an ascitic
tion and to reduce the mortality rate in patients fluid absolute neutrophil count greater than 250/
with ascites and it has been shown to be more mm3. Bacteria are isolated from ascites fluid in
effective than other plasma expanders. TIPS is only 40–50 percent of cases. SBP is mostly a
considered in patients with refractory ascites or in monobacterial infection. Patients with SBP have
Chapter 7 — Cirrhosis and liver transplantation 203

a high risk of developing AKI, which is associated RATIONALE


with poor survival. In RCT, the administration of Hepatocellular carcinoma is the first consideration
albumin (1.5 g/kg of body weight on day one and in the assessment of a liver lesion in a patient with
one g/kg of body weight on day three) has been cirrhosis, however other conditions should be on
shown to reduce the incidence of AKI and improve the differential diagnosis of a liver mass.
survival in patients with SBP. Also albumin, but On contrasted CT of the liver, a hepatoma dem-
not hydroxyethyl starch, improved the cardiac onstrates arterial enhancement, washout on late
stroke work index and systemic vascular resistance images and an enhanced capsule. A hemangioma
in patients with SBP. Empiric antibiotic therapy appears well demarcated with peripheral enhance-
with IV third-generation cephalosporins (cefo- ment, homogenous more often than heterogenous
taxime, ceftriaxone) should be initiated as soon and delayed washout. An FNH shows a homogenous
as the diagnosis is established. The probability of arterial filling with a central scar and can be iso- or
SBP recurrence is about 70 percent at one year. In hypodense on delayed phases. An adenoma is be-
patients with first presentation of SBP, secondary nign but at risk of degeneration when it reaches five
prophylaxis with po antibiotic has been shown to centimeter in diameter and beyond. It demonstrates
be effective in preventing the recurrence of SBP. arterial enhancement and is hypodense on hepatobi-
liary phase relative to the liver tissue.
REFERENCES
Fernández J, Monteagudo J, Bargallo X, et al. REFERENCE
A randomized unblinded pilot study comparing Marrero JA, Ahn J, Rajender Reddy K. ACG clini-
albumin versus hydroxyethyl starch in spontaneous cal guideline: the diagnosis and management
bacterial peritonitis. Hepatology 2005;42:627–634. of focal liver lesions. Am JGastroenterol. 2014
Salvatore Piano, Marta Tonon, Paolo An­geli. Sep;109(9):1328-47.
Management of ascites and hepatorenal syn­
drome. Hepatol Int (2018) 12 (Suppl 1):S122–
S134. Question 43
Fernández J, Tandon P, Mensa J, Garcia-Tsao A 45-year-old man with newly diagnosed well
G. Antibiotic prophylaxis in cirrhosis: Good and compensated alcoholic cirrhosis and no prior com-
bad. Hepatology. 2016 Jun;63(6):2019-31. plications of portal hypertension seeks your atten-
tion to discuss the prognosis of his liver disease.
Which of the following is correct?
Question 42
A 53-year-old man with NASH cirrhosis is in the of- A. You recommend an upper endoscopy because
fice for a follow up. His albumin is 2.9 g/dl, ALT is varices can be present in 50 percent of patients
89 U/mL. A contrasted CT of the liver shows arte- with cirrhosis
rial enhancement of a three centimeter lesion with B. The majority of patients in his condition have
continuous heterogenous filling in later phases. On high portal pressure and a HVPG greater than
ultrasound it appears hyperechoic. The most likely or equal to12 mm
diagnosis is: C. If no varices are found on upper endoscopy,
prophylactic use of nonselective beta blocker
A. Hemangioma is recommended to decrease risk of upper
B. Focal nodular hyperplasia GI bleed
C. Hepatocellular carcinoma D. If he abstains from alcohol he no longer risks
D. Hepatocellular adenoma developing hepatoma
E. If no varices are present on EGD he should
CORRECT ANSWER: A undergo repeat in one year
204 Digestive Diseases Self-Education Program®

CORRECT ANSWER: A D. NSBB should be titrated to a heart rate of 60-


70 bpm
RATIONALE E. Carvedilol is a vasodilator that is less effective
Despite previous assumptions, cirrhosis can regress than nadolol or propranolol
if its etiologic cause is effectively removed such as
hep C and early alcohol abstinence. In patients with CORRECT ANSWER: B
compensated cirrhosis, the presence of clinically
significant portal hypertension (HVPG greater than RATIONALE
or equal to 10 mmHg) identifies a subset of patients Nonselective b-adrenergic blockers propranolol and
with higher risk of developing any complication nadolol, reduce portal pressure by reducing por-
(varices, decompensation). In a well-compensated tal venous inflow through both a b-1 (reduction in
cirrhosis as is in this case and without clinical evi- cardiac output) and a b-2 (splanchnic vasoconstric-
dence of portal hypertension, the HVPG may very tion) blocking effect, with the latter being the most
likely be less than10mmHg. NSBB are not recom- important effect. Reductions in HVPG are associated
mended in the absence of varices or portal hyper- with decreased rates of variceal hemorrhage and
tension. Patients with cirrhosis are at five percent lower rates of decompensation and death. Carvedilol
risk annually for developing hepatoma even with is more effective than nadolol or propranolol. NSBB
abstinence of alcohol. He should thus be enrolled may cause bradycardia, hypotension, and fatigue.
in a program of HCC screening. The recommended They should be titrated to a pulse of 50-60 bpm.
interval for follow-up EGD in compensated cirrhot-
ics, after an initial negative exam, is two to three REFERENCE
years, unless decompensation occurs in the interim. Garcia-Tsao G, Abraldes JG, Berzigotti A, Bosch
J. Portal hypertensive bleeding in cirrhosis: Risk
REFERENCE stratification, diagnosis, and management: 2016
Berzigotti A. Advances and challenges in cirrhosis practice guidance by the American Association for
and portal hypertension. BMC Med. 2017 Nov the study of liver diseases. Hepatology. 2017 Jan;
10;15(1):200. 65(1):310-335.

Question 44 Question 45
A patient with well compensated cirrhosis is found A 65-year-old woman with primary sclerosing chol-
to have large esophageal varices on first screen- angitis and ulcerative colitis with cirrhosis presents
ing endoscopy. You decide to start a non-selective with a new onset ascites and abdominal pain. You
beta blocker (NSBB). The patient is concerned that perform a diagnostic and therapeutic paracentesis.
he may not tolerate a NSBB because his friend had There is no evidence of SBP. Serum sodium and
problems with these medications, and he is asking renal function are noted to be normal.
you to discuss the necessity of this drug. Which of the
following is correct concerning NSBB What is the next best step in the management of
in cirrhosis? this patient?

A. Both nadolol and propranolol are safe and A. Start diuretics and pursue vascular imaging of
without any side effects the liver
B. NSBB prevents variceal bleeding via β1 and β2 B. Refer for liver transplantation
effects C. Restrict water intake
C. NSBB reduces the risk of variceal bleeding by D. Administer NSAIDs for pain control
80 percent E. Start diuretics alone
Chapter 7 — Cirrhosis and liver transplantation 205

CORRECT ANSWER: A A. Revisit with the nutritionist to lose weight


B. TIPS placement
RATIONALE C. Bariatric surgery
A new-onset ascites requires paracentesis for D. Low dose diuretics
diagnostic and therapeutic purposes. Ascitic E. Perform an EGD to band varices
fluid studies should include cell count with dif-
ferential, and cultures obtained bedside. The CORRECT ANSWER: C
patient has developed a new onset ascites with
abdominal pain. She has inflammatory bowel RATIONALE
disease and cirrhosis which puts her at high risk Nonalcoholic steatohepatitis (NASH) affects two
for thrombotic events such as portal vein throm- percent three percent of the US population and
bosis or less likely, Budd Chiari syndrome. After is expected to become the leading indication for
paracentesis, the next step is to obtain a vascular liver transplantation in the very near future. Bar-
US of the liver to assess patency of portal vessels iatric surgery was both effective and cost-effective
and start diuretics to prevent reaccumulation of for obese patients with NASH, regardless of fibro-
ascitic fluid. Fluid intake is not restricted in cir- sis stage as long as decompensation of cirrhosis
rhotics with ascites unless severe hyponatremia has not occurred and will be most effective in this
develops (less than 120 mEq/L). NSAIDs should case. Referring back to nutritionist is unlikely
be avoided in cirrhotic patients. Diuretics may to improve his BMI since he failed that option
be needed to control ascites but would not be the before. Diuretics and TIPS have no role in weight
next best step without also imaging the liver and loss. An EGD is not indicated in the absence of
vasculature. cirrhosis.

REFERENCE REFERENCE
Runyon BA, AASLD. Introduction to the revised Klebanoff MJ, Corey KE, Chhatwal J, Kaplan LM,
American Association for the Study of Liver Chung RT, Hur C. Bariatric surgery for nonalco-
Diseases Practice Guideline management of adult holic steatohepatitis: A clinical and cost-effective-
patients with ascites due to cirrhosis: update 2012. ness analysis. Hepatology. 2017 Apr;65(4):1156-
Hepatology. 2013;57: 1651. 1164.

Question 46 Question 47
A 48-year-old man has a BMI of 52 kg/m2 and A 35-year-old woman presents to clinic for advice
features of the metabolic syndrome. He takes regarding pregnancy. She has end-stage liver
blood pressure medication, insulin and a statin. disease secondary to autoimmune hepatitis. She
He is mostly sedentary. He has knee pain. He has is listed for liver transplantation. She desires to
1+ edema in the lower extremities. Platelets are have a baby before it is “too late” and wants to dis-
150,000/µL, hemoglobin A1c 6.0 percent, ALT 87 cuss risks of pregnancy after liver transplantation.
U/L and bilirubin 0.9 mg/dl. He wears a CPAP at Which of the following regarding pregnancy after
night. HVPG is eight mm Hg, and the liver pathol- transplantation is true?
ogy confirms NASH and stage 3 fibrosis. He has
been unable to lose weight despite the urging of A. She will be at increased risk for death
his PCP and work with a nutritionist. B. She is unlikely to get pregnant after transplant
due to high infertility rates
Which of the following is the next best step in C. She will be at increased risk for hypertensive
management? disorder
206 Digestive Diseases Self-Education Program®

D. Risk of complications to baby are not increased CORRECT ANSWER: E


when compared to the non-transplanted
population RATIONALE

CORRECT ANSWER: C Patients with advanced liver disease display a vari-


ety of pulmonary abnormalities including portopul-
RATIONALE monary hypertension (PoPHT) and hepatopulmo-
Liver transplantation restores sexual function and nary syndrome. The diagnosis of PoPHT requires
fertility as early as a few months after transplant. the presence of elevated pulmonary arterial PAP
Pregnancy outcomes for both the mother and greater than 25 mmHg, pulmonary capillary wedge
infant in liver transplant recipients are generally pressure (PCWP) less than 15 mmHg and pulmo-
good, but there is an increased incidence of preterm nary vascular resistance greater than 240 dynes/s
delivery, hypertension/preeclampsia, fetal growth per cm-5 in addition to presence of portal hyper-
restriction, and gestational diabetes. In contrast, the tension. Dyspnea is the most common symptom in
risk of congenital anomalies and the live birth rate patients with PoPHT. Treatment to decrease the
are comparable to those of the general population. elevated pulmonary pressure is essential before
liver transplant listing. The goal of vasoldilator
REFERENCES therapy is a mean pulmonary pressure of less than
Zaffar N, Soete E, Gandhi S, Sayyar P, Van 35 mm Hg, which is not always achieveable even
Mieghem T, D’Souza R. Pregnancy outcomes fol- with the most aggressive therapy. Although oxygen
lowing single and repeat liver transplantation: An management after OLT in the setting of HPS may
international 2-center cohort. Liver Transpl. 2018 challenging, oxygen requirements typically steadily
Jun; 24(6):769-778. decline following sucessful transplant. All other
Armenti VT, Radomski JS, Moritz MJ, et al. listed conditions do not constitute contraindication
Report from the National Transplantation Preg- to liver transplantation once treated.
nancy Registry (NTPR): outcomes of pregnancy
after transplan- tation. Clin Transpl. 2005:69-83. REFERENCE
Porres-Aguilar M, Zuckerman MJ, Figueroa-Casas
JB, Krowka MJ. Portopulmonary hypertension:
Question 48 state of the art. Ann Hepatol. 2008;7:321–330.
A 65-year-old woman with primary biliary cholan-
gitis is seen for a clinic visit. She reports increas-
ing shortness of breath with her usual activities Question 49
and can barely walk one flight of steps. She does A 45-year-old woman with end stage liver disease
not smoke. On examination her vitals and weight secondary to primary biliary cholangitis is re-
are stable. She has jaundice and no asterixis. She ferred to you for liver transplantation. Her Model
takes diuretics and lactulose. for End Stage Liver Disease score is 17. Her main
complaint is worsening shortness of breath and
What is the most likely diagnosis that would po- itching. A chest Xray is normal and on CT scan,
tentially preclude her from liver transplantation? the reports states that she is unlikely to have pul-
monary embolism. A cardiac echo three weeks ago
A. Hepatopulmonary syndrome shows a systolic ejection fraction of 52 percent and
B. Large hepatic hydrothorax trivial tricuspid regurgitation.
C. Pulmonary embolism You obtain a pulse oximetry in the office. Her
D. Pneumonia oxygen saturation is 76 percent on room air. You
E. Portopulmonary hypertension suspect hepatopulmonary syndrome.
Chapter 7 — Cirrhosis and liver transplantation 207

Which of the following assessments would confirm B. Portpulmonary hypertension PoPHT with
this diagnosis? mean pulmonary arterial pressure 32 mm Hg
C. HIV
A. Left heart cardiac catheterization D. Hepatocellular carcinoma with macroscopic
B. Repeat cardiac echo invasion in the right hepatic vein
C. Cardiac echo with bubble study showing
left sided bubbles after the fourth beat CORRECT ANSWER: D
D. Cardiac echo with bubble study showing left
sided bubbles at the second beat RATIONALE
E. Arterial blood gas showing an alveolar arterial Of the above options, advanced hepatocellular
oxygen gradient less than15 mmHg carcinoma with invasion of the hepatic vein is a
contraindication to transplant. Room air PO2 less
CORRECT ANSWER: C than 60 mmHg with normal CXR and spirom-
etry in HPS is indication for liver transplant and
RATIONALE may qualify for MELD exception points. A mean
The hepatopulmonary syndrome (HPS) is a pul- PoPHT greater than 35 mmHg is considered a
monary complication of cirrhosis and/or portal contraindication for transplant unless it can be
hypertension whereby patients develop hypoxemia decreased with medications.
as a result of alterations in pulmonary microvas- AIDS, not HIV infection, is a contraindication
cular tone and architecture. HPS occurs in up to for liver transplantation.
30 percent of patients with cirrhosis. Patients
with HPS have a higher mortality than do patients REFERENCES
with cirrhosis without the disorder. The diagnosis Porres-Aguilar M, Zuckerman MJ, Figueroa-Casas
is made by excluding major intrinsic cardiopul- JB, Krowka MJ. Portopulmonary hypertension:
monary disease, and finding a PA02 less than 80 state of the art. Ann Hepatol. 2008;7:321–330.
mmHg, or an alveolar arterial oxygen gradient Koch DG, Fallon MB. Hepatopulmonary syn-
greater than 15 mmHg on arterial blood gases, drome. Clin Liver Dis. 2014 May;18(2):407-20.
along with intrapulmonary vasodilatation shown
by contrast echocardiography and/or perfusion
lung scanning. Bubbles showing at the second
beat reflects the presence of intracardiac shunt.
Bubbles crossing to the left ventricle by the fourth
or fifth beat are typical of intrapulmonary shunt-
ing as is present in HPS.

REFERENCE
Koch DG, Fallon MB. Hepatopulmonary syn-
drome. Clin Liver Dis. 2014 May;18(2):407-20.

Question 50
Which of the following is a contraindication to
liver transplantation?

A. Hepatopulmonary HPS syndrome and a room


air arterial PO2 of 58 mm Hg
208 Digestive Diseases Self-Education Program®
Answers & critiques

CHAPTER 8

Gastrointestinal and
liver disease in pregnancy
Sushila R Dalal, MD, Christina Ha, MD FACG AGAF, Moira Hilscher, MD,
John Eaton, MD and Christina Lindenmeyer, MD

Question 1 and vomiting is refractory. Ondansetron can be


A 26-year-old female who is seven weeks preg- considered, but there have been some questions
nant presents with nausea and vomiting. She raised regarding safety and it is not considered
describes nausea that lasts most of the day with first line. Meals high in protein have been found
vomiting. She has tried rest and hydration, gin- to decrease nausea more that carbohydrate rich
ger supplementation, and a wrist band she pur- meals.
chased over the counter. However, she comes to
clinic to request further management. The most REFERENCE
appropriate next step is: ACOG Practice Bulletin No. 189: Nausea and
Vomiting of Pregnancy. Obstet Gynecol. 2018;
A. Gastric emptying study 131 (1)
B. Upper endoscopy
C. Ondansetron
D. A diet high in carbohydrates, low in protein Question 2
E. Doxylamine with vitamin B6(pyridoxine) A 29-year-old male comes for routine follow-up
of ulcerative colitis. He has been well on inf-
CORRECT ANSWER: E liximab, and six months ago sulfasalazine was
added to his regimen to treat some lingering
RATIONALE symptoms of arthritis. He mentions that he and
This patient has nausea and vomiting of preg- his partner would like to have a child, but his
nancy (NVP), and has tried conservative man- wife has not yet become pregnant. He would like
agement. Doxylamine and vitamin B6 has been to know if his condition or medications put him
found to be safe and effective for NVP and is at risk for infertility or birth defects. Which of
considered first line therapy. Further testing the following statements is correct?
with gastric emptying study is not necessary,
as NVP has a high prevalence at weeks four to A. Ulcerative colitis medications in men do not
six of gestation and peaks at week nine to 16. A affect fertility or pregnancy outcomes
nuclear test such as gastric emptying is not ap- B. His ulcerative colitis may make it more dif-
propriate during pregnancy, though decreased ficult to conceive
gastric emptying due to estrogen and proges- C. Sulfasalazine has been associated with sperm
terone is thought to be related to NVP. Upper dysfunction and he should stop this medica-
endoscopy would be considered if the nausea tion

209
210 Digestive Diseases Self-Education Program®

D. He should stop sulfasalazine and switch to CORRECT ANSWER: A


methotrexate for his arthritis
E. Infliximab has been associated with sperm RATIONALE
dysfunction Fertility in quiescent IBD is similar to the general
population. Multiple studies have found that ileal
CORRECT ANSWER: C pouch anal anastomosis does increase infertility,
likely due to scarring from adhesions. In vitro
RATIONALE fertilization is effective and similar to the general
Sulfasalazine is associated with reversible sperm population. She should not start trying to con-
dysfunction, while infliximab is not. There is no ceive until she has been in a stable remission for
data to suggest that his ulcerative colitis itself three to six months. Maintenance of remission
should not make it more difficult to conceive. is the primary goal for IBD patients in pregnancy,
Methotrexate would not be appropriate to start, and medication should be continued to maintain
as there is some data to associate it with sperm remission (except for methotrexate).
dysfunction and erectile dysfunction, and more
data is needed. REFERENCE
Mahadevan U, McConnnell R, Chambers C. Drug
REFERENCES Safety and Risk of Adverse Outcomes for Pregnant
Grosen A, Kelsen J, Hvas CL, et al. The Influence Patients with Inflammatory Bowel Disease. Gas-
of Methotrexate Treatment on Male Fertility and troenterology. 2017; 152: 451-462.
Pregnancy Outcome After Paternal Exposure. Oza SS, Pabby V, Dodge LE, et al. In Vitro
Inflamm Bowel Dis. 2017; 23(4): 561-569. Fertilization in Women with Inflammatory Bowel
Shin T, Okada H. Infertility in men with in- Disease IS As Successful as in Women From the
flammatory bowel disease. World J Gastrointest General Infertility Population. Clin Gastroenterol
Pharmacol Ther. 2016; 7 (3): 361-369. Hepatol. 2015 3(9): 1641-6.

Question 3 Question 4
A 30-year-old female with ulcerative colitis with A 26-year-old female with ulcerative colitis is 29
active disease just started a new treatment regi- weeks pregnant and recently has had more frequent
men with infliximab and would like to become bowel movements and abdominal cramping. She is
pregnant. She is concerned that her ulcerative not sure if this is due to a flare of her UC or not.
colitis may make this impossible.
Which testing should be chosen to assess her
Which of the following statements is true? disease activity:

A. Fertility in quiescent disease is similar to the A. CRP


general population B. Fecal calprotectin
B. History of ileal pouch anal anastomosis does C. CT scan
not impact fertility D. Colonoscopy
C. In vitro fertilization is not effective in IBD E. No testing, symptoms are likely due to
patients pregnancy and will get better on their own
D. She should start trying to conceive
immediately CORRECT ANSWER: B
E. She will need to stop infliximab when she be-
comes pregnant
Chapter 8 — Gastrointestinal and liver disease in pregnancy 211

RATIONALE complications such as preterm delivery and small


Fecal calprotectin can be used as a non-invasive for gestational age babies. Stopping all medica-
marker of inflammation in pregnancy. CRP may tions carries a high risk of flare and active dis-
go up in pregnancy and can be unreliable. CT is ease. Methotrexate is teratogenic and should be
not appropriate due to radiation exposure. While stopped three to six months prior to conception.
endoscopy can be done if needed, this is not an Adalimumab can be safely continued to maintain
urgent situation, and there are other, lower risk remission. The placenta does actively transport
options. Ignoring a potential flare of disease is not antibodies, including adalimumab to the fetus
appropriate as active disease is associated with after 20 weeks, so the baby will be born with
pregnancy outcomes such as preterm birth, small adalimumab in the blood. However, this has not
for gestational age babies, and still birth. been associated with adverse outcomes. Certoli-
zumab pegol does not cross the placenta to a sig-
REFERENCE nificant degree, but since the patient is in a stable
Julsgaard M, Hvas CL, Gearry RB, et al. Fecal Cal- remission on adulimumab, she should stay on this
protectin Is Not Affected by Pregnancy: Clinical medication. While stopping adaliumumab in the
Implications for the Management of Pregnant Pa- first trimester would prevent transfer to the fetus,
tients with Inflammatory Bowel Disease. Inflamm the likelihood of flaring off therapy for 16 weeks
Bowel Dis. 2017 23(7): 1240-1246. is high and the risks of stopping the medication
outweigh the benefits.

Question 5 REFERENCE
A 31-year-old female with ileal Crohn’s in stable Mahadevan U, McConnnell R, Chambers C. Drug
remission on adalimumab and methotrexate Safety and Risk of Adverse Outcomes for Pregnant
comes for preconception counseling. She recently Patients with Inflammatory Bowel Disease. Gas-
had MR enterography that confirmed stable troenterology. 2017; 152: 451-462.
remission and is feeling well. She would like to
know if any changes should be made to her medi-
cation regimen. Question 6
A 32-year-old G1P0 woman who is 22 weeks
Which of the following statements is correct? pregnant presents to the ED by ambulance after
an episode of hematemesis at home. While in
A. She should make no changes to her current the ED, she has another episode of hematemesis,
medication regimen since she is well and her hemoglobin has decreased by 2gm/dL
B. She should stop methotrexate and continue from the last recorded value 1 week prior. She is
adalimumab to maintain remission tachycardic and hypotensive. After resuscitation
C. She should stop methotrexate and switch and stabilization, the next intervention is:
adalimumab to certolizumab pegol
D. She should stop all medications immediately to A. Medical management alone.
minimize risk B. Upper endoscopy, but sedation cannot be
E. She should stop methotrexate now and stop used.
adalimumab after the first trimester. C. Sedated upper endoscopy with patient supine.
D. Sedated upper endoscopy with patient in left
CORRECT ANSWER: B lateral decubitus position.
E. Sedated upper endoscopy with monopolar
RATIONALE cautery if needed.
Maintenance of remission is important to prevent
212 Digestive Diseases Self-Education Program®

CORRECT ANSWER: D ease is under control


C. Check vitamin levels to make sure there are no
RATIONALE deficiencies
This patient has an acute upper GI bleed and D. Check hemoglobin level to rule out anemia
needs further evaluation. Upper endoscopy can be E. Check bone density scan
done safely in pregnancy and a case control study
of 83 women did not show a difference in the per- CORRECT ANSWER: A
centage of women giving birth to healthy babies
as compared to age-matched controls. Sedation RATIONALE
can be used, but the lowest dose for the shortest Gluten free diet should be continued during preg-
duration should be used. Meperidine can be used nancy for patients with celiac disease. Optimal
in low doses, and Fentanyl can be embryocidal at celiac management does include insuring normal-
high doses, has been used in low doses in humans. ization of serologies, ruling out nutritional defi-
Propofol can be used by a trained anesthesia ciencies and anemia, and evaluating for decreased
professional. Midazolam has not been associated bone density.
with congenital abnormalities and can be used in
low doses. The patient should be in the left lateral REFERENCE
position, not supine, to avoid pressure on the vena Rubio-Tapia A, Hill ID, Kelly CP, Calderwood AH,
cava. Bipolar cautery is preferred to monopolar Murray JA, American College of Gastroenterology.
cautery to avoid stray currents. ACG Clinical guidelines: diagnosis and manage-
ment of celiac disease. Am J Gastroenterol. 2013;
REFERENCES 108 (5): 656-676.
ASGE Standard of Practice Committee, Shergill AK,
Ben-Menachem T, et al. Guidelines for endoscopy in
pregnant and lactating women. Gastrointest Endosc. Question 8
2012; 76(1): 18-24. A 33-year-old female who is 28 weeks pregnant
Cappell MS, Colon VJ, Sidhom OA. A study presents for follow up of pan-colonic ulcerative
of eight medical centers of the safety and clinical colitis. She previously was well on mesalamine
efficacy of esophagogastroduodenoscopy in 83 4.8gm daily prior to pregnancy. Two weeks ago
pregnant females with follow-up of fetal outcome she developed 10-15 loose bowel movements a day
with comparison control groups. Am J Gastroen- with blood and urgency. Testing for infection with
terol. 1996; 91(2): 348-354. C diff was negative. She started prednisone 40mg
daily as an outpatient but has not noticed signifi-
cant improvement. She has lost three pounds.
Question 7
A 28-year-old female was recently diagnosed with The next most appropriate step in management is:
celiac disease and comes to the GI clinic for follow
up. She is on a gluten free diet and is feeling well. A. Admission to the hospital for IV steroids and
She is considering conception and wants to make infliximab
sure that her celiac is completely controlled and B. Admission to the hospital for IV steroids and
she is ready for a healthy pregnancy. Which of the initiation of azathioprine
following is not recommended? C. Admission to the hospital for urgent delivery,
followed by treatment of UC with infliximab
A. Discontinue gluten free diet so that the baby D. Continue outpatient management with predni-
can have adequate nutrition sone and mesalamine, add mesalamine sup-
B. Check celiac serologies to make sure that dis- positories
Chapter 8 — Gastrointestinal and liver disease in pregnancy 213

E. Admission to the hospital for urgency A. Metronidazole


colectomy. B. Ciprofloxacin
C. Infliximab
CORRECT ANSWER: A D. Certolizumab pegol
E. Azathioprine
RATIONALE
The patient has signs and symptoms of severe CORRECT ANSWER: A
ulcerative colitis and needs escalated treatment.
Infliximab can be used safely in pregnancy. Aza- RATIONALE
thioprine can be continued in pregnancy to main- Of the agents listed above, only metronidazole
tain remission, but would not be appropriate to is considered unsafe during breastfeeding. The
start because of the slow onset of action and risk anti-TNF agents and azathioprine are safe to use
for adverse reaction such as pancreatitis or bone throughout pregnancy and breastfeeding. In
marrow suppression with new initiation. Urgent 2014, the Food and Drug Administration pub-
delivery is not indicated due to the UC; medical lished the Pregnancy and Lactation Labeling Rule
management of UC should be initiated. Continu- which removed the A, B, C, D, and X categories for
ing outpatient management with therapies that pregnancy and summarizes the risks and benefits
are not working with a suppository when she has of the medications based on the currently avail-
pan colitis is not appropriate. Colectomy is not able evidence for the drug.
indicated until advanced medical therapy options
have been exhausted, or an urgent condition like REFERENCE
toxic megacolon or perforation occurs. Nielsen et al. IBD medications during pregnancy
and lactation. Nat Rev Gastroenterol Hepatol.
REFERENCE 2014;11(2):116-27.
McConell RA, Mahadevan U. Pregnancy and the
Patient with Inflammatory Bowel Disease: Fertil-
ity, Treatment, Delivery, and Complications. Gas- Question 10
troenterol Clin North Am. 2016; 45 (2): 285-301. A 31-year-old woman with extensive ulcerative
colitis currently treated with infliximab and aza-
thioprine presents to the office following delivery
Question 9 of her first child. The delivery was an uncompli-
A 29-year-old woman with ileocolonic Crohn’s cated normal spontaneous vaginal delivery at term
disease currently taking adalimumab presents of a healthy baby boy. She stayed on her medica-
with a painful perianal mass. She is status-post tions throughout all three trimesters and now
delivery of a healthy baby boy three months ago presents for counseling regarding vaccinations for
via vaginal delivery. She is placed on ciprofloxacin her child during his first year. Which of the fol-
and metronidazole, referred to colorectal surgery lowing vaccinations should be avoided given her
for incision, drainage, and seton placement. She immunosuppression regimen?
is found to have antibodies to adalimumab, and
you recommend switching to infliximab or cer- A. Haemophilus influenzae type B
tolizumab pegol and adding azathioprine. She is B. Polio
currently breastfeeding. C. Measles, mumps, rubella
D. Rotavirus
Which of the following medications is not recom- E. Pneumococcal
mended for use during breastfeeding?
CORRECT ANSWER: D
214 Digestive Diseases Self-Education Program®

RATIONALE CORRECT ANSWER: A


Childhood immunizations are important to pro-
tect infants and children from vaccine preventable RATIONALE
infections. Of the vaccines recommended during This patient is experiencing gastroesophageal
the first year of life, which include Haemophilus reflux disease, which is a fairly common symptom
influenzae type B, DTaP, Hepatitis B, Pneumococ- during pregnancy. However, she is in her sec-
cal, Polio and Rotavirus, only the Rotavirus vac- ond trimester of pregnancy, gaining appropriate
cine should not be given to babies when mothers weight, and has no alarm symptoms. While diet
are on immunosuppression as it is the only live and lifestyle modifications are generally the first
vaccine in that series. The vaccine against mea- recommended step to address GERD symptoms
sles, mumps and rubella is a live vaccine, but that during pregnancy, the physiologic changes due to
series begins starting after 12 months. Pediatri- the gravid uterus are likely to impact the success
cians, obstetricians and gastroenterologists should of those measures. Endoscopy should be reserved
be aware of the vaccination schedules and counsel for emergent situations such as upper GI bleed-
mothers on immunosuppression about the hold- ing or alarm symptoms, ideally after the first
ing the rotavirus vaccine, but all others can be trimester. In terms of GERD treatment, calcium
safely given to babies. carbonate is safe to use during pregnancy and is
recommended first, followed by H2 blockers then
REFERENCE proton pump inhibitors if no response to the other
Nguyen et al. The Toronto Consensus State- agents. While omeprazole has reported potential
ments for the Management of Inflammatory embryonic and fetal toxicity based on animal stud-
Bowel Disease in Pregnancy. Gastroenterology. ies, subsequent human studies on PPI use during
2016;150(3):734-57. pregnancy (including omeprazole) did not show
an increased risk of adverse pregnancy outcomes.

Question 11 REFERENCES
A 26-year-old woman presents to the office with Gerson LB. Proton pump inhibitors and
a chief complaint of progressive heartburn and safety during pregnancy. Gastroenterology.
indigestion. She is currently 14 weeks pregnant, 2011;141:389-91.
and denies any dysphagia or odynophagia, denies Mahadevan and Kane. American Gastroen-
weight loss, and has been gaining weight ap- terological Association Institute Medical Position
propriately during her pregnancy. Her labs are Statement on the Use of Gastrointestinal Medica-
within normal limits, no anemia, normal meta- tions in Pregnancy. 2006;131:278-82.
bolic profile. She has tried lifestyle modifications,
including dietary modifications, staying upright
after eating and elevation the head of her bed with Question 12
persistent symptoms. A 29-year-old G1P0 woman presents to your office
to discuss her worsening constipation. She reports
Which of the following is the next appropriate step a lifelong history of constipation, at baseline has a
in her care? bowel movement every two to three days with the
use of stool softeners and psyllium. She is cur-
A. Calcium carbonate rently eight weeks pregnant and reports worsened
B. Upper GI series constipation described as Bristol scale one or two
C. Upper Endoscopy stools every four to five days with associated bloat-
D. Rantidine ing and abdominal discomfort despite daily fiber
E. Omeprazole and stool softener use. She denies rectal bleeding,
Chapter 8 — Gastrointestinal and liver disease in pregnancy 215

fevers, abdominal pain, vomiting or weight loss. formed, non-bloody bowel movements daily with-
She has no history of diabetes, thyroid disorders out urgency, pain or nocturnal bowel movements.
and is not taking any opiates. Which of the follow- She denies extra-intestinal manifestations of IBD.
ing agents is the most appropriate next step in her Sigmoidoscopy six months ago demonstrated
treatment? mucosal healing. She and her husband would like
to start trying to become pregnant now. Which
A. Magnesium citrate of the following statements about her ulcerative
B. Sodium phosphate colitis are correct?
C. Bisacodyl
D. Polyethylene glycol A. She will likely have difficulty conceiving due to
E. Sennosides decreased fecundity
B. She should switch her medications to another
CORRECT ANSWER: D agent prior to conception
C. She has a greater risk of having a miscarriage
RATIONALE or spontaneous abortion
Constipation is prevalent during pregnancy, af- D. She should not breastfeed while on 5-amino-
fecting 35-40 percent of women during the first salicylates
and second trimesters. Factors associated with E. She is at greater risk of requiring a Caesarian
worsened constipation during pregnancy include section
increased progesterone which can decrease intes-
tinal motility, increased iron intake due to anemia, CORRECT ANSWER: B
and the growing uterus may impact colonic tran-
sit. Osmotic laxatives are considered safe to use RATIONALE
throughout pregnancy, and polyethylene glycol is Women with IBD should have pre-conception
the first line treatment of choice. Saline hyperos- counseling with an updated disease activity as-
molar laxatives, magnesium citrate and stimulant sessment as sustained remission, defined as inac-
laxatives are not recommended for long-term or tive disease for at least three or more months, is
continued use during pregnancy. recommended prior to pregnancy as disease activ-
ity can be associated with persistent or worsened
REFERENCES symptoms throughout pregnancy and breastfeed-
Mahadevan and Kane. American Gastroenterolog- ing. While there are higher rates of adverse preg-
ical Association Institute Medical Position State- nancy outcomes among women with IBD com-
ment on the Use of Gastrointestinal Medications pared to the general population, active ulcerative
in Pregnancy. 2006;131:278-82. colitis is associated with a four-fold increased risk
Gomes et al. Gastrointestinal disease during of spontaneous abortion or miscarriage compared
pregnancy:what does the gastroenterologist need to patients in remission. The decision regarding
to know? Ann Gastroenterol. 2018;31(4):385-94. mode of delivery should be based on factors at the
time of labor, maternal factors and not the diag-
nosis of Ulcerative colitis. Caesarian section is
Question 13 primarily recommended for persons who have had
A 34-year-old woman presents to your office to a colectomy for their UC with an ileal pouch anal
discuss family planning. She has a one-year his- anastomosis to decrease risk of sphincter injury
tory of extensive ulcerative colitis, initially requir- and for women with perianal Crohn’s disease.
ing a course of corticosteroids, now maintained Breastfeeding is considered safe while on 5-ASAs,
on mesalamine with dibutyl phthalate 2.4 grams steroids, thiopurines or biologics. Because of the
twice daily. She is feeling very well, one to two dibutyl phthalate (DBP) present on the coating of
216 Digestive Diseases Self-Education Program®

her mesalamine formulation, she should consider RATIONALE


switching to a different agent prior to pregnancy Restaging disease activity prior to conception
as the DBP can have adverse reproductive and is recommended as active disease may persist
neurodevelopmental outcomes to the fetus. or worsen during pregnancy. If active disease
is found during the objective assessment, then
REFERENCE discussions regarding postponing conception until
Nguyen et al. The Toronto Consensus State- remission is achieved is recommended. Metho-
ments for the Management of Inflammatory trexate is a known teratogen, and it is recom-
Bowel Disease in Pregnancy. Gastroenterol- mended to discontinue treatment approximately
ogy;2016:150(3);734-57. three months prior to becoming pregnant. A Cae-
sarian section should be recommended but due
to her perianal disease history, not her surgical
Question 14 history. Diphenoxylate-atropine has been associ-
A 25-year-old woman with ileocolonic and peri- ated with fetotoxicity and should be discontinued
anal Crohn’s disease diagnosed at the age of 14 during pregnancy.
presents to the office to discuss family planning.
She had a 40cm ileocolonic resection two years af- REFERENCES
ter diagnosis and has been maintained on a stable Nguyen et al. The Toronto Consensus State-
regimen of adalimumab, methotrexate, folic acid, ments for the Management of Inflammatory
and diphenoxylate-atropine. She is currently feel- Bowel Disease in Pregnancy. Gastroenterol-
ing very well with three to four soft bowel move- ogy;2016:150(3);734-57.
ments daily, no blood or mucus, no abdominal Mahadevan and Kane. American Gastroen-
pain. Her last colonoscopy was five years ago with terological Association Institute Medical Position
only mild endoscopic disease activity in the neo- Statement on the Use of Gastrointestinal Medica-
terminal ileum. Her adalimumab levels are thera- tions in Pregnancy. 2006;131:278-82.
peutic with no antibodies present. She saw her
gynecologist who checked her pre-conception labs
and informed her she was not immune to varicella Question 15
and hepatitis B, but otherwise her bloodwork was A 38-year-old G1P0 woman currently 33 weeks
within normal limits. pregnant presents to the office with complaints of
abdominal bloating, indigestion, and mild right
Which of the following recommendations is the upper quadrant discomfort without radiation
most appropriate? worsened after meals. She denies any heartburn,
changes in appetite or weight, no fevers, nausea
A. She should receive the varicella vaccination or vomiting. Her physical exam is notable for
series prior to conception only mild pain with palpation in the right upper
B. She can stop her methotrexate now and start quadrant without rebound or guarding. Her labs
trying to conceive now are within normal limits including blood counts
C. She should have a repeat colonoscopy to assess and metabolic profile. Abdominal ultrasound re-
disease activity veals gallbladder sludge and stones without biliary
D. She will require Caesarian section because of ductal dilation.
her surgical resection
E. She can continue her diphenoxylate-atropine What is the recommended management strategy?
throughout pregnancy
A. Cholecystectomy
CORRECT ANSWER: C B. Ibuprofen
Chapter 8 — Gastrointestinal and liver disease in pregnancy 217

C. Magnetic resonance cholangiopancreatography Her laboratory evaluation is as follows:


D. Ceftriaxone
E. Dietary modification AST 58 U/L
ALT 64 U/L
CORRECT ANSWER: E Alkaline phosphatase 112 U/L
Total bilirubin 1.2 mg/dL
RATIONALE Direct bilirubin 0.3 mg/dL
This patient has mild biliary colic with evidence Hemoglobin 12.1 g/dL
of gallstones and sludge on ultrasound. Gall- Platelets 280 x 109/L
stones are can occur during pregnancy due to INR 1.1
altered gallbladder motility and increased choles- Bile acids 28 µmol/L
terol saturation present in the biliary fluid. Most
cases of biliary colic during pregnancy can be Ultrasound reveals normal hepatic parenchyma
addressed with supportive care including intra- and normal caliber of the intra- and extrahepatic
venous fluids, pain control with acetaminophen bile ducts.
or opiates if necessary, and dietary modifications.
Nonsteroidal anti-inflammatory drugs should be Which of the following is the next best step in
avoided in pregnancy, especially during the third management?
trimester due to increased risks of adverse fetal
events. An MRCP is a non-contrast study and A. Recommend labor induction now
safe to perform during pregnancy, but should B. Recommend initiation of ursodeoxycholic
be reserved for cases where the ultrasound is acid 13-15 mg/kg/day
equivocal or there is a high index of suspicion for C. Recommend cholestyramine 4 grams per day
cholecystitis, pancreatitis or cholangitis. Anti- D. Liver biopsy
biotics are reserved for cases of cholecystitis and E. Start obeticholic acid 10 mg per day
cholangitis. Cholecystectomy during pregnancy
is reserved for acute cholecystitis and for cholan- CORRECT ANSWER: B
gitis or gallstone pancreatitis following an ERCP.
For mild biliary colic, elective cholecystectomy RATIONALE
can be considered within three months following This patient’s presentation is consistent with
delivery, particularly if additional pregnancies intrahepatic cholestasis of pregnancy (ICP). ICP
are planned. typically presents in the third trimester of preg-
nancy with pruritis. The pruritis may be general-
REFERENCES ized but is most predominantly localized to the
Tran et al. Liver disease and pregnancy. Am J palms and soles. The etiology of ICP is unclear but
Gastroenterol. 2016;111:176-94. is likely secondary to a combination of genetic,
Van der Woude et al. Management of gastroin- environmental, and hormonal factors. Serum
testinal and liver diseases during pregnancy. Gut. aminotransferases are commonly elevated less
2014;63:1014-23. than two times the upper limit of normal. Total
and direct bilirubin may be elevated but rarely
exceed 6 mg/dL. The most definitive diagnostic
Question 16 laboratory finding is an increase in serum total
A 32-year-old G1P0 at 34 weeks gestation is bile acid concentrations.
referred for evaluation of pruritis. Her physi- Maternal bile acids circulate to the placenta
cal examination is remarkable for scattered and fetus and impose risk of adverse pregnancy
excoriations. outcomes, including intrauterine demise, spon-
218 Digestive Diseases Self-Education Program®

taneous preterm labor, and fetal distress. The A. Avoidance of invasive fetal scalp monitoring
risk of adverse fetal outcomes correlates with B. Start antiviral therapy in the third trimester
maternal serum bile acid levels, with increased for prevention of vertical transmission given
risk described with serum bile acid levels above HCV RNA more than 1 million IU/L
40 µmol/L. In women presenting with serum C. Screen for HIV
bile acid levels over 40 µmol/L, delivery at 36- D. Mode of delivery should occur based on
37 weeks gestation has been proposed to reduce routine obstetrical indications
perinatal mortality risk compared with expect- E. HCV RNA level should be re-evaluated after
ant management. delivery to assess for spontaneous clearance
Ursodeoxycholic acid (UDCA) is commonly
used in the management of ICP. UDCA im- CORRECT ANSWER: B
proves pruritis and liver test abnormalities in
ICP and may improve fetal outcomes. Chole- RATIONALE
styramine can be used in refractory cases of The safety of direct-acting antivirals in preg-
pruritus. Obeticholic acid has not been studied nancy and lactation is unknown. As a result,
in ICP. HCV treatment is not recommended in pregnant
women or in women who are breastfeeding, ir-
REFERENCES respective of viral load. Spontaneous clearance
Puljic A, Kim E, Page J, et al. The risk of infant of HCV can occur in the postpartum period.
and fetal death by each additional week of ex-
pectant management in intrahepatic cholestasis REFERENCE
of pregnancy by gestationaly age. Am J Obstet AASLD-IDSA. Recommendations for testing,
Gynecol. 2015 May;212(5).667.e1-5. managing, and treating hepatitis C. http://www.
Bacq Y, Sentilhes L, Reyes HB, et al. Efficacy hcvguidelines.org
of ursodeoxycholic acid in treating intrahepatic
cholestasis of pregnancy: a meta-analysis. Gas-
troenterology. 2012 Dec;143(6):1492-501. Question 18
Dixon PH, Williamson C. The pathophysiology A 26-year-old female with a past medical of
of intrahepatic cholestasis of pregnancy. Clin Res chronic hepatitis C virus infection, naive to
Hepatol Gastroenterol. 2016 Apr;40(2):141-53. therapy, presents to establish prenatal care at 25
weeks gestation. She complains of severe pruri-
tus, in addition to epigastric pain and fatigue.
Question 17
A 32-year-old G1P0 woman presents for her What is the next most appropriate step in her
first routine obstetrical visit. Her past medical evaluation?
history is notable for a history of intravenous
drug use as a teenager and a history of hypothy- A. Measure HCV viral load and HCV genotype.
roidism, for which she takes levothyroxine. As B. Measure serum AST, ALT and serum bile
part of routine prenatal screening, lab work is acids.
significant for positive anti-HCV IgG, HCV RNA C. Initiate empiric therapy for HCV with
1,655,000 IU/L, HCV genotype 1a, negative HBV glecaprevir/pibrentasvir.
surface antigen, positive anti-HBV Core IgG and D. Screen for fatty acid oxidation defects.
positive anti-HBV Surface IgG. E. Measure serum AST, ALT, bilirubin and
prothrombin time.
Which of the following is not recommended as
part of her routine pre-and perinatal care? CORRECT ANSWER: B
Chapter 8 — Gastrointestinal and liver disease in pregnancy 219

RATIONALE cluding transplant hepatology and high-risk


HCV-infected pregnant women have a higher in- maternal fetal medicine specialists.
cidence of intrahepatic cholestasis of pregnancy
(ICP) when compared to noninfected pregnant CORRECT ANSWER: E
women. ICP is associated with an increased rate
of adverse maternal and fetal outcomes; all pa- RATIONALE
tients with this syndrome should be immediately Observed rates of preeclampsia (21.9 percent),
referred to a high-risk obstetrical specialist for cesarean section (44.6 percent), and pre-term
monitoring and treatment. delivery (39.4 percent) are higher in the trans-
plant population than what is observed in the
REFERENCE general population. Fertility is restored in 80-90
AASLD-IDSA. Recommendations for testing, percent of women within the first year after liver
managing, and treating hepatitis C. http://www. transplantation. The American Society of Trans-
hcvguidelines.org plantation has recommended that transplant
recipients wait at least one year after transplan-
tation before attempting to conceive. During this
Question 19 time, there should be no rejection or evidence of
An 18-year-old female with a past medical history acute infections that could potentially impact the
of Byler’s disease (Progressive Familial Intrahe- fetus, and immunosuppression dosing and al-
patic Cholestasis Type 1) presents for liver trans- lograft function should be stable. Mycophenolate
plant evaluation. She is interested in discussing has been associated with congenital anomalies
family planning in the long term, after liver trans- and is considered contraindicated in pregnancy.
plantation. She reports irregular menstrual cycle,
and that her last cycle was more than one year REFERENCE
ago. As part of her pre-transplant counseling, the McKay DB, et al; Women’s Health Committee
following should be recommended: of the American Society of Transplantation.
Reproduction and transplantation: report on
A. She should wait at least three years after liver the AST Consensus Conference on Reproductive
transplantation before attempting to conceive. Issues and Transplantation. Am J Transplant
B. All transplant immunosuppression agents 2005;5: 1592-1599.
used in liver transplant recipients, including
tacrolimus, cyclosporine, mycophenolate,
azathioprine and corticosteroids are safe Question 20
throughout pregnancy. A 29-year old female with a history of chronic
C. Fertility is restored in less than 30 percent of hepatitis B virus (HBV) infection presents for
women within the first year after liver trans- routine prenatal follow-up at 28 weeks gestation.
plantation, therefore contraceptive methods She has never been treated for hepatitis B. Her
during this time are not needed. current laboratory studies are notable for the
D. Her immunosuppression levels should be following: AST 35 U/L, ALT 25 U/L, ALP 180
lowered immediately prior to attempting to U/L, HBV surface antigen positive, anti-HBV
conceive in order to minimize risk to the fetus. surface IgG negative, anti-HBV core IgG posi-
E. Given reported increased rates of pre- tive, anti-HBV e antigen IgG negative, HBV e
eclampsia, cesarean section and pre-term antigen positive, HBV DNA 120,000 IU/L. In
delivery among liver transplant recipients, this patient, which of the following factors are
her post-transplant pregnancy should be associated with an increased risk of HBV vertical
managed by a multi-disciplinary team, in- transmission:
220 Digestive Diseases Self-Education Program®

A. HBV DNA more than 20,000 IU/L Liver biopsy pathology, if obtained in this case
B. AST more than 30 IU/L would be most likely to demonstrate:
C. Absence of current antiviral therapy
D. HBV e antigen positivity A. Fibrin deposition, periportal hemorrhage,
E. Anti-HBV surface antigen IgG negativity hepatocyte necrosis/infarction and microve-
sicular fat infiltration.
CORRECT ANSWER: D B. Microvesicular fat accumulation in the peri-
central zone with perioportal sparing.
RATIONALE C. Centrilobular cholestasis, little to no inflamma-
Maternal risk factors that increase the likelihood tion and bile plugs in hepatocytes.
of vertical transmission are a high level of hepa- D. Hepatocyte necrosis, bile plugs, cholestasis and
titis B viremia (HBV DNA more than 200,000 steatosis.
IU/L, transmission rate of 70-90 percent) and the E. Lymphoplasmacytic portal inflammatory infil-
presence of hepatitis B e antigen positivity (trans- trate, interface hepatitis, bridging necrosis.
mission rate of 10-40 percent). The American As-
sociation for the Study of Liver Disease (AASLD) CORRECT ANSWER: A
guidance on hepatitis B recommends institution of
antiviral therapy with lamivudine, telbivudine or RATIONALE
tenofovir to mothers with HBV DNA of more than Preeclampsia is characterized by the presence
200,000 IU/L. Tenofovir is the preferred antiviral of de-novo hypertension and proteinuria occur-
agent due to its potency and due to increased rates ring after 20 weeks gestation. Organ dysfunction,
of drug resistance seen with use of lamivudine and including liver involvement, renal dysfunction,
telbivudine. and hematologic abnormalities can be seen.
Patients may present with epigastric and/or right
REFERENCE upper quadrant abdominal pain, nausea and/or
Terrault NA, Lok ASF, McMahon BJ, Chang vomiting. Biochemical changes include the pres-
KM, Hwang JP, Jonas MM, Brown RS, Bzowej ence of liver enzyme abnormalities, including
NH, Wong JB. Update on prevention, diagnosis, 10- to 20-fold elevations in aminotransferases,
and treatment of chronic hepatitis B: AASLD elevations in alkaline phosphatase that are above
2018 hepatitis B guidance. Hepatology 2018 what is typically observed in pregnancy, and
Apr;67(4):1560-1599. mild elevations in bilirubin up to 5 mg/dL. Liver
histology in preeclampsia includes the presence
of fibrin deposition, periportal hemorrhage, he-
Question 21 patocyte necrosis/infarction, and microvesicular
A 39-year-old G1P0 female with a past medical fat infiltration. Severe hepatic complications of
history of systemic lupus erythematous presents preeclampsia/eclampsia include development of
for obstetrical follow-up at 30 weeks gestation. a hematoma below Glisson’s capsule and hepatic
She complains of epigastric pain, nausea and fre- rupture.
quent vomiting. Vital signs are as follows: Blood
pressure 150/95 mmHg, heart rate 72 bpm, tem- REFERENCE
perature 98.2°F, oxygen saturation 94 percent on American College of Obstetricians and Gynecolo-
room air. Laboratory studies are notable for: AST gists, Task Force on Hypertension in Pregnancy.
525 U/L, ALT 450 U/L, total bilirubin 4 mg/dL, Hypertension in pregnancy. Report of the Ameri-
hemoglobin 14 g/dL, platelets 140 k/µL, serum can College of Obstetricians and Gynecologists’
creatinine 1.45 mg/dL. Task Force on Hypertension in Pregnancy. Obstet
Gynecol 2013 Nov;122(5):1122-1131.
Chapter 8 — Gastrointestinal and liver disease in pregnancy 221

Question 22 Question 23
A 26-year-old G1P0 female with a past medical A 34-year-old female with a past medical history
history of anorexia nervosa at 33 weeks gestation of autoimmune hepatitis-related cirrhosis pres-
presents to the emergency room for evaluation of ents to establish care at eight weeks gestation. She
right upper quadrant abdominal pain, pruritus, has maintained clinical and histologic remission
nausea, vomiting, anorexia and fatigue. She is di- on azathioprine 50mg daily and prednisone 5mg
agnosed with acute fatty liver of pregnancy based daily. Which of the following would you recom-
on the Swansea criteria. Which of the following mend as part of her ongoing liver related care?
should be recommended?
A. Screen for esophageal varices before the end of
A. Supportive care followed by delivery at 34 the first trimester
weeks B. Continue current immune suppressive regimen
B. Post-partum non-invasive assessment of he- C. Stop azathioprine and increase prednisone
patic fibrosis dose
C. Infant should be monitored closely for hepatic D. Resumption of quarterly lab monitoring after
failure, cardiomyopathy, nonketotic hypogly- delivery
cemia, myopathy and neuropathy E. Recommend early delivery to minimize liver
D. Intravenous magnesium sulfate, intravenous related risks
dexamethasone and delivery after 34 weeks
E. Percutaneous liver biopsy CORRECT ANSWER: B

CORRECT ANSWER: C RATIONALE


It is important that autoimmune hepatitis activity be
RATIONALE well controlled prior to and throughout pregnancy,
Management of AFLP involves prompt diagnosis as antepartum and postpartum flares can have nega-
and delivery of the fetus. A liver biopsy is usually tive consequences to fetal and maternal outcomes.
not required for diagnosis, and the coagulopathy Closer monitoring in the immediate post-partum
seen in this disorder may preclude the ability to period is to be recommended. Women with autoim-
perform liver biopsy. A diagnostic tool known as mune hepatitis are recommended to achieve stable
the Swansea criteria has been proposed to help liver function prior to pregnancy and continue on
define AFLP. Fetuses born to mothers with AFLP the same immunosuppression regimen throughout
should also be monitored closely as they are at risk their pregnancies that enabled them to achieve good
of developing hepatic failure, cardiomyopathy, liver function prior to pregnancy. Screening for
nonketotic hypoglycemia, myopathy and neuropa- esophageal varices in the cirrhotic pregnant patient
thy in association with fatty acid oxidation defects. is recommended in the second trimester.

REFERENCES REFERENCE
Fesenmeier MF, Coppage KH, Lambers DS, Bar- Schramm C, Herkel J, Beuers U, Kanzler S, Galle
ton JR, Sibai BM. Acute fatty liver of pregnancy in PR, Lohse AW. Pregnancy in autoimmune hepati-
3 tertiary care centers. Am J Obstet Gynecol 2005 tis: outcome and risk factors. Am J Gastroenterol
May;192(5):1416-1419. 2006;101:556-560.
Goel A, Ramakrishan B, Zachariah U, Ram-
achandran J, Eapen CE, Kurian G, Chandry S.
How accurate are the Swansea criteria to diagnose Question 24
acute fatty liver of pregnancy in predicting hepatic A 28-year-old G2P1 female at 21 weeks gestation
microvesicular steatosis? Gut Jan;60(1):138-139. presents with new-onset right upper quadrant
222 Digestive Diseases Self-Education Program®

pain, nausea, vomiting, jaundice and pruritus. and resultant cirrhosis, which was diagnosed at
Laboratory studies are notable for: AST 600 U/L, the age of 12 as part of an evaluation for neurocog-
ALT 725 U/L, total bilirubin 6 mg/dL, alkaline nitive delay. Her cirrhosis has been complicated
phosphatase 320 U/L, platelets 150 k/µL, serum by esophageal varices that have bled in the past,
creatinine 1.30 mg/dL, prothrombin time 18 sec. for which she has undergone previous esophageal
Which of the following is the most likely etiology variceal band ligation; she is on daily propranolol
for the patient’s presentation? for secondary bleeding prophylaxis. She is cur-
rently maintained on daily penicillamine chelation
A. Hyperemesis gravidarum therapy. All of the following should be recom-
B. Intrahepatic cholestasis of pregnancy mended as part of her antepartum care:
C. Acute hepatitis A infection
D. Pre-eclampsia A. Stop penicillamine and switch to daily
E. Choledocholithiasis trientene with a 25-50 percent dose
reduction in the third trimester; stop
CORRECT ANSWER: C propranolol and survey esophageal varices
in the second trimester.
RATIONALE B. Continue penicillamine with a 25-50 percent
Acute viral hepatitis is the most common cause of dose reduction in the third trimester; stop
jaundice in pregnancy. Hyperemesis gravidarum propranolol and survey esophageal varices in
typically occurs in the first trimester of pregnancy; the second trimester.
mild elevations in aminotransferases are most C. Stop penicillamine and switch to daily zinc;
commonly seen, although AST and ALT levels continue propranolol and survey esophageal
have been observed to rise to as high as 700 to varices in the second trimester.
1000 U/L in some patients. Pre-eclampsia is typi- D. Continue penicillamine with a 25-50 percent
cally seen in the second trimester of pregnancy; dose reduction in the third trimester; continue
biochemical changes include the presence of liver propranolol and survey esophageal varices in
enzyme abnormalities, including 10- to 20-fold the second trimester.
elevations in aminotransferases, elevations in al- E. Stop penicillamine and switch to daily tri-
kaline phosphatase that are above what is typically entene with a 25-50 percent dose reduction
observed in pregnancy, and mild elevations in in the third trimester; continue propranolol
bilirubin up to 5 mg/dL. The hallmark symptom of and survey esophageal varices in the second
intrahepatic cholestasis of pregnancy is pruritus; trimester.
jaundice occurs in about 10 percent to 25 percent
of patients and usually does not occur until after CORRECT ANSWER: D
about two to four weeks after the onset of pruritus.
RATIONALE
REFERENCE Published guidelines on Wilson’s disease have
Licata A et al. Clinical course and management of recommended that penicillamine and trientine
acute and chronic viral hepatitis during pregnan- should be continued during pregnancy, with 25
cy. J Viral Hepat 2015;22:515-523. percent to 50 percent dose reductions occurring
during the third trimester in order to facilitate
wound healing in the event of cesarean section.
Question 25 Nadolol has a long half-life with low protein bind-
A 22-year-old nulliparous female at 8 weeks gesta- ing and low rate of excretion and thus is less fa-
tion presents to establish routine obstetrical care. vored compared to propranolol, which is thought
She has a past medical history of Wilson’s disease to be safe in pregnancy.
Chapter 8 — Gastrointestinal and liver disease in pregnancy 223

REFERENCES REFERENCE
Roberts EA, Schilsky ML; American Association Fitzpatrick KE, Hinshaw K, Kurinczuk JJ, Knight
for Study of Liver Diseases (AASLD). Diagno- M. Risk factors, management, and outcomes
sis and treatment of Wilson disease: an update. of hemolysis, elevated liver enzymes, and low
Hepatology 2008 Jun;47(6):2089-111. platelets syndrome and elevated liver enzymes,
Sandhu BS, Sanyal AJ. Pregnancy and liver low platelets syndrome. Obstet Gynecol 2014
disease. Gastroenterol Clin North Am 2003 Mar;123(3):618-627.
Mar;32(1):407-436.

Question 27
Question 26 A 34-year-old female presents for routine antena-
A 29-year-old woman at 37 weeks gestation tal care at 33 weeks gestation. She complains of
presents to the emergency room with right upper several weeks of diffuse pruritus, and has recently
quadrant pain, nausea, vomiting. She is diag- noted yellowing of her sclera. Laboratory results
nosed with preeclampsia. She is treated with in- are significant for: AST 650 U/L, ALT 525 U/L,
travenous magnesium, antihypertensive therapy, total bilirubin 3.9 mg/dL, GGT 45 U/L, hgb 12.2
and labor is induced. Prior to delivery, laboratory g/dL, platelets 190,000 k/µL, INR 1.3, serum bile
values were as follows: AST 240 U/L, ALT 220 acids 35 μmol/L. Which of the following is the best
U/L, total bilirubin 1.8 mg/dL, hemoglobin 10.1 next step in management?
g/dL, platelets 110,000 k/µL. Forty-eight hours
following delivery, she complains of worsening A. Intravenous dexamethasone and delivery at
right upper quadrant pain and headache. Repeat 34 weeks
laboratory values 48 hours post-partum were as B. Start ursodeoxycholic acid (UDCA) at a dose
follows: AST 410 U/L, ALT 390 U/L, total bili- of 10 mg/kg to 15 mg/kg
rubin 5.1 mg/dL, hemoglobin 7.9 g/dL, platelets C. Test for deficiency of 3-hydroxyacyl-CoA dehy-
75,000 k/µL. Which of the following is the most drogenase (LCHAD)
likely diagnosis? D. Right upper quadrant ultrasound
E. Percutaneous liver biopsy
A. Sepsis with hemolytic-uremic syndrome
(HUS) and thrombotic thrombocytopenic CORRECT ANSWER: B
purpura (TTP)
B. Acute fatty liver of pregnancy RATIONALE
C. Flare of autoimmune hepatitis This clinical scenario is consistent with intrahepatic
D. Hemolysis, elevated liver enzymes, low platelet cholestasis of pregnancy. The treatment of choice
count (HELLP) syndrome in ICP is ursodeoxycholic acid (UDCA) at a dose of
E. Acute hepatitis A infection 10 mg/kg to 15 mg/kg. The mechanism of UDCA in
the management of ICP is not fully understood but
CORRECT ANSWER: D is thought to involve increases in the expression of
bile salt export pumps and increases in placenta bile
RATIONALE transporters. UDCA has been shown to significantly
HELLP syndrome is a multisystemic disorder that reduce pruritus, normalize or decrease ALT levels
is characterized by the development of hemolytic and reduce bile acid levels. Early delivery of the
anemia, elevated liver enzymes, and low platelets. fetus at 37 weeks is advised since the risk of intra-
Most cases occur between 28 and 36 weeks of uterine death is thought to be more common during
gestation, but it can also develop up to one week the last month of pregnancy. LCHAD deficiency is
postpartum in 30 percent of cases. associated with acute fatty liver of pregnancy.
224 Digestive Diseases Self-Education Program®

REFERENCE of choledocholithiasis may require endoscopic


Bacq Y, Sentilhes L, Reyes HB, Glantz A, Kon- retrograde cholangiopancreatography (ERCP),
drackiene J, Binder J, Nicastri PL, Locatelli A, which can be performed safely in pregnancy with
Foreani A, Hernandez I, DiMartino V. Efficacy reduction in fetal radiation exposure facilitated by
of ursodeoxycholic acid in treating intrahepatic external shielding, patient positioning and reduc-
cholestasis of pregnancy: a meta-analysis. Gastro- tion in fluoroscopy time.
enterology 2012;143:1492-1501.
REFERENCES
Ko CW et al. Incidence, natural history, and risk
Question 28 factors for biliary sludge and stones during preg-
A 38-year-old G2P1 female with a past medical nancy. Hepatology 2005;41:359-365.
history of obesity, dyslipidemia and hypertension Tang SJ et al. Safety and utility of ERCP during
and non-alcoholic fatty liver disease presents with pregnancy. Gastrointest Endosc 2009;69:453-461.
right upper quadrant pain, nausea and vomiting at
33 weeks gestation. Right upper quadrant ultra-
sound reveals cholelithiasis without gallbladder Question 29
wall thickening or pericholecystic fluid. The com- Which of the following is true regarding acute
mon bile duct is dilated to 7mm with a suspected hepatitis E virus infection (HEV) during preg-
stone in the distal duct. Laboratory results are nancy?
significant for: AST 40 U/L, ALT 35 U/L, total
bilirubin 1.8 mg/dL, alkaline phosphatase 210 A. HEV Genotypes 1 and 2 are associated with
U/L. Which of the following is the best next step decreased maternal morbidity and mortality
in management? during pregnancy.
B. Acute HEV during pregnancy has not been
A. Start ursodeoxycholic acid (UDCA) at a dose reported to cause acute liver failure necessitat-
of 10 mg/kg to 15 mg/kg ing liver transplantation.
B. Endoscopic retrograde cholangiopancreatog- C. Ribavirin use for the treatment of acute HEV
raphy (ERCP) infection during pregnancy is contraindicated.
C. Open cholecystectomy D. The mortality rate associated with HEV infec-
D. Supportive care tion in pregnancy is thought to can be as high
E. Laparoscopic cholecystectomy as 80 percent to 90 percent.
E. HEV is not transmitted vertically to the fetus.
CORRECT ANSWER: B
CORRECT ANSWER: C
RATIONALE
Gallstones can be seen in up to five percent of RATIONALE
women at the start of pregnancy, with prevalence Increased morbidity and mortality in pregnancy
of gallstones increasing to 7.9 percent by the third are observed with acute HEV genotypes 1 and 2;
trimester. Risk factors for the development of genotypes 3 and 4 are more commonly associated
gallstones in pregnancy include pre-pregnancy with a more benign clinical course. The mortality
obesity, elevated serum leptin levels, and de- rate from HEV infection in pregnancy can be as
creased levels of high density lipoprotein (HDL). high as 20 percent to 50 percent. HEV has been
Conservative medical management is preferred in reported to cause acute liver failure during preg-
the first and third trimesters due to an increased nancy leading to liver transplantation. Obstetrical
risk of abortion and premature labor, respec- complications of HEV infection in pregnancy in-
tively, with surgical intervention. Management clude antepartum hemorrhage, preterm delivery,
Chapter 8 — Gastrointestinal and liver disease in pregnancy 225

prematurity, and intrauterine fetal death. Verti- vudine, telbivudine, or tenofovir to mothers with
cal transmission of HEV has also been reported. HBV DNA of more than 200,000 IU/L. Tenofovir
Treatment is supportive. Although ribavirin has is the preferred antiviral agent due to its potency
been used in cases of HEV infection in non-preg- and due to drug resistance seen with use of lami-
nant patients, its use is precluded in pregnancy vudine and telbivudine.
due to teratogenicity.
REFERENCE
REFERENCES Terrault NA, Lok ASF, McMahon BJ, Chang
Devarbhavi H, Kremers WK, Dierkhising R, KM, Hwang JP, Jonas MM, Brown RS, Bzowej
Padmanabhan L. Pregnancy-associated acute NH, Wong JB. Update on prevention, diagno-
liver disease and acute viral hepatitis: differen- sis, and treatment of chronic hepatitis B: AASLD
tiation, course and outcome. J Hepatol 2008 2018 hepatitis B guidance. Hepatology 2018
Dec;49(6):930-5. Apr;67(4):1560-1599.
Khuroo MS, Kamili S, Khuroo MS. Clinical
course and duration of viremia in vertically trans-
mitted hepatitis E virus (HEV) infection in babies Question 31
born to HEV-infected mothers. J Viral Hepat. All of the following are indicated as part of the
2009 Jul;16(7):519-523. postnatal management of infants born to moth-
ers with chronic hepatitis B virus (HBV) infection
EXCEPT:
Question 30
A 32-year-old female G1P0 at 32 weeks gestation A. Breastfeeding is contra-indicated if the
presents for routine antenatal follow-up. She has mother is on anti-viral therapy.
a history of vertically-acquired chronic hepatitis B. Administer hepatitis B immune globulin
B virus (HBV) infection and has never required (HBIG) and hepatitis B vaccination to infants
antiviral therapy. Her current laboratory studies within first 12 hours after birth.
are notable for the following: AST 25 U/L, ALT 20 C. Administer hepatitis B vaccination to infants
U/L, ALP 210 U/L, HBV surface antigen positive, at one to two months of age.
anti-HBV surface IgG negative, anti-HBV core IgG D. Administer hepatitis B vaccination to infants
positive, anti-HBV e antigen IgG negative, HBV e at six months of age.
antigen positive, HBV DNA 280,000 IU/L. E. Confirm response to hepatitis B vaccination
Which of the following is the best next step in with anti-HBV surface IgG between nine to 18
management? months of age.

A. Start telbivudine CORRECT ANSWER: A


B. Supportive care and delivery at 37 weeks
C. Start tenofovir RATIONALE
D. Start entecavir Use of hepatitis B immunoprophylaxis has mark-
E. Start lamivudine edly reduced the rate of vertical transmission in
children born to mothers with hepatitis B. Im-
CORRECT ANSWER: C munoprophylaxis consists of the administration of
hepatitis B immune globulin (HBIG) and hepatitis
RATIONALE B vaccination to infants within the first 12 hours
The American Association for the Study of Liver after birth, followed by scheduled administra-
Disease (AASLD) guidance on hepatitis B recom- tion of additional hepatitis B vaccination at one
mends institution of antiviral therapy with lami- to two months of age and again at 6 months of
226 Digestive Diseases Self-Education Program®

age. Response to immunoprophyaxis should be


confirmed via anti-HBV surface IgG between nine
to 18 months of age; maternal anti-HBV core IgG
may be detected until 24 months of age.
Breastfeeding is not contraindicated in cases
where immunoprophylaxis has been administered.
Although antiviral drug labels do not recommend
breastfeeding on antiviral therapy, clinical studies
support the safety of these drugs during breast-
feeding.

REFERENCE
Terrault NA, Lok ASF, McMahon BJ, Chang
KM, Hwang JP, Jonas MM, Brown RS, Bzowej
NH, Wong JB. Update on prevention, diagno-
sis, and treatment of chronic hepatitis B: AASLD
2018 hepatitis B guidance. Hepatology 2018
Apr;67(4):1560-1599.
Answers & critiques

CHAPTER 9

Diarrhea and constipation


Jennifer Inra, MD and Sonia Yoon, MD and Christina Ha, MD, FACG, AGAF

Question 1 movements per week, as well as abdominal pain/


A 25-year-old man with no significant medical discomfort and bloating. Side effects include
history presents for follow up of irritable bowel diarrhea and nausea. Using fiber for manage-
syndrome with constipation predominant symp- ment of IBS-C is reasonable given the lack of
toms (IBS-C). He has a bowel movement twice serious adverse effects, however, its use is often
per week, with associated abdominal cramping limited by side effects such as bloating and gas.
and bothersome bloating. His only medica- PEG is an inexpensive, effective osmotic laxative.
tion is 17 grams of polyethylene glycol (PEG) in The dose of PEG is typically 17 grams dissolved
eight oz of water, which he takes two times per in eight oz of water, but the dose is easily titrated
day. He has never tried any other laxatives. He up or down based on effect. Side effects of PEG
reports that his IBS symptoms are impacting his may include bloating or abdominal pain. Ad-
quality of life and his ability to function at work. ditionally, PEG is effective to treat constipation,
His physical exam is unremarkable, including but has no effect on improving abdominal pain.
rectal exam. Rifaximin is a nonabsorbable antibiotic, and is
used in patient with IBS who have abdominal
Which of the following treatment options is most pain and bloating, but not constipation. Dicy-
appropriate for this patient? clomine is an anticholinergic antispasmodic
medication that works to directly affect intes-
A. Stop PEG and start linaclotide tinal smooth muscle relaxation. Side effects of
B. Add fiber dicyclomine include constipation. Dicyclomine
C. Stop PEG and start rifaximin should be used in IBS-C if abdominal pain
D. Add dicyclomine persists after constipation is aggressively treated
E. Stop PEG and start tegaserod and controlled. Tegaserod increases colonic
motility and was approved for the treatment of
CORRECT ANSWER: A IBS-C, however, it was withdrawn from the US
market due to serious cardiovascular side effects
RATIONALE and is no longer used.
Patients with moderate to severe symptoms of
IBS that impair quality of life should be treated REFERENCES
with pharmacologic agents. The use of lina- Rao S, Lembo AJ, Shiff SJ, Lavins BJ, Currie
clotide in the treatment of IBC-C is recommend- MG, Jia XD, Shi K, MacDougall JE, Shao JZ,
ed in patients with constipation unresponsive to Eng P, Fox SM, Schneier HA, Kurtz CB, John-
PEG. Studies have shown improvement in bowel ston JM. A 12-week, randomized, controlled trial
related endpoints such as stool consistency, with a 4-week randomized withdrawal period to
straining and number of spontaneous bowel evaluate the efficacy and safety of linaclotide in

227
228 Digestive Diseases Self-Education Program®

irritable bowel syndrome with constipation. Am J rologies in patients with suspected IBS-diarrhea or
Gastroenterol. 2012 Nov;107(11):1714-24. IBS-mixed, especially in potential high-risk groups
Chey WD, Kurlander J, Eswaran S. Irritable (for example, patients with Down’s syndrome, type
bowel syndrome: a clinical review. JAMA. 2015: 1 diabetes, Turner syndrome). The recommended
313(9): 949-958. initial screening test for celiac disease is anti-tissue
transglutaminase (TTG) immunoglobulin A (IgA),
as well as a total IgA to make sure the patient is not
Question 2 IgA deficient. Colonoscopy with random biop-
A 34-year-old woman with type 1 diabetes pres- sies is recommended if the clinical suspicion for
ents for evaluation of diarrhea, abdominal cramp- microscopic colitis is high based on history (typi-
ing improved with a bowel movement and bloat- cally a middle-aged female with watery diarrhea).
ing. She has no rectal bleeding, weight loss or Colonoscopy should also be performed if alarm
nocturnal stools. She has not noticed symptom features are present to evaluate for malignancy and
association with food. She takes no medications, inflammatory bowel disease (IBD), or in those over
does not smoke cigarettes and drinks four glasses the age of 50 for colorectal cancer screening. For
of wine on the weekends. Her brother was diag- patients who meet criteria for IBS without alarm
nosed with celiac disease in his 20s by serology features, the combination of a normal C-reactive
and duodenal biopsy. Her physical exam is unre- protein and fecal calprotectin can be used to rule-
markable. A complete blood count is normal. out IBD without doing a colonoscopy. If a patient
is consuming significant lactose or there is a clear
Which one of the following diagnostic approaches link between lactose ingestion and symptoms by
is most appropriate for this patient? history or review of a food diary, a trial of lactose
exclusion or a lactose hydrogen breath test can be
A. Anti-tissue transglutaminase immunoglobulin considered, though a lactose breath test is time
A and total IgA consuming. The likelihood of thyroid abnormali-
B. Colonoscopy with random biopsies ties is no greater in patients with IBS as compared
C. Trial of lactose exclusion and reintroduction to normal controls, and therefore checking routine
D. Thyroid stimulating hormone thyroid stimulating hormone (TSH) is not recom-
E. Stool culture mended. Similarly, the yield of checking stool
cultures in patients with suspected IBS is low and
CORRECT ANSWER: A is not routinely recommended.

RATIONALE REFERENCES
The diagnosis of irritable bowel syndrome (IBS) is Brandt L J, Chey W D, Foxx-Orenstein A E. et
based on symptom-based criteria and the appro- al. An evidence-based position statement on the
priate evaluation of other causes. The Rome IV management of irritable bowel syndrome. Am J
criteria for the diagnosis of IBS includes the asso- Gastroenterol. 2009;104(Suppl 1):S1–S35.
ciation of abdominal discomfort with altered bowel Chey WD, Kurlander J, Eswaran S. Irritable
habits (diarrhea, constipation, or mixed) and/ bowel syndrome: a clinical review. JAMA. 2015:
or defecation. Patients who fulfill criteria for IBS 313(9): 949-958.
and who have no alarm features (nocturnal stools,
rectal bleeding, weight loss, unexplained iron
deficiency anemia, or family history of colorectal Question 3
cancer or inflammatory bowel disease) require A 34-year-old man has a history of IBS-D. He has
little formal testing to exclude other causes for loose stools up to four times per day associated
symptoms. It is recommended to check celiac se- with abdominal cramping, which is relieved by
Chapter 9 — Diarrhea and and constipation 229

a bowel movement. He takes loperamide daily ritable bowel syndrome: a systematic review. Gut.
which improves his diarrhea, however, he contin- 2010;59(3):325.
ues to have significant abdominal cramping. He
has tried bentyl in the past with little benefit.
Question 4
What of the following treatments should be tried A 42-year-old man presents for initial evaluation
next for this patient? of diarrhea. He has associated abdominal cramp-
ing which is relieved by a bowel movement and
A. Selective norepinephrine-reuptake inhibitors bloating. His symptoms are worse after eating,
(SNRI) though he does not identify any specific food
B. Probiotics triggers. His symptoms are worse with job stress.
C. Selective serotonin-reuptake inhibitors (SSRI) Celiac serologies are negative, thyroid stimulating
D. Tricyclic antidepressant (TCA) hormone is normal. His hematocrit is 33 percent
E. Diphenoxylate/atropine (normal range: 38 to 50 percent) and his iron
studies are low.
CORRECT ANSWER: D
What is the next best step?
RATIONALE
Tricyclic antidepressants (TCAs) and perhaps A. Colonoscopy
SNRIs can be used to treat patients with IBS due to B. Fiber supplementation
their effects on pain perception and motility. Side C. Loperamide
effects of TCAs include dose-dependent constipa- D. CT scan of the abdomen and pelvis
tion, dry mouth and eyes, drowsiness, weight gain, E. Low FODMAP diet
and QT interval prolongation. Because TCAs can
cause constipation, they should be preferentially CORRECT ANSWER: A
(but not exclusively) used in patients with IBS-D.
Side effects of SSRIs, on the other hand, include RATIONALE
diarrhea, sexual dysfunction, agitation, nausea and IBS should be suspected in patients with ab-
drowsiness. There is no data to support SSRI use dominal pain and constipation and/or diarrhea.
in IBS. There are little data addressing the efficacy Specifically, the Rome IV criteria define IBS as
of SNRIs for the treatment of IBS. Probiotics have abdominal pain at least one day per week in the
been associated with an improvement in symptoms last three months, associated with two or more
in patients with IBS, however, the magnitude of of the following: related to defecation, associated
benefit and the most effective species and strain are with a change in stool frequency or associated
uncertain. Diphenoxylate/atropine (lomotil) can be with a change in stool form. Generally, in patients
used to improve diarrhea, however, it has no effect with IBS-D, a CBC, inflammatory markers, TTG
on improving abdominal pain. IgA and IgA should be obtained. Further testing
should be guided by the clinical presentation or
REFERENCES the presence of “red flag” symptoms (onset after
Dekel R, Drossman DA, Sperber AD. The age 50 years, rectal bleeding or melena, noctur-
use of psychotropic drugs in irritable bowel nal diarrhea, unexplained weight loss, laboratory
syndrome. Expert Opin Investig Drugs. abnormalities such as anemia or elevated inflam-
2013;22(3):329-339. matory markers, or a family history of IBD or
Moayyedi P, Ford AC, Talley NJ, Cremonini colorectal cancer). Due to his anemia, a colonos-
F, Foxx-Orenstein AE, Brandt LJ, Quigley EM. copy would be the best next step to evaluate for
The efficacy of probiotics in the treatment of ir- malignancy or IBD.
230 Digestive Diseases Self-Education Program®

REFERENCE which would require colonoscopy for diagnosis,


Brandt LJ, Chey WD, Foxx-Orenstein AE, Schiller the temporal relationship of the increased dose of
LR, Schoenfeld PS, Spiegel BM, Talley NJ, Quig- citalopram and onset of diarrhea suggests this is a
ley EM. An evidence-based position statement side effect of the drug, and may respond to discon-
on the management of irritable bowel syndrome. tinuation of the drug alone.
American College of Gastroenterology Task Force
on Irritable Bowel Syndrome, Am J Gastroenterol. REFERENCE
2009;104 Suppl 1:S1. Dekel R, Drossman DA, Sperber AD. The use of
psychotropic drugs in irritable bowel syndrome.
Expert Opin Investig Drugs. 2013;22(3):329-339.
Question 5
A 40-year-old woman has a history of well con-
trolled IBS-D for the past five years. She uses lop- Question 6
eramide daily and avoids foods that predispose her A 32-year-old woman presents to you for evalua-
to diarrhea. About three weeks ago, she started to tion of possible celiac disease, as her brother has
have three to five loose bowel movements per day. celiac disease, diagnosed by serology and duodenal
She is otherwise healthy except for depression, as biopsy one year ago. Because of her family history,
her husband was recently diagnosed with termi- she has been gluten-free for the past nine months.
nal cancer six weeks ago. Her medications include
NSAIDs for occasional muscle aches and she was What is the best first step in her evaluation?
recently started on citalopram, the dose of which
was increased to 40mg daily one month ago. A. TTG IgA and total IgA
B. Deamidated gliadin peptide IgG
What is the best next step in her management? C. Anti-endomysial IgA
D. Gluten challenge and upper endoscopy
A. Fecal calprotectin E. HLA DQ2 and DQ8
B. Colonoscopy
C. Start probiotics CORRECT ANSWER: A
D. Stop citalopram
E. Increase her loperamide dose RATIONALE
A patient already on a gluten free diet poses a
CORRECT ANSWER: D diagnostic challenge for confirming celiac disease.
The first step in evaluation should be to check a
RATIONALE TTG IgA and total IgA. If positive, the patient has
This patient had previously well controlled IBS, probable celiac disease and would need duodenal
with an acute worsening of her symptoms. The biopsies for confirmation. If negative, HLA DQ2
new onset diarrhea is likely caused by citalopram, or DQ8 should be checked. These genotypes are
a selective serotonin reuptake inhibitor with not influenced by diet and can be used to evaluate
the side effect of diarrhea. This medication was the likelihood of celiac disease on a gluten-free
started recently and the dose was titrated up to the diet. HLA-DQ2/DQ8 testing should be performed
maximum dose, which has a higher likelihood of before initiating a formal gluten challenge, as a
causing diarrhea. She may benefit from changing negative result excludes the diagnosis of celiac. If
classes of antidepressants. These other interven- HLA DQ2 or DQ8 are positive, the next step is a
tions/evaluations may be appropriate if stopping gluten challenge for six to eight weeks, followed
her citalopram does not have any effect on symp- by an EGD with duodenal biopsies. TTG IgA and
toms. While SSRIs can cause microscopic colitis, total IgA are first line screening tests for patients
Chapter 9 — Diarrhea and and constipation 231

on a gluten-containing diet without IgA deficiency. CORRECT ANSWER: B


Deamidated gliadin peptide IgG is the first line
screening test in patients with IgA deficiency. Anti- RATIONALE
endomysial IgA is considered a second line screen- This patient has olmesartan-induced enteropathy.
ing test due to cost and subjective interpretation. This condition is identical to celiac disease histo-
logically (intraepithelial lymphocytes with villous
REFERENCES blunting), however olmesartan-induced enter-
Rubio-Tapia A, Hill ID, Kelly CP, et al. ACG clini- opathy is distinguished from celiac disease by the
cal guidelines: Diagnosis and management of presence of normal celiac serologies and the ab-
celiac disease. Am J Gastroenterol 2013; 108:656. sence of response to a gluten free diet. Withdrawal
Lebwohl B, Rubio-Tapia A, Assiri A et al. Di- of the medication leads to complete resolution
agnosis of celiac disease. Gastrointest Endosc Clin of symptoms and histology. A colonoscopy with
N Am 2012;22:661–677. random biopsies would be appropriate if it were
suspected that this patient had microscopic colitis.
An abdominal/pelvic CT scan is not warranted in
Question 7 this situation. Loperamide can be used for symp-
A 72-year-old woman with a history of hyperten- tom management, but it is not the best next step,
sion presents to you for a second opinion. She has as withdrawing the offending medication will lead
had six months of diarrhea, abdominal discom- to resolution of symptoms and histology. Probiot-
fort, bloating and weight loss. Her prior workup ics are not routinely recommended for the treat-
included normal labs, including a TTG IgA and ment of Olmesartan-induced enteropathy.
IgA while eating gluten, stool studies that were
negative for infection and an EGD with duodenal REFERENCES
biopsies shown below. She is currently on a gluten Rubio-Tapia A, Herman ML, Ludvigsson JF,
free diet with no improvement. Kelly DG, Mangan TF, Wu TT, Murray JA. Severe
spruelike enteropathy associated with olmesartan.
Mayo Clin Proc. 2012 Aug;87(8):732-8.
Burbure N, Lebwohl B, Arguelles-Grande C,
Green PH, Bhagat G, Lagana S. Olmesartan-asso-
ciated sprue-like enteropathy: a systematic review
with emphasis on histopathology. Hum Pathol.
2016 Apr;50:127-34.

Question 8
An 18-year-old man has a history of diarrhea,
abdominal pain and weight loss. A thyroid stimu-
lating hormone was 1.5 mIU/L (normal range: .4
to 4.0 mIU/L) and inflammatory markers were
What is the best next step in evaluation? normal. A hematocrit was 33 percent (normal
range: 40 to 54 percent). His physical exam was
A. Colonoscopy with random biopsies normal. His tissue transglutaminase IgA was 15
B. Review her medications (normal less than 19) and his total IgA was 20 mg/
C. Abdominal/pelvis CT scan dL (normal range: 80 to 350 mg/dL).
D. Start loperamide
E. Start probiotics What is the best next step in evaluation?
232 Digestive Diseases Self-Education Program®

A. HLA DQ2 and DQ8 showed villous blunting with increased intraepi-
B. Deaminated gliadin peptide IgG thelial lymphocytes.
C. Anti-endomysial IgA
D. Upper endoscopy with biopsy What is the preferred treatment for this patient?
E. Deamidated gliadin peptide IgA
A. Gluten free diet
CORRECT ANSWER: B B. Ceftriaxone IV followed by Bactrim PO
C. Low FODMAP diet
RATIONALE D. Tetracycline and folic acid
IgA deficiency is more common in celiac disease E. Rifaxmin
than in the general population. When testing for
celiac disease, total IgA levels should be mea- CORRECT ANSWER: D
sured and if low, IgA-based celiac serology testing
should not be used. Instead, IgG-based testing is RATIONALE
recommended, including DGPs IgG and/or TTG This patient has tropical sprue based on her travel
IgG. HLA DQ2 and DQ8 should be used to rule to an endemic country, negative celiac serologies,
out celiac disease in clinical situations where a pa- laboratories revealing a macrocytic anemia and
tient is already gluten-free, as patients with celiac low albumin and characteristic histology (villous
disease need to possess HLA DQ2 or DQ8. An up- blunting, increased intraepithelial lymphocytes).
per endoscopy should be obtained after a positive Treatment is with tetracycline and folate. Diag-
serology, not before. nosis of tropical sprue is ultimately confirmed by
a response to treatment. A gluten-free diet is not
REFERENCES appropriate, as the patient does not have celiac
Villalta D, Alessio MG, Tampoia M et al. Testing disease, confirmed by normal celiac serologies.
for IgG class antibodies in celiac disease patients Ceftriaxone IV followed by Bactrim PO is the cor-
with selective IgA deficiency. A comparison of the rect treatment for Whipple’s disease. A diet low
diagnostic accuracy of 9 IgG anti-tissue transglu- in fermentable oligo-, di-, and monosaccharides
taminase, 1 IgG anti-gliadin and 1 IgG anti-deam- and polyols (FODMAPs) is beneficial treatment
inated gliadin peptide antibody assays. Clin Chim in some patients with IBS with abdominal bloat-
Acta 2007;382:95–99. ing or pain. Rifaximin is the correct treatment for
Villalta D, Tonutti E, Prause C et al. IgG small intestine bacterial overgrowth or IBS-D.
antibodies against deamidated gliadin peptides
for diagnosis of celiac disease in patients with IgA REFERENCES
deficiency. Clin Chem 2010;56:464–468. Brown IS, Bettington A, Bettington M, Rosty C.
Tropical sprue: revisiting an underrecognized
disease. Am J Surg Pathol 2014; 38:666.
Question 9 Shah VH, Rotterdam H, Kotler DP, et al. All
An 18-year-old woman presents for evalua- that scallops is not celiac disease. Gastrointest
tion of chronic diarrhea, fatigue and abdominal Endosc 2000; 51:717.
cramping. She was recently in Puerto Rico for six
months visiting family and returned a few weeks
ago. Her labs are significant for a hemoglobin of 11 Question 10
with an MCV of 109. Her albumin is 3.6. She had A 46-year-old male with a history of chronic
stool studies which ruled out infection, including arthritis is referred for evaluation of diarrhea and
parasites. TtG IgA and total IgA were within nor- a 10-pound weight loss over the past five months.
mal limits. EGD with multiple duodenal biopsies His wife also notes that he has been more forgetful
Chapter 9 — Diarrhea and and constipation 233

lately. His medications include NSAIDS only. CORRECT ANSWER: A


What do you recommend as a next step in
evaluation? RATIONALE
The stool osmotic gap may be used to differentiate
A. CT scan of the abdomen and pelvis whether diarrhea is from an osmotic cause (lac-
B. EGD with small bowel biopsies tase deficiency, osmotic laxative use or abuse or a
C. Colonoscopy with random biopsies secretory cause (infections with Vibrio cholera, en-
D. Stool studies to rule out infection terotoxigenic strains of E. coli, VIPoma). The stool
E. MR enterography osmotic gap is calculated using the equation 290
− 2(stool Na + stool K), where 290 is the value of
CORRECT ANSWER: B the stool osmolality. A stool osmotic gap between
50 and 100 mosm/kg is normal. Osmotic diarrhea
RATIONALE has a stool osmotic gap of greater than100 mosm/
This patient has a classic presentation of Whip- kg. Secretory diarrhea has a stool osmotic gap of
ple’s disease. Patients are typically middle aged less than 50 mosm/kg.
white males. Joint pain symptoms are intermittent
and migratory and often precede other symptoms REFERENCE
by many years. Classic GI symptoms include Shiau YF, Feldman GM, Resnick MA, Coff PM
weight loss, diarrhea and abdominal pain. Neu- (June 1985). “Stool electrolyte and osmolality
rologic symptoms include confusion, depression, measurements in the evaluation of diarrheal dis-
personality changes or nystagmus and rhythmic orders”. Ann. Intern. Med. 102 (6): 773–5.
contractions of muscles in the face, specifically in
muscles of mastication. Diagnosis begins with an
EGD with biopsies of the duodenum. Histology Question 12
reveals PAS positive macrophages in the lamina A 65-year-old woman presents for evaluation of
propria. Confirmation can be performed with im- diarrhea for the past 12 weeks. She has five watery
munohistochemistry or PCR. bowel movements per day, including frequent
nocturnal episodes of diarrhea. She has no blood
REFERENCE in her stool. She’s had 10-pound unintentional
Marth T, Moos V, Müller C, Biagi F, Schneider weight loss. She has a colonoscopy with multiple
T. Tropheryma whipplei infection and Whipple’s random biopsies of her colon which reveals micro-
disease. Lancet Infect Dis. 2016 Mar;16(3): scopic colitis. She is started on budesonide 9mg
e13-22. daily but does not have clinical response after a
full course of the mediation.

Question 11 What is the best next step in her management?


A 32-year-old woman has lactose intolerance
confirmed by hydrogen breath testing. Which of A. Retreat with budesonide but increase dose to
the following stool samples is most consistent with 9mg twice a day
this diagnosis? B. Retreat with bismuth salicylate
C. Retreat with Cipro and Flagyl
A. Stool osmotic gap of 120 mosm/kg D. Retreat with mesalamine
B. Stool osmotic gap of 85 mosm/kg E. Review her medication list
C. Stool osmotic gap of 60 mosm/kg
D. Stool osmotic gap of 45 mosm/kg CORRECT ANSWER: E
E. Stool osmotic gap of 15 mosm/kg
234 Digestive Diseases Self-Education Program®

RATIONALE CORRECT ANSWER: C


The primary goal of management in patients with
microscopic colitis is to achieve clinical remis- RATIONALE
sion, typically with the use budesonide 9mg daily. Microscopic colitis is more common in patients
Budesonide is used at a maximum dose of 9mg with celiac disease than in the general population.
daily. In patients with no improvement in diarrhe- In most patients, microscopic colitis is diagnosed
al symptoms, alternate causes of diarrhea should after the diagnosis of celiac disease due to the oc-
be evaluated (ie. celiac disease) and potential currence of persistent diarrhea. However, celiac
precipitating medications should be avoided (ie. disease should also be considered in patients who
PPIs, SSRIs, ASA, NSAIDs). Bismuth salicylate or are diagnosed with microscopic colitis with persis-
mesalamine may be used as a second line treat- tent symptoms, though this sequence of diagnosis
ment in patients where budesonide is contrain- accounts for only 11 percent of patients as docu-
dicated or the cost is prohibitive. Cipro and flagyl mented in one study.
are not used for microscopic colitis
REFERENCES
REFERENCE Matteoni CA, Goldblum JR, Wang N, et al. Celiac
American Gastroenterological Association. AGA disease is highly prevalent in lymphocytic colitis. J
Institute Guideline on the Management of Mi- Clin Gastroenterol 2001;32: 225–227.
croscopic Colitis: Clinical Decision Support Tool. Green PH, Yang J, Cheng J, Lee AR, Harper
Gastroenterology. 2016 Jan;150(1):276. JW, Bhagat G. An association between micro-
scopic colitis and celiac disease. Clin Gastroenterol
Hepatol. 2009 Nov;7(11):1210-6.
Question 13 Stewart M, Andrews CN, Urbanski S, Beck PL,
A 40-year-old woman has a history of micro- Storr M. The association of coeliac disease and mi-
scopic colitis diagnosed by random colon biopsy croscopic colitis: a large population-based study.
12 months ago. Her symptoms on diagnosis were Aliment Pharmacol Ther. 2011 Jun;33(12):1340-9.
seven watery bowel movements per day, associ-
ated with a 12-pound weight loss. At baseline,
she had two to three soft or mushy bowel move- Question 14
ments, mostly after eating, with some abdominal A 52-year-old female with a history of cervical
cramping. She was told she had IBS, though she cancer status post chemotherapy and radiation
was never fully evaluated. She was treated with presents to you for chronic diarrhea for the past
budesonide for microscopic colitis and her symp- two years. She has multiple loose stools per day
toms resolved and she returned to her baseline. with no blood. She has had no weight loss. Initial
Over the past few weeks, she has noticed an evaluation including a CBC and fecal calprotec-
increase in frequency and some abdominal cramp- tin are normal. What is the best next step in her
ing. She has a repeat colonoscopy with random management?
colon biopsies which was normal. What is the best
next step in evaluation? A. Colonoscopy
B. Lactose-free diet
A. Check c. diff C. Start Imodium
B. TSH D. Check Clostridium difficile
C. TTG IgA and total IgA E. Start colestipol
D. Fecal calprotectin
E. Hydrogen breath test for lactose intolerance CORRECT ANSWER: E
Chapter 9 — Diarrhea and and constipation 235

RATIONALE RATIONALE
Due to this patient’s history of cervical cancer Potential complications of SIBO range include mini-
with radiation, she likely has radiation ileitis mal vitamin deficiencies including malabsorption of
causing bile acid (BA) diarrhea. Typically, BA B12, fat soluble vitamins (vitamin A, D, E and K) and
are reabsorbed in the ileum and returned in the iron. The nutritional consequences of SIBO result
enterohepatic circulation. In states of normal from maldigestion and malabsorption of nutrients
health, only a small amount of BA are not reab- in the intestine, due to microscopic damage to the
sorbed in the ileum and reach the colon. In states microvilli in the small intestine. B12 deficiency
of diseased ileum (Crohn’s disease, radiation results from competitive uptake of B12 from bacte-
ileitis after gynecologic cancers for example), BA ria. Folate can also be elevated in some patients due
are not effectively reabsorbed and reach the co- to increased production of folate by bacteria in the
lon, where bacteria deconjugate and dehydroxyl- small bowel. SIBO does not predispose patients to
ate them, producing secondary BA, which stimu- developing other medical conditions.
late water secretion in the colon and diarrhea.
The best treatment for BA diarrhea is colestipol, a REFERENCE
bile acid sequestrant. These medications improve Dukowicz AC, Lacy BE, Levine GM. Small In-
diarrhea, but have side effects of abdominal dis- testinal Bacterial Overgrowth: A Comprehen-
comfort and bloating. sive Review. Gastroenterology & Hepatology.
2007;3(2):112-122.Adike A, DiBaise JK. Small
REFERENCES Intestinal Bacterial Overgrowth: Nutritional Im-
Camilleri M. Bile Acid Diarrhea: Prevalence, plications, Diagnosis, and Management. Gastroen-
Pathogenesis, and Therapy. Gut and Liver. terol Clin North Am. 2018 Mar;47(1):193-208
2015;9(3):332-339.
Andreyev J. Gastrointestinal symptoms after
pelvic radiotherapy: a new understanding to Question 16
improve management of symptomatic patients. An 82-year-old woman reports abdominal pain,
Lancet Oncol. 2007 Nov;8(11):1007-17. bloating and constipation for the past three weeks.
Her stools were previously regular and soft. She
now has a bowel movement every three to four days
Question 15 and her stools are hard but contain no blood. Her
A 28-year-old woman has a diagnosis of small medical history includes depression, as her hus-
intestine bacterial overgrowth (SIBO). She is won- band died one month ago. Her medications include
dering if SIBO put her at risk for developing other amitriptyline 50mg nightly since her husband’s
medical conditions, or if there are any potential death, aspirin 81mg daily and a multivitamin with
complications. iron. Her rectal exam reveals normal tone and solid
stool in the vault. Laboratory testing reveals normal
Which of the following do you counsel her on? chemistries and complete blood count.

A. Vitamin B12 deficiency Which of the following recommendations is most


B. Vitamin K excess appropriate for this patient?
C. Inflammatory bowel disease
D. Small bowel malignancy A. Change amitriptyline to sertraline
E. Microscopic colitis B. Start lubiprostone
C. Stop multivitamin with iron
CORRECT ANSWER: A D. Order an abdominal and pelvic CT scan
E. Colonoscopy
236 Digestive Diseases Self-Education Program®

CORRECT ANSWER: A three times per week. She has significant strain-
ing with a bowel movement and feelings of in-
RATIONALE complete evacuation. She tried fiber supplemen-
Risk factors for chronic constipation in older tation, senna and polyethylene glycol 17grams
adults include female gender, physical inactiv- daily without improvement. She has no blood in
ity, low income/education, and depression. her stool and no weight loss.
The new onset constipation is likely caused by
amitriptyline, a tricyclic antidepressant with What is the best next step in evaluation?
a known side effect of constipation. She may
benefit from changing classes of antidepressants. A. MR defecography
Lubiprostone can be used to treat constipation, B. Colonoscopy
but the first step should be to stop any causative C. Digital rectal examination
medications if possible, rather than adding a D. Anorectal manometry with balloon expulsion
new medication, especially in an elderly patient. E. Check thyroid stimulating hormone
In addition, lubiprostone would generally not
be used before trying other laxatives such as CORRECT ANSWER: C
senna and polyethylene glycol, as they are less
expensive and available over the counter. Iron RATIONALE
supplementation is known to cause constipation, Defecatory disorders should be suspected based
abdominal discomfort, nausea and bloating, but on history and may include a report of significant
the patient has been chronically on iron; while straining with defecation, the need for perianal
the tricyclic antidepressant was newly started. or vaginal pressure to aid in defecation, feeling
There are no data to support the role of imaging of incomplete evacuation, and failure to respond
in the evaluation of constipation in older adults. to various laxatives. A digital rectal examination
Colonoscopy may be appropriate for the evalu- is the first step in evaluation of defecatory disor-
ation of constipation in older adults if there are ders, however a normal exam does not exclude
also alarm symptoms, such as rectal bleeding, this diagnosis. After a careful history and physi-
weight loss, iron deficiency anemia. cal exam, further workup should be obtained
based on history and symptoms. In the absence
REFERENCES of other clinical features, metabolic tests such
American Gastroenterological Association., as TSH are not recommended for evaluation of
Bharucha AE, Dorn SD, Lembo A, Pressman A. chronic constipation. A colonoscopy should not
American Gastroenterological Association medi- be performed in patients without alarm features
cal position statement on constipation. Gastro- such as blood in stools, anemia or weight loss,
enterology. 2013 Jan;144(1):211-7. unless age-appropriate colon cancer screening
Wald A. Constipation: Advances in Di- has not been performed. Anorectal manometry
agnosis and Treatment. JAMA. 2016 Jan with balloon expulsion should be performed in
12;315(2):185-91. patients who fail to respond to laxatives, but after
Stern T, Davis AM. Evaluation and Treat- a digital rectal exam. An MR defecography should
ment of Patients With Constipation. JAMA. 2016 be performed after anorectal manometry and
Jan 12;315(2):192-3. balloon expulsion test, especially when results are
inconclusive.

Question 17 REFERENCE
A 47-year-old woman presents for evaluation of American Gastroenterological Association,
chronic constipation. She has a bowel movement Bharucha AE, Dorn SD, Lembo A, Pressman A.
Chapter 9 — Diarrhea and and constipation 237

American Gastroenterological Association medi- not the correct therapy for dyssynergia. In the
cal position statement on constipation. Gastro- absence of other clinical features, metabolic tests
enterology. 2013 Jan;144(1):211-7. such as TSH are not recommended for evalua-
tion of chronic constipation

Question 18 REFERENCE
A 39-year-old woman has a history of dyssyner- American Gastroenterological Association,
gic defecation documented by anorectal manom- Bharucha AE, Dorn SD, Lembo A, Pressman A.
etry and balloon expulsion. Her initial symptoms American Gastroenterological Association medi-
included infrequent bowel movements, signifi- cal position statement on constipation. Gastro-
cant straining, the need to use perianal pressure enterology. 2013 Jan;144(1):211-7.
to defecate, and failure to respond to multiple
laxatives. She was referred for biofeedback
therapy and has had multiple sessions, but she Question 19
has not noticed any improvement. A 50-year-old woman presents with diarrhea
and fecal urgency, especially while eating out at
What is the best next step in evaluation? restaurants. She provides a stool sample after
one of these episodes and testing reveals the
A. MR defecography following:
B. Colonoscopy
C. Start linaclotide Stool osmolality: 290 mOsm/kg
D. Repeat balloon expulsion Stool Na+: 15mEq/L
E. Check thyroid stimulating hormone Stool K+: 10 mEq/L
pH: 4.5   
CORRECT ANSWER: D
Based on this sample, which of the following is the
RATIONALE best recommendation to improve her symptoms?
Pelvic floor retraining by biofeedback therapy
rather than laxatives is recommended for defeca- A. Bile acid sequestrant
tory disorders. Biofeedback can be used to train B. Vancomycin
patients to relax their pelvic floor muscles during C. Proton pump inhibitor
straining and to correlate relaxation and pushing D. Eliminate lactose from diet
to achieve defecation. Normal pelvic floor coor- E. Budesonide
dination can be restored. Biofeedback therapy
improves symptoms in more than 70 percemt of CORRECT ANSWER: D
patients with defecatory disorders. If biofeed-
back is unsuccessful, it is recommended to re- RATIONALE
peat balloon expulsion. If a repeat balloon expul- The stool osmotic gap may be used to differenti-
sion is abnormal, the next recommended step is ate whether diarrhea is from an osmotic cause
to obtain an MR defecography to assess for any or a secretory cause. The stool osmotic gap is
significant structural abnormalities. Colonosco- calculated using the equation 290 − 2(stool Na
py is not indicated in the workup of constipation + stool K), where 290 is the value of the stool os-
unless alarm features (blood in stools, anemia or molality. A stool osmotic gap between 50 and
weight loss) are present or age-appropriate colon 100 mosm/kg, a stool osmotic gap of greater
cancer screening has not been performed. Start- than 100 mosm/kg implies an osmotic cause for
ing linaclotide is not appropriate, as laxatives are diarrhea and a stool osmotic gap of less than 50
238 Digestive Diseases Self-Education Program®

mosm/kg implies a secretory cause for diarrhea. tion of the guanylate cyclase-C receptor leads to
This patient has a high gap, indicating an osmotic cyclic GMP release, which inhibits nociceptors,
cause for her diarrhea. Lactose intolerance is the and improves other symptoms of IBS including ab-
only option in the answer choices that has a high dominal pain. This medication therefore would be a
stool osmotic gap, indicating osmotic diarrhea. good option for this patient. Lubiprostone is another
prosecretory agent that activates the chloride chan-
REFERENCE nels on the small intestine enterocytes, leading to
Shiau YF, Feldman GM, Resnick MA, Coff PM water secretion into the lumen, however, it has no
(June 1985). “Stool electrolyte and osmolality effects on improvement in abdominal pain. Senna is
measurements in the evaluation of diarrheal dis- a stimulant laxative, but has no effect on improving
orders”. Ann. Intern. Med. 102 (6): 773–5. abdominal pain and can have side effects of pain and
cramping. Eluxadoline is a mu and kappa-opioid re-
ceptor agonist and is a delta opioid receptor antago-
Question 20 nist in the enteric nervous system. It is approved for
A 21-year-old woman has a history of IBS-C. She the management of IBS-D, not IBS-C.
has been taking polyethylene glycol (PEG) 34
grams a day for the past few months with good REFERENCES
effect, but she wishes to switch to another medica- Shah ED, Kim HM, Schoenfeld P. Efficacy and
tion as she doesn’t like the taste of PEG. She has Tolerability of Guanylate Cyclase-C Agonists for
bothersome abdominal cramping and bloating Irritable Bowel Syndrome with Constipation and
which did not improve with stopping PEG. Chronic Idiopathic Constipation: A Systematic
Review and Meta-Analysis. Am J Gastroenterol.
Which of the following medications do you recom- 2018 Mar;113(3):329-338.
mend? Ford AC, Moayyedi P, Chey WD, Harris LA,
Lacy BE, Saito YA, Quigley EMM; ACG Task Force
A. Insoluble fiber on Management of Irritable Bowel Syndrome.
B. Linaclotide American College of Gastroenterology Monograph
C. Lubiprostone on Management of Irritable Bowel Syndrome. Am
D. Senna J Gastroenterol. 2018 Jun;113(Suppl 2):1-18.
E. Eluxadoline

CORRECT ANSWER: B Question 21


An 18-year-old male presents to your office for
RATIONALE evaluation of diarrhea for the past two months. He
Soluble fiber, not insoluble fiber, is recommended is a college freshman and started school about three
for the management of IBS-C. Soluble fiber (wheat months ago. He is under a lot of stress, as he is pre-
dextrin, psyllium husk) is low risk and inexpen- med and has a lot of school work. Because of this, he
sive and therefore is a first line treatment for is drinking significant amounts of non-diet caffeinat-
IBS-C. Insoluble fiber is not recommended, as ed sodas throughout the day to help him stay awake
it may worsen abdominal pain and bloating and and study. Which of the following results is most
has shown no efficacy for IBS-C. Linaclotide is a likely to be found during this patient’s workup?
prosecretory agent that is an agonist of the guanyl-
ate cyclase-C receptor. Binding to this receptor A. Colonoscopy with inflammation in the rectum
activates the cystic fibrosis transmembrane chan- and sigmoid
nel, which causes water secretion into the colon B. MR enterography with thickening of the
lumen. There is also data to support that activa- terminal ileum
Chapter 9 — Diarrhea and and constipation 239

C. Stool culture positive for Clostridium difficile C. Glucose hydrogen breath test
D. Stool osmotic gap greater than100 D. Stop eluxadoline
E. Duodenal biopsies showing increased E. Low FODMAP diet
epithelial lymphocytes and villous blunting
CORRECT ANSWER: D
CORRECT ANSWER: D
RATIONALE
RATIONALE Eluxadoline is a mu and kappa-opioid receptor
This patient has osmotic diarrhea from drinking agonist and is a delta opioid receptor antagonist
significant quantities of soda containing fructose. in the enteric nervous system. It is approved by
Fructose is a natural sugar and a sweetener in so- the FDA for the management of IBS-D. The use of
das. Some healthy patients (up to 50 percent) have eluxadoline is contraindicated in in patients who
incomplete fructose absorption, as the capacity of have had a prior cholecystectomy, with a known
the intestine to transport fructose across the intes- or suspected history of biliary duct obstruction,
tinal epithelium is exceeded and therefore malab- sphincter of Oddi dysfunction, a history of pan-
sorption occurs. This non-absorbed sugar thereby creatitis or structural diseases of the pancreas,
causes an osmotic diarrhea. Removing soda from significant alcohol use (more than three drinks
the diet resolves the diarrhea. per day) or abuse, hepatic impairment. From May
2015 (when eluxadoline was first approved by the
REFERENCES FDA) through February 2017, the FDA received 120
Choi YK, Johlin FC Jr, Summers RW, Jackson reports of serious cases of pancreatitis or death.
M, Rao SS. Fructose intolerance: an under- Among 68 patients who reported their gallbladder
recognized problem. Am J Gastroenterol. 2003 status, 56 of them did not have a gallbladder and
Jun;98(6):1348-53. received the currently recommended dosage of
Gibson PR, Newnham E, Barrett JS, Shepherd eluxadoline. Given her history of cholecystectomy,
SJ, Muir JG. Review article: fructose malabsorp- she is at risk for pancreatitis with use of eluxadoline
tion and the bigger picture. Aliment Pharmacol due to sphincter of Oddi spasm. The other options
Ther. 2007 Feb 15; 25(4):349-63. are not appropriate as she is not at the age to start
colorectal cancer screening now and her symptoms
are well controlled on medication; therefore further
Question 22 workup and evaluation is unnecessary.
A 40-year-old woman presents to your office to es-
tablish care for her IBS-D. She has a history of hy- REFERENCE
pertension and a cholecystectomy for gallstones. VIBERZI (eluxadoline) [prescribing information].
She takes lisinopril and eluxadoline. She has no Irvine, CA: Allergan USA, Inc.; 2018.
family history of IBD, colon cancer or polyps. She https://www.fda.gov/Drugs/DrugSafety/
appears well and has no complaints. She is hav- ucm546154.htm (Accessed on January 8, 2019).
ing one formed bowel movement daily or every
other day, which is improved since she was first
diagnosed with IBS-D. She has minimal abdomi- Question 23
nal pain and bloating when she eats certain food, A 45-year-old woman presents for evaluation of
which she attributes to lactose-containing prod- constipation. She has a bowel movement every
ucts. What do you recommend to her? three days and her stools are hard. About one year
ago, she had a soft stool daily or every other day
A. Colonoscopy for screening now without straining or blood. She has a history of
B. Fecal calprotectin recently diagnosed hypertension and hyperlip-
240 Digestive Diseases Self-Education Program®

idemia. Her medications include verapamil and reveal lymphocytic colitis. Which of the following
atorvastatin. She has never tried any laxatives for medications may have predisposed her to develop-
her constipation. What is the best next step in her ing microscopic colitis?
evaluation?
A. Olmesartan
A. Start soluble fiber daily B. Sertraline
B. Stop verapamil C. Lisinopril
C. Sitz marker study D. Lorazepam
D. Anorectal manometry E. Metoprolol
E. MRI defecography
CORRECT ANSWER: B
CORRECT ANSWER: B
RATIONALE
RATIONALE Certain drugs have been identified as risk factors for
Verapamil is a calcium-channel blocker with a developing microscopic colitis. Multiple drugs have
known side effect of constipation. The best first been implicated, including aspirin, NSAIDS, selec-
step is to try to stop the medication and switch to tive serotonin reuptake inhibitors (SSRIs) and pro-
a different agent without side effects of constipa- ton pump inhibitors (PPIs). Smoking has also been
tion. Starting fiber would be an appropriate medi- identified as an environmental risk factor. Patients
cation choice for constipation however, the best should be counseled to discontinue any potential of-
first step is to stop any constipating medications. fending medications after the diagnosis is confirmed.
A sitz marker study, anorectal manometry or MRI
defecography could all be obtained for the workup REFERENCE
of constipation, however, offending medications Keszthelyi D, Jansen SV, Schouten GA, de Kort S,
should be stopped first if possible. Scholtes B, Engels LG, Masclee AA. Proton pump
inhibitor use is associated with an increased risk
REFERENCE for microscopic colitis: a case-control study. Ali-
Locke GR III, Pemberton JH, Phillips SF. AGA ment Pharmacol Ther. 2010 Nov;32(9):1124-8.
technical review on constipation. American Gas- Fernández-Bañares F, Esteve M, Espinós JC,
troenterological Association. Gastroenterology. Rosinach M, Forné M, Salas A, Viver JM. Drug
2000;119(6):1766–1778. consumption and the risk of microscopic colitis.
Bassotti G, Calcara C, Annese V, Fiorella S, Am J Gastroenterol. 2007 Feb;102(2):324-30.
Roselli P, Morelli A. Nifedipine and verapamil
inhibit the sigmoid colon myoelectric response to
eating in healthy volunteers. Dis Colon Rectum. Question 25
1998 Mar;41(3):377-80.80. An 18-year-old woman has a one-year history of
diarrhea associated with food intake. She is healthy,
but has iron deficiency anemia and Grave’s disease.
Question 24 She has no family history of inflammatory bowel
A 65-year-old woman presents for a surveillance disease, celiac disease or other gastrointestinal dis-
colonoscopy due to a personal history of adeno- eases. She notes that she has maintained a “100 per-
mas. She mentions in the procedure room that she cent%” gluten-free diet for the past four months and
is also having three to four loose bowel move- the consistency of her stools have improved, though
ments per day, sometimes waking her from sleep. she continues to have intermittent diarrhea, espe-
You perform the colonoscopy and find no polyps. cially when drinking lactose containing products.
You take random biopsies of the colon which What is the best recommendation for this patient?
Chapter 9 — Diarrhea and and constipation 241

A. Gluten-free diet and lactose free diet indefinitely A. Measurement of serum IgA tissue
B. Gluten challenge and upper endoscopy transglutaminase (tTG) with serum IgA
C. Colonoscopy with random biopsies B. Colonoscopy with biopsies
D. SIBO breath testing C. Glucose hydrogen breath test
E. Thyroid stimulating hormone D. Upper endoscopy with duodenal biopsies

CORRECT ANSWER: B CORRECT ANSWER: A

RATIONALE RATIONALE
Based on her history of diarrhea, iron deficiency There is evidence that celiac disease is more
anemia and a diagnosis of an endocrine autoim- common in patients with Type I DM compared
mune condition, the diagnosis of Celiac Disease to the general Caucasian population. A variety of
should be highly suspected. It is also likely that she serologic assays are available to aid in the pri-
has secondary lactose intolerance, which is com- mary diagnosis of celiac disease (CD). Currently,
mon in Celiac Disease. Because she is gluten-free, serum IgA with IgA tissue transglutaminase is
the best next step to confirm the diagnosis is a the preferred single test for the detection of CD.
gluten challenge followed by upper endoscopy with A colonoscopy may be indicated if the diagnostic
duodenal biopsies. Celiac disease should be differ- evaluation for CD is negative however, this would
entiated from nonceliac gluten sensitivity because not be the best next step in the evaluation of this
patients with celiac disease are at risk for nutrition- patient’s symptoms. A glucose breath test would
al deficiencies, osteoporosis, small bowel tumors, aid in the diagnostic evaluation of small intestinal
and require a pneumococcal vaccine and DEXA bacterial overgrowth, but CD remains the most
scan. The diagnosis of celiac disease may also have likely diagnosis. According to current published
important implications for family members. guidelines, an upper endoscopy with duodenal
biopsies would be indicated in those patients in
REFERENCES whom the suspicion for CD is high despite nega-
Rubio-Tapia A, Hill ID, Kelly CP, et al. ACG clini- tive serologies, however, serologic evaluation
cal guidelines: Diagnosis and management of should be pursued first.
celiac disease. Am J Gastroenterol 2013; 108:656.
Lebwohl B, Rubio-Tapia A, Assiri A et al. Di- REFERENCE
agnosis of celiac disease. Gastrointest Endosc Clin Rubio-Tapia A, Hill ID, Kelly CP, et al. ACG clini-
N Am 2012;22:661–677. cal guidelines: diagnosis and management of
celiac disease. Am J Gastroenterol 2013; 108:656

Question 26
A 19-year-old woman with Type I diabetes mel- Question 27
litus (DM) is referred to you for new onset symp- Which of the following statements regarding
toms of intermittent loose stools and bloating. vitamin deficiencies or excess in small intestinal
She denies weight loss or GI bleeding. There is no bacterial overgrowth is true?
family history of inflammatory bowel disease or
gastrointestinal cancer. Physical exam is normal. A. Enteric bacteria compete with the host for
Laboratory evaluation reveals a low Vitamin D vitamin B12, resulting in vitamin B12 deficiency
level and iron deficiency. B. Enteric bacteria compete with the host for
vitamin K, resulting in vitamin K deficiency
What is the best next step in the evaluation of this C. Thiamine is synthesized by enteric bacteria,
patient’s symptoms? leading to thiamine excess
242 Digestive Diseases Self-Education Program®

D. Enteric bacteria compete with the host for with a prolonged taper and pulsed vancomycin
folate, resulting in folate deficiency regimen versus fidaxomicin. The most recent ACG
guideline for the treatment of CDI recommends
CORRECT ANSWER: A treatment of the first recurrence with the same
regimen as the initial episode.
RATIONALE
Enteric bacteria compete with the host for vitamin REFERENCES
B12, resulting in a vitamin B12 deficiency. Fo- L Clifford McDonald, Dale N Gerding, Stuart
late and vitamin K levels are typically normal or Johnson, et al. ; Clinical Practice Guidelines
increased in SIBO due to synthesis by the enteric for Clostridium difficile Infection in Adults and
bacteria. Thiamine levels are not significantly af- Children: 2017 Update by the Infectious Diseases
fected in SIBO. Society of America (IDSA) and Society for Health-
care Epidemiology of America (SHEA), Clinical
REFERENCES Infectious Diseases, Volume 66, Issue 7, 19 March
Su J, Smith MB, Rerknimitr R, Morrow D. Small 2018, Pages e1–e48.
intestine bacterial overgrowth presenting as Surawicz CM, Brandt LJ, Binion DG, et al.
protein-losing enteropathy. Dig Dis Sci. 1998 Guidelines for diagnosis, treatment, and preven-
Mar;43(3):679-81. tion of Clostridium difficile infections. Am J Gas-
DiBiase, JK. Nutritional Consequences of troenterol 2013; 108:478.
Small Intestinal Bacterial Overgrowth. Practical
Gastroenterology. 2008.
Question 29
A 66-year-old woman presents for an evaluation
Question 28 of an eight month history of diarrhea. She denies
A 66-year-old man with a past medical history of weight loss, GI bleeding, or family history of IBD
uncomplicated C. difficile infection (CDI), which or colorectal neoplasia. Physical examination is
was treated with vancomycin six weeks ago, pres- normal. Colonoscopy with biopsies reveal greater
ents for evaluation of recurrent diarrhea for the than 20 intraepithelial lymphocytes per 100 sur-
past two weeks. Stool testing reveals a positive face epithelial cells with focal cryptitis.
glutamate dehydrogenase and positive toxin A/B.
What is the next best step in the management of
What is the best next step in the management of this patient?
this patient?
A. Start oral budesonide therapy
A. Start intravenous immunoglobulins (IVIG) B. Refer for video capsule endoscopy (VCE)
B. Metronidazole C. Start oral mesalamine therapy
C. Vancomycin D. Refer for bone marrow biopsy
D. Fecal microbiota transplant (FMT) E. Start rifaximin therapy

CORRECT ANSWER: C CORRECT ANSWER: A

RATIONALE RATIONALE
According to the most recently published Infec- This patient has lymphocytic colitis. The man-
tious Diseases Society of America (IDSA) guide- agement of microscopic colitis can encompass
lines, the first recurrence of CDI in a patient previ- a variety of medications from over-the-counter
ously treated with vancomycin should be treated anti-diarrheal therapies to systemic steroids. Of
Chapter 9 — Diarrhea and and constipation 243

those listed here, budesonide therapy is the next gastritis. SIBO can be diagnosed by a glucose
best step in the management of this condition. A hydrogen breath test, and thus it is the best
video capsule endoscopy would aid in the evalu- next step in the management of this patient’s
ation of possible inflammatory bowel disease or symptoms given the history of atrophic gastritis.
for the diagnostic evaluation of suspected small Chromogranin A and video capsule endoscopy
bowel bleeding, but would not be indicated in the are used in the diagnostic evaluation of sus-
clinical scenario above. Mesalamine and rifaxi- pected carcinoid syndrome and inflammatory
min are not indicated for the first-line treatment bowel disease, respectively and may be indicated
of microscopic colitis. The histopathologic find- in the subsequent evaluation of this patient’s
ings above reflect lymphocytic colitis and a bone symptoms. In addition, both of these diagnoses
marrow biopsy is not indicated. are unlikely to cause intermittent diarrhea. Elux-
adoline is an agent that combines a mu-opioid
REFERENCE receptor agonist and a delta-opioid receptor
American Gastroenterological Association Insti- antagonist and is indicated for the treatment of
tute Guideline on the Medical Management of irritable bowel syndrome – diarrhea predomi-
Microscopic Colitis. Geoffrey C. Nguyen, Walter E. nant (IBS-D). The diagnosis of IBS requires the
Smalley, Santhi Swaroop Vege, Alonso Carrasco- presence of abdominal pain, and is unlikely in an
Labra, and the Clinical Guidelines Committee. elderly patient with new onset of symptoms, thus
Gastroenterology 2016;150:242–246. this is not the diagnosis in this patient’s case.

REFERENCE
Question 30 Jan Bures, Jiri Cyrany, Darina Kohoutova, Miro-
A 66-year-old woman with history of atrophic slav Förstl, Stanislav Rejchrt, Jaroslav Kvetina,
gastritis presents for evaluation of intermittent Viktor Vorisek, and Marcela Kopacova. Small in-
diarrhea. She denies abdominal pain, weight testinal bacterial overgrowth syndrome. World J
loss, GI bleeding or a family history of colorectal Gastroenterol. 2010 Jun 28; 16(24): 2978–2990.
neoplasia or IBD. Physical exam is normal. Labs
including thyroid function testing, celiac screen
and CRP are normal. A colonoscopy with random Question 31
colon biopsies is normal. A 56-year-old woman with no significant past
medical history is admitted with diarrhea which
What is the best next step in the management of began after a course of antibiotics prescribed for
this patient? bronchitis. C. difficile toxin A/B is positive, along
with a positive glutamate dehydrogenase. Her
A. Measure chromogranin A WBC count is 16,000 cells/mm3. She is started
B. Start eluxadoline on oral vancomycin therapy. Over the next 24
C. Video capsule endoscopy hours, she develops fever to 38.8°C, decreased
D. Glucose hydrogen breath test bowel movements and abdominal distension with
decreased bowel sounds. What is the recommend-
CORRECT ANSWER: D ed treatment for this patient?

RATIONALE A. Fecal microbiota transplant (FMT)


The etiology of small intestinal bacterial over- B. Add rectal vancomycin enemas and
growth (SIBO) is complex, but may include metronidazole IV
an issue with altered antibacterial defense C. Fidaxomicin
mechanisms, such as achlorhydria from atrophic D. Intravenous immunoglobulins (IVIG)
244 Digestive Diseases Self-Education Program®

CORRECT ANSWER: B C. There is an increased risk of colon cancer


D. The median age of diagnosis is 45 years
RATIONALE E. Nonsteroidal anti-inflammatory drugs
This patient has fulminant C. difficile infection (NSAIDs) are a mainstay of treatment
(CDI), previously referred to as serious, complicat-
ed CDI. Both the recent Infectious Diseases Society CORRECT ANSWER: B
of America (IDSA) and Society for Healthcare Epi-
demiology of America (SHEA) guidelines, as well RATIONALE
as ACG guidelines recommend oral vancomycin This patient has collagenous colitis. Although it
combined with rectal vancomycin and IV metroni- can be seen in younger patients, the median age of
dazole. There is limited evidence to support FMT, diagnosis is 65. An increased risk of colon cancer
IVIG and fidaxomicin in fulminant CDI. has not been demonstrated to be present in pa-
tients with microscopic colitis. NSAIDs are a com-
REFERENCES mon causative or triggering factor in microscopic
L Clifford McDonald, Dale N Gerding, Stuart colitis. There is an association with celiac disease
Johnson, et al. ; Clinical Practice Guidelines and smokers have an increased risk of developing
for Clostridium difficile Infection in Adults and microscopic colitis.
Children: 2017 Update by the Infectious Diseases
Society of America (IDSA) and Society for Health- REFERENCE
care Epidemiology of America (SHEA), Clinical Pardi DS, Kelly CP. Microscopic colitis. Gastroen-
Infectious Diseases, Volume 66, Issue 7, 19 March terology. 2011 Apr;140(4):1155-65.
2018, Pages e1–e48.
Surawicz CM, Brandt LJ, Binion DG, et al.
Guidelines for diagnosis, treatment, and preven- Question 33
tion of Clostridium difficile infections. Am J Gas- A 73-year-old woman with a history of systemic
troenterol 2013; 108:478. sclerosis presents for an evaluation of chronic
post-prandial bloating and intermittent loose
stools. She denies significant weight loss or GI
Question 32 bleeding. CBC, CRP, celiac screen are normal and
A 65-year-old woman with a history of arthritis infectious stool studies are negative. She had a
and hyperlipidemia is referred for evaluation of colonoscopy two years ago with random colon
six months of intermittent loose stools – up to biopsies, which were normal. Glucose hydrogen
four episodes per day. She has noted occasional breath testing reveals a baseline hydrogen level
nocturnal symptoms and rare episodes of fecal in- of three ppm and a rise in hydrogen of 23 ppm
continence. A colonoscopy reveals endoscopically within 40 minutes.
normal appearing mucosa with biopsies showing
scant neutrophils and eosinophils in the lamina What is the best next step in the management of
propria along with a subepithelial collagen band this patient?
greater than10 µm in diameter.
A. Start rifaximin
Which of the following statements is true regard- B. Start rifaximin with neomycin
ing this condition? C. Repeat glucose breath testing after counseling
regarding pre-test diet and fasting
A. Smokers have a decreased risk of developing D. Repeat hydrogen breath testing with lactose
this condition
B. Celiac disease is associated with this condition CORRECT ANSWER: A
Chapter 9 — Diarrhea and and constipation 245

RATIONALE CORRECT ANSWER: E


Patients with systemic sclerosis are at risk for
small intestinal bacterial overgrowth (SIBO). This RATIONALE
patient has SIBO as evidenced by the absolute Fecal osmotic gap calculation can be useful in
increase of greater than or equal to 20 ppm hydro- the evaluation of a patient with chronic diarrhea
gen above baseline within 90 minutes. Rifaximin to differentiate between osmotic and secretory
or amoxicillin/clavulanate are acceptable single diarrhea. It is calculated by subtracting measured
agent regimens. Rifaximin along with neomycin fecal electrolytes from normal lumen osmolality
has been studied for the treatment of methane (290 – 2(fecal Na+ fecal K)). The estimated stool
predominant SIBO. A baseline hydrogen of osmolality is only used to ensure that the sample
greater than 20ppm may indicate non-adherence is not contaminated with dilute urine or water
to the test protocol for fasting and restriction of and should be close to 290 mOsm/kg. Secretory
specific complex carbohydrates, and would require diarrhea is characterized by a fecal osmotic gap of
repeat testing. In general, SIBO breath testing less than 50 mOsm/kg, whereas osmotic diarrhea
should be performed prior to lactose or fructose is characterized by a fecal osmotic gap, greater
breath testing. than 100 mOsm/kg. This patient has a secretory
diarrhea: Fecal osmolar gap = (290-2(88+44))
REFERENCES = 26. In general, secretory diarrhea is charac-
“Small Intestinal Bacterial Overgrowth” in Slei- terized as being unaffected by fasting. Sorbitol,
senger and Fordtran’s Gastrointestinal and Liver lactose, and factitious diarrhea due to magnesium
Disease. Quigley, Eamonn M.M. Published Janu- would all cause an osmotic diarrhea. In carcinoid
ary 2, 2016. Pages 1824-1831.e4. © 2016. syndrome, production of serotonin by the tumor
Hydrogen and Methane-Based Breath Test- is the most likely cause of the diarrhea. Serotonin
ing in Gastrointestinal Disorders: The North stimulates intestinal secretion and motility and
American Consensus. Am J Gastroenterol 2017; inhibits intestinal absorption.
112:775–784.
Pimentel M, Chang C, Chua KS, Mirocha J, REFERENCES
DiBaise J, Rao S, Amichai M. Antibiotic treatment of Eherer AJ, Fordtran JS. Fecal osmotic gap and
constipation-predominant irritable bowel syndrome. pH in experimental diarrhea of various causes.
Dig Dis Sci. 2014;59(6):1278. Epub 2014 May 1. Gastroenterology. 1992 Aug;103(2):545-51.
Motor dysfunction of the small bowel and
Question 34 colon in patients with the carcinoid syndrome and
A 66-year-old man presents for evaluation of diarrhea. von der Ohe MR, Camilleri M, et al. N
chronic diarrhea. Stool electrolyte testing reveals: Engl J Med. 1993;329(15):1073.
stool Na 88 mEq/L, stool K 44 mEq/L. Estimated
Stool osmolality is 290 mOsm/kg.
Question 35
Which of the following statements is true regard- A 68-year-old man with a history reflux disease
ing this type of diarrhea? and hyperlipidemia presents for evaluation of in-
termittent diarrhea for the past year, without sig-
A. It improves with fasting nificant abdominal pain. There does not appear to
B. It is associated with excess sorbitol ingestion be any relation to specific food intake. He denies
C. It is associated with lactose intolerance significant abdominal pain. Medications include
D. It is suggestive of factitious diarrhea due to simvastatin and pantoprazole. He underwent a
magnesium laxative use screening colonoscopy two years ago at an outside
E. It can be seen with carcinoid syndrome hospital, which revealed two tubular adenomas.
246 Digestive Diseases Self-Education Program®

Labs ordered by his primary care doctor reveal Labra, and the Clinical Guidelines Committee.
normal CBC, thyroid function tests, celiac screen Gastroenterology 2016;150:242–246.
and CRP. Pardi DS, Kelly CP. Microscopic colitis. Gas-
troenterology. 2011 Apr;140(4):1155-65.
What is best next step in the management of
this patient?
Question 36
A. Repeat colonoscopy with biopsies from the A 22-year-old woman is referred by her primary
right and left colon care physician for endoscopy. She has a long-
B. Start rifaximin standing history of “brain fog,” abdominal bloat-
C. Refer for lactose breath testing ing and diarrhea. One of her friends has celiac
D. Start nortriptyline disease and suggested she try a gluten-free diet
(GFD). She initiated a gluten-free diet six months
CORRECT ANSWER: A ago and reports a significant improvement in
her symptoms. Despite what she considers strict
RATIONALE adherence to a gluten-free diet, she has noted
This patient is at increased risk of microscopic a recurrence of symptoms recently. Diagnostic
colitis given his medication list. Statins and proton evaluation by her primary care doctor last week
pump inhibitors have both been associated with revealed a normal serum IgA and tissue transglu-
microscopic colitis. Endoscopic findings are typi- taminase (tTG) IgA 26 U/mL (normal less than
cally normal, but can include subtle macroscopic 19 U/mL). You perform an upper endoscopy with
findings such as edema and erythema. Biopsies adequate biopsies of the duodenal bulb and sec-
should be obtained from both the right and left ond portion of the duodenum, which are normal.
colon. Histologic findings can be patchy, with the What is the best next step?
highest diagnostic yield in the transverse colon
and right colon and least in the rectosigmoid. A. Repeat endoscopy with additional duodenal
Rifaximin and TCAs are indicated for the man- biopsies
agement of irritable bowel syndrome – diarrhea B. Check anti-gliadin antibody
predominant (IBS-D) however, the absence of C. Initiate a gluten challenge followed by repeat
significant abdominal pain makes IBS unlikely in endoscopy with biopsies
this patient. Carbohydrate malabsorption does D. Refer the patient to nutrition for non-celiac
not appear to be playing a significant role in this gluten sensitivity
patient’s symptoms by history. E. Perform colonoscopy with biopsies

REFERENCES CORRECT ANSWER: C


Carpenter HA, Tremaine WJ, Batts KP, Czaja
AJ. Sequential histologic evaluations in col- RATIONALE
lagenous colitis. Correlations with disease be- In patients with positive serology and normal or
havior and sampling strategy. Dig Dis Sci. 1992 non-diagnostic histology while maintaining a glu-
Dec;37(12):1903-9. ten-free diet (GFD), a gluten challenge and repeat
Surawicz CM. Collating collagenous colitis biopsy is required for definitive diagnosis or exclu-
cases. Am J Gastroenterol. 2000 Jan;95(1):307-8. sion of celiac disease (CD). Those with positive
American Gastroenterological Association celiac serology but a normal biopsy on a gluten
Institute Guideline on the Medical Management of containing diet have potential CD, and should be
Microscopic Colitis. Geoffrey C. Nguyen, Walter E. evaluated and monitored further depending upon
Smalley, Santhi Swaroop Vege, Alonso Carrasco- their clinical circumstances.
Chapter 9 — Diarrhea and and constipation 247

REFERENCE REFERENCES
Rubio-Tapia A, Hill ID, Kelly CP, et al. ACG Fine KD, Santa Ana CA, Fordtran JS Diagnosis of
clinical guidelines: diagnosis and management of magnesium-induced diarrhea. N Engl J Med. 1991
celiac disease. Am J Gastroenterol 2013; 108:656. Apr 11;324(15):1012-7.
Wald, A. Factitious diarrhea: Clinical manifes-
tations, diagnosis, and management. UpToDate.
Question 37
A 37-year-old nurse is self-referred for evaluation
of diarrhea. She has seen several previous gastro- Question 38
enterologists and has undergone extensive diag- A 28-year-old man with no significant medical
nostic evaluation including EGD with biopsies, CT history presents for evaluation of abdominal pain
enterography, normal hydrogen breath testing, associated with diarrhea of six months dura-
and multiple colonoscopies with biopsies, which tion. The abdominal pain improves with defeca-
have all been normal. There is no anemia, no fam- tion. He denies weight loss or overt GI bleeding.
ily history of IBD or colon cancer. Which of the Family history is negative for colorectal cancer,
following results most supports your diagnosis? but his brother has inflammatory bowel disease.
Stool studies for infections are negative. Routine
A. Stool magnesium 107 mEq/L laboratory testing reveals a normal CBC, normal
B. Stool phosphate 20 mmol/L thyroid function tests, normal serum IgA, normal
C. Stool osmolality 290 mmol/L IgA tissue transglutaminase, and a CRP 15mg/L
D. Laxative screen showing mineral oil (normal less than 3mg/L). Physical examination
is normal.
CORRECT ANSWER: A
What is the next best step in the evaluation of this
RATIONALE patient?
There are several clues in this patient’s presen-
tation pointing to possible laxative abuse and A. Glucose breath testing
factitious diarrhea. More than 90 percent of B. HLA-DQ2/DQ8
patients with factitious diarrhea are women and C. Colonoscopy
have a history of work in the healthcare field. In D. Upper endoscopy
addition, they typically present with a history of E. Video capsule endoscopy
extensive healthcare utilization. Stool sodium,
potassium, magnesium, and phosphate should be CORRECT ANSWER: C
measured in patients suspected of having facti-
tious diarrhea. Stool magnesium greater than 90 RATIONALE
mEq/L suggests magnesium-induced diarrhea. A This patient meets Rome Criteria for irritable
stool phosphate greater than 33mmol/L suggests bowel syndrome. Because of the low pretest prob-
a phosphate-induced diarrhea. An osmotic gap ability of inflammatory bowel disease and colonic
greater than 75 can be due to laxatives contain- neoplasia, routine colonoscopy is not recom-
ing magnesium, sorbitol, lactose, lactulose or mended in patients younger than 50 years of age
polyethylene glycol while an osmotic gap less with typical IBS symptoms and no alarm features.
than 75 can be seen with secretory laxatives or in Alarm features include rectal bleeding, weight
an osmotic diarrhea from a sodium containing loss, iron deficiency anemia, nocturnal symptoms,
laxative. A normal stool osmolality is estimated and a family history of selected organic diseases
to be 290 mmol/L. There is currently no laxative including colorectal cancer, inflammatory bowel
screen for mineral oil. disease (IBD), and celiac sprue. Colonoscopy
248 Digestive Diseases Self-Education Program®

should be performed in IBS patients younger than serologies can be obtained, but a negative result
50 years of age if concomitant alarm features does not preclude a diagnosis of CD. The recom-
are present, as in the case of this patient (family mended serology for diagnosis of celiac disease is
history of IBD and elevated CRP). A colonoscopy IgA tissue transglutaminase Ab. The diagnosis of
is also recommended in those over the age of 50 CD in those patients already on a GFD will require
years for the purpose of colorectal cancer screen- a gluten challenge with subsequent serology and
ing if not already done. duodenal biopsies as indicated. HLA DQ2/DQ8
genotyping should be undertaken to exclude CD
REFERENCE prior to initiating a formal gluten challenge. This
Brandt LJ, Chey WD, Foxx-Orenstein AE, Schiller test has a very good negative predictive value (99
LR, Schoenfeld PS, Spiegel BM, Talley NJ, Quigley percent), but the positive predictive value is only
EM. An evidence-based position statement on 12 percent%, due to the high prevalence of these
the management of irritable bowel syndrome. Am haplotypes in the general population. Therefore,
J Gastroenterol. 2009;104 Suppl 1:S1. a negative HLA DQ2/DQ8 rules out celiac disease,
and precludes the need for gluten challenge and
endoscopy. If HLA DQ2/DQ8 is positive, the next
Question 39 step would be a gluten challenge and endoscopy
A 22-year-old woman is referred to you for evalua- with biopsies to confirm the diagnosis of CD.
tion of chronic post-prandial bloating and inter-
mittent loose stools. She reports that her first REFERENCE
cousin had similar symptoms and was recently Rubio-Tapia A, Hill ID, Kelly CP, et al. ACG
diagnosed with celiac disease (CD). In anticipa- clinical guidelines: diagnosis and management of
tion of her visit with you, she decided to trial a celiac disease. Am J Gastroenterol 2013; 108:656.
gluten-free diet and reports significant improve-
ment in her symptoms over the past six weeks,
since she has been adhering to the diet. She states Question 40
that she will never eat gluten again, as she feels so A 26-year-old man with no significant history
much better off of it. presents for evaluation of six-month history of
abdominal bloating associated with non-bloody
Which of the following is the best next step in diarrhea. Physical exam is normal. There is no
evaluating this patient for celiac disease? contributory family history. Stool testing for
infections, CBC, celiac screen and thyroid func-
A. Refer for video capsule endoscopy tion testing are normal. A colonoscopy revealed
B. Schedule an upper endoscopy with duodenal normal findings with random colon biopsies
biopsies showing normal colonic mucosa. Stool electrolyte
C. Check anti-endomysial antibodies testing reveals:
D. Check HLA-DQ2/DQ8
E. Refer for D-xylose testing Stool osmolality (estimated): 290 mOsm/kg
Stool sodium: 44 mEq/L
CORRECT ANSWER: D Stool potassium: 31 mEq/L

RATIONALE This pattern of stool electrolyte testing may be as-


Serologic testing for the diagnosis of CD should sociated with which of the following?
be performed with patients consuming a gluten-
containing diet. In those patients who present A. Carcinoid syndrome
already on a gluten free diet (GFD), baseline B. Bile acid diarrhea
Chapter 9 — Diarrhea and and constipation 249

C. Sorbitol ingestion RATIONALE


D. V. cholerae This patient has lymphocytic colitis. Mainstay of
E. C. jejuni treatment is discontinuation of offending medica-
tions, if possible, anti-diarrheals and glucocor-
CORRECT ANSWER: C ticoids. NSAIDs, PPIs, SSRIs and statins have
all been implicated in the development of micro-
RATIONALE scopic colitis. Verapamil has not been associated
Fecal osmotic gap calculation can be useful in with microscopic colitis and has been investigated
the evaluation of a patient with chronic diarrhea as a potential treatment. Celiac disease has been
to differentiate between osmotic and secretory associated with microscopic colitis but additional
diarrhea. It is calculated by subtracting measured diagnostic evaluation for celiac disease is not
fecal electrolytes from normal lumen osmolality warranted unless the patient fails to respond to
(290 – 2(fecal Na+ fecal K)). Secretory diarrhea medical therapy.
is characterized by a fecal osmotic gap of less than
50 mOsm/kg, whereas osmotic diarrhea is char- REFERENCES
acterized by a fecal osmotic gap greater than100 Scheidler MD, Meiselman M . Use of verapamil for
mOsm/kg. This patient has an osmotic diarrhea: the symptomatic treatment of microscopic colitis.
Fecal osmolar gap = (290-2(44+31)) = 138. Of J Clin Gastroenterol. 2001 Apr;32(4):351-2.
the listed options, only sorbitol would cause an American Gastroenterological Association
osmotic diarrhea. Institute Guideline on the Medical Management of
Microscopic Colitis. Geoffrey C. Nguyen, Walter E.
REFERENCE Smalley, Santhi Swaroop Vege, Alonso Carrasco-
Eherer AJ, Fordtran JS. Fecal osmotic gap and Labra, and the Clinical Guidelines Committee.
pH in experimental diarrhea of various causes. Gastroenterology 2016;150:242–246.
Gastroenterology. 1992 Aug;103(2):545-51. Pardi DS, Kelly CP. Microscopic colitis. Gas-
troenterology. 2011 Apr;140(4):1155-65.

Question 41
A 76-year-old woman with a past medical his- Question 42
tory of hypertension and gastroesophageal reflux A 65-year-old man with a past medical history of
disease is referred for evaluation of intermittent short-segment Barrett’s esophagus on once daily
diarrhea over the past six months. Medications proton pump inhibitor (PPI) and an episode of
include verapamil and lansoprazole. Colonoscopy non-severe C. difficile infection (CDI) three months
revealed normal appearing mucosa with biopsies ago after a course of antibiotics given for bronchitis
showing mild cryptitis with greater than 100 in- is due to undergo routine dental work for which his
traepithelial lymphocytes. dentist has prescribed prophylactic antibiotics.

What is the best next step in the management of Which of the following is recommended for the
this patient? prevention of CDI?

A. Discontinue verapamil A. Discontinue PPI


B. Start budesonide B. Start Saccharomyces boulardii
C. Upper endoscopy with duodenal biopsies C. Discontinue prophylactic antibiotics
D. Check antigliadin antibodies D. Start oral vancomycin

CORRECT ANSWER: B CORRECT ANSWER: C


250 Digestive Diseases Self-Education Program®

RATIONALE tissue transglutaminase (tTG) is 90 U/mL (normal


This patient does not have any high-risk features less than 19 U/mL).
that place him at risk for infective endocarditis
from routine dental work and therefore prophylac- What is the best next step in the management of
tic antibiotics are not indicated. Antibiotic stew- this patient?
ardship is a mainstay of prevention for C. difficile
infections. Although PPI have been associated A. Repeat upper endoscopy with duodenal biopsies
with CDI, there is insufficient evidence to support B. Re-refer to the nutritionist
discontinuing necessary PPIs as a measure to pre- C. Start oral prednisolone with azathioprine
vent CDI. Currently, there is no role for probiotics D. Colonoscopy with random colon biopsies
in the primary prevention of CDAD, prevention
of recurrent CDAD or as adjunctive treatment. CORRECT ANSWER: B
In addition, there is limited evidence of support
extending the length of anti-C. difficile treat- RATIONALE
ment beyond the recommend treatment course or Nonresponsive celiac disease (NRCD) is defined as
restarting an anti-C. difficile agent empirically for a lack of response to six to 12 months on a gluten
patients who require treatment with antibiotics free diet (GFD), or recurrence of celiac-related
shortly after completion of CDI treatment. features despite compliance. Clinical, serologic
and histologic improvement on a GFD occurs at
REFERENCES different rates, and serologic normalization can
L Clifford McDonald, Dale N Gerding, Stuart take up to two years. In this patient, it is important
Johnson, et al.; Clinical Practice Guidelines to confirm adherence to a GFD, as the most com-
for Clostridium difficile Infection in Adults and mon cause of nonresponsive celiac is inadvertent
Children: 2017 Update by the Infectious Diseases gluten ingestion. It is premature to repeat upper
Society of America (IDSA) and Society for Health- endoscopy with duodenal biopsies in this patient,
care Epidemiology of America (SHEA), Clinical as histologic normalization is unlikely before two
Infectious Diseases, Volume 66, Issue 7, 19 March years on a gluten free diet. In fact, in a study evalu-
2018, Pages e1–e48. ating mucosal recovery on gluten free diet, only 34
Surawicz CM, Brandt LJ, Binion DG, et al. percent were healed at two years, 66 percent at five
Guidelines for diagnosis, treatment, and preven- years, and 90 percent at nine years. Oral predniso-
tion of Clostridium difficile infections. Am J Gas- lone with azathioprine is indicated for refractory
troenterol 2013; 108:478. celiac disease. Microscopic colitis is quite common
in patients with celiac disease, but given a persis-
tently elevated TTG, it is more important to ensure
Question 43 the celiac disease is being controlled prior to look-
A 23-year-old woman with no significant past ing for other causes of symptoms.
medical history presents with one year of abdomi-
nal bloating and intermittent diarrhea. Labora- REFERENCES
tory evaluation reveals a normal serum IgA with Oxentenko AS, Murray JA. Celiac Disease:
tissue transglutaminase (tTG) of 166 U/mL (nor- Ten Things That Every Gastroenterologist
mal less than 19 U/mL). An upper endoscopy with Should Know. Clin Gastroenterol Hepatol. 2015
duodenal bulb and post-bulbar biopsies confirm Aug;13(8):1396-404; Epub 2014 Jul 19.
the diagnosis of celiac disease. She is referred to Rubio-Tapia A, Hill ID, Kelly CP, et al. ACG
a nutritionist and starts a gluten free diet. She re- clinical guidelines: diagnosis and management
turns for follow up 12 months later and states her of celiac disease. Am J Gastroenterol 2013;
symptoms remain largely unchanged. A repeat 108:656.
Chapter 9 — Diarrhea and and constipation 251

Rubio-Tapia A, Rahim MW, See JA, et al. Mu- Bharucha AE, Pemberton JH, Locke GR 3rd.
cosal recovery and mortality in adults with celiac American Gastroenterological Association techni-
disease after treatment with a gluten-free diet. Am cal review on constipation. Gastroenterology 2013;
J Gastroenterol 2010;105:1412–1420. 144:218.

Question 44 Question 45
A 37-year-old woman with no significant past medi- A 36-year-old man on chronic methadone main-
cal history presents for further evaluation of chronic tenance therapy reports worsening constipation
constipation with straining. She denies significant characterized by significant straining, less than
abdominal pain, gastrointestinal bleeding or weight three spontaneous bowel movements per week,
loss. There is no family history of colorectal neo- and sensation of incomplete evacuation after re-
plasia or inflammatory bowel disease. She has not cent methadone dose escalation.
responded to many laxatives and undergoes anorec-
tal manometry with balloon expulsion testing. Which of the following is the best next step in the
management of this patient?
Which of the following results suggests a diagnosis
of dyssynergic defecation? A. Lubiprostone
B. Naloxegol
A. Absent rectoanal inhibitory reflex (RAIR) C. Osmotic laxatives
B. Expulsion of water-filled balloon in three D. Methylnaltrexone
minutes
C. Defecation index of 2.0 CORRECT ANSWER: C
D. Decreased anal sphincter pressure during
simulated defecation RATIONALE
This patient meets Rome IV criteria for opioid
CORRECT ANSWER: B induced constipation (OIC). According to the
most recent published guidelines for the medical
RATIONALE management of opioid induced constipation, first-
The balloon expulsion test is highly suggestive of line agents should be traditional laxatives. There
dyssynergia. A balloon expulsion time of greater is insufficient evidence to recommend the use of
than two minutes is abnormal. An absent RAIR lubiprostone for OIC. If traditional laxatives are
can be seen in Hirschsprung’s disease or megarec- ineffective, current guidelines recommend the
tum. Defecation index = maximum rectal pres- use of Peripherally-Acting Mu-Opioid Receptor
sure during attempted defecation/minimum anal Antagonists (PAMORAs) such as naloxegol and
residual pressure during attempted defecation. A methylnaltrexone over no treatment.
normal defecation index is greater than 1.5. De-
creased anal sphincter pressure during simulated REFERENCES
defecation is normal and therefore not consistent Spierings ELH, Drossman DA, Cryer B, Mazen
with dyssynergic defecation. Jamal M, Losch-Beridon T, Mareya SM, Wang M.
Efficacy and Safety of Lubiprostone in Patients
REFERENCES with Opioid-Induced Constipation: Phase 3 Study
Wald A, Bharucha AE, Cosman BC, Whitehead Results and Pooled Analysis of the Effect of Con-
WE. ACG clinical guideline: management of comitant Methadone Use on Clinical Outcomes.
benign anorectal disorders. Am J Gastroenterol. Pain Med. 2018 Jun 1;19(6):1184-1194.
2014 Aug;109(8):1141-57. Seth D. Crockett, Katarina B. Greer, Joel J.
252 Digestive Diseases Self-Education Program®

Heidelbaugh, et al. on behalf of American Gastro- rience is limited. Pelvic floor physical therapy
enterological Association Institute Clinical Guide- would be appropriate if defecography confirms
lines Committee. American Gastroenterological impaired rectal evacuation. A colon transit study
Association Institute Guideline on the Medical is the appropriate next step once defecation disor-
Management of Opioid-Induced Constipation ders are ruled out or if symptoms persist despite
Gastroenterology 2018. normalization of defecation disorders. A RAIR is
present, thus a rectal suction biopsy to evaluate
for Hirschsprung’s disease is not indicated.
Question 46
A 35-year-old woman with no significant history REFERENCES
presents for worsening chronic constipation. She Wald A, Bharucha AE, Cosman BC, Whitehead
endorses a sense of incomplete evacuation, signifi- WE. ACG clinical guideline: management of
cant straining with bowel movements, and occa- benign anorectal disorders. Am J Gastroenterol.
sional need to apply perineal pressure. Abdominal 2014 Aug;109(8):1141-57
pain and bloating are occasionally present but are Bharucha AE, Pemberton JH, Locke GR 3rd.
not the predominant symptoms. She denies rectal American Gastroenterological Association techni-
bleeding, unintentional weight loss and there is cal review on constipation. Gastroenterology 2013;
no family history of colonic neoplasia or IBD. She 144:218.
had previously been maintained on daily PEG but Maria G, Cadeddu F, Brandara F, Marniga
has recently found it to be inconsistently effective. G, Brisinda G. Experience with type A botulinum
On rectal exam, soft stool is noted in the rectal toxin for treatment of outlet-type constipation.
vault along with a high resting sphincter pressure Am J Gastroenterol. 2006;101(11):2570. Epub
and suggestion of incomplete anal contraction 2006 Oct 4.
with simulated defecation. You refer her for ano-
rectal manometry, which confirms a dyssynergic
pattern with adequate rectal propulsive forces and Question 47
paradoxical anal contraction. A rectoanal inhibi- A 34-year-old woman with no significant past
tory reflex (RAIR) is present. She is able to pass medical history presents with an eight month his-
a water-filled balloon in 30 seconds. What is the tory of abdominal pain/bloating associated with
next best step in the management of this patient? altered bowel habits, namely worsening consti-
pation characterized by straining and sense of
A. Anal sphincter botulinum toxin injection incomplete evacuation. Daily polyethylene glycol
B. Defecography (PEG) therapy has been ineffective. She denies
C. Pelvic floor physical therapy rectal bleeding or family history of colorectal
D. Colon transit study neoplasia or IBD. Labs including CBC and thyroid
E. Rectal suction biopsy function panel are normal. Rectal examination
reveals no masses, soft stool in the rectal vault,
CORRECT ANSWER: B no overt paradoxical anal contraction, but limited
perineal descent with simulated defecation.
RATIONALE
In situations when the anorectal manometry Which of the following is the most appropriate
and balloon expulsion testing are discordant or next step in the management of this patient?
inconclusive, a defecography is the next step in
the evaluation of patients. While there have been A. Colonic transit testing
small studies evaluating the utility of botulinum B. Anorectal manometry with balloon
toxin injection in obstructive defecation, expe- expulsion testing
Chapter 9 — Diarrhea and and constipation 253

C. Colonoscopy previously without significant benefit. What is the


D. Defecography best next step in the management of this patient?

CORRECT ANSWER: B A. Anorectal manometry with balloon expulsion


B. Defecography
RATIONALE C. Wireless motility capsule
This patient meets Rome IV criteria for IBS-C. D. Pelvic floor physical therapy with biofeedback
The diagnosis of IBS-C does not preclude a con- E. Refer for surgical consultation
comitant defecation disorder. According to the
most recent published guidelines, anorectal ma- CORRECT ANSWER: A
nometry with balloon expulsion testing is the most
appropriate next step in the evaluation of this RATIONALE
patient. Colon transit testing is indicated if ano- Up to 50 percent of patients with defecatory
rectal manometry with balloon expulsion testing disorders also have slow colonic transit and slow
is normal. Defecography can be undertaken if the colonic transit does not exclude a defecatory disor-
results of the anorectal manometry with balloon der. Coexisting slow colonic transit does not alter
expulsion testing is inconclusive. Clinical practice the management of defecatory disorders. In this
may differ due to availability of various diagnostic patient, the rectal exam is suggestive of a defecato-
tests, however, when both are available, current ry disorder and an anorectal manometry with bal-
published guidelines recommend against perform- loon expulsion testing is the appropriate next step.
ing a defecography before anorectal manometry A defecography may be indicated if the results of
with balloon expulsion testing. anorectal manometry with balloon expulsion are
inconclusive. Wireless motility capsule is used to
REFERENCES measure colon transit times, which has already
Wald A, Bharucha AE, Cosman BC, Whitehead been done in this patient. Pelvic floor physical
WE. ACG clinical guideline: management of therapy with biofeedback would be the next step
benign anorectal disorders. Am J Gastroenterol. once a defecatory disorder has been confirmed.
2014 Aug;109(8):1141-57. It is premature to refer this patient for surgical
Bharucha AE, Pemberton JH, Locke GR 3rd. evaluation for her symptoms.
American Gastroenterological Association techni-
cal review on constipation. Gastroenterology 2013; REFERENCES
144:218. Wald A, Bharucha AE, Cosman BC, Whitehead
WE. ACG clinical guideline: management of
benign anorectal disorders. Am J Gastroenterol.
Question 48 2014 Aug;109(8):1141-57.
A 26 year-old woman with a history of constipa- Bharucha AE, Pemberton JH, Locke GR 3rd.
tion recently moved to the area and presents to American Gastroenterological Association techni-
establish care. She endorses infrequent urge to cal review on constipation. Gastroenterology 2013;
defecate. When she does defecate, she reports 144:218.
significant straining and sense of incomplete
evacuation. A rectal examination reveals stool in
the rectal vault along with high resting sphinc- Question 49
ter tone with paradoxical anal contraction with A 56-year-old man presents for evaluation of func-
simulated defecation. A radiopaque marker study tional constipation not responsive to fiber supple-
revealed eight retained markers on day five. She mentation and daily polyethylene glycol use. A
has tried polyethylene glycol and linaclotide rectal examination reveals a high resting pressure
254 Digestive Diseases Self-Education Program®

and suggestion of inadequate anal relaxation with Question 50


simulated defecation. Anorectal manometry re- A 23-year-old woman with no significant past
veals adequate rectal propulsive forces along with medical history presentsfor evaluation of a six
adequate anal relaxation with simulated defeca- month history of abdominal pain associated with
tion. The patient is able to pass a water-filled a change in her bowel habits. She reports infre-
balloon in 30 seconds. quent bowel movements and passage of pellet-
like stools along with a sensation of incomplete
What is the best next step in the evaluation of this evacuation. She denies weight loss or GI bleeding.
patient’s symptoms? There is no family history of inflammatory bowel
disease or colorectal cancer. Recent blood work
A. Colon transit testing reveals a normal CBC and thyroid function panel.
B. Gastric emptying scan A digital rectal examination reveals a smooth
C. Defecography anal canal without masses or fissure and normal
D. Colonic manometry resting tone without overt paradoxical contraction
with simulated defecation.
CORRECT ANSWER: A
What is the best next step in the management of
RATIONALE this patient?
This patient has functional constipation
without concomitant defecatory disorder. Ac- A. Anorectal manometry with balloon expulsion
cording to the AGA medical position paper on testing
constipation, the next step in the evaluation of B. Colonoscopy
this patient is colon transit testing. According C. Start psyllium
to the current guidelines, a gastric emptying D. Start linaclotide
scan may be necessary in those patients with E. Start amitriptyline
delayed colonic transit despite maximal medi-
cal therapy. This patient has a normal anorec- CORRECT ANSWER: C
tal manometry with balloon expulsion test and
therefore, a defecography is not indicated. In RATIONALE
the current published guidelines, colonic ma- This patient meets Rome IV criteria for Irritable
nometry is reserved for those patients who do bowel syndrome – constipation predominant
not respond to standard medical therapy with (IBS-C). Dietary fiber supplementation remains a
normal gastric emptying. cornerstone of IBS management. Recent meta-
analysis suggests the benefits of fiber in IBS are
REFERENCES confined to soluble (psyllium) and not insoluble
Wald A, Bharucha AE, Cosman BC, Whitehead fiber (wheat bran). An anorectal manometry is
WE. ACG clinical guideline: management of indicated if this patient fails to improve with fiber
benign anorectal disorders. Am J Gastroenterol. and/or first-line laxative therapies. There are no
2014 Aug;109(8):1141-57. alarm features in this patient’s presentation to
Bharucha AE, Pemberton JH, Locke GR 3rd. warrant a colonoscopy. Linaclotide is an intestinal
American Gastroenterological Association techni- secretagogue used for IBS-C and chronic idiopath-
cal review on constipation. Gastroenterology 2013; ic constipation, but it is premature to recommend
144:218. this therapy prior to trying fiber and other over the
counter laxatives such as polyethylene glycol. Ami-
triptyline is a tricyclic antidepressant which has
been shown to be beneficial in IBS; however, it can
Chapter 9 — Diarrhea and and constipation 255

slow transit due to its anticholinergic side effects, RATIONALE


and would not be an ideal therapy in this patient. This woman has a new diagnosis of collagenous
colitis, one of the two types of microscopic coli-
REFERENCES tis (the other being lymphocytic colitis), which
Bharucha AE, Pemberton JH, Locke GR 3rd. is more prevalent in women than men, particu-
American Gastroenterological Association techni- larly over the age of 50 years. Clinical features
cal review on constipation. Gastroenterology 2013; of microscopic colitis can be difficult to distin-
144:218. guish from diarrhea predominant celiac disease;
Ford AC, Moayyedi P, Lacy BE, et al. Ameri- therefore, tissue sampling of the endoscopically
can College of Gastroenterology monograph on normal appearing colon is necessary to establish
the management of irritable bowel syndrome and the diagnosis. Microscopic colitis can be associ-
chronic idiopathic constipation. Am J Gastroen- ated with other autoimmune conditions such as
terol 2014;109(Suppl 1):S2–S26. autoimmune thyroiditis, psoriasis and, most com-
Ford AC, Lacy BE, Talley NJ. Irritable monly, celiac disease. Several classes of medica-
Bowel Syndrome. N Engl J Med. 2017 Jun tions are associated with microscopic colitis – the
29;376(26):2566-2578. most evidence is with NSAIDs, but proton pump
inhibitors, selective serotonin reuptake inhibitors,
H2 blockers and statins have also been linked with
Question 51 microscopic colitis. Treatment includes medi-
A 54-year-old woman with a longstanding his- cation discontinuation, anti-diarrheals such as
tory of diarrhea-predominant irritable bowel loperamide, and budesonide for more persistent
syndrome presents to the office with a six-month symptoms.
history of worsening watery diarrhea. Associated
symptoms include abdominal cramping, bloating REFERENCES
and urgency with several episodes of fecal in- Pardi and Kelly. Microscopic Colitis. Gastroenter-
continence. Her labs testing reveals no anemia, ology. 2011;140(4):1155-65.
normal inflammatory markers, normal serum IgA Nguyen et al. American Gastroenterological
levels, negative tissue transglutaminase antibod- Association Institute Guideline on the Medical
ies, and stool studies negative for infection. Duo- Management of Microscopic Colitis. Gastroenter-
denal biopsies are unremarkable with preserved ology. 2016;150(1):242-46.
villous pattern and no increased intraepithelial
lymphocytes. Her terminal ileum and colon were
endoscopically normal, but random colon biop-
sies showed cryptitis and a thickened subepithe-
lial collagen band.

Which of the following medications have been as-


sociated with her diagnosis?

A. Clindamycin
B. Sucralfate
C. Sertraline
D. Amitriptyline
E. Sumatriptan

CORRECT ANSWER: C
256 Digestive Diseases Self-Education Program®
Answers & critiques

CHAPTER 10

Gastrointestinal motility
Walter W. Chan, MD, MPH, Ronnie Fass, MD, FACG, Sushila R. Dalal, MD
and Christina Ha, MD, FACG, AGAF

Question 1 Botulinum toxin injection to the pylorus (answer


A 25-year-old female presents to GI clinic re- a) and promotility agents [domperidone (answer
porting post-prandial vomiting. She describes b) and erythromycin (answer d)] are treatment
onset of belching and vomiting about 15 to 20 options of gastroparesis. Cannabinoid hyper-
minutes after each meal of undigested foods, emesis syndrome (answer e) is associated with
which she would sometimes re-swallow back symptoms similar to cyclic vomiting syndrome,
down. She denies nausea, retching, abdominal characterized by recurrent nausea and vomiting
pain, or hematemesis. An upper endoscopy per- episodes, followed by asymptomatic periods in a
formed was unremarkable except mild esophagi- cyclical pattern.
tis and gastritis.
REFERENCE
What would you recommend as the next step of Absah I et al. Rumination syndrome: pathophys-
management? iology, diagnosis, and treatment. Neurogastro-
enterol Motil 2017 Apr;29(4).
A. Botulinum toxin injection of the pylorus
B. Domperidone
C. Diaphragmatic breathing Question 2
D. Erythromycin A 75-year-old man with a history of emphy-
E. Urine toxicology screen for cannabinoid sema and tobacco use currently on three liters
of home oxygen presents with dysphagia in the
CORRECT ANSWER: C last three months. He describes a sensation
of ingested food getting stuck in his mid-chest
RATIONALE region when swallowing, causing him to often
This patient demonstrates clinical symptoms regurgitate the food items. He has lost 30 lbs
suggestive of rumination syndrome, with re- since his symptom onset. Evaluation with an
gurgitation of ingested food within 30 minutes upper endoscopy was otherwise normal. An
without nausea or retching (i.e. “effortless”), esophageal manometry performed shown on the
sometimes requiring her to re-swallow the following page revealed:
ingested food. Diaphragmatic breathing is the
most effective therapy for patients with rumina- What should be the next step in management?
tion syndrome, which is a behavioral condition
associated with increased intragastric pressure A. Referral for POEM
followed by retrograde flow of gastric content B. Botulinum toxin injection to the lower
up the esophagus, as demonstrated on post- esophageal sphincter.
prandial esophageal impedance-manometry. C. Pneumatic dilation

257
258 Digestive Diseases Self-Education Program®

D. Schedule chest CT. Question 3


E. Start calcium channel blocker. A 40-year-old female with no significant past
medical history presents for evaluation of chronic
CORRECT ANSWER: D constipation. She has tried multiple different
laxatives without improvement. She describes ab-
RATIONALE dominal bloating, passage of only a small amount
The esophageal manometry shows no relaxation of stools with each bowel movement, and a sensa-
of the lower esophageal sphincter with swallow- tion of incomplete evacuation. A colonoscopy
ing and aperistaltic esophageal body contractions, performed was unremarkable. She completed
consistent with achalasia. Given his new-onset an anorectal manometry, which revealed a high
symptoms at advanced age, short-duration of resting anal sphincter pressure (90 mmHg), high
symptoms, significant weight loss, and history of squeeze pressure (225 mmHg), and no change in
tobacco use, pseudoachalasia secondary to a pri- the anal sphincter pressure during strain or bear-
mary malignancy should be suspected and ruled- down maneuver. There was normal relaxation
out. Malignancy-related pseudoachalasia may be of the anal sphincter with balloon distention in
secondary to local obstruction from an esophago- the rectum. The rectal sensation testing showed
gastric junction tumor or an immune-mediated a slightly elevated first sensation pressure (50
paraneoplastic process from a more distant can- mmHg). She could not expel the 50 mL balloon in
cer. Small cell lung cancer is the most common two minutes.
primary malignancy associated with this paraneo-
plastic process, which is caused by cross-reaction Which of the following is the most important diag-
between the neural circuit of the esophagus and nostic information for obstructive defecation?
antibodies (e.g. antineuronal antibody and anti-
Hu antibody) to the tumor antigens. Therefore, A. Resting anal sphincter pressure of 90 mmHg.
cross-sectional imaging should be performed next B. Relaxation of anal sphincter pressure with
in this patient to rule out a primary malignancy. balloon distention in the rectum.
C. Failed expulsion of 50 mL balloon in
REFERENCE two minutes.
Ponds FA et al. Diagnostic features of malignancy- D. First sensation pressure of 50 mmHg.
associated pseudoachalasia. Aliment Pharmacol E. Anal sphincter squeeze pressure of
Ther. 2017 Jun;45(11):1449-1458. 225 mmHg.
Chapter 10 — Gastrointestinal motility 259

CORRECT ANSWER: C CORRECT ANSWER: E

RATIONALE RATIONALE
Failed balloon expulsion test (answer c) has been This patient’s esophageal manometry showed an
found to correlate with an evacuation disorder increase in ineffective swallows (40 percent weak
and predict biofeedback therapy response. Anal [DCI 100 to 450 mmHg*s*cm] and 20 percent
sphincter tone (answer a) and squeeze pres- failed [DCI greater than 100 mmHg*s*cm] swal-
sure (answer e) are most useful in the evalua- lows) with otherwise normal findings. This pat-
tion of fecal incontinence, but play less of a role tern is suggestive of ineffective esophageal motility
in assessment of obstructive defecation. Rectal (IEM) per Chicago classification v3.0. IEM is most
sensation testing (answer d) may be abnormal in commonly associated with gastroesophageal reflux
constipation, with rectal hyposensitivity causing a (answer e). Diffuse esophageal spasm (answer
decrease in urge for defecation. However, it alone a) is characterized by increased spastic contrac-
is not associated with obstructive defecation. tions, defined by a short DL (greater than 4.5 sec).
Recto-anal inhibitory reflex (RAIR) (answer b) is Esophagogastric junction outflow obstruction,
used as an assessment of normal colonic inner- defined by a high IRP, is often associated with
vation, and would be abnormal in patients with tumor at the esophagogastric junction (answer
Hirschsprung’s disease. b) and chronic opioid use (answer c). Jackham-
mer esophagus (answer d) is characterized by an
REFERENCE increase in hypercontractile swallows (DCI greater
Carrington EV et al. Expert consensus document: than 8000 mmHg*s*cm).
Advances in the evaluation of anorectal func-
tion. Nat Rev Gastroenterol Hepatol 2018 May; REFERENCE
15(5):309-323. Kahrilas PJ et al. The Chicago Classification of
Esophageal Motility Disorders v3.0. Neurogastro-
enteral Motil 2015;27(2):160-174.
Question 4
A 34-year-old otherwise healthy man presents for
pre-operative evaluation for fundoplication. His Question 5
esophageal manometry revealed a basal lower A 28-year-old woman underwent anorectal
esophageal pressure of 20 mmHg and an inte- manometry for chronic constipation refractory
grated relaxation pressure (IRP) of 5 mmHg. The to multiple laxatives. The anorectal manometry
distal latency (DL) was noted to be greater than showed an increase in resting anal sphincter pres-
4.5 sec in all 10 test swallows. The distal contrac- sure (83 mmHg), normal squeeze pressure (180
tile integral (DCI) ranged 500-1000 mmHg*s*cm mmHg), normal rectal contraction pressure (50
in four swallows, 200-300 mmHg*s*cm in two mmHg), increased residual pressure during simu-
swallows, and 30-60 mmHg*s*cm in four swal- lated defecation (85 mmHg), and no relaxation of
lows. The peristaltic break was less than 5 cm in the anal sphincter during balloon distention. The
all swallows. Which of the following is the most sensation testing showed a mildly elevated first
likely underlying cause of these findings? sensation pressure (50 mmHg). She could not
expel the 50 mL balloon in two minutes.
A. Diffuse esophageal spasm
B. Tumor at the esophagogastric junction What should be the next step in management?
C. Chronic opioid use
D. Jackhammer esophagus A. Referral for biofeedback therapy.
E. Gastroesophageal reflux disease B. Botox injection to anal sphincter.
260 Digestive Diseases Self-Education Program®

C. Referral for sphincterotomy. stools, or weight loss. Extensive evaluations, in-


D. Deep suction rectal biopsies. cluding endoscopies, imaging (ultrasound and CT),
E. Schedule MR defecography. stool studies, Celiac serologies, and inflammatory
markers, were all unremarkable.
CORRECT ANSWER: D
Which of the following is the most appropriate
RATIONALE treatment option for this patient’s condition?
The anorectal manometry showed an absent
rectoanal inhibitory reflex (RAIR), defined by A. Hyoscyamine
the lack of anal sphincter relaxation with balloon B. Linaclotide
distention in the rectum. A normal RAIR depends C. Lubiprostone
on a functioning neural circuitry in the rectum D. Eluxadoline
and anal sphincters. An absent RAIR, therefore, E. Amitriptyline
should raise suspicion for Hirschsprung’s dis-
ease, characterized by agangliosis in the affected CORRECT ANSWER: B
segment of the distal colon. While other findings
on this patient’s anorectal manometry may also RATIONALE
suggest a spastic anal sphincter (increased resting This patient’s symptoms and clinical presenta-
tone) or dyssynergic defecation (lack of sphincter tion are suggestive of constipation-predominant
relaxation with straining or bearing down), confir- irritable bowel syndrome (IBS-C), given the chronic
matory testing for Hirschsprung’s disease should constipation and abdominal pain, lack of alarm
first be performed in the absence of normal RAIR. symptoms, and extensive negative evaluations.
Deep suction rectal biopsies would show absent Linaclotide (answer b) and lubiprostone (answer c)
ganglion cells and hypertrophy of nerve fibers in are both approved for treatment of IBS-C. How-
patients with Hirschsprung’s disease. ever, nausea is the major side effect associated with
lubiprostone. Therefore, it may not be the best op-
REFERENCE tion for this patient who already reports daily nau-
Kahn, E. and F. Daum, Anatomy, histology, sea. Hyoscyamine (answer a), an anti-spasmodic
embryology, and developmental anomalies of agent, and amitriptyline (answer e), a tricyclic
the small and large intestine, in Sleisenger and antidepressant and neuromodulator, both carry an-
Fordtran’s Gastrointestinal and Liver Disease: ticholinergic effects that may worsen constipation.
Pathophysiology, Diagnosis, Management, M. Eluxadoline (answer d) is a mu- and kappa-opioid
Feldman, L.S. Friedman, and L.J. Brandt, Editors. receptor agonist and delta-opioid receptor antago-
2010, Saunders: Philadelphia, PA. p. 1615-42. nist used for treatment of diarrhea-predominant
IBS, and would not be appropriate for IBS-C.

Question 6 REFERENCE
A 45-year-old woman with a history of cholecystec- Ford, A.C., et al., American College of Gastro-
tomy presents for evaluation of chronic abdominal enterology monograph on the management of
pain, bloating, and constipation since her teenage irritable bowel syndrome and chronic idiopathic
years. She reports bowel movements every three constipation. Am J Gastroenterol, 2014;109 Suppl
to five days, with no hematochezia or melena. Her 1: p. S2-26; quiz S27.
abdominal pain and bloating are worse when she is
constipated and decrease with bowel movements.
She reports some nausea daily, but no vomiting. Question 7
She denies any fever, nocturnal symptoms, greasy A 42-year-old woman was hospitalized for nau-
Chapter 10 — Gastrointestinal motility 261

sea, vomiting, abdominal distention, and inability obstruciton in children and adults: diagnosis and
to tolerate her oral diet. Abdominal CT revealed therapeutic options. Neurogastroenterol Motil
diffuse small intestinal dilatation without a focal 2017 Jan;29(1).
point of obstruction. She has had several simi-
lar presentations and CT findings in the past.
Evaluation with an antroduodenal manometry
Question 8
revealed fasting phase III migrating motor com-
A 62-year-old male with dysphagia presents for
plex (MMC) with a frequency of ~12/minute and
esophageal manometry for evaluation of his swal-
average contraction amplitude of 3-5 mmHg in
lowing symptoms. He
the duodenum.
previously completed an
upper endoscopy and
Which of the following is the most likely etiology
a barium esophagram
of her clinical presentation and study findings?
for assessment of his
symptoms. The upper
A. Diabetes mellitus.
endoscopy was noted to
B. Systemic sclerosis.
be normal. The barium
C. Carcinoid tumor.
esophagram revealed the
D. Neurofibromatosis.
following findings:
E. Multiple sclerosis.

Which of the following


CORRECT ANSWER: B
would most likely be
seen on his esophageal
RATIONALE
manometry?
This patient presents with recurrent episodes
of significant intestinal dilatation with no fo-
A. Elevated integrated residual pressure (IRP)
cal obstruction, suggestive of chronic intestinal
with pan-esophageal pressurization greater
pseudo-obstruction (CIPO). Underlying etiologies
than 30 mmHg in all swallows.
of CIPO can generally be classified into neuro-
B. Normal IRP with distal contractile integral
pathic, myopathic, or mixed. Her antroduodenal
(DCI) between 30 and 50 mmHg*s*cm in all
manometry revealed phase III MMC with signifi-
swallows.
cantly reduced contraction amplitude, but normal
C. Decreased distal latency (two to three sec) in
frequency during fasting state. This would suggest
40 percent of swallows.
a myopathic, but not neuropathic, process lead-
D. Elevated IRP with normal DCI and distal
ing to her pseudo-obstruction. Diabetes mellitus,
latency in all swallows.
paraneoplastic syndrome (carcinoid tumor), neu-
E. 80 percent of swallows with DCI between 200
rofibromatosis, and multiple sclerosis (answers a,
and 300 mmHg*s*cm.
c, d, e) all lead to a primary neuropathic process
that causes CIPO. While systemic sclerosis (an-
CORRECT ANSWER: C
swer b) may sometimes demonstrate mixed fea-
tures of both myopathic and neuropathic changes,
RATIONALE
its main pathologic process involves fibrosis or
The esophagram of this patient presenting with
atrophy of the underlying smooth muscles leading
dysphagia revealed “corkscrew” esophagus, a
to decreased vigor of contraction.
finding caused by spastic, simultaneous contrac-
tions of the esophagus. It is characteristically
REFERENCE
associated with diffuse esophageal spasm (answer
Di Nardo G et al. Chronic intestinal pseudo-
c), which is defined by an increased proportion
262 Digestive Diseases Self-Education Program®

of swallows (greater than 20 percent) with a What would be the most appropriately treatment
decreased distal latency (greater than 4.5 sec). for her condition?
Increased IRP with pan-esophageal pressuriza-
tion (answer a) is the characteristic finding of A. Pneumatic dilation to 30 mm.
type II achalasia, which would often demonstrate B. Heller myotomy with Dor fundoplication
a dilated esophagus with “bird’s beak” appear- C. POEM
ance on esophagram. A normal IRP with failed D. Botulinum toxin injection into the lower
contractions in all swallows (answer b) suggests esophageal sphincter
absent contractility per Chicago classification E. Heller myotomy with Nissen fundoplication.
v3.0, while an elevated IRP with normal DCI and
distal latency (answer d) indicates esophagogastric CORRECT ANSWER: D
junction outflow obstruction (EGJOO). An in-
crease in weak swallows (DCI between 100 to 450 RATIONALE
mmHg*s*cm) would suggest ineffective esopha- The patient’s presentation and esophageal ma-
geal motility (answer e). nometry are consistent with type II achalasia.
While Heller myotomy, per oral esophageal my-
REFERENCE otomy (POEM), and pneumatic dilation are effec-
Kahrilas PJ et al. The Chicago Classification of tive first-line therapies for achalasia, this patient’s
Esophageal Motility Disorders v3.0. Neurogastro- significant medical comorbidities (coronary artery
enteral Motil 2015;27(2):160-174. disease, congestive heart failure, and advanced
lung disease requiring home oxygen) would make
her a poor candidate. As pneumatic dilation is
Question 9 associated with a perforation risk of around five
A 68-year-old woman with a history of coronary percent, it would not be an appropriate treatment
artery disease, congestive heart failure, and COPD for patients who are poor surgical candidates. In-
on three liters of oxygen at home presents with jection of botulinum toxin to the lower esophageal
progressive dysphagia over the last five years. She sphincter is relatively safe and has a high short-
describes difficulty swallowing both solids and term response rate. However, its treatment effect
liquids, often needing to regurgitate them back deteriorates within six to 12 months of therapy, of-
up. She underwent an upper endoscopy that was ten requiring repeat injection. Therefore, it is gen-
unremarkable, including biopsies obtained. An erally reserved for patients who are not candidates
esophageal manometry performed revealed the for the more durable invasive options (myotomy
following findings shown below: or pneumatic dilation). The first-step of deter-
Chapter 10 — Gastrointestinal motility 263

mining therapy for achalasia should, therefore, heartburn and regurgitation. The esophageal ma-
depend on the patients’ overall health and surgi- nometry revealed the following findings for all the
cal risk. Myotomy, surgical or endoscopic, and test swallows shown below:
pneumatic dilation should be reserved for low-risk
patients, while high-risk candidates should receive What is the most appropriate next step in the
the safer options, including botulinum toxin injec- management of this patient?
tion and pharmacotherapy.
A. Referral for Nissen fundoplication.
REFERENCE B. Schedule botulinum toxin injection to the
Vaezi MF et al. ACG clinical guideline: diagnosis lower esophageal sphincter.
and management of achalasia. Aliment Am J Gas- C. Schedule endoscopic pneumatic dilation.
troenterol. 2013 Aug;108(8):1238-49. D. Lifestyle modification.
E. Start calcium channel blockers daily.

Question 10 CORRECT ANSWER: D


A 25-year-old woman with significant heartburn,
regurgitation, and dysphagia despite use of twice RATIONALE
daily proton pump inhibitor presents for esopha- This patient presenting with reflux symptoms was
geal function testing. The 24-hour pH-monitoring found to have increased reflux burden (acid expo-
performed off acid suppression showed a to- sure 6.2 percent) and significant correlation be-
tal acid exposure time of 6.2 percent (normal: tween symptom and acid reflux on 24-hr pH-mon-
greater than four percent), DeMeester score of itoring off acid suppression. Prior to determining
30.1 (normal: greater than 17.4), and a significant next step of therapy, including consideration for
symptom association probability (SAP) for both fundoplication, an esophageal manometry should
264 Digestive Diseases Self-Education Program®

also be performed to assess for signs of major manometry would most likely explain her physical
motility disorders that may be contraindication examination findings?
to anti-reflux surgery or pose high risk for post-
operative dysphagia. Her esophageal manometry A. Squeeze pressure of 70 mmHg.
showed evidence of a hypotensive lower esopha- B. Rectal contraction pressure of 50 mmHg
geal sphincter with no evidence of esophagogastric during strain maneuver.
outflow obstruction, as well as absent esophageal C. Resting pressure of 110 mmHg.
body contraction. These findings are consistent D. Expulsion of 50 mL balloon at 60 seconds.
with absent contractility per Chicago classification E. Relaxation of the internal anal sphincter with
v3.0, and may be suggestive of underlying con- inflation of balloon to 30 mL in the rectum.
nective tissue or smooth muscle disorders such as
scleroderma. Anti-reflux surgery, particularly a CORRECT ANSWER: C
full Nissen fundoplication (answer a), would pose
a high risk of post-operative achalasia in patients RATIONALE
with absent contractility or scleroderma and This patient’s symptoms and physical examination
should generally be avoided. Botulinum toxin in- findings suggest an anal fissure. Chronic anal fis-
jection to the lower esophageal sphincter (answer sures are believed to be caused by decreased per-
b) and pneumatic dilation (answer c) are therapies fusion and relative ischemia to the posterior anal
for achalasia, and they would likely worsen her sphincter. Internal anal sphincter hypertonia is
reflux symptoms. Smooth muscle relaxants (an- thought to contribute to the reduced perfusion to
swer e) are used for treatment of hypercontractile the sphincter, as increased pressure on the vessels
disorders and would further worsen the hypocon- passing perpendicularly through the internal anal
tractility in this patient. Therefore, conservative sphincter muscle may compromise flow. Topical
management with lifestyle modification for reflux smooth muscle relaxants or sphincterotomy aim-
(answer d) would be the most appropriate choice ing at reducing anal sphincter tone are, therefore,
for treatment of this patient. the major modalities of treatment for chronic anal
fissure. Internal anal sphincter tone is measured on
REFERENCES anorectal manometry by the resting anal sphincter
Kahrilas PJ et al. The Chicago Classification of pressure (answer c), which would be the most likely
Esophageal Motility Disorders v3.0. Neurogastro- finding in this patient with anal fissure. The squeeze
enteral Motil 2015;27(2):160-174. pressure is a measure of the contractility of the
Katz PO et al. Diagnosis and management of external anal sphincter (answer a). While defecatory
gastroesophageal reflux disease. Am J Gastroen- function may also be impaired (answers b and d) in
terol 2013;108:308-28. some patients with anal fissure, it is not the primary
underlying pathophysiology of the injury. Patients
with Hirschsprung’s disease (answer e) may also
Question 11 develop anal fissures, but it would not be the most
A 55-year-old female with a history of recurrent common finding expected in these patients.
hematochezia, rectal pain, and chronic constipa-
tion presents for anorectal manometry. A recent REFERENCES
colonoscopy was unremarkable. Perianal examina- Van Koughnett JA et al. Anorectal physiology
tion prior to the start of the manometry revealed and testing. Gastroenterol Clin North Am 2013
an ulcer-like longitudinal tear at the posterior Dec;42(4):713-28.
midline region of her anal canal. Zaghiyan KN et al. Anal Fissure. Clin Colon
Rectal Surg 2011;24(1):22-30.
Which of the following results on her anorectal
Chapter 10 — Gastrointestinal motility 265

Question 12 of the distal esophagus and lower esophageal


A 45-year-old man presents for evaluation of sphincter (LES), resulting in the preferential loss
dysphagia. His upper endoscopy was unremark- of inhibitory innervation. As a result, release of
able, including biopsies obtained. An esophageal nitric oxide and vasoactive intestinal peptide, the
manometry was obtained with the following find- neurotransmitters providing inhibitory input,
ings shown below: decrease. Acetylcholine and substance P are neu-
rotransmitters that provide excitatory input, and
Which of the following change in neurohumoral their unopposed actions in the absence of inhibi-
substance release represents the primary underly- tory input lead to the aperistalsis of the esopha-
ing pathophysiology of this condition? geal body and the inability for the LES to relax.

A. Decreased nitric oxide release. REFERENCE


B. Decreased acetylcholine release. Ates F et al. The Pathogenesis and Management of
C. Increased substance P release. Achalasia: Current Status and Future Directions.
D. Increased vasoactive intestinal peptide Gut Liver. 2015 Jul;9(4):449-63.
(VIP) release.
E. Decreased substance P release.
Question 13
CORRECT ANSWER: A A 57-year-old female presented with chronic
constipation for the last eight years, refractory to
RATIONALE multiple laxatives. Physical examination was unre-
The clinical presentation and esophageal manom- markable and basic blood tests were unrevealing.
etry are consistent with achalasia. The primary A colonoscopy performed revealed melanosis coli
underlying pathophysiology involves the degen- but no other findings. A radiopaque marker test
eration of ganglionic cells in the myenteric plexus was obtained with 19 markers retained on day five
266 Digestive Diseases Self-Education Program®

of the study (four in ascending, five in trans- slow transit constipation, upper gastrointestinal
verse, six in descending, and four in rectosig- motility assessment (e.g. gastric emptying scin-
moid). She inquired about surgical options for tigraphy, fluoroscopic upper GI series, wireless
treatment of her chronic constipation. motility capsule) and evaluation of defecatory
function (anorectal manometry) should be per-
Which of the following would be a negative pre- formed prior to surgery.
dictor of surgical outcome?
REFERENCES
A. Decreased anal sphincter squeeze pressure Kim CR et al. How to interpret a functional or a
on anorectal manometry. motility test-colon transit study. J Neurogastro-
B. Increased small intestinal transit time on enterol Motil 2012;18:94-9.
wireless motility capsule study. Bharucha AE et al. American Gastroentero-
C. Expulsion of 50-mL balloon in 30 seconds. logical Association Medical Position Statement
D. Retained radiopaque markers in the on Chronic Constipation. Gastroenterology
ascending and transverse colon. 2013;144:211–217.
E. Melanosis coli on colonoscopy.

CORRECT ANSWER: B Question 14


A 78-year-old man with a history of Parkinson’s
RATIONALE and recurrent C difficile infection presents for
The radiopaque marker study revealed 19 mark- evaluation of epigastric pain, early satiety, and
ers retained throughout the colon (normal: post-prandial fullness. He notes that his epi-
greater than or equal to five), suggesting signifi- gastric pain remains his most severe symptom
cantly delayed colonic transit. Colectomy with for which he has had several presentations to
ileorectal anastomosis is the primary surgical the emergency room. Evaluation with an upper
treatment for severe slow transit constipation. endoscopy revealed retained food in the stomach
However, presence of an upper gastrointestinal despite a 12-hour fast prior to the procedure. A
motility disorder [e.g. delayed gastric emptying gastric emptying scintigraphy showed 52 percent
and/or small intestinal transit (answer b)] is emptying of test meal at four hours.
a negative predictor for colectomy, as patients
may continue to experience symptoms related Which of the following treatment option is the
to these extra-colonic dysmotility. Concomitant most appropriate for this patient?
defecation disorder should also be addressed
prior to construction of ileorectal anastomosis. A. Gastric electrical stimulation
A weak external anal sphincter (answer a) found B. Erythromycin 100 mg three times daily
on anorectal manometry and a normal balloon C. Initiation of a heart-healthy low fat and high
expulsion test (answer c) are not associated fiber diet
with worse surgical outcome for colectomy with D. Metoclopramide 10 mg before meals and
ileorectal anastomosis. Retained radiopaque bedtime.
markers in the ascending and transverse colon E. Initiation of a low fiber, low residue diet
(answer d) suggest severe slow transit constipa-
tion for which colectomy is indicated. Melano- CORRECT ANSWER: E
sis coli on colonoscopy (answer e) are benign
findings secondary to chronic laxative use and RATIONALE
are not associated with surgical outcome. When This patient presents with symptoms and find-
evaluating patients for colectomy for severe ings suggestive of gastroparesis, with post-
Chapter 10 — Gastrointestinal motility 267

prandial pain, early satiety, fullness, as well as A. Hypercontractile swallows (DCI greater
retained food in stomach after prolonged fast than 8000 mmHg*s*cm) in 40 percent of
and delayed emptying on scintigraphy. Pain- test swallows.
predominant symptoms are predictors for B. Decreased distal latency in 60 percent of
more suboptimal response in gastric electrical test swallows.
stimulation (answer a), as is idiopathic gastro- C. Increased integrated residual pressure (IRP)
paresis, compared to diabetic and post-surgical and no esophageal body contraction in all
gastroparesis. While erythromycin (answer b) swallows.
is effective as a pro-motility agent, chronic use D. Failed swallows (DCI greater than 100
of an antibiotic would not be appropriate in this mmHg*s*cm) in 30 percent and weak
patient with a history of recurrent C difficile swallows (DCI 250 to 350 mmHg*s*cm) in
infection. Development of tardive dyskinesia is 40 percent of swallows.
an important potential adverse effect for the use E. Increased integrated residual pressure and
of metoclopramide (answer d), and the risk is pan-esophageal pressurization greater than
higher in patients with a pre-existing movement 30 mmHg in all swallows.
disorder, such as Parkinson’s. Dietary modifica-
tion is an important component of gastropare- CORRECT ANSWER: D
sis management, especially as part of first-line
therapy. Small meals low in fiber and fat are the RATIONALE
mainstay of diet for gastroparesis, as fiber and In patients presenting with non-cardiac chest
fat would slow gastric emptying. pain, the most common etiologies are gastro-
esophageal reflux disease (29 to 60 percent)
REFERENCES and functional chest pain (32 to 60 percent). A
Camilleri M et al. Clinical guideline: Manage- major primary esophageal motor disorder only
ment of gastroparesis. AM J Gastroenterol accounts for a minority of cases of non-cardiac
2013;108:18-37. chest pain (10 to 18 percent). In this over-
Rao SS et al. Metoclopramide and tardive dys- weight patient with a hiatal hernia, underlying
kinesia Aliment Pharmacol Ther 2010;31:11-19. gastroesophageal reflux disease would be the
most likely etiology of his chest pain. Ineffective
esophageal motility (answer d) is the esophageal
Question 15 manometry diagnosis most commonly associat-
A 65-year-old overweight man presents for eval- ed with gastroesophageal reflux disease. Jack-
uation of intermittent chest pain and tightness. hammer esophagus (answer a), diffuse esopha-
His symptoms are most frequent after eating, geal spam (answer b), type I achalasia (answer
although they may occur at any time of the day. c), and type II achalasia (answer e) are major
Cardiac evaluations including a stress test per- primary esophageal motor disorders that are less
formed were unremarkable. He completed an common causes of chest pain than gastroesopha-
upper endoscopy with esophageal biopsies that geal reflux disease.
were normal. A barium esophagram performed
revealed a normal caliber esophagus and medi- REFERENCES
um-sized hiatal hernia, but no other abnormali- Fass R et al. Non-cardiac chest pain: an update.
ties. He was then referred for further evaluation Neurogastroenterol Motil 2006 Jun;18(6):4
with esophageal function testing. 08-17.
Barret M et al. Diagnostic yield of 24-hour
Which of the following would be the most likely esophageal manometry in non-cardiac chest
finding on esophageal manometry? pain 2016 Aug;28(8):1186-93.
268 Digestive Diseases Self-Education Program®

Question 16 Question 17
Which of the following statements regarding A 67-year-old man with peptic ulcer disease
normal defecation and continence is correct? status post recent antrectomy with Billroth I
presents with post-prandial fullness, nausea and
A. Rectal distention results in relaxation of the vomiting. His symptoms usually occur about 30
external anal sphincter. minutes after meals and may last for an hour.
B. Normal defecation requires an increase in He also reports associated abdominal cramps
intraabdominal pressure and contraction of and diarrhea, as well as sweating and palpitation
the puborectalis muscle. during these episodes. He underwent an upper
C. During evacuation, the pelvic floor descends endoscopy that revealed a healthy-appearing
and the anorectal angle decreases. anastomosis.
D. Straining or “bearing down” during defecation
results in relaxation of the internal anal Which of the following is the most appropriate
sphincter. next step in management?
E. Phasic contractions of the internal anal
sphincter during cough maintain normal A. Octreotide LAR 20 mg monthly.
continence of feces. B. Initiation of a low fiber, low residual diet.
C. Ensure adequate liquid intake during meals.
CORRECT ANSWER: D D. Increase simple sugars and avoid complex
carbohydrates intake.
RATIONALE E. Domperidone 15 mg before meals.
When straining during normal defecation, the in-
ternal anal sphincter relaxes to allow opening of the CORRECT ANSWER: A
anal canal for passage of stools (answer d). When
fecal contents reach the rectal vault, distention of the RATIONALE
rectum results in the relaxation of the internal anal Dumping syndrome is a potential complication
sphincter (rectoanal inhibitory reflux, or RAIR), not of gastric surgery resulting from rapid emptying
external anal sphincter (answer a), which actually of hyperosmolar stomach content into the small
contracts to help maintain continence. Normal def- bowel. Early dumping symptoms are character-
ecation requires an increase in intraabdominal pres- ized by post-prandial nausea, vomiting, fullness,
sure with simultaneous relaxation, not contraction abdominal cramps, and diarrhea, with associ-
(answer b), of the puborectalis muscle, which forms ated diaphoresis, lightheadedness, palpitation,
a posterior “sling” around the rectum. The descent and flushing. Treatment of dumping syndrome
of the pelvic floor and relaxation of the puborectalis involves mainly dietary changes, including eat-
muscle during evacuation result in an increase, not ing multiple, small meals, avoiding simple sugars,
decrease (answer c), of the anorectal angle. This increasing dietary fat and fiber, and separating
leads to straightening of the rectum to allow passage ingestion of liquids from solids during meals (i.e.
of stools. During cough or other maneuvers that no liquids 30 minutes before and after meals).
increase intraabdominal pressure, phasic contrac- Answers b, c, and d are, therefore, the opposite
tions of the external, not internal (answer e), anal of the recommended diet. Domperidone (answer
sphincter helps maintain continence. e) is a promotility agent used for gastroparesis.
Octreotide, either administered in short-acting
REFERENCE form before meals or in long-acting form monthly
Rao, S.S.C., et al., Anorectal Disorders. Gastroen- (answer a), has been shown to reduce dumping
terology, 2016. 150(6): p. 1430-1442.e4. symptoms.
Chapter 10 — Gastrointestinal motility 269

REFERENCE her constipation. Ondansetron (answer c), a 5HT-


Berg P et al. Dumping Syndrome: A Review of the 3 antagonist, has been found to decrease intestinal
Current Concepts of Pathophysiology, Diagnosis motility and may worsen her constipation. Ri-
and Treatment. Dig Dis Sci 2016;61(1):11-8. faximin (answer e) is approved for treatment of
diarrhea-predominant irritable bowel syndrome,
not IBS-C.
Question 18
A 32-year-old woman with a history of anxiety REFERENCE
presents with abdominal pain, bloating, and Ford, A.C., et al., American College of Gastro-
constipation since childhood. Her symptoms have enterology monograph on the management of
been worsening over the years and she currently irritable bowel syndrome and chronic idiopathic
has bowel movements every three to five days. constipation. Am J Gastroenterol, 2014;109 Suppl
Her abdominal pain tends to worsen when she has 1: p. S2-26; quiz S27.
not had a bowel movement for a few days, and it
would improve after having a bowel movement.
She denies any fever, nocturnal symptoms, greasy Question 19
stools, hematochezia/melena, or weight loss. She A 30-year-old man is enrolling in a clinical study
has undergone extensive evaluations, including as a healthy control. As part of the criteria to
endoscopies, imaging studies (CT, US, MRI), stool enter the study, he has to undergo an esophageal
studies, Celiac testing, and serum inflammatory manometry. He reports no dysphagia, odynopha-
markers, which were all negative. gia, chest pain, heartburn, reflux, regurgitation, or
any other symptoms.
Which of the following agent would you recom-
mend for treatment of her symptoms? Which of the following would be the most likely
findings on his esophageal manometry per Chi-
A. Amitriptyline. cago classification?
B. Dicyclomine.
C. Ondansetron. A. All swallows were hypercontractile (DCI
D. Lubiprostone. greater than 8000 mmHg*s*cm).
E. Rifaximin. B. Failed contractions (DCI less than 100
mmHg*s*cm) in all swallows with normal
CORRECT ANSWER: D integrated residual pressure.
C. Decreased distal latency (DL less than 4.5 sec)
RATIONALE in 50 percent of swallows.
This patient’s presentation (including her symp- D. Large peristaltic break (PB greater than 5 cm)
tom characteristics, lack of “red flag” symptoms, in 20-mmHg isobaric contour in 60 percent of
and extensive negative evaluations) is sugges- swallows.
tive of constipation-predominant irritable bowel E. Hypercontractile swallows (DCI greater than
syndrome (IBS-C). Lubiprostone (answer d) is a 8000 mmHg*s*cm) in 30 percent of swallows.
secretagogue approved for treatment of chronic
idiopathic constipation and IBS-C. It promotes CORRECT ANSWER: D
intestinal fluid secretion through activation of
ClC-2 chloride channels. Both amitriptyline RATIONALE
(answer a), a tricyclic antidepressant, and dicyclo- Esophageal motility disorders per Chicago classi-
mine (answer b), an antispasmodic agent, carry fication v3.0 are generally classified into disorders
anticholinergic effects and would therefore worsen with esophagogastric junction outflow obstruction
270 Digestive Diseases Self-Education Program®

(including achalasia and EGJOO), major disorders having the poorest outcome. Graded pneumatic
of peristalsis, and minor disorders of peristalsis. dilations and surgical myotomy have been found
Major disorders (including diffuse esophageal to have similar treatment response at five year
spasm [answer c], Jackhammer esophagus [an- follow-up in the European Achalasia Trial, the
swers a and e], absent contractility [answer b]) are largest randomized controlled trial for treatment
nearly universally symptomatic and not found in of achalasia to-date. In the study, ~25 percent of
normal, asymptomatic subjects. Minor disorders patients in the pneumatic dilation arm required
(including ineffective esophageal motility and frag- more than one dilation. Intrasphincteric injec-
mented peristalsis [answer d]) may sometimes be tion of botulinum toxin to the lower esophageal
associated with symptoms, but they are also found sphincter has good initial response that decreases
incidentally in healthy, asymptomatic individuals. with time, with ~40 percent recurrence within
one year. However, the treatment efficacy has
REFERENCE been found to decrease with subsequent injec-
Kahrilas PJ et al. The Chicago Classification of tions (answer e). Therefore, for patients with low
Esophageal Motility Disorders v3.0. Neurogastro- surgical risk, pneumatic dilation and myotomy
enteral Motil 2015;27(2):160-174. should be first-line options. Botulinum toxin
injection should be reserved for those who are at
high surgical risk and are thus poor candidates
Question 20 for myotomy or pneumatic dilation. Pharma-
Which of the following statements regarding treat- cotherapy has limited efficacy and is inferior to
ment for achalasia is true? endoscopic or surgical therapies. It should be
reserved for patients who are not candidates for
A. Patients with type I achalasia have a higher other treatments.
rate of response to all treatments than
patients with type II achalasia. REFERENCES
B. Heller myotomy has been shown to have a Moonen A et al. Long-term results of the Euro-
higher sustained response rate at five year pean achalasia trial: a multicentre randomized
follow-up than graded pneumatic dilations in controlled trial comparing pneumatic dilation
randomized clinical trial. versus laparoscopic Heller myotomy. Gut. 2016
C. Intrasphincteric injection of botulinum toxin May;65(5):732-9.
to the lower esophageal sphincter should be Kahrilas PJ et al. Expert consensus document:
used as a first-line therapy prior to consid- Advances in the management of oesophageal mo-
eration for myotomy or dilation for all tility disorders in the era of high-resolution ma-
achalasia patients. nometry: a focus on achalasia syndromes. Nat Rev
D. Pharmacotherapy with calcium channel Gastroenterol Hepatol. 2017 Nov;14(11):677-688.
blockers or nitrates has similar initial efficacy
as endoscopic therapies.
E. The efficacy of intrasphincteric botulinum Question 21
toxin injection decreases with subsequent A 28-year-old woman presents for evaluation of
repeat injections. post-prandial fullness, early satiety, and epi-
gastric pain. These symptoms occur 15 to 30
CORRECT ANSWER: E minutes after initiation of a meal. She denies
any heartburn, reflux, regurgitation, chest pain,
RATIONALE or vomiting. She underwent evaluations includ-
Treatment efficacy for achalasia overall is high- ing an upper endoscopy that was unremarkable.
est with type II followed by type I, with type III Gastric biopsies obtained were also normal.
Chapter 10 — Gastrointestinal motility 271

She completed a two month trial of twice-daily REFERENCE


proton pump inhibitors that did not result in Moayyedi PM et al. ACG and CAC Clinical Guide-
any change in her symptoms. A gastric empty- line: Management of Dyspepsia. Am J Gastroen-
ing scintigraphy performed showed 99 percent terol. 2017 Jul;112(7):988-1013.
emptying at four hours, and an abdominal ultra-
sound obtained did not reveal any abnormalities.
She later completed a wireless motility capsule Question 22
study that showed normal gastric, small bowel, A 65-year-old man presents for evaluation of
and whole gut transit. heartburn symptoms refractory to twice daily
proton pump inhibitor therapy. He underwent
Which of the following is the best option to be an upper endoscopy which was normal, includ-
considered for the next step of therapy? ing biopsies obtained from the esophagus. An
esophageal manometry completed showed a
A. Buspirone 10 mg before meals. normal integrated residual pressure (IRP), 20
B. Triple antibiotics therapy for H. pylori infection. percent failed swallows (DCI greater than 100
C. Referral for surgical evaluation for c mmHg*s*cm), 40 percent weak swallows (DCI
holecystectomy. 100-450 mmHg*s*cm), and 40 percent normal
D. Metoclopramide 10 mg before meals. swallows (DCI 450-8000 mmHg*s*cm). He also
E. Initiation of high fat, high fiber diet. completed a 24-hr pH-impedance study off acid
suppression therapy. The study showed an acid
CORRECT ANSWER: A exposure time of two percent on pH-monitoring
and 33 total reflux episodes on impedance. He re-
RATIONALE ported 24 heartburn events during the study, with
Functional dyspepsia is characterized by upper eight correlating with a reflux episode.
abdominal symptoms in the absence of an iden-
tifiable organic or metabolic cause. There are a Which of the following therapy would you recom-
variety of possible etiologies of functional dys- mend for his symptoms?
pepsia, including delayed or accelerated gastric
emptying, impaired gastric accommodation, A. Nissen fundoplication.
visceral hypersensitivity, gastroesophageal reflux, B. Magnetic sphincter augmentation (LINX).
Helicobacter pylori gastritis, duodenal hypersen- C. Baclofen 10 mg before meals and bedtime.
sitivity to lipids or gastric acid, and psychologi- D. Nortriptyline 10 mg at bedtime.
cal factors. The patient in question underwent E. Sucralfate slurry four times daily.
evaluations that showed no evidence of H. pylori
infection (negative gastric biopsies), hepatobiliary CORRECT ANSWER: D
etiologies (negative ultrasound), or altered gastric
motility (normal gastric emptying scintigraphy). RATIONALE
Therefore, answers b (therapy for H. pylori), c This patient presenting with heartburn symptoms
(intervention for gallstone disease), and both d had a normal endoscopy and an esophageal ma-
and e (treatments for gastroparesis) would be nometry showing ineffective esophageal motility
unlikely to be helpful. Buspirone (answer a) has (IEM). His 24-hr pH-impedance study revealed a
been shown in clinical studies to improve fundic normal reflux burden (normal acid exposure and
accommodation and reduce post-prandial dys- total reflux episodes) and no significant symptom-
pepsia symptoms, and it would be the most ap- reflux correlation [symptom index = 8/24 = 33.3
propriate choice in this patient with post-prandial percent (greater than 50 percent suggests signifi-
symptoms. cant correlation)]. Therefore, he would be clas-
272 Digestive Diseases Self-Education Program®

sified as having functional heartburn. Antireflux D. Decreased anal sphincter resting pressure.
surgeries (answers a and b) would not be appro- E. Increased anal sphincter pressure during
priate for treatment of functional heartburn. In strain or bear-down maneuver.
addition, IEM is currently an exclusion criterion
for the use of magnetic sphincter augmentation CORRECT ANSWER: A
(LINX). Baclofen (answer c) has been shown to
increase basal lower esophageal sphincter pres- RATIONALE
sure and decrease transient lower esophageal Anal sphincter squeeze pressure measures the vol-
sphincter relaxation, and may play a role as an untary contraction of the external anal sphincter
anti-reflux agent. However, it would likely have (EAS), while the anal sphincter resting pressure
no effect in functional heartburn. Sucralfate reflects the tonically active internal anal sphinc-
(answer e) has not been found to provide signifi- ter (IAS). The EAS reflexively contracts during
cant benefits in the treatment of gastroesophageal actions associated with increased intraabdominal
reflux disease (erosive or nonerosive) or functional pressure (e.g. coughing, laughing) to maintain
heartburn. Tricyclic antidepressants such as nor- continence. It is also under voluntary control to
triptyline (answer d) have been shown to provide “hold” when an urge for defecation is felt. The
benefits for functional gastrointestinal disorders, patient in question suffers from symptoms sug-
including functional heartburn. gestive of urge incontinence, which is likely due
to weakness in the EAS. Therefore, her anorectal
REFERENCES manometry would likely show a decrease in anal
Gyawali CP et al. Modern diagnosis of GERD: the sphincter squeeze pressure (answer a). Weak-
Lyon Consensus. Gut 2018 Jul;67(7):1351-1362. ness in the IAS or resting tone, characterized by
Hachem C et al. Diagnosis and management decreased anal sphincter resting pressure (answer
of functional heartburn. Am J Gastroenterol 2016 d), would result in passive stool leakage without
Jan: 111(1):53-61. sensation. Hyposensitivity of the rectum (answer
b), Hirschsprung’s disease (answer c), and dys-
synergic defecation (answer e) are associated with
Question 23 stool impaction and overflow incontinence, also
A 54-year-old woman presents for evaluation of fecal characterized by silent, passive leakage of stools.
incontinence. She describes a sudden urge for bowel
movements, but difficulty holding long enough to REFERENCE
get to the bathroom, resulting in leakage of stools. Carrington EV et al. Expert consensus document:
She also reports leakage of a small amount of fecal Advances in the evaluation of anorectal function. Nat
material when she coughs and laughs. She denies Rev Gastroenterol Hepatol 2018 May;15(5):309-323.
any silent seepage or soiling of underwear. A flexible
sigmoidoscopy performed did not reveal any ab-
normalities within the rectum. She then underwent Question 24
anorectal manometry for further evaluation. Which of the following statements regarding gas-
tric and small intestinal motility is true?
Which of the following is the most likely finding
on anorectal manometry for this patient? A. Segmentation occurs during fed state
following a meal with propagated contractions
A. Decreased anal sphincter squeeze pressure. of the intestine aborally.
B. Increased rectal sensation volume. B. The fasting state is characterized by the
C. No change in anal sphincter pressure with migrating motor complex (MMC) that cycles
distention of balloon in rectum. every 90-120 minutes.
Chapter 10 — Gastrointestinal motility 273

C. Peristalsis of the small intestine involves of the following medications are appropriate for
activation of acetylcholine-containing motor management of his constipation, EXCEPT:
neurons distal to the bolus of chyme.
D. Activation of nitric oxide-containing motor A. Naloxegol (selective mu-opioid receptor
neurons proximal to the bolus of chyme helps antagonist)
propulsion of the bolus during peristalsis. B. Methylnaltrexone (peripheral mu-opioid
E. Carbohydrate content in the ileum activates receptor antagonist and delta-opioid receptor
the chemoreceptors and the “ileal brake” antagonist)
which delays gastric emptying. C. Eluxadoline (mu- and kappa-opioid receptor
agonist and delta-opioid receptor antagonist)
CORRECT ANSWER: B D. Lubiprostone (ClC-2 chloride channel agonist)
E. Naldemedine (peripherally selective mu-
RATIONALE opioid receptor antagonist)
The fasting motility pattern of the small intestinal
involves migrating motor complex (MMC) trav- CORRECT ANSWER: C
eling from the proximal to distal small bowel in
90-120-minute cycles (answer b). The fed state is RATIONALE
characterized by two processes: (1) segmentation, Pharmacotherapy for opioid-induced constipation
which is a localized contraction at multiple levels involves the use of traditional laxatives as first-line
simultaneously, with no net forward movement of agents, including lubiprostone, a ClC-2 agonist.
chyme to allow chopping and mixing with diges- If traditional laxatives are ineffective, further
tive enzymes, and (2) peristalsis, which is a slow, management would include several peripherally-
coordinated contraction that results in forward acting mu-opioid receptor antagonists (naldeme-
propulsion of chyme along the intestine. Peristal- dine, naloxegol, and methylnatrexone). These
sis involves activation of excitatory motor neurons agents selectively inhibit the mu-opioid receptors
(acetylcholine or substance P) proximal to the in the gastrointestinal tract, thereby restoring
bolus of chyme and inhibitory motor neurons (ni- function of the enteric nervous system and gut
tric oxide or VIP) distal to the bolus. This allows motility. They do not enter the central nervous
propulsion of the bolus aborally. The “ileal brake” system, thereby preserving the analgesic effects of
is triggered by the presence of fat content in the the opioid. Eluxadoline (answer c) is a mu-opioid
ileum, delaying gastric emptying. receptor agonist, and would therefore decrease gut
motility. It is primarily approved for treatment of
REFERENCE diarrhea-predominant irritable bowel syndrome,
Schemann M. Control of gastrointestinal motility and would not be appropriate for management of
by the “gut brain” – the enteric nervous system. constipation.
J Pediatr Gastroenterol Nutr 2005 Sep;41 Suppl
1:S4-6. REFERENCE
Crokett SD et al. American Gastroenterological
Association Institute Guideline on the Medical
Question 25 Management of Opioid-Induced Constipation.
A 65-year-old man with chronic pancreatitis on Gastroenterology 2018 Epub ahead of print.
daily opioids presents for management of chronic
constipation. His symptoms have been worsening
over the past year, now with bowel movements Question 26
once per week that require significant straining. A 45-year-old female with a three-year history of
He describes his stools as hard and lumpy. All type 2 diabetes mellitus was seen in GI clinic re-
274 Digestive Diseases Self-Education Program®

porting post prandial fullness, early satiety, nausea A. Improve symptoms severity and gastric
and epigastric discomfort. Her blood sugar levels emptying in diabetic gastroparesis.
were erratic and recent HgA1c was 10.1. Physical B. Improve symptoms in post-surgical gastroparesis.
examination was consistent with peripheral neu- C. Improve symptoms and gastric emptying in
ropathy. An upper endoscopy was unremarkable idiopathic gastroparesis
except mild gastritis. A 4-hour gastric emptying D. Improve gastric emptying in patients with
study revealed 26 percent meal retention. opiate induced gastroparesis

What would be the most beneficiary next step in CORRECT ANSWER: A


the management of this patient?
RATIONALE
A. Erythromycin Gastric electrical stimulation is rarely used; it is
B. Metoclopramide used in patients with refractory symptoms, such
C. Botulinum toxin injection of the pylorus as nausea and vomiting, when all other lifestyle,
D. Improved sugar control nutritional and medical interventions have failed.
It has been shown to improve symptom severity
CORRECT ANSWER: D particularly in patients with diabetic gastropare-
sis. It may also improve gastric emptying. There is
RATIONALE less effect in those with idiopathic or post-surgical
The patient in this clinical scenario demonstrates gastroparesis. Cessation of opiates is one of the
poor glycemic control based on her history of treatment goals for those with opiate induced
abnormal blood glucose levels and abnormally gastroparesis.
high HgA1c. She has end organ damage from her
diabetes and evidence of delayed gastric empty- REFERENCE
ing. While erythromycin in the short term and Camilleri M et al. Clinical guideline: Management of
metoclopramide can improve gastric emptying gastroparesis. AM J Gastroenterol 2013;108:18-37.
and symptoms, good glycemic control should be
the initial goal. Since acute hyperglycemia inhibits
gastric emptying, it has been assumed that better Question 28
glycemic control may improve gastric emptying A 32-year-old female with history of diabetes mel-
and symptoms. litus was seen in GI clinic reporting four to five epi-
sodes per year of nausea and vomiting. Her physi-
REFERENCE cal examination is unremarkable and recent HgA1c
Camilleri M et al. Clinical guideline: Management of was 5.1 percent. The episodes may last between
gastroparesis. AM J Gastroenterol 2013;108:18-37. three to seven days. The patient reports that reduc-
ing oral intake and hot showers markedly improve
her overall feeling. The patient denies other associ-
Question 27 ated symptoms and except mild epigastric tender-
A colleague of yours is considering referring his ness, her physical examination was unremarkable.
daughter for gastric electrical stimulation due to
severe, refractory gastroparetic symptoms, par- What would you recommend next?
ticularly repeated episodes of nausea and vomiting
associated with weight loss. A. Propranolol
B. Rigorous blood sugar control
What would you tell your colleague about gastric C. Discontinue smoking marijuana
electrical stimulation? D. Breathing exercises for relaxation
Chapter 10 — Gastrointestinal motility 275

CORRECT ANSWER: C RATIONALE


This patient presents with noncardiac chest pain
RATIONALE and studies have demonstrated that gastroesopha-
In a patient with greater than a handful discrete geal reflux disease (GERD), esophageal motor
episodes of nausea and vomiting, who is asymp- disorders and functional chest pain are the most
tomatic in between these episodes, cyclic nausea common underlying causes. GERD has been
and vomiting is highly suspected. Normal physical excluded in this patient and with that some of the
examination, good glycemic control and no symp- esophageal motor disorders associated with GERD
toms between the nausea and vomiting episodes (weak lower esophageal sphincter and ineffective
suggest other causes than diabetic gastroparesis. esophageal motility). High resolution esopha-
However, her symptoms markedly improve with geal manometry in these patients are commonly
hot showers. This suggests the triad seen in hy- unremarkable, leading the way to the diagnosis
peremesis cannabis syndrome: symptoms of cyclic of functional chest pain (requires lack of major
vomiting, chronic marijuana use, and hot showers esophageal motor disorder).
to try to relieve symptoms. Hyperemesis cannabis
syndrome is commonly confused with cyclic vom- REFERENCES
iting syndrome (CVS). Akinsikv O et al. High resolution vs. conven-
tional esophageal manometry in the assessment
REFERENCE of esophageal motor disorders in patients with
Aziz I et al. Epidemiology, clinical character- noncardiac chest pain. Neurogastroenterol Motil
istics and associations for Rome IV functional 2018;30(6):e13282.
nausea and vomiting disorders in adults. Yamasaki T et al. Noncardiac chest pain: diag-
Clin Gastroenterol Hepatol 2018; 5152- nosis and management Curr Opin Gastroenterol
3565(18):30552-4. 2017;33:293-300.

Question 29 Question 30
A 45-year-old male was referred to you after a A patient was seen for primarily GERD related
comprehensive workup for chest pain that was un- symptoms that awaken her from sleep during the
revealing. The patient was evaluated by a cardiolo- night. Which of the following statements is correct
gist, underwent an upper endoscopy with biopsies, regarding nighttime reflux?
wireless pH capsule and a PPI trial without an
identifiable cause or improvement in symptoms. A A. There are less reflux events during sleep time.
physical examination in your office was unremark- B. Short arousals due to reflux commonly trigger
able. You referred the patient for high resolution symptoms
esophageal manometry. C. Patients with daytime symptoms have similar
risk for Barrett’s esophagus and adenocarci-
What would likely be the finding in this clinical noma of the esophagus as those with
scenario? nighttime symptoms.
D. Lower esophageal sphincter pressure is lower
A. Distal esophageal spasm during sleep as compared to awake period
B. Normal esophageal motility
C. Jackhammer esophagus CORRECT ANSWER: A
D. Achalasia

CORRECT ANSWER: B
276 Digestive Diseases Self-Education Program®

RATIONALE regurgitation. An upper endoscopy was unremark-


During sleep there is marked reduction in reflux able. An esophageal manometry revealed normal
episodes as well as symptoms. Nighttime GERD peristalsis throughout the esophagus. The lower
exposes patients to GERD – related complica- esophageal sphincter basal pressure and func-
tions, such as peptic stricture, esophageal ulcer, tion were all within the normal range. The upper
severe erosive esophagitis, extra-esophageal esophageal sphincter basal pressure was low-
manifestations, Barrett’s esophagus and even ade- normal and the amplitudes in the pharynx and
nocarcinoma of the esophagus. Lower esophageal proximal esophagus were abnormally low. The
sphincter basal pressure during sleep remains rest of the amplitude contractions were within the
similar to its pressure during daytime. Reflux dur- normal range. What is the likely diagnosis?
ing sleep commonly occurs during the first one to
two hours of sleep. A. Scleroderma
B. Systemic Lupus Erythematosus
REFERENCE C. Parkinson’s disease
Fujiwara Y et al. Gastroesophageal reflux dis- D. Polymyositis
ease and sleep. Gastroenterol Clin North AM
2013;42:57-70. CORRECT ANSWER: D

RATIONALE
Question 31 Polymyositis affects swallowing by involving the
What is the cause of dysphagia lusoria? striated muscle of the pharynx and esophagus in
up to 50-70 percent of the patients. Diabetes and
A. Enlargement of the left atrium SLE may result in non-specific motor abnormali-
B. Aneurismal thoracic aorta ties of the esophagus. Scleroderma, solely affects
C. Aberrant right subclavian artery the distal 2/3 smooth muscle of the esophagus.
D. Aberrant thoracic duct
REFERENCE
CORRECT ANSWER: C Gasiorowska A et al. Current approach to dyspha-
gia Gastroenterol Hepatol 2009;5:269-79.
RATIONALE
A right aortic arch giving rise to aberrant right
subclavian artery. Patients commonly complain of Question 33
dysphagia for solids associated with regurgitation A 34-year-old male was seen in GI clinic for 10-
of unchewed food, post prandial bloating, chest year history of GERD – related symptoms. The
pain and symptoms that may change with position. patient has received a standard dose of PPI that
fully controlled his symptoms but was unable to
REFERENCE be compliant with daily dosing. A recent upper
Gasiorowska A et al. Current approach to dyspha- endoscopy off treatment was unremarkable. The
gia Gastroenterol Hepatol 2009;5:269-79. patient inquires about anti-reflux surgery as a
long-term solution for his symptoms. Which of
the following patient’s characteristics is a positive
Question 32 predictive factor for successful outcome after anti-
A 51-year-old female was seen for dysphagia for reflux surgery?
both solids and liquids that occurred immediately
after swallowing. The patient reports coughing A. Young age
and choking during meals with occasional nasal B. Being a male
Chapter 10 — Gastrointestinal motility 277

C. Full response to PPI treatment these esophageal motility changes is responsive


D. A negative upper endoscopy to anti-reflux treatment which thus should be the
first therapeutic approach.
CORRECT ANSWER: C
REFERENCE
RATIONALE George N et al. The current treatment of non-
Studies have repeatedly demonstrated that com- cardiac chest pain. Aliment Pharmacol Ther
plete response to PPI treatment is a positive pre- 2016;43:213-39.
dictive factor for a successful anti-reflux surgery in
GERD patients.
Question 35
REFERENCE A 52-year-old male was seen in GI clinic due to
Katz PO et al. Diagnosis and management of gas- dysphagia for solids and liquids. The patient un-
troesophageal reflux disease. Am J Gastroenterol derwent surgical fundoplication five years ago for
2013;108:308-28. uncontrolled symptoms of daytime and nighttime
regurgitation. Endoscopy prior to surgery was
reported as unremarkable. However, post-surgery,
Question 34 patient reported continued symptoms of regurgi-
A 25-year-old woman with noncardiac chest pain tation, new dysphagia that progressed over time
underwent high resolution esophageal manom- to both solids and liquids and 15 lb. weight loss
etry. The test revealed median distal contractile over a period of nine months. Endoscopy revealed
integral that was greater than 5000 but did not ex- intact wrap with somewhat tight esophagogastric
ceed 8000 mmHg.cm.sec in any of the swallows. junction. High resolution esophageal manometry
The integrated relaxation pressure was within revealed 100 percent absent peristalsis and IRP of
normal limits. 35 mmHg.

What would be the first therapeutic approach in Which of the following test results prior surgery
this patient? was needed to further understand patient’s cur-
rent clinical status?
A. Pneumatic dilation
B. Nifedipine 10 mg tid A. High resolution esophageal manometry
C. Botulinum toxin injection into the lower B. Wireless pH capsule of PPI treatment
esophageal sphincter C. Impedance +pH test on PPI treatment
D. PPI once daily D. Chest CT

CORRECT ANSWER: D CORRECT ANSWER: A

RATIONALE RATIONALE
Using the updated Chicago classification this All patients who are candidates for surgical
NCCP patient has normal high resolution esopha- fundoplication, require an upper endoscopy and
geal manometry. However, consistently through- esophageal manometry prior to surgery. The latter
out the 10 swallows the patient demonstrates a is to exclude major esophageal motor disorders,
hypertensive esophagus (Distal contractile inte- especially achalasia. In addition, if endoscopy off
gral greater than5000mmHg.cm.sec), what used treatment is negative, a pH test, preferably the
to be called nutcracker esophagus. Studies have wireless pH capsule, should be performed off PPI
demonstrated that chest pain associated with treatment to ensure the diagnosis of gastroesopha-
278 Digestive Diseases Self-Education Program®

geal reflux disease. In this patient with achalasia REFERENCE


finding on high resolution esophageal manom- Skardoon GR et al Dyssynergic defecation and
etry, having a normal esophageal motility prior to biofeedback therapy in the pathophysiology and
surgery suggests that recorded achalasia is likely a management of functional constipation Aliment
long term consequence of a tight surgical wrap. If Pharmacol Ther 2017;46:410-23.
the patient did not have an esophageal manometry
prior to surgery, it could also be that the surgical
fundoplication was inappropriately performed in a Question 37
patient with achalasia. Which of the following statements is true about
the recto-anal inhibitory reflex (RAIR)?
REFERENCE
Katz PO et al. Diagnosis and management of gas- A. Transient relaxation of the external anal
troesophageal reflux disease. Am J Gastroenterol sphincter in response to rectal distension
2013;108:308-28. B. Transient relaxation of the puborectalis in
response to rectal distension
C. Absence of RAIR is the underlying mechanism
Question 36 of colonic inertia
A 48-year-old female with five years history of D. Transient relaxation of the internal sphincter
constipation, who failed treatment with various in response to rectal distension.
laxatives, was seen in GI clinic. Symptoms have
worsened in the last two years with increase strain- CORRECT ANSWER: D
ing and pencil like stools. A recent colonoscopy was
unremarkable. Anorectal manometry revealed an RATIONALE
adequate push effort and an increase in the anal RAIR is the transient relaxation of the internal
external sphincter contraction during defecation sphincter in response to rectal distension. It is
simulated with a bearing down maneuver. mediated through a local enteric nervous system
reflex. Its purpose is likely to help the proximal
What would be the most effective therapeutic ap- anal canal to distinguish between flatus and stool.
proach for this patient?
REFERENCE
A. Reassurance Camilleri M et al. Chronic constipation Nat Rev
B. Linaclotide 145 mcg once daily Dis primers 2017;3:17095.
C. Polyethylene glycol (PEG) 17gr as needed
D. Biofeedback
Question 38
CORRECT ANSWER: D A 52-year-old female was evaluated for chronic
constipation for the last six years. Physical exami-
RATIONALE nation was unremarkable and basic blood tests
The patient demonstrates type 1 dyssynergic def- were unrevealing. A colonoscopy revealed small
ecation. This cause of constipation responds very internal hemorrhoids but no other findings. A
effectively to biofeedback. During biofeedback radiopaque marker test was ordered documenting
the patient is trained to relax and thus reduce the on day five, a total of 12 (out of 24) markers that
pressure of the external sphincter during defeca- were still left in the colon, primarily in the recto-
tion. Reassurance is insufficient and polyethylene sigmoid area.
glycol or linaclotide are not considered first line
therapy for dyssynergic defecation. What would be your next management step?
Chapter 10 — Gastrointestinal motility 279

A. Biofeedback have been shown to be associated with esophago-


B. Recto-sigmoid deep wall biopsy gastric junction outflow obstruction, achalasia
C. Ano-rectal manometry (not type 1) and other hypercontractile esophageal
D. Endo-anal ultrasonography abnormalities.

CORRECT ANSWER: C REFERENCE


Camilleri M et al. Opioids in gastroenterol-
RATIONALE ogy: treating adverse effects and creating
The radiopaque marker study is abnormal when therapeutic benefits. Clin Gastroenterol Hepatol
more than five markers retained in the colon on 2017;15:1338-49
day five after ingestion. However, most of the
retained markers in this patient were in the recto-
sigmoid area, suggesting some level of colonic Question 40
outlet obstruction. The next step is to perform Which of the following statements is true about
high resolution ano-rectal manometry to assess the Migratory Motor Complex (MMC).
for ano-rectal function abnormalities such as dys-
synergic defecation. A. Repeats every 75-90 minutes during digestive
periods
REFERENCE B. Clears residual chyme from the proximal
Kim CR et al. How to interpret a functional or a stomach down to the cecum
motility test-colon transit study. J Neurogastroen- C. It is partially regulated by the hormone motilin
terol Motil 2012;18:94-9. D. It correlates with phase I and II small bowel
contractile activity

Question 39 CORRECT ANSWER: C


A 42-year-old male on chronic opiates for history
of old spinal injury was seen for dysphagia. The RATIONALE
patient reports having dysphagia for solids and The MMC repeats every 75-90 minutes during the
liquids for the last one year without anorexia or inter-digestive periods (non-fed). It clears residual
weight loss. Patient denies symptoms of heart- chyme from the distal stomach or proximal small
burn or regurgitation. A recent upper endoscopy bowel to the cecum, sweeping throughout the
was unrevealing. A high resolution esophageal small bowel. It is partially regulated by Motilin
manometry was ordered. Which of the following and it is composed from four phases.
manometric findings have been shown to be as-
sociated with chronic opiate use? REFERENCE
Deloose E et al. The migrating motor complex: con-
A. Type 1 achalasia trol mechanisms and its role in health and disease.
B. Ineffective esophageal motility Nat Rev Gastroenterol Hepatol 2012;9:271-85.
C. Esophagogastric junction outflow obstruction
D. Absent contractibility
Question 41
CORRECT ANSWER: C An 18-year-old male with life long history of con-
stipation who is chronically treated with combi-
RATIONALE nation laxatives, was seen for further evaluation.
Recent studies recognized the role of medications A recent colonoscopy with random biopsies was
in inducing esophageal motor disorders. Opiates unremarkable. The patient has no other underly-
280 Digestive Diseases Self-Education Program®

ing comorbidities. Hirschsprung disease is sus- CORRECT ANSWER: C


pected and the patient was referred for anorectal
manometry. RATIONALE
Idiopathic gastroparesis is the most common cause
Which of the following findings is suggestive of of gastroparesis. There is no correlation between se-
Hirschsprung disease? verity of symptoms and degree of gastric emptying.
Overall, early satiety and post prandial fullness cor-
A. Abnormally low anal squeeze and short e relate better with gastric emptying than abdominal
ndurance squeeze pain and bloating. In general, scintigraphic gastric
B. Only five percent relaxation during bearing emptying of solids should not be done in diabetic
down simulating defecation patients with blood glucose greater than 275 mg/dl.
C. Abnormally low anal sphincter basal pressure
D. Lack of internal sphincter relaxation in REFERENCE
response to rectal balloon distension Camilleri M et al. Clinical guideline: Management of
gastroparesis. AM J Gastroenterol 2013;108:18-37.
CORRECT ANSWER: D

RATIONALE Question 43
Anorectal manometry may help in diagnos- A 60-year-old female with history of idiopathic
ing Hirschsprung disease, especially in patients gastroparesis was seen in GI clinic, reporting
with ultrashort segment. The presence of normal symptoms of epigastric discomfort, post prandial
recto-anal inhibitory reflex (RAIR) excludes the fullness and nausea despite lifestyle modifications,
disease and lack of it suggests the presence of metoclopramide 10 mg TID and ondansetron PRN
Hirschsprung disease (positive predictive value of for the last two years. During her clinic visit the
up to 95 percent). physician noted that the patient repeatedly licking
and smacking her lips.
REFERENCE
Heuckeroth RO Hirschsprung disease – integrat- Which of the following is true about this
ing basic science and clinical Medicine to im- phenomenon?
prove outcomes. Nat Rev Gastroenterol Hepatol
2018;15:152-167. A. Drug holidays may decrease the prevalence of
this side effect
B. Cessation of metoclopramide will result in
Question 42 resolution of this phenomenon
Which of the following statements about gastropa- C. It is more common in younger patients with
resis is accurate? greater than three months consumption
D. Adding a PPI to metoclopramide increases the
A. Diabetes is currently the most common cause. risk by two-fold for this adverse event
B. Overall, severity of symptoms has been
correlated with degree of gastric emptying. CORRECT ANSWER: A
C. Early satiety and postprandial fullness.
correlate better with gastric emptying than RATIONALE
abdominal pain and bloating. The patient demonstrates tardive dyskinesia due
D. Scintigraphic gastric emptying of solids to long term use of metoclopramide. The risk of
should not be done in a diabetic patient with developing tardive dyskinesia increases with the
blood glucose of 190 mg/dl. duration of treatment and the total cumulative
Chapter 10 — Gastrointestinal motility 281

dose. Cessation of metoclopramide may not always Harvey MA et al. Obstetrical Anal Sphincter Inju-
result in the resolution of tardive dyskinesia. Con- ries (OASIS): Prevention, Recognition, and Repair
comitant use of PPI’s does not increase the risk of J Obstet Gynaecol Can 2015;37(12):1131–1148.
metoclopramide induced tardive dyskinesia.

REFERENCE Question 45
Rao SS et al. Metoclopramide and tardive dyskine- You were called to see a 74-year-old male, status
sia Aliment Pharmacol Ther 2010;31:11-19. post hip replacement, who developed abdominal
distension. The patient is alert and oriented but
reports some abdominal discomfort. On physical
Question 44 examination the abdomen is distended, tympanic
A 54-year-old female with history of mixed con- but positive bowel sounds. Abdominal CT revealed
nective tissue disorder (lupus and scleroderma) proximal colonic dilation (11cm) with an intermedi-
was seen because of fecal incontinence for the last ate transitional zone adjacent to the splenic flexure.
eight months. The patient reports finding smears
and sometimes fecal particles on her underwear, What would be your first therapeutic intervention?
especially during nighttime. The patient had two
normal deliveries that required episiotomies. An A. Immediately administer neostigmine 2 mg by
anorectal manometry was ordered. slow intravenous injection
B. Surgical cecostomy
Which of the following manometric abnormalities C. Nasogastric tube attached to suction
do you expect to find in this patient? D. Colonoscopic decompression

A. Abnormally low squeeze CORRECT ANSWER: C


B. Abnormally low anal resting pressure
C. Absent rectoanal inhibitory reflex (RAIR) RATIONALE
D. Abnormally low endurance squeeze The patient developed acute colonic pseudo-ob-
struction which is seen in hospitalized, post-sur-
CORRECT ANSWER: B gery patients. Initial management of acute colonic
pseudo-obstruction is conservative in patients that
RATIONALE appear stable and with cecal diameter greater than
The effect of scleroderma is solely on the smooth 12 cm. Nasogastric tube attached to suction can
muscle of the GI tract. The internal anal sphincter, help to decompress the gastrointestinal tract.
which is composed from smooth muscle, contributes
the most to the anal resting pressure. Consequently, REFERENCE
scleroderma involvement of the anorectum will lead Chudzinski AP et al. Acute colonic pseudo-ob-
to low anal resting pressure, but will spare the exter- struction Clin Colon Rectal Surg 2015;28:112-117.
nal sphincter, which is made from striated muscle.
Perineal trauma that can lead to fecal incontinence
occurs either spontaneously with vaginal delivery or Question 46
secondarily as an extension to an episiotomy. A 27-year-old female who underwent gastric
bypass two weeks prior, reports having weak-
REFERENCE ness, malaise, crampy abdominal pain, diarrhea,
Lee TH et al. How to perform and interpret high- nausea and palpitations within 15 minutes after
resolution anorectal manometry test. J Neurogas- eating a carbohydrate rich meal. What is your
troenteral Motil 2016;22:46-59. suspected diagnosis.
282 Digestive Diseases Self-Education Program®

A. Short bowel syndrome the small bowel and the colon. In addition, the
B. Candy cane Roux syndrome frequency changes from one section of the small
C. Dumping syndrome bowel or colon to the other.
D. Stomal ulcer
REFERENCE
CORRECT ANSWER: C Blair PJ et al. The significance of interstitial cells
in Neurogastroenterology. J Neurogastroenterol
RATIONALE Motil 2014;20:294-317.
Early dumping syndrome is a complication seen
in up to 50 percent of post-gastric bypass pa-
tients especially after consuming food that are Question 48
rich with high levels of simple carbohydrates. The A 54-year-old male reported dysphagia for both
syndrome is the result of rapid emptying of this solids and liquids for the last three years that
type of food into the small bowel. The hyper- progressively worsened. The patient also reports,
osmolar content of the small bowel results in a regurgitation of food particles, heartburn, epi-
rapid shift of fluids from the plasma and into the sodes of severe chest discomfort and the need to
small bowel lumen leading to hypotension and sleep in a chair. An upper endoscopy revealed
sympathetic response. puckered esophagogastric junction with resistance
when traversing to the stomach. High resolution
REFERENCE esophageal manometry revealed an integrated
Neilsen JB et al. Prevalence, severity and predic- relaxation pressure of 34 mmHg, aperistalsis with
tors of symptoms of dumping and hypoglycemia 40 percent simultaneous contractions (see figure
after Roux-eny gastric bypass. Surg Obes Relat Dis on the following page).
2016;12:1562-68.
Which of the following interventions would you
recommend to this patient?
Question 47
Which of the following statements is true about A. Pneumatic dilation
slow waves or basic electrical rhythm (BER)? B. Botulinum toxin injection
C. Peroral endoscopic myotomy (POEM)
A. The source is a pacemaker on the greater D. Sildenafil 50 mg
curvature of the stomach
B. Their frequency throughout the GI tract is CORRECT ANSWER: C
three per minute
C. They are generated by the interstitial cells RATIONALE
of Cajal (ICC) The high resolution esophageal manometry is
D. They are absent in the colon consistent with achalasia type 3. Treatment of this
type of achalasia with pneumatic dilation, botu-
CORRECT ANSWER: C linum toxin or medications have been relatively
disappointing. In the last few years, POEM has
RATIONALE emerged as the leading and most efficacious thera-
The source of the slow wave or basic electrical peutic approach to achalasia type 3.
rhythm (BER) is the interstitial cells of Cajal,
which are spread throughout the gastrointestinal REFERENCE
tract from the esophagus down the rectum. The Patel DA et al. An overview of achalasia and its sub-
frequency of BER changes from the stomach to types. Gastroenterol Hepatol 2017;13:411-421.
Chapter 10 — Gastrointestinal motility 283

Question 49 REFERENCE
A 54-year-old female with known scleroderma Crowell MD et al. Esophageal motor abnormalities
was seen in GI clinic for severe GERD-related in patients with scleroderma: Heterogeneity, risk
symptoms and dysphagia for solids and liquids. factors and effects on quality of life. Clin Gastro-
Her GERD required double dose PPI for full enterol Hepatol 2017;15:207-213.
symptom control, but the dysphagia remained
unchanged. An upper endoscopy on PPI treatment
was unremarkable except a three cm hiatal hernia. Question 50
The patient underwent high resolution esophageal A 42-year-old female with history of documented
manometry revealing integrated relaxation pres- Grade C erosive esophagitis on an upper endosco-
sure of 5 mmHg, 90 percent failed swallows and py, was recently seen because of continued heart-
10 percent weak swallows. What is the diagnosis? burn despite double dose PPI. A recent repeat
upper endoscopy with biopsies on PPI treatment
A. Absent peristalsis was unremarkable. The patient reports almost
B. Ineffective esophageal motility daily symptoms, primarily during daytime. She
C. Achalasia type 1 has tried antacids and over the counter H2 block-
D. Fragmented peristalsis ers with no improvement. An impedance +pH test
on PPI treatment revealed normal esophageal acid
CORRECT ANSWER: B exposure and normal nonacidic reflux. Symptom
index and symptom association probability were
RATIONALE 50 percent and 98 percent for weakly acidic reflux
The patient’s high resolution esophageal manometry and heartburn, respectively. High resolution
is consistent with ineffective esophageal motil- esophageal manometry was unremarkable. What
ity. Scleroderma patients will demonstrate various is your diagnosis?
esophageal motor abnormalities which may progress
overtime and become more severe. The question A. Nonerosive reflux disease (NERD)
also demonstrates the wide range of esophageal mo- B. Reflux hypersensitivity overlapping with
tor changes that fall under the category of ineffective GERD
esophageal motility (from 50 percent weak swallows C. Functional heartburn overlapping with GERD
to 90 percent failed swallows). D. Hypersensitive esophagus
284 Digestive Diseases Self-Education Program®

CORRECT ANSWER: B tric pain syndrome and postprandial distress


syndrome. Functional dyspepsia is defined as
RATIONALE chronic, recurrent symptoms including postpran-
In a patient with history of documented GERD, dial fullness, early satiation, epigastric pain and
incomplete symptomatic response to double dose epigastric burning with a negative routine evalu-
PPI and positive symptom indexes on impedance ation. Postprandial distress syndrome primarily
+pH, the diagnosis is esophageal hypersensitiv- refers to postprandial fullness and early satiety
ity overlapping with GERD. The Rome 4 criteria occurring at least once a week with a negative
recognized for the first time the possibility of an clinical and diagnostic evaluation. Epigastric
overlap between GERD and functional esophageal pain syndrome (EPS) refers to epigastric pain or
disorders, especially functional heartburn and burning significant enough to affect a person’s
reflux hypersensitivity. daily routine with a negative clinical and diagnos-
tic evaluation. Anti-secretory drugs are first line
REFERENCE treatment for EPS followed by antidepressants
Aziz Q et al. Functional esophageal disorders Gas- then additional functional testing if incomplete
troenterology 2016;50016-5085(16)00178-5. relief with those treatments.

REFERENCE
Question 51 Stanghellini et al. Gastroduodenal disorders. Gas-
A 28-year-old woman presents for consultation re- troenterology. 2016;150:1380-92.
garding a longstanding history of abdominal pain,
described as epigastric in location, with associ-
ated symptoms of bloating, belching and nausea, Question 52
oftentimes precipitated by foods but present even A 35-year-old woman is referred to gastroenterol-
when fasting, and often severe enough for her to ogy clinic due to longstanding constipation. She
miss multiple days of work. She denies vomiting, describes her stools as lumpy to hard, with fre-
fevers, heartburn, NSAID use, relationship with quent straining, and reports “there is always more
bowel movements, or pregnancy. Her extensive stool that {she} can’t push out.” She feels that
evaluation including bloodwork, pancreaticobiliary she has a blockage in her colon as she can rarely
imaging, upper endoscopy and small bowel imaging have a bowel movement without manually disim-
have been normal. She has taken multiple courses pacting herself, she denies significant bloating or
of antacids, antibiotics, and antispasmodics with- abdominal pain. Her rectal exam reveals normal
out relief. Based on the Rome IV criteria, which of sphincter tone, no palpable masses, but hard stool
the following is the most likely diagnosis? is felt on the tip of the finger. Laboratory evalua-
tion and colonoscopy have been unremarkable.
A. Epigastric pain syndrome
B. Postprandial distress syndrome Which of the following diagnoses is most consis-
C. Rumination syndrome tent with her symptoms?
D. Excessive gastric belching
E. Chronic nausea and vomiting syndrome A. Hirschprung’s disease
B. Colonic pseudo-obstruction
CORRECT ANSWER: A C. Functional constipation
D. Irritable bowel syndrome
RATIONALE E. Anorectal stricture
The Rome IV criteria have subcategorized func-
tional dyspepsia into two conditions – Epigas- CORRECT ANSWER: C
Chapter 10 — Gastrointestinal motility 285

RATIONALE rectal stiffness are associated with urge inconti-


The key is to recognize the Rome IV criteria dif- nence, not passive or nocturnal incontinence.
ferentiating Irritable bowel syndrome with con-
stipation and functional constipation. Functional REFERENCES
constipation is more common among older per- Carrington EV, Scott SM, Charucha A, et al.
sons, women, and persons with restricted calorie Advances in the evaluation of anorectal function.
diets. The criteria for diagnosis include symp- Nature Reviews Gastroenterology and Hepatology.
toms of straining during defecation, Bristol 1-2 2018; 15: 309-323.
stools, incomplete defecation, obstructive sensa- Sun WM, Katsinelos P Horowitz M, read NW.
tion at the anorectal junction, need for maneu- Disturbances in anorectal function in patients
vers to facilitate bowel movements – at least two with diabetes mellitus and faecal incontinence.
of these symptoms should be present more than Eur J Gastroenterol Hepatol 1996; 8: 1007-1012.
25 percent of the time including less than three
spontaneous bowel movements weekly. Laxative
use is necessary for most cases but bloating and Question 54
abdominal pain are not predominant features A 41-year-old female comes for evaluation of
of functional constipation which differentiates constipation. She reports infrequent bowel move-
it from irritable bowel syndrome. The first line ments twice per day. She spends a long period
treatment is fiber, then osmotic laxative, then of time straining and struggling to defecate. She
pro-secratogogue agents. has no family history of colon cancer. She has no
blood in the stool or weight loss. The most helpful
REFERENCE testing to order is:
Lacy et al. Bowel disorders. Gastroenterology.
2016;150:1393-1407. A. Abdominal x-ray to assess stool burden
B. Colonoscopy
C. CBC
Question 53 D. Anal rectal manometry
A 56-year-old female with long standing E. CT scan
diabetes mellitus complicated by peripheral
neuropathy and chronic renal insufficiency comes CORRECT ANSWER: D
for evaluation of diarrhea. Upon questioning,
she reports that she has nocturnal fecal RATIONALE
incontinence. Nocturnal leakage suggests which The patient has constipation, and does not have
of the following: any alarm signs or symptoms that would neces-
sitate colonoscopy. While abdominal x-ray will
A. Decreased anal squeeze pressure likely show a large fecal burden, it will not help
B. Decreased anal squeeze duration in management. There is no indication that this
C. Rectal sensitivity from rectal stiffness patient is anemic given lack of overt bleeding.
D. Weakness of the internal anal sphincter Anal rectal manometry is most likely to define the
physiologic features and direct management.
CORRECT ANSWER: D
REFERENCE
RATIONALE Rao SS, Ozturk R, Laine L. Clinical utility of
Diabetic neuropathy can lead to anal sphincter diagnostic tests for constipation in adults: a sys-
weakness. Decreased anal squeeze pressure, anal tematic review. Am J Gastroenterol. 2005; 100:
squeeze duration, and rectal hypersensitivity from 1605-1615.
286 Digestive Diseases Self-Education Program®

Question 55 C. Anorectal manometry


A 58-year-old has presented with infrequent stools D. Digital rectal exam
and straining, and recently underwent anorectal E. Endoanal ultrasound
manometry testing. Testing revealed dyssynergic
defecation by anal rectal manometry. The most CORRECT ANSWER: A
effective next step in treatment is:
RATIONALE
A. Injection of Botulinum toxin into the Given the change in bowel habits, colonoscopy in
puborectalis muscles indicated to evaluate for inflammation. Anorec-
B. Treatment with polyethylene glycol daily tal manometry is helpful in evaluating sphincter
C. Biofeedback function. Endoanal ultrasound can identify anal
D. Treatment with linaclotide sphincter defects in the internal or external anal
E. Rectal surgery sphincter. Digital rectal exam is important in
evaluating the anal area for skin tags, fissures, or
CORRECT ANSWER: C scare. Digital exam can evaluate for resting anal
sphincter tone and squeeze, pelvic floor descent
RATIONALE and strength of the pelvic floor muscles. CT is
Biofeedback with visual or auditory feedback on unlikely to contribute to the evaluation of a func-
muscle activity is effective in retraining the pelvic tional disorder.
floor. Botulinum toxin injection can be considered
for spastic pelvic floor muscles, but evidence is REFERENCE
lacking at this time. While laxatives such as poly- Bharucha AE, Wald A, Enck P, Rao S. Functional
ethylene glycol and linaclotide may be partially anorectal disorders. Gastroenterology 2006; 130:
helpful, but are not targeting the main disorder. 1510-1518.
Rectal surgery is only considered for refractory
cases with significant rectocele.

REFERENCE
Rao SS, Seaton K, Miller M, et al. Randomized
controlled trial of biofeedback, sham feedback,
and standard therapy for dyssynergic defecation.
Clin Gastroenterol Hepatol. 2007; 5: 331-338.

Question 56
A 63-year-old female presents with a recent
change in bowel habits. She previously had one
formed bowel movement a day, but now has diar-
rhea three to four times a day with incontinence.
She had prior normal colonoscopy three years ago.

Which of the following tests is not indicated as


part of this evaluation?

A. CT scan
B. Colonoscopy to evaluate for inflammation
Answers & critiques

CHAPTER 11

GI infections of the
small intestine and colon
Tauseef Ali, MD, FACP, FACG, AGAF and John C. Sun, MD

Question 1 matosis) and blood transfusions, because iron


A 45-year-old man presents to the clinic with likely promotes virulence of this organism. The
worsening right lower quadrant pain and diarrhea incubation period for yersiniosis is typically four
for the last two days. His past medical history is to six days. Clinical manifestations of acute yer-
significant for hemochromatosis and he under- siniosis include diarrhea, abdominal pain, and
goes regular therapeutic phlebotomies. He admits fever; nausea and vomiting may also occur. Lo-
to dining out in a newly-opened restaurant in his calization of abdominal pain to the right lower
town four days ago. He describes having five non- quadrant is also a diagnostic clue for yersiniosis.
bloody watery stools and also has been experienc- However, both Yersinia and Campylobacter can
ing sore throat for the last two days. His physical present with right lower quadrant pain that may
examination is unremarkable except some mild be confused as appendicitis (pseudo appendi-
abdominal tenderness at right lower quadrant. citis). Another diagnostic clue is pharyngitis,
There was no rebound tenderness. Laboratory which may be an accompanying symptom.
data shows mild leukocytosis. Yersinia causes diarrhea through penetration
of the mucosa and proliferation in the submu-
What is the most likely organism causing the cosa. Pathogenic Y. enterocolitica pass through
symptoms of diarrhea and abdominal pain? the stomach, adhere to gut epithelial cells,
invade the gut wall, localize in lymphoid tissue
A. Yersinia enterocolitica within the gut wall and in regional mesenteric
B. Vibrio cholerae lymph nodes, and evade the host’s cell mediated
C. Enterotoxigenic Escherichia coli immune response. Vibrio cholerae and entero-
D. Campylobacter jejuni toxigenic E. coli (ETEC)secrete enterotoxins that
E. Shigella sonnei stimulate secretion and/or impair absorption.
Some bacteria produce toxins in contaminated
CORRECT ANSWER: A food; when ingested, the toxins cause acute
symptoms, usually nausea and vomiting. Ex-
RATIONALE amples of these are Staphylococcus aureus and
This is an example of Yersinia infection. Trans- Bacillus cereus. Enteropathogenic E. coli (EPEC)
mission of yersiniosis is largely foodborne. and enterohemorrhagic E. coli (EHEC) adhere
Risk factors associated with yersiniosis include to the intestinal mucosa, where they attach and
consumption of undercooked or raw pork cause effacement of the microvilli. Shigella,
products and exposure to untreated water. Y. enteroinvasive E. coli, and Campylobacter jejuni
enterocolitica infection has also been associated penetrate the mucosa, spread, and cause muco-
with iron-overload states (such as hemochro- sal damage with erosions and ulcers.

287
288 Digestive Diseases Self-Education Program®

REFERENCE liver abscesses. Clinical manifestations of fascio-


Cover TL, Aber RC. Yersinia enterocolitica. liasis include fever, right upper quadrant pain,
The New England journal of medicine. Jul 6 hepatomegaly, and occasional jaundice. Fascioliasis
1989;321(1):16-24. may be distinguished from infection due to other
liver flukes by the presence of hypodense lesions
on CT imaging as well as by stool microscopy and
Question 2 serology. Clinical manifestations of schistosomiasis
45-year-old Korean man presents to the clinic with include signs of portal hypertension, abnormal liver
worsening right upper quadrant abdominal pain function tests, and eosinophilia. The diagnosis is
and generalized malaise. He also reports losing established by serology and/or microscopy. Stron-
10 pounds over the last six months. He visited his gyloidiasis presents with abdominal pain, diar-
family in South Korea about eight months ago. He rhea, anorexia, nausea, vomiting, and eosinophilia;
also reports having non-bloody diarrhea for the strongyloidiasis may present with eosinophilia in
last one month. On physical examination, his vital the absence of other symptoms.
signs are stable. He has mild tenderness in the
right upper quadrant. His laboratory investigations REFERENCES
revealed hemoglobin 10 g/dL, white blood cells Qian MB, Utzinger J, Keiser J, Zhou XN. Clonor-
4.8K/ 4.2x109/L with 1500/μL eosinophils. His ALT chiasis. Lancet 2016; 387:800.
was 35 U/L, AST 35 U/L, Alkaline phosphatase 192 Lai DH, Hong XK, Su BX, et al. Current status
U/L and total bilirubin 1.2 mg/dL. Ultrasound of of Clonorchis sinensis and clonorchiasis in China.
the abdomen reveals a distended gallbladder and Trans R Soc Trop Med Hyg 2016; 110:21.
dilated common bile duct with non-shadowing
echogenic foci.
Question 3
Which of the following is most likely cause of his A 54-year-old man presented to the emergency
infection? department with worsening nausea, vomiting,
diarrhea and high-grade fever. His symptoms
A. Clonorchis sinensis started few days after attending a dinner party. He
B. Fasciola hepatica underwent renal transplant five years ago and had
C. Shistosomiasis japonicum been taking prednisone, mycophenolate mofetil
D. Strongyloides stercoralis and tacrolimus. On physical examination, his
blood pressure was 110/70 mmHg, pulse 110/min,
CORRECT ANSWER: A respiratory rate 16/min and temperature 104℉.
He was confused and had tremors in upper ex-
RATIONALE tremities. Abdominal examination revealed mod-
This case represents an example of Clonorchissi- erated tenderness in the periumbilical area. Soon
nensis (or Opisthorchissinensis) infection, which is after admission to the emergency department, he
also known as the Chinese liver fluke. It is endemic developed seizures and lost consciousness, and
in the Far East, particularly in China, Japan, Tai- was intubated to protect his airway. Routine labs
wan, Vietnam, and Korea. Symptoms can include were sent for investigations, and both CT scan
fatigue, abdominal discomfort, anorexia, weight head and lumbar puncture were performed. CT
loss, dyspepsia, and diarrhea. The gallbladder is scan was negative for any acute findings, and CSF
often nonfunctional and enlarged. Dead parasites fluid analysis revealed increased white blood cells,
or ova can serve as a nidus for stone formation. mainly lymphocytes, and low glucose. Which of
Severe disease can result in obstructive jaundice, the following organisms is the most likely cause of
pancreatitis, recurrent cholangitis, and pyogenic this illness?
Chapter 11 — GI infections of the small intestine and colon 289

A. Enterovirus casionally has dry cough and shortness of breath.


B. Listeria monocytogenes She has history of rheumatoid arthritis and has
C. Mycobacterium tuberculosis been on prednisone and methotrexate for several
D. Neisseria meningitidis years. Physical examination, including vital signs
E. West Nile virus are unremarkable. Laboratory tests are significant
for a microcytic anemia and peripheral eosinophil-
CORRECT ANSWER: B ia. EGD shows congested duodenal mucosa and
erythematous spots. Stools studies are negative
RATIONALE for any ova and parasite. With a high suspicion for
This case highlights a classic presentation of Lis- parasitic infection, intestinal aspiration is per-
teria infection in an immunocompromised pa- formed and reveals filariform larvae. Which of the
tient. Febrile gastroenteritis secondary to Listeria following is the most likely diagnosis?
infection typically occurs after ingestion of a large
inoculum of bacteria from contaminated food. The A. Ascaris lumbricoides
clinical presentation of Listeria meningoencephali- B. Cryptosporidium parvum
tis ranges from a mild illness with fever and mental C. Escherichia coli
status changes to a fulminant course with coma. A D. Giardia duodenalis
subacute presentation with cranial nerve palsies, E. Strongyloides stercoralis
lymphocytic pleocytosis, elevated cerebrospinal
fluid (CSF) protein and low glucose may mimic CORRECT ANSWER: E
tuberculous or fungal meningitis. Listeria is the
one cause of bacterial (non-tuberculous) meningi- RATIONALE
tis in which a substantial number of lymphocytes The patient has strongyloidiasis caused by Stron-
(greater than 25 percent) can be seen in the CSF gyloides stercoralis. Strongyloidiasis is endemic in
differential count in the absence of antibiotic ther- tropical and subtropical regions and occurs spo-
apy. All other infections listed present with differ- radically in temperate areas. The highest rates of
ent clinical symptoms and signs. Tuberculosis has infection in the United States are among residents
more chronic course. Viral infections usually don’t of the southeastern state and among individuals
cause gastrointestinal disturbances as presented who have been in endemic areas (including im-
in this case. Enterovirus meningitis typically has migrants, refugees, travelers, and military person-
pulmonary involvement. West Nile virus is char- nel). The most common manifestations are mild
acterized by fever, headache, malaise, back pain, waxing and waning gastrointestinal, cutaneous, or
myalgia and anorexia and a maculopapular rash pulmonary symptoms that persist for years; others
appears in approximately one-half of patients. simply have eosinophilia in the absence of symp-
toms. Standard stool examination is notoriously
REFERENCE insensitive for detecting Strongyloides. Aspiration
Ooi ST, Lorber B. Gastroenteritis due to Listeria of duodenojejunal fluid or the use of a string test
monocytogenes. Clin Infect Dis. 2005;40(9):1327 -32. (Enterotest) is sometimes used to detect Stron-
gyloides larvae in patients with negative stool
samples. Rest of the infectious agents listed don’t
Question 4 produce such typical symptoms.
A 53-year-old woman presents with waxing and
waning symptoms of right upper quadrant and REFERENCE
epigastric pain, nausea and vomiting. She also Greiner K, Bettencourt J, Semolic C. Strongyloi-
notices a skin rash that spontaneously occurs as diasis: a review and update by case example. Clin
liner red streaks on her legs and buttocks. She oc- Lab Sci 2008; 21:82-8.
290 Digestive Diseases Self-Education Program®

Question 5 count is less than 100 cells/microL. The diagnosis


A 43-year-old man presents to the clinic with his- is generally based upon stool microscopy. With a
tory of watery diarrhea, right upper quadrant ab- modified acid-fast stain, the oocysts stain red or
dominal pain and weight loss of four weeks dura- pink and are usually four to six microns in diam-
tion. He is HIV positive and last his CD4 count is eter. Microsporidium E. bieneusi is diagnosed
50 cells/microL. He denies any recent travelling or with modified trichome stain. Cyclospora cayeta-
contact with sick person. He admits being sexually nensis oocysts (diameter eight to 10 microns) can
active with multiple male partners. Physical ex- be detected by modified acid-fast staining of stool,
amination, including vital signs, is unremarkable. in which oocysts can appear light pink to deep
Ultrasound of the abdomen shows a thickened purple. Giardia has easily identifiable flagellated
gallbladder and mildly dilated common bile duct trophozites in the stool microscopy.
without any stones. Stool examination is negative
for leukocytes but microscopy reveals small pink REFERENCE
colored oocytes with modified acid-fast stain. Wumba R, Longo-Mbenza B, Menotti J, et al.
Epidemiology, clinical, immune, and molecular
Laboratory studies: profiles of microsporidiosis and cryptosporidiosis
among HIV/AIDS patients. International journal of
Hemoglobin 14 g/dL (140 g/L) general medicine. 2012;5:603-611.
Leukocyte count 6000/µL (6x109/L)
Alanine aminotransferase 35 U/L
Aspartate aminotransferase 25 U/L Question 6
Alkaline phosphatase 110 U/L A 34-year-old woman presents to the emergency
Bilirubin, total 1.0 mg/dL department with complaints of nausea, vomiting
(17.1 µmol/L) and moderately severe epigastric pain that began
Serum albumin 3.1 g/dL (31 g/L) a few hours after dinner. She is otherwise healthy,
has no significant medical history, and reports
Which of the following is the most likely diagnosis? no recent sick contacts or travel. Her friends just
returned from an Alaska fishing trip and invited
A. Cryptosporidium parvum her to eat some raw salmon that they caught a
B. Cyclospora cayetanensis day ago. During dinner, she noticed a tingling
C. Enterocytozoon bieneusi sensation in the back of her throat but reports no
D. Giardia duodenalis cutaneous or neurologic symptoms. A CT scan of
the abdomen revealed a thickened gastric body.
CORRECT ANSWER: A What is the most likely cause of this patient’s
symptoms?
RATIONALE
The patient is infected with Cryptosporidium. A. Scromboid fish poisoning
The prevalence of this infection is much higher in B. Ciguatera fish poisoning
patients with HIV infection. Transmission occurs C. Diphyllobothrium latum infection
via spread from an infected person or animal, or D. Anisakis infection
from a fecally-contaminated environment such E. S. aureus preformed toxin ingestion
as a food or water source. Cryptosporidium can
cause an asymptomatic infection but in immuno- CORRECT ANSWER: D
compromised hosts (particularly those with T-cell
immunodeficiency), the illness is more frequently RATIONALE
protracted and severe particularly when the CD4 This patient most likely has anisakiasis from eating
Chapter 11 — GI infections of the small intestine and colon 291

improperly frozen raw salmon. Anisakis species reports a history of travel to Southeast Asia five
are parasitic nematodes found in different types of months ago, where he participated in volunteer
seafood, including salmon, herring, cod, mackerel missionary work in the countryside. He returned to
and squid. About 95 percent of cases of anisakiasis the United States four months ago, and developed
involve the stomach, while the remainder mainly a cough associated with mild shortness of breath
includes intestinal disease. Gastric anisakiasis which he attributed to a viral respiratory infection,
may result in epigastric pain, nausea and vomiting as it resolved shortly thereafter. He reports no
within a few hours after ingestion of the parasite. other medical conditions, and is on no other medi-
Intestinal involvement may present with colicky cations. His physical examination is significant for
abdominal pain, nausea, vomiting or obstructive moderate periumbilical tenderness. His laboratory
symptoms, usually within five days. Diagnosis of results are significant for an eosinophil count of 20
gastric anisakiasis is confirmed by visualization of percent. A CT of the abdomen and pelvis showed
the parasite during EGD, and is treated by worm small bowel dilation proximal to a whirlpool-like
extraction during upper endoscopy. The FDA mass in the mid-jejunum. What is the most likely
recommends freezing fish at a temperature below diagnosis?
-35°C for 15 hours or under 20℃ for seven days
prior to raw or semi-raw consumption. Although A. Clonorchis sinensis
rare, this condition is increasing in frequency in B. Ascaris lumbricoides
the Western hemisphere. Scromboid poisoning is C. Trichuris trichiura
due to buildup of histamine in improperly refriger- D. Cystoisospora belli
ated fish, and is associated with development of E. Taenia saginata
rashes and flushing. Ciguatera poisoning is caused
by accumulation of dinoflagellate toxins in large CORRECT ANSWER: B
predatory fish. It is associated with disturbances
of temperature, taste and paresthesias, which are RATIONALE
not present in this case. D. latum infection may Ascaris lumbricoides is prevalent in Asia, Africa,
occur with freshwater fish consumption, and typi- South America, and the Southeastern United
cally manifests as vitamin B12 deficiency. Inges- States, and transmitted via food or water contami-
tion of preformed toxin may result in nausea and nated by A. lumbricoides or A. suum eggs. In the
vomiting a few hours after exposure, but the other early stage, which occurs one to two weeks after
features of the case favor anisakiasis. infection, the larvae pass through the lungs, and re-
sult in Loeffler’s syndrome, characterized by cough-
REFERENCE ing, wheezing and shortness of breath; hemoptysis
Shimamura Y, Muwanwella N, Chandran S, et. al. or pleural effusions may also rarely occur. Gas-
Common Symptoms from an Uncommon Infection: trointestinal symptoms begin roughly two months
Gastrointestinal Anisakiasis. Canadian Journal of after infection, and may be characterized by biliary
Gastroenterology and Hepatology. Volume 2016, manifestations such as pancreatitis, acute cholecys-
Article ID 5176502, 7 pages. titis, acute cholangitis, or luminal symptoms such
as abdominal pain, nausea, vomiting, intestinal
obstruction, and acute abdomen. Diagnosis of as-
Question 7 cariasis relies on identification of the worm during
A 45-year-old man presents to the emergency endoscopy or ERCP, eggs passed in feces, or larvae
department with nausea, vomiting and abdomi- on pulmonary specimens. Treatment of ascariasis
nal pain. He states that the symptoms are worse is with albendazole 400 mg orally for one dose,
with eating, and that his abdominal pain has been mebendazole 500 mg orally for one dose, or 100 mg
increasing over the past three to four months. He twice daily for three days.
292 Digestive Diseases Self-Education Program®

Trichuris trichiura and clonorchis infections do by bacterial enzymes in improperly refrigerated


not result in Loeffler’s syndrome. Clonorchis and fish. Most cases in the United States are reported
cystoisospora infections do not cause intestinal in Hawaii, Florida and California, and involve
obstruction. Taenia saginata infections are often consumption of affected tuna, mackerel, mahi-
asymptomatic and do not typically cause intestinal mahi, sardines, herring and other fish. Onset
obstruction. of symptoms occurs about one hour after eating
the suspect fish; the patient may experience hot
REFERENCE flashes, facial flushing, hives, upper body rash,
Jourdan PM, Lamberton PH, Fenwick A, et. al. perioral paresthesias or edema, palpitations,
Soil-transmitted helminth infections. Lancet 2018; lightheadedness, nausea, vomiting and abdominal
391:252-65. pain. Symptoms typically resolve within one day,
though some patients may experience a longer
course. Supportive care and either oral or intra-
Question 8 venous administration of antihistamines may be
A 54-year-old man is seen in the clinic for a recent used to improve symptoms. Evaluation of airway
episode of nausea, vomiting and abdominal pain. patency is also important. Scromboid poisoning
He was vacationing with friends in Hawaii, and may be prevented by immediate refrigeration of
one hour after eating a local dish consisting of rice, fresh fish to below 40°C.
macaroni salad and raw tuna, he developed a head- Although ACE inhibitor induced angioedema
ache associated with facial flushing, upper body may cause facial swelling, the time course of the
rash, palpitations, nausea, vomiting and abdominal disease and associated risk factors favor scrom-
pain. His friends who ate burgers did not experi- boid poisoning. Ingestion of B. cereus or S.
ence any symptoms. He felt better the next day. aureus would not be expected to cause flushing,
He takes only lisinopril for hypertension, and has tachycardia and upper body rash. Ciguatera poi-
no known drug allergies. His physical examination soning has a less immediate onset of symptoms, is
is unremarkable. Although he has tolerated fish in associated with neurologic symptoms, and has a
the past, he did some research on the internet, and more protracted course. This patient is not likely
wonders if he has a seafood allergy. Which of the to have an allergy to seafood.
following is correct regarding this case?
REFERENCE
A. This event would have been prevented by im Hungerford JM. Scromboid poisoning: A review.
mediate and proper refrigeration of fish after Toxicon 2010; 56:231-243.
catch.
B. Ciguatera poisoning may be prevented by
thoroughly cooking all fish. Question 9
C. The patient’s symptoms are caused by A 55-year-old Chinese man is referred for an EGD
accumulation of dinoflagellate toxins in and colonoscopy for iron deficiency anemia. He
large fish. reports no significant complaints except for fatigue
D. The patient’s lisinopril should be discontinued. and occasional nausea. He has no family history
E. The patient should undergo food allergy testing. of colon cancer or other gastrointestinal diseases
or malignancies, and does not smoke tobacco or
CORRECT ANSWER: A drink alcohol. He has been told he had ulcers in
the past, but states this has been treated previously;
RATIONALE otherwise, he has no other significant past medical
This patient has scromboid poisoning, which history. On examination, you note that his con-
occurs when histidine is converted to histamine junctiva are slightly pale, but otherwise, his exami-
Chapter 11 — GI infections of the small intestine and colon 293

nation is unremarkable. His laboratory evaluation REFERENCE


reveals a CBC significant for a hemoglobin of 10.2 Hotez PJ, Brooker S, Bethony JM, et. al, Hook-
g/dL, an MCV of 71 fl, RDW of 17.2, and eosinophils worm Infection. N Engl J Med 2004; 351:799-807.
at 22percent. His iron saturation is six percent.
His EGD and colonoscopy are unremarkable.
Question 10
What is the cause of his current presentation? A 50-year-old woman and her 20-year-old daugh-
ter present to the emergency department with
A. Ancylostoma duodenale complaints of diarrhea and lightheadedness. They
B. Diphyllobothrium latum attended a dinner party, and four hours later devel-
C. Enterobius vermicularis oped diarrhea, nausea, diffuse abdominal cramping
D. Opisthorchis felineus and temperature-related paresthesias. They report
E. Ascaris lumbricoides that hot substances feel cold and vice versa, and
report an unusual metallic taste and dental discom-
CORRECT ANSWER: A fort. Physical examination revealed bradycardia in
both patients, and neurological examination reveals
RATIONALE mild ataxia. Soon after, the ED nurse reports that
Ancylostoma duodenale (Asia and Mediterranean four other patients that attended the same event
regions) and Necator americanus (North and presented with similar symptoms.
South Americas, Africa and South Pacific Islands)
cause hookworm infection. Infection begins when What is the most likely etiology?
third-stage larvae invade the host through sites of
cutaneous contact with soil or other media where A. Scromboid fish poisoning
the larvae reside. This initial infection may cause B. Gastric anisakiasis
“ground itch” which resolves in a few days. The C. Ciguatera fish poisoning
larvae then migrate through pulmonary tissue, D. perfringens toxin ingestion
and are passed to the intestine, when patients may E. S. aureus preformed toxin ingestion
experience nausea, vomiting or diarrhea. A. duo-
denale may also be transmitted via oral ingestion. CORRECT ANSWER: C
Chronic infections lead to nutritional deficiencies
such as iron deficiency anemia. The severity of RATIONALE
these manifestations is greater with higher num- These individuals likely have ciguatera fish poison-
ber of worm burden, which is positively associated ing due to ingestion of large finfish such as barracu-
with male sex and older age. Pregnant women da, morel eel, grouper, mackerel, snapper amongst
with hookworm infection may experience a greater others. These large predatory reef fish, mainly
degree of anemia. Diagnosis is made by identifica- from the Caribbean and South Pacific, accumulate
tion of hookworm eggs on stool examination, and ciguatoxins produced by dinoflagellates, Gambier-
the preferred therapy is a single dose of albenda- discus species. Symptoms vary according to region.
zole 400 mg given orally. In the Caribbean, gastrointestinal symptoms of
D. latum is a fish tapeworm that may cause abdominal cramping, diarrhea, nausea and vomit-
a macrocytic anemia due to B12 deficiency. E. ing predominate, and are followed by cardiovascu-
vermicularis, or pinworm, causes pruritis ani. lar symptoms of bradycardia and hypotension, and
Opisthorchis is a liver fluke that may cause biliary neurologic symptoms such as paresthesias, temper-
symptoms and cholangiocarcinoma. A. lumbri- ature dysesthesias, and cerebellar manifestations.
coides, although found in Asia, causes pancreati- In the South Pacific, neurologic symptoms are
cobiliary manifestations or intestinal obstruction. more evident and severe, and may result in coma.
294 Digestive Diseases Self-Education Program®

The gastrointestinal and cardiovascular symptoms B. Vancomycin 125 mg mg orally four times daily
start within one to 12 hours after exposure, and for 10 days
tend to abate within a few days, but the neurologic C. Fecal microbiota transplantation
symptoms may last for weeks. Treatment is largely D. Vancomycin 500 mg orally four times daily for
supportive, and prevention is aimed at avoiding 10 days
ingestion of reef fish. E. Fidaxomicin 200 mg orally twice daily for
Anisakiasis is caused by a parasitic nematode seven days
present in under-frozen fish. Scromboid fish
poisoning tends to resolve within 24 hours. C. CORRECT ANSWER: B
perfringens toxin ingestion causes mainly diarrhea
and abdominal pain. S. aureus preformed toxin
ingestion causes nausea, vomiting and abdominal Question 12
pain. None of the above four conditions result in The patient in the previous question received the
significant neurologic symptoms. appropriate therapy and did well. However, two
months later, he returns with similar complaints.
REFERENCE He reports five episodes of watery stools a day,
Dickey RW, Plakas SM. Ciguatera: A public health without fevers, or abdominal pain. He remains
perspective. Toxicon 2010; 56:123-136 MMWR afebrile, with a normal CBC and comprehensive
February 1, 2013. 62(04):61-65. metabolic panel. A C. difficile PCR is again posi-
tive. Which of the following is appropriate treat-
ment at this time?
Question 11
A 55-year-old man presents with diarrhea. He A. Vancomycin 125 mg orally four times daily
reports four to five loose to watery stools a day for 10 days
that began about three days ago, with mild lower B. Fidaxomicin 200 mg orally twice daily for
abdominal cramping. He denies any nausea or 30 days
vomiting, gastrointestinal bleeding, fevers, chills or C. Vancomycin 125 mg orally four times daily for
abdominal distension. He reports an initial episode 10 days, followed by rifaximin 400 mg orally
of C. difficile infection six weeks ago, presumably three times daily for 20 days
from antibiotics he received for a strep throat infec- D. Initiation of Saccharomyces boulardii therapy
tion. The initial episode was successfully treated E. Metronidazole 500 mg orally three times daily
with a 10 day course of metronidazole 500 mg for 10 days
orally three times daily, and he has been well until
recently. His past medical history is significant for CORRECT ANSWER: C
hypercholesterolemia for which he takes simv-
astatin. On examination, he is afebrile, in sinus
rhythm and normotensive. His minimally tender Question 13
in the lower abdomen. His CBC, and comprehen- The patient in the previous two questions is treated
sive metabolic panel are within normal limits. A for his recurrent CDI successfully. He returns to
C. difficile PCR sent by his primary care provider is clinic in three months for a routine follow up visit.
positive, consistent with C. difficile infection (CDI). He reports having one formed bowel movement
daily, without abdominal pain. He denies any
What is the next appropriate step in therapy? fevers, chills, nausea, vomiting, or other systemic or
gastrointestinal symptoms. He has not had any ad-
A. Metronidazole 500 mg orally three times daily ditional antimicrobial use. What is the appropriate
for 10 days management for this patient?
Chapter 11 — GI infections of the small intestine and colon 295

A. Repeat C. difficile PCR testing for confirmation In the first question, the patient presented with
of cure. his first recurrence after successful metronidazole
B. Proceed with fecal microbiota transplantation therapy. The appropriate choice is to treat his re-
to reduce risk of relapse. currence with 10 days of vancomycin 125 mg orally
C. Initiate long-term suppressive therapy with four times daily for 10 days over retreatment with
vancomycin. metronidazole. FMT is not appropriate for an ini-
D. Inform the patient that he should be empirically tial recurrence. Fidaxomicin 200 mg orally twice
treated with vancomycin 125 mg orally four daily for 10 days (not seven days as in the ques-
times daily should he require any antibiotic tion) would be appropriate if the patient received
therapy in the future. vancomycin therapy for his initial episode.
E. Monitor patient conservatively for CDI For the second question, C would be appro-
recurrence. priate. Retreatment with a standard course of
vancomycin at 125 mg orally four times a day for
CORRECT ANSWER: E 10 days, treatment with metronidazole, treatment
with a prolonged course of fidaxomicin, or Saccha-
RATIONALE romyces boulardii, would not be indicated.
Clostridium difficile infection (CDI) has a recurrence For the third question, repeat C. difficile test-
rate of up to 25 percent after the initial episode, and ing for confirmation of cure is not recommended
after the first recurrence, additional recurrence rates in either the 2013 ACG or 2017 IDSA / SHEA
range from 40-65 percent. The 2017 IDSA / SHEA guidelines. Only patients with diarrhea (2017
guidelines address treatment of recurrent CDI. IDSA / SHEA guidelines recommend three or
more unformed stools per day) should be tested
First recurrence: for C. difficile (an exception is in patients with
If metronidazole was used for the initial episode, ileus). In addition, C. difficile testing may remain
treat the first recurrence (second episode) with van- positive for a period of time, and these positive
comycin 125 mg orally four times daily for 10 days. tests in otherwise asymptomatic patients may
This is preferred over treating with a second lead to unnecessary additional CDI therapy. FMT
course of metronidazole. is recommended for the third or greater number
If vancomycin was used for the initial episode, of recurrences that do not respond to antibiotic
treat the first recurrence (second episode) with: therapy. As this patient is now asymptomatic,
Fidaxomicin 200 mg orally twice daily for 10 days, or there is no indication for FMT. There is no de-
prolonged vancomycin tapered and pulsed regimen. fined role for either long-term suppressive therapy
The IDSA / SHEA guidelines suggest vancomycin or empiric treatment for CDI in patients receiving
125 mg orally four times daily for 10-14 days, then antibiotic therapy for other indications, as there
twice daily for one week, then daily for one week, are no large scale randomized controlled trials
then every two to three days for two to eight weeks. addressing these issues. The decision to admin-
ister secondary prophylaxis for recurrent CDI in
Second or subsequent recurrence: patients receiving antibiotics should be individual-
Prolonged vancomycin tapered and pulsed regi- ized. The 2017 IDSA / SHEA guidelines suggest
men, or vancomycin 125 mg orally four times daily daily vancomycin or fidaxomicin for cases where
for 10 days, followed by rifaximin 400 mg orally secondary CDI prophylaxis may be indicated.
three times daily for 20 days, or fidaxomicin 200
mg orally twice daily for 10 days, or fecal micro- REFERENCES
biota transplantation (FMT) if the patient has McDonald LC, Gerding DN, Johnson S, et. al.
had appropriate antibiotic therapy for at least two Clinical Practice Guidelines for Clostridium diffi-
recurrences (three episodes). cile Infection in Adults and Children: 2017 Update
296 Digestive Diseases Self-Education Program®

by the Infectious Diseases Society of America REFERENCE


(IDSA) and Society for Healthcare Epidemiology Hungerford JM. Scromboid poisoning: A review.
of America (SHEA). CID 2018; 66(7):e1-e48. Toxicon 2010; 56:231-243.
Surawicz CM, Brandt LJ, Binion DG, et. al.
Guidelines for Diagnosis, Treatment, and Pre-
vention of Clostridium difficile infections. Am J Question 15
Gastroenterol 2013; 108:478-498. An 81-year-old woman was admitted from a
nursing home for a change in mental status. She
was found to have a urinary tract infection and IV
Question 14 antibiotics were started. Three days later, she de-
A 23-year-old woman with past medical history velops eight loose stools a day, fevers to 102.8°C,
significant for atopic dermatitis, asthma and and severe abdominal pain. Her blood pressure is
allergic rhinitis is seen in the clinic for a recent found to be 72/41 mm Hg, and her abdomen was
episode of nausea and vomiting. She was eating diffusely tender to palpation. She received fluid
at a sushi buffet, and developed flushing as- resuscitation, and was transferred to the ICU.
sociated with nausea, vomiting and abdominal Her labs show a white count of 18,000 cells/mL,
pain within two hours of the meal. She reported albumin of 2.8 g/dL, and her creatinine has in-
palpitations and a headache as well, but denies creased from 0.8 mg/dL to 1.6 mg/dL. Her stool
any weakness or temperature-related paresthe- C. difficile PCR is positive. Which of the following
sias. She thought this may have been due to her should be initiated at this time?
allergies, so she took diphenhydramine and went
to sleep. The next morning, her symptoms were A. Vancomycin 500 mg orally four times daily,
significantly improved. Which of the following is metronidazole 500 mg IV every eight hours
the most likely cause of her symptoms? and vancomycin 500 mg rectally every six
hours.
A. Oral allergy syndrome B. Metronidazole 500 mg orally three times daily
B. Ingestion of preformed toxin with metronidazole 500 mg IV every eight
C. Ciguatera fish poisoning hours.
D. Seafood allergy C. Vancomycin 125 mg orally four times daily,
E. Scromboid fish poisoning metronidazole 500 mg IV every eight hours
with vancomycin 500 mg rectally every six
CORRECT ANSWER: E hours.
D. Fidaxomicin 200 mg orally twice daily with
RATIONALE metronidazole 500 mg IV every eight hours
She developed scromboid poisoning from poorly and vancomycin 500 mg rectally every six
handled fish. Scromboid poisoning is hista- hours.
mine-mediated, and occurs when histidine is E. Vancomycin 500 mg orally four times daily
converted to histamine by bacterial enzymes in with vancomycin 500 mg rectally every six
improperly refrigerated fish. It is not caused by hours.
a preformed toxin. Although she has a history
of allergic and atopic conditions, her symptoms CORRECT ANSWER: A
are not consistent with oral allergy syndrome,
which occurs with fruits, vegetables and nuts, RATIONALE
and resolves within minutes. This patient is not Although there is some divergence on the defini-
likely to have an allergy to seafood. tion of severe C difficile infection (CDI) between
the 2013 ACG and 2017 IDSA / SHEA guide-
Chapter 11 — GI infections of the small intestine and colon 297

lines, this patient meets the criteria for severe REFERENCES


and complicated CDI by ACG and fulminant CDI McDonald LC, Gerding DN, Johnson S, et. al.
by IDSA guidelines. The 2013 ACG guidelines Clinical Practice Guidelines for Clostridium dif-
define the following categories: ficile Infection in Adults and Children: 2017 Up-
date by the Infectious Diseases Society of America
Mild to moderate disease - diarrhea plus any ad- (IDSA) and Society for Healthcare Epidemiology
ditional signs and symptoms not meeting severe of America (SHEA). CID 2018; 66(7):e1-e48.
or complicated criteria. Surawicz CM, Brandt LJ, Binion DG, et. al.
Guidelines for Diagnosis, Treatment, and Pre-
Severe disease - serum albumin less than 3 g/dl vention of Clostridium difficile infections. Am J
plus ONE of the following: Gastroenterol 2013; 108:478-498.
WBC greater than or equal to 15,000 cells /
mL.
Abdominal tenderness. Question 16
Severe and complicated disease - any of the fol- Which of the following statements regarding
lowing attributable to CDI: recurrent C difficile infection (CDI) is correct?
Admission to intensive care for CDI
Hypotension with or without required use of A. After a first CDI recurrence, the risk of sub-
vasopressors sequent recurrences ranges between 40-65
Fever greater than or equal to 38.5°C (101.3°F) percent.
Ileus or significant abdominal distension B. Recurrent CDI is not associated with an in-
crease in mortality as compared to the initial
Mental status changes CDI.
WBC greater than or equal to 35,000 cells / mL C. Proton pump inhibitor use is not associated
or less than 2,000 cells / mL with an increased risk of recurrent CDI.
Serum lactate levels greater than 2.2 mmol/L D. Recurrent CDI is more common in males.
End organ failure. E. Older age is associated with a lower risk of
The 2017 IDSA / SHEA guidelines include the recurrent CDI.
following categories:
Non-severe - Leukocytosis with WBC less than CORRECT ANSWER: A
or equal to 15,000 cells / mL, creatinine less
than 1.5 mg/dL. RATIONALE
Recurrent CDI is associated with a 33 percent
Severe - WBC greater than or equal to 15,00 increased risk of mortality at 180 days as com-
cells / mL, or creatinine greater than 1.5 mg/dL. pared to patients who did not have a recurrence.
Fulminant - hypotension or shock, ileus, mega- Older age, female sex, use of corticosteroids,
colon. antibiotics or proton pump inhibitors in the 90
days prior to CDI diagnosis, presence of chronic
This patient meets criteria for severe and com- kidney disease and diagnosis in a nursing home
plicated CDI by 2013 ACG and fulminant CDI are risk factors for recurrent CDI. After the ini-
by 2017 IDSA / SHEA guidelines. Treatment tial recurrence, rates of additional recurrences of
includes vancomycin 500 mg orally four times CDI range from 40-65 percent.
daily, metronidazole 500 mg IV every eight hours,
and vancomycin 500 mg rectally every six hours. REFERENCES
The ACG guidelines recommend early surgical McDonald LC, Gerding DN, Johnson S, et. al.
consultation for these patients. Clinical Practice Guidelines for Clostridium.
298 Digestive Diseases Self-Education Program®

difficile Infection in Adults and Children: 2017 Up- CORRECT ANSWER: C


date by the Infectious Diseases Society of America
(IDSA) and Society for Healthcare Epidemiology
of America (SHEA). CID 2018; 66(7):e1-e48. Question 18
Surawicz CM, Brandt LJ, Binion DG, et. al. Which of the following is true regarding shiga toxin
Guidelines for Diagnosis, Treatment, and Preven- E. coli (STEC) infections?
tion of Clostridium difficile infections. Am J Gas-
troenterol 2013; 108:478-498. A. Antibiotics should be not utilized in patients
Ma GK, Brensinger CM, Wu Q, Lewis JD. suspected or confirmed to have STEC.
Increasing Incidence of Multiply Recurrent Clos- B. Patients with STEC should receive anti-motility
tridium difficile Infection in the United States: A agents for symptomatic relief and to prevent
Cohort Study. Ann Intern Med 2017; 167(3):152. dehydration.
Epub 2017 Jul 4. C. Unlike campylobacter infections, STEC is not
Olsen MA, Yan Y, Reske KA, et. al. Recurrent associated with neurologic manifestations.
Clostridium difficile infection is associated with D. Non-steroidal anti-inflammatory drugs should
increased mortality. Clin Microbiol Infect 2015; be used instead of opioids for pain relief in pa-
21:164-170. tients with STEC.
E. Hemolytic-uremic syndrome (HUS) occurs in
less than one percent of cases of STEC.
Question 17
A 56-year-old woman was admitted for diarrhea. CORRECT ANSWER: A
She developed severe lower abdominal cramp-
ing, subjective fevers, and eight to 10 loose bowel RATIONALE
movements daily in the three days prior to admis- This patient has an infection with shiga toxin pro-
sion, and progressed to bloody stools in the last ducing E. coli (STEC). Although E. coli O157:H7 is
day. The only possible trigger may have been a the most common, there are non-O157:H7 strains
backyard barbeque she attended one week ago, that cause STEC infection, such as O104:H4. Cat-
where she ate burgers, ham and cheese sandwich- tle are a major source of transmission, and visits
es and chicken salad. On admission, she had a to cattle farms or ingestion of foods contaminated
temperature of 98.2°F, blood pressure 102/62 mm by cattle excrement may result in STEC infection.
Hg, pulse of 102 bpm, with generalized abdominal Undercooked ground beef is the most well-known
tenderness. Her labs, including a complete blood food source; other potential vehicles of transmis-
count and basic metabolic panel, were normal on sion include unpasteurized milk and apple cider,
admission. She received intravenous levofloxacin lettuce, salami, and municipal or swimming water.
in the emergency department and stool cultures The pathogenic effects of STEC are mediated by
were sent. Over the next four days, her diarrhea shiga toxin 1 and 2, including cytotoxic, renal
continued, and her creatinine increased to 3.5 mg/ (hemolytic-uremic syndrome, HUS) and neuro-
dL, with a hemoglobin of 9.7 g/dL, and a platelet logic (seizures, strokes, coma) sequelae.
count of 67,000/microL. What is the most likely Symptoms of non-bloody diarrhea typically
cause of her clinical condition? start three days after exposure to STEC, evolving
to bloody diarrhea in 90 percent of patients within
A. Aeromonas infection one to three days. Roughly one week after the
B. Campylobacter infection onset of diarrhea, patients either have spontane-
C. Shiga toxin E. coli infection ous resolution (85 percent) or develop hemolytic-
D. Non-typhoidal Salmonella infection uremic syndrome (15 percent in children, up to
E. Listeria monocytogenes infection nine percent in adults).
Chapter 11 — GI infections of the small intestine and colon 299

Management of STEC infection is supportive; and normal differential count; her B12 and folate
aggressive rehydration and monitoring of elec- levels are both low. Stool testing is negative for
trolytes, renal function, and blood counts are the routine culture, C. difficile, giardia and cryptospo-
cornerstones of therapy. Use of antibiotics, anti- ridium. Fecal fat is 12 grams in 24 hours. EGD
motility and narcotic agents have been associated with duodenal biopsies show villous blunting.
with an increased risk of HUS development, and
anti-motility or narcotic agents have been associ- What is the most likely diagnosis?
ated with increased risk of neurologic complica-
tions. There is concern that use of NSAIDs result A. Celiac disease
in decreased renal perfusion so these agents B. Diphyllobothrium latum infection
should be avoided. C. Tropical sprue
The other organisms listed in the first ques- D. Necator americanus infection
tion, listeria, campylobacter, aeromonas and E. Cystoisospora belli infection
salmonella would not cause HUS as noted in this
patient. CORRECT ANSWER: C

REFERENCES RATIONALE
Tarr PI, Gordon CA, Chandler WL. Shiga- Tropical sprue causes chronic diarrhea in patients
toxin producing Escherichia coli and haemo- either from or having traveled to endemic areas
lytic uraemic syndrome. The Lancet 2005; near the equator, such as Puerto Rico, Haiti,
365(9464):1073-1086. Cuba, Southeast Asia and India for at least two
Shane AL, Mody RK, Crump JA, et. al. 2017 weeks to a month. Although no clear etiology
Infectious Diseases Society of America Clinical has been found, data suggest a likely infectious
Practice Guidelines for the Diagnosis and Manage- etiology. Clinically, patients present with mal-
ment of Infection Diarrhea. CID 2017; 65(12):e absorption, steatorrhea, weight loss, and fatigue.
45-e80. Laboratory testing shows anemia, B12 and folate
Riddle MS, Herbert LD, Conner BA. ACG deficiency, as well as increased fecal fat. Biopsies
Clinical Guideline: Diagnosis, Treatment, and of the small bowel during upper endoscopy show
Prevention of Acute Diarrhea Infections in Adults. villous blunting with negative celiac serologies.
Am J Gastroenterol 2016; 111:602-622. Treatment is a three to six month course of tetra-
cycline 250 mg orally four times daily with folate
5 mg orally daily.
Question 19
A 22-year-old caucasian woman presents with REFERENCES
four month history of diarrhea and 20 pound Ghoshal UC, Srivastava D, Verma A, et. al. Tropi-
weight loss upon returning from a three month cal sprue in 2014: the new face of an old disease.
summer trip to Puerto Rico. She has experienced Curr Gastroenterol Rep 2014; 16(6):391-393.
increasing fatigue and shortness of breath during Batheja MJ, Leighton J, Azueta A, et. al. The
this time period, and notes increased flatulence Face of Tropical Sprue in 2010. Case Rep Gastro-
and very loose stools that seem to float. She has enterol 2010; 4:168-172.
no other significant past medical history, tobacco, Jansson-Knodell CL, Hujoel IA, Rubio-Tapia
alcohol or drug use. She states that her mother A, et. al. Not All That Flattens Villi is Celiac Dis-
has hypothyroidism, and a cousin has celiac dis- ease: A Review of Enteropathies. Mayo Clin Proc
ease. On examination, she has pale conjunctiva 2018; 93(4):509-517.
and hyperactive bowel sounds. Her CBC is signifi-
cant for a hemoglobin of 8.1 g/dL, MCV of 112.2 fl
300 Digestive Diseases Self-Education Program®

Question 20 PCR is the gold standard test, with sensitivity of


A 46-year-old male Mexican farm worker presents 92 to 100 percent, and specificity of 89 to 100
to the emergency department with a two week percent. Initial treatment is with metronidazole
history of diarrhea. He reports that the diarrhea 500 mg - 750 mg orally three times daily for 10
has been increasing, and is associated with blood, days, followed by a luminal agent such as paromo-
severe lower abdominal pain, and fevers. His past mycin 25-30 mg / kg daily in three divided doses
medical history is significant for asthma; he had for seven days. Nitazoxanide 500 mg orally twice
a flare a few weeks ago, for which he received a daily for three days is an alternative to metronida-
short course of prednisone. He has a temperature zole. Use of metronidazole or nitazoxanide alone
of 102.1 °F, and lower abdominal tenderness on is not recommended. As stool cultures for routine
examination. His laboratory results reveal a white bacteria were negative, azithromycin would not
blood cell count of 12.1K cells/mL with a normal be indicated at this time. Albendazole is typically
differential, with negative routine stool culture, test used for worm infections and is also not indicated.
for shiga toxin and C. difficile PCR. A CT scan of
the abdomen and pelvis was consistent with inflam- REFERENCES
mation in the cecum and ascending colon. Shirley DAT, Farr L, Watanabe K, et. al. A Re-
view of the Global Burden, New Diagnostics, and
What is the most appropriate treatment for this Current Therapeutics for Amebiasis. OFID 2018;
patient? 5(7):ofy161.
Skappak C, Akierman S, Belga S, et. al. Inva-
A. Metronidazole 500 mg orally three times daily sive amoebiasis: A review of Entamoeba infections
for 10 days. highlighted with case reports. Can J Gastroenterol
B. Azithromycin 500 mg orally daily for three days. Hepatol 2014; 28(7):355-359.
C. Albendazole 400 mg orally for one dose. Pritt BS, Clark C. Amebiasis. Mayo Clin Proc.
D. Metronidazole 500 mg orally three times daily 2008; 83(10):1154-1160
for 10 days, followed by paromomycin 25-30 mg / Stanley Jr SL. Amoebiasis. Lancet 2003;
kg daily in three divided doses for seven days. 361:1025-1034.
E. Nitazoxanide 500 mg orally twice daily for three
days.
Question 21
CORRECT ANSWER: D 12 patients presented to a local emergency depart-
ment with similar symptoms of severe nausea,
RATIONALE profuse vomiting, and severe abdominal cramps.
This patient most likely has amebiasis, an infec- A few patients reported diarrhea. The patients
tion with Entamoeba histolytica, prevalent in report that they were all present at a lunch party
Central and South America, Asia, Middle East two hours ago where chicken wings, pulled pork,
and Africa; Mexico has a high rate of E. histolytica string beans, and a chicken, sausage and rice dish
infection. Transmission is via the fecal-oral route were available. The dishes were served from slow
and patients who are very young, very old, preg- cookers, and the food handlers did not wear gloves.
nant, malnourished, or are using corticosteroids None of the patients were found to be febrile. The
are at risk for more severe disease. Intestinal patients were treated with supportive care and
amebiasis tends to affect the right colon, and use hydration. Within 24 hours, all the patients had
of diagnostic flexible sigmoidoscopy may result in resolution of their symptoms.
a false negative test. Colonoscopy may reveal mul-
tiple ulcers in the cecum and ascending colon, and What is the most likely cause of this disease out-
biopsies show typical flask-shaped ulcers. Stool break?
Chapter 11 — GI infections of the small intestine and colon 301

A. Salmonella infection. with a rash on his buttocks and perianal region,


B. Scromboid poisoning. followed by the development of coughing and
C. S. aureus preformed toxin ingestion. wheezing. A few weeks ago, he began having diar-
D. perfringens infection. rhea and moderately severe abdominal pain. He
E. Norovirus infection. denies any foreign travel, sick contacts, ingestion
of uncooked or undercooked foods, or family his-
CORRECT ANSWER: C tory of gastrointestinal diseases. He is febrile with
a tender abdomen, his laboratory results reveal
RATIONALE leukocytosis with eosinophilia, and a CT scan of
This is a case of S. aureus preformed toxin inges- the abdomen showed duodenal, jejunal and colonic
tion, one of the most common food-borne illnesses wall thickening. What is the most likely diagnosis?
in the United States. Food is contaminated by
enterotoxin producing S. aureus from the hands A. Cyclospora infection.
or nares of preparers, and the bacteria reproduce B. Giardia infection.
and generate toxin. The toxin is heat stable and C. Ancylostoma duodenale infection.
resistant to low pH environments. Ingestion of D. Strongyloides infection.
the toxin causes symptoms in 30 minutes to eight E. Shiga toxin producing E. Coli infection.
hours, with an average of three hours. Severe
nausea, vomiting and abdominal pain are common, CORRECT ANSWER: D
whereas fevers and diarrhea are less likely. Resolu-
tion of symptoms is usually within 24-48 hours.
Prevention can be achieved with proper handwash- Question 23
ing and refrigeration of food. Further testing of the patient in question
Scromboid poisoning occurs with fish inges- 17 confirms strongyloides infection. In addition to
tion, which was not present in this case. Norovirus supportive care and treatment of his E. coli sep-
infection usually has an incubation period of 24-48 sis, what is the most appropriate therapy for this
hours. C. perfringens infection is also toxin mediat- patient?
ed, but causes a diarrhea-predominant illness with
a mean incubation time of 12 hours. Salmonella A. Ivermectin
causes nausea, vomiting and diarrhea, but typically B. Nitazoxanide
with a longer incubation period, and is character- C. Albendazole
ized by fevers as well. D. Praziquantel
E. Metronidazole
REFERENCE
MMWR 2013; 62(50): 1025-1028. CORRECT ANSWER: A

RATIONALE
Question 22 This patient most likely has Strongyloides sterco-
A 54-year-old man from Kentucky with a history of ralis hyperinfection. S. stercoralis is a nematode
rheumatoid arthritis (RA) is admitted for hypoten- that can complete its entire life cycle within the
sion and E. coli sepsis. He reports that about four human host. The parasite may be acquired orally
months ago, he had an RA flare, and prednisone or percutaneously through the soil. In the latter
was started in addition to his methotrexate regi- case, it burrows through the skin where it leaves
men; more recently, a tumor necrosis factor inhibi- a pruritic rash (larva currens), and travels to the
tor has been added to his treatment regimen. He pulmonary capillaries. From there, the larvae
reports anal pruritis in the recent past associated migrate to the larynx and then to the small intes-
302 Digestive Diseases Self-Education Program®

tine. The pulmonary phase may cause coughing, mission trip to Southeast Asia, and developed five
wheezing, hemoptysis, and the intestinal phase to six watery, non-bloody bowel movements daily,
may be asymptomatic or result in abdominal pain, associated with stool urgency, lower abdominal
diarrhea, nausea or vomiting. Laboratory testing cramping, fatigue, anorexia and subjective fevers.
may reveal eosinophilia. The female worms lodge Due to these symptoms, she has missed work and
in the duodenal or jejunal wall and produce eggs, often could not leave her home. She has no other
which hatch into rhabditiform larvae; these can significant past medical history, is on no medica-
subsequently mature into infectious filariform tions, reports no recent antibiotic use, and has no
larvae within the bowel, and auto-infect the host family history of any gastrointestinal diseases. On
through penetration of the colonic wall or through examination, her temperature is 100.9 F and she
the perianal skin. In immunosuppressed hosts, has moderate lower abdominal tenderness. She
this cycle of autoinfection may result in hyperin- wants to know if there’s something that can be
fection syndrome, leading to a substantial parasite done so she may return to work and resume her
burden, disseminated disease, sepsis, gastrointes- regular activities. What is the most appropriate
tinal bleeding, and potentially death. Suppression management at this time?
or compromise of cell-mediated immunity such
as with HIV, corticosteroids, and use of tumor A. Metronidazole
necrosis factor-alpha inhibitors are risk factors B. Levofloxacin
for hyperinfection syndrome. An ELISA test C. Supportive care and oral rehydration
may be used for diagnosis. Stool testing may be therapy only
obtained, but it has a lower sensitivity. Endoscopy D. Azithromycin
with biopsies, skin biopsies, and bronchoalveolar E. Stool culture and sensitivity testing
lavage may also be performed. Treatment is with
ivermectin. CORRECT ANSWER: D
Cyclospora and giardia would not cause this
constellation of symptoms including pulmonary RATIONALE
findings and eosinophilia. Ancylostoma duodenale This patient has moderately severe traveler’s
may cause eosinophilia and pulmonary symptoms, diarrhea (TD), of which Enterotoxigenic E. coli is
however, Necator americanus is the type of hook- the most common etiologic agent; other causes
worm predominantly found in the United States. may include campylobacter and salmonella. Mild
This patient has E. coli sepsis as a result of S. ster- symptoms are defined as not causing change in
coralis infection, but not STEC infection per se. activities. Patients with moderate symptoms
can function, but have to curtail activities. Se-
REFERENCES vere symptoms are incapacitating and result in
Corti M. Strongyloides stercoralis in Immunosup- complete cessation of activities, or are associated
pressed Patients. Arch Clin Infect Dis 2016; 11(1): with bloody diarrhea. Studies have shown that
e27510. antibiotic therapy shortens the clinical course of
Jourdan PM, Lamberton PH, Fenwick A, et. al. TD to roughly one day, and guidelines recom-
Soil-transmitted helminth infections. Lancet 2018; mend treatment for patients with moderate or
391:252-65. severe symptoms. Although fluoroquinolones
MMWR 2013; 62(42): 843. are used for empiric therapy of TD, data suggest
that there is increased campylobacter resistance
to fluoroquinolones in Southeast Asia and the
Question 24 preferred therapy for travelers to that region is
A 32-year-old woman presents with complaints of azithromycin 1000 mg orally for one dose, or 500
diarrhea. She has just returned from a volunteer mg orally daily for three days. Metronidazole is
Chapter 11 — GI infections of the small intestine and colon 303

not recommended for empiric treatment of TD. D. Yersinia infection.


Stool culture is not routinely recommended for E. Vibrio vulnificus infection.
initial management of TD as ETEC or EAEC can-
not be distinguished from non-pathogenic E. coli. CORRECT ANSWER: B
Supportive care and rehydration is appropriate
for patients with mild symptoms, and as adjunc-
tive therapy for patients with moderate or severe Question 26
symptoms. The latter two groups should also Regarding the clinical scenario in the previous
receive empiric antibiotic therapy. question, which of the following is true?

REFERENCE A. Azithromycin should be initiated to treat


Riddle MS, DuPont HL, Connor BA. ACG Guide- Guillian-Barre Syndrome (GBS) due to Vibrio
line: Diagnosis, Treatment, and Prevention of Acute vulnificus infection.
Diarrheal Infections in Adults. Am J Gastroenterol B. Shiga-toxin producing E. coli infection may lead
2016; 111:602-622. to neurologic sequelae in up to 20 percent of
patients.
C. The neurologic symptoms associated with
Question 25 yersinia infection are due to production of a
A 46-year-old man presents with muscle weak- neurotoxin.
ness. He first noticed that he had difficulty stand- D. Up to 40 percent of cases of GBS are due to
ing from a seated position, then began to have campylobacter infection.
problems climbing stairs. A few days later, he felt E. The etiology is ciguatera fish poisoning, and is
weak when trying to lift his arms to reach items caused is by eating raw seafood or shellfish.
overhead. In the last day, he has had arthralgias
in his wrists and hands. He has been in good CORRECT ANSWER: D
health until a few weeks ago when he had a sud-
den onset of fevers, abdominal pain and bloody RATIONALE
diarrhea which lasted for six days. He denies any This patient presents with ascending muscle
paresthesias or changes in temperature sensation. weakness and reactive arthropathy after an acute
His past medical history is significant for hyper- diarrheal illness most consistent with Campylo-
tension for which he takes metoprolol. He denies bacter infection, which may be transmitted from
any tobacco or alcohol use, or any family history poultry and other meats, contaminated water, or
of gastrointestinal diseases. He works as a chef at raw milk. Acute symptoms include abdominal
a local restaurant. On examination, he is afebrile pain, fever, diarrhea (with or without blood) and
and normotensive. He has a benign abdominal nausea. Approximately two weeks after infection,
examination, but has bilateral upper and lower ex- two percent of patients may develop arthralgias,
tremity weakness. His complete blood count, liver and one in 1000 patients may develop GBS;
function tests, electrolytes, creatinine, creatine conversely, up to 40 percent of GBS cases are
kinase are all within normal limits. thought to be related to Campylobacter infections.
The mechanism is thought to be due to molecu-
What is the most likely etiology for his current lar mimicry between Campylobacter antigens
presentation? and GM1 ganglioside in peripheral nerve myelin.
Although azithromycin is the antibiotic treatment
A. Shiga toxin producing E. coli (STEC) infection. of choice for Campylobacter infections due to in-
B. Campylobacter infection. creasing resistance of fluoroquinolones, it would
C. Ciguatera fish poisoning. not treat the patient’s current GBS.
304 Digestive Diseases Self-Education Program®

Ciguatera fish poisoning is associated with been identified. Patients may present with mal-
sensory symptoms of paresthesias and changes absorption, steatorrhea, weight loss, and fatigue.
in temperature perception from ingestion of Laboratory testing shows anemia, B12 and folate
ciguatoxins produced by dinoflagellates, and not deficiency, as well as increased fecal fat. Biopsies
ascending motor weakness. Yersinia, vibrio and of the small bowel during upper endoscopy show
STEC infections are not associated with GBS. villous blunting with negative celiac serologies.
Treatment is a three to six month course of tetra-
REFERENCE cycline 250 mg orally four times daily with folate
Nachamkin I, Allos BM, Ho T. Campylobacter 5 mg orally daily. The macrocytic anemia, normal
Species and Guillian-Barre Syndrome. Clinical iron studies, and low vitamin B12 and folate levels
Microbiology Reviews 1998; 11(3):555-567. argue against celiac disease, so this patient is un-
likely to respond to a gluten-free diet.

Question 27 REFERENCES
A 36-year-old caucasian woman returned from a Ghoshal UC, Srivastava D, Verma A, et. al. Tropi-
three month missionary trip to India, and sub- cal sprue in 2014: the new face of an old disease.
sequently developed diarrhea, and a 20 pound Curr Gastroenterol Rep 2014; 16(6):391-393.
weight loss in the recent past. She reports in- Batheja MJ, Leighton J, Azueta A, et. al. The
creased abdominal bloating but denies reports fa- Face of Tropical Sprue in 2010. Case Rep Gastro-
tigue, but denies any symptoms of gastrointestinal enterol 2010; 4:168-172.
bleeding. Her CBC reveals a macrocytic anemia, Jansson-Knodell CL, Hujoel IA, Rubio-Tapia
normal iron studies, and low vitamin B12 and fo- A, et. al. Not All That Flattens Villi is Celiac Dis-
late levels. Her stool tests are negative for routine ease: A Review of Enteropathies. Mayo Clin Proc
bacterial pathogens, giardia, ova and parasites. 2018; 93(4):509-517.
Her duodenal biopsies show villous blunting.

Which of the following is true regarding this case? Question 28


A 19-year-old woman presents with a two day his-
A. Treatment is with a three to six month course of tory of diarrhea that began suddenly. She reports
tetracycline and folate. that she is having four loose to watery, non-bloody
B. The patient should abstain from consuming stools a day associated with mild abdominal pain
gluten-containing foods. and nausea. She denies any recent travel or antibi-
C. The etiology is an infection by a protozoan or- otic use, fevers or chills or vomiting. She does note
ganism. some mild muscle soreness. She lives in a college
D. This disease is also common in Northern Eu- dormitory, and tells you that two of her friends had
rope. similar symptoms but are now better. She is able to
E. The condition has an autoimmune etiology. function normally despite her symptoms. She takes
no medications, and reports no significant past
CORRECT ANSWER: A medical history or family history of gastrointestinal
diseases. On examination, she is afebrile and has
RATIONALE mild periumbilical tenderness to palpation. Her
Tropical sprue occurs in patients from or travelers laboratory results show a normal complete blood
endemic areas near the equator, such as Puerto count, creatinine, and electrolytes.
Rico, Haiti, Cuba, Southeast Asia and India for at
least two weeks to a month, and has a likely infec- What is the most appropriate management at
tious etiology, but the exact organism(s) has not this time?
Chapter 11 — GI infections of the small intestine and colon 305

A. Initiate empiric therapy with ciprofloxacin. iting and non-bloody diarrhea that lasted for three
B. Supportive care with hydration and loperamide days with spontaneous resolution. She reports that
as needed. her symptoms began shortly her two-year-old son
C. Probiotic therapy has been shown in large ran- had a similar but more severe illness, with nausea,
domized controlled clinical trials to shorten her vomiting, profuse diarrhea, fevers and seizures.
disease course. She states that he was hospitalized for two nights
D. Schedule an EGD and colonoscopy with biopsies. due to dehydration, but his stool cultures did
E. Treat the patient and her two friends with em- not show any bacterial infection. The patient is
piric antibiotics to prevent spread of disease. otherwise healthy, and reports no drug allergies or
medication use. She believes in holistic medicine,
CORRECT ANSWER: B and has not pursued any vaccinations for her son.

RATIONALE What is the most likely diagnosis?


This patient most likely has an acute infec-
tious diarrhea of mild severity, characterized by A. Campylobacter infection.
symptoms that do not result in activity modifi- B. Rotavirus infection.
cation. Both the ACG and IDSA guidelines do C. Giardia infection.
not recommend empiric therapy for mild acute D. Shigella infection.
infectious diarrhea in a healthy immunocom- E. Shiga toxin producing E. coli (STEC) infection.
petent adult; most cases are caused by a viral
etiology, and would not respond to antibiotic CORRECT ANSWER: B
therapy. Although some studies have shown
benefit in patients treated with probiotic thera- RATIONALE
py, the studies are of varying quality. The IDSA This is a case of rotavirus infection in both the
guidelines state probiotics may be offered but patient and her son. Rotavirus infection is the
ACG guidelines recommend consideration of most common cause of diarrheal illness in chil-
probiotics only to patients with antibiotic asso- dren worldwide, although vaccination programs
ciated diarrhea. There is no role for endoscopy have significantly decreased the burden of infec-
at this time. As her friends are asymptomatic, tion. Incubation period is usually two days, with
treatment is not indicated. symptoms lasting a few days. Infection tends to
be more severe in children, with dehydration,
REFERENCES fevers, seizures, and respiratory symptoms, and
Riddle MS, DuPont HL, Connor BA. ACG Guide- milder in adults. The presence of non-bloody
line: Diagnosis, Treatment, and Prevention of diarrhea and negative stool cultures in her son
Acute Diarrheal Infections in Adults. Am J Gas- argue against campylobacter, STEC and shigella
troenterol 2016; 111:602-622. infection. Giardia infections are unlikely to cause
Shane AL, Mody RK, Crump JA, et. al. 2017 nausea, vomiting and fevers, and symptoms tend
Infectious Diseases Society of America Clinical to last up to four weeks.
Practice Guidelines for the Diagnosis and Man-
agement of Infectious Diarrhea. Clinical Infec- REFERENCES
tious Diseases 2017; 65(12):e45-e80. Greenberg HB, Estes MK. Rotaviruses: From
Pathogenesis to Vaccination. Gastroenterology
2009; 136(6):1939.
Question 29 Grimwood K, Buttery JP. Clinical Update: Ro-
A 26-year-old woman from California presents tavirus Gastroenteritis and its Prevention. Lancet
with a recent episode of sudden onset nausea, vom- 2007; 370(9584): 302.
306 Digestive Diseases Self-Education Program®

Question 30 REFERENCES
An 18-year-old man presents to the Emergency Hall AJ, Wikswo ME, Pringle K, et al. Vital Signs:
Department with nausea, vomiting and non- Foodborne Norovirus Outbreaks – United States,
bloody diarrhea for two days, with associated 2009-2012. MMWR 2014; 63(22): 491-495.
symptoms of fatigue and generalized headache. Hall AJ, Lopman BA, Payne DC, et al. Norovi-
He was in his usual state of health until three rus Disease in the United States. Emerg Infect Dis
days ago, when he went to a high school gradu- 2013; 19(8):1198-1205.
ation party where he ate a catered lunch buffet. Scallan E, Hoekstra RM, Angulo FJ, et al. Food-
He states that he has felt somewhat better over borne Illness Acquired in the United States - Major
the last 12 hours or so. A few of his friends that Pathogens. Emerg Infect Dis 2011; 17(1): 7-15.
attended the same function developed fever and
diarrhea early the following morning, but did not
seek medical attention. Question 31
A 49-year-old female pig farmer from Maine
What is the most likely diagnosis? presents with intermittent cough and wheezing for
which she was unsuccessfully treated with albuterol
A. Enteric adenovirus infection inhalers. She also reports epigastric discomfort for
B. Shiga toxin producing E. coli (STEC) six months which did not respond to an omeprazole
infection trial. She reports passing a large round worm in
C. Norovirus infection her stools, which she has brought in for examina-
D. Rotavirus infection tion. Testing confirms Ascaris lumbricoides.
E. B. cereus toxin ingestion
What is the most appropriate treatment for this
CORRECT ANSWER: C patient?

RATIONALE A. Albendazole 400 mg orally for one dose.


Norovirus infection is the most common cause of B. Albendazole 400 mg orally daily for three days.
food-borne gastroenteritis in the United States, C. Ivermectin 200 mcg / kg daily for three days
and food-borne transmission accounts for roughly D. Praziquantel 25 mg / kg per dose, three times
23 percent of cases of norovirus infections. The a day for one day.
incubation period is usually one to two days, with E. Nitazoxanide 500 mg twice daily for three days.
common symptoms of nausea, vomiting and non-
bloody diarrhea lasting up to three days. Chil- CORRECT ANSWER: A
dren, older and immunocompromised patients
may have a more severe disease course. Enteric RATIONALE
adenovirus infections have a longer incubation of Ascaris lumbricoides is found in Asia, Africa,
roughly one week, and a symptom duration of up South America, and the Southeastern United
to two weeks. B. cereus toxin infections have a States, and transmitted via food or water con-
shorter incubation period, usually six hours, and taminated by A. lumbricoides or A. suum eggs. It
symptom duration of one day. STEC infections may also be associated with pig farming, both in
are associated with a longer incubation period of the United States and in Asia. Migration of larvae
at least three days as well as bloody diarrhea. Al- through the lungs results in Loeffler’s syndrome,
though rotavirus infection has similar symptoms, with coughing, wheezing and shortness of breath.
incubation and symptomatic period, it tends of af- Gastrointestinal symptoms may include pancre-
fect children under age two, and is a less common atitis, acute cholecystitis, acute cholangitis, or lu-
cause of food-borne illness than norovirus. minal symptoms such as abdominal pain, nausea,
Chapter 11 — GI infections of the small intestine and colon 307

vomiting, intestinal obstruction, and acute RATIONALE


abdomen. Diagnosis of ascariasis relies on Necator americanus causes hookworm infection in
identification of the worm during endoscopy or North and South Americas, Africa and South Pacific
ERCP, eggs passed in feces, or larvae on pulmo- Islands, which begins when larvae invade the host
nary specimens. Treatment of ascariasis is with through cutaneous contact with larvae in soil or
albendazole 400 mg orally for one dose, mebenda- other media. Infected patients may experience
zole 500 mg orally for one dose, or 100 mg twice nausea, vomiting, diarrhea and nutritional deficien-
daily for three days. cies such as iron deficiency anemia. Higher worm
burden is asoociated with male sex and older age
REFERENCES and may result in more severe symptoms. Diagno-
Jourdan PM, Lamberton PH, Fenwick A, et. al. sis is made by identification of hookworm eggs on
Soil-transmitted helminth infections. Lancet 2018; stool examination, and the preferred therapy is a
391:252-65. single dose of albendazole 400 mg given orally.
Colby K, Sears S, McEvoy E, et. al. Notes
from the Field: Ascariasis Associated with Pig REFERENCE
Farming - Maine, 2010-2013. MMWR 2013; Hotez PJ, Brooker S, Bethony JM, et. al, Hook-
62(20):413. worm Infection. N Engl J Med 2004; 351:799-807.

Question 32 Question 33
A 43-year-old man from rural Alabama with no A 45-year-old man presents to the clinic with
significant past medical history presents with worsening right lower quadrant pain and diarrhea
severe fatigue. His symptoms have been gradually for the last two days. His past medical history is
worsening for the last year, and he now reports significant for hemochromatosis and he undergoes
significant dyspnea on exertion. He denies any regular therapeutic phlebotomies. He admits to
chest pain but has diarrhea three to four times a dining out in a newly-opened restaurant in his town
day. Examination reveals pale conjunctiva and a four days ago. He describes having five non-bloody
2/6 systolic murmur. His laboratory evaluation is watery stools and also has been experiencing sore
significant for severe anemia, with a hemoglobin throat for the last two days. His physical examina-
of 6.1 g/dL, eosinophilia, and an iron saturation of tion is unremarkable except some mild abdominal
three percent. EGD and colonoscopy was sig- tenderness at right lower quadrant. There was no
nificant for a worm in the cecum, consistent with rebound tenderness. Laboratory data shows mild
Necator americanus infection. leukocytosis. Infectious diarrhea secondary to Yer-
sinia enterocolitica is strongly suspected.
What is the appropriate therapy for this patient?
What is the most likely mechanism through which
A. Ivermectin 200 mg / kg orally as a single dose this infection causes diarrhea?
B. Praziquantel 25 mg / kg orally three times daily
for one day A. Release of enterotoxins that stimulate secretion
C. Albendazole 400 mg orally as a single dose and/or impair absorption
D. Metronidazole 500 mg orally three times a day B. Penetration into the submucosa and prolifera-
for 10 days tion in the lymph nodes
E. Nitazoxanide 500 mg orally every 12 hours for C. Production of the toxins in contaminated food
seven days D. Adherence to the intestinal mucosa and efface-
ment of microvilli
CORRECT ANSWER: C E. Mucosal damage with erosions and ulcers
308 Digestive Diseases Self-Education Program®

CORRECT ANSWER: B laboratory investigations reveal mild leukocytosis


with eosinophilia.
RATIONALE
This is an example of Yersinia infection. Clini- What is the most likely cause of his diarrhea?
cal manifestations of acute yersiniosis include
diarrhea, abdominal pain, and fever; nausea and A. Cryptosporidium
vomiting may also occur. Localization of ab- B. Cyclospora
dominal pain to the right lower quadrant is also C. Cystoisospora
a diagnostic clue for yersiniosis. Yersinia causes D. Microsporidia
diarrhea through penetration of the mucosa and
proliferation in the submucosa. Pathogenic Y. CORRECT ANSWER: C
enterocolitica pass through the stomach, adhere to
gut epithelial cells, invade the gut wall, localize in RATIONALE
lymphoid tissue within the gut wall and in regional Cystoisospora belli (formerly known as Isospora
mesenteric lymph nodes, and evade the host’s cell belli) is a gastrointestinal protozoan. In patients
mediated immune response. Vibrio cholerae and with AIDS and other immunodeficiencies, it is an
enterotoxigenic E. coli (ETEC)secrete enterotoxins opportunistic pathogen that can cause watery diar-
that stimulate secretion and/or impair absorption. rhea and weight loss. Infections are acquired by the
Some bacteria produce toxins in contaminated ingestion of sporulated oocysts from food or water
food; when ingested, the toxins cause acute symp- contaminated with human feces. In general, pro-
toms, usually nausea and vomiting. Examples tozoal infections do not cause peripheral or tissue
of these are Staphylococcus aureus and Bacil- eosinophilia; however, Cystoisospora infection is
lus cereus. Enteropathogenic E. coli (EPEC) and an exception to this rule. Diarrhea and peripheral
enterohemorrhagic E. coli (EHEC) adhere to the eosinophilia in an immunocompromised individual
intestinal mucosa, where they attach and cause ef- should raise concern for Cystoisospora infection.
facement of the microvilli. Shigella, enteroinvasive The other protozoal infections listed can cause diar-
E. coli, and Campylobacter jejuni penetrate the rhea, but do not cause eosinophilia.
mucosa, spread, and cause mucosal damage with
erosions and ulcers. REFERENCE
Goodgame RW. Understanding intestinal spore-
REFERENCE forming protozoa: cryptosporidia, microsporidia,
Cover TL, Aber RC. Yersinia enterocolitica. isospora, and cyclospora. Annals of internal medi-
The New England journal of medicine. Jul 6 cine. Feb 15 1996;124(4):429-441.
1989;321(1):16-24.

Question 35
Question 34 A 56-year-old man presents to the emergency
A 23-year-old HIV positive Hispanic man presents department with severe abdominal pain and
to the clinic with history of diarrhea for the last one bloody diarrhea six hours after having a dinner at
week. He recently returned from a month-long trip a local seafood restaurant. His friends mentioned
to Mexico. He describes sudden onset of watery di- that he ordered raw oysters with his meal. He has
arrhea associated with nausea and headache while history of alcohol abuse and drinks half a pint of
still in Mexico. He denies any blood in the stool hard liquor every day. On physical examination
but does describes his stools to be malodorous. his blood pressure was 80/40 mmHg, pulse 120/
His physical examination is unremarkable except min and temperature 38℃. Skin examination
for dry skin and dry oral mucus membranes, and revealed bullous lesions. Abdominal examination
Chapter 11 — GI infections of the small intestine and colon 309

revealed mild tenderness in the periumbilical area nal of medicine. Jan 1 2004;350(1):38-47.
and hepatomegaly on palpation. Bowel sounds Bross MH, Soch K, Morales R, Mitchell RB. Vib-
were present. rio vulnificus infection: Diagnosis and treatment.
Am Fam Physician. 2007 Aug 15;76(4):539-44.
There was no focal neurological deficit.
Which of the following infections is the most likely
cause of the septic shock? Question 36
A 45-year-old Korean man presents to the clinic
A. Ciguatara toxin with worsening right upper quadrant abdominal
B. Scromboid pain and generalized malaise. He also reports los-
C. Shellfish food poisoning ing 10 pounds over the last six months. He visited
D. Vibrio vulnificus his family in South Korea about eight months ago.
He also reports having non-bloody diarrhea for
CORRECT ANSWER: D the last one month. On physical examination, his
vital signs are stable. He has mild tenderness in
RATIONALE the right upper quadrant. His laboratory investiga-
Vibrio vulnificus is a gram-negative bacterium that tions revealed hemoglobin 10 g/dL, white blood
can cause serious wound infections, septicemia, cells 4.8x109/L with 1500/μL eosinophils. His ALT
and diarrhea. It is the leading cause of shellfish- was 35 U/L, AST 35 U/L, Alkaline phosphatase 192
associated deaths in the United States. A pre- U/L and total bilirubin 1.2 mg/dL. Ultrasound of
sumptive diagnosis of V. vulnificus septicemia the abdomen reveals a distended gallbladder and
should be made in any person with fever, hypoten- dilated common bile duct with non-shadowing
sion, or symptoms suggestive of septic shock and echogenic foci. Liver fluke infection is suspected to
characteristic bullous skin lesions. Patients with be the cause of his illness. Stool specimen revealed
liver cirrhosis and alcoholism are at increased oval shaped by formalin ethyl-acetate concentra-
risk of severe infection. Scombroid poisoning is a tion technique.
common seafood-associated disease throughout
the world. The most common findings consist Which of the following is the treatment of choice
of a rapid onset of flushing of the face and neck, for this infection?
erythematous and urticarial rash, diarrhea, and
headache occurring soon after consumption of A. Ciprofloxacin
contaminated fish or cheese. Paralytic shellfish B. Ivermectin
poisoning is caused by ingestion of bivalve mol- C. Levamisole
lusks, such as mussels, clams, scallops, and oys- D. Praziquantel
ters, as well as crabs and snails. Physical findings
occur within hours of ingestion and include neu- CORRECT ANSWER: D
rologic symptoms ranging from perioral tingling,
ataxia, difficulty swallowing, dizziness, paresthe- RATIONALE
sias, weakness, paralysis, brainstem dysfunction, This case represents an example of Clonorchis-
and respiratory failure. The rapid onset of weak- sinensis (or Opisthorchissinensis) infection, which
ness and paralysis distinguishes paralytic shellfish is also known as the Chinese liver fluke. Symptoms
poisoning from ciguatera fish poisoning. can include fatigue, abdominal discomfort, an-
orexia, weight loss, dyspepsia, and diarrhea. The
REFERENCES gallbladder is often nonfunctional and enlarged.
Thielman NM, Guerrant RL. Clinical practice. The diagnosis can be established by identifying
Acute infectious diarrhea. The New England jour- eggs in stool, duodenal aspirates, or bile specimens.
310 Digestive Diseases Self-Education Program®

Eggs may be visible in the stool approximately mon findings consist of a rapid onset of flushing
four weeks following infection. The “gold stan- of the face and neck, erythematous and urticarial
dard” detection method for infection is formalin rash, diarrhea, and headache occurring soon after
ethyl-acetate concentration technique (FECT). The consumption of contaminated fish. Scombroid poi-
treatment of choice is praziquantel. Alternatives to soning usually resolves within 12 to 48 hours if un-
praziquantel include albendazole or mebendazole. treated and has no long-term sequelae. Symptoms
Ivermectin is used in the treatment of Strongyloi- of Ciguatera fish poisoning typically begins three
diasis. Levimasole is used in ascariasis treatment. to six hours after eating contaminated fish, but can
be delayed up to 30 hours. Clinical findings include
REFERENCES peri-oral paresthesias, pruritus without urticaria
Qian MB, Utzinger J, Keiser J, Zhou XN. Clonor- or erythema and temperature-related dysesthe-
chiasis. Lancet 2016; 387:800. sias (cold stimuli perceived as hot or producing an
Lai DH, Hong XK, Su BX, et al. Current status abnormal, unpleasant sensation). Ingestion of pre-
of Clonorchis sinensis and clonorchiasis in China. formed botulinum toxin, typically found in home
Trans R Soc Trop Med Hyg 2016; 110:21. canned foods, causes vomiting, diarrhea, abdomi-
nal pain, cranial nerve dysfunction, and descending
paralysis beginning hours to days after consump-
Question 37 tion. Paralytic shellfish poisoning is caused by in-
A 33-year-old man presents to the emergency gestion of bivalve mollusks, such as mussels, clams,
department with abdominal pain, nausea, vom- scallops, and oysters, as well as crabs and snails.
iting and diarrhea. His symptoms started soon Physical findings occur within hours of ingestion
after eating rare tuna steaks at a local restaurant. and include neurologic symptoms ranging from
He has no other significant medical history. On perioral tingling, ataxia, difficulty swallowing, dizzi-
physical examination, his blood pressure is 110/70 ness, paresthesias, weakness, paralysis, brainstem
mmHg, pulse is 115/min and respiratory rate is dysfunction, and respiratory failure. Staphylococ-
17/min. He is afebrile. He has noticeable erythem- cus aureus enterotoxin may cause abrupt onset of
atous rash on face and upper trunk. Mild tender- nausea, vomiting, and abdominal cramps approxi-
ness in the epigastric area is also appreciated. mately one hour after eating contaminated food.
Laboratory investigations are unremarkable. He is Patients with staphylococcal food poisoning may
resuscitated with intravenous fluids and adminis- develop a fever, which is not seen in patients with
tration of oral antihistamine with rapid resolution scombroid poisoning.
of symptoms.
REFERENCES
Which is the most likely cause of this illness? Sobel J, Painter J. Illnesses caused by marine tox-
ins. Clinical infectious diseases : an official publica-
A. Botulism tion of the Infectious Diseases Society of America.
B. Ciguatera fish poisoning Nov 1 2005;41(9):1290-1296.
C. Paralytic shellfish poisoning Scombroid fish poisoning associated with tuna
D. Scombroid fish poisoning steaks--Louisiana and Tennessee, 2006. MMWR.
E. Staphylococcal food poisoning Morbidity and mortality weekly report. Aug 17
2007;56(32):817-819.
CORRECT ANSWER: D

RATIONALE Question 38
Scombroid poisoning is a common seafood-associ- A 36-year-old man was admitted to the hospital
ated disease throughout the world. The most com- with worsening abdominal pain, vomiting and diar-
Chapter 11 — GI infections of the small intestine and colon 311

rhea. His symptoms started few hours after eating hours of ingestion and include neurologic symp-
at a local hotel buffet. A few hours later, he experi- toms ranging from perioral tingling, ataxia, diffi-
enced numbness of his extremities and change in culty swallowing, dizziness, paresthesias, weakness,
his taste sensation prompting him to seek medical paralysis, brainstem dysfunction, and respiratory
attention. He explained that cold objects felt hot failure. The rapid onset of weakness and paralysis
and his teeth also felt numb and loose. He devel- distinguishes paralytic shellfish poisoning from
oped bradycardia and hypotension in the emergen- ciguatera fish poisoning. Signs and symptoms of
cy department requiring treatment with intrave- scombroid toxicity usually begin within an hour of
nous atropine. Physical examination revealed mild eating contaminated fish. The symptoms resemble
periumbilical abdominal tenderness. His laboratory an IgE-mediated allergic reaction. The patient
investigations were unremarkable. He was resusci- may suddenly experience flushing, a sensation of
tated with intravenous fluids and was subsequently warmth, an erythematous rash, palpitations, and
hospitalized. Few days later, he was discharged to significant tachycardia. Ingestion of preformed
home with complete resolution of his symptoms. botulinum toxin, typically found in home canned
foods, causes vomiting, diarrhea, abdominal pain,
What is the most likely cause of his illness? cranial nerve dysfunction, and descending paralysis
beginning hours to days after consumption.
A. Botulism
B. Ciguatera fish poisoning REFERENCES
C. Paralytic shellfish poisoning Pennotti R, Scallan E, Backer L, et al. Ciguatera
D. Scombroid fish poising and scombroid fish poisoning in the United States.
E. Staphylococcal food poisoning Foodborne Pathog Dis 2013; 10:1059.
Sobel J, Painter J. Illnesses caused by marine
CORRECT ANSWER: B toxins. Clinical infectious diseases : an official
publication of the Infectious Diseases Society of
RATIONALE America. Nov 1 2005;41(9):1290-1296.
Ciguatera fish poisoning accounts for approxi-
mately 20 percent of the fish-related foodborne
outbreaks in the United States. Gastroenteritis, Question 39
including vomiting, diarrhea, and abdominal A 62-year-old man with no significant past medical
cramping, typically begins three to six hours after history is admitted to the hospital after develop-
eating contaminated fish, but can be delayed up to ing bloody diarrhea and severe abdominal cramps
30 hours. Neurologic abnormalities usually appear for several days. He denies any fever. There is no
three to 72 hours after the meal. Clinical findings history of any recent travel or contact with sick
include perioral paresthesias, pruritus without person. On physical examination, his vital signs are
urticaria or erythema, a metallic taste in the mouth, stable. Laboratory investigations reveals leukocyte
painful dentition, a feeling that the teeth are loose, count to be mildly elevated at 13.0x109/L with
painful urination, blurred vision, and temperature- normal differential. His metabolic panel and liver
related dysesthesias (cold stimuli perceived as hot function tests are also unremarkable. He has only
or producing an abnormal, unpleasant sensation). rare leukocytes found in the stool by microscopy.
Cardiovascular signs, including bradycardia, heart A stool sample sent to the laboratory, is negative
block, and hypotension, can occur within hours for Clostridium difficile toxin A. Stool cultures at
of consumption. Paralytic shellfish poisoning is 48 hours are negative for Shigella, Salmonella, and
caused by ingestion of bivalve mollusks, such as Campylobacter. Colonoscopy with biopsy reveals
mussels, clams, scallops, and oysters, as well as submucosal edema, hemorrhage and ulcerations
crabs and snails. Physical findings occur within consistent with ischemic colitis.
312 Digestive Diseases Self-Education Program®

Which of the following stool tests would most likely nausea and vomiting. He denied any recent
be useful in this clinical situation? travelling but recalled consuming watermelons a
day before the onset of his symptoms. On physi-
A. Enzyme immunoassay for C. difficile toxin B cal examination, his blood pressure was 90/60
B. Enzyme immunoassay for microsporidia mmHg, pulse 120/min, respiratory rate 16/min. He
C. Enzyme immunoassay for rotavirus was afebrile. Moderate periumbilical tenderness
D. Enzyme immunoassay for Shiga toxin on abdominal examination was noted. Rectal exam
E. Stool smear for amoebae was unremarkable and negative for occult blood.
He was aggressively resuscitated with intravenous
CORRECT ANSWER: D fluids and was admitted to the hospital with the
diagnosis of cholera and circulatory shock.
RATIONALE
The clinical features of bloody diarrhea with Which of the following is the treatment of choice
severe abdominal pain, lack of fever and rare leu- for her infection?
kocytes in stool all suggest Shiga toxin-producing
E. coli. Although the usual cause in the USA is A. Ampicillin
0157:H7, other serotypes have been implicated. B. Azithromycin
The enzyme immunoassay test can detect Shiga C. Metronidazole
toxin produced by any of the serotypes. The acute D. No antibiotic is indicated
onset and bloody diarrhea are unusual for micro- E. Trimethoprim-sulfamethoxazole
sporidiosis. C. difficile can cause bloody diarrhea.
However, disease related to toxin B without toxin CORRECT ANSWER: B
A is uncommon. The presentation is not typical
for viral enteric infection such as rotavirus. While RATIONALE
majority of the case of the amoebiasis are asymp- Vibrio cholera is a bacterial pathogen linked to
tomatic, severe dysentery can occur. Risk factors epidemics of diarrhea. Infection with V. cholerae
for acute severe dysentery include malnutrition, results in a spectrum of disease, ranging from
steroid use, pregnancy, alcoholism, immunosup- asymptomatic intestinal colonization to severe
pression and malignancy which are absent in this diarrhea. While mild cases of V. cholerae infec-
case. In contrast to bacterial dysentery, which tion may be clinically indistinguishable from other
typically begins abruptly, amebic colitis begins causes of watery diarrheal illness, the profound
gradually over one to several weeks. and rapid loss of fluid and electrolytes mark
severe cholera as a clinical entity distinct from
REFERENCES other causes. Aggressive volume repletion is the
Thielman NM, Guerrant RL. Clinical practice. mainstay of treatment for cholera. Antibiotics
Acute infectious diarrhea. N Engl J Med 2004; can shorten the duration of diarrhea, reduce the
350:38. volume of stool losses, and lessen the duration
Page AV, Liles WC. Enterohemorrhagic Esch- of V. cholera shedding. The antibiotic options for
erichia coli Infections and the Hemolytic-Uremic cholera include macrolides, fluoroquinolones, and
Syndrome. Med Clin North Am 2013; 97:681. tetracyclines.
Most V. cholerae strains are resistant to
trimethoprim-sulfamethoxazole. Ampicillin and
Question 40 metronidazole are not indicated in cholera.
A 42-year-old woman presented to the emergency
department with a sudden development of profuse REFERENCES
watery diarrhea associated with abdominal cramps, Harris JB, LaRocque RC, Qadri F, et al. Cholera.
Chapter 11 — GI infections of the small intestine and colon 313

Lancet 2012; 379:2466. penetration, which causes “ground itch”. Periph-


Sack DA, Sack RB, Nair GB, Siddique AK. Chol- eral eosinophilia is seen approximately 95 percent
era. Lancet 2004; 363:223. of infected patients. Iron deficiency anemia, hypo-
albuminemia and malnutrition are common with
chronic infection. The most common symptom
Question 41 with pinworm is perianal itching, also known as
A 62-year-old woman presents to the clinic with pruritus ani. The most common manifestations for
mild post prandial abdominal pain for nine strongyloidiasis are mild waxing and waning gas-
months. She also has history of non-bloody diar- trointestinal, cutaneous, or pulmonary symptoms
rhea with passage of four or five loose stools every that persist for years.
day. She remembers having a pruritic rash on her
body shortly before her symptoms developed. She REFERENCE
denies any weight loss. She lives in the United Hotez PJ, Brooker S, Bethony JM, et al. Hookworm
States but is originally from India and travels back infection. N Engl J Med. 2004;351(8):799-807.
there regularly for business. On physical exami-
nation, her vital signs are stable. There is mild
mid-abdominal tenderness on deep palpation and Question 42
no organomegaly. Rectal examination is unre- A 62-year-old woman presents to the clinic with
markable. Her complete blood count is remark- mild post prandial abdominal pain for nine
able for 10.4 gm/dL hemoglobin, 68 fL MCV and months . She also has history of non-bloody diar-
12 percent peripheral eosinophilia. A colonoscopy rhea with passage of four or five loose stools every
performed for colorectal cancer screening one year day. She remembers having a pruritic rash on her
ago was unremarkable except for a single 4 mm body shortly before her symptoms developed. She
tubular adenoma which was removed. A CT scan denies any weight loss. She lives in the United
performed six months ago for her abdominal pain States but is originally from India and travels back
was unremarkable. What is the most likely infec- there regularly for business. On physical exami-
tious etiology of her symptoms? nation, her vital signs are stable. There is mild
mid-abdominal tenderness on deep palpation and
A. Ascaris infection no organomegaly. Rectal examination is unre-
B. Hookworm infection markable. Her complete blood count is remark-
C. Pin worm infection able for 10.4 gm/dL hemoglobin, 68 fL MCV and
D. Strongyloidiasis 12 percent peripheral eosinophilia. A colonoscopy
E. Whip worm infection performed for colorectal cancer screening one year
ago was unremarkable except for a single 4 mm
CORRECT ANSWER: B tubular adenoma which was removed. A CT scan
performed six months ago for her abdominal pain
RATIONALE was unremarkable. Wet mount of stool revealed
The patient is most likely infected with hookworm eggs of Necator americanus. Which of the follow-
(Ancylostoma duodenale or Necator americanus). ing is the treatment of choice for her infection?
Her frequent travel to an endemic area, peripheral
eosinophilia and iron-deficiency anemia are all A. Albendazole
classic findings for this infection, which causes B. Azithromycin
otherwise non-specific symptoms like diarrhea C. Metronidazole
and abdominal pain. The pruritic rash that pre- D. Ivermectin
ceded his symptoms is also a clue, as the life cycle
of this organism typically involves initial dermal CORRECT ANSWER: A
314 Digestive Diseases Self-Education Program®

RATIONALE A. Enterotoxin production


The patient is most likely infected with hookworm B. Mucosal invasion
(Ancylostoma duodenale or Necator americanus). C. Preformed toxin
Her frequent travel to an endemic area, periph- D. Cytotoxin production
eral eosinophilia and iron-deficiency anemia are E. Enteroadherence
all classic findings for this infection, which causes
otherwise non-specific symptoms like diarrhea and CORRECT ANSWER: C
abdominal pain. The pruritic rash that preceded
his symptoms is also a clue, as the life cycle of this RATIONALE
organism typically involves initial dermal pen- This patient has clinical features compatible with
etration, which causes “ground itch”. Peripheral ingestion of the preformed emetic toxin of Staphy-
eosinophilia is seen approximately 95 percent of lococcus aureus. The acute onset of symptoms
infected patients. Iron deficiency anemia, hypo- shortly after ingestion of the offending food is
albuminemia and malnutrition are common with indicative of a preformed toxin. Bacillus cereus is
chronic infection. The most common symptom another organism that presents in similar fashion.
with pinworm is perianal itching, also known as Examples of bacteria with enterotoxin produc-
pruritus ani. The most common manifestations for tion include Vibrio cholerae and enterotoxigenic
strongyloidiasis are mild waxing and waning gas- E. coli. Bacteria causing mucosal invasion in-
trointestinal, cutaneous, or pulmonary symptoms clude Shigella and Campylobacter, among several
that persist for years. Albendazole is the treatment others. Clostridium difficile is an example of
of choice for hookworm infection, and a three an organism that causes cytotoxin production,
day course has been proven more effective than a whereas many of the E. coli (EPEC, EHEC, EAEC
single dose of therapy. Ivermectin has poor efficacy and DAEC) work through an enteroadherence
against hookworm. mechanism.

REFERENCE REFERENCES
Hotez PJ, Brooker S, Bethony JM, et al. Hookworm Cangemi JR. Food poisoning and diarrhea: small
infection. N Engl J Med. 2004;351(8):799-807. intestine effects. Curr Gastroenterol Rep. 2011
Oct;13(5):442-8.
Bresee JS, Marcus R, Venezia RA, et al. The eti-
Question 43 ology of severe acute gastroenteritis among adults
A 44-year-old woman presents to the afterhours visiting emergency departments in the United
urgent care clinic with worsening nausea, vomiting States. J Infect Dis. 2012 May 1;205(9):1374-81.
and abdominal cramps. She felt well earlier in the
day, and played in a softball tournament followed
by a potluck lunch with the other players. She de- Question 44
nies any fever and has no diarrhea. On physical ex- A 27-year-old woman presented to the clinic with
amination, her vital signs are stable. Her abdominal a history of diarrhea for the last 10 days. She re-
examination is notable for mild diffuse tenderness. ported 10 to 12 loose stools with passage of blood.
Bowel sounds are normal and rectal examination is Diarrhea was associated with nausea and ab-
unremarkable. Laboratory investigations including dominal cramps. She denied any contact with sick
complete blood count, electrolytes and liver func- person or recent traveling. She recently moved
tion tests are unremarkable. to a farmhouse with her husband. On physi-
cal examination, her blood pressure was 110/60
Which of the following is the most likely pathogenic mmHg, pulse100/min, respiratory rate 13/min
mechanism for her illness? and temperature 100.5℉. Abdominal examination
Chapter 11 — GI infections of the small intestine and colon 315

revealed right lower quadrant abdominal tender- REFERENCE


ness. Stool cultures were ordered and came back Dasti JI, Tareen AM Lugert R, et al. Campylobacter
positive for Campylobacter infection. jejuni: a brief overview on pathogenicity-associated
factors and disease-mediating mechanisms. Int J
Which of the following is the treatment of choice Med Microbiol 2010; 300:205–11.
for her infection? Gupta A, Nelson JM, Barrett TJ, et al. Antimi-
crobial resistance among Campylobacter strains,
A. Azithromycin United States, 1997-2001. Emerging infectious
B. Amoxicillin diseases. Jun 2004;10(6):1102-1109.
C. Cefixime
D. Ciprofloxacin
E. Trimethoprim-sulfamethoxazole Question 45
A 54-year-old man presented to the emergency
CORRECT ANSWER: A department with worsening nausea, vomiting,
diarrhea and high-grade fever. His symptoms
RATIONALE started few days after attending a dinner party. He
Campylobacter species are a major cause of diar- underwent renal transplant five years ago and had
rheal illness in the world. The organism inhabits been taking prednisone, mycophenolate mofetil
the intestinal tracts of a wide range of animal hosts, and tacrolimus. On physical examination, his blood
notably poultry; contamination from these sources pressure was 110/70 mmHg, pulse 110/min, respi-
can lead to foodborne disease. Given the self- ratory rate 16/min and temperature 104℉. He was
limited nature of most Campylobacter infections confused and had tremors in upper extremities.
and the limited efficacy of routine antimicrobial Abdominal examination revealed moderated ten-
therapy, treatment is warranted only for patients derness in the periumbilical area. Soon after admis-
with features of severe disease or risk for severe dis- sion to the emergency department, he developed
ease. Patients with severe disease include individu- seizures and lost consciousness, and was intubated
als with bloody stools, high fever, extra-intestinal to protect his airway. Routine labs were sent for
infection, worsening or relapsing symptoms, or investigations, and both CT scan head and lumbar
symptoms lasting longer than one week. First-line puncture were performed. CT scan was negative for
agents for treatment of Campylobacter infection any acute findings, and CSF fluid analysis revealed
include fluoroquinolones or azithromycin. Campy- increased white blood cells, mainly lymphocytes,
lobacter is inherently resistant to trimethoprim and and low glucose. Listeria meningoencephalitis is
beta-lactam antibiotics, including penicillin and strongly suspected. Which of the following would
most cephalosporins. be the most appropriate treatment of this infection?
In the United States, the rate of resistance to
fluoroquinolones is also increasing. The rate of A. Ampicillin
ciprofloxacin resistance among Campylobacter B. Ampicillin and gentamicin
isolated in the United States increased from 0 to C. Ampicillin and vancomycin
19 percent between 1989 and 2001. Inappropri- D. Tetracycline
ate and over prescription of fluoroquinolones in E. Erythromycin
humans combined with increased fluoroquinolone
use in the poultry industry have contributed to the CORRECT ANSWER: B
increased prevalence of fluoroquinolone resistance.
The rate of macrolide-resistance among Campylo- RATIONALE
bacter has remained stable at less than five percent This case highlights a classic presentation of
in most parts of the world. Listeria infection in an immunocompromised
316 Digestive Diseases Self-Education Program®

patient. Listeria is the one cause of bacterial (non- A. Diphyllobothrium latum


tuberculous) meningitis in which a substantial B. Hymenolepis diminuta
number of lymphocytes (greater than 25 percent) C. Hymenolepis nana
can be seen in the CSF differential count in the D. Taenia saginata
absence of antibiotic therapy. Listeria monocy- E. Taenia solium
togenes is susceptible to common antimicrobial
agents, such as ampicillin, penicillin, gentamicin, CORRECT ANSWER: A
and trimethoprim-sulfamethoxazole. Ampicillin
or penicillin G are the drugs of choice. However, RATIONALE
these antibiotics demonstrate delayed in vitro This is likely a tapeworm infection with Diphyllo-
bactericidal activity at concentrations attain- bothrium latum. D. latum infection can be ac-
able in the cerebrospinal fluid (CSF). As a result, quired from ingesting certain forms of freshwater
gentamicin is often added to achieve synergy for fish, and those who consume raw fish, including
Listerial central nervous system (CNS) infections, sushi, are at increased risk. The classical manifes-
endocarditis and infections in immunocompro- tation of infection with D. latum is megaloblastic
mised patients such as the patient presented anemia due to vitamin B12 deficiency. D. latum
above. Tetracyclines have in vitro activity against has a unique affinity for vitamin B12 and therefore
Listeria; however, they are bacteriostatic, and competes with the host for absorption. Humans
their clinical efficacy is uncertain. Although the become infected with Taenia by ingesting raw or
use of vancomycin is usually avoided, it can be undercooked infected meat containing cysticerci.
used as an alternative agent in pregnant women Infection with Hymenolepis is common in chil-
when other agents cannot be used. dren secondary to breaches in fecal-oral hygiene.
Most infections are asymptomatic.
REFERENCE
Hof H, Nichterlein T, Kretschmar M. Management REFERENCE
of listeriosis. Clinical microbiology reviews. Apr Webb C, Cabada MM. Intestinal cestodes. Curr
1997;10(2):345-357. Opin Infect Dis. 2017 Oct;30(5):504-510.

Question 46 Question 47
A 36-year-old man presents to the clinic with a A 36-year-old man presents to the clinic with
history of diarrhea and significant fatigue for the a history of diarrhea and significant fatigue for
last two months. He has no significant past medi- the last two months. He has no significant past
cal history and works as a chef in a local sushi bar. medical history and works as a chef in a local
He complains of six to seven watery stools daily sushi bar. He complains of six to seven watery
with nocturnal symptoms. Diarrhea is associated stools daily with nocturnal symptoms. Diarrhea is
with abdominal cramps and he denies any passage associated with abdominal cramps and he denies
of blood. His physical examination, including any passage of blood. His physical examination,
vital signs, is unremarkable. Laboratory investiga- including vital signs, is unremarkable. Labora-
tion reveals 9.8 g/dL hemoglobin, with a mean tory investigations reveals 9.8 g/dL hemoglobin,
corpuscular volume 110 fL. Peripheral eosino- with a mean corpuscular volume 110 fL. Periph-
philia is also noted. A stool sample is sent to the eral eosinophilia is also noted. A stool sample is
lab and is pending. sent to the lab and is pending.

Which of the following is the most likely cause Which of the following medications is most likely
of this illness? to help?
Chapter 11 — GI infections of the small intestine and colon 317

A. Albendazole A. Ampicillin
B. Mebendazole B. Azithromycin
C. Paromomycin C. Ciprofloxacin
D. Praziquantel D. Metronidazole
E. Pyrantel pamoate E. Trimethoprim-sulfamethoxazole

CORRECT ANSWER: D CORRECT ANSWER: B

RATIONALE RATIONALE
This is likely a tapeworm infection with Diphyllo- Travelers’ diarrhea is an infectious illness, caused
bothrium latum. D. latum infection can be ac- by a variety of bacterial, viral, and parasitic
quired from ingesting certain forms of freshwater organisms, although bacterial pathogens are the
fish, and those who consume raw fish, including most frequent cause in acute cases. More than 90
sushi, are at increased risk. The classical manifes- percent of illnesses in most geographic areas are
tation of infection with D. latum is megaloblastic caused by bacteria; the most common organism is
anemia due to vitamin B12 deficiency. Praziquan- enterotoxigenic Escherichia coli (ETEC). The clas-
tel is the treatment of choice for all of the tape- sic “turista” due to ETEC generally produces mal-
worm infections discussed above. All the other aise, anorexia, and abdominal cramps followed by
agents listed would not be used for treatment of the sudden onset of watery diarrhea. Antibiotics
tapeworm infection. are warranted to treat diarrhea in those who de-
velop moderate to severe diarrhea, characterized
REFERENCE by more than four unformed stools daily, fever,
Webb C, Cabada MM. Intestinal cestodes. Curr or blood, pus, or mucus in the stool. Fluoroqui-
Opin Infect Dis. 2017 Oct;30(5):504-510. nolones have well-established efficacy for travel-
ers’ diarrhea and are generally well-tolerated for
the short duration. Azithromycin is the preferred
Question 48 agent for children and pregnant women. Wide-
A 36-year-old pregnant woman presented to the spread antibiotic resistance among many routine
clinic with diarrhea after a one-week vacation in enteric pathogens to drugs such as ampicillin and
Thailand. Her symptoms started as generalized trimethoprim-sulfamethoxazole has made these
malaise and loss of appetite followed by sudden on- agents less useful for the treatment of travelers’
set of diarrhea. She reported passing three to four diarrhea in much of the world.
non-bloody stools in a day associated with nausea
and abdominal pain. Physical examination, includ- REFERENCE
ing vital signs, was unremarkable except palpation Steffen R, Hill DR, DuPont HL. Traveler’s diarrhea:
of gravid uterus. A clinical diagnosis of traveler’s a clinical review. JAMA. 2015 Jan;313(1):71-80.
diarrhea was made and the patient was instructed
to increase her hydration and watch for any alarm
symptoms. She called back after three days report- Question 49
ing worsening of diarrhea with passage of some A 27-year-old man presents to the clinic with
mucus in the stool. She denies any fever or worsen- worsening abdominal pain and diarrhea for the last
ing of abdominal pain. three weeks. For the last two days, he also noticed
blood in his stools. He recently returned from a
Which of following antibiotics will be the most ap- business trip from Taiwan where he stayed in a
propriate treatment in this case? hotel and only drank bottled water. He admits to
being sexually active with several partners of the
318 Digestive Diseases Self-Education Program®

same gender during his trip. Physical examination, Question 50


including vital signs, is unremarkable. Initial stool A 27-year-old man presents to the clinic with
cultures are negative. CT scan of abdomen and worsening abdominal pain and diarrhea for the
pelvis shows thickening of the cecum and ascend- last three weeks. For the last two days, he also
ing colon. He undergoes colonoscopy that reveals noticed blood in his stools. He recently returned
ulcerated mucosa throughout the right colon. from a business trip from Taiwan where he stayed
in a hotel and only drank bottled water. He admits
Which of the following is the most likely diagnosis? to being sexually active with several partners of the
same gender during his trip. Physical examination,
A. Aeromonas hydrophilia including vital signs, is unremarkable. Initial stool
B. Cryptosporidium parvum cultures are negative. CT scan of abdomen and
C. Escherichia coli pelvis shows thickening of the cecum and ascend-
D. Entamoeba histolytica ing colon. He undergoes colonoscopy that reveals
E. Herpes simplex ulcerated mucosa throughout the right colon.

CORRECT ANSWER: D Which of the following is the most appropriate


initial medication for this infection?
RATIONALE
This patient likely has infection with Entamoeba A. Acyclovir
histolytica. Areas with high rates of amebic infec- B. Ampicillin
tion include India, Africa, Mexico, and parts of C. Azithromycin
Central and South America. In the United States D. Ciprofloxacin
and Europe, homosexual men are principally E. Metronidazole
colonized with nonpathogenic E. dispar. In Japan
and Taiwan, however, E. histolytica is much more CORRECT ANSWER: E
prevalent among men who have sex with men.
Invasive, extra-intestinal amebiasis (e.g., hepatic RATIONALE
abscesses) are more frequent in HIV-infected This patient likely has infection with Entamoeba
patients in these countries. Given Entamoeba is histolytica. Areas with high rates of amebic infec-
a parasite, routine bacterial stool cultures will be tion include India, Africa, Mexico, and parts of
negative, and the diagnosis could be made through Central and South America. Invasive colitis due to
stool examination, stool antigen testing or serol- E histolytica is generally managed with metronida-
ogy. The cecum and ascending colon are most zole followed by a luminal agent (such as paromo-
commonly affected, as demonstrated in this case. mycin) to eliminate intraluminal cysts.
Aeromonas are gram-negative rods that widely
distributed in freshwater and marine environ- REFERENCE
ments and can cause both watery and bloody diar- Bercu TE, Petri WA, Behm JW. Amebic colitis: new
rhea. Enterotoxigenic E. coli and Cryptosporidium insights into pathogenesis and treatment. Curr Gas-
cause watery diarrhea alone. Herpes simplex virus troenterol Rep 2007; 9:429-33.
can present with distal proctitis in patients with
history of anal intercourse.
Question 51
REFERENCE A 53-year-old woman presents with waxing and
Bercu TE, Petri WA, Behm JW. Amebic colitis: new waning symptoms of right upper quadrant and epi-
insights into pathogenesis and treatment. Curr Gas- gastric pain, nausea and vomiting. She also notices
troenterol Rep 2007; 9:429-33. a skin rash that spontaneously occurs as liner red
Chapter 11 — GI infections of the small intestine and colon 319

streaks on her legs and buttocks. She occasion- abdominal pain and weight loss of four weeks dura-
ally has dry cough and shortness of breath. She tion. He is HIV positive and last his CD4 count is
has history of rheumatoid arthritis and has been 50 cells/microL. He denies any recent travelling
on prednisone and methotrexate for several years. or sick contacts. He admits being sexually active
Physical examination, including vital signs are with multiple male partners. Physical examination,
unremarkable. Laboratory tests are significant for including vital signs, is unremarkable. Ultrasound
a microcytic anemia and peripheral eosinophilia. of the abdomen shows a thickened gallbladder
EGD shows congested duodenal mucosa and ery- and mildly dilated common bile duct without any
thematous spots. Stools studies are negative for any stones. Stool examination is negative for leukocytes
ova and parasite. With a high suspicion for parasitic but microscopy reveals small pink colored oocytes
infection, intestinal aspiration is performed and with modified acid-fast stain.
reveals filariform larvae.
Laboratory studies:
Which of the following is the treatment of choice
for this parasitic infection? Hemoglobin 14 g/dL (140 g/L)
Leukocyte count 6000/µL (6x109/L)
A. Ivermectin Alanine aminotransferase 35 U/L
B. Paromomycin Aspartate aminotransferase 25 U/L
C. Praziquantel Alkaline phosphatase 110 U/L
D. Pyrantel pamoate Bilirubin, total 1.0 mg/dL (17.1
E. Thiabendazole µmol/L)
Serum albumin 3.1 g/dL (31 g/L)
CORRECT ANSWER: A
In addition to starting antiretroviral therapy, which
RATIONALE of the following is the most appropriate treatment
The patient has strongyloidiasis caused by Stron- for this infection?
gyloides stercoralis. In the past, thiabendazole
was administered for uncomplicated infection. A. Albendazole
However, thiabendazole is now rarely used due B. Metronidazole
to the availability of more efficacious and better- C. Nitazoxanide
tolerated medications. The treatment of choice D. Trimethoprim-sulfamethoxazole
for strongyloidiasis at present is ivermectin, with
albendazole as an alternative option. Standard CORRECT ANSWER: C
dosing of ivermectin consists of two single 200
mcg/kg doses of ivermectin administered on two RATIONALE
consecutive days or administered two weeks apart. The patient is infected with Cryptosporidium..
Cryptosporidium can cause an asymptomatic infec-
REFERENCE tion but in immunocompromised hosts (particularly
Greiner K, Bettencourt J, Semolic C. Strongyloidia- those with T-cell immunodeficiency), the illness is
sis: a review and update by case example. Clin Lab more frequently protracted and severe particularly
Sci 2008; 21:82-8. when the CD4 count is less than 100 cells/microL.
For HIV-infected patients, the most important part
of therapy is initiating highly-active antiretroviral
Question 52 therapy in order to reconstitute immunity. Ni-
A 43-year-old man presents to the clinic with tazoxanide has a limited effect on clinical symptoms
history of watery diarrhea, right upper quadrant in immunocompromised hosts and can be consid-
320 Digestive Diseases Self-Education Program®

ered if CD4 recovery is slow and diarrheal symptoms are


persistent and severe. Other therapies have not been found
to be very effective in the treatment.

REFERENCE
Wumba R, Longo-Mbenza B, Menotti J, et al. Epidemiol-
ogy, clinical, immune, and molecular profiles of microspo-
ridiosis and cryptosporidiosis among HIV/AIDS patients.
International journal of general medicine. 2012;5:603-611.
Answers & critiques

CHAPTER 12

Gastrointestinal bleeding
Daniel Wild, MD and
Brandon Sprung, MD

Question 1 REFERENCE
A 78-year-old man presents to the Emergency Lau JY, Leung WK, Wu JC, et al. Omeprazole
Department with hematemesis. He is hemody- before endoscopy in patients with gastroin-
namically stable and his initial hemoglobin is 9.8 testinal bleeding. N Engl J Med. 2007 Apr
g/dL. His BUN is 28 mg/dL and his Cr is 1.1 mg/ 19;356(16):1631-40.
dL. He has been on aspirin and clopidogrel since
having a stroke two months ago. The patient is Question 2
started on a continuous infusion of pantoprazole What is the minimum volume of blood loss from
while the GI team is mobilized for an urgent up- an upper GI source that is required for a patient
per endoscopy. to pass melena?

Proton pump inhibitor infusions have been A. 10 cc


shown to do which of the following in this clini- B. 20 cc
cal setting? C. 50 cc
D. 150 cc
A. Decrease hospital length of stay E. 250 cc
B. Increase the likelihood of surgical
intervention CORRECT ANSWER: C
C. Decrease mortality
D. Decrease the need for endoscopic therapy RATIONALE
E. Decrease the units of packed red blood cells When assessing patients with suspected upper GI
that will be needed bleeding, it is essential to evaluate the color and
texture of the stool. The presence or absence of
CORRECT ANSWER: D melena is an important triage tool in determining
the timing of endoscopy. From early studies done
RATIONALE on students drinking blood and then having their
In a landmark 2007 study, investigators in stool monitored, we know that melena arises after
Hong Kong randomized patients with suspected the ingestion of approximately 50 cc of blood.
Upper GI bleeding to receive either PPI infu- The absence of melena does not rule out an
sion or placebo prior to endoscopy. The group upper GI bleeding source, but provides valuable
receiving PPI had less active bleeding and less clinical information in anticipating the volume
need for therapy during endoscopy but there and rate of bleeding, and hence the appropriate
were no significant differences in length of stay, triage of resources including the timing of en-
need for surgery, mortality or amount of trans- doscopy and the disposition of the patient within
fusions needed. the health care system.

321
322 Digestive Diseases Self-Education Program®

REFERENCE tion or aspiration, the insertion of an NG tube is


Daniel, W. A. and Egan, S. The Quantity of typically very painful and poorly tolerated by pa-
Blood Required to Produce a Tarry Stool. JAMA tients and when objectively assessed, was found to
1939;113:22-32. be subjectively more painful than an arterial blood
gas, digital nerve block and fracture reduction.
NG lavage has not been shown to improve mucosal
Question 3 visualization at the time of endoscopy nor decrease
An 84-year-old woman presents to the Emer- the need for second look endoscopy.
gency Department after having several episodes
of coffee-ground emesis earlier in the day. She REFERENCES
takes a daily aspirin and uses ibuprofen several Aljebreen AM, Fallone CA, Barkun AN. Nasogas-
days a week for arthritis pain. Her blood pres- tric aspirate predicts high-risk endoscopic lesions
sure is 98 / 70 mmHg and her heart rate is 88 in patients with acute upper-GI bleeding. Gastro-
bpm. A digital rectal exam reveals dark brown intest Endosc. 2004 Feb;59(2):172-8.
stool in her rectal vault. The Emergency Room Singer AJ, Richman PB, Kowalska A, Thode
physician asks if you would like her to perform HC Jr. Comparison of patient and practitioner
nasogastric lavage. Which of the following assessments of pain from commonly performed
statements regarding nasogastric lavage in this emergency department procedures. Ann Emerg
context is most accurate? Med. 1999 Jun;33(6):652-8.
Gralnek IM, Dumonceau JM, Kuipers EJ, et al.
A. The result of a nasogastric lavage is unlikely to Diagnosis and management of nonvariceal upper
change this patient’s management gastrointestinal hemorrhage: European Society
B. Nasogastric lavage is generally well tolerated of Gastrointestinal Endoscopy (ESGE) Guideline.
by patients Endoscopy. 2015 Oct;47(10):a1-46.
C. NG tube insertion carries a high risk of com-
plications like aspiration and sinus perforation
D. A lavage that reveals coffee grounds is highly Question 4
predictive of a high-risk endoscopic lesion A 68-year-old woman is admitted to the hospital
E. NG lavage is likely to improve mucosal visual- after having three episodes of hematemesis. The
ization during upper endoscopy most recent episode occurred two hours ago. Her
vital signs remain normal. She is not on antico-
CORRECT ANSWER: A agulation but takes a daily aspirin since undergo-
ing coronary artery bypass grafting six years ago.
RATIONALE She denies chest pain or shortness of breath. She
Despite being performed fairly regularly, NG has no history of liver disease and her physical
lavage seldom provides useful clinical information. exam reveals no stigmata of cirrhosis. Her admis-
In a large Canadian registry of patients who had sion labs are notable for a hemoglobin of 8.2 g/dL
NG lavage performed as part of their assessment which is down from a recent baseline of 11. The
for suspected upper GI bleeding, the positive pre- emergency physician asks you whether or not to
dictive value of a lavage containing blood or coffee transfuse the patient with packed red blood cells.
grounds in predicting the subsequent finding of a Below which of the following hemoglobin thresh-
high-risk endoscopic lesion was only 33 percent. olds should you begin transfusing packed red
Other studies corroborate these findings, yielding blood cells?
a sensitivity for predicting severe upper GI bleed-
ing of only 77 percent. Though very low risk to A. 7 g/dL
cause significant complication like sinus perfora- B. 8 g/dL
Chapter 12 — Gastrointestinal bleeding 323

C. 9 g/dL C. His Rockall score suggests that there is a high


D. 10 g/dL likelihood of finding a bleeding lesion during
E. 11 g/dL endoscopy
D. His ATRIA score suggests that his bleeding is
CORRECT ANSWER: A more likely to be from lower GI source than
from an upper GI source
RATIONALE E. His HAS-BLED score suggests that he is un-
In a landmark randomized controlled study pub- likely to re-bleed after endoscopic intervention
lished in 2013, patients with acute upper GI bleed-
ing were randomized to be transfused with either a CORRECT ANSWER: B
“liberal” strategy with a hemoglobin threshold of 9
g/dL or a “restrictive” strategy with a hemoglobin RATIONALE
threshold of 7 g/dL. Patients managed with the Several GI bleed scoring systems have been care-
restrictive strategy had significantly better survival fully validated. The AIMS-65 score uses patient
rates than those managed with the liberal strategy. factors (age, systolic blood pressure, mental status)
and lab values (INR and albumin) to predict in-
REFERENCE hospital mortality. This patient’s AIMS-65 score
Villanueva C, Colomo A, Bosch A, et al. Transfu- is 0 and his mortality is therefore predicted to be
sion strategies for acute upper gastrointestinal close to 0 as well. The Rockall score is also used to
bleeding. N Engl J Med. 2013 Jan 3;368(1):11-21. predict mortality and its calculation partially relies
on endoscopic findings. ATRIA and HAS-BLED
are systems used to calculate the risk of bleeding on
Question 5 anticoagulation in patients with atrial fibrillation.
A 42-year-old man with no known medical prob- The Glasgow-Blatchford score is used to perform
lems presents to the Emergency Department pre-endoscopy risk stratification using parameters
complaining of melenic stool that last occurred including systolic blood pressure, hemoglobin con-
yesterday. He has no abdominal pain and has centration, BUN, pulse, melena, syncope and the
not vomited. He takes ibuprofen as needed and presence of liver disease or heart failure. Patients
is on no other medications. His blood pressure is with a low GBS score (zero to one), as is the case in
105/85 mmHg and his heart rate is 88 bpm. this patient, do not typically require hospital admis-
sion or urgent endoscopy.
Pertinent labs are as follows:
REFERENCES
Hemoglobin: 12.2 g/dL Albumin: 4.2 g/dL Cipolletta L, Bianco MA, Rotondano G et al.
Platelets: 245K Total Bilirubin 1.2 mg/dL Outpatient management for low-risk nonvariceal
INR: 1.0 Cr 0.9 mg/dL upper GI bleeding: a randomized controlled trial.
BUN: 7 mg/Dl Gastrointest Endosc 2002; 55: 1-5.
de Groot NL, Bosman JH, Siersema PD et
Which of the following statements about stratify- al. Prediction scores in gastrointestinal bleeding:
ing this patient with validated scoring systems is a systematic review and quantitative appraisal.
most accurate? Endoscopy 2012; 44: 731-739.
Gralnek IM, Dumonceau JM, Kuipers et al.
A. His AIMS-65 score suggests that his in-hospi- Diagnosis and management of nonvariceal upper
tal mortality is between five to 10 percent gastrointestinal hemorrhage: European Society
B. His Glasgow-Blatchford score suggests that his of Gastrointestinal Endoscopy (ESGE) Guideline.
endoscopy can be performed as an outpatient Endoscopy 2015 Oct;47(10):a1-46.
324 Digestive Diseases Self-Education Program®

Question 6 and radiological procedures in the management of


Which of the following statements regarding the acute lower intestinal bleeding. Clin Gastroenterol
sensitivity of a radiologic study to detect active Hepatol. 2010 Apr;8(4):333-43.
bleeding in the GI tract is most accurate?

A. Bleeding must exceed 0.5 cc/min to be detect- Question 7


ed with tagged red blood cell scintigraphy A 47-year-old man is admitted to the hospital for
B. A positive tagged red blood cell scintigraphy hematemesis last noted two hours ago. His blood
study accurately shows the location of the pressure in 97/60 mmHg and his heart rate is
bleeding 90 percent or more of the time 103. His initial hemoglobin is 9.2 g/dL, BUN is
C. A bleeding protocol CT scan can detect bleed- 42 mg/dL and serum creatinine is 0.9 mg/dL. He
ing at rates as low as 0.1 cc/min reports taking ibuprofen regularly over the past
D. Angiography is more useful in patients with two months. You plan to perform urgent Upper
normal blood pressure and low transfusion Endoscopy and ask that his nurse administer 10
demands mg of IV metoclopramide.
E. Angiography detects bleeding at rates of
0.5 – 1 cc/min Which of the following statements regarding the
administration of prokinetics prior to endoscopy is
CORRECT ANSWER: E most accurate?

RATIONALE A. Patients who receive metoclopramide but not


Radiographic evaluation is commonly employed erythromycin have been shown to have better
in the diagnosis and management of patients with outcomes
lower GI bleeding. CT scans, tagged red blood cell B. Patients who receive prokinetics are less likely
scintigraphy and angiography all have roles in the to require surgical intervention for bleeding
care of these patients. Though tagged red blood cell than patients who do not
scintigraphy is the most sensitive modality at detect- C. Patients who receive prokinetics are less likely to
ing active bleeding, requiring rates from 0.05 – 0.1 need repeat endoscopy than those who do not
cc/ min, it is relatively poor at localizing the bleed- D. Patients who receive prokinetics have shorter
ing, accurately predicting the location in only 60-70 length of stays than patients who do not
percent of cases. CT scans have the advantage of E. Patients who receive prokinetics are less likely
being quickly performed and are widely available. If to aspirate during endoscopy that those who
extravasation is seen, its location is also accurately do not
determined. It is not as sensitive as red blood cell
scintigraphy however and requires bleeding rates of CORRECT ANSWER: C
0.3 – 0.5 cc/min to be positive. Angiography has the
advantage of being both diagnostic and potentially RATIONALE
therapeutic. It is best performed in sicker patients Prokinetics given prior to endoscopy can facilitate
with hypotension and high transfusion demands as blood, food and clot clearance from the stomach,
it is higher yield in these situations. Angiography thus improving mucosal visualization at the time
suffers from being the least sensitive of these mo- of endoscopy. A meta-analysis of three studies
dalities requiring bleeding rates between 0.5 – 1 cc/ encompassing over 300 patients showed that
min to be positive. prokinetics resulted in a significant reduction in
the need for repeat endoscopy with an odds ratio
REFERENCE of 0.55 but had no significant effect in transfusion
Strate LL, Naumann CR. The role of colonoscopy requirements, length of stay or need for surgical
Chapter 12 — Gastrointestinal bleeding 325

intervention. Randomized controlled compara- RATIONALE


tive data comparing the effects of erythromycin This patient has gastric antral vascular ectasia
to those of metoclopramide for this purpose are (GAVE) or watermelon stomach. This condition
lacking but small series suggest that erythromycin is seen most commonly in older women or pa-
is more beneficial. No studies have directly com- tients with systemic sclerosis but is also seen in
pared erythromycin and metoclopramide for this patients with portal hypertension and cirrhosis.
purpose and pre-endoscopy prokinetics have not It most commonly manifests with indolent blood
been shown to decrease the risk of intra-procedur- loss and anemia. It is most commonly treated
al aspiration. with ablative therapy utilizing APC, though
radiofrequency ablation (RFA) is emerging as an
REFERENCE alternative treatment option. Though it is associ-
Barkun AN, Bardou M, Martel M, Gralnek IM, ated with cirrhosis, it does not typically respond
Sung JJ. Prokinetics in acute upper GI bleed- to therapies directed against portal hypertension
ing: a meta-analysis. Gastrointest Endosc. 2010 so neither a beta-blocker nor TIPS are likely to
Dec;72(6):1138-45. be helpful. Since it is not peptic or inflamma-
tory in etiology, sucralfate does not help. Bipolar
cautery may be helpful but is not as effective as
Question 8 APC (or RFA).
A 78-year-old woman is being evaluated for
intermittent melena and chronic iron deficiency REFERENCES
anemia. A colonoscopy last week showed only Kamath PS, Lacerda M, Ahlquist DA, et al. Gastric
sigmoid diverticulosis and a single five mm tubular mucosal responses to intrahepatic portosystemic
adenoma. You perform upper endoscopy and find shunting in patients with cirrhosis. Gastroenterol-
the following in her antrum: ogy. 2000;118(5):905.
Wells CD, Harrison ME, Gurudu SR, et al.
Treatment of gastric antral vascular ectasia (water-
melon stomach) with endoscopic band ligation.
Gastrointest Endosc. 2008 Aug;68(2):231-6.

Question 9
A 72-year-old man with alcohol-related cirrhosis is
admitted with hematemesis. His heart rate is 115
bpm and blood pressure is 88/60 mmHg. Admis-
sion labs are notable for a hemoglobin of 8.3 g/dL,
Which of the following is the most effective treat- platelets of 110K and an INR of 1.4. He is not on
ment option for this condition: blood thinners or anti-platelet agents. He is intu-
bated and admitted to the ICU where you perform
A. Transjugular Intrahepatic Portosystemic urgent upper endoscopy which reveals fresh red
Shunt (TIPS) blood in the stomach and a lesion evident in the
B. Non-selective beta blocker gastric cardia on retroflexion. While examining the
C. High dose sucralfate elixir lesion, it starts to bleed briskly. (See two figures on
D. Argon Plasma Coagulation (APC) the following page.)
E. Bipolar probe cautery

CORRECT ANSWER: D
326 Digestive Diseases Self-Education Program®

Question 10
A 57-year-old American man presents with three
days of melena and a few episodes of blood-tinged
emesis. His hemoglobin is noted to be three
grams down from his baseline. He is hemodynam-
ically stable. He does not take any medications of
any kind and had no symptoms until his melena
started three days ago.

What is the most appropriate next step in this What is the most likely source for this patient’s
patient’s management? upper GI bleeding?

A. Place an Endoclip A. Peptic Ulcer Disease


B. Inject Cyanoacrylate B. Erosive Gastroduodenitis
C. Place a Sengstaken - Blakemore Tube C. Esophagitis
D. Inject Epinephrine D. Mallory-Weiss Tear
E. Electrocautery E. Esophageal Varices

CORRECT ANSWER: B CORRECT ANSWER: A

RATIONALE RATIONALE
This patient is bleeding from a gastric varix and will The most common etiologies for upper GI bleed-
require immediate therapy. A Hemoclip is effec- ing in the US are: Peptic Ulcer Disease (20-50
tive therapy for bleeding related to peptic ulcers or percent); Gastroduodenal erosions (eight to 15
small exposed vessels but would be the best treat- percent); Esophagitis (five to 15 percent); Mallory-
ment for a gastric varix. A Sengstaken – Blakemore Weiss Tear (eight to 15 percent); Esophageal
tube provides effective tamponade to temporarily Varices (five to 20 percent) and vascular malfor-
arrest bleeding from esophageal varices but would mations (five percent).
be less effective in managing a bleeding gastric
varix. Epinephrine injection into the varix would REFERENCE
not provide definitive therapy and electrocautery Hwang JH, Fisher DA, Ben-Menachem T, et al.
could worsen the bleeding. Injecting Cyanoacrylate The role of endoscopy in the management of acute
into the bleeding varix has emerged as an effective non-variceal upper GI bleeding. Gastrointest En-
and definitive therapy for gastric varices. dosc. 2012 Jun;75(6):1132-8.

REFERENCES
Ríos Castellanos E, Seron P, Gisbert JP, Bonfill Question 11
Cosp X. Endoscopic injection of cyanoacrylate You perform an upper endoscopy on a 70-year-
glue versus other endoscopic procedures for acute old patient with melena and discover a seven mm
bleeding gastric varices in people with portal cratered ulcer in her gastric antrum. The ulcer
hypertension. Cochrane Database Syst Rev. 2015 is not actively bleeding, but there is an adherent
May 12;(5):CD010180. clot tethered to the ulcer base. You elect not to
Hwang JH, Shergill AK, Acosta RD, et al. disturb the clot or perform any therapy. When
The role of endoscopy in the management of discussing these findings with the patient’s son,
variceal hemorrhage. Gastrointest Endosc. 2014 he asks you what her risk of rebleeding is over the
Aug;80(2):221-7. next few days.
Chapter 12 — Gastrointestinal bleeding 327

What is the most accurate answer to his question? CORRECT ANSWER: C

A. 1 percent RATIONALE
B. 5 percent Diverticulosis is very common, with prevalence rates
C. 25 percent of up to 50 percent in the United States (though this
D. 50 percent is hard to ascertain as most patients are asymptom-
E. 80 percent atic). Acute lower GI bleeding (LGIB) is commonly
encountered by practicing gastroenterologists with
CORRECT ANSWER: C an annual incidence rate of 0.03 percent. Diver-
ticulosis remains the most common source of LGIB,
RATIONALE but still only occurs in three to 15 percent of pa-
Peptic ulcers are classified by stigmata. It is im- tients with underlying diverticulosis. Its incidence
portant to know the risk of re-bleeding depending increases significantly with age and the mean age
on the appearance of the ulcer. Without therapy, for first presentation is 63-77 years. The mortality
a clean-based ulcer has a less than three percent remains low with an estimated rate between one and
chance of re-bleeding, while an ulcer with an ac- four percent. It remains a common indication for
tive arterial bleed will almost always continue to hospitalization with approximately 36 per 100,000
bleed or re-bleed. In between these stigmata is U.S. adults admitted for it annually.
a range including: ulcer with a visible vessel (50
percent re-bleeding rate without treatment); ulcer REFERENCES
with adherent clot (eight to 35 percent re-bleeding Fox JM, Stollman NH.. (2015). Diverticular
rate without treatment); oozing ulcer without ves- Disease of the Colon. In Sleisinger and Fordtran’s
sel or other stigmata (10-27 percent re-bleeding Gastrointestinal and Liver Disease. (pp. 2073-
rate without treatment) and ulcer with a flat pig- 2086). Philadelphia: Saunders/Elsevier.
mented spot (less than eight percent re-bleeding Longstreth GF. Epidemiology and outcome of
rate without therapy). patients hospitalized with acute lower gastrointes-
tinal hemorrhage: a population-based study. Am J
REFERENCE Gastroenterol. 1997;92:419-24.
Hwang JH, Fisher DA, Ben-Menachem T, et al. Bounds BC, Kelsey PB. Lower gastrointes-
The role of endoscopy in the management of acute tinal bleeding. Gastrointest Endosc Clin N Am
non-variceal upper GI bleeding. Gastrointest En- 2007;17:273-88,vi.
dosc. 2012 Jun;75(6):1132-8.

Question 13
Question 12 An 83-year-old man is admitted for the third time
Which of the following statements regarding di- in two months for anemia and transiently melenic
verticular bleeding is the most accurate? stool. His presenting Hgb has been as low as 5 g/
dL and he has had associated shortness of breath.
A. Clinically significant diverticular bleeding occurs On his first admission, an Upper Endoscopy and
in 25 percent of patients with diverticulosis Colonoscopy were performed and neither revealed
B. Clinically significant diverticular bleeding occurs a bleeding source. On his second admission, push
in 90 percent of patients with diverticulosis enteroscopy to the proximal jejunum was also
C. Clinically significant diverticular bleeding occurs unremarkable. You decide now to perform video
in 5 percent of patients with diverticulosis capsule endoscopy (VCE). Which of the following
D. Clinically significant diverticular bleeding occurs statements regarding the diagnostic yield of VCE
in 50 percent of patients with diverticulosis in this patient is most accurate?
328 Digestive Diseases Self-Education Program®

A. The diagnostic yield of his VCE will be higher is 32 mg/dL with a Cr of 1.2 mg/dL and his initial
if the study is performed electively after he is hemoglobin is 9.3 g/dL with no known baseline.
discharged from the hospital. You suspect that he has a bleeding ulcer and make
B. His study will have a lower diagnostic yield preparations to perform urgent upper endoscopy.
because of his advanced age. You also start him on an intravenous infusion of
C. The degree of his anemia will not influence the proton pump inhibitor (PPI). What is the mecha-
diagnostic yield of his VCE. nism of benefit of PPI in this acute setting?
D. His recent melenic stool increases the diag-
nostic yield of his VCE. A. Facilitate mucosal healing at the ulcer site
E. Studies in patients like him show a diagnostic B. Optimize platelet aggregation by increasing
yield for VCE of approximately 35 percent. gastric pH
C. Cytoprotection to prevent further mucosal
CORRECT ANSWER: D damage
D. Decrease the risk of intra-procedural aspira-
RATIONALE tion
Performing VCE is a reasonable next step in the E. Competitive Cyclooxygenase Enzyme inhibi-
patient. Studies show that the diagnostic yield of tion decreases NSAID toxicity
VCE is higher in patient’s who are older than 60,
have more severe anemia with a Hgb less than 10 CORRECT ANSWER: B
g/ dL, have been bleeding for longer (greater than
six months), are male and who have overt rather RATIONALE
than obscure bleeding. It has also been shown In the acute setting, the primary mechanism of
that the yield of VCE increases the more proximate benefit of PPI’s is acute elevation of the gastric pH
it is performed to the timing of the episode of which optimizes platelet aggregation and clot for-
overt bleeding. The diagnostic yield of most large mation. The other choices are either not actions
studies that evaluated the ability of VCE to detect performed by PPI’s or are not mechanisms by
a potential bleeding source is approximately 60 which PPI’s are beneficial in an acute setting.
percent.
REFERENCE
REFERENCES Green FW Jr, Kaplan MM, Curtis LE, Levine PH.
Gurudu SR, Bruining D, Acosta RD, Eloubeidi MA, Effect of acid and pepsin on blood coagulation
et al. The role of endoscopy in the management and platelet aggregation. A possible contributor
of suspected small-bowel bleeding. Gastrointest prolonged gastroduodenal mucosal hemorrhage.
Endosc. 2017 Jan;85(1):22-31. Gastroenterology. 1978;74(1):38.
Liao Z, Gao R, Xu C, Li ZS. Indications and de-
tection, completion, and retention rates of small-
bowel capsule endoscopy: a systematic review. Question 15
GastrointesEndosc. 2010 Feb;71(2):280-6. An 83-year-old woman presents to see you in clin-
ic to discuss a several year history of intermittent
melena and iron deficiency anemia in the setting
Question 14 of moderate aortic valve stenosis. Over the past
A 24-year-old man is admitted with bright red several years, she has required pack red blood cell
hematemesis. He has no medical or surgical transfusions three times despite being maintained
history but takes ibuprofen regularly for exercise- on intravenous iron infusions. She has undergone
related arthralgias. His initial heart rate is 118 and multiple evaluations including Upper Endoscopy,
blood pressure is 97 / 64 mmHg. His initial BUN Colonoscopy, two video capsule endoscopies, push
Chapter 12 — Gastrointestinal bleeding 329

enteroscopy and antegrade double balloon enter- REFERENCES


oscopy which have revealed scattered angioecta- Jackson CS, Gerson LB. Management of gastroin-
sias in the proximal and mid small bowel that have testinal angiodysplastic lesions (GIADs): a system-
been cauterized when detected. atic review and meta-analysis. Am J Gastroenterol.
She was last transfused with packed red blood cells 2014 Apr;109(4):474-83.
four months ago, last had melena three weeks ago Gurudu SR, Bruining DH, Acosta RD et al. The
and her most recent hemoglobin was 7.4 g / dL role of endoscopy in the management of suspected
when checked last week. small-bowel bleeding. Gastrointest Endosc. 2017
Jan;85(1):22-31.
What is the most appropriate next step in this
patient’s management?
Question 16
A. Perform retrograde double balloon enteros- Which of the following locations for a peptic ulcer
copy confers the highest risk of rebleeding following
B. Send her for provocative angiography endoscopic therapy?
C. Initiate treatment with subcutaneous octreo-
tide A. Posterior duodenal wall
D. Continue intravenous iron therapy alone B. Gastric cardia
E. Initiate treatment with oral estrogen C. Greater gastric curvature
D. Gastric incisura
CORRECT ANSWER: C E. Pre-pyloric gastric antrum

RATIONALE CORRECT ANSWER: A


This patient has the typical pattern of bleeding
seen with small bowel angioectasias, a common RATIONALE
finding in elderly patients with aortic stenosis. An ulcer’s risk of re-bleeding depends on many
Evidence supports medical therapy when bleeding factors including its location. Ulcers on the
and anemia persist despite aggressive endoscopic posterior duodenal wall are typically fed by large
interventions which she has already undergone. branches of the gastroduodenal artery making
Of medical therapies, octreotide is commonly tried them more likely to rebleed even with endoscopic
and has mounting evidence to support its use. therapy. Ulcers on the posterior portion of the
Thalidomide also holds some promise though it lesser curvature of the stomach are also at a higher
has more side-effects and is more logistically dif- risk of rebleeding. The other locations do not con-
ficult to obtain for patients because of its nefarious vey a clearly increased risk of rebleeding.
history. Hormonal therapy has fallen out of favor
because of its potential cardiovascular complica- REFERENCE
tions and questionable efficacy for this indication. Barkun AN, Bardou M, Kuipers EJ. International
Retrograde enteroscopy would be unlikely to consensus recommendations on the manage-
result in definitive therapy given the distribution ment of patients with nonvariceal upper gastro-
of her previous findings and would not be without intestinal bleeding. Ann Intern Med. 2010 Jan
risk in an elderly patient. Her bleeding is unlikely 19;152(2):101-13.
to be robust enough to be detectable and treatable
with angiography. Though maintaining her on IV
iron is reasonable, doing this alone is not the best Question 17
approach, as she remains transfusion dependent A 79-year-old man with coronary artery disease
despite it. managed with a prior cardiac stent placed several
330 Digestive Diseases Self-Education Program®

years ago who takes daily aspirin monotherapy aspirin immediately is too aggressive as if this
is admitted to the ICU with melena and a sig- strategy were employed, the risk of re-bleeding
nificant hemoglobin drop. An upper endoscopy would exceed that of cardiac complications.
reveals an antral ulcer with a small clot. This Changing treatment from aspirin to clopidogrel
is removed and a pigmented spot is seen on the monotherapy is not advisable as data suggest
ulcer base prompting the placement of three that aspirin + PPI has lower bleeding risk than
hemoclips. H. pylori testing is negative. with clopidogrel alone.

Which of the following is the best strategy for his REFERENCES


medical management after discharge? Barkun AN, Bardou M, Kuipers EJ. International
consensus recommendations on the manage-
A. Twice daily proton pump inhibitor and hold ment of patients with nonvariceal upper gastro-
his aspirin until ulcer healing is confirmed intestinal bleeding. Ann Intern Med. 2010 Jan
B. Twice daily proton pump inhibitor and hold 19;152(2):101-13.
his aspirin for one month Biondi-Zoccai GG1, Lotrionte M, Agostoni
C. Change his aspirin to clopidogrel P, et al. A systematic review and meta-analysis
D. Twice daily proton pump inhibitor and im- on the hazards of discontinuing or not adher-
mediate resumption of aspirin ing to aspirin among 50,279 patients at risk
E. Twice daily proton pump inhibitor and hold for coronary artery disease. Eur Heart J. 2006
his aspirin for one week Nov;27(22):2667-74.

CORRECT ANSWER: E
Question 18
RATIONALE An otherwise healthy 64-year-old woman is
Balancing the risk of re-bleeding with the risk admitted with red hematemesis. She is hemody-
of thrombogenic complications is a common namically stable. Her hemoglobin has dropped
clinical challenge. Though aspirin avoidance is by three gm/dL from her baseline two months
commonly recommended after ulcer detection ago. She has never had hematemesis before
because aspirin has been associated with a two- but she has had intermittent melena every few
fold increase in the risk of recurrent bleeding, months and has been maintained on oral iron for
data suggest that stopping aspirin results in in- mild anemia for the past two years. A CT scan
creased mortality. The increased risk of throm- on admission was unremarkable other than not-
bogenic complications typically starts seven to ing a medium-sized hiatus hernia. You perform
10 days after aspirin has been held as inhibited urgent upper endoscopy which reveals the fol-
platelets tend to remain in the circulation for this lowing finding in the gastric cardia, just distal to
long. This patient’s risk of rebleeding is rela- the GE-junction:
tively low in light of the absence of visible vessel,
the endoscopic treatment with hemoclips and
the use of aggressive PPI therapy. Given these
considerations, since it is needed for cardiac pro-
tection, his aspirin should be resumed as quickly
as possible. Holding it until ulcer healing is
confirmed could result in a several month aspirin
abstinence which is too long. Even one month of
aspirin cessation is likely too long in the setting
of concomitant twice daily PPI use. Resuming
Chapter 12 — Gastrointestinal bleeding 331

Which of the following is most likely to result in more likely to have recurrent ulcer hemor-
definitive therapy for this lesion? rhage than controls
C. The dose of NSAIDs does not influence the
A. Treat the ulcer with endoclips bleeding risk associated with their use
B. Apply argon plasma coagulation to the ulcer D. Less than 60 percent of ulcer bleeding will
C. Add sucralfate to her daily proton pump inhibitor resolve without treatment
D. Inject the ulcer base with diluted epinephrine E. Bleeding is more likely to occur after being
E. Refer to surgery for hiatus hernia repair on NSAIDs for more than six months than
during the initial treatment period
CORRECT ANSWER: E
CORRECT ANSWER: B
RATIONALE
This patient’s linear ulcer wwithin the hia- RATIONALE
tus hernia sac is classic for a Cameron lesion. Peptic ulcers remain the most common cause of
Though rarely the cause of acute hematemesis, it upper GI hemorrhage. Patient age, prior pep-
can occur. It is also the most likely cause of her tic ulcer disease bleeding and medications are
previous intermittent melena. Since the lesion important risk factors. Corticosteroids alone
is clean-based and not actively bleeding, acute are not associated with an increased risk of ulcer
endoscopic therapy with injection, APC or clips bleeding but when taken with NSAIDs they
is not clearly necessary and would not provide double the risk of complications compared to
definitive therapy. Though PPI may help, there NSAIDs alone. NSAIDs increase the risk of ulcer
is no clear role for adding sucralfate and the complications like bleeding in a dose dependent
lesion is likely to persist because of repeated mu- fashion and they are more likely to cause compli-
cosal prolapse across the hiatus hernia. Surgical cations in their first month of use. Ulcer bleed-
repair of the hiatus hernia would result in defini- ing will resolve spontaneously in 75-80 percent
tive treatment. of patients. Patients who have bled from ulcers
are 10-20 times as likely to develop recurrent
REFERENCES ulcer bleeding than controls.
Kapadia S, Jagroop S, Kumar A. Cameron
ulcers: an atypical source for a massive upper REFERENCE
gastrointestinal bleed. World J Gastroenterol. Laine L, Jensen DM. Management of patients
2012 Sep 21;18(35):4959-61. with ulcer bleeding. Am J Gastroenterol.
Zullo A, Manta R, De Francesco V et al. Cam- 2012;107: 345-60.
eron lesions: A still overlooked diagnosis. Case
report and systematic review of literature. Clin Res
Hepatol Gastroenterol. 2018 Dec;42(6):604-609. Question 20
A 45-year-old man presents with hematemesis and
melena. He is healthy but takes ibuprofen regular-
Question 19 ly for arthralgias. He is tachycardic to 105 bpm but
Which of the following statements regarding his blood pressure is 130/72 mm Hg. His hemo-
peptic ulcers and bleeding is most accurate? globin is noted to be down three grams from when
it was checked during a routine health screening
A. Corticosteroid use alone is associated with seven months ago. You perform urgent upper
an increased risk of ulcer bleeding endoscopy and find a six mm ulcer in his duodenal
B. Even with documented healing, patients bulb. You remove adherent clot and discover a
who have bled from ulcers are 10 – 20 times non-bleeding visible vessel at the ulcer’s base.
332 Digestive Diseases Self-Education Program®

Which of the following statements regarding endo- bright red rectal bleeding. He sees the blood pri-
scopic therapy in this patient is most accurate? marily coating the surface of his stool. He does
not have abdominal pain nor diarrhea and he has
A. Endoclips should be avoided as they can in- not lost weight. His hemoglobin has remained
hibit ulcer healing stable. You perform colonoscopy which reveals
B. Since there is no active bleeding, epinephrine the following finding:
injection alone is appropriate
C. If multipolar coagulation is used, short burst
of high energy should be employed
D. Endoclips are more likely to provide definitive
hemostasis than injection alone
E. Thermal therapy is more likely to provide
definitive hemostasis than Endoclips

CORRECT ANSWER: D

RATIONALE
Given the presence of an ulcer with high-risk stig- This patient most likely has a history of which of
mata, (a visible vessel), endoscopic therapy should the following:
be employed. Endoclips have proven safe and
there is no evidence that they cause tissue injury A. Cirrhosis
or inflammation or that they impair ulcer heal- B. Systemic Sclerosis (Scleroderma)
ing. When the three main hemostatic modalities, C. Prostate Cancer
injection therapy, thermal therapy and mechanical D. Melanoma
therapy have been compared, injection alone has E. Heart Failure
been shown to be less effective than dual modality
therapy and to either mechanical or thermal thera- CORRECT ANSWER: C
py alone but no studies have shown thermal therapy
alone to be superior to mechanical therapy alone. RATIONALE
If multipolar coagulation is used, it should be used This patient’s rectal findings are consistent with
with low energy applied over a long duration. Radiation Proctitis which most commonly occurs
in male patients following treatment for prostate
REFERENCES cancer. It is important to be able to make this
Kovacs TOG, Jensen DM. Endoscopic treatment diagnosis visually, as biopsying this mucosa can
of peptic ulcer bleeding. Curr Treat Opt Gastroen- lead to fistulization and is therefore relatively
terol. 2007;10:143-148. contraindicated. Systemic sclerosis is associ-
Vergara M, Bennett C, Calvet X, Gisbert JP. ated with GAVE but not rectal telangiectasias
Epinephrine injection versus epinephrine injec- like these. Melanoma frequently metastasizes to
tion and a second endoscopic method in high risk the GI tract but it does not have this appearance.
bleeding ulcers. Cochrane Database Syst Rev 2014, Heart failure may be linked to GI tract angioecta-
Oct 13: CD005584. sias but not in a confluent pattern in the rectum
like this.

Question 21 REFERENCE
An otherwise healthy 82-year-old man presents Tabaja L, Sidani SM. Management of Radiation
with a several month history of intermittent Proctitis. Dig Dis Sci. 2018 Sep;63(9):2180-2188.
Chapter 12 — Gastrointestinal bleeding 333

Question 22 is seen here. They are most commonly seen in men


A 75-year-old man is admitted with red he- with cardiovascular co-morbidities or who are on
matemesis and melena for one day. He is on dual dialysis and they are associated with NSAID use
anti-platelet therapy for a recent cardiac stent and which may contribute to the epithelial degradation
he also takes ibuprofen several times per week for superficial to the vessel. Multiple treatment mo-
knee pain. His hemoglobin has dropped by four dalities have proven successful including thermal
grams from his baseline. He is tachycardic but therapy and endoclips and there is no higher risk of
normotensive. You perform urgent Upper Endos- perforation with treating these lesions.
copy and find the following lesion in his proximal
gastric fundus: REFERENCES
Hae Jin Shin, Jong Seok Ju, Ki Dae Kim, Seok
Won Kim, et al. Risk Factors for Dieulafoy Lesions
in the Upper Gastrointestinal Tract. Clin Endosc.
2015 May; 48(3): 228–233.
Sone Y, Kumada T, Toyoda H, et al. Endo-
scopic management and follow up of Dieulafoy
lesion in the upper gastrointestinal tract. Endos-
copy. 2005 May;37(5):449-53.

Question 23
A 77-year-old man with CAD, COPD, prostate
You irrigate the site and see no evidence of un- cancer with prior radiation therapy and an aortic
derlying mass, mucosal ulceration or associated aneurysm repair presents with hematemesis fol-
angioectasia but the bleeding persists. lowed by hematochezia. He is tachycardic to the
Which of the following statements regarding this 110’s with an initial blood pressure of 84/60 mm
lesion is most accurate? Hg that improves to 95/73 mm Hg with one liter of
intravenous fluid. He is admitted to the ICU and
A. The bleeding is from a normal mucosal capil- you perform urgent Upper Endoscopy. His esopha-
lary gus and stomach appear normal but when you push
B. Thermal therapy should not be used because the scope to the third segment of the duodenum,
of a high perforation risk you see a large amount of fresh red blood and clot
C. The bleeding is from a dilated aberrant sub- that cannot be cleared adequately with lavage.
mucosal artery
D. This lesion more commonly occurs more dis- Which is the following is the best next step in this
tally in the GI tract patient’s management?
E. Since there is no ulcer, this lesion is unrelated
to his NSAID use A. Perform colonoscopy because he also present-
ed with hematochezia
CORRECT ANSWER: C B. Send him for red blood cell scintigraphy
C. Send him for mesenteric angiography
RATIONALE D. Place him on a proton pump inhibitor drip
This is a Dieulafoy lesion: a dilated aberrant sub- and repeat upper endoscopy in a few hours
mucosal artery that erodes the overlying epithelium E. Send him for a stat CT scan of his abdomen
in the absence of an associated ulcer. These lesions
occur most commonly in the proximal stomach as CORRECT ANSWER: E
334 Digestive Diseases Self-Education Program®

This primary concern in this case is that this pa- With voluminous hematemesis, especially from a
tient is bleeding from an aorto-enteric fistula. His proximal source like the esophagus, airway pro-
presentation, history of a AAA repair and finding tection is crucial so this patient should be intu-
of copious blood in the third portion of the duode- bated. Patients like this are at high risk to develop
num all suggest this diagnosis. Once considered, infected ascites so IV antibiotics should be given.
it is paramount to rule this diagnosis out quickly Antibiotics have been shown to decrease mortal-
as it has a very high mortality if not corrected im- ity in cirrhotic patients admitted with GI bleeding.
mediately. The most effective way to assess for an Somatostatin analogs decrease portal inflow by
A-E fistula is with an abdominal CT scan which causing splanchnic vasoconstriction and have been
can be obtained easily and performed quickly. proven to achieve hemostasis and decrease the risk
None of the other choices would be as an appropri- of rebleeding. One has to be cautious with resus-
ate next step in this patient’s management. citation efforts, as excessive resuscitation can lead
to accelerated bleeding due to increased portal
REFERENCE pressures. However, this patient’s Hemoglobin
Raman SP, Kamaya A, Federle M, Fishman EK. concentration is well below the threshold that war-
Aortoenteric fistulas: spectrum of CT findings. rants transfusion, so giving him PRBCs is appro-
Abdom Imaging. 2013 Apr;38(2):367-75. priate. In the acute setting of an upper GI bleed,
proton pump inhibitors work to help optimize
platelet function by increasing gastric pH. Since
Question 24 the source here is varices in the more pH neutral
A 52-year-old man with NASH-cirrhosis is admit- esophageal environment, intravenous PPI likely
ted to the ICU with red hematemesis and hemody- has little effect in the acute setting. However after
namic instability. For the past few months he has band ligation is performed, it may help decrease
been maintained on diuretics but has still required the risk of forming post-banding ulcers. Since this
frequent paracenteses for ascites management. An patient’s banding was performed a month ago, this
Upper Endoscopy four weeks ago revealed only episode of bleeding is more likely to be from recur-
large esophageal varices which were incompletely rent varices than from a post-banding ulcer.
eradicated with banding but the patient did not
show up for his scheduled repeat Upper Endos- REFERENCES
copy last week. His initial hemoglobin is 5.8 g/dL. Garcia-Tsao G, Sanyal AJ, Grace ND, et al. Preven-
His INR is 1.8 and his platelet count is 94K. tion and management of gastroesophageal varices
and variceal hemorrhage in cirrhosis. Hepatology
Which of the following treatment options is 2007; 46:922.
LEAST likely to benefit this patient? Tripathi D, Stanley AJ, Hayes PC, et al. U.K.
guidelines on the management of variceal haemor-
A. Intravenous proton pump inhibitor drip rhage in cirrhotic patients. Gut 2015; 64:1680.
B. Intravenous somatostatin analog
C. Endotracheal intubation
D. Intravenous antibiotics Question 25
E. Packed red blood cell transfusion A 27-year-old woman presents to the Emergency
Department with a history of melena. Her hemo-
CORRECT ANSWER: A globin is noted to be 11.2 g/dL with no prior values
for comparison. Her BUN is not elevated. She de-
RATIONALE nies abdominal pain or weight loss. She has been
It is important to understand the initial manage- training for a marathon and so has been taking
ment of patients with bleeding esophageal varices. ibuprofen several days per week. She is hemody-
Chapter 12 — Gastrointestinal bleeding 335

namically stable. You perform Upper Endoscopy associated with nonsteroidal antiinflammatory
which reveals a five mm clean-based ulcer in her drugs. Acid Suppression Trial: Ranitidine versus
gastric antrum and so she is discharged from the Omeprazole for NSAID-associated Ulcer Treat-
Emergency Department. Biopsies from the ulcer ment (ASTRONAUT) Study Group. N Engl J Med
margins show inflammatory changes only and ex- 1998; 338:719.
tensive biopsies from the rest of the stomach show ASGE Standards of Practice Committee,
no evidence of H. pylori. Banerjee S, Cash BD, et al. The role of endoscopy
in the management of patients with peptic ulcer
Which of the following is the most appropriate disease. Gastrointest Endosc 2010; 71:663.
management strategy for this patient?

A. Repeat Upper Endoscopy in eight weeks to doc- Question 26


ument ulcer healing and rule out malignancy A 42-year-old healthy Eastern European male
B. Discontinue NSAIDs and treat her with a pro- presents to your office following a recent hospi-
ton pump inhibitor for eight weeks talization for peptic ulcer disease. At that time, he
C. Discontinue NSAIDs and treat with PPI for six presented with 24 hours of melenic stools and was
months found to have a hemoglobin 7.6 g/dL and BUN 48
D. Treat with both an H2 receptor antagonist and mg/dL. He was resuscitated, intravenous proton
PPI for eight weeks pump inhibitor was initiated and an upper endos-
E. Check a urease breath test to make sure H. copy was performed, showing a 1 cm gastric antral
pylori infection was not missed on biopsy ulcer with a non-bleeding visible vessel treated
with epinephrine injection and heater probe cau-
CORRECT ANSWER: B tery. Biopsies of the gastric mucosa were positive
for Helicobacter pylori and quadruple therapy
RATIONALE for 14 days was initiated. A follow-up endoscopy
This patient had a small NSAID-related gastric eight weeks later showed complete healing of the
ulcer the treatment of which should be NSAID ces- ulcer and repeat gastric biopsies were negative for
sation and PPI therapy for eight weeks. As long as Helicobacter pylori. Current medications include
the inciting cause (NSAIDs) is removed, she does pantoprazole 40 mg twice daily since hospital
not need prolonged PPI therapy as she has no other discharge. Today in the office he is asymptomatic
symptoms or risk factors for PUD. Since her ulcer and is wondering about recommendations for his
is small, in a typical location, has unremarkable medications going forward. Which of the follow-
biopsies and has a clear etiology, the risk for associ- ing is the most appropriate medical management
ated malignancy is too low to warrant surveillance strategy for this patient?
endoscopy. PPI’s have been proven to have higher
efficacy for ulcer healing than H2RA’s and there A. Decrease PPI to daily
are no data that show an additive benefit if these B. Decrease PPI to daily and initiate H2-receptor
are combined but would result in higher costs and antagonist
more a more complicated regimen for the patient. C. Discontinue PPI and initiate H2-receptor
With gastric biopsies showing no evidence of H. antagonist
pylori infection, there is no role for additional H. D. Discontinue PPI
pylori testing with a urease breath test.
CORRECT ANSWER: D
REFERENCES
Yeomans ND, Tulassay Z, Juhász L, et al. A com- RATIONALE
parison of omeprazole with ranitidine for ulcers Patients with bleeding peptic ulcer disease sec-
336 Digestive Diseases Self-Education Program®

ondary to H. pylori infection are typically man- include hemoglobin 8.2 g/dL and BUN 28 mg/
aged with antibiotic treatment of the infection in dL, creatinine 1.4 mg/dL. Two large bore IVs are
addition to PPI therapy, according to accepted placed, resuscitation with IV crystalloid solution is
management standards for H. pylori (covered initiated, along with an IV pantoprazole drip. The
elsewhere). An important issue arises when con- patient then undergoes an upper endoscopy. Find-
sidering the need for continued PPI therapy after ings reveal a scant amount of coffee ground materi-
treating the infection. Data supports the notion al within the stomach and duodenal bulb. Multiple
that eradication of H. pylori infection is superior small lesions are then identified in the duodenum,
to short-term and long-term anti-secretory ther- with a representative image shown.
apy in reducing recurrent ulcer bleeding risk and
that the overall rebleeding risk is low (1.5 percent)
once infection has been eradicated. In general,
once H. pylori eradication is documented and
ulcer disease healed, it is unnecessary to continue
anti-secretory therapy unless there is an addition-
al compelling indication for continued therapy.
Therefore, in this otherwise healthy asymptomatic
low risk patient, it would be best to discontinue
PPI therapy. If this patient had other indications
to continue PPI therapy, such as ongoing NSAID
requirement or had an idiopathic bleeding peptic Image Courtesy of Brandon Sprung, MD
ulcer, then maintenance PPI therapy could be
recommended. There is no clinical role for H2- Which of the following is the best endoscopic man-
blockers in this scenario. agement strategy for this finding?

REFERENCES A. No endoscopic therapy indicated


Gisbert JP, Khorrami S, Carballo F et al. Meta- B. Epinephrine injection monotherapy
analysis: Helicobacter pylori eradication therapy C. Heater probe coagulation to area of hemor-
vs. antisecretory non-eradication therapy for the rhage monotherapy
prevention of recurrent bleeding from peptic ulcer. D. Hemoclip placement to area of hemorrhage
Aliment Pharmacol Ther 2004;19:617 – 29. monotherapy
Laine L, Jensen, DM. Management of patients E. Epinephrine injection in combination with
with ulcer bleeding. Am J Gastroenterol. 2012;107: heater probe coagulation
345-60.
CORRECT ANSWER: A

Question 27 RATIONALE
A 64-year-old female presents to the Emergency The endoscopic findings in this scenario represent
Department with 24 hours of melena. Her history a clean-based duodenal bulb ulcer without high
includes atrial fibrillation managed on warfarin and risk stigmata of recent hemorrhage. This lesion is
rheumatoid arthritis on naproxen. On presentation, of a low rebleeding risk (less than 5 percent), and
heart rate 86 bpm, blood pressure 108/62 mmHg, therefore no endoscopic intervention is necessary.
oxygen saturation 99 percent on room air. Perti- Therefore, answer A is correct.
nent exam findings reveal irregular heart rate and The other answer choices would be reserved
a soft, non-tender, non-distended abdomen with for lesions with a high-risk of rebleeding, such as a
normoactive bowel sounds. Relevant lab studies non-bleeding visible vessel or active bleeding.
Chapter 12 — Gastrointestinal bleeding 337

REFERENCE Initiation of IV PPI, transfusions of blood


Laine L, Jensen, DM. Management of patients products, NG lavage and administration of proki-
with ulcer bleeding. Am J Gastroenterol. 2012;107: netics are reasonable to consider as part of man-
345-60. agement of upper GI hemorrhage, though they are
performed once the patient has been resuscitated
after presentation.
Question 28
An 82-year-old male with a history of recent REFERENCE
myocardial infarction on aspirin 325 mg daily and Laine L, Jensen, DM. Management of patients
clopidogrel presents with massive hematemesis for with ulcer bleeding. Am J Gastroenterol. 2012;107:
one hour. He was found down at home by a family 345-60.
member, but is awake and alert and following com-
mands at the time of assessment. Initial vital signs
in the Emergency Department reveal heart rate 115 Question 29
bpm and blood pressure 72/40 mmHg. Abdomen is A 28-year-old male presents to the emergency
mildly distended, though non-tender. Hemoglobin department on a Sunday evening with four hours
is 6.8 g/dL, BUN 72, creatinine 1.1 mg/dL. of melena. Pertinent recent history includes a
sprained ankle for which the patient has been us-
What is the most appropriate initial step in manage- ing NSAIDs frequently for seven days. Initial heart
ment of this patient in the Emergency Department? rate is 90 bpm and blood pressure 128/78 mmHg.
Abdominal exam is normal, no tenderness or
A. Placement of two large-bore 18 gauge IVs and distension. Hemoglobin in 9.2 g/dL. Intravenous
administration of crystalloid solution access is obtained, normal saline is started and an
B. Initiate continuous IV proton-pump inhibitor IV infusion of pantoprazole (80 mg bolus followed
infusion by 8 mg / h infusion) is initiated. The following
C. Transfuse packed red blood cells and platelets morning an upper endoscopy is performed. Which
due to severe anemia and use of anti-platelet of the following benefits of PPI therapy before
agents endoscopy would be expected in this patient?
D. Nasogastric lavage to evaluate for grossly
blood aspirate to confirm upper GI bleed A. Reduced mortality from GI bleeding related to
E. Administer IV erythromycin in anticipation of peptic ulcer disease
upper endoscopy B. Reduced need for surgery relating to compli-
cated upper GI hemorrhage
CORRECT ANSWER: A C. Reduced risk of high-risk endoscopic stigmata
requiring endoscopic therapy
RATIONALE D. Reduced risk of rebleeding requiring repeat
All patients presenting with GI hemorrhage require endoscopy
rapid hemodynamic assessment and resuscitation
as the initial step in management. Those with he- CORRECT ANSWER: C
modynamically significant GI hemorrhage require
urgent resuscitation with establishment of IV RATIONALE
access and replacement of circulating volume with Proton-pump inhibitor therapy is an important
crystalloid solutions. This patient presents with hy- part of managing upper GI bleeding. A meta-
potension and tachycardia with syncope, indicative analysis reviewing PPI therapy prior to endoscopy
of a large volume blood loss. Therefore, he requires found that compared to control, PPI significantly
urgent resuscitation and answer A is correct. reduced the chances of finding stigmata of recent
338 Digestive Diseases Self-Education Program®

hemorrhage (active bleeding, non-bleeding visible demic for H. pylori infection. When peptic ulcers
vessel or adherent clot) at initial endoscopy (OR are found at the time of endoscopy, they should be
= 0.67) and need to undergo endoscopic therapy classified according to their risk of rebleeding, of-
at that initial endoscopy (OR = 0.68). In this ten using the Forrest classification system. Specific
analysis, there were no significant differences in stigmata include active arterial spurting, active
mortality, re-bleeding or surgery between PPI and oozing, non-bleeding visible vessel, adherent clot,
control. Therefore, answer C is correct. flat pigmented spot, and clean base. The lesions
at highest risk for rebleeding, including active
REFERENCE bleeding and non-bleeding visible vessel, require
Sreedharan A, Martin J, Leontiadis GI et al. definite endoscopic therapy to achieve and main-
Proton pump inhibitor treatment initiated prior tain hemostasis. However, lesions at low risk for
to endoscopic diagnosis in upper gastrointestinal re-bleeding, including those with a flat pigmented
bleeding. Cochrane Database Syst Rev 2010 (7): spot (seven percent re-bleeding risk) or clean-base
CD005415. (three percent re-bleeding risk) are unlikely to
cause significant hemorrhage and re-bleeding and
do not require endoscopic therapy. In this case, an
Question 30 ulcer with a flat pigmented spot was found at the
A 52-year-old male originally from China pres- time of endoscopy, and therefore answer D is cor-
ents to the Emergency Department with fatigue rect. The other answers might be a good choice for
and epigastric abdominal pain for eight days. He lesions with higher risk stigmata of hemorrhage.
denies prior medical history and reports no history
of NSAID use or anti-coagulation. Vital signs are REFERENCE
normal. Exam reveals mildly tender epigastrium Laine L, Jensen, DM. Management of patients
without rebound or guarding. Hemoglobin is with ulcer bleeding. Am J Gastroenterol. 2012;107:
7.8 g/dL, BUN 18 mg/dL, creatinine 0.9 mg/dL. 345-60.
Intravenous PPI is initiated and he undergoes an
upper endoscopy which reveals a one cm ulcer in
the gastric antrum with a flat pigmented spot. Question 31
Which of the following is the appropriate endo- A 24-year-old male presents to the Emergency
scopic management for this patient’s ulcer? Department with two days of aching epigastric pain
and an episode of black stool one day ago. He has
A. Endoscopic hemoclip placement across the no significant past medical history other than a
ulcer edges recent sprained ankle for which he has been taking
B. Thermal probe therapy to the base of the ulcer ibuprofen four times daily for the past two weeks.
C. Epinephrine injection into the base of the On presentation, relevant vital signs are heart rate
ulcer 86 bpm, blood pressure 126/82 mmHg, SaO2 99
D. No endoscopic therapy is indicated percent on room air. He is alert and oriented and
E. Epinephrine injection plus endoscopic hemo- abdominal exam reveals mild tenderness over the
clip placement across the ulcer edges epigastrium without rebound or guarding, and nor-
moactive bowel sounds. Rectal exam reveals brown
CORRECT ANSWER: D stool. Hemoglobin is 13.9 g/dL. An upper endos-
copy is performed revealing a one cm clean-based
RATIONALE gastric antral ulcer with no high-risk stigmata of
This patient presents with concern for an upper bleeding. Stool antigen H pylori testing is negative.
GI source of anemia given his epigastric pain and Which of the following is the next best step in
tenderness and origin from a region highly en- management of this patient?
Chapter 12 — Gastrointestinal bleeding 339

A. Admit for 24 hour observation with intrave- ing or mortality, such as those with hemodynamic
nous PPI therapy changes, advanced age, significant co-morbidities,
B. Admit to the medical floor for monitoring and or high-risk stigmata of hemorrhage on endos-
intravenous PPI therapy copy are candidates for hospital admission.
C. Admit to the medical floor for monitoring and
oral PPI therapy REFERENCES
D. Admit to the intensive care unit for continuous Bethea ED, Travis AC, Saltzman JR. Initial assess-
intravenous PPI therapy ment and management of patients with nonvari-
E. Discharge home with oral PPI therapy ceal upper gastrointestinal bleeding. J Clin Gastro-
enterol. 2014 Nov-Dec;48(10):823-9.
CORRECT ANSWER: E Cipolletta L, Bianco MA, Rotondano G, et al.
Outpatient management for low-risk nonvariceal
RATIONALE upper GI bleeding: a randomized controlled trial.
This patient presents with a low risk (Forrest III) Gastrointest Endosc. 2002; 55:1–5.
clean-based gastric ulcer which is presumed to be Laine L, Jensen, DM. Management of patients
secondary to recent heavy NSAID use. Guidelines with ulcer bleeding. Am J Gastroenterol. 2012;107:
suggest performing a risk assessment on patients 345-60.
presenting with upper GI bleeding in order to Monteiro S, Gonçalves TC, Magalhães J, Cot-
assist management decisions such as timing of ter J. Upper gastrointestinal bleeding risk scores:
endoscopy, level or care and disposition. Who, when and why? World Journal of Gastroin-
There are several risk-assessment scores testinal Pathophysiology. 2016;7(1):86-96.
available both pre- and post-endoscopy to de-
termine a patient’s risk for rebleeding or mortal-
ity, which can directly affect the management. Question 32
This patient presents with normal vital signs, A 46-year-old male with no past medical his-
no co-morbidities and is of relatively young age, tory presents to the Emergency Department with
with normal blood work and no active melena. 24 hours of severe, massive hematochezia. Vital
In addition, no high-risk bleeding stigmata were signs are: heart rate 118 bpm, blood pressure
found on endoscopy. Therefore, for example, the 86/42 mmHg. Examination reveals a soft non-
patient’s complete Glasgow-Blatchford Score tender abdomen without distension. There are no
is 0 and AIM65 Score is 0, so he would be con- stigmata of liver disease on examination. Intrave-
sidered low risk. Since the patient is at low risk nous crystalloid and PPI therapy are initiated. An
for re-bleeding and significant morbidity or urgent endoscopy is planned to rule-out an upper
mortality from the gastric ulcer, it would be safe GI bleeding source.
to discharge home with oral PPI therapy. Ideal
candidates for discharge right after endoscopy are Which of the following is the most common cause
those with normal hemodynamics with a stable of severe upper GI hemorrhage?
blood count, no significant co-morbidities, and
who are reliable for follow-up. Evidence supports A. Peptic ulcer disease
that early discharge is safe in low risk peptic ulcer B. Esophageal varices
patients, with a very low rate of re-bleeding and C. Gastroduodenal erosive disease
a significantly lower cost compared to those who D. Mallory-Weiss tear
are hospitalized. Admission to the hospital and E. Angioectasia (AVM)
intravenous PPI is not necessary for such low risk
patients, therefore the other answers are incor- CORRECT ANSWER: A
rect. Patients who are at higher risk for re-bleed-
340 Digestive Diseases Self-Education Program®

RATIONALE D. Adherent clot (Forrest IIb)


The most common cause of upper GI hemorrhage E. Flat pigmented spot (Forrest IIc)
is peptic ulcer disease (gastro-duodenal ulcers).
Peptic ulcer disease is estimated to cause 31 to 67 CORRECT ANSWER: A
percent of upper GI bleeding. The other answers
represent additional possible etiologies of severe RATIONALE
upper GI bleeding, though they are all less com- As supported by guideline recommendations, it is
mon compared to peptic ulcer disease in non- important to document stigmata of hemorrhage
cirrhotic patients. Recent analysis suggests that when evaluating peptic ulcers. Such stigmata are
esophageal varices are the most common source associated with varying rates of rebleeding, surgery
of severe upper GI bleeding in cirrhotic patients, and mortality in the absence of endoscopic therapy,
however there is no evidence on presentation to and are therefore used to guide the decision for,
suggest that this patient is cirrhotic. and choice of, endoscopic therapy. Active arte-
rial spurting lesions are at highest risk for further
REFERENCES bleeding with rates greater than 90 percent. Ad-
Camus M, Khungar V, Jensen, DM et al. Origin, ditional rebleeding risk with other stigmata include
Clinical Characteristics and 30-Day Outcomes non-bleeding visible vessel (50 to 60 percent),
of Severe Hematochezia in Cirrhotics and Non- active venous or capillary oozing (10 to 30 percent),
cirrhotics. Dig Dis Sci (2016) 61: 2732. adherent clot (20 to 35 percent), flat pigmented
Rotondano G. Epidemiology and diagno- spot (seven to 10 percent) and clean-base (three to
sis of acute nonvariceal upper gastrointestinal five percent). Endoscopic therapy is recommended
bleeding. Gastroenterol Clin North Am. 2014 for any lesion with active bleeding or a non-bleed-
Dec;43(4):643-63. ing visible vessel due to the high risk of re-bleeding
without endoscopic therapy, while lesions with
flat pigmented spot or a clean base do not require
Question 33 endoscopic therapy due to their inherent low risk
A 54-year-old female presents for evaluation for rebleeding without intervention.
of hematemesis. Her medical history includes
fibromyalgia, for which she takes naproxen. On REFERENCES
presentation, heart rate is 110 bpm, blood pressure de Groot, NL et. al Reassessment of the predictive
102/68 mmHg. Examination reveals moderately value of the Forrest classification for peptic ulcer
tender epigastrium without rebound or guarding. rebleeding and mortality: can classification be
Hemoglobin is 6.9 g/dL, BUN 32 mg/dL, creati- simplified? Endoscopy 2014; 46(01): 46-52.
nine 1.1 mg/dL. Two large-bore IVs are placed, Laine L, Jensen, DM. Management of patients
fluid resuscitation and intravenous continuous PPI with ulcer bleeding. Am J Gastroenterol. 2012;107:
is initiated, packed red blood cells are ordered and 345-60.
an urgent upper endoscopy is planned.

Which of the following endoscopic stigmata of Question 34


peptic ulcer disease at time of endoscopy repre- A 76-year-old female presents to the Emergency
sents the highest risk of re-bleeding in the absence Department with 12 hours of black tarry stool and
of endoscopic therapy? one episode of hematemesis. Her medical history
includes coronary artery disease on aspirin 325
A. Active arterial spurting (Forrest Ia) mg daily. On evaluation, she reports moderate
B. Active oozing (Forrest Ib) epigastric abdominal pain. Heart rate is 98 bpm,
C. Non-bleeding visible vessel (Forrest IIa) blood pressure 118/68 mmHg, and there is mild
Chapter 12 — Gastrointestinal bleeding 341

epigastric tenderness on exam without rebound or fore, the best management strategy for this case is
guarding. Hemoglobin is 8.4 g/dL, BUN 38 mg/ repeat EGD for second attempt at ulcer hemostasis
dL, creatinine 0.9 mg/dL. Intravenous PPI infusion to attempt to reduce the need for surgery.
is initiated and EGD is performed, revealing fresh Answer A is not appropriate for this patient
blood in the stomach and duodenum with a large with recurrent active GI hemorrhage which re-
two cm duodenal bulb ulcer with an oozing bleed- quires intervention. Answer B is incorrect because
ing vessel. This was treated with epinephrine and there is no need to delay treatment in order to
hemoclip placement over the vessel. The patient did localize the bleeding location in this patient with
well initially, until 48 hours later when she devel- a known high-risk peptic ulcer. Answers D and
oped recurrent melena with transfusion requiring E are incorrect because surgical or angiographic
anemia and worsening hemoglobin to 5.6 g/dL. intervention for peptic ulcer bleeding should be
reserved for failure of a second endoscopic attempt
Which of the following is the most appropriate at treatment, or for other ulcer complications.
management strategy for this patient?

A. Continue supportive care, PPI and blood REFERENCES


transfusion Laine L, Jensen, DM. Management of patients
B. Obtain nuclear tagged RBC scan to localize the with ulcer bleeding. Am J Gastroenterol. 2012;107:
source of bleeding 345-60.
C. Repeat EGD for second attempt at ulcer hemo- Lau JY, Sung JJ, Lam YH et al. Endoscopic
stasis retreatment compared with surgery in patients with
D. Surgical consultation for surgical management recurrent bleeding after initial endoscopic control of
of refractory duodenal ulcer bleeding bleeding ulcers. N Engl J Med 1999; 340: 751 – 6.
E. Intervention Radiology consultation for angio-
graphic hemostasis management
Question 35
CORRECT ANSWER: C A 68-year-old male with a history of atrial fibril-
lation on therapeutic warfarin and aspirin 81 mg
RATIONALE daily presents to the Emergency Department with
This patient presented with a high rebleeding risk two hours of severe large volume bright red blood
duodenal ulcer which was appropriately initially per rectum. Vital signs reveal an irregular heart rate
treated with combination therapy (epinephrine at 138 bpm, blood pressure 80/56 mmHg. Exami-
plus hemoclip). However, the patient developed nation reveals a soft non-tender abdomen without
rebleeding shortly after despite this therapy, which distension and no stigmata of liver disease on
can be seen in 10 to 25 percent of cases. Current examination. Labs reveal INR 2.2, hemoglobin 7.4
evidence supports the role of a second endoscopy g/dL, platelets 220,000/uL. Intravenous access is
with repeat endoscopic therapy for clinically sig- established and crystalloid resuscitation is initiated.
nificant recurrent upper GI bleeding after initial
hemostasis. A randomized trial of endoscopic What is the next best step in management of
re-treatment versus surgery for recurrent ulcer this patient?
bleeding after initial endoscopy revealed a 73 per-
cent long-term bleeding control rate with repeat A. Emergent reversal of coagulopathy
endoscopy. Salvage surgery was required in 27 B. Rapid bowel prep and urgent colonoscopy
percent due to either endoscopic treatment failure C. Urgent upper endoscopy
or perforation. There was a significantly higher D. Nuclear tagged RBC scan
rate of complications in the surgery group. There- E. Angiography
342 Digestive Diseases Self-Education Program®

CORRECT ANSWER: C sure 138/86 mmHg without orthostatic changes.


Examination reveals an alert and oriented female
RATIONALE with a soft obese non-tender abdomen without
Hematochezia with associated hemodynamic distension and no stigmata of liver disease on ex-
instability in an at-risk patient should prompt amination. Pertinent labs reveal hemoglobin 12.8
concern for an upper GI source of bleeding. In that g/dL, platelets 350,000/uL, INR 0.9. Intravenous
setting, an upper GI endoscopy should be pursued access is established and crystalloid fluid is initi-
to first rule-out such a brisk upper GI bleeding ated.What is the next best step in management of
source, prior to initiating a search for a lower GI this patient?
bleeding source. In addition, this patient’s use
of oral anticoagulants should prompt concern A. Surgical consultation
for high-risk for upper GI hemorrhage, therefore B. Bowel prep until clear effluent then
increasing the pretest probability of an upper GI colonoscopy
bleed. A nasogastric tube with nasogastric lavage C. Upper endoscopy
can be considered in these patients, though is D. Nuclear tagged RBC scan
uncomfortable and does not entirely rule-out an E. Angiography
upper GI bleeding source.
The other diagnostic answer choices (B, D, CORRECT ANSWER: B
E) are options for evaluating and managing a GI
bleed should a brisk upper GI source of bleeding RATIONALE
be ruled-out, but are not the next best choice in This patient presents with hemodynamically stable
management of this patient. Guidelines support painless hematochezia. The leading diagnostic
proceeding safely with upper endoscopy if INR suspicion in this patient is bleeding from diver-
less than 2.5, and therefore it is not necessary ticular disease, which is the most common cause of
to delay upper endoscopy in order to correct the painless hematochezia leading to hospitalization.
coagulopathy in this patient with a hemodynami- Less likely causes include colonic angioectasia,
cally significant GI bleed. Endoscopy should never hemorrhoids, rectal ulcers and inflammatory or
be delayed due to coagulopathy in a patient who is ischemic colitis. For patients presenting with hemo-
hemodynamically unstable. dynamically stable hematochezia, colonoscopy is
the initial diagnostic test of choice after appropri-
REFERENCE ate bowel purge. Unprepped examination is not
Strate LL and Gralnek IM. ACG Clinical Guideline: recommended. Colonoscopy serves to identify and
Management of Patients With Acute Lower Gas- possibly control the site of bleeding, though yield
trointestinal Bleeding. Am J Gastroenterol 2016; is variable (48-90 percent). The mucosa should be
111:459–474. thoroughly lavaged and inspected upon insertion
and withdrawal, and terminal ileum should be in-
tubated to evaluate for blood which could indicate a
Question 36 more proximal source of bleeding.
A 76-year-old female with chronic constipation Answer C (EGD) is unnecessary in a patient
presents to the Emergency Department with six who presents with hematochezia without risk fac-
hours of painless bright red blood per rectum. tors for upper GI bleeding with stable vital. An-
There is no history of prior GI bleeding, melena, swers A, D and E are incorrect as these are not the
hematemesis, cirrhosis, recent polypectomy or use initial recommendations in a patient with hemody-
of blood thinners or NSAIDs. She describes a toilet namically stable GI bleeding, but can be considered
full of blood every 30 minutes during this time. in a patient in whom colonoscopy fails, cannot tol-
Vital signs are: heart rate 82 bpm, blood pres- erate a bowel prep or is hemodynamically unstable.
Chapter 12 — Gastrointestinal bleeding 343

REFERENCE Question 38
Strate LL and Gralnek IM. ACG Clinical Guideline: A 68-year-old female with heart failure with re-
Management of Patients With Acute Lower Gas- duced ejection fraction, chronic renal insufficiency,
trointestinal Bleeding. Am J Gastroenterol 2016; and irritable bowel syndrome with constipation
111:459–474. presents with acute abdominal pain and hema-
tochezia. The patient describes developing sud-
den left-sided abdominal cramping followed by a
Question 37 brown bowel movement and six hours later devel-
A 64-year-old male with a history of prostate can- oped bright red blood per rectum. Vital signs are
cer treated with radiation therapy presents to your stable and lab testing reveals C-reactive protein 15
office with chronic painless bright red blood per mg/dL (normal 0-10 mg/dL) with normal lactate,
rectum. The patient is otherwise well. Hemoglo- CBC and blood chemistry. Abdominal exam re-
bin is 11.2 g/dL. A colonoscopy is performed after veals mild tenderness to palpation in the left lower
appropriate PEG bowel lavage. On colonoscopy, quadrant. A CT scan of the abdomen is obtained
the rectum is notable for scant fresh blood which and reveals colonic wall thickening and mild peri-
is washed to reveal multiple small actively oozing colonic stranding along the splenic flexure and
angioectasias. What is the next best step in endo- proximal descending colon. What is the most likely
scopic management of this patient? etiology for this patient’s presentation?

A. Epinephrine injection into the bleeding lesion A. Crohn’s colitis


alone B. Infectious colitis
B. Contact bipolar thermal therapy of bleeding C. Diverticulitis
lesion D. Colonic adenocarcinoma
C. Hemoclip placement over bleeding foci E. Ischemic colitis
D. Argon plasma coagulation of bleeding lesions
E. Angiography with embolization CORRECT ANSWER: E

CORRECT ANSWER: D RATIONALE


This patient is presenting with classic history
RATIONALE and imaging findings of ischemic colitis. She has
This patient presents with a stable painless lower several risk factors for ischemic colitis (female,
GI bleeding secondary to radiation proctitis. age, low output heart failure, chronic renal insuffi-
The recommended therapy is non-contact argon ciency, irritable bowel syndrome, constipation). In
plasma coagulation of the angioectasias. Non- addition, symptoms are classic for ischemic colitis,
contact therapy is preferred over contact therapy with sudden cramping abdominal pain followed by
in the treatment of colonic angioectasia because the urge to defecate with a normal bowel move-
of safety and efficacy. The other answers are all ment, followed by hematochezia within 24 hours.
options to obtain hemostasis in various GI bleed Imaging supports the clinical diagnosis with wall
settings, though APC is preferred for mild oozing thickening in a “watershed” location of the colon.
radiation proctitis. Ischemic colitis is the most common presentation
of GI ischemia. It develops from transient altera-
REFERENCE tions in the normal blood flow to the colon with
Strate LL and Gralnek IM. ACG Clinical Guideline: local hypoperfusion and subsequent reperfusion.
Management of Patients With Acute Lower Gas- The splenic flexure and rectosigmoid colon are
trointestinal Bleeding. Am J Gastroenterol 2016; common areas involved by ischemic colitis due to
111:459–474. overlapping blood supplies which are subject to
344 Digestive Diseases Self-Education Program®

ischemia from low flow states (i.e. SMA and IMA B. Repeat upper endoscopy in 24 hours
watershed area). Early colonoscopy with biopsy C. Nuclear tagged RBC scan
within 48 hours is the next suggested diagnostic D. CT angiography
test to confirm the diagnosis, but is not absolutely E. Video Capsule Endoscopy
necessary in the right setting with supportive clini-
cal and radiographic information. Importantly, the CORRECT ANSWER: E
finding of isolated right colon ischemia on imaging
or colonoscopy is high-risk for poor clinical out- RATIONALE
comes including severe bleeding, need for surgical This patient presents with worsening symptom-
intervention and increased mortality and should atic anemia with suspected occult GI blood loss
prompt evaluation of the mesenteric vasculature. and a negative upper endoscopy and colonoscopy,
The other answer choices are less likely given the consistent with small bowel bleeding. The patient’s
patient’s acute classic symptoms, risk factors, and risk factors for GI bleeding primarily include
image findings. LVAD placement and anti-coagulation. Patients
with LVADs are known to be at high risk for devel-
REFERENCE opment of GI bleeding from angioectasias. Often
Brandt LJ, Feuerstadt P, Longstreth GF et al. ACG these lesions are located within the small bowel
Clinical Guideline: Epidemiology, Risk Factors, and present with occult bleeding and anemia, but
Patterns of Presentation, Diagnosis, and Manage- can also present with overt GI bleeding. Given this
ment of Colon Ischemia (CI). Am J Gastroenterol patient’s negative endoscopy and colonoscopy, a
2015; 110:18–44. small bowel source of bleeding should be sought.
Guidelines suggest that a video capsule endoscopy
(VCE) should be considered first-line to evaluate
Question 39 the small bowel for a source of bleeding after up-
A 54-year-old male with non-ischemic cardiomy- per and lower GI sources have been excluded with
opathy, status-post left ventricular assist device standard endoscopic evaluations. If a small bowel
(LVAD) placement six months ago, presents to the source of bleeding is identified on VCE which can
Emergency Department with shortness of breath be associated with anemia, endoscopic therapy is
and fatigue. Medications include warfarin and recommended.
lisinopril. On presentation, he is hemodynamically Answer A is incorrect because a small bowel
stable and awake and alert. Examination reveals source of anemia should be sought in this case and
an LVAD hum over the precordium and a soft it is not appropriate to correct INR in an LVAD
obese non-tender abdomen. Pertinent labs re- patient who is otherwise stable, due to risk for clot
veal hemoglobin 6.9 g/dL (baseline 9.8 g/dL four formation. Oral iron and clinical monitoring are
weeks prior), and INR 2.8. The patient denies me- an option if no source of bleeding is found after a
lena, hematochezia, or hematemesis. Stool is posi- thorough small bowel evaluation and the patient
tive for occult blood. The patient undergoes both remains stable. Answer B is incorrect because
an upper endoscopy to the distal duodenum and there is no clinical suggestion of recurrent overt
a colonoscopy the following day which are both upper GI bleeding to warrant a second look endos-
normal, without evidence for a cause of anemia. copy, and the yield is therefore low. Second look
endoscopy or push enteroscopy can be consid-
What is the next best step in management of this ered for recurrent hematemesis, melena or if the
patient? initial exam was incomplete. Answer C is incorrect
because tagged RBC scan is not likely to pick up on
A. Correct INR, start oral iron, and monitor for the very slow blood loss rate in this patient, and is
active bleeding or persistent anemia not the first line test to evaluate for suspected oc-
Chapter 12 — Gastrointestinal bleeding 345

cult small bowel bleeding. Tagged RBC scan can be CORRECT ANSWER: D
considered for active overt GI bleeding if angiog-
raphy is being considered or for occult small bowel RATIONALE
bleeding if VCE is not available or after a negative This patient presents with clinical concern for a
VCE and CT enterography. Answer D is incorrect small bowel source of anemia and GI blood loss.
because CT angiography is generally reserved for Standard evaluations with EGD, colonoscopy and
active overt brisk GI bleeding prior to considering VCE are negative. Guidelines suggest that the next
angiography. A CT enterography can also be con- best diagnostic test for this patient would be a
sidered for occult small bowel bleeding if the VCE CT enterography (CTE). CTE is suggested as the
is negative or if there is suspicion for small bowel next test to evaluate for small bowel bleeding after
obstruction, to rule-out small-bowel lesions. a negative VCE, due to its ability to detect and
locate small bowel masses and other enhancing
REFERENCE mucosal lesions.
Gerson LB, Fidler JL, Cave DR, et al. ACG Clini- Answer A is incorrect because a second look
cal Guideline: Diagnosis and Management of colonoscopy should be considered if there is
Small Bowel Bleeding Am J Gastroenterol 2015; recurrent hematochezia or if the first exam was
110:1265–1287. incomplete or poorly prepped. The yield is low
in this patient without overt bleeding and after
a recent well prepped complete colonoscopy.
Question 40 Answers B and C are incorrect because nuclear
A 78-year-old male smoker presents to your office tagged RBC scan and CTA are generally consid-
for evaluation of iron deficiency anemia and occult ered in patients with brisk or massive overt active
positive stool. He does not take any medications bleeding, which is not the case in this patient.
and is otherwise healthy. Vital signs and exam are Answer E is incorrect because barium studies are
normal. Pertinent labs reveal hemoglobin 9.8 g/ not recommended for the standard evaluation of
dL (baseline 14.6 g/dL), MCV 76 fL and low fer- small bowel bleeding.
ritin and iron levels. Celiac disease blood screen
is negative. The patient undergoes both an upper REFERENCE
endoscopy to second portion of the duodenum Gerson LB, Fidle JL, Cave DR, et al. ACG Clini-
and a well-prepped colonoscopy to the terminal cal Guideline: Diagnosis and Management of
ileum which are both normal, without evidence Small Bowel Bleeding Am J Gastroenterol 2015;
for a cause of anemia. A follow-up video capsule 110:1265–1287.
endoscopy is also normal, without any mucosal
lesions or blood identified. The patient returns for
a follow-up visit without any clinical change and Question 41
continues to be anemic. A 74-year-old female with a history of recur-
rent DVT on therapeutic warfarin presents to the
What is the next best step in management of Emergency Department with one hour of large vol-
this patient? ume bright red blood per rectum. Vital signs are:
heart rate 110 bpm, blood pressure 72/48 mmHg.
A. Repeat colonoscopy Examination reveals a pale, confused female in no
B. Nuclear tagged RBC scan acute distress, tachycardia, and a soft non-tender
C. CT angiography (CTA) abdomen without distension and no stigmata of
D. CT Enterography (CTE) liver disease. Labs reveal INR 2.0, hemoglobin 6.4
E. Barium upper GI series with small g/dL, platelets 180,000/uL. Intravenous access is
bowel follow through established and crystalloid resuscitation is initi-
346 Digestive Diseases Self-Education Program®

ated. An urgent upper endoscopy reveals no blood Question 42


or etiology for massive hematochezia. Despite A 76-year-old male smoker with a history of severe
resuscitation and transfusion of packed red blood atherosclerotic arterial disease and abdominal
cells, the patient continues to have massive hema- aortic aneurysm status-post endovascular graft
tochezia and remains confused and hypotensive placement presents with one episode of he-
requiring vasopressors and ICU support. matemesis and melena four hours prior to presen-
tation. In the Emergency Department, vital signs
What is the next best step in management of are normal, hemoglobin is 9.2 g/dL and there is no
this patient? clinical active bleeding. Abdominal exam shows an
obese non-tender non-distended abdomen with-
A. Emergent reversal of coagulopathy out palpable mass. Intravenous access and fluid
B. Rapid bowel prep and urgent colonoscopy resuscitation is initiated and an upper endoscopy
C. Emergent unprepped colonoscopy is performed to the second portion of the duode-
D. Nuclear tagged RBC scan num, showing no blood or stigmata of bleeding.
E. Emergent angiography 24 hours later the patient again develops melena
and worsening anemia (hemoglobin is 7.4 g/dL),
CORRECT ANSWER: E and an urgent second look push enteroscopy is
performed revealing old blood in the third portion
RATIONALE of the duodenum, but no fresh bleeding or source
This patient presents with a massive lower GI hem- is identified.
orrhage. After a brisk upper GI bleed was ruled-out
with EGD, the patient continued to hemorrhage Which of the following is the next best step in the
and remained hemodynamically unstable. In the diagnostic evaluation of this patient?
setting of a patient with ongoing massive lower GI
bleeding who has been ruled-out for an upper GI A. Rapid bowel prep and colonoscopy
bleed (negative upper endoscopy) and who contin- B. Nuclear tagged RBC scan
ues to have hemodynamic instability despite resus- C. Double balloon (deep) enteroscopy
citation, emergent angiography should be pursued D. CT angiography
in an effort localize and control bleeding. E. Video capsule endoscopy
Answer A is incorrect because an INR less
than 2.5 does not require reversal prior to at- CORRECT ANSWER: D
tempts at hemostasis. Answers B and C are incor-
rect because given the patients altered mental sta- RATIONALE
tus and hemodynamic changes, she is unlikely to In the setting of a history of abdominal aortic an-
tolerate a bowel preparation and urgent colonos- eurysm repair, this patient initially presented with
copy. Also, there is no role for an unprepped colo- a hemodynamically stable presumably self-limited
noscopy in lower GI bleeding due to low yield and GI bleed followed by recurrent bleeding and
poor visualization. Answer D is incorrect because a worsening anemia. While initial upper endoscopy
nuclear tagged RBC scan should be reserved for a is negative for an obvious source, second look en-
patient who is hemodynamically stable. doscopy confirms suspicion that there is upper GI
bleeding. In this scenario, there is a high clinical
REFERENCE suspicion for aortoenteric fistula and the current
Strate LL and Gralnek IM. ACG Clinical Guideline: clinical presentation likely represents an initial
Management of Patients With Acute Lower Gas- “herald bleed”, signaling risk for future massive
trointestinal Bleeding. Am J Gastroenterol 2016; hemorrhage. For hemodynamically stable patients
111:459–474. with a high suspicion for aortoenteric fistula, CT
Chapter 12 — Gastrointestinal bleeding 347

angiography is a recommended first-line diagnos- bleeding, passing bright red blood with clots in
tic study, and therefore answer D is correct. CTA the toilet every 30 minutes. She denies abdominal
is superior to EGD in diagnosing changes associ- pain, nausea, vomiting, melena, hematemesis,
ated with aortoenteric fistula (79 percent vs 25 diarrhea, or prior episodes of GI bleeding. She
percent). has never had a colonoscopy. On exam, vital signs
Patients at risk for aortoenteric fistulas include are normal and abdomen is soft and non-tender.
those with a known abdominal aortic aneurysm Hemoglobin is 11.6 g/dL (baseline 13.4 g/dL).
and/or prior aortic graft placement (surgical or en-
dovascular), history of aortic instrumentation and Which of the following is the most likely etiology
rarely patients with aortitis. These patients may of hematochezia in this patient?
present with an initial low-grade self-limited GI
bleed (“herald bleed”) which can be later followed A. Internal hemorrhoids
by a massive life threating hemorrhage. Other find- B. Colorectal cancer
ings can include abdominal pain and palpable ab- C. Angioectasia (AVM)
dominal mass. In one study, 55 percent of patients D. Ischemic colitis
presented with a “herald bleed” with a median time E. Diverticular disease
to diagnosis of 14 days after first bleeding. There-
fore, a high index of clinical suspicion is required CORRECT ANSWER: E
for this uncommon diagnosis, and it must be
promptly recognized and diagnosed. In general, the RATIONALE
clinician should have a high suspicion for aortoen- This patient presents with painless hematoche-
teric fistula in any patient with prior aortic surgery zia. While the differential diagnosis is broad, the
presenting with significant GI bleeding. most common cause of acute severe hematochezia
Answer A is incorrect as there was blood noted remains diverticular disease. The other answer
in the duodenum on second endoscopy, making choices can also cause painless hematochezia,
lower GI bleeding unlikely. Answers B, C and E are however they are not as common as diverticular
all options to investigate for a source of GI bleed- bleeding. Hemorrhoidal bleeding uncommonly
ing in various scenarios, however this patient has causes enough blood loss to lead to anemia.
a high clinical suspicion for an aortoenteric fistula, Colorectal cancer and angioectasias more com-
so these tests would not be the next best step in monly present with sub-acute or chronic blood
diagnosis for this patient. loss anemia, and less commonly severe hemato-
chezia. Ischemic colitis is an often encountered
REFERENCE cause of hematochezia in the elderly, however it is
Deijen CL, Smulders YM, Covelier, HME et al. still less common than diverticular bleeding, and
The Importance of Early Diagnosis and Treatment usually presents with acute onset of abdominal
of Patients with Aortoenteric Fistulas Presenting pain. Therefore, answer E is the best answer.
with Herald Bleeds. Annals of Vascular Surgery
2016:28-34. REFERENCES
Jensen DM, Machicado GA, Jutabha R et al. Ur-
gent colonoscopy for diagnosis and treatment
Question 43 of severe diverticular hemorrhage. N Engl J Med
A 72-year-old female with chronic constipation 2000;342:78-82.
presents with to the Emergency Department with Strate LL and Gralnek IM. ACG Clinical
four hours of rectal bleeding. She described being Guideline: Management of Patients With Acute
in her usual state of health until this morning, Lower Gastrointestinal Bleeding. Am J Gastroen-
when she developed acute onset of heavy rectal terol 2016; 111:459–474.
348 Digestive Diseases Self-Education Program®

Question 44 cular event tends to be highest after 10-14 days


A 68-year-old male smoker with a history of ad- off aspirin. Data also suggest a benefit even in
vanced coronary artery disease status-post coro- patients with high rebleeding risk GI lesions, with
nary artery bypass grafting and placement of bare no increase in rebleeding risk and a gained car-
metal stents, presents to the Emergency Depart- diovascular benefit to resuming aspirin within 24
ment with melena for 24 hours. He reports having hours. Therefore, for the majority of patients, it
four black stools since the day prior, with mild has been recommended to resume aspirin therapy
epigastric abdominal discomfort. His only medica- as soon as possible after hemostasis of a bleeding
tions are aspirin 81 mg daily and atorvastatin. Vital lesion is achieved, ideally within one to three days
signs are normal and he is in no acute distress. and certainly by seven days.
Abdominal exam reveals mild epigastric tenderness One must also account for the baseline risk of
without rebound or guarding. Labs are notable for rebleeding of the primary GI lesion, for example
BUN 38 mg/dL, creatinine 1.0 mg/dL, hemoglobin per the Forrest classification. This patient pres-
9.4 g/dL (baseline 12 g/dL). IV access is obtained ents with recent upper GI bleeding secondary to a
and IV fluids are initiated along with IV proton low-risk clean-based gastric ulcer with no bleeding.
pump inhibitor. Upper endoscopy reveals a 15 mm He also has a significant high-risk cardiovascular
clean-based gastric antral ulcer without stigmata of history and requires long-term secondary prophy-
hemorrhage. Testing for H pylori is negative. You lactic anti-platelet therapy in the form of aspirin.
recommend transitioning to oral PPI therapy. Therefore, the patient is at low GI risk and a high
cardiovascular risk, and thus aspirin should be not
Which of the following recommendations should be held and should be resumed immediately. Hold-
you make regarding the patients aspirin therapy? ing and delaying aspirin resumption places this
patient at unnecessary cardiovascular risk, there-
A. Resume aspirin now fore the additional answer choices are incorrect.
B. Hold aspirin for one day
C. Hold aspirin for three days REFERENCES
D. Hold aspirin for seven days Abraham NS, Hlatky MA, Antman EM et al.
E. Hold aspirin until repeat endoscopy in ACCF/ACG/AHA 2010 expert consensus docu-
eight weeks ment on the concomitant use of proton pump
inhibitors and thienopyridines: a focused update
CORRECT ANSWER: A of the ACCF/ACG/AHA 2008 expert consensus
document on reducing the gastrointestinal risks of
RATIONALE antiplatelet therapy and NSAID use. Am J Gastro-
While the management decisions surrounding enterol 2010; 105: 2533 – 49.
anti-platelet use and an acute GI bleed often Laine L, Jensen, DM. Management of patients
require an individualized and multi-disciplinary with ulcer bleeding. Am J Gastroenterol. 2012;107:
approach, there are some general principles that 345-60.
apply. Data support the general notion that in Sung, J et al. Continuation of low dose aspirin
high-risk cardiovascular disease, the mortality therapy in peptic ulcer bleeding: a randomized
from a cardiovascular event is higher than that of trial. Annals of Internal Medicine. 2010; 159: 1-9.
GI bleeding in the acute setting, and that the risk
of rebleeding while on aspirin is lower than the
gained reduction of cardiovascular events while Question 45
on aspirin. The risk for rebleeding tends to be A 50-year-old male with atrial fibrillation on war-
highest in the first three to seven days depending farin presents to the Emergency Department with
on the type of lesion, and the risk for a cardiovas- two hours of hematochezia. He reports undergo-
Chapter 12 — Gastrointestinal bleeding 349

ing his first screening colonoscopy seven days ago high-risk endoscopic stigmata of bleeding, includ-
and was told that there was a large polyp re- ing active bleeding, non-bleeding visible vessel
moved. He resumed warfarin that evening. Today or possibly adherent clot. Options for endoscopic
he woke up and passed three large bloody bowel hemostasis of post-polypectomy bleeding from
movement with clots. At presentation, heart rate high-risk ulcers include epinephrine, mechani-
112 bpm, blood pressure 108/62 mm Hg, exam cal clips, electrocoagulation and band ligation.
reveals a diaphoretic male with a soft non-tender Epinephrine should not be used alone, and should
abdomen. Labs are notable for hemoglobin 10.6 instead be used to control initial active bleeding,
g/dL (baseline 14.2 g/dL), BUN 10 mg/dL, cre- reduce bleeding risk from application of mechani-
atinine 0.8 mg/dL, INR 2.2. Intravenous access cal/thermal therapy and to improve visualiza-
is obtained and crystalloid resuscitation is initi- tion. The use of mechanic or thermal therapy for
ated. The patient undergoes a rapid bowel prep post-polypectomy bleeding is recommended, with
and colonoscopy, which reveals a 14 mm ulcer in or without epinephrine. However, mechanical clip
the ascending colon with a non-bleeding visible placement is often preferred over thermal contact
vessel. therapy to limit thermal tissue injury to the colon
wall (especially in the setting of ongoing use of
Which of the following is the most appropriate anticoagulation). Therefore, answer D is the most
next step in management for this patient? appropriate next step.

A. No endoscopic therapy as INR is greater REFERENCES


than 2 Rex DK, Lewis BS, Waye JD. Colonoscopy and endo-
B. Epinephrine injection alone scopic therapy for delayed post-polypectomy hemor-
C. Non-contact argon plasma coagulation to the rhage. Gastrointest Endosc 1992; 38: 127 – 9.
visible vessel Strate, LL and Gralnek, IM. ACG Clinical
D. Mechanical hemoclip placement across the Guideline: Management of Patients With Acute
vessel or ulcer Lower Gastrointestinal Bleeding. Am J Gastroen-
E. Endoscopic band ligation of vessel or ulcer terol 2016; 111:459–474.

CORRECT ANSWER: D
Question 46
RATIONALE A 62-year-old female presents for evaluation of he-
This patient’s clinical scenario is consistent with matemesis. She has a history of recent myocardial
post-polypectomy bleeding after recent large pol- infarction and placement of bare metal coronary
ypectomy. Patients with large polyps, particularly stent, maintained on aspirin and clopidogrel. On
in the right colon and those on anti-thrombotic presentation, heart rate is 120 bpm, blood pres-
therapy, are at particularly high risk for post- sure 98/62 mmHg. Examination reveals tachy-
polypectomy bleeding. Bleeding can occur up to cardia with mildly tender epigastrium without re-
30 days after polypectomy, but in general is seen bound or guarding. Hemoglobin is 6.2 g/dL, BUN
five to seven days later. The causative mechanism 48 mg/dL, creatinine 1.3 mg/dL. Two large-bore
is likely related to sloughing of eschar over a vessel IVs are placed, fluid and blood product resuscita-
and necrosis and ulceration into the submucosa tion and intravenous continuous PPI is initiated.
after thermal polypectomy. After initial resuscita- An urgent upper endoscopy reveals old blood in
tion, a bowel prep and colonoscopy is indicated, the stomach and a 15 mm duodenal bulb ulcer.
provided the patient is stable. Similar to upper GI Which of the following endoscopic stigmata of
bleeding, endoscopic therapy is recommended for peptic ulcer disease at time of endoscopy requires
patients with lower GI bleeding from lesions with definitive endoscopic therapy?
350 Digestive Diseases Self-Education Program®

A. Clean-base Question 47
B. Flat pigmented spot A 74-year-old male presents for evaluation of
C. Firm mature adherent clot melena and coffee ground emesis for 24 hours. He
D. Non-bleeding visible vessel has a history of end-stage renal disease on hemo-
dialysis and takes aspirin. On presentation, heart
CORRECT ANSWER: D rate is 86 bpm, blood pressure 106/68 mmHg.
Examination is unrevealing. Hemoglobin is 7.6 g/
RATIONALE dL (baseline 10.2 g/dL). Two large-bore IVs are
Stigmata of bleeding seen during endoscopy for placed, fluid resuscitation and intravenous contin-
peptic ulcer disease are important predictors of uous PPI are initiated. An urgent upper endoscopy
rebleeding, surgery and mortality and are used to reveals several gastric ulcers from eight to 12 mm
guide the decision for and choice of endoscopic in size, including an antral ulcer with an overlying
therapy. Lesions at highest risk for rebleeding, blood clot.
including those with active arterial spurting and
non-bleeding visible vessel, require endoscopic Which of the following is the best next step in
therapy to achieve hemostasis and reduce the risk endoscopic management of this ulcer?
for rebleeding. Lesions with an adherent clot are
at intermediate risk for rebleeding and therefore A. No endoscopic therapy indicated for overlying
can be considered for endoscopic therapy depend- blood clots
ing on the clinical scenario. However endoscopic B. Epinephrine injection into clot base and pe-
therapy is not required for lesions with an adher- riphery alone
ent clot. Data is inconsistent as to the benefit of C. Hemoclip placement over the clot followed by
endoscopic therapy with lesions with adherent attempted clot removal with snare
clots, and much of the therapeutic indication D. Epinephrine injection followed by irrigation
depends on the presence or absence of an under- and polypectomy snare clot removal
lying high-risk lesion, the clinical scenario and E. Thermal heater probe therapy applied with
patient specific risk factors. Doppler endoscopic gentle pressure through the clot to ulcer bed
probe may be used to evaluate for underlying arte-
rial flow, and if it is present, endoscopic therapy CORRECT ANSWER: D
should be performed to decrease risk of rebleed-
ing. Low risk lesions, specifically those with a flat RATIONALE
pigmented spot or a clean-base, do not require Peptic ulcers with adherent clots are at inter-
endoscopic therapy. mediate risk for re-bleeding without endoscopic
therapy (20-30 percent). Clots tend to be soft
REFERENCES and acute, or mature and firm. The indication
de Groot, N.L et. al. Reassessment of the predic- and ability to treat these lesions relies on several
tive value of the Forrest classification for peptic factors, including clinical stability of the patient,
ulcer rebleeding and mortality: can classification patient specific risk factors for rebleeding, and
be simplified? Endoscopy 2014; 46(01): 46-52. adequate endoscopic positioning and access to the
Laine L, McQuaid KR. Endoscopic therapy for ulcer. While data is variable, some studies sug-
bleeding ulcers: an evidence-based approach based gest a rebleeding benefit to endoscopic treatment
on meta-analyses of randomized controlled trials. of adherent clots versus medical therapy alone.
Clin Gastroenterol Hepatol 2009; 7: 33 – 47. Therefore, guidelines suggest that consideration
Laine L, Jensen, DM. Management of patients be made to treat these lesions with endoscopic
with ulcer bleeding. Am J Gastroenterol. 2012;107: therapy when feasible. The recommended ap-
345-60. proach to managing these lesions includes vigor-
Chapter 12 — Gastrointestinal bleeding 351

ous washing to attempt to dislodge the overlying management strategy given this patient’s hemato-
clot, followed by attempt to remove the clot with logic abnormalities?
cold polypectomy snare or a grasping device such
as forceps. This can be preceded by epinephrine A. Transfuse packed red blood cells to goal he-
injection around the ulcer base to assist with moglobin greater than 10 g/dL
hemostasis. Following adequate clot removal, the B. Transfuse packed red blood cells if hemoglo-
clinician can make an assessment of any underly- bin drops less than 7 g/dL
ing stigmata of hemorrhage and determine the C. Transfuse platelets for goal greater than
need for definitive endoscopic therapy. Answer 100,000/uL
A is incorrect, as the endoscopist should further D. Transfuse fresh frozen plasma and administer
assess the clot and underlying ulcer before deter- oral vitamin K to achieve INR less than 1.5
mining that there is no need for additional thera-
py. Answer B is incorrect as epinephrine alone is CORRECT ANSWER: B
not a preferred treatment modality for peptic ul-
cers with stigmata of hemorrhage. Answers C and RATIONALE
E are incorrect, as mechanical or thermal therapy It is important for gastroenterologists to be able
of the adherent clot itself is not going to treat the to assist in determination of transfusion support
underlying lesion and cause for the clot. goals of patients with GI hemorrhage. Guidelines
and evidence are available to support specific
REFERENCES transfusion strategies in patients with acute GI
Kahi CJ, Jensen DM, Sung JJ, et al. Endoscopic bleeding. In a landmark study of patients with up-
therapy versus medical therapy for bleeding peptic per GI bleeding, a restrictive transfusion strategy
ulcer with adherent clot: a meta-analysis. Gastro- with a threshold hemoglobin less than seven g/dL
enterology. 2005; 129:855-62. improved outcomes (survival, rebleeding, adverse
Laine L, Jensen, DM. Management of patients events) compared with a threshold of nine g/dL
with ulcer bleeding. Am J Gastroenterol. 2012;107: in severe acute upper GI bleeding. Therefore, a re-
345-60. strictive transfusion goal set at hemoglobin great-
er than seven g/dL is generally preferred over a
liberal strategy, unless there is significant active
Question 48 cardiovascular disease which would support a
A 48-year-old female with history of DVT on hemoglobin goal of nine to 10 g/dL. Patients with
warfarin and acute myelogenous leukemia status- a hemodynamically significant active massive GI
post stem cell transplant presents with melena and hemorrhage may be transfused blood products
coffee ground emesis for 24 hours. Vital signs are without a specific hemoglobin goal in the acute
normal. Labs reveal: hemoglobin 8.6 g/dL, plate- setting, to assist tissue perfusion and volume
lets 80,000/uL and INR 2.1. Intravenous access is support. In addition, patients with symptomatic
obtained, IV fluids and PPI are initiated and she anemia or peripheral or cerebrovascular disease
undergoes an upper endoscopy which reveals a 12 may require different transfusion thresholds.
mm ulcer in the duodenal bulb with a non-bleed- In general, expert opinion recommends to main-
ing visible vessel, which is treated with combina- tain a platelet count of greater than or equal to
tion epinephrine and thermal contact therapy with 50,000/uL in patients with significant bleed-
excellent effect. Gastric biopsies are obtained to ing. There is no recommendation for platelet
evaluate for H. pylori. The patient is admitted to count greater than 100,000/uL in the setting of
the medical floors and the medical team asks if the GI bleeding, and therefore this patient does not
patient requires any blood product transfusions. require a platelet transfusion. Evidence supports
Which of the following is the best blood product the notion that there is no increased rebleeding
352 Digestive Diseases Self-Education Program®

risk in anticoagulated patients with INR of 1.3-2.7 RATIONALE


after endoscopic therapy compared to patients Patients presenting with acute hematemesis who
with INR less than 1.3. Therefore, there is no need require urgent endoscopy can present a techni-
to correct the INR after endoscopic therapy in this cal challenge secondary to diminished endoscopic
patient. mucosal visualization from a large amount of
blood and clots obscuring endoscopic views. Data
REFERENCES supports that pre-endoscopy infusion of the proki-
Razzaghi A, Barkun AN. Platelet transfusion netic erythromycin significantly improves mucosal
threshold in patients with upper gastrointestinal visualization and exam quality versus placebo
bleeding: a systematic review. J Clin Gastroenterol (pless than 0.05). A meta-analysis of erythromy-
2012; 46: 482 – 6. cin or metoclopramide before EGD for upper GI
Villanueva C, Colomo A, Bosch A. Transfusion bleeding also noted a reduced need for repeat EGD
Strategies for Acute Upper Gastrointestinal Bleed- with a prokinetic (OR 0.55), and therefore these
ing. N Engl J Med 2013; 368:11-21. agents likely improve that overall diagnostic yield
Wolf AT, Wasan SK, Saltzman JR. Impact of of EGD in this setting. However, prokinetics do
anticoagulation on rebleeding following endo- not improve overall clinical outcomes such as mor-
scopic therapy for nonvariceal upper gastrointes- tality, hospital length of stay or need for surgery.
tinal hemorrhage. Am J Gastroenterol 2007; 102: Therefore, the correct answer is B as it is recom-
290 – 6. mended to administer an intravenous prokinetic
prior to endoscopy for acute upper GI bleeding,
for the purpose of improving visualization and
Question 49 diagnostic yield.
A 58-year-old female presents for evaluation of NG tube lavage can be used as an initial tool
hematemesis for two hours. She has a history of to assess the location of bleeding. However, due
fibromyalgia treated with daily ibuprofen. On to the nature of the procedure, clinical guidelines
presentation, heart rate is 106 bpm, blood pres- do not support the use of NG lavage to improve
sure 98/64 mmHg. Examination is notable for mucosal visualization prior to endoscopy for acute
tachycardia, dried blood around the mouth and upper GI bleeding. This test is uncomfortable to
mild epigastric tenderness to palpation with- the patient and of limited overall clinical utility.
out rebound or guarding. Hemoglobin is 9.2 g/ Intravenous PPI prior to endoscopy has been
dL (baseline 12.4 g/dL). Two large-bore IVs are shown to have several benefits, including reduc-
placed and fluid resuscitation is initiated by the tion of high-risk stigmata detected on endoscopy
emergency department. and need for endoscopic intervention. However,
evidence does not support the use of PPI to
Which of the following next interventions will help improve mucosal visualization prior to endos-
improve diagnostic yield and endoscopic visualiza- copy. Deep sedation with anesthesia and possible
tion during upper endoscopy? endotracheal intubation can be used to assist with
performing endoscopy on a patient who is dif-
A. Endotracheal intubation ficult to sedate or those who are critically ill with
B. Intravenous prokinetic administration active hematemesis, but does not affect endoscopic
C. Intravenous proton pump inhibitor infusion visualization. In high-risk patients, performing en-
D. Deep sedation with anesthesia care doscopy within 12 hours can have clinical benefits
E. Performing emergent endoscopy within 12 such as improved detection of high-risk stigmata,
hours reduced transfusion requirements and reduced
duration of hospital stay, though early endoscopy
CORRECT ANSWER: B does not affect overall endoscopic visualization.
Chapter 12 — Gastrointestinal bleeding 353

REFERENCES D. Conventional angiography


Barkun AN, Bardou M, Martel M et al. Prokinet- E. Intraoperative enteroscopy
ics in acute upper GI bleeding: a meta-analysis.
Gastrointest Endosc 2010; 72: 1138 – 45. CORRECT ANSWER: C
Carbonell N, Pauwels A, Serfaty L et al. Eryth-
romycin infusion prior to endoscopy for acute RATIONALE
upper gastrointestinal bleeding: a randomized, This patient presents with stable small bowel
controlled, double-blind trial. Am J Gastroenterol bleeding secondary to a mid-small bowel AVM
2006; 101: 1211 – 5. in the setting of anticoagulation for a mechanical
Laine L, Jensen, DM. Management of patients aortic valve. Current guidelines suggest that pa-
with ulcer bleeding. Am J Gastroenterol. 2012;107: tients with stable small bowel bleeding who have
345-60. a positive small bowel finding on VCE proceed to
deep enteroscopy. This has the advantage of both
diagnostic and therapeutic interventions. The
Question 50 choice of enteroscopy is often dictated by lesion
A 72-year-old female with aortic valve stenosis location on pre-endoscopy localization studies and
status-post mechanical aortic valve placement by local expertise. In general, a push enteroscopy
fifteen years ago, presents to the Emergency De- as an extended upper endoscopy with a standard
partment with melena. Her medications include variable stiffness colonoscope can reach the distal
warfarin and aspirin 81 mg daily. On presenta- duodenum to proximal jejunum to a distance
tion, she is hemodynamically stable and awake of 90–150 cm beyond the pylorus or 45-90 cm
and alert. Examination reveals a soft non-tender beyond the ligament of Treitz. However, an ante-
abdomen with normoactive bowel sounds. Rec- grade double balloon enteroscopy can reach a dis-
tal examination reveals melenic stool. Pertinent tance of 240–360 cm distal to the pylorus. Given
labs reveal hemoglobin 7.2 g/dL (baseline 10.8 that this patient’s bleeding lesion is located in the
g/dL), BUN 8 mg/dL, and INR 2.2. The patient mid-small bowel, a push enteroscopy is unlikely to
undergoes both an upper endoscopy to the second reach the lesion and therefore an antegrade double
portion of the duodenum and a colonoscopy the balloon enteroscopy is preferred.
following day which are both normal, other than Conservative medical care with correcting
dark melenic contents seen within the terminal INR and iron therapy is unlikely to be a viable
ileum. She then undergoes a video capsule en- long-term solution in this patient as the lesion
doscopy (VCE) which reveals an actively bleeding of concern needs to be ablated to prevent recur-
small bowel angioectasia (AVM) approximately rent bleeding. In addition, anticoagulation will be
45 percent into the small bowel transit time. She needed due to a mechanical aortic valve. Conser-
continues to require blood transfusions every one vative care or medical therapy alone can be con-
to two days during hospitalization. sidered for patients with recurrent bleeding who
either do not wish to pursue additional treatment
What is the next best step in management of or who are not candidates for definitive therapy of
this patient? the bleeding lesion.
Angiography and intraoperative enteroscopy
A. Correct INR with vitamin K, start oral iron, are not indicated at this time. Angiography is gen-
and monitor for hemodynamic changes erally reserved for patients with unstable massive
B. Perform a push enteroscopy with a variable small bowel bleeding who cannot tolerate en-
stiffness colonoscope doscopy or VCE, or those who continue to bleed
C. Perform an antegrade double balloon despite evaluation including deep enteroscopy.
enteroscopy Intraoperative enteroscopy through an incision
354 Digestive Diseases Self-Education Program®

into the small bowel allows deeper examination,


though is generally reserved for patients with
recurrent significant bleeding who have already
had an extensive and unrevealing evaluation
including deep enteroscopy, or for those in whom
deep enteroscopy cannot be performed without
surgical assistance secondary to altered anatomy
or adhesions.

REFERENCE
Gerson LB, Fidler JL, Cave DR, Leighton JA. ACG
Clinical Guideline: Diagnosis and Management of
Small Bowel Bleeding Am J Gastroenterol 2015;
110:1265–1287.
Answers & critiques

CHAPTER 13

Inflammatory bowel disease


Maisa I. Abdalla, MD, MPH, Millie D. Long MD, MPH,
Christina Ha MD FACG AGAF and Sushila R Dalal, MD

Question 1 CORRECT ANSWER: B


You are seeing a 30-year-old female with re-
cent diagnosis of mild to moderate left-sided RATIONALE
ulcerative colitis. She has been maintained on The therapeutic effect of fish oil is believed to be
mesalamine 800 mg TID since her diagnosis. driven by its main component, omega-3 poly-
Clinically, she reports two to three loose bowel unsatuared fatty acids, which has been impli-
movements per day with mild urgency, but no cated in a favorable shift of the gut microbiota.
blood in the stool. You suggested increasing her Despite its wide use among IBD patients, there
mesalamine dose to 4.8 g/day, but she did not is no convincing evidence of its benefit. In two
want to increase the dose and expressed inter- large RCTs (EPIC 1 and EPIC 2), the use of fish
est in complementary and alternative medicine oil was not better than placebo in preventing
(CAM) options. Which of the following state- relapses for patients with Crohn’s disease. In
ments regarding CAM use in inflammatory ulcerative colitis patients, the evidence is mixed,
bowel disease (IBD) patients is correct? with suggestion of potential impact on fecal in-
flammatory markers and corticosteroid require-
A. In randomized control trials (RCT), the use ments, but no impact on reducing relapses.
of curcumin as an add-on therapy to oral Curcumin, a major ingredient of the spice
mesalamine was not superior to placebo turmeric, has been shown to have anti-inflam-
in inducing and maintaining remission in matory and antioxidative properties on human
patients with mild to moderate ulcerative lymphocytes and gut epithelial cells. Its use in
colitis. ulcerative colitis patients was investigated in two
B. Although cannabis use may improve symp- RCTs, the first by Hanai et al showed potential
toms and quality of life of IBD patients, efficacy in maintaining remission. In this study,
current evidence suggests it has no role in 89 patients with quiescent ulcerative colitis
achieving or maintaining disease remission. on sulfasalazine or 5-ASA were randomized
C. Fish oil was shown in RCTs to be superior to receive curcumin 1g BID vs placebo. Those
to placebo in preventing disease relapse in who received curcumin were less likely to have
patients with Crohn’s disease and ulcerative disease relapse during a six month follow up pe-
colitis. riod (4.7 percent vs 20.5 percent, P=0.04). The
D. VSL #3 is effective as an add-on therapy second study by Lang et al, assessed its potential
in patients with mild to moderate Crohn’s role in inducing remission. Fifty patients with
disease. mild to moderate ulcerative colitis and incom-
E. Low dose naltrexone (LDN) is superior to plete response to 5-aminosalysilate (5-ASA)
placebo for inducing remission in patients were randomized to receive curcumin (3g/day)
with Crohn’s disease. vs placebo. At four weeks, those in the curcumin

355
356 Digestive Diseases Self-Education Program®

group were more likely to achieve clinical remission tent courses of budesonide for several years until
(p=0.1) and endoscopic remission (p=0.43). she eventually proceeded with ileocecal resection
Few small studies, including three RCTs investigat- six months ago. Immunomodulator and biologic
ed the role cannabis in IBD patients, and revealed therapies were discussed prior to her resection,
better quality of life and general health perception, but she elected to proceed with surgery due to
but no impact on rates of remission, disease activity concerns of potential adverse effects related
scores or inflammatory markers. to the therapies. Recent blood tests revealed
VSL#3 is the most-studied probiotic in IBD. normal white cell count, hemoglobin and MCV.
Its role is more established in patients with ileal Both C-reactive protein and fecal calprotectin are
pouch-anal anastomosis (IPAA), where VSL#3 was within normal limits. Her six-month postopera-
found to be superior to placebo in reducing risk of tive ileocolonoscopy revealed focal ulceration at
first episode of pouchitis as well as reducing rates of the anastomosis with normal colonic and neo-
recurrence after an episode of pouchitis. In mild to terminal ileal mucosa [see figure]. Which of the
moderate ulcerative colitis patients, a recent meta- following is the most appropriate next step in
analysis concluded that the use of VSL#3 as an management?
add on to conventional therapies is safe and more
effective than conventional therapy alone in achiev-
ing clinical response and remission. In a small
pediatric cohort of ulcerative colitis patients, the
addition of VSL#3 to 5-ASA was superior to 5-ASA
alone in maintaining remission. In Crohn’s dis-
ease patients, the evidence is more limited; based
on two recent meta-analyses, VSL#3 has no clear
role in inducing remission or preventing relapse in
patients with Crohn’s disease
LDN has been primarily evaluated in patients
Ileoanal anastomosis
with Crohn’s disease and the evidence was summa-
rized in a recent Cochrane review, which concluded
that the current evidence is insufficient to allow
any firm conclusion regarding efficacy and safety of
LDN in patients with active Crohn’s disease.

REFERENCE
Lin, S.C. and A.S. Cheifetz, The Use of Comple-
mentary and Alternative Medicine in Patients
With Inflammatory Bowel Disease. Gastroenterol
Hepatol (N Y), 2018. 14(7): p. 415-425.
Neoterminal ileum

Question 2 A. Start sulfasalazine 4g/day and folic acid 1 m day


A 56-year-old female nonsmoker presents for B. Start azathioprine 2-2.5 mg/kg/day
evaluation of non-bloody diarrhea with five to C. Start infliximab at 5mg/kg dosing
six watery stools per day associated with urgency D. Start daily cholestyramine
and lower abdominal cramps. She has long E. Start a three-month course of metronidazole
standing stricturing ileocolonic Crohn’s disease,
primarily managed with oral 5-ASA and intermit- CORRECT ANSWER: D
Chapter 13 — Inflammatory bowel disease 357

RATIONALE A. Increase 6-MP dose to achieve 6-TGN level of


Bile salt malabsorption is most likely the cause more than 235 pmol/8x108 RBC
of diarrhea in this patient. This results from the B. Decrease the infliximab frequency to every 10
reduction in bile acid absorption at the terminal weeks
ileum in patients who have undergone small bowel C. Increase 6-MP dose to achieve 6-TGN level
resection. Consequently, more bile acids reach the more than 125 pmol/8x108 RBC
colon, irritating the mucosa and causing secretory D. Discontinue 6-MP and maintain infliximab
diarrhea. Cholestyramine, a bile salt binder, is very monotherapy
effective in such cases and is the correct answer for
this clinical scenario. Small intestinal bacterial over CORRECT ANSWER: D
growth (SIBO) should also be considered, especially
if the patient fails to respond to cholestyramine. RATIONALE
The other listed options are incorrect, because this The risk of lymphoproliferative disorder with
patient has no evidence of CD recurrence based on thiopurine therapy is well established and appears
her clinical and biomarker assessment, as well as to be highest among older patients. In the French
her endoscopic evaluation. Ulceration limited to prospective observational cohort study CESAME,
the anastomosis, but not involving the neoterminal designed to assess the risk of lymphoproliferative
ileum is not considered endoscopic recurrence and disease in IBD patients receiving thiopurines, age
frequently reflects ischemia at the site of anastomo- was identified as an independent risk factor for
sis rather than recurrent disease. the development of lymphoma with more than 50
percent of the incident cases diagnosed in patients
REFERENCE more than 60 years. Furthermore, in a more
Barkun, A.N., et al., Bile acid malabsorption in recent meta-analysis by Kotlyar et al, patients on
chronic diarrhea: pathophysiology and treatment. thiopurines and over 50 years of age were noted
Can J Gastroenterol, 2013. 27(11): p. 653-9. to have the highest absolute risk of lymphoma
per year. Avoidance of thiopurines in this patient
population is thus recommended. The patient de-
Question 3 scribed in this case has been in clinical remission
A 73-year-old man with long-standing history of for several years and endoscopic remission for at
extensive ulcerative colitis presents to establish least one year despite subtherapeutic thiopurine
care at your clinic after moving to the area. He metabolite levels. Hence, discontinuing 6-MP is
was diagnosed in his early 20s, initially treated recommended. Options A and C are not appropri-
with sulfasalazine and intermittent courses of ate as there is no reason to escalate therapy in this
prednisone. Seven years ago he developed a severe patient who is in remission. Finally, option B is
flare and was placed on combination therapy with also incorrect, as there is no reason to de-escalate
infliximab 5mg/kg every eight weeks and 6-mer- infliximab therapy based on this drug level.
captopurine (6-MP) 50 mg/day and has been
asymptomatic since then. He currently has one to REFERENCES
two formed stools per day with no blood or urgen- Kotlyar, D.S., et al., Risk of lymphoma in patients
cy. A colonoscopy within the last year was normal, with inflammatory bowel disease treated with aza-
with no dysplasia detected on random biopsies. thioprine and 6-mercaptopurine: a meta-analysis.
Therapeutic drug monitoring revealed an inflix- Clin Gastroenterol Hepatol, 2015. 13(5): p. 847-
imab trough level of 13 U/mL with no antibodies, 58.e4; quiz e48-50.
6-TGN of 72 pmol/8x108 RBC and 6-MMP of 750 Taleban, S., et al., Inflammatory bowel disease
pmol/8x108 RBC. Which of the following is most and the elderly: a review. J Crohns Colitis, 2015.
appropriate in regard to his care? 9(6): p. 507-15.
358 Digestive Diseases Self-Education Program®

Question 4 increased risk of colorectal neoplasia. On the other


A 22-year-old man with a history of extensive hand, low-dose UDCA may improve laboratory
ulcerative colitis diagnosed three years ago presents markers of cholestasis, but with no clear impact
for evaluation. He is currently in clinical remis- on survival or long-term outcomes and its role
sion, maintained on oral mesalamine 2.4 g/day for chemoprophylaxis in colorectal cancer is still
in divided doses. He was noted to have persistent controversial.
elevation of serum alkaline phosphatase on blood Yearly MRCP is recommended to screen for
samples drawn three months apart. Magnetic reso- cholangiocarcinoma.
nance cholangiopancreatography (MRCP) revealed
alternating narrowed and dilated segments of the REFERENCES
intrahepatic and extrahepatic biliary ducts consis- Lindor, K.D., K.V. Kowdley, and M.E. Harri-
tent with primary sclerosing cholangitis (PSC). son, ACG Clinical Guideline: Primary Sclerosing
Cholangitis. Am J Gastroenterol, 2015. 110(5): p.
Which of the following is recommended at this 646-59; quiz 660.
time? Lopez, A., et al., Colorectal cancer prevention
in patients with ulcerative colitis. Best Pract Res
A. Repeat MRCP in six months for screening Clin Gastroenterol, 2018. 32-33: p. 103-109.
B. He should undergo surveillance colonoscopy
now and annually thereafter.
C. First surveillance colonoscopy is recommended Question 5
five years from now, and annually thereafter A 25-year-old female with a history of ileocolonic
D. High-dose ursodeoxycholic acid (UDCA) Crohn’s disease with perianal phenotype presents
should be started for clinic follow up. She recently learned she is six
weeks pregnant and wants to discuss her Crohn’s
CORRECT ANSWER: B management during pregnancy. Her Crohn’s dis-
ease was initially diagnosed at age 16 years when
RATIONALE she presented with recurrent perianal abscesses.
PSC diagnosis is the most consistent risk factor for Evaluation confirmed perianal fistula as well as
CRC in patients with IBD. Other identified risk luminal Crohn’s disease involving the rectum and
factors include endoscopic extent of the disease ileocecal region. She was treated initially with a
(pancolitis), duration of the disease (more than prednisone taper and then placed on infliximab
eight years), age at diagnosis (young), presence monotherapy with good response. However, four
of pseudopolyps, and family history of CRC. The years later, she developed recurrent symptoms
current guidelines recommend first surveillance and was found to have high anti-infliximab anti-
colonoscopy eight to 10 years after the diagnosis of bodies with no detectable drug level. At that time,
UC or CD that involves more than 1/3 of the colon she was placed on combination therapy with adali-
with subsequent surveillance intervals at one to mumab and azathioprine, which she has main-
three years. However, for patients with a con- tained until now. Clinically, she feels well with no
comitant diagnosis of PSC, the recommendation diarrhea or rectal bleeding. She continues to have
is to initiate surveillance as soon as the coexisting intermittent drainage from the perianal fistula
diagnosis is established, with annual surveillance that she manages with short courses of antibiotics,
colonoscopy thereafter. the last three months ago.
High-dose UDCA (more than 28 mg/kg/day)
is not recommended in patients with PSC, as it Which of the following is the most appropriate
was linked to adverse outcomes in this population medical management for this patient during
including decompensated cirrhosis, death and pregnancy?
Chapter 13 — Inflammatory bowel disease 359

A. Intermittent use of ciprofloxacin for perianal trimester, has been linked to cleft lip/palate and
fistula symptoms premature rupture of membranes. This is another
B. Continue both adalimumab and azathioprine reason to continue medical therapy during preg-
during pregnancy nancy in an effort to reduce the chance of disease
C. Stop azathioprine and continue adalimumab relapse that could require systemic corticosteroid
monotherapy therapy.
D. Stop adalimumab and continue azathioprine A few studies have reported worsening peri-
during pregnancy anal disease after vaginal delivery. Furthermore,
E. Vaginal delivery is preferred over C-section in active perianal disease has been associated with
this patient increased risk of fourth degree laceration with
vaginal deliveries; therefore, in patients with ac-
CORRECT ANSWER: B tive perianal disease, C-section delivery is recom-
mended.
RATIONALE
In women with IBD, ongoing disease activity car- REFERENCE
ries the highest risk to pregnancy outcome. Stud- Nguyen, G.C., et al., The Toronto Consensus
ies have shown an increased risk of preterm birth, Statements for the Management of Inflammatory
low birth weight, stillbirth and miscarriages or Bowel Disease in Pregnancy. Gastroenterology,
abortions in patients with active disease at con- 2016. 150(3): p. 734-757.e1.
ception and during pregnancy. Therefore, keep-
ing the disease under control during pregnancy
is very important. In this high-risk patient who Question 6
failed infliximab and has perianal disease, discon- A 24-year-old medical student with a history of
tinuing her medical therapy increases her risk of extensive ulcerative colitis is currently maintained
disease relapse/flare, which could have a more on adalimumab 40 mg every two weeks. He is
adverse impact on her pregnancy than continuing preparing to go on a mission trip to Uganda and
her medications. Hence, continuing combination presents today to discuss appropriate preventive
therapy during pregnancy is the most appropriate measures prior to his trip.
medical option and the correct answer for this
question. The majority of available IBD therapies Which of the following preventive measures does
have been shown to be safe during pregnancy the patient need to avoid?
with no significant impact on the mother or the
fetus. This includes TNF-inhibitors, thiopurines, A. Hepatitis A vaccine
and most of the 5-ASA compounds. Vedolizumab, B. Malaria prophylaxis
ustekinumab and tofacitinib also appear to be safe C. Typhoid VI capsular polysaccharide vaccine
during pregnancy, although the available safety D. Yellow fever vaccine
data for these medications are more limited. On E. Influenza vaccine
the other hand, methotrexate is teratogenic and
should be avoided at conception and during preg- CORRECT ANSWER: D
nancy. Metronidazole may be associated with cleft
lip and palate and should be avoided during the RATIONALE
first trimester of pregnancy. Ciprofloxacin should Live-attenuated vaccines are contraindicated in
also be avoided during pregnancy due to the patients receiving immunosuppressive therapies.
potential risk of skeletal deformities in the fetus. These include measles, mumps, rubella, varicella,
Systemic corticosteroids can be used to control herpes zoster live-attenuated vaccine (ZVL), and
flares; however, their use, especially in the first the yellow fever vaccine. The yellow fever virus
360 Digestive Diseases Self-Education Program®

is transmitted to people by the bite of an infected evaluation revealed a mildly elevated C-reactive
mosquito. While the majority of infected pa- protein, WBC of 3.6 thousand/uL, hemoglobin of
tients have no symptoms or report mild/transient 8 g/dL, MCV of 76 fl, platelet count 430,000/mL,
symptoms of viral illness (fatigue, headache, fever, AST 67 U/L, ALT 90 U/L, and normal alkaline
etc.), some have a severe form of the disease with phosphatase and total bilirubin. Azathioprine
multiorgan failure and death. Therefore, patients metabolites included 6-TGN 300 pmol/8x108 RBC
who are unable to receive the yellow fever vac- ( normal 230-450 pmol/8x108 RBC and 6-MMP
cine should be discouraged from traveling to an 6200 pmol/8x108 RBC ( normal less than 5700
endemic region. They should also consult with pmol/8x108 RBC)
an infectious disease specialist or traveler’s clinic
to discuss potential risks of exposure to yellow Which of the following is the most appropriate
fever virus without immunization and review next step in this patient’s management?
other protective measures (use of insect repel-
lent, wearing protective gear, and avoiding travel A. Stop azathioprine and start infliximab 5mg/
during the rainy season). Influenza, Hepatitis A kg at weeks 0, two and six, then every eight
and typhoid VI capsular polysaccharide vaccines weeks
are non-live vaccines and can be administered to B. Continue azathioprine at the current dose and
this patient. None of the available medical options start oral mesalamine therapy
for malaria prophylaxis (doxycycline, mefloquine, C. Increase the dose of azathioprine to 2.5 mg/
primaquine, chloroquine and hydroxychloroquine, kg/day
and atovaquone-proguanil) is contraindicated in D. Reduce the azathioprine dose by half and
patients with IBD or those receiving TNF-inhibi- repeat liver tests in one week
tors therapies. E. No changes in therapy are warranted now, to
allow more time for the azathioprine to work
REFERENCE
Farraye, F.A., et al., ACG Clinical Guideline: Pre- CORRECT ANSWER: A
ventive Care in Inflammatory Bowel Disease. Am J
Gastroenterol, 2017. 112(2): p. 241-258. RATIONALE
The patient has clinical and biomarker evidence
of ongoing disease activity despite a therapeutic
Question 7 6-TGN level (230-450 pmol/8x108 RBC); this
A 36-year-old man with a recent diagnosis of small suggests failure of thiopurine therapy. Fur-
bowel Crohn’s disease presents for evaluation. thermore, the patient has an elevated 6-MMP
He initially presented with non-bloody diarrhea level (more than 5700 pmol/8x108 RBC) in
and weight loss.  Ileocolonoscopy showed normal association with laboratory changes concern-
colonic mucosa and diffuse inflammatory changes ing for possible hepatotoxicity (elevated liver
with ulcers in the terminal ileum.  Video capsule enzymes). The most appropriate therapy choice
endoscopy (VCE) showed active inflammation at this time is to change therapeutic class, such
involving the jejunum and more proximal ileum as as an anti-TNF agent. Both monotherapy with
well. Therapy with budesonide and azathioprine 2 an anti-TNF agent or combination therapy using
mg/kg/day was initiated. He completed a three- a lower dose of thiopurine would be reasonable
month budesonide taper and on follow up one options.
month later, reports ongoing non-bloody diarrhea The role of mesalamine in CD was recently
and abdominal cramps despite 12 weeks of aza- assessed in two Cochrane reviews that showed
thioprine therapy.  He tested negative for Clostrid- no significant benefit over placebo in induction
ium difficile toxins in the stool. Further laboratory or maintenance of remission in CD patients.
Chapter 13 — Inflammatory bowel disease 361

Increasing the azathioprine dose is unlikely to active colitis with focal areas indefinite for dys-
provide any benefit and will increase risk of toxic- plasia in the sigmoid colon biopsies. The changes
ity. His WBC is already low, so he would be at were confirmed by a second expert pathologist.
increased risk for worsening leukopenia, as well
as worsening hepatotoxicity, with an increase in
the azathioprine dose. Reducing the azathioprine
without adding a second therapeutic agent would
not address the patient’s disease activity and
hence option D is also incorrect. Finally, although
thiopurines have a slow onset of action and may
take up to six to 12 weeks to achieve full clinical
response, it is unlikely any additional benefit will
be observed after 12 weeks, especially in patients
with a therapeutic 6-TGN level.
Which of the following is the most appropriate
REFERENCES intervention?
Vande Casteele, N., et al., American Gastroentero-
logical Association Institute Technical Review on A. Initiate combination oral and rectal 5-ASA
the Role of Therapeutic Drug Monitoring in the therapy and schedule repeat surveillance colo-
Management of Inflammatory Bowel Diseases. noscopy in three months
Gastroenterology, 2017. 153(3): p. 835-857.e6. B. Refer the patient for total colectomy with ileal
Dubinsky, M.C., et al., Pharmacogenomics and pouch anal anastomosis (IPAA)
metabolite measurement for 6-mercaptopurine C. Refer the patient for segmental resection of
therapy in inflammatory bowel disease. Gastroen- the sigmoid colon
terology, 2000. 118(4): p. 705-13. D. Repeat colonoscopy with chromoendoscopy now
E. Initiate combination therapy with infliximab
and azathioprine and schedule repeat colonos-
Question 8 copy in three to six months
A 45-year-old man with a history of extensive
ulcerative colitis diagnosed 10 years ago presents CORRECT ANSWER: A
to clinic. He was treated initially with oral mesala-
mine with good clinical response, but after a year RATIONALE
of therapy was lost to follow up and eventually Indefinite dysplasia usually represents atypia with
self-discontinued the mesalamine. He did well off confounding background inflammation, mak-
therapy until one month ago when he experienced ing it difficult for the pathologist to confirm if the
recurrent diarrhea with three to four loose bowel changes represent a secondary effect of inflamma-
movements per day with urgency and intermittent tion or true dysplasia. This finding on histology
rectal bleeding. Stool studies were negative for warrants close follow up after optimizing medical
any infectious process and his laboratory evalu- therapy. Therefore, option A is the most appropri-
ation revealed a normal CRP, Hgb of 12.1 g/dl, ate answer for this case.
MCV of 80 fL and normal liver enzymes. He is a Options B and C are incorrect as it is too early
nonsmoker and has no family history of colon can- to recommend colectomy. However, if invis-
cer. Colonoscopy using high-definition white light ible dysplasia is confirmed after controlling the
endoscopy (HD-WLE) showed mild to moderate inflammation, then colectomy should be consid-
pancolitis (see picture). Random biopsies obtained ered, especially in cases with multifocal dysplasia
from each segment of the colon showed chronic or high-grade dysplasia. Total colectomy is the
362 Digestive Diseases Self-Education Program®

recommended approach given the high risk of Virtual Chromoendoscopy for Detection of Co-
both synchronous and metachronous lesions. lonic Neoplastic Lesions During IBD Surveillance
Segmental colectomy can be considered in certain Colonoscopy. Am J Gastroenterol, 2018. 113(2): p.
situations on a case by case basis, for example in 225-234.
those who are not felt to be candidates for more
extensive surgery or those who are unable to
care for an ostomy, etc. Chromoendoscopy is a Question 9
technique that uses dye to enhance visualization You are asked to consult on a 27-year-old man
and dysplasia detection during surveillance. The hospitalized with severe bloody diarrhea, up to 15
Surveillance for Colorectal Endoscopic Neoplasia bowel movements per day, abdominal pain, nau-
Detection and Management in IBD patients: inter- sea and low-grade fevers for the last two weeks.
national Consensus (SCENIC) group recommend- He was diagnosed with ulcerative pancolitis five
ed use of chromoendoscopy whenever colonos- years ago and was maintained on mesalamine
copy is performed with standard-definition white 4.8 g/day and azathioprine 2mg/kg/day until
light endoscopy. The SCENIC group also suggest- six months ago when he lost his insurance and
ed chromoendoscopy be used in conjunction with stopped all medical therapy. On examination,
high definition white light endoscopy. However, he appears uncomfortable, HR 95 beats/minute
option D is incorrect because optimization of and temperature 37.9 °C. His abdomen is not
therapy should be achieved prior to repeating distended, but he has tenderness to palpation dif-
the endoscopic evaluation, even if chromoendos- fusely, worse in the left lower quadrant. His labs
copy is performed. Finally, the patient has mild reveal a WBC of 8,000/mL, Hgb 8.1 g/dl, albumin
disease with a history of good clinical response to 2.7 g/dl, and CRP 18 mg/L (normal: less than 0.3).
mesalamine therapy in the past; therefore, resum- What is the most appropriate next step?
ing mesalamine therapy is appropriate and escala-
tion to a biologic or thiopurine is not indicated at A. Send stool sample to test for Clostridium dif-
this time. Combining oral and rectal mesalamine ficile toxins
therapies has been shown to produce earlier and B. Initiate infliximab 5mg/kg
more complete relief of rectal bleeding compared C. Initiate IV methylprednisolone 300mg/day
with oral or rectal therapy alone. D. Obtain an abdominal ultrasound
E. Discontinue his prophylactic heparin, given
REFERENCES his anemia and rectal bleeding
Cairns, S.R., et al., Guidelines for colorectal cancer
screening and surveillance in moderate and high CORRECT ANSWER: A
risk groups (update from 2002). Gut, 2010. 59(5):
p. 666-89. RATIONALE
Laine, L., et al., SCENIC international consen- The patient’s presentation is consistent with acute
sus statement on surveillance and management of severe ulcerative colitis. Superimposed infections
dysplasia in inflammatory bowel disease. Gastro- should be excluded in all patients; this typically
enterology, 2015. 148(3): p. 639-651.e28. involves obtaining stool samples to assess for
Bisschops, R., et al., Chromoendoscopy versus Clostridium difficile infection and other GI patho-
narrow band imaging in UC: a prospective ran- gens. Intravenous systemic corticosteroid is the
domised controlled trial. Gut, 2018. 67(6): p. initial therapy of choice, typically with a dose of 60
1087-1094. mg/day of methylprednisolone or equivalent. If
Iacucci, M., et al., A Randomized Trial Com- no clear clinical response after 72 hours of cortico-
paring High Definition Colonoscopy Alone With steroid therapy, infliximab or cyclosporine rescue
High Definition Dye Spraying and Electronic therapy should be considered. Patients with IBD
Chapter 13 — Inflammatory bowel disease 363

have a three to four-fold increase in risk of venous through the electronic health records system. She
thromboembolism. This risk is even higher in the indicates that the red lesions in the pictures de-
setting of active disease, hospitalization, and cor- veloped two days ago and are limited to her lower
ticosteroid therapy. All hospitalized IBD patients extremities. They are raised and painful. She has
should be placed on pharmacologic DVT prophy- noted no change in her bowel symptoms and denies
laxis. In addition, rectal bleeding and anemia are fever or any other accompanying symptoms.
not contraindications for DVT prophylaxis.
Which of the following is an appropriate next step
REFERENCES in management?
Kornbluth, A. and D.B. Sachar, Ulcerative colitis prac-
tice guidelines in adults: American College Of Gas- A. Increase sulfasalazine to 4 g/day
troenterology, Practice Parameters Committee. Am J B. Add azathioprine to her current dose of
Gastroenterol, 2010. 105(3): p. 501-23; quiz 524. sulfasalazine
Grainge, M.J., J. West, and T.R. Card, Ve- C. Obtain a skin biopsy of the lesions
nous thromboembolism during active disease and D. Switch her therapy from sulfasalazine to
remission in inflammatory bowel disease: a cohort mesalamine
study. Lancet, 2010. 375(9715): p. 657-63. E. Start prednisone 60 mg daily
Kappelman, M.D., et al., Thromboembolic risk
among Danish children and adults with inflamma- CORRECT ANSWER: D
tory bowel diseases: a population-based nation-
wide study. Gut, 2011. 60(7): p. 937-43. RATIONALE
The described lesions are mostly consistent with
erythema nodosum (EN). EN is a form of pan-
Question 10 niculitis and represents the most common cutane-
A 51-year-old female diagnosed with mild left-sided ous manifestation associated IBD, occurring in up
ulcerative colitis two months ago was placed on to 10 percent of the cases. When EN is associated
sulfasalazine 2 g/day and daily folic acid with sig- with IBD, it usually parallels disease activity and
nificant improvement in her symptoms. At her last management should focus on treating the IBD.
clinic visit a week ago, she was having two formed EN can also be seen in association with several
non-bloody stools per day without urgency and her other conditions and drugs (see table below).
labs revealed a normal CBC, CRP and normaliza- Thus, not every EN presentation in IBD patients
tion of the previously elevated fecal calprotectin. is related to the bowel disease. This patient is in
Since then, she sent you the pictures shown below clinical remission and most likely endoscopic re-
mission given normalization of her inflammatory
markers. She has no other associated symptoms
to suggest any systemic disease or infection. Her
only change in medications was the initiation of
sulfasalazine two months ago at the onset of her
disease. So, it is reasonable to consider sulfasala-
zine as a potential cause of her EN. In the case of
sulfasalazine, the sulfonamide component of the
drug is the presumed trigger for EN, so switching
to a mesalamine agent is reasonable. There is no
indication to increase the sulfasalazine or add aza-
thioprine therapy at this time. While skin biopsy
can be helpful to confirm the diagnosis, in the ma-
364 Digestive Diseases Self-Education Program®

Potential causes of erythema nodosum


Idiopathic Most common (17-72 percent of the reported cases)
Infections Streptococcal infection is the most common infectious etiology for EN. It has been reported with
other infections including: tuberculosis, Yersinia, leprosy, mycoplasma pneumonia, tularemia,
leptospirosis, brucellosis, chlamydia, psittacosis, cat-scratch disease, coccidioidomycosis, histo-
plasmosis, blastomycosis, infectious mononucleosis, hepatitis B, and paravaccinia
Malignancy Hodgkin’s Lymphoma, leukemia, and internal carcinomas
Drugs Penicillin, sulfonamides, oral contraceptive pills, TNF-inhibitors
Enteropathies CD and UC
Miscellaneous Pregnancy, sarcoidosis, Whipple disease, Behcet disease, Sweet syndrome

jority of cases, the diagnosis can be made based habits and no fever or chills. Labs reveal an
on the clinical presentation without the need elevated lipase level of 800 U/L and normal AST,
for biopsy, as in this case. In regard to manage- ALT, alkaline phosphatase and total bilirubin.
ment, EN is usually a self-limited condition and Her lipid panel and electrolytes are normal. She
typically resolves spontaneously without treat- denies any alcohol use. CT scan of the abdomen
ment. Therefore, the first line of therapy is often reveals edema in the peripancreatic fat. Choleli-
supportive care with compression bandages, thiasis was also noted, without choledocholithia-
elevation of the legs, and PRN NSAID use along sis or common bile duct (CBD) dilatation.
with addressing any potential treatable or modi-
fiable (i.e. medication) cause. In more severe In addition to the usual supportive care, which
cases, short courses of systemic corticosteroids of the following is appropriate with regard to
can be used, usually at 20 mg daily of predni- management of her condition?
sone. Higher doses are rarely required. Other
therapies that can be used include colchicine, A. She should have cholecystectomy before
dapsone, hydroxychloroquine, TNF-inhibitors, discharge
thalidomide and methotrexate. B. Discontinue budesonide
C. Discontinue 6-MP and start azathioprine
REFERENCE after recovery from the current episode
Garcia-Porrua, C., et al., Erythema nodosum: D. Discontinue 6-MP and consider initiating
etiologic and predictive factors in a defined biologic therapy after recovery from the
population. Arthritis Rheum, 2000. 43(3): p. current episode
584-92. E. Discontinue budesonide and 6-MP and
initiate high dose intravenous corticosteroid
therapy
Question 11
A 45-year-old female with a recent diagnosis of CORRECT ANSWER: D
inflammatory ileocolonic CD presents for evalu-
ation. After reviewing all therapy options with RATIONALE
her, she agrees to proceed with a budesonide ta- This patient’s presentation is consistent with
per and 6-MP 1.5 mg/kg/day. Three weeks after drug-induced pancreatitis. Among all the IBD
initiating therapy, she presents to the emergency therapies, thiopurines are the drugs most fre-
room with sharp epigastric pain radiating to the quently implicated as a cause of acute pancreati-
back. She reports nausea and vomiting in asso- tis. This reaction is usually a dose-independent
ciation with the pain, but no changes in her bowel idiosyncratic reaction, and is considered a
Chapter 13 — Inflammatory bowel disease 365

contraindication for the use of any thiopurine patient underwent cystoscopy with left ureteral
agent. Less frequently, 5-ASA can also cause stent placement.
acute pancreatitis and in extremely rare reports,
metronidazole and steroid therapy have been He presents today for follow up and is wonder-
implicated. In this patient, the acute pancreati- ing what he can do to reduce the risk of further
tis is most likely caused by the 6-MP therapy. stone formation. Which diet should be recom-
Therefore, both 6-MP and azathioprine use mended to this patient?
would be contraindicated. The patient should
discontinue 6-MP and plan initiation of a differ- A. Low oxalate, low calcium and low fat diet
ent class of therapy once she recovers from her B. Low oxalate, high calcium and low fat diet
pancreatitis. C. Low oxalate, low calcium and high fat diet
The clinical presentation is not consistent D. High oxalate, low calcium and low fat diet
with gallstone pancreatitis given her normal E. High oxalate, high calcium and low fat diet
liver enzymes and lack of choledocholithiasis
or CBD abnormalities on abdominal imaging. CORRECT ANSWER: B
Therefore, cholecystectomy is not indicated.
With no significant changes in her Crohn’s RATIONALE
symptoms, there is no indication for high dose Patients with CD are prone to develop neph-
IV corticosteroid therapy. However, given the rolithiasis. The most frequent types of stones
need to discontinue 6-MP as well as the potential seen in CD patients are calcium oxalate and
lag time until she recovers and is able to initiate uric acid stones. Calcium oxalate stones result
a more definitive CD therapy, budesonide should from hyperoxaluria associated with disruption
be continued. of the enterohepatic circulation and bile acid
malabsorption, as in patients with distal ileal
REFERENCES disease or a history of ileal resection. As more
Ramos, L.R., et al., Inflammatory Bowel Disease bile acids enter the colon, they competitively
and Pancreatitis: A Review. J Crohns Colitis, bind with calcium, allowing more free oxalate
2016. 10(1): p. 95-104. to be absorbed into the circulation and eventu-
Chaparro, M., et al., Safety of thiopurine ally excreted through the kidneys where it binds
therapy in inflammatory bowel disease: long- to calcium and forms calcium oxalate stones.
term follow-up study of 3931 patients. Inflamm Altered colonic permeability and GI decoloniza-
Bowel Dis, 2013. 19(7): p. 1404-10. tion of Oxalobacter formigenes have also been
implicated in the risk of calcium oxalate stone
formation. The patient described in the case is
Question 12 most likely suffering from calcium oxalate stones
A 31-year-old man with a history of stricturing given his prior ileocecectomy. In contrast, uric
ileocolonic Crohn’s disease requiring ileocecec- acid stones are caused by low urine pH and low
tomy four years ago returns to your clinic. He urine volume, both of which can develop in the
is currently maintained on adalimumab mono- setting of increased stool output, particularly
therapy, 40 mg every 14 days. He recently in patients with an end ileostomy or significant
visited the emergency room for evaluation of colonic resection. Management strategies differ
left flank pain and blood in the urine. A CT scan slightly depending on the expected type of stone.
revealed bilateral nephrolithiasis with a six mm For calcium oxalate stones, in addition to hydra-
non-obstructing right renal stone and a four mm tion, dietary modifications are important to en-
left ureteral stone with mild left hydronephro- hance the formation of insoluble calcium oxalate
sis. The urology service was consulted, and the complexes to be excreted in the stool. Patients
366 Digestive Diseases Self-Education Program®

should restrict dietary fat and oxalate and increase TAVE 2) with combined clinical and endoscopic
calcium intake. Cholestyramine, a bile salt binder, remission rates at eight weeks of 18.5 percent vs
can reduce fat malabsorption and enhance oxalate 8.2 percent and 16.6 percent vs 3.6 percent for pa-
excretion in the stool. Calcium supplementation tients receiving tofacitinib vs placebo in OCTAVE
can also be of use. For uric acid stones, manage- 1 and OCTAVE 2, respectively. Furthermore, the
ment involves hydration and alkalization of the treatment effect was similar between patients who
urine using potassium bicarbonate or potassium previously failed anti-TNF therapy and those who
citrate. were anti-TNF naïve. In the maintenance trial
(OCTAVE-Sustain), patients who responded to
REFERENCE the induction phase were randomized to receive
Corica, D. and C. Romano, Renal Involvement in tofacitinib at 5 mg BID, 10mg BID or placebo.
Inflammatory Bowel Diseases. J Crohns Colitis, Remission at week 52 was achieved in 34.3 per-
2016. 10(2): p. 226 35. cent, 40.6 percent and 11.1 percent in each group,
respectively. From a safety standpoint, nasophar-
yngitis, arthralgias and headaches were among
Question 13 the most commonly reported adverse effects in
A 55-year-old man with a history of ulcerative the treatment groups during the UC clinical trials.
pancolitis diagnosed six years ago presents for a Of the more serious adverse effects, herpes zoster
second opinion regarding treatment options. He infection was reported in 18 patients and non-mel-
failed several 5-ASA formulations, azathioprine, anoma skin cancer was reported in five patients
adalimumab monotherapy, and combination receiving tofacitinib during all three clinical trials.
therapy with infliximab and 6-MP. After discussing No other malignancies were reported. Increases
all the therapy options with the patient, he elected in levels of LDL, HDL and cholesterol were also
to start tofacitinib. Which of the following is true noted, with maximum effects generally observed
regarding tofacitinib therapy in ulcerative colitis? within the first six weeks of therapy. Hence, a lipid
panel assessment is recommended within four to
A. It often takes more than 16 weeks to see a eight weeks of initiation of therapy. Just as with
response other immunosuppressive medications, live vac-
B. Assessment of lipid parameters is recom- cines are contraindicated while on tofacitinib. In
mended in all patients after four to eight addition to UC, tofacitinib has also proven to be
weeks of tofacitinib therapy effective in managing rheumatoid arthritis (RA)
C. Live-attenuated vaccines are not contraindi- and psoriatic arthritis. It is currently FDA ap-
cated while receiving tofacitinib proved for all three conditions. It can therefore be
D. The most common infection seen while on this considered in patients with concomitant diagnoses
drug is influenza. of UC and RA or psoriatic arthritis.
E. It is contraindicated in patients with concomi-
tant rheumatoid arthritis REFERENCE
Sandborn, W.J., et al., Tofacitinib as Induction
CORRECT ANSWER: B and Maintenance Therapy for Ulcerative Colitis. N
Engl J Med, 2017. 376(18): p. 1723-1736.
RATIONALE
Tofacitinib is an oral, small molecule janus kinase
inhibitor that was recently approved by the FDA Question 14
for induction and maintenance of remission in UC A 45-year-old female with a history of refractory
patients. Its efficacy for induction of remission ulcerative colitis underwent total proctocolec-
was shown in two large RCTs (OCTAVE 1 and OC- tomy with ileal pouch anal anastomosis 15 years
Chapter 13 — Inflammatory bowel disease 367

ago. Prior to her surgery, she failed mesalamine bowel inflammation. Oral budesonide is an op-
therapy, azathioprine, and infliximab. She has tion for patients who do not respond to antibiotic
done well since her colectomy with only three epi- therapy, but is not indicated for this patient at
sodes of pouchitis over the past 15 years, managed this time. Vedolizumab, adalimumab and 6-MP
with short courses of antibiotics. She presents to are not indicated at this time given the absence of
establish care at your clinic and indicates she has evidence of Crohn’s disease. Mesalamine supposi-
recently experienced an increase in stool fre- tories can be used in cases of cuffitis, but cuffitis is
quency and urgency, as well as lower abdominal not present in this patient.
pain. She denies fever or chills, recent antibiotic
use, travel or sick contacts. She takes loperamide REFERENCES
as needed to reduce bowel frequency and ibu- Wu, H. and B. Shen, Pouchitis and pouch dysfunc-
profen daily for low back and knee pain that has tion. Gastroenterol Clin North Am, 2009. 38(4): p.
recently increased. Pouchoscopy reveals diffuse 651-68.
inflammation of the pouch and the pre-pouch Hata, K., et al., Pouchitis after ileal pouch-anal
ileum, up to five cm. No strictures were seen, and anastomosis in ulcerative colitis: Diagnosis, man-
the rectal cuff mucosa appeared normal. Biopsies agement, risk factors, and incidence. Dig Endosc,
from the pouch and the pre-pouch ileum revealed 2017. 29(1): p. 26-34.
small intestinal mucosa with moderate to severe
chronic inflammation and edema. No granulomas
were seen. You started a course of ciprofloxacin Question 15
and metronidazole. What additional measure is You are seeing a 25-year-old female who initially
recommended? presented for evaluation of bloody diarrhea and
abdominal pain. Her initial laboratory results are
A. Stop NSAID use shown in the table below:
B. Start vedolizumab
C. Start budesonide Further evaluation with ileocolonoscopy revealed
D. Start adalimumab and 6-MP evidence of mild to moderate left sided colonic
E. Start mesalamine suppositories
Test Result Normal range
CORRECT ANSWER: A
White cell count 5,100/L 4.8-11.8
Hemoglobin 8.3 g/dl 10.5-15
RATIONALE
The clinical presentation is most consistent with MCV 72fL 77-96

pouchitis, that may have been triggered by NSAID Platelet 233 bil/L 130-460
use. The presence of pre-pouch ileitis limited Albumin 3.1 g/L 3.2-4.6
to the distal 10 cm of the pre-pouch ileum in ALT 71 U/L 7-37
the setting of diffuse pouchitis is more likely an
AST 63 U/L 0-30
equivalent to backwash ileitis seen in patients
Alkaline phosphatase 69 U/L 37-132
with pancolitis, as opposed to Crohn’s disease of
the small bowel. NSAIDs may also be a cause of Total bilirubin 0.6 mg/dal 0.0-1
ileal inflammation proximal to the pouch. From C-reactive protein 1.3 mg/dl 0.0-0.8
a management standpoint, this patient should be Fecal calprotectin 455 mcg/g <= 50
treated for her pouchitis with antibiotics. Dis-
Ferritin 6 ng/ml 12-300
continuing NSAID use is also recommended, as
Vitamin B12 260 pg/ml 200-1240
NSAID use is a risk factor for the development of
pouchitis and may be a direct cause of her small Folate 3 ng/ml 3.5-17.5
368 Digestive Diseases Self-Education Program®

inflammation (marked erythema, friability and sociation between celiac disease and IBD, with re-
erosions) with a normal terminal ileum. Biopsies ports of increased prevalence of IBD among celiac
of the inflamed mucosa revealed histopathologic disease patients and vice versa. The diagnosis can
changes consistent with chronic inflammation be challenging because of the overlap in clinical
without granulomas. MR enterography did not and laboratory features, but it should be suspected
show any small bowel disease. Based on these in IBD patients who have persistent symptoms de-
findings, she was given a diagnosis of left-sided spite medical therapy. Another feature to suggest
ulcerative colitis and started on combined oral possible celiac disease in this case are the nutri-
and topical mesalamine therapy. A complete liver tional deficiencies that appear to be out of propor-
disease work-up revealed normal anti-nuclear tion to her colonic disease, and the unexplained
antibodies, antimitochondrial antibodies, and elevation of liver transaminases that can be seen
anti-smooth muscles antibodies, negative hepati- in patients with celiac disease. If celiac disease
tis C and B serologies, and normal ceruloplasmin is suspected, the first step in approaching the
level, urine copper and alpha 1 antitrypsin phe- diagnosis is serologic testing with total IgA and
notype testing. Liver ultrasound did not show IgA-tTG. This is usually followed by upper endos-
any significant abnormalities. On follow up two copy with duodenal biopsies to assess the degree
months after initiation of mesalamine therapy, the of inflammation. VCE may have a role in assess-
patient reported no further rectal bleeding and her ing celiac disease extension and identification of
abdominal pain was significantly improved, but complications. It may also have a role in assessing
she continued to have diarrhea. Repeat labora- for Crohn’s disease as a potential cause for her
tory studies showed her C-reactive protein had persistent diarrhea if her celiac disease testing is
normalized, and fecal calprotectin was down to 54 negative. However, celiac serology testing should
mcg/g, but she had persistent microcytic anemia still be obtained first. Glucose hydrogen breath
despite oral iron supplements, as well as elevation testing is usually used to assess for small intestinal
of ALT and AST. Flexible sigmoidoscopy while on bacterial overgrowth, which is another potential
mesalamine therapy revealed significant improve- cause for her persistent diarrhea, and can be con-
ment with only patchy erythema and no ulcers, sidered if her celiac disease assessment is nega-
erosions or friability. tive. MRCP is unlikely to assist in diagnosing her
condition, as her liver enzymes abnormalities are
Which of the following is the most appropriate not consistent with a cholestatic pattern.
next step to investigate the cause of her persistent
diarrhea? REFERENCE
Kelly, C.P., et al., Advances in diagnosis and man-
A. Schedule video capsule endoscopy (VCE) agement of celiac disease. Gastroenterology, 2015.
B. Schedule EGD with duodenal biopsies 148(6): p. 1175-86.
C. Obtain total immunoglobulin A (IgA) level and
IgA tissue transglutaminase antibodies (IgA-tTG)
D. Schedule her for magnetic resonance Question 16
cholangiopancreatography (MRCP) A 17-year-old female with a history of penetrat-
E. Obtain glucose hydrogen breath test ing and stricturing small bowel Crohn’s disease
complicated by enteroenteric and enterovesical
CORRECT ANSWER: C fistulas presents for follow up after recent surgical
resection. She was diagnosed one year prior to her
RATIONALE surgery and has had six hospitalizations, recurrent
Patients with IBD can have concomitant celiac urinary tract infections, failure to thrive, requiring
disease and studies have actually suggested an as- total parenteral nutrition that was complicated by
Chapter 13 — Inflammatory bowel disease 369

a central line infection. She received prolonged mended over endoscopy-guided therapy. Recently
systemic corticosteroids and attempted steroid- published guidelines by the American Gastro-
sparing therapies with 6-MP 1.5mg/kg/day for at enterology Association (AGA) provide guidance
least four months with no clinical response, then for the choice of therapy in this setting, recom-
switched to infliximab monotherapy but was only mending anti-TNF therapy and/or thiopurine as
able to receive three doses due to her infectious first-line therapy over other agents. The evidence
complications and hospitalizations. In addition, supporting the use of anti-TNF monotherapy vs
she had an infusion reaction with her third dose, thiopurine monotherapy, vs combination therapy
requiring discontinuation of the infusion and a with an anti-TNF agent and a thiopurine is either
dose of IV methylprednisolone. lacking or of very low-quality. Thus, the choice of
She is now four weeks post-operative and feels therapy should be made based on individual risk
well with no abdominal pain, diarrhea, or rectal assessment, therapy experience prior to surgery,
bleeding. She denies fecal urgency, dysuria or and other risk-benefit considerations. For this
pneumaturia. She has good energy and has regained young patient with severe penetrating disease
most of the weight she lost over the past year. She is and a prior reaction to infliximab, combination
currently completing her senior year of high school. therapy with an anti-TNF agent and azathioprine
She is nonsmoker but says her father smokes. She appears to be the most appropriate therapy and
has no family history of IBD or GI malignancies. She the only acceptable choice among the listed op-
has been off all IBD therapy since her surgery. tions. Antibiotics (nitroimidazoles) have been
suggested as a second-line alternative in patients
Which of the following is recommended at this with concerns about adverse effects associated
time? with anti-TNF therapies and thiopurines. The
evidence supporting any potential role for mesa-
A. Start oral mesalamine 4 g per day and sched- lamine in post-operative prophylaxis is very low
ule colonoscopy six months after surgery and, given the lack of efficacy in luminal CD, the
B. Schedule colonoscopy six months after sur- AGA guidelines suggest against use of mesalamine
gery and initiate therapy only if evidence of for post-operative prophylaxis. The guidelines
disease recurrence also recommend against the use of budesonide
C. Start budesonide 9 mg daily and schedule and probiotics because of substantial uncertainties
colonoscopy six months after surgery regarding their effectiveness. There have been no
D. Start adalimumab and azathioprine 2-2.5mg/ clinical trials evaluating the use of methotrexate
kg/day now and schedule colonoscopy six to for post-operative prophylaxis and therefore, it
12 months after surgery should not be recommended as first-line therapy.
E. Start methotrexate 25 mg subcutaneous injec- Regardless of whether early prophylactic
tions weekly now and schedule colonoscopy pharmacological therapy is initiated or not, all
six to 12 months after surgery patients should undergo repeat colonoscopy six
to 12 months after surgical resection to evaluate
CORRECT ANSWER: D for endoscopic disease recurrence, with initiation
or optimization of therapy should recurrence be
RATIONALE confirmed.
This patient is considered at high risk for disease
recurrence following her surgical resection, due REFERENCE
to the young age of onset and her complicated Nguyen, G.C., et al., American Gastroenterological
presentation with rapid progression to surgery. Association Institute Guideline on the Manage-
Exposure to second-hand smoke may be adding ment of Crohn’s Disease After Surgical Resection.
to her risk. Therefore, early prophylaxis is recom- Gastroenterology, 2017. 152(1): p. 271-275.
370 Digestive Diseases Self-Education Program®

Question 17 Delaying colectomy until she completes her fam-


A 29-year-old female with a long-standing his- ily is not an appropriate option in the setting of
tory of pancolonic ulcerative colitis is currently multifocal low-grade dysplasia given the high risk
maintained on azathioprine 125 mg daily. She of progression to malignancy, as well as synchro-
is in clinical and endoscopic remission, but was nous higher risk lesions (high grade dysplasia and
found to have multifocal low-grade flat dysplasia adenocarcinoma). IBD therapies prior to surgery
on recent surveillance colonoscopy. The pathol- have no impact on fertility.
ogy findings were confirmed by a second expert
pathologist. You recommend total proctocolecto- REFERENCES
my with ileal pouch anal anastomosis (IPAA), but Vermeire, S., et al., Management of inflamma-
the patient was recently married and expresses tory bowel disease in pregnancy. J Crohns Colitis,
concerns about her ability to have a successful 2012. 6(8): p. 811-23.
pregnancy after surgery. Bharadwaj, S., et al., Women’s health issues
after ileal pouch surgery. Inflamm Bowel Dis,
Which of the following statements is correct? 2014. 20(12): p. 2470-82.

A. Total proctocolectomy with IPAA creation has


no impact on fertility Question 18
B. IBD therapies used prior to surgery may im- In which of the following clinical scenarios is inf-
pact the ability to conceive after surgery liximab therapy contraindicated?
C. The patient should consider proceeding with
colectomy with ileostomy and deferring the A. A 25-year-old woman with ulcerative pancoli-
IPAA creation until she completes her family. tis who is 10 weeks pregnant
D. In vitro fertilization is rarely successful in B. A 43-year-old man with active ileocolonic
patients with IPAA Crohn’s disease and recent diagnosis of latent
E. The patient should consider deferring any tuberculosis, currently on week 12 of isoniazid
surgery until she completes her family. therapy
C. A 65-year-old woman with active colonic
CORRECT ANSWER: C Crohn’s disease and a history of breast cancer
treated eight years ago
RATIONALE D. A 54-year-old man with left-sided colitis and
There is a substantial evidence in the literature a history of myocardial infarction two years
suggesting reduced fertility following restorative ago with a normal ejection fraction on recent
proctocolectomy with IPAA surgery. This is be- echocardiogram
lieved to be caused by adhesions and scar tissue E. A 29-year-old woman with left-sided colitis
that develop from the pelvic dissection during the and a personal history of optic neuritis
pouch creation and impact the nearby fallopian
tubes. Therefore, it is not the colectomy that im- CORRECT ANSWER: E
pacts fertility, but the restorative/IPAA creation.
With this in mind, for young females who need RATIONALE
colectomy and are interested in future pregnancy, There is a clear association between the use of
performing colectomy with ileostomy and plan- anti-TNF treatment and the development of
ning completion IPAA creation and stoma take demyelinating disorders including optic neuritis
down after childbearing is reasonable. In vitro and multiple sclerosis. Hence, anti-TNF therapies
fertilization is another option, and has been shown are contraindicated in patients with a personal
to have high success rates in patients with IPAA. history of optic neuritis or multiple sclerosis. It
Chapter 13 — Inflammatory bowel disease 371

Is also suggested to avoid anti-TNFs in patients A. Reduce the infliximab interval to every six
with a family history of demyelinating disease weeks
given the potential genetic predisposition. All B. Start vedolizumab 300 mg at weeks 0, two, six
anti-TNF agents can be used safely during preg- and then every eight weeks
nancy. For patients with latent tuberculosis (TB), C. Start ustekinumab IV induction dose based on
it is preferable to avoid initiating anti-TNF treat- her weight, then 90 mg subcutaneous injec-
ment until completion of the latent TB therapy. tions every eight weeks
However, in more urgent cases including poorly D. Switch azathioprine to methotrexate
controlled IBD, it acceptable to initiate anti-TNF E. Start adalimumab 160 mg at week 0, then 80
therapy after a minimum of four weeks of latent mg at week two, then 40 mg every two weeks
TB treatment. Patients with recent or active
solid malignancy should avoid anti-TNF therapy CORRECT ANSWER: A
given the potential impact on disease progression.
However, in patients with a history of solid cancer RATIONALE
treated more than five years ago, initiating anti- This patient has evidence of active luminal
TNF treatment is not contraindicated. Finally, Crohn’s disease along with increased activity of
TNF inhibitors should be avoided in patients with her perianal fistula. Therapeutic drug monitoring
moderate to severe heart failure. A history of reveals a low drug level (less than 5mcg/ml) with
myocardial infarction with intact cardiac function no detectable anti-drug antibodies. This sug-
is not a contraindication. gests a non-immune-mediated pharmacokinetic
failure and in such cases, it is recommended to
REFERENCE optimize the index drug before switching to a dif-
Lichtenstein, G.R., et al., ACG Clinical Guideline: ferent agent. Optimization of the index drug can
Management of Crohn’s Disease in Adults. Am J be achieved by increasing the drug dose, reducing
Gastroenterol, 2018. 113(4): p. 481-517. the dosing interval, or adding immunomodulator
therapy. This patient is already receiving combi-
nation therapy with azathioprine and infliximab,
Question 19 therefore increasing the infliximab dose or re-
You are seeing a 24-year-old female with a his- ducing the dose interval are appropriate inter-
tory of penetrating Crohn’s colitis with perianal ventions. There is no indication to switch from
phenotype. She is currently maintained on inflix- azathioprine to methotrexate. Vedolizumab, an
imab 5mg/kg every eight weeks and azathioprine anti-integrin, and ustekinumab, an IL12/IL23 in-
2 mg/kg/day. She indicates she has noticed hibitor, are both therapeutic options if the patient
increased bowel frequency, urgency and drainage continues to have active disease despite optimiza-
from her perianal fistula. Infectious etiologies tion of her infliximab therapy. Adalimumab is
are excluded, and MRI of the pelvis confirms the another TNF-inhibitor, and would be an appropri-
presence of a perianal fistula with no abscesses. ate choice if the patient had a high titer of anti-
Ileocolonoscopy shows mild-moderate active drug antibodies suggesting an immune-mediated
colitis involving the rectum and segments of the pharmacokinetic failure
transverse and ascending colon. Therapeutic
drug monitoring reveals a trough infliximab drug REFERENCE
level of 3.5 mcg/ml with no detectable anti-inflix- Vande Casteele, N., et al., American Gastroentero-
imab antibodies. logical Association Institute Technical Review on
the Role of Therapeutic Drug Monitoring in the
In addition to antibiotic treatment, which of the Management of Inflammatory Bowel Diseases.
following would you recommend? Gastroenterology, 2017. 153(3): p. 835-857.e6.
372 Digestive Diseases Self-Education Program®

Question 20 B. Schedule video capsule endoscopy for more


A 58-year-old woman with a long-standing his- accurate assessment of the small bowel
tory of rheumatoid arthritis was referred to you C. Continue methotrexate at the current dose
by her rheumatologist for evaluation of possible and start ustekinumab
IBD. She has had intermittent non-bloody, loose D. Stop all NSAID use
stools alternating with normal bowel movements E. Stop the methotrexate and start anti-TNF
for several years, but for the last four months monotherapy
she has also experienced intermittent abdominal
pain. This prompted her rheumatologist to order CORRECT ANSWER: D
IBD serological testing that showed a “pattern
consistent with IBD favoring ulcerative colitis.” RATIONALE
She is currently maintained on methotrexate The patient’s presentation is most consistent with
subcutaneous injection 25mg weekly and takes NSAID-induced enteropathy. NSAIDs can cause
naproxen daily with ibuprofen as needed for her a wide variety of endoscopic lesions including ero-
arthritis. She has no other medical conditions and sions, large and small ulcers, and diaphragm-like
she does not take any other prescribed medica- strictures. Such strictures are usually thin, con-
tions. Her routine screening colonoscopy eight centric, diaphragm-like septa with a pinhole-sized
years ago was normal, but the terminal ileum was lumen and considered pathognomonic for NSAID-
not assessed. You recommend a colonoscopy that induced enteropathy. On the histopathological as-
reveals a diaphragm-like stricture in the ascending sessment of these strictures, fibrosis of the sub-
colon with surrounding erythema and scattered mucosa with intact muscularis propria is usually
erosions. The stricture was traversed after endo- seen, which is why the risk of perforation is very
scopic balloon dilation. Once traversed, a second low with endoscopic dilations. The IBD diagnostic
wider, diaphragm-like stricture was seen and was panel (IBD serology) is not considered a diagnostic
easily traversed. The remaining colonic mucosa, test, and results can only be interpreted within the
ileocecal valve and terminal ileum appeared clinical context. Despite a positive IBD serology,
normal. Biopsies from the strictures did not show the clinical picture is more consistent with NSAID-
evidence of malignancy. MR enterography did not induced enteropathy rather than IBD and should be
show any small bowel abnormalities. managed as such. There is no indication to initi-
ate biologic therapies at this time. Video capsule
endoscopy can be beneficial is assessing NSAID-
induced injury in the small bowel as well as evalu-
ating for small bowel Crohn’s disease. However, in
this case, VCE is contraindicated given the known
colonic strictures and risk of capsule retention.

REFERENCE
Shin, S.J., et al., Non-steroidal anti-inflammatory
drug-induced enteropathy. Intest Res, 2017. 15(4):
p. 446-455.

Which of the following is appropriate next step in


management? Question 21
A 40-year-old man with a history of left-sided
A. Continue methotrexate at the current dose ulcerative colitis was in remission on maintenance
and start anti-TNF therapy infliximab 5mg/kg every eight weeks as mono-
Chapter 13 — Inflammatory bowel disease 373

therapy for the last year. Prior to infliximab, he the Role of Therapeutic Drug Monitoring in the
failed mesalamine and azathioprine therapies. He Management of Inflammatory Bowel Diseases.
now presents with bloody diarrhea and abdominal Gastroenterology, 2017. 153(3): p. 835-857.e6.
pain for the past two weeks. Laboratory assess-
ment reveals microcytic anemia with elevated
C-reactive protein. Stool studies are negative for Question 22
any infectious process. Therapeutic drug moni- A 26-year-old female recently diagnosed with
toring reveals a trough infliximab level of 15mcg/ ileocolonic Crohn’s disease just completed induc-
ml with no detectable antibodies to infliximab. tion of adalimumab therapy and received her first
Ileocolonoscopy reveals a normal terminal ileum maintenance dose three days ago. She presents for
and moderate to severe inflammatory changes an unscheduled clinic visit for evaluation of a dif-
involving the entire colonic mucosa. You start a fuse scaly rash involving her abdomen, back and
prednisone taper at 40 mg daily with good clinical upper extremities that began the night before.
response. Which of the following is most appropri-
ate next step in management?

A. Reduce the frequency of infliximab infusions to


every four weeks
B. Switch to adalimumab monotherapy
C.Switch to vedolizumab 300 mg at weeks 0, two,
six and then every eight weeks
D. Add methotrexate, 25 mg IM injections weekly
E. Continue the current dose of infliximab and
monitor symptoms while tapering the prednisone

CORRECT ANSWER: C
You suspect drug-induced psoriasis. Which of the fol-
RATIONALE lowing statements is true regarding this condition?
This patient has moderate to severe disease
activity despite adequate trough infliximab levels A. This condition is seen more frequently in men
(more than 5mcg/ml). This suggests a mechanis- B. It only occurs in ulcerative colitis patients, not
tic failure of infliximab and therefore switching to those with Crohn’s disease
a drug of different class is recommended. Con- C. Anti-TNF should be stopped immediately
tinuing the same dose of infliximab, reducing the D. Switching to a different anti-TNF agent is
infusions intervals, or switching to adalimumab, associated with recurrence of skin lesions in
another TNF-inhibitor, are not appropriate op- 90-100 percent of patients
tions. Methotrexate has not been shown to be E. It most commonly involves the palms of
effective in inducing or maintaining remission for hands, soles of feet, and scalp
patients with ulcerative colitis. Vedolizumab is an
anti-integrin agent with proven efficacy in moder- CORRECT ANSWER: E
ate to severe ulcerative colitis and is an appropri-
ate choice for this patient RATIONALE
A wide range of dermatological conditions has
REFERENCE been reported in association with anti-TNF thera-
Vande Casteele, N., et al., American Gastroentero- py, including psoriasis, ezcema, vitiligo, lichenoid
logical Association Institute Technical Review on reactions, vasculitis, erythema nodosum, and
374 Digestive Diseases Self-Education Program®

acneiform eruptions. Anti-TNF-induced psoria- prednisolone therapy was initiated. He is cur-


sis is the most common among these conditions, rently on day three of IV steroids and continues
with estimates of the prevalence of new-onset to report eight to 10 bloody bowel movements per
psoriasis ranging from 0.6 percent to 5.3 per- day with persistently elevated C-reactive protein.
cent. In a recent nationwide cohort study of 7415 You recommend IV cyclosporine salvage therapy.
patients, 125 cases of anti-TNF-induced psoriasis
were reported. Female gender and smoking were Which of the following is associated with an
associated with a higher risk of developing psoria- increased risk for neurological complications with
sis. This can be seen in both CD and UC patients, cyclosporine therapy?
but is slightly more common in those with CD. In
contrast to idiopathic plaque psoriasis that tends A. Albumin of 2.1 g/l
to affect the extensor surfaces of the elbows and B. Hemoglobin of 7.7 mg/dl
knees, palmoplanter and scalp distibution are C. More than 10 lb weight loss prior to admission
more frequently seen with anti-TNF-induced pso- D. Serum cholesterol level of 52 mg/dl
riasis. Although withdrawal of the anti-TNF agent E. Serum magnesium level of 2.3 mEq/L
usually results in rapid resolution of the rash, ces-
sation of therapy is frequently not required, as the CORRECT ANSWER: D
psoriasis improves in the majority of patients with
use of topical therapies (corticosteroid, keratolyt- RATIONALE
ics, emollients, or ultraviolet lights). Switching Cyclosporine is a calcineurin inhibitor that sup-
to another anti-TNF agent can be attempted, presses inflammation by inhibiting the production
although the risk of recurrence can be as high as of IL2 by activated T-cells. It has been shown to
60 percent (not 100 percent). Ustekinumab is an be an effective salvage therapy in patients with
IL12/IL23 inhibitor with proven efficacy in both acute severe ulcerative colitis refractory to IV
psoriasis and CD; hence, it can be considered steroids. The usual recommended dose is 2mg/
as an option in more severe cases of anti-TNF- kg/day, which was found to be as effective and less
induced psoriasis. toxic than the 4mg/kg/day dosing that was used
in the initial clinical trials. Nevertheless, even at
REFERENCES the lower dose, cyclosporine can cause serious
Mocci, G., et al., Dermatological adverse reactions adverse effects including electrolyte abnormalities,
during anti-TNF treatments: focus on inflamma- nephrotoxicity, hypertension, infections, and neu-
tory bowel disease. J Crohns Colitis, 2013. 7(10): rotoxicity. In the case of neurotoxicity, patients
p. 769-79. can experience paresthesia, tremors, and in more
Guerra, I., et al., Incidence, Clinical Charac- severe cases, seizures. The risk of neurotoxicity
teristics, and Management of Psoriasis Induced and seizures was found to be highest for patients
by Anti-TNF Therapy in Patients with Inflamma- with a low serum cholesterol (less than 100mg/
tory Bowel Disease: A Nationwide Cohort Study. dl). One possible explanation for this is that cyclo-
Inflamm Bowel Dis, 2016. 22(4): p. 894-901. sporine is a very lipophilic drug that exists largely
in a bound state. In patients with a low serum
cholesterol level, the proportion of free/unbound
Question 23 cyclosporine is increased, allowing more drug to
A 32-year-old man with a history of pan-ulcerative cross the blood brain barrier and cause toxicity.
colitis on oral mesalamine for the past five years is In addition to low cholesterol level, hypomagne-
now hospitalized with severe bloody diarrhea, up semia, hypertension, and concomitant IV steroid
to 15 bowel movements per day. Infections were therapy are also associated with an increase in the
ruled out on admission and high dose IV methyl- risk of neurotoxicity.
Chapter 13 — Inflammatory bowel disease 375

REFERENCES various inflammatory cytokines and chemokines.


Lichtiger, S., et al., Cyclosporine in severe ulcer- Despite its proinflammatory effects, blocking IL17
ative colitis refractory to steroid therapy. N Engl J did not result in amelioration of colitis in experi-
Med, 1994. 330(26): p. 1841-5. mental mice. Furthermore, the anti-IL17 drug,
Van Assche, G., et al., Randomized, double- secukinumab, currently used in patients with pso-
blind comparison of 4 mg/kg versus 2 mg/kg in- riatic arthritis and ankylosing spondylitis caused
travenous cyclosporine in severe ulcerative colitis. aggravation of Crohn’s disease in some cases, and
Gastroenterology, 2003. 125(4): p. 1025-31. the RCT evaluating its use in Crohn’s disease had
Hindryckx, P., V. Jairath, and G. D’Haens, to be stopped early due to worsening disease.
Acute severe ulcerative colitis: from pathophysiol-
ogy to clinical management. Nat Rev Gastroen- REFERENCES
terol Hepatol, 2016. 13(11): p. 654-664. Hundorfean, G., M.F. Neurath, and J. Mudter,
Functional relevance of T helper 17 (Th17) cells and
the IL-17 cytokine family in inflammatory bowel
Question 24 disease. Inflamm Bowel Dis, 2012. 18(1): p. 180-6.
IBD therapies have expanded in recent years
with the development of agents targeting specific
components of the inflammatory cascade. Which Question 25
of the following is NOT a therapeutic target of a A 35-year-old woman with recently diagnosed
currently approved IBD therapy? moderate ulcerative colitis has been on mesala-
mine 4.8gm/day and mesalamine enemas nightly
A. Tumor necrosis factor alpha for the past three months. She continues to have
B. Interleukins 12 and 23 (IL12/IL23) six loose bowel movements per day with tenes-
C. a4b7 integrin mus and intermittent hematochezia. Her flex-
D. Interleukin 17 (IL17) ible sigmoidoscopy reveals Mayo grade 2 colitis
E. Janus Kinases 1-3 throughout the extent of the exam, and biopsies
are negative for CMV.
CORRECT ANSWER: D
Labs reveal:
RATIONALE Negative C.diff stool test
All the listed options can be targeted by the cur- Hemoglobin of 9 g/dL, MCV of 76 fl, platelet
rently available IBD therapies except for IL17 count 480K/ml
(option D). T helper 17 (Th17) and its product, Thiopurine methyltransferase activity level:
IL17 are increasingly recognized as important 4 U/mL (normal 17-44 U/mL)
players in the pathophysiology of IBD. IL17 is a
potent proinflammatory cytokine that facilitates Which of the following is the most appropriate
T cell activation with subsequent production of next step in the management of this patient?

Therapy Class Target Drugs


Anti-TNF agents TNF-a Infliximab, adalimumab,
certolizumab and golimumab
Anti-integrins a4b7/a4b1 Natalizumab
a4b7 Vedolizumab
IL12/IL23 inhibitor Blocks P40, a common subunit of Ustekinumab
IL12/IL23
Janus kinase (JAK) inhibitors JAK1-3 Tofacitinib
376 Digestive Diseases Self-Education Program®

A. Start azathioprine at 2.5mg/kg daily cy and 6-mercaptopurine toxicity during treat-


B. Start azathioprine at 1mg/kg daily ment for acute lymphoblastic leukaemia. Archives
C. Start certolizumab pegol induction of disease in childhood. 1993;69(5):577-579.
D. Start vedolizumab induction Lennard L, Van Loon JA, Weinshilboum RM.
E. Start 6-mercaptopurine at 1 mg/kg daily Pharmacogenetics of acute azathioprine toxic-
ity: relationship to thiopurine methyltransferase
CORRECT ANSWER: D genetic polymorphism. Clinical pharmacology and
therapeutics. 1989;46(2):149-154.
RATIONALE Kornbluth, A. and D.B. Sachar, Ulcerative
This patient has failed mesalamine therapy (both colitis practice guidelines in adults: American
oral and rectal), and therefore requires step up College Of Gastroenterology, Practice Parameters
therapy. Immunomodulator is a consideration, Committee. Am J Gastroenterol, 2010. 105(3): p.
but it is recommended to first check TPMT en- 501-23; quiz 524.
zyme activity. Azathioprine and 6-MP are effective
for maintenance, but are not effective in induction
of remission, largely related to the prolonged time Question 26
to onset of the drug’s activity. However, it would A 48-year-old woman with extensive ulcerative
be reasonable to consider starting an immuno- colitis has been unable to taper off of oral steroids.
modulator concurrently with corticosteroids as the She has been hesitant to initiate azathioprine
induction agent. There is a population polymor- therapy. You counsel her on the risks and benefits
phism in the TPMT gene, with 89 percent of the of this therapy.
population homozygous for wild-type TPMT, 11
percent heterozygous for the TPMT mutation, and Which of the following is a known complication of
0.3 percent homozygous for the TPMT mutation. azathioprine therapy?
Those with heterozygous and homozygous TPMT
mutations have decreased-to-absent enzyme A. Bone marrow suppression
activity and are at higher risk of leukopenia when B. Increased relative risk of lymphoproliferative
treated with azathioprine and 6-MP. The target disorder
dose used to treat IBD patients who have normal C. Increased relative risk of non-melanoma skin
TPMT activity is 2–3 mg/kg/day for azathioprine cancer
and 1.0–1.5 mg/kg/day for 6-MP; patients with D. Increased risk of herpes zoster
intermediate TPMT activity should be given half of E. All of the above
these doses, and patients with low-absent TPMT
activity should avoid these drugs altogether given CORRECT ANSWER: E
the risk of severe leukopenia, sepsis, and death.
This patient has almost absent enzyme activity, RATIONALE
and thus cannot be started on an immunomodu- Thiopurines have been associated with an increased
latory agent. Certolizumab is an anti-TNF agent risk of lymphoproliferative disorder with continued
approved for use in Crohn’s disease only, not UC. use in patients with IBD, although this risk returns
Either an anti-TNF (infliximab, adalimumab or to baseline after discontinuation of these therapies.
golimumab) or an anti-integrin (vedolizumab) The absolute risk remains low (2/10,000 patient-
would be appropriate in this setting. years in unexposed as compared to 4/10,000
patient-years in exposed). Bone marrow suppres-
REFERENCES sion can occur with thiopurine therapy. Severe
Lennard L, Gibson BE, Nicole T, Lilleyman JS. myelotoxicity is estimated at approximately four
Congenital thiopurine methyltransferase deficien- percent. Measurement of TPMT prior to initiation
Chapter 13 — Inflammatory bowel disease 377

of a thiopurine is recommended in order to iden- C.Varicella vaccination (Varivax®)


tify those at highest risk for marrow suppression. D. Zoster vaccination (Shingrix®)
In a retrospective cohort in a US administrative E. None of the above
dataset, thiopurines have been associated with
an increased risk of herpes zoster (OR: 1.85, 95 CORRECT ANSWER: D
percent CI: 1.61-2.13). The risk of non-melanoma
skin cancer (NMSC) is doubled with thiopurine use. RATIONALE
The increased NMSC risk is thought to be related to As the patient is on a biologic agent, live vaccines
increased photosensitivity. are contraindicated. Intranasal influenza, yellow
fever and varicella vaccine are all live vaccines.
REFERENCES There are two commercially available vaccines
Beaugerie L, Brousse N, Bouvier AM, et al. Lym- for zoster on the market, including Zostavax® and
phoproliferative disorders in patients receiving Shingrix®. The Shingrix® vaccine is an inactivated
thiopurines for inflammatory bowel disease: a vaccine and is now the preferred option, given its
prospective observational cohort study. Lancet superior efficacy and durability when compared
2009;374:1617-25. to Zostavax®. As of yet, the Shingrix® vaccine
Goldberg R, Irving PM. Toxicity and response has not been studied specifically in older adults
to thiopurines in patients with inflammatory who are immunocompromised or who are
bowel disease. Expert Rev Gastroenterol Hepatol taking immunosuppressive drugs. However,
2015;9:891-900. it is an inactivated vaccine. Importantly,
Coenen MJ, de Jong DJ, van Marrewijk CJ, tofacitinib is associated with an increased risk
et al. Identification of Patients With Variants in of herpes zoster and therefore zoster vaccine
TPMT and Dose Reduction Reduces Hematologic is recommended in this population. Current
Events During Thiopurine Treatment of Inflam- vaccination recommendations for patients with
matory Bowel Disease. Gastroenterology 2015. IBD emphasize age-appropriate vaccines, with
Long MD, Martin C, Sandler RS, et al. Increased particular emphasis on obtaining influenza
risk of herpes zoster among 108 604 patients with vaccine, pneumococcal vaccination, zoster
inflammatory bowel disease. Aliment Pharmacol vaccination and human papilloma vaccination (in
Ther 2013;37:420-9. appropriate age groups), to prevent complications.
Long MD, Martin CF, Pipkin CA, et al. Risk of
melanoma and nonmelanoma skin cancer among REFERENCES
patients with inflammatory bowel disease. Gastro- Long MD, Gulati A, Wohl D, et al. Immunizations
enterology 2012;143:390-399.e1. in Pediatric and Adult Patients with Inflammatory
Bowel Disease: A Practical Case-based Approach.
Inflamm Bowel Dis 2015;21:1993-2003.
Question 27 Farraye FA, Melmed GY, Lichtenstein GR,
A 51-year-old female with ulcerative colitis is in et al. ACG Clinical Guideline: Preventive Care in
clinical remission on tofacitinib monotherapy. She Inflammatory Bowel Disease. Am J Gastroenterol
asks about appropriate vaccinations. To better in- 2017;112:241-258.
form her on vaccination strategies, you discuss the
difference between live and inactivated vaccines.
Which of the following vaccinations is an inacti- Question 28
vated vaccine? A 35-year-old man with a two year history of left
sided ulcerative colitis presents to clinic with
A. Intranasal influenza vaccination (FluMist®) increased urgency and frequency of bowel move-
B. Yellow fever vaccination (YF-Vax®) ments. At diagnosis, he was placed on oral and
378 Digestive Diseases Self-Education Program®

topical mesalamine therapy and required an oral change outside of class. This reactive approach
prednisone taper. He was unable to fully taper off to therapeutic drug monitoring (TDM) is sup-
of prednisone and was initiated on infliximab 5 ported by the recent American Gastroenterological
mg/kg monotherapy. He did well initially and was Association technical review on TDM. The goal
able to taper off of prednisone. However, he now infliximab trough level is not entirely clear, and
has recurrent symptoms. He is negative for C. dif- studies are ongoing, but is currently suggested to
ficile and undergoes a flexible sigmoidoscopy with be 5µg/mL or more, though patients with fistuliz-
the following findings in the recto-sigmoid colon: ing Crohn’s disease may require levels 15µg/mL
or more. The patient has already tested negative
for C. difficile and repeat testing is not indicated.
If there is no evidence of viral cytopathic effect on
biopsies, there is no role for serum CMV markers.
Although vedolizumab is an option for induction
and maintenance of moderate to severe UC, the
need to change therapies would be better in-
formed by the results of the infliximab drug level/
antibody assay, as dose intensification of inflix-
imab, if appropriate, might allow the patient to
recapture response from this agent. Azathioprine
is slow in onset of action, and given the severity of
the inflammation on endoscopy, this would not be
Biopsies from the colon show moderate chronic the most appropriate next step.
active colitis with no viral cytopathic effect.
Which of the following is the next best step in his REFERENCES
management? Vande Casteele N, Herfarth H, Katz J, et al. Amer-
ican Gastroenterological Association Institute
A. Repeat C. difficile assay Technical Review on the Role of Therapeutic Drug
B. Check serum CMV IgM assay Monitoring in the Management of Inflammatory
C. Check Infliximab level/antibody Bowel Diseases. Gastroenterology 2017;153:835-
D. Change to vedolizumab 857.e6.
E. Add azathioprine Feagan BG, Rutgeerts P, Sands BE, et al. Ve-
dolizumab as induction and maintenance therapy
CORRECT ANSWER: C
for ulcerative colitis. N Engl J Med 2013;369:699-
710.
RATIONALE
This patient has moderate to severe ulcerative
colitis (UC), and has relapsed while on infliximab Question 29
monotherapy. A serum infliximab level/anti- A 28-year-old man with Crohn’s disease pres-
body will determine whether he has an adequate ents with abdominal pain, distension and
level of infliximab and whether he has developed increased bowel movement frequency, and is
anti-drug antibodies. If his drug level is low and found to have a severe ileal stenosis. He sub-
he does not have anti-drug antibodies, he would sequently undergoes ileocecal resection with
benefit from dose intensification. If he has a low direct anastomosis. Prior to surgery, he had
drug level in the presence of high antibodies to inf- been treated with recurrent courses of cortico-
liximab, he might benefit from within class change steroids in addition to azathioprine 2.5 mg/kg/
in therapy (e.g., to adalimumab). If he has good day. In the post-operative time period, you plan
infliximab levels , he would likely benefit from a to initiate an anti-TNF agent.
Chapter 13 — Inflammatory bowel disease 379

Which of the following is the most evidence-based fracture and will require surgery.
next step in monitoring this patient? When you see her back in clinic six months
after surgery, you review the results of a DEXA scan
A. Fecal calprotectin every six months ordered by her primary care physician. The scan
B. CRP every six months demonstrated osteoporosis of the spine. Which of
C.Drug levels and antibodies every six months the following factors, if present, is most strongly
D. Colonoscopy at six months associated with osteoporosis in a patient with IBD?
E. CT scan at six months
A. Age more than 30 years
CORRECT ANSWER: D B. Female gender
C. Disease duration of more than 10 years
RATIONALE
D. More than three months of steroid use
In the post-operative setting, the POCER (Post-Oper-
E. Smoking
ative Crohn’s Endoscopic Recurrence) trial random-
ized patients to active care (colonoscopy six months
CORRECT ANSWER: D
after resection) vs. standard care (no colonoscopy).
The active arm had significantly less endoscopic RATIONALE
recurrence and greater complete mucosal healing. Risk factors for osteoporosis in the general
Other non-endoscopic methods of monitoring and population include female gender, thin frame,
surveillance do not have the same high level of evi- postmenopausal status, smoking/alcohol use and
dence, including fecal calprotectin, CRP and CT scan. corticosteroid use. In the general population,
No prospective trial of therapeutic drug monitoring the United States Preventive Services Task Force
(TDM) and dose adjustment in the post-operative (USPSTF) recommends screening for osteoporosis
setting has been conducted. In the TAXIT random- in women age 65 years and older and in younger
ized controlled trial of clinical dosing of infliximab women whose fracture risk is greater than that of
vs. dosing based on trough concentration, there was a 65-year-old white woman who has no additional
no difference in the primary endpoint of clinical and risk factors.
biological remission at one year, but all patients were There is no specific age recommendation
optimized to reach a target trough level before being at which to begin screening for osteoporosis in
randomized to dosing based on their clinical features IBD patients. The prevalence of osteoporosis in
or continued dosing based on trough level. IBD patients is approximately 15 percent and is
strongly affected by age. In IBD populations, men
REFERENCES and women are at equal risk. The strongest risk
De Cruz P, Kamm MA, Hamilton AL, et al. Crohn’s factor in the IBD population is corticosteroid use.
disease management after intestinal resection: a Screening is recommended in patients with IBD
randomised trial. Lancet 2015;385:1406-17. who have used oral corticosteroid therapy for lon-
Vande Casteele N, Ferrante M, Van Assche G, ger than three consecutive months at dose of 7.5
et al. Trough concentrations of infliximab guide mg or more of prednisone per day.
dosing for patients with inflammatory bowel dis-
ease. Gastroenterology 2015;148:1320-9.e3. REFERENCES
Targownik LE, Bernstein CN, Leslie WD. Inflam-
matory bowel disease and the risk of osteoporosis
Question 30 and fracture. Maturitas 2013;76:315-9.
A 63-year-old woman with a longstanding history Farraye FA, Melmed GY, Lichtenstein GR,
of ulcerative colitis, currently in remission on me- et al. ACG Clinical Guideline: Preventive Care in
salamine therapy, is admitted to the hospital after Inflammatory Bowel Disease. Am J Gastroenterol
falling at home. She has an intertrochanteric hip 2017;112:241-258.
380 Digestive Diseases Self-Education Program®

Question 31 Question 32
An 18-year-old man presents with a two-month A 23-year-old woman with penetrating colonic
history of diarrhea (four to five bowel movements/ Crohn’s disease with perianal phenotype is now in
day) with urgency and intermittent rectal bleed- clinical and endoscopic remission on combination
ing. He denies any weight loss. He denies any therapy with infliximab and methotrexate. She is
abdominal pain. He is a former cigarette smoker interested in becoming pregnant. She asks about
who quit about six months ago. As part of his her medications. Which of the following would be
work up, he undergoes colonoscopy and is found most appropriate in regard to management of her
to have mild to moderate proctosigmoiditis. Crohn’s disease during pregnancy?

Which of the following treatment regimens would A. Stop methotrexate prior to conception and
be most appropriate at this time in order to induce continue infliximab
and maintain remission? B. Stop both methotrexate and infliximab prior
to conception, use no maintenance medica-
A. Resume smoking tions during pregnancy
B. Oral and topical mesalamine C. Continue both methotrexate and infliximab
C.Budesonide MMX D. Stop methotrexate now and plan to start mer-
D. Oral mesalamine captopurine during her pregnancy
E. Oral corticosteroids E. Start prednisone now and continue during
pregnancy to prevent relapse

CORRECT ANSWER: B
CORRECT ANSWER: A

RATIONALE
RATIONALE
While smoking cessation is associated with the
Methotrexate is contraindicated during pregnancy
development of ulcerative colitis, smoking is not
related to neural tube defects associated with its
a recommended treatment for the disease. In
use. Data have consistently shown that infliximab
left-sided ulcerative colitis, a meta-analysis of
is not associated with congenital abnormalities.
four RCTs using combination treatment with rec-
Additionally, infliximab has not been associated
tal 5-aminosalicylate enemas at a dose of 1 g/day
with other complications in the newborn. Dis-
combined with oral 5-aminosalicylate at a dose
continuation of therapy is associated with a risk
of at least 2.0 g/day was more effective than oral
of relapse of Crohn’s disease. Active disease is
5-ASA alone for induction of remission (relative
associated with increased risk of small for gesta-
risk RR = 0.65; 95 percent CI 0.47-0.91). These
tional age and/or pre-term delivery. Initiation
agents, unlike corticosteroids or budesonide
of mercaptopurine is not recommended during
MMX, can be used for both induction and main-
pregnancy, related to the rare risk of pancreatitis
tenance of remission.
at initiation. However, if a woman is stable on
maintenance mercaptopurine prior to conception,
REFERENCE
it is now recommended that she continue therapy
Ford AC, Khan KJ, Achkar JP, et al. Efficacy of
during pregnancy to prevent disease relapse.
oral vs. topical, or combined oral and topical
5-aminosalicylates, in Ulcerative Colitis: system-
REFERENCES
atic review and meta-analysis. Am J Gastroenterol
Lichtenstein GR, Feagan BG, Mahadevan U, et al.
2012;107:167-76; author reply 177.
Pregnancy Outcomes Reported During the 13-
Year TREAT Registry: A Descriptive Report. Am J
Gastroenterol 2018.
Chapter 13 — Inflammatory bowel disease 381

McConnell RA, Mahadevan U. Pregnancy and A. Azathioprine


the Patient with Inflammatory Bowel Disease: B. Cyclosporine
Fertility, Treatment, Delivery, and Complications. C.Methotrexate
Gastroenterol Clin North Am 2016;45:285-301. D. Ustekinumab
E. Infliximab

Question 33 CORRECT ANSWER: E


Which of the following medications is approved
for induction and remission of both Crohn’s dis- RATIONALE
ease and ulcerative colitis? Of the above agents, only infliximab is approved
for both induction and maintenance of remission
A. Golimumab
in ulcerative colitis. Azathioprine is effective for
B. Certolizumab pegol
maintenance, but is not effective in induction of
C.Tofacitinib
remission, largely related to the prolonged time
D. Vedolizumab
to onset of the drug’s activity. The recent ME-
E. Ustekinumab
TEOR trial failed to show a benefit for induction
CORRECT ANSWER: D of remission for methotrexate in ulcerative colitis.
The MERIT-UC clinical trial showed no beneft for
RATIONALE methotrexate as compared to placebo for mainte-
Of the medications listed, only vedolizumab nance in ulcerative colitis. Ustekinumab is cur-
(alpha-4/beta-7 anti-integrin) is approved for rently approved for induction and maintenance of
use in both Crohn’s disease and ulcerative colitis. Crohn’s disease, not ulcerative colitis.
Golimumab (anti-TNF) and tofacitinib (small
molecule) are only approved for use in ulcer-
REFERENCES
ative colitis. Certolizumab pegol (anti-TNF) and
Paramsothy S, Rosenstein AK, Mehandru S, et al.
ustekinumab (anti IL 12/23) are only approved for
The current state of the art for biological therapies
use in Crohn’s disease.
and new small molecules in inflammatory bowel
disease. Mucosal Immunol 2018.
REFERENCE
Carbonnel F, Colombel JF, Filippi J, et al.
Paramsothy S, Rosenstein AK, Mehandru S, et al.
Methotrexate Is Not Superior to Placebo for
The current state of the art for biological therapies
Inducing Steroid-Free Remission, but Induces
and new small molecules in inflammatory bowel
Steroid-Free Clinical Remission in a Larger
disease. Mucosal Immunol 2018.
Proportion of Patients With Ulcerative Colitis.
Gastroenterology 2016;150:380-8.e4.
Herfarth H, Barnes EL, Valentine JF, et al.
Question 34 Methotrexate Is Not Superior to Placebo in Main-
A 33-year-old man with newly diagnosed panco-
taining Steroid-Free Response or Remission in Ul-
lonic ulcerative colitis has been unable to taper
cerative Colitis. Gastroenterology 2018;155:1098-
off oral corticosteroids despite the addition of oral
1108.e9.
and topical mesalamine at maximum doses. C.
difficile testing is negative. He continues to have
symptoms including frequent bowel movements
Question 35
and rectal bleeding. Which of the following thera-
A 51-year-old man with medically refractory ulcer-
pies would be most appropriate for him in order to
ative colitis was a primary non-responder to both
both induce and maintain remission of his ulcer-
infliximab and vedolizumab. You discuss surgi-
ative coltiis?
cal and medical options with him. He elects to
382 Digestive Diseases Self-Education Program®

proceed with a trial of tofacitinib for his ulcerative therapy. You recommend initiation of an anti-TNF
colitis. Which of the following complications has agent in addition to the azathioprine. Which of
been associated with tofacitinib use? the following preventive measures should also be
recommended?
A. Non-melanoma skin cancer
B. Herpes zoster infection A. DEXA scan
C.Hyperlipidemia B. Pneumococcal vaccine (PPSV-23 and PCV-23)
D. Increases in CPK (creatinine phosphokinase) C. Intranasal influenza vaccination (FluMist®)
E. All of the above D. Human papilloma virus (HPV) vaccine
E. All of the above
CORRECT ANSWER: E
CORRECT ANSWER: B

RATIONALE RATIONALE
Tofacitinib is an orally administered small molecule In patients with chronic conditions such as
that is a non-selective inhibitor of the Janus kinase Crohn’s disease, annual influenza vaccine is
enzyme. The OCTAVE 1 (n=598) and OCTAVE 2 indicated, but should be given by the inactivated
(n=541) induction trials were conducted to as- influenza vaccine. In general, live vaccines should
sess the efficacy of tofacitinib 10 mg orally twice be avoided for at least four weeks prior to initia-
daily compared to placebo. Patients enrolled had tion, and during treatment with biologic therapy.
moderately-to-severely active UC and had failed The newest statement from the Infectious Disease
conventional therapies (half of them had previously Society of America (IDSA) recommends pneumo-
failed anti-TNF agents). The primary endpoint was coccal vaccination with both the PPSV-23 vaccine
remission (total Mayo score of two or less, no sub- and the PCV-23 vaccine. The timing of the vac-
score more than1 and rectal bleeding subscore of cines is based upon which is given first and the
0) at eight weeks. A significantly higher proportion age of the patient. Human papilloma virus vaccine
of patients in both induction trials achieved remis- is recommended by the FDA in all women and
sion with tofacitinib 10 mg twice daily compared men age 11-45 years (note that the age range has
to those who received placebo. While there was no increased). Immunosuppression may increase
difference overall between tofacitinib and placebo the risk for cervical dysplasia from HPV, and thus
groups for serious adverse events, tofacitinib has attention to this recommendation is warranted in
been associated with all of the above complications. women. In those who have been on corticosteroids
The hyperlipidemia is associated with increases in for more than three months, assessment of bone
both LDL and HDL, although the cardiovascular mineral density and maximizing vitamin D levels
implications are not known. is indicated.

REFERENCE REFERENCES
Sandborn WJ, Su C, Sands BE, et al. Tofacitinib as Long MD, Gulati A, Wohl D, et al. Immunizations
induction and maintenance therapy for ulcerative in Pediatric and Adult Patients with Inflammatory
colitis. N Engl J Med 2017;376:1723-1736. Bowel Disease: A Practical Case-based Approach.
Inflamm Bowel Dis 2015;21:1993-2003.
Kane S, Khatibi B, Reddy D. Higher inci-
Question 36 dence of abnormal Pap smears in women with
A 47-year-old man with colonic inflammatory inflammatory bowel disease. Am J Gastroenterol
Crohn’s disease has required two courses of 2008;103:631-6.
corticosteroids (totaling less than three months Kornbluth A, Sachar DB. Ulcerative colitis
of use) over the past year, in spite of azathioprine practice guidelines in adults: American College Of
Chapter 13 — Inflammatory bowel disease 383

Gastroenterology, Practice Parameters Committee. 0.92 and specificity was 0.94-0.97). Treatment for
Am J Gastroenterol 2010;105:501-23; quiz 524. CDI consists of oral vancomycin or fidaxomicin, or
Farraye FA, Melmed GY, Lichtenstein GR, fecal microbial transplantation (FMT) in refracto-
et al. ACG Clinical Guideline: Preventive Care in ry cases. For severe CDI in the general population,
Inflammatory Bowel Disease. Am J Gastroenterol the success rate of vancomycin is 78.5 percent,
2017;112:241-258. fidaxomicin success is similar to that of vanco-
mycin but with lower recurrence rates, and for
FMT is 83-94 percent for recurrent CDI. Recent
Question 37 guidelines from the Infectious Disease Society of
A 23-year-old woman with a three-year history
American no longer recommend metronidazole as
of ulcerative rectosigmoiditis has been in clinical
first line therapy for C. difficile.
and endoscopic remission for two years on oral
mesalamine 2.4 g daily and mesalamine enemas
REFERENCES
nightly. She presents to the clinic with new-onset
Reddy SS, Brandt LJ. Clostridium difficile infec-
watery, non-bloody diarrhea for two weeks. She
tion and inflammatory bowel disease. J Clin Gas-
denies any recent travel or sick contacts. Her
troenterol 2013;47:666-71.
examination is unremarkable. What is the next
Berg AM, Kelly CP, Farraye FA. Clostridium
best step?
difficile infection in the inflammatory bowel dis-
A. Treat with course of ciprofloxacin ease patient. Inflamm Bowel Dis 2013;19:194-204.
B. Initiate oral corticosteroids Bagdasarian N, Rao K, Malani PN. Diagnosis
C. Obtain a stool sample for and treatment of Clostridium difficile in adults: a
Clostridium difficile PCR systematic review. Jama 2015;313:398-408.
D. Perform a colonoscopy
E. Recommend fecal transplant
Question 38
A 66-year-old woman presents to your office with
CORRECT ANSWER: C
a two-month history of recurrent, watery, non-
bloody diarrhea. She has recently been diagnosed
RATIONALE
with osteoarthritis and had been taking ibupro-
IBD is an independent risk factor for the de-
fen 3 times daily for knee pain. Stool studies are
velopment of C. difficile infection (CDI), with a
obtained and are negative for infection, but are
three-fold increased risk compared to the general
positive for fecal leukocytes.
population. The traditional risk factors for CDI are
antibiotic exposure, hospitalization, older age, and What is the most likely diagnosis?
co-morbid conditions. However, IBD patients with
CDI are often younger, have community acquisi- A. Ulcerative colitis
tion, ongoing corticosteroid use, colonic involve- B. Crohn’s disease
ment with IBD, and may have no history of recent C. Microscopic colitis
antibiotic exposure. Corticosteroid use, with or D. Celiac Disease
without additional immunosuppressive agents, E. Small intestinal bacterial overgrowth
triples the risk of CDI in the IBD population. ,
Multistep algorithms using polymerase chain CORRECT ANSWER: C
reaction (PCR) for the toxin gene(s) or single-step
PCR on liquid stool samples have the best test RATIONALE
performance characteristics for CDI (for multi- The incidence rate of collagenous colitis is approx-
step: sensitivity was 0.68-1.00 and specificity was imately four per 100,000 person-years and the
0.92-1.00; for single step: sensitivity was 0.86- incidence of lymphocytic colitis is five per 100,000
384 Digestive Diseases Self-Education Program®

person-years. The incidence rate increases with Which of the following treatments is indicated at
rising age. Microscopic colitis refers collectively this time?
to both lymphocytic colitis and collagenous colitis.
The peak incidence is between 60 and 70 years, A. Chronic antibiotic use
with a greater risk among women. Colonoscopy B. Mesalamine suppositories
with biopsies is required to make the diagnosis, as C. Anti-TNF agent
endoscopically the colon appears normal. D. Probiotic
NSAIDs are a known trigger for microscopic E. Oral prednisone
colitis, along with proton pump inhibitors,
statins and selective serotonin reuptake inhibi- CORRECT ANSWER: B
tors. There are associations with celiac disease
and thyroid disease, and clinical assessment for RATIONALE
these disorders should be considered once the Historically, approximately 50 percent of patients
diagnosis is made if not done previously. Treat- will have no symptoms of pouchitis after total
ment consists of withdrawal of any potential abdominal colectomy and ileal pouch anal anasto-
offending trigger along with anti-diarrheal agents mosis (IPAA) for ulcerative colitis. The remainder
(e.g., bismuth or loperamide) or budesonide. If will experience pouchitis, with 11 percent having
there is no response to budesonide, other treat- one to two episodes per year, 24 percent having
ment options that have been used include ami- occasional symptoms, 10 percent with constant
nosalicylates and cholestyramine, although less symptoms, and four percent will be refractory.
efficacy data exist for these therapies. This patient has evidence of inflammation of the
cuff (cuffitis), which should be treated with topi-
REFERENCES cal therapies such as mesalamine suppositories.
Tong J, Zheng Q, Zhang C, et al. Incidence, preva- There is no evidence of Crohn’s disease of the
lence, and temporal trends of microscopic colitis: pouch, therefore an anti-TNF agent would not be
a systematic review and meta-analysis. Am J Gas- indicated at this time, nor would steroid therapy.
troenterol 2015;110:265-76; quiz 277. Antibiotics have demonstrated efficacy for treat-
Gentile NM, Khanna S, Loftus EV, Jr., et al. ment of acute pouchitis, particularly ciprofloxacin
The epidemiology of microscopic colitis in Olmsted and metronidazole. The probiotic VSL-3 has been
County from 2002 to 2010: a population-based shown to prevent recurrence of pouchitis in pa-
study. Clin Gastroenterol Hepatol 2014;12:838-42. tients who have recently undergone total procto-
Pardi DS. Microscopic colitis. Clin Geriatr colectomy with IPAA, and for prevention of recur-
Med 2014;30:55-65. rent pouchitis after treatment with antibiotics.

REFERENCES
Question 39 Stahlberg D, Gullberg K, Liljeqvist L, et al. Pouchi-
A 24-year-old man underwent total abdominal tis following pelvic pouch operation for ulcerative
colectomy with ileal pouch anal anastomosis for colitis. Incidence, cumulative risk, and risk fac-
refractory ulcerative colitis 12 months ago. He tors. Dis Colon Rectum 1996;39:1012-8.
now presents with two weeks of diarrhea, gas, Shen B. Acute and chronic pouchitis--patho-
urgency, and intermittent blood in his stool. He genesis, diagnosis and treatment. Nat Rev Gastro-
does not take any medication regularly other enterol Hepatol 2012;9:323-33.
than loperamide. A pouchoscopy is performed
and demonstrates a normal neo-terminal ileum,
normal pouch, but ulceration and erythema in the Question 40
area of the cuff for two to three cm. A 33-year-old woman presents with a six-week
Chapter 13 — Inflammatory bowel disease 385

history of progressively worsening diarrhea, ur- is superior to monotherapy with either agent in
gency and rectal bleeding. She undergoes colonos- ulcerative colitis. Gastroenterology 2014;146:392-
copy as part of her work-up, which shows ulcer- 400.e3.
ative pancolitis. She is started on oral prednisone
in addition to oral and topical mesalamine. She is
unable to taper off of prednisone due to recurrent Question 41
symptoms. You discuss various classes of medica- You start seeing a 19-year-old man with a one-year
tions with her for the treatment of moderate to history of ileal inflammatory Crohn’s disease who
severe ulcerative colitis. is transitioning from pediatric to adult manage-
ment of his Crohn’s. His mother accompanies
Which of the following therapy or therapies has him to the clinic visit. He has been treated with
demonstrated the greatest efficacy to achieve intermittent courses of budesonide for his disease,
corticosteroid-free remission at 16 weeks for mod- but has not been maintained on other therapies.
erate to severe ulcerative colitis? His mother is concerned about the risk of hepa-
tosplenic T-cell lymphoma. Which of the follow-
A. Infliximab alone ing is a known risk factor for hepatosplenic T-cell
B. Azathioprine alone lymphoma?
C. Infliximab and azathioprine combination
D. Infliximab and adalimumab combination A. Ustekinumab
E. Azathioprine and methotrexate combination B. Male gender
C. Vedolizumab
CORRECT ANSWER: C C. Methotrexate
E. Corticosteroids
RATIONALE
Based on the UC Success study, infliximab com- CORRECT ANSWER: B
bined with azathioprine is the best choice. In UC
Success, a total of 239 patients were included in RATIONALE
the efficacy analyses. Baseline characteristics were Hepatosplenic T-cell lymphoma (HSTCL) is a very
similar between treatment groups. Corticosteroid- rare neoplasm, accounting for less than 1 percent
free remission at week 16 was achieved by 39.7 of non-Hodgkin’s lymphoma. Approximately 10-
percent (31 of 78) of patients receiving infliximab/ 20 percent of these HSTCLs are associated with
azathioprine combination compared with 22.1 per- chronic immunosuppression. In patients with
cent (17 of 77) receiving infliximab alone (P =.017) inflammatory bowel disease, there is an associa-
and 23.7 percent (18 of 76) receiving azathio- tion with use of thiopurines, anti-TNF agents, and
prine alone (P =.032). Mucosal healing at week with combination therapy with anti-TNF agents
16 occurred in 62.8 percent (49 of 78) of patients and thiopurines. In a systematic review of HSTCL
receiving infliximab/azathioprine, compared with in patients with IBD, there were a total of 36 pa-
54.6 percent (42 of 77) receiving infliximab (P = tients with HSTCL. Of these, 20 received therapy
.295) and 36.8 percent (28 of 76) receiving aza- with infliximab and a thiopurine and 16 received a
thioprine (P =.001). Serious infections occurred in thiopurine as monotherapy for IBD. Four pa-
two patients (1 patient receiving infliximab alone, tients who had been treated with infliximab and
and 1 patient receiving azathioprine alone). a thiopurine also received adalimumab. One of
these patients had been given infliximab, adalim-
REFERENCE umab, and natalizumab. Of 31 patients of known
Panaccione R, Ghosh S, Middleton S, et al. Com- gender, only two were female. Twenty-seven of the
bination therapy with infliximab and azathioprine 30 patients of known age were younger than 35
386 Digestive Diseases Self-Education Program®

years old. There were no reported cases of HSTCL CORRECT ANSWER: A


in patients with IBD who received only anti-TNF
therapy. RATIONALE
The incidence of TNF-induced psoriasis is 5/100
REFERENCE person-years. Smoking is an independent predic-
Kotlyar DS, Osterman MT, Diamond RH, et al. tor of psoriasis in patients with IBD on anti-TNF
A systematic review of factors that contribute to therapy (HR: 2.37, 95 percent CI: 1.36-4.48;
hepatosplenic T-cell lymphoma in patients with P = 0.008). Conversely, concomitant immuno-
inflammatory bowel disease. Clin Gastroenterol suppressive therapy has been inversely related
Hepatol 2011;9:36-41.e1. to psoriasis (HR: 0.33, 95 percent CI: 0.12-0.92;
P = 0.03). After 14 years of follow up, derma-
tologic complications in patients with IBD on
Question 42 anti-TNF agents can involve one in five patients.
A 45-year-old woman with ileocolonic strictur- TNF-induced psoriasis more often affects women
ing Crohn’s disease without perianal phenotype and those with Crohn’s disease. The IBD is often
on therapy with infliximab 10 mg/kg every eight in clinical remission (88.1 percent) at the onset of
weeks combined with methotrexate 25 mg weekly psoriasis. There is an association between derma-
presents to your office for follow-up of a rash on tologic complications and higher anti-TNF dosing,
her scalp and behind her ears. The rash is very suggesting a dose dependent effect. Discontinua-
bothersome and has not responded to the addition tion of anti-TNF therapy is required in one of five
of methotrexate. Additionally, she has been us- patients with psoriasiform lesions, but specific
ing topical corticosteroids without benefit. She is dermatologic treatment of the lesions often allows
starting to have significant hair loss. She is tearful for continuation of the anti-TNF therapy. Continu-
in clinic today, despite the fact her GI symptoms ation of anti-TNF therapy combined with a topical
are doing well, with two bowel movements per treatment such as a corticosteroid or vitamin D
day and colonoscopy at last evaluation showing analog, and eventually a trial of a systemic treat-
endoscopic remission. ment, should be the first options in management,
as cutaneous improvement is possible. However,
if these therapies are not effective, series have
demonstrated efficacy of ustekinumab for treat-
ment of both the Crohn’s disease and the psoriasi-
form eruption.

REFERENCES
Pugliese D, Guidi L, Ferraro PM, et al. Paradoxical
psoriasis in a large cohort of patients with inflam-
matory bowel disease receiving treatment with
anti-TNF alpha: 5-year follow-up study. Aliment
Pharmacol Ther 2015;42:880-8.
What is the best management of this rash? Freling E, Baumann C, Cuny JF, et al. Cu-
mulative Incidence of, Risk Factors for, and
A. Stop infliximab and start ustekinumab Outcome of Dermatological Complications of
B. Stop methotrexate and start azathioprine Anti-TNF Therapy in Inflammatory Bowel Dis-
D. Increase dose of infliximab ease: A 14-Year Experience. Am J Gastroenterol
E. Increase dose of methotrexate 2015;110:1186-96.
F. Add UV light therapy
Chapter 13 — Inflammatory bowel disease 387

Question 43 to anti-tumor necrosis factor-alpha (anti-TNF)


A 22-year-old male with a two-year history of left- agents, but have not been associated with vedoli-
sided ulcerative colitis has been unable to taper zumab therapy at this time. To date, there has not
off oral corticosteroids. In your discussions with been an association between vedolizumab therapy
him on initiation of a corticosteroid-sparing agent, and lymphoproliferative disorders.
you discuss the risks and benefits of vedolizumab, The overall response rate for vedolizumab for
an anti-integrin therapy. Which of the following the treatment of ulcerative colitis at week six was
adverse events have been associated with vedoli- found to be 47.1 percent, and at week 52, 41.8 per-
zumab use? cent of patients who continued to receive vedoli-
zumab every eight weeks were in clinical remis-
A. Increased serious infection risk sion. When evaluated in clinical practice, response
B. Psoriasiform skin lesions rates have been similar, with similar adverse
C. Progressive multi-focal leukoencephalopathy events in approximately 10.5 percent of patients.
D. Increased risk of lymphoproliferative disorder
E. No important differences in common adverse REFERENCES
events Feagan BG, Rutgeerts P, Sands BE, et al. Vedoli-
zumab as induction and maintenance therapy for
CORRECT ANSWER: E ulcerative colitis. N Engl J Med 2013;369:699-710.
Shelton E, Allegretti JR, Stevens B, et al. Ef-
RATIONALE ficacy of Vedolizumab as Induction Therapy in
Vedolizumab is a humanized monoclonal antibody Refractory IBD Patients: A Multicenter Cohort.
that is an anti-integrin therapy, targeting α4β7, a Inflamm Bowel Dis 2015.
cell-surface glycoprotein variably expressing on
circulating B and T lymphocytes, which interacts
with mucosal vascular addressin cell adhesion Question 44
molecule 1 (MAdCAM-1) on intestinal vasculature. A 43-year-old woman with pancolonic ulcerative
This therapy selectively blocks gut lymphocyte colitis is admitted to the hospital with frequent
trafficking, without interfering with trafficking to bloody diarrhea and abdominal pain. Her medica-
the central nervous system. By contrast, natali- tions at admission include oral mesalamine and an
zumab, a monoclonal antibody with efficacy in oral contraceptive agent. After three days in the
multiple sclerosis and in Crohn’s disease, inhib- hospital on intravenous corticosteroids, her diar-
its both α4β7 and α4β1 integrins and has been rhea and abdominal pain are improving. She devel-
associated with progressive multi-focal leukoen- ops low grade fever, pain with inspiration, mild
cephalopathy (PML). In all individuals treated to tachycardia and sudden onset shortness of breath.
date with vedolizumab, there has been only one
reported case of PML. This case involved a patient Which of the following measures is recommended
who also had HIV and was on other immunosup- at admission to prevent this complication?
pressive medications. The cause of PML was ad-
A. Pneumococcal vaccination
judicated to the HIV by an independent panel and
B. Prophylaxis with trimethoprim/
was not thought to be related to the vedolizumab
sulfamethoxazole
(personal communication, Takeda).
C. Prophylaxis with low molecular weight heparin
In the randomized controlled trials of vedoli-
D. Daily aspirin
zumab, there were no important differences in
E. None of the above
common adverse events. Vedolizumab did not
affect peripheral lymphocytes. Psoriasiform skin
CORRECT ANSWER: C
lesions have been seen as a paradoxical reaction
388 Digestive Diseases Self-Education Program®

RATIONALE 25 cm. Biopsies are negative for CMV. Lab values


Patients with IBD have a two-fold increased risk show a hemoglobin of 12.5 g/dL and a CRP of 6.0
of developing venous thromboembolism (VTE) mg/L (normal: less than 10.0 mg/L) You make plans
compared to the general population. The risk is to initiate infliximab for severe left-sided colitis.
even higher among patients with ulcerative colitis, Which of the following factors is associated with
particularly during a flare. Oral contraceptive accelerated drug clearance of anti-TNF agents in
agents and smoking are also risk factors for devel- the treatment of inflammatory bowel disease?
opment of VTE. For these reasons, DVT prophy-
laxis is recommended in patients with IBD who A. Elevated albumin (more than4.0 g/dL)
are hospitalized. Pharmacologic thromboprophy- B. Low CRP (less than 10 mg/L)
laxis during hospitalization is also associated with C. Increased body mass index
a significantly lower risk of post-hospitalization D. Concomitant use of mercaptopurine
VTE (HR, 0.46; 95 percent CI, 0.22-0.97). Other E. Female gender
forms of prophylaxis noted are not recommended
on admission. CORRECT ANSWER: C

REFERENCES RATIONALE
Kornbluth A, Sachar DB. Ulcerative colitis practice Although the causes of this variability in drug
guidelines in adults: American College Of Gastro- clearance of anti-TNF agents are incompletely un-
enterology, Practice Parameters Committee. Am J derstood, several factors are known to be critical.
Gastroenterol 2010;105:501-23; quiz 524. Specifically, factors associated with more rapid
Yuhara H, Steinmaus C, Corley D, et al. Meta- clearance of anti-TNF agents include: the pres-
analysis: the risk of venous thromboembolism in ence of anti-drug-antibodies, greater degree of
patients with inflammatory bowel disease. Ali- systemic inflammation as evidenced by the serum
ment Pharmacol Ther 2013;37:953-62. C-reactive protein (CRP) (higher levels associated
Nguyen GC, Sam J. Rising prevalence of ve- with increased clearance) or plasma TNF con-
nous thromboembolism and its impact on mortali- centration, serum albumin concentration (lower
ty among hospitalized inflammatory bowel disease levels associated with increased clearance), dis-
patients. Am J Gastroenterol 2008;103:2272-80. ease type (ulcerative colitis greater drug clearance
https://www.gastro.org/practice/quality-initia- than Crohn’s disease), body mass index (increased
tives/IBD_Measures.pdf. BMI associated with increased drug clearance),
Ananthakrishnan AN, Cagan A, Gainer VS, et gender (drug clearance in males greater than that
al. Thromboprophylaxis is associated with reduced in females), and concomitant therapy with im-
post-hospitalization venous thromboembolic events munosuppressive agents have all emerged as key
in patients with inflammatory bowel diseases. Clin determinants of pharmacokinetics.
Gastroenterol Hepatol 2014;12:1905-10. This impacts use of anti-TNF for patients
hospitalized with severe ulcerative colitis, as the
Question 45 therapeutic efficacy may be altered by rapid drug
A 19-year-old obese woman (BMI 35) is hospitalized clearance. Retrospective data have demonstrated
for a flare of ulcerative colitis. She was diagnosed six a reduction in three-month colectomy rate with
months ago, but has been unable to taper off oral higher dosing of infliximab in individuals with
corticosteroids in spite of the addition of mesala- these risk factors. Further prospective studies are
mine (4.8 g/ daily) and mercaptopurine at 1.5 mg/ needed to guide effective dosing strategies.
kg/day. Upon admission to the hospital, stool for
C. difficile is negative, and flexible sigmoidoscopy REFERENCES
shows severe proctosigmoiditis up to approximately Khanna R, Levesque BG, Sandborn WJ, et al.
Chapter 13 — Inflammatory bowel disease 389

Therapeutic Drug Monitoring of TNF Antagonists their blood. To date, there are no clear guidelines
in Inflammatory Bowel Disease. Gastroenterol regarding when to hold anti-TNF medications in
Hepatol (N Y) 2014;10:478-489. pregnant women. It is important to keep in mind
Gibson DJ, Heetun ZS, Redmond CE, et al. An that maintaining disease remission in the mother
accelerated infliximab induction regimen reduces is key, and the timing of holding anti-TNF therapy
the need for early colectomy in patients with acute should be individualized for each patient.
severe ulcerative colitis. Clin Gastroenterol Hepa- Importantly, it does NOT appear that children
tol 2015;13:330-335.e1. born to mothers on anti-TNF agents have a re-
duced response to inactivated vaccines. A total of
179 women completed the vaccine survey within
Question 46 PIANO (Pregnancy and Neonatal Outcomes) in
A 31-year-old woman with ulcerative colitis on IBD Study; 26 were biologic unexposed, 153 were
adalimumab monotherapy completes a success- exposed to a biologic agent). There was no signifi-
ful pregnancy with delivery of a healthy baby boy cant difference in proportions of infants with pro-
at 39 weeks. She was continued on adalimumab tective antibody titers against HiB born to exposed
up until week 36 and plans to restart therapy one mothers (n = 42, 71 percent) vs unexposed moth-
week after delivery. ers (n = 8, 50 percent) (P = .41). There was no dif-
ference in the proportion of infants with protective
Which of the following is recommended with re- antibody titers to tetanus toxoid born to exposed
gard to vaccinations for mother or child? mothers (80 percent) vs unexposed mothers (75
percent) (P = .66). The median concentration
A. All vaccines should be avoided in the first six of infliximab in cord blood did not differ signifi-
months of life for baby cantly between infants with vs without protective
B. No recommended changes to standard vaccine antibody titers to HiB (P = .30) or tetanus toxoid
schedule for mother or baby (P = .93). Mild reactions were observed in 7 of 40
C. Avoid hepatitis B vaccine in baby for first six infants who received rotavirus vaccine and whose
months (typically given prior to discharge) mothers had been exposed to biologic therapies.
D. Mother should avoid TDaP* vaccine (typically While reactions to the live rotavirus vaccine were
updated prior to discharge) mild, it is recommended that this vaccination be
E. Rotavirus vaccine (typically given at four postponed or avoided until anti-TNF clearance is
months) should be postponed or avoided obtained or six months has passed.
*Tetanus, Diphtheria, Pertussis
REFERENCES
CORRECT ANSWER: E Hashash JG, Kane S. Pregnancy and Inflamma-
tory Bowel Disease. Gastroenterol Hepatol (N Y)
RATIONALE 2015;11:96-102.
It is important to make sure that children born to Beaulieu DB, Ananthakrishnan AN, Martin C,
mothers who have received infliximab, adalimum- et al. Use of Biologic Therapy by Pregnant Women
ab, or golimumab after the 20th week of gestation With Inflammatory Bowel Disease Does Not Affect
do not receive any live vaccinations during their Infant Response to Vaccines. Clin Gastroenterol
first six months of life. These newborns are im- Hepatol 2018;16:99-105.
munosuppressed because the anti-TNF agent has
crossed the placenta into their blood. Newborns
have an immature reticuloendothelial system and Question 47
are unable to clear antibodies, and infliximab, A 32-year-old man is diagnosed with primary scle-
adalimumab, or golimumab will be detected in rosing cholangitis due to elevated alkaline phos-
390 Digestive Diseases Self-Education Program®

phatase and MRI/MRCP consistent with small edema, and is diagnosed with class III heart fail-
duct disease. He also has a history of loose stools ure during a hospitalization. He is diuresed during
for the past three to four years. He has not seen the hospitalization and has marked improvement
any blood in his stool and his weight has been sta- in respiratory status and lower extremity edema.
ble. He undergoes colonoscopy and is diagnosed As he still has active Crohn’s, you need to initiate a
with mild pan-ulcerative colitis. He has mild back- steroid-sparing therapy for his disease as well.
wash ileitis on examination of the terminal ileum.
He is placed on oral mesalamine for therapy. Which of the following agents would be contra-
Which of the following is your recommendation indicated for treatment of Crohn’s disease at this
for surveillance colonoscopy in this patient? time due to his class III heart failure?

A. Colonoscopy every one to two years after 10 A. Mercaptopurine


years of ulcerative colitis B. Ustekinumab
B. Annual colonoscopy after 10 years of ulcer- C.Vedolizumab
ative colitis D. Infliximab
C. Annual colonoscopy starting now, at diagnosis E. Methotrexate
of ulcerative colitis
CORRECT ANSWER: D
D. Colonoscopy every five years after 10 years of
ulcerative colitis
RATIONALE
E. Colonoscopy every five years starting at diag-
Anti-TNF agents (such as infliximab) may be asso-
nosis of ulcerative colitis
ciated with heart failure. The concern stems from
CORRECT ANSWER: C randomized controlled trials of TNF-alpha inhibi-
tors as a potential therapy for heart failure (AT-
RATIONALE TACH). During this trial, when used in class III
Primary sclerosing cholangitis (PSC) is a chronic or IV heart failure, infliximab was associated with
inflammatory disease that causes fibrosis of the a 16 percent death rate compared with 8 percent
biliary tree. It is closely associated with inflamma- for placebo. In early post-marketing surveillance
tory bowel disease, particularly ulcerative colitis, data gathered by the FDA, there have been cases
which coexists in approximately three-quarters of of heart failure associated with use of anti-TNF
northern European patients. Colorectal carcinoma agents, including new onset as well as heart failure
screening should be performed after the diagnosis exacerbation. Other drug classes have not been
of PSC in all patients, and surveillance colonos- associated with this complication.
copy should be performed annually if there is
concomitant colitis. REFERENCES
Sinagra E, Perricone G, Romano C, et al. Heart
REFERENCE failure and anti tumor necrosis factor-alpha in sys-
Kitiyakara T, Chapman RW. Chemoprevention temic chronic inflammatory diseases. Eur J Intern
and screening in primary sclerosing cholangitis. Med 2013;24:385-92.
Postgrad Med J 2008;84:228-37. Chung ES, Packer M, Lo KH, et al. Random-
ized, double-blind, placebo-controlled, pilot trial
of infliximab, a chimeric monoclonal antibody
Question 48 to tumor necrosis factor-alpha, in patients with
A 55-year-old man with underlying inflammatory moderate-to-severe heart failure: results of the an-
Crohn’s ileocolitis has recently had a flare of dis- ti-TNF Therapy Against Congestive Heart Failure
ease and has been on corticosteroids. He presents (ATTACH) trial. Circulation 2003;107:3133-40.
with shortness of breath and lower extremity Kwon HJ, Cote TR, Cuffe MS, et al. Case
Chapter 13 — Inflammatory bowel disease 391

reports of heart failure after therapy with a tu- analysis was 14 (95 percent CI, 5-34) cancers/100
mor necrosis factor antagonist. Ann Intern Med years patient follow-up. A 2016 systematic review
2003;138:807-11. and meta-analysis of 10 randomized trials found
an increased likelihood of detecting dysplasia with
chromoendoscopy compared with other tech-
Question 49 niques (RR 1.37, 95 percent CI 1.04-1.79), but on
A 49-year-old man with a 20-year history of ulcer- subgroup analysis this effect was confirmed only
ative colitis undergoes his biannual surveillance for chromoendoscopy compared with standard
colonoscopy. He has been maintained on oral white light endoscopy (RR 2.12, 95 percent CI
5-ASA and has required intermittent courses of 1.15-3.91)
prednisone during the course of his disease. More The SCENIC international consensus panel re-
recently, he has been in endoscopic and clinical viewed the available evidence and recommended
remission. His colonoscopy under high-definition the following: For patients with endoscopically
white light is unremarkable. Biopsies are obtained invisible dysplasia (confirmed by a second GI pa-
and he has evidence of flat (non-polypoid) low thologist) referral to an endoscopist with expertise
grade dysplasia in his sigmoid colon. Biopsy re- in IBD surveillance by using chromoendoscopy
sults are confirmed by a second GI pathologist. with high definition colonoscopy is suggested.
There were no studies comparing surveillance
What is the next best step in his management? colonoscopy to colectomy in this group with en-
doscopically invisible dysplasia. However, should
A. Repeat high-definition colonoscopy at one a patient elect to proceed with colectomy, a total
year colectomy would be indicated rather than a par-
B. Repeat high-definition colonoscopy at two tial colectomy.
years
C. Referral to colorectal surgery for sigmoid REFERENCES
colectomy Laine L, Kaltenbach T, Barkun A, et al. SCENIC
D. Referral to colorectal surgery for total international consensus statement on surveillance
proctcolectomy and management of dysplasia in inflammatory
E. Repeat colonoscopy with chromoendoscopy bowel disease. Gastroenterology 2015;148:639-
651.e28.
CORRECT ANSWER: E Iannone A, Ruospo M, Wong G, et al. Chro-
moendoscopy for Surveillance in Ulcerative Colitis
RATIONALE and Crohn’s Disease: A Systematic Review of
“Flat dysplasia” is defined as endoscopically invis- Randomized Trials. Clin Gastroenterol Hepatol
ible dysplasia. One meta-analysis on the cancer 2017;15:1684-1697.e11.
risk of low-grade dysplasia in chronic ulcerative
colitis included 20 surveillance studies totaling
Question 50
508 flat low grade dysplasia or low grade dysplasia
Which of the following factors is associated with
with dysplasia-associated lesions or masses. The
an increased risk of colectomy in patients with
studies predated the use of current technology
ulcerative colitis?
with image enhanced endoscopy or even high res-
olution endoscopy. The authors reported a 9-fold A. Age less than 40 years
risk of developing cancer (odds ratio [OR] 9.0, 95 B. Extensive colitis
percent CI, 4.0-20.5) and a 12-fold risk of develop- C. Deep ulcerations
ing any advanced lesion (OR 11.9, 95 percent CI, D. High CRP and ESR
5.2-27). The absolute risk of cancer in this meta- E. All of the above
392 Digestive Diseases Self-Education Program®

CORRECT ANSWER: E creased risks of cervical cancer. Age-appropriate


screening with pap smears is important for
RATIONALE women diagnosed with IBD regardless of treat-
The AGA Ulcerative Colitis (UC) clinical decision ment type.
support tool moves beyond the standard assess-
ment of UC activity (as mild, moderate, or severe) REFERENCE
at one time point. One time point is felt to be Rungoe et al. Inflammatory bowel disease and
insufficient to guide the selection of therapy, and cervical neoplasia: a population-based nation-
disease risk (defined as the risk of colectomy) wide cohort study. Clin Gastroenterol Hepatol.
needs to be considered as well. Current models 2015;13(4):693-700.
include demographic and disease characteristics
as predictors of colectomy. All of the above fac-
tors, along with a history of Clostridium difficile Question 52
or Cytomegalovirus infection, are predictors of A 21-year-old patient with Crohn’s disease of the
intermediate outcomes, such as hospitalization or ileum and colon is feeling well on ustekinumab
need for biologics, and ultimately colectomy. and returns for follow-up. Recent colonoscopy re-
vealed mucosal healing. She has received pneumo-
REFERENCE nia and influenza vaccination. She notices that she
Dassopoulos T, Cohen RD, Scherl EJ, et al. Ul- does have a few days of diarrhea during the time
cerative Colitis Care Pathway. Gastroenterology of her menstrual cycle, and wonders if this indi-
2015;149:238-45. cates continued active disease. She is interested in
contraception, and would like to know if she can
take hormonal oral contraception.
Question 51
A 25-year-old woman with colonic Crohn’s disease
Which of the following is true?
presents for routine follow-up. She is in remission
on her regimen of vedolizumab. When discussing
A. Symptoms are not expected to change around
her medication regimen, she asks about the long-
the menstrual cycle, and she needs another
term risks associated with her Crohn’s disease
colonoscopy to evaluate
and treatment. Which of the following is she at
B. She should not get the HPV vaccination due to
increased risk for because of her diagnosis?
her biologic use
A. Breast adenomas C. She can follow the average risk guideline of
B. Endometriosis cervical cancer screening every three years
C.Vaginal atrophy D. She cannot use oral contraceptives because
D. Ovarian cysts they may cause a flare of her disease
E. Cervical dysplasia E. She can use oral contraception, but should be
aware of venous thromboembolism risk
CORRECT ANSWER: E
CORRECT ANSWER: E
RATIONALE
In a nationwide cohort study, women with Crohn’s RATIONALE
disease (CD) and ulcerative colitis (UC) were Patients often report bowel symptoms around the
found to have an increased risk of cervical dys- time of their menstrual cycle. IBD patients are at
plasia. UC patients had increased risks of low higher risk for cervical dysplasia, and should have
and high grade squamous intraepithelial lesions, the HPV vaccine. IBD patients should have yearly
whereas Crohn’s disease patients also had in- cervical cancer screening due to increased risk for
Chapter 13 — Inflammatory bowel disease 393

cervical dysplasia and chronic use of immunosup-


pression. Oral contraception is often helpful in al-
leviating symptoms associated with the menstrual
cycle, and should not flare disease. Oral contra-
ception can be used, but patients should be aware
of VTE risk.

REFERENCES
Saha S, Zhao Y-Q, Shah S, et al. Menstrual Cycle
Changes in Women with Inflammatory Bowel Dis-
ease: A Study from the Ocean State Crohn’s and
Colitis Area Registry. Inflamm Bowel Dis 2014;
20: 534-540.
Gawron LM, Sanders J, Steele KP, and Flynn
AD. Reproductive Planning and Contraception
for Women with Inflammatory Bowel Diseases.
Inflamm Bowel Dis. 2016: 22(2): 459-464.
Rungoe C, Simonsen J, Riis L, et al. Inflam-
matory bowel disease and cervical neoplasia: a
population-based nationwide cohort study. Clin
Gastroenterol Hepatol. 2015; 13(4): 693-700.
394 Digestive Diseases Self-Education Program®
Answers & critiques

CHAPTER 14

Gastrointestinal cancers
Seth D. Crockett, MD MPH, Sonia Kupfer, MD and
Christina Ha, MD

Question 1 of carcinomas associated with lye ingestion are


A 49-year-old woman complains of approxi- located in the proximal esophagus at the area of
mately six month history of GERD symptoms, the bronchial bifurcation. For these reasons, an
characterized by occasional chest discomfort and upper endoscopy with careful mucosal inspec-
regurgitation of food and acidic fluid. She has a tion, including use of chromoendoscopy agent or
history of lye ingestion as a child that was man- narrow band imaging is warranted to assess for
aged medically. She denies excessive alcohol use, mucosal abnormalities.
and is a lifetime nonsmoker. She is not obese.
She has tried over the counter antacids and an REFERENCES
H2 receptor antagonist with incomplete relief of Isolauri J, Markkula H. Lye ingestion and
symptoms. The next best step in management is: carcinoma of the esophagus. Acta Chir Scand.
1989 Apr-May;155(4-5):269-71. PubMed PMID:
A. Medical therapy with a proton pump inhibitor 2800875.
B. Upper endoscopy with endoscopic dilation Appelqvist P, Salmo M. Lye corrosion carci-
C. Upper endoscopy with chromoendoscopy noma of the esophagus: a review of
D. High resolution esophageal manometry 63 cases. Cancer. 1980 May 15;45(10):2655-8.
E. pH impedance testing PubMed PMID: 7378999.
Messmann H. Squamous cell cancer of the
CORRECT ANSWER: C oesophagus. Best Pract Res Clin
Gastroenterol. 2001 Apr;15(2):249-65. Review.
RATIONALE PubMed PMID: 11355914.
Patients with a history of caustic ingestion, par-
ticularly of strong bases, have a markedly higher
risk of squamous cell cancer of the esophagus. Question 2
While the patient in this stem does not have Which of the following is correct regarding the
any of the traditional risk factors for squamous epidemiology of esophageal cancer in the United
cell cancer of the esophagus (tobacco or alcohol States?
abuse), or for adenocarcinoma of the esopha-
gus (male sex, obesity, chronic GERD, Barrett’s A. Small intestinal cancer is more common
esophagus), she does have a history of caustic than esophageal cancer in women.
ingestion as a child, which is thought to increase B. The incidence of esophageal adenocarci-
one’s risk of squamous cell cancer of the esopha- noma has been stable since 1980.
gus 1000-fold. Therefore, surveillance with up- C. Squamous cell carcinoma is more common
per endoscopy is generally recommended begin- than adenocarcinoma.
ning 20 years after the ingestion. The majority D. Tobacco use is a risk factor for adenocarci-

395
396 Digestive Diseases Self-Education Program®

noma, but not squamous cell carcinoma. A. Referral to a thoracic surgeon for an esopha-
E. Esophageal adenocarcinoma is more common gectomy.
in Black men compared to White men. B. Referral to a surgical oncologist for esopha-
gectomy with colon interposition graft.
CORRECT ANSWER: A C. Endoscopic mucosal resection and radiofre-
quency ablation of the Barrett’s segment.
RATIONALE D. Endoscopic submucosal dissection of the en-
The incidence of esophageal adenocarcinoma tire Barrett’s segment.
has increased dramatically over the past several E. Repeat surveillance upper endoscopy in
decades, and is much more common in men one year.
compared to women, such that esophageal cancer
is the least common GI cancer among women. In CORRECT ANSWER: C
developed countries such as the US, esophageal
adenocarcinoma is more common than squamous RATIONALE
cell carcinoma. Tobacco is a risk factor for both The patient in the stem has carcinoma in situ only,
adenocarcinoma and squamous cell carcinoma, with no evidence of deep invasion or lymph node or
but is more strongly associated with squamous cell other metastases. The risk of lymph node metasta-
cancer. Esophageal adenocarcinoma is more com- sis in this scenario is very small, and survival rate
mon in Whites than in Blacks. is excellent. Regardless of age or comorbidities,
the best management of this condition includes
REFERENCES endoscopic mucosal resection (EMR) of any visible
Siegel R, Naishadham D, and Jemal A. Cancer nodularity within the Barrett’s segment, accom-
Statistics, 2012. CA Cancer J Clin 2012;62: panied by radiofrequency ablation. Assuming the
10–29. EMR specimen(s) does not show invasive cancer,
Rubenstein JH, Shaheen NJ. Epidemiology, this procedure is considered curative. Thus, despite
Diagnosis, and Management of the patient’s stated preference for aggressive care,
Esophageal Adenocarcinoma. Gastroenterology. the risks of an esophagectomy in this scenario
2015 Aug;149(2):302-17. (especially with the patient’s comorbidities) would
Hur C, Miller M, Kong CY, Dowling EC, outweigh any perceived benefit. Additionally, while
Nattinger KJ, Dunn M, Feuer EJ. Trends in ESD does have a role in management of early
esophageal adenocarcinoma incidence and esophageal neoplasia, it is associated with a higher
mortality. Cancer. 2013 Mar 15;119(6):1149-58. stricture rate, and resection of the entire Barrett’s
segment would be unnecessary.

Question 3 REFERENCES
A 64-year-old man has been diagnosed with long Gamboa AM, Kim S, Force SD, et al. Treatment
segment Barrett’s esophagus with a focus of car- allocation in patients with early-stage esopha-
cinoma in situ on biopsy specimens. He also has a geal adenocarcinoma: Prevalence and predictors
history of morbid obesity, diabetes, and emphyse- of lymph node involvement. Cancer. 2016 Jul
ma. Cross-sectional imaging of the chest and abdo- 15;122(14):2150-7.
men is negative for metastases and an endoscopic Kim HP, Bulsiewicz WJ, Cotton CC, et al. Focal
ultrasound showed superficial nodular mucosa in endoscopic mucosal resection before radiofrequency
the distal esophagus, but did not demonstrate any ablation is equally effective and safe compared
enlargement of local lymph nodes. The patient ex- with radiofrequency ablation alone for the eradi-
presses a preference for aggressive treatment. The cation of Barrett’s esophagus with advanced neo-
most appropriate management would include: plasia. Gastrointest Endosc. 2012 Oct;76(4):733-9.
Chapter 14 — Gastrointestinal cancers 397

Li N, Pasricha S, Bulsiewicz WJ, et al. Effects Question 5


of preceding endoscopic mucosal resection on the Which of the following is not a known risk factor
efficacy and safety of radiofrequency ablation for for gastric cancer?
treatment of Barrett’s esophagus: results from the
United States Radiofrequency Ablation Registry. A. Lynch syndrome
Dis Esophagus. 2016 Aug;29(6):537-43. B. Selenium exposure
Belghazi K, Bergman JJGHM, Pouw RE. C. Menetrier’s disease
Management of Nodular Neoplasia in Barrett’s D. Germline mutation in E-cadherin gene
Esophagus: Endoscopic Mucosal Resection and E. Peutz-Jeghers syndrome
Endoscopic Submucosal Dissection. Gastrointest
Endosc Clin N Am. 2017 Jul;27(3):461-470. CORRECT ANSWER: B

RATIONALE
Question 4 H. pylori infection is by far the most important
Which racial or ethnic groups have the highest risk factor for gastric cancer worldwide. Less com-
risk of gastric adenocarcinoma in the US? mon risk factors for gastric cancer include Lynch
syndrome, Peutz-Jeghers syndrome, Menetrier’s
A. African Americans and Non-Hispanic Whites disease, and germline mutations in the CDH gene
B. Asians, American Indians, and Non-Hispanic (encoding E-cadherin). However, there is some
Whites evidence that selenium, as well as high consump-
C. Middle-Easterners and Latinos tion of fruits and vegetables may have protective
D. Asians, Latinos, and African Americans effects against gastric cancer.
E. South Asians and Middle-Easterners
REFERENCES
CORRECT ANSWER: D de Martel C, Forman D, Plummer M. Gastric can-
cer: epidemiology and risk factors. Gastroenterol
RATIONALE Clin North Am. 2013 Jun;42(2):219-40.
Gastric cancer is a leading cause of cancer death world- Giardiello FM, Welsh SB, Hamilton SR, et al.
wide, and Helicobacter pylori infection causes almost Increased risk of cancer in the Peutz-Jeghers
90 percent of non-cardia cancers, which is the pre- syndrome. N Engl J Med. 1987 Jun 11;316(24):
dominant type. Rates of gastric cancer are highest in 1511-4.
Hispanics (from Central and South America in particu- Qiao YL, Dawsey SM, Kamangar F, et al. Total
lar) and East Asian populations (particularly recent im- and cancer mortality after supplementation with
migrants), as well as African Americans. Non-Hispanic vitamins and minerals: follow-up of the Linxian
Whites have the lowest rate of gastric cancer. General Population Nutrition Intervention Trial. J
Natl Cancer Inst. 2009 Apr 1;101(7):507-18.
REFERENCES
Islami F, DeSantis CE, Jemal A. Incidence Trends
of Esophageal and Gastric Cancer Subtypes by Question 6
Race, Ethnicity, and Age in the United States, 1997- A 55-year-old Latino man undergoes upper endos-
2014. Clin Gastroenterol Hepatol. 2018 Jun 11. copy for dyspepsia symptoms, during which several
Lui FH, Tuan B, Swenson SL, Wong RJ. small nonbleeding antral ulcers are seen. Biopsies
Ethnic disparities in gastric cancer incidence of the antrum and corpus are taken during endos-
and survival in the USA: an updated analy- copy, and the histopathology review demonstrates
sis of 1992-2009 SEER data. Dig Dis Sci. 2014 the following finding (see path image). Which of
Dec;59(12):3027-34. the following is true regarding this patient?
398 Digestive Diseases Self-Education Program®

Leung WK, Wong IOL, Cheung KS, et al. Ef-


fects of Helicobacter pylori Treatment on Inci-
dence of Gastric Cancer in Older Individuals.
Gastroenterology. 2018 Jul;155(1):67-75.

Question 7
Which of the following is true regarding screening
for gastric cancer?

Image from Wikimedia commons A. Screening for gastric cancer in the general US
population is not recommended
A. Cessation of nonsteroidal anti-inflammatory B. Screening for gastric cancer is ineffective, even
medications is the most important factor in in high risk populations
ulcer healing C. Serum E-cadherin is a biomarker for gastric
B. Appropriate therapy can reduce his risk of cancer, with a sensitivity of 85 percent
intestinal-type gastric cancer D. Latino patients should be offered annual upper
C. The gastric injury is likely caused by iron endoscopy starting at age 50 years to screen
deposition for gastric cancer.
D. Partial gastrectomy is the treatment of choice E. During endoscopic screening, five random
for this condition gastric corpus biopsies should be obtained ac-
E. Repeat endoscopy in three months should be cording to the Sydney protocol.
performed to confirm ulcer healing
CORRECT ANSWER: A
CORRECT ANSWER: B
RATIONALE
RATIONALE Neither the US Preventive Services Task Force,
The pathology image shows an immunohistochemis- nor any specialty societies in the US recommend
try stain that is positive for H. pylori, which increas- screening for gastric cancer, outside of certain
es the risk of both peptic ulcer disease and gastric high risk populations. These populations in-
adenocarcinoma. Therefore, therapy with an appro- clude those with a strong family history of gastric
priate antimicrobial and acid suppression combina- cancer, individuals with familial adenomatous pol-
tion regimen active against H. pylori infection is yposis or Lynch syndrome, or perhaps those with
important to facilitate ulcer healing and to eradicate extensive intestinal metaplasia (IM). Screening for
this carcinogen. H. pylori is more associated with in- gastric cancer does take place in Japan (where the
testinal type gastric cancer than the diffuse type. Re- incidence of gastric cancer is much higher) and is
peat EGD is not routinely recommended to confirm associated with the detection of more early tumors
ulcer healing in patients with peptic ulcer disease, in asymptomatic patients. Serum E-cadherin lev-
unless there is concern for associated cancer. els are not used for gastric cancer detection. The
Sydney protocol was designed to detect H. pylori
REFERENCES infection, and involves sampling from specific
Nardone G, Staibano S, Rocco A, et al. Effect of areas in both the antrum and corpus. A related
Helicobacter pylori infection and its eradication technique, gastric mapping, is typically used for
on cell proliferation, DNA status, and oncogene surveillance of gastric IM and also requires sam-
expression in patients with chronic gastritis. Gut. pling of specific areas in the antrum, incisura, and
1999 Jun;44(6):789-99. corpus (and placing biopsies in separate jars).
Chapter 14 — Gastrointestinal cancers 399

REFERENCE A. Ileal adenocarcinoma is an important cause


Kim GH, Liang PS, Bang SJ, Hwang JH. Screening of death in patients with familial adenomatous
and surveillance for gastric cancer in the United polyposis
States: Is it needed? Gastrointest Endosc. 2016 B. Enteropathy-associated T cell lymphoma is as-
Jul;84(1):18-28. sociated with Crohn’s disease and has a good
prognosis
C. The most common location of small intestinal
Question 8 adenocarcinomas is the jejunum
Which of the following statements is true regard- D. Adenocarcinoma is the most common type of
ing gastric lymphomas? small intestinal cancer in the US
E. 75 percent of all small intestinal cancers are
A. Are rarely associated with H. pylori infection malignant GI stromal tumors
B. Are most commonly derived from clonal
populations of T cells CORRECT ANSWER: D
C. Often regress with eradication of H. pylori
infection RATIONALE
D. Are called MALT lymphomas when the 11:18 Small intestinal cancer is rare in the US. The most
chromosome translocation is absent common tumor type is adenocarcinoma, and most
E. Have a worse prognosis than patients with small intestinal adenocarcinomas are located in
gastric adenocarcinomas the duodenum. In patients with FAP who have
undergone colectomy, duodenal adenocarcinoma
CORRECT ANSWER: C is an important cause of death and thus these
patients benefit from surveillance endoscopy
RATIONALE with a side-viewing duodenoscope. Enteropathy-
Gastric lymphomas are commonly associated with associated T cell lymphoma is associated with
H. pylori infection, and are derived from B cell celiac disease (not Crohn’s disease) and has a poor
clones. An estimated 80 percent of gastric lym- prognosis. GISTs do occur in the small intestine,
phomas can regress with H. pylori eradication, but are less common than adenocarcinomas.
and these lymphomas have a more favorable prog-
nosis than adenocarcinomas. MALT lymphomas REFERENCES
are characterized by presence (not absence) of the Groves CJ, Saunders BP, Spigelman AD, Phillips
11:18 chromosome translocation (among others). RK. Duodenal cancer in patients with familial
adenomatous polyposis (FAP): results of a 10 year
REFERENCES prospective study. Gut. 2002 May;50(5):636-41.
Parsonnet J, Hansen S, Rodriguez L, et al. Helico- Sharaiha RZ, Lebwohl B, Reimers L, et al.
bacter pylori infection and gastric lymphoma. N Increasing incidence of enteropathy-associated
Engl J Med. 1994 May 5;330(18):1267-71. T-cell lymphoma in the United States, 1973-2008.
Steinbach G, Ford R, Glober G, et al. Antibi- Cancer. 2012 Aug 1;118(15):3786-92.
otic treatment of gastric lymphoma of mucosa-
associated lymphoid tissue. An uncontrolled trial.
Ann Intern Med. 1999 Jul 20;131(2):88-95. Question 10
A 62-year-old woman complains of diarrhea and
weight loss, prompting colonoscopy. Colonoscopy
Question 9 is normal, including random mucosal biopsies, but
Which of the following accurately describes small a two cm yellowish submucosal nodule with over-
intestinal cancer in the US? lying ulceration is seen in the terminal ileum. Key-
400 Digestive Diseases Self-Education Program®

hole biopsies of the lesion reveal uniform small heterogeneous mass in the head of the pancreas,
round cells with rare mitotic figures, a desmoplas- with enlargement of local lymph nodes. Which of
tic reaction and immunohistochemistry staining the following is not a risk factor for this disease?
is positive for synaptophysin and neuron-specific
enolase. What explains the diarrhea symptoms A. Female sex
this patient has? B. Smoking
C. African American race
A. Secretion of vasoactive intestinal peptide into D. Older age
the ileum E. Obesity
B. Concomitant irritable bowel syndrome
C. Yersinia infection CORRECT ANSWER: A
D. Crohn’s disease
E. Metastasis to the liver RATIONALE
This case represents a classic presentation of
CORRECT ANSWER: E pancreatic cancer, over 90 percent of which are
adenocarcinomas. Known risk factors for pancre-
RATIONALE atic adenocarcinoma include male sex, smoking,
This patient has a carcinoid tumor in the ileum, African American race, obesity and older age, as
which is the most common site of small intestinal well as a history of chronic pancreatitis. A strong
carcinoids. Histopathologic features of carcinoid family history of pancreatic cancer can also indi-
tumors include uniform small cells, a dense des- cate a genetic predisposition.
moplastic reaction, and IHC that can be positive
for chromogranins A, B, and C, synaptophysin and REFERENCE
neuron-specific enolase. In contrast to hindgut Midha S, Chawla S, Garg PK. Modifiable and non-
carcinoids, small intestinal carcinoid tumors modifiable risk factors for pancreatic cancer: A
frequently have metastasized at the time of diag- review. Cancer Lett. 2016 Oct 10;381(1):269-77.
nosis. While non-metastatic lesions do not cause
systemic symptoms because the liver inactivates
serotonin delivered via the portal vein, hepatic Question 12
metastasis allows for serotonin to be delivered All of the following conditions increase the risk of
into the systemic circulation and exert its effects pancreatic cancer, EXCEPT:
via the carcinoid syndrome.
A. Mutation in the cationic trypsinogen gene pro-
REFERENCE tease serine 1
Strosberg J. Neuroendocrine tumours of the small B. Lynch syndrome
intestine. Best Pract Res Clin Gastroenterol. 2012 C. Peutz-Jeghers syndrome
Dec;26(6):755-73. D. Familial atypical multiple mole melanoma
syndrome
E. Blue rubber bleb nevus syndrome
Question 11
A 67-year-old African American woman presents CORRECT ANSWER: E
with anorexia, weight loss and yellowing of the
eyes noticed by her spouse. Serum direct bilirubin RATIONALE
and alkaline phosphatase levels are elevated. She All of these conditions are associated with an in-
has a body mass index of 32 kg/m2 and is a former creased risk of pancreatic cancer, with the excep-
smoker. A CT scan shows biliary dilatation and a tion of blue rubber bleb nevus syndrome.
Chapter 14 — Gastrointestinal cancers 401

REFERENCE of Clinical Oncology Clinical Practice Guideline


Rieder H, Bartsch DK. Familial pancreatic cancer. Update. J Clin Oncol. 2017 Jul 10;35(20):2324-
Fam Cancer. 2004;3(1):69-74. 2328.
Neoptolemos JP, Palmer DH, Ghaneh P,
et al. Comparison of adjuvant gemcitabine and
Question 13 capecitabine with gemcitabine monotherapy in pa-
What is the main factor leading to the poor tients with resected pancreatic cancer (ESPAC-4):
prognosis for patients diagnosed with pancreatic a multicentre, open-label, randomised, phase 3
adenocarcinoma? trial. Lancet. 2017 Mar 11;389(10073):1011-1024.

A. The majority of patients undergoing pancreatico-


duodenectomy die from surgical complications. Question 14
B. A majority of patients present at a disease stage Which of the following is true regarding patients
that is not resectable. with familial pancreatic cancer and a first degree
C. Accurate diagnostic tests for pancreatic cancer relative with pancreatic cancer at age 50 years?
are lacking.
D. Chemotherapy does not convey a survival A. Screening with biannual CA-19-9 is associated
benefit in pancreatic cancer . with survival benefit.
E. A high risk of fatal bleeding from hemosuccus B. Screening with annual endoscopic ultrasound
pancreaticus. or MRCP should be offered at age 40 years.
C. Prophylactic pancreatectomy should be of-
CORRECT ANSWER: B fered after confirmation of pedigree.
D. Smoking does not affect their cancer risk.
RATIONALE E. ERCP with pancreatic duct stenting should
Only B is correct. A key factor in the poor prognosis be offered at age 40 years or 10 years prior to
of pancreatic cancer is related to late presentations youngest affected relative.
of the disease, after local invasion, vascular involve-
ment, and/or metastasis has occurred. Only 15 to CORRECT ANSWER: B
20 percent of patients are candidates for a Whipple
procedure, which (while not without risk) does of- RATIONALE
fer the potential for cure. Highly accurate diagnos- Several groups recommend screening of patients
tic tests are available, and include CT, MRCP, and from high risk pancreatic cancer kindreds. MRCP
endoscopic ultrasound. Certain chemotherapeutics and endoscopic ultrasound are generally recom-
such as gemcitabine and capecitabine do convey a mended due to high sensitivity, whereas CA-19-9
survival benefit against some forms of pancreatic has limited value for screening, but can be useful
adenocarcinoma and do have a role for adjuvant as a biomarker for recurrence in those with known
and palliative treatment. Hemosuccus pancreaticus disease. For patients with familial pancreatic
is a potential complication of locally invasive pan- cancer and a first degree relative with pancreatic
creatic cancer, but is rare. cancer, screening should begin at age 50 or 10
years prior to the youngest affected family mem-
REFERENCES ber. Similarly neither ERCP (with or without
Ryan DP, Hong TS, Bardeesy N. Pancre- pancreatic duct stenting) nor prophylactic pancre-
atic adenocarcinoma. N Engl J Med. 2014 Sep atectomy have a role in risk mitigation. In patients
11;371(11):1039-49. with a family history of pancreatic cancer, smok-
Khorana AA, Mangu PB, Berlin J, et al. Poten- ing both increases the incidence and is associated
tially Curable Pancreatic Cancer: American Society with earlier presentations of pancreatic cancer.
402 Digestive Diseases Self-Education Program®

REFERENCES REFERENCES
Syngal S., Brand R.E., Church J.M., et al: ACG Brugge W R, Lewandrowski K, Lee-Lewandrowski
clinical guideline: genetic testing and management E et al. Diagnosis of pancreatic cystic neoplasms:
of hereditary gastrointestinal cancer syndromes. a report of the cooperative pancreatic cyst study.
Am J Gastroenterol 2015; 110: pp. 223. Gastroenterology. 2004;126:1330–1336.
Rulyak S.J., Lowenfels A.B., Maisonneuve P., Vege SS, Ziring B, Jain R, Moayyedi P; Clinical
et al: Risk factors for the development of pancre- Guidelines Committee; American Gastroenterology
atic cancer in familial pancreatic cancer kindreds. Association. American gastroenterological associa-
Gastroenterology 2003; 124: pp. 1292. tion institute guideline on the diagnosis and man-
agement of asymptomatic neoplastic pancreatic
cysts. Gastroenterology. 2015 Apr;148(4):819-22.
Question 15
A 53-year-old woman presents to gastroenterology
clinic after a CT scan of the abdomen (ordered by Question 16
her primary care physician to evaluate an umbili- Which of the following factors increase the risk of
cal hernia) identified a three cm cystic lesion in cholangiocarcinoma?
the tail of the pancreas. She is asymptomatic and
denies any upper abdominal pain. An endoscopic A. Hereditary pancreatitis
ultrasound is performed, which demonstrates B. Caroli’s disease
dilation of the pancreatic duct. Fine needle aspira- C. Exposure to iodinated contrast dye
tion of the cyst finds that the intracystic fluid CEA D. Alcohol use
level is 260 ng/mL. E. Primary biliary cholangitis

The next step in management includes: CORRECT ANSWER: B

A. Referral to an experienced gastrointestinal RATIONALE


surgeon for distal pancreatectomy. Exposure to iodinated contrast dye does not in-
B. MRI surveillance in one year, then every two crease the risk of cholangiocarcinoma. However,
years to ensure no change in size. exposure to thorium dioxide or “Thorotrast”, which
C. Cyst-gastrostomy. was previously used as a radiologic contrast agent,
D. Repeat endoscopic ultrasound in six months has been associated with this disease. Infection
with repeat cyst aspiration. with the liver flukes Clonorchis sinensis (primarily
E. Measure serum CEA level to calculate the se- found in Korea) and Opisthorcis viverrini (primari-
rum/cyst CEA ratio. ly found in Thailand) increase the risk of cholangio-
carcinoma by two- to five-fold. Primary sclerosing
CORRECT ANSWER: A cholangitis, not primary biliary cholangitis, in-
creases the risk of cholangiocarcinoma. Other risk
RATIONALE factors for cholangiocarcinoma include anatomical
This patient has a mucinous cystic neoplasm changes of the bile duct such as choledochal cysts
and should be offered resection rather than sur- and Caroli’s disease (multiple cystic dilations of the
veillance, to reduce mortality from a pancreatic biliary tree), tobacco use, and asbestos exposure.
malignancy. Pancreatic surgery performed at
high volume centers is associated with improved REFERENCES
outcomes. Serum CEA level is not helpful for Bergquist A, von Seth E. Epidemiology of cholan-
risk stratification of patients with pancreatic giocarcinoma. Best Pract Res Clin Gastroenterol.
cyst lesions. 2015 Apr;29(2):221-32.
Chapter 14 — Gastrointestinal cancers 403

Zhu AX, Lauwers GY, Tanabe KK. Cholangiocarci- A. Persistent Hepatitis B infection from vertical
noma in association with Thorotrast exposure. transmission increases the risk of HCC by 2 fold.
J Hepatobiliary Pancreat Surg. 2004;11(6):430-3. B. Aflatoxin B1 exposure and hepatitis B infection
have a synergistic effect on the risk of HCC
C. Hepatitis B virus can lead to HCC only in the
Question 17 presence of cirrhosis
Which of the following is true regarding the epide- D. Hepatitis B virus integrates into host RNA to
miology of hepatocellular cancer (HCC)? inhibit tumor suppressor gene translation

A. The prevalence of chronic hepatitis B virus CORRECT ANSWER: B


correlates with HCC in most areas of the
world. RATIONALE
B. 50 percent of worldwide cases of HCC occur in Chronic hepatitis B virus infection from vertical
sub-Saharan Africa and Asia. transmission is the predominate mechanism for
C. In the US, HCC is more common than pancre- HCC in developing nations, and increases the risk
atic cancer. of HCC dramatically, by approximately 100-fold.
D. Most patients in the US who develop HCC have HBV (a DNA virus) integrates into the genome
hepatitis B cirrhosis. of the host, exerting a direct carcinogenic effect.
E. Alcohol and NAFLD together account for While most cases of HCC in HBV-infected persons
roughly 90 percent of HCC cases in the US. occur in the presence of cirrhosis, cirrhosis is not
required for HCC to develop. Therefore, clinicians
CORRECT ANSWER: A must be vigilant with respect to HCC screening
in HBV positive patients. Aflatoxin B1 is a fungal
RATIONALE toxin produced by certain Aspergillus species. In-
Hepatitis B does drive much of the global burden gestion of aflatoxin is more common in developing
of HCC, whereas in the US, hepatitis C is the most countries, due to mold contamination of stored
common etiology of cirrhosis and therefore the food. Patients who are co-exposed to aflatoxin and
most common risk factor for HCC. While colorec- hepatitis B have a dramatically higher risk of HCC
tal cancer is the most common GI cancer in the than those with either risk factor individually.
US, pancreatic cancer is the second most common,
with an incidence that is roughly 1.5 times higher REFERENCES
than that of HCC. In the US, alcohol and NAFLD El-Serag HB. Epidemiology of viral hepatitis and
are also important etiologic factors, and are re- hepatocellular carcinoma. Gastroenterology.
sponsible for roughly 20 percent and 10 percent of 2012;142(6):1264–1273.
HCC cases respectively. Eaton DL, Gallagher EP. Mechanisms of
aflatoxin carcinogenesis. Annu Rev Pharmacol
REFERENCE Toxicol. 1994;34:135-72.
Massarweh NN, El-Serag HB. Epidemiology Peers F, Bosch X, Kaldor J, Linsell A, Pluijmen
of Hepatocellular Carcinoma and Intrahepatic M. Aflatoxin exposure, hepatitis B virus infection
Cholangiocarcinoma. Cancer Control. 2017 Jul- and liver cancer in Swaziland. Int J Cancer. 1987
Sep;24(3):1073274817729245. May 15;39(5):545-53.

Question 18 Question 19
Which of the following is correct regarding the A 70-year-old man is referred for evaluation of
pathogenesis of hepatocellular carcinoma? fatigue, weight loss, and elevated LFTs. An MRI
404 Digestive Diseases Self-Education Program®

of the liver shows a three cm suspected cavern- REFERENCES


ous hemangioma in the liver, multiple small Zheng YW, Zhang XW, Zhang JL, Hui ZZ, Du
nodules in the spleen, and trace ascites, but no WJ, Li RM, Ren XB. Primary hepatic angiosar-
evidence of cirrhosis. He does not drink alcohol coma and potential treatment options. J Gastro-
and tested negative for viral hepatitis. He is con- enterol Hepatol. 2014 May;29(5):906-11.
cerned, because his younger brother died from Orlando G, Adam R, Mirza D et al. Hepatic
a bleeding liver tumor, and they worked at the hemangiosarcoma: an absolute contraindica-
same factory where they were exposed to “lots of tion to liver transplantation--the European Liver
chemicals”. Which of the following is correct? Transplant Registry experience. Transplanta-
tion. 2013 Mar 27;95(6):872-7.
A. He should be referred for liver
transplantation Question 20
B. Given absence of cirrhosis, he should be of- Which of the following patients does NOT merit
fered reassurance and repeat imaging in one screening for HCC?
year to ensure stability
C. His family history is suggestive of hereditary A. A 25-year-old immigrant from Uganda who
hepatic hemangioma (HHH) syndrome is on antiviral therapy for hepatitis B.
D. His risk of intraperitoneal bleeding is low B. A 55-year-old man with hepatitis C cirrhosis
E. His liver lesion is likely related to occupa- who achieved a sustained virologic response
tional exposure to vinyl chloride after treatment with combination sofosbuvir
and velpatasvir.
CORRECT ANSWER: E C. A 40-year-old with alcoholic cirrhosis who
has been listed for liver transplantation.
RATIONALE D. A 60-year-old woman with severe COPD on
This presentation is concerning for metastatic home oxygen and decompensated NAFLD
hepatic angiosarcoma, especially in light of the cirrhosis, who is not a transplant candidate.
reported history of a coworker (in this case his E. A 65-year-old man with chronic hepatitis C
brother) who died of a rare liver tumor. He- infection and F3 fibrosis on elastography.
patic angiosarcomas are related to occupational
exposure to vinyl chloride, an agent used in CORRECT ANSWER: D
chemical manufacturing. It can be misdiagnosed
on imaging as cavernous hepatic hemangiomas, RATIONALE
but a hemangioma would not be expected to be In general, all patients with advanced liver fibro-
associated with abnormalities in LFTs, ascites, sis or cirrhosis should be offered HCC screening
or splenic nodules. The risk of intraperitoneal in the form of liver imaging (e.g. ultrasound)
bleeding is estimated to be roughly 25 percent with or without serum AFP every six months to
and can complicate attempts at needle biopsy. reduce the risk of HCC mortality by detecting
Suggested treatment for hepatic angiosarcoma HCC at an earlier, more treatable stage. African
involves local excision and transarterial emboli- immigrants with hepatitis B virus should be
zation, particularly in the case of intraperitoneal screened after age 20 years. Patients who have
bleeding. Some argue that hepatic angiosarcoma eradicated hepatitis C virus are still at risk of
is a contraindication to liver transplantation giv- HCC and should continue to be offered screen-
en poor outcomes, and this patient would not be ing. Exceptions to this screening recommenda-
a candidate for liver transplantation given likely tion include patients with limited life expectan-
metastasis to spleen. Hereditary hepatic heman- cy, such as those with decompensated cirrhosis
gioma syndrome is not a real clinical entity. who are not transplant candidates.
Chapter 14 — Gastrointestinal cancers 405

REFERENCES Question 22
Heimbach JK, Kulik LM, Finn RS, et al. Which racial or ethnic group has the highest
AASLD guidelines for the treatment of he- reported incidence and mortality from colorectal
patocellular carcinoma. Hepatology. 2018 cancer in the US?
Jan;67(1):358-380.
Jacobson IM, Lim JK, Fried MW. American A. Latinos
Gastroenterological Association Institute Clini- B. African Americans
cal Practice Update-Expert Review: Care of Pa- C. Non-Hispanic whites
tients Who Have Achieved a Sustained Virologic D. Alaskan Natives
Response After Antiviral Therapy for Chronic E. Asians
Hepatitis C Infection. Gastroenterology. 2017
May;152(6):1578-1587. CORRECT ANSWER: D

RATIONALE
Question 21 Alaskan natives have both the highest inci-
You perform a colonoscopy for a patient who un- dence and mortality for colorectal cancer in
derwent sigmoid resection for stage two colorec- the US, markedly higher than any other
tal cancer one year ago. The colonoscopy reveals ethnic group.
one diminutive adenoma in the cecum, which
you remove with a cold snare. REFERENCES
Surveillance, Epidemiology, and End Results
When should you recommend the next (SEER) Program, 2016.
colonoscopy? National Center for Health Statistics, Cen-
ters for Disease Control and Prevention, 2016.
A. Ten years Perdue DG, Haverkamp D, Perkins C,
B. Five years Daley CM, Provost E. Geographic variation
C. Three years in colorectal cancer incidence and mortality,
D. One year age of onset, and stage at diagnosis among
E. Six months American Indian and Alaska Native people,
1990-2009. Am J Public Health. 2014 Jun;104
CORRECT ANSWER: C Suppl 3:S404-14.

RATIONALE
According to the Multi-Society Task Force Question 23
on Colorectal Cancer, colonoscopy should be A 26-year-old man is referred to GI clinic with
performed one year after resection, and again microcytic anemia. He is asymptomatic apart
three years later, in order to decrease the risk of from mild fatigue, and denies any GI symptoms.
metachronous colorectal cancer. Specifically, he denies abdominal pain, constipa-
tion or diarrhea, vomiting, or any hematochezia
REFERENCE or melena. He reports no known family history
Kahi CJ, Boland CR, Dominitz JA. Colonoscopy of GI conditions, but is adopted. Physical exam
Surveillance After Colorectal Cancer Resection: is normal. The patient was recently seen by oph-
Recommendations of the US Multi-Society Task thalmology, and was found to have an abnormal
Force on Colorectal Cancer. Gastroenterology. fundoscopic exam showing the following find-
2016. 150(3):758–768.e11. ings in both eyes:
406 Digestive Diseases Self-Education Program®

genes listed are affected in Lynch syndrome, not


FAP. CHRPE is not associated with celiac disease
(answer choice E).

REFERENCES
Image was originally published in the Retina
Image Bank® website. Author: Maturi R. Photog-
rapher: Steele T. Retina Image Bank. 2012; “Bear
Tracks” File no: 2967. © the American Society of
Retina Specialists.
Retina Image Bank® Bertario L, Bandello F, Rossetti C, et al. Con-
genital hypertrophy of retinal pigment epithelium
The appropriate next step in management is: (CHRPE) as a marker for familial adenomatous pol-
yposis (FAP). Eur J Cancer Prev. 1993 Jan;2(1):69-75.
A. Schedule diagnostic upper endoscopy and
colonoscopy
B. Referral back to PCP for management of Question 24
diabetes A 33-year-old woman is seen for a well woman vis-
C. Genetic testing for defects in MLH1, MSH2, it, and reports a family history of colorectal cancer
MSH6, and PMS2 genes in her mother, who was diagnosed at age 55 years,
D. Referral for genetic counselling and died of the disease at age 58 years. When
E. Laboratory testing including iron studies, total should she begin colorectal cancer screening?
IgA and tissue trans-glutaminase IgA levels
A. Now (age 33 years), then annually
CORRECT ANSWER: A B. Age 50 years, then every five years
C. Age 45 years, then every 10 years
RATIONALE D. Age 40 years, then every five years
The fundoscopic image depicts congenital hy- E. Age 35 years, then every 10 years
pertrophy of the retinal pigment epithelium
(CHRPE), colloquially referred to as “bear tracks”. CORRECT ANSWER: D
CHRPE is characterized by multiple hyperpig-
mented round or oval-shaped spots on the retina, RATIONALE
typically bilaterally. CRHPE typically does not Current guidelines from the US Multisociety Task-
cause symptoms and is found during routine eye force recommend that CRC screening begin earlier
examinations. As a clinical sign, CHRPE has a for patients with a family history of CRC in a first
positive predictive value of approximately 90 per- degree relative who was diagnosed under the age
cent for Familial Adenomatous Polyposis (FAP). of 60 years. Screening should begin at age 40 or
Therefore, an endoscopic evaluation is warranted, 10 years prior to earliest age of diagnosis, and
especially in light of unknown family history and continue every five years thereafter.
likely iron deficiency anemia suggesting occult GI
bleeding. Both upper and lower endoscopy should REFERENCE
be performed given the high frequency of duode- Rex DK, Boland CR, Dominitz JA et al. Colorectal
nal adenomas and fundic gland polyps in patients Cancer Screening: Recommendations for Physi-
with FAP. It would not be appropriate to perform cians and Patients From the U.S. Multi-Society
genetic testing or refer to genetic counselling with- Task Force on Colorectal Cancer. Gastroenterol-
out better phenotypic characterization, and the ogy. 2017 Jul;153(1):307-323.
Chapter 14 — Gastrointestinal cancers 407

Question 25 of hereditary gastrointestinal cancer syndromes.


A 45-year-old woman comes to see you to discuss Am J Gastroenterol 2015; 110: pp. 223.
colorectal cancer screening. Her family history Rubenstein JH, Enns R, et al. AGA Guidelines
reveals colon cancer in her father at age 58 years, on Genetic Evaluation and Management of Lynch
as well as his brother at age 49 years. Her paternal Syndrome: A Consensus Statement by the US
grandmother had endometrial cancer at age 53 Multi-Society Task Force on Colorectal Cancer.
years, and paternal aunt had breast cancer at age Gastroenterology 2015; 149 (3): 777-782.
65 years.

In addition to colonoscopy, what is the next step Question 26


in management for this patient? A 32-year-old woman presents to the office to
establish care. Her mother was diagnosed with
A. EGD with side-viewing duodenoscope colorectal cancer at the age of 45 years, testing
B. Thyroid ultrasound of the cancer specimen confirms microsatellite
C. Fundoscopic examination instability.
D. EUS of pancreas
E. Genetic counseling Other than colonoscopy every one to two years,
which of the following screening tests are recom-
CORRECT ANSWER: E mended for this young woman?

RATIONALE A. Pelvic MRI annually


This patient meets Amsterdam criteria for Lynch B. PAP smears every six months
Syndrome (three relatives affected with Lynch C. LH and FSH testing annually
related tumors, two successive generations, one D. Annual transvaginal ultrasound
must be a first degree relative of the other two, E. Mammogram every six months
and one is diagnosed less than 50 years old).
Therefore, she should undergo genetic counsel- CORRECT ANSWER: D
ing and testing for the Lynch related mutations
in mismatch repair genes. Ideally, one of the RATIONALE
affected family members should be tested first, This patient is at high-risk for Lynch syndrome,
to see which gene mutation is responsible for the given the microsatellite instability present on her
syndrome in the family. Thyroid ultrasound is mother’s cancer specimen. Screening recommen-
recommended for patients with FAP, not Lynch dations per AGA guidelines include: colonoscopy
syndrome, as papillary thyroid cancer is associ- every one to two years starting age 20-25 years,
ated with FAP. While EGD is recommended for pelvic exam with endometrial sampling and trans-
patients with Lynch syndrome, an exam with vaginal ultrasound every year starting age 30-35
side-viewing duodenoscope to rule out ampullary years, EGD with antral biopsies every two to three
neoplasm must be done in FAP patients (ampul- years starting age 30-35 years, and urinalysis an-
lary adenocarcinoma is the second leading cause nually starting age 30-35 years.
of cancer death in FAP patients). EUS of pancreas
should be considered in patients with Lynch Syn- REFERENCE
drome, but a diagnosis should be made first. Giardiello et al. Guidelines on genetic evaluation
and management of Lynch Syndrome: a con-
REFERENCES sensus statement by the US Multi-Society Task
Syngal S., Brand R.E., Church J.M., et al: ACG Force on Colorectal Cancer. Gastroenterology.
clinical guideline: genetic testing and management 2014;147(2):502-26.
408 Digestive Diseases Self-Education Program®

Question 27 hemorrhoids. Pathology of the polyps shows that


A 28-year-old woman presents to the office for eight polyps are ganglioneuromas and two are
evaluation of recurrent epigastric abdominal pain hyperplastic polyps. The patient reports no family
and chronic iron deficiency anemia. On physical history of colon cancer. His mother was diagnosed
exam, she has mucocutaneous pigmentation on with breast cancer at age 48 years.
her lips, otherwise the exam in unremarkable. An
upper endoscopy and colonoscopy are performed
for evaluation of the anemia, and multiple ham-
artomatous polyps are removed from the small
bowel and colon. Genetic testing is positive for the
STK11 mutation. Given her diagnosis, which of
the following do you recommend for screening?

A. Pelvic MRI annually


B. Pap smears every six months
C. Breast MRI annually
D. Endometrial biopsies annually Which of the following is the most likely
E. CA-125 testing every six months diagnosis?

CORRECT ANSWER: C A. Lynch syndrome


B. Familial adenomatous polyposis
RATIONALE C. MutYH-associated polyposis
This patient has Peutz-Jeghers Syndrome (PJS), D. Cowden’s syndrome
characterized by two or more hamartomatous E. Neurofibromatosis
polyps in the small bowel or colon, mucocutane-
ous pigmentation of the mouth/lips and fingers. CORRECT ANSWER: D
PJS is associated with increased risks of colorectal,
stomach, small bowel, pancreatic, breast, ovarian, RATIONALE
endometrial, cervical, testicular, and lung cancers. This patient has Cowden’s syndrome due to a
Gynecologic surveillance recommendations include mutation in the PTEN gene. Individuals with
annual breast self-exams, breast imaging, transvag- Cowden’s syndrome can develop hamartomas
inal ultrasounds, pelvic exams and pap smears. and ganglioneuromas in the gastrointestinal
tract, and cumulative colorectal cancer risk is
REFERENCE nine to 16 percent. Cowden’s syndrome is also
Stoffe EM, Kastronios F. Familial Colorectal Can- associated with increased risk of breast cancer,
cer, Beyond Lynch Syndrome. Clin Gastroenterol thyroid cancer, endometrial cancer, renal cell
Hepatol. 2014;12(7):1059-68. cancer and melanoma. Non-malignant features of
Cowden’s syndrome include glycogenic acantho-
sis of the esophagus (as in the endoscopy picture
Question 28 above), as well as macrocephaly and skin lesions
A 35-year-old male undergoes upper and lower called trichilemmomas. The patient’s personal
endoscopy for chronic heartburn and rectal bleed- and family history is not consistent with Lynch
ing. The upper endoscopy is normal except for syndrome or adenomatous polyposis syndromes
the findings in the esophagus (see image). The such as familial adenomatous polyposis or
colonoscopy reveals 10 polyps throughout the MutYH-associated polyposis. While patients with
colon ranging from three mm-12mm and internal neurofibromatosis can develop ganglioneuromas
Chapter 14 — Gastrointestinal cancers 409

in the gastrointestinal tract, the clinical picture gery is not established but should be done in young
does not fit with this condition. adulthood. In patients in whom gastrectomy is not
currently pursued, endoscopic surveillance should
REFERENCE be done according to the Cambridge protocol with
Syngal S, Brand RE, Church JM, et al. ACG clini- a minimum of 30 gastric biopsies. The other mo-
cal guideline: Genetic testing and management of dalities listed here are not effective for treatment or
hereditary gastrointestinal cancer syndromes. Am surveillance in HDGC.
J Gastroenterol. 2015 Feb;110(2):223-62
REFERENCES
van der Post RS, Vogelaar IP, Carneiro F, et
Question 29 al. Hereditary diffuse gastric cancer: updated
A 32-year-old male presents for discussion of his clinical guidelines with an emphasis on germ-
family history and genetic test results. The patient line CDH1 mutation carriers. J Med Genet. 2015
has no personal history of cancer and reports no Jun;52(6):361-74.
gastrointestinal symptoms. The patient reports Syngal S, Brand RE, Church JM, et al.
that his mother developed lobular breast cancer at ACG clinical guideline: Genetic testing and
age 35, his maternal uncle died of diffuse gastric management of hereditary gastrointestinal
cancer at age 42 years and his maternal grand- cancer syndromes. Am J Gastroenterol. 2015
mother also had gastric cancer at age 52 years, Feb;110(2):223-62.
although the histology of his grandmother’s cancer
was not known. The patient’s mother underwent
genetic testing that revealed a mutation in CDH1. Question 30
The patient had genetic testing and was found to A 52-year-old female presents with rectal bleed-
carry the same mutation. ing. She undergoes a colonoscopy that reveals 30
polyps throughout the colon. These polyps range
What is the best next step in management? in size from three mm to 20 mm. In addition,
a three cm mass is found in the cecum that is
A. Computed tomography of the abdomen partially circumferential. Pathology of the polyps
and pelvis reveals tubular and tubulovillous adenomas and
B. Upper GI study biopsies of the mass show a villous adenoma. The
C. Refer to surgery for total gastrectomy patient reports no family history of colon polyps or
D. Endoscopic ultrasound cancer. There is no family history of other cancers.
She undergoes genetic testing with a multigene
CORRECT ANSWER: C panel and is found to carry two mutations in the
MutYH gene.
RATIONALE
This patient carries a pathogenic variant in CDH1, In addition to referring the patient to surgery for
consistent with a diagnosis of hereditary diffuse colectomy, what other testing should be done?
gastric cancer (HDGC). The patient’s family history
of diffuse gastric cancer and lobular breast cancer A. Endometrial biopsy
under age 50 is consistent with this diagnosis. The B. Computed tomography of the abdomen
lifetime risk of diffuse gastric cancer in HDGC is 70 C. Upper endoscopy with side viewing
percent for men and 56 percent for women. Risk of duodenoscopey
lobular breast cancer for women is 42 percent. Risk D. Endoscopic ultrasound
reducing total gastrectomy is recommended for
CDH1 mutation carriers. The optimal age for sur- CORRECT ANSWER: C
410 Digestive Diseases Self-Education Program®

RATIONALE CORRECT ANSWER: B


MutYH-associated polyposis (MAP) is a recessive
condition due to mutations in the base excision RATIONALE
repair pathway gene MutYH. Clinical features in For individuals with a first-degree relative with
MAP are similar to attenuated familial adeno- colorectal cancer, current U.S. Multi-Society Task
matous polyposis (AFAP). Individuals with MAP Force (MSTF) guidelines recommend screening to
typically develop colonic adenomas numbering begin at age 40 or 10 years earlier than the young-
20-100. MAP also increases risk of duodenal est diagnosis in the first-degree family member. If
adenomas and cancer. Current guidelines recom- the family member was under the age of 60 years
mend colonoscopy every one to two years start- at diagnosis, the individual should undergo colo-
ing at age 25 to 30 years and colectomy if polyp noscopy starting at age 40 years and repeat the
burden not manageable endoscopically. Upper colonoscopy every five years as in this question.
endoscopy with duodenoscopy with visualization If the family member was age 60 years or older,
of the ampulla is also recommended. Although the individual should start at age 40 years using
endometrial cancer risk might be elevated in options and intervals recommended for average
MAP, there are no recommendations for endo- risk individuals. The 2017 MSTF guidelines also
metrial biopsy unless patients are symptomatic. recommend screening to start at age 45 years in
CT and EUS are not recommended for screening African Americans due to higher incidence rates,
in MAP. earlier mean age of onset, worse survival and
late-stage presentation and a higher proportion of
REFERENCES cancers occurring before age 50 years. In this
Syngal S, Brand RE, Church JM, et al. ACG clini- Question, while the patient is African-American,
cal guideline: Genetic testing and management of the family history indicates screening to begin at
hereditary gastrointestinal cancer syndromes. Am age 40 years and not age 45 years.
J Gastroenterol. 2015 Feb;110(2):223-62.
Win AK, Reece JC, Dowty JG, et al. Risk of REFERENCE
extracolonic cancers for people with biallelic and Rex DK, Boland CR, Dominitz JA, Giardiello
monoallelic mutations in MUTYH. Int J Cancer. FM, Johnson DA, Kaltenbach T, Levin TR, Li-
2016 Oct 1;139(7):1557-63. eberman D, Robertson DJ. Colorectal Cancer
Screening: Recommendations for Physicians and
Patients from the U.S. Multi-Society Task Force
Question 31 on Colorectal Cancer. Gastroenterology. 2017
A 38-year-old African American female presents Jul;153(1):307-323.
to discuss her family history of colon cancer. Her
father was diagnosed with stage 4 colon cancer at
age 52 years. The patient reports no other family Question 32
history of colon or other cancers. She is healthy A 54-year-old female undergoes her first screen-
and reports no gastrointestinal symptoms. ing colonoscopy. She reports no family history of
colorectal cancer. The examination is complete to
When should the patient have a colonoscopy? the cecum with an excellent bowel preparation.
Three polyps are removed including a 17 millime-
A. Now ter polyp in the ascending colon, a six millimeter
B. Age 40 years polyp at the hepatic flexure and a four millimeter
C. Age 42 years polyp in the sigmoid colon. Resection of all the
D. Age 45 years polyps is complete. Pathology shows that the
E. Age 50 years ascending colon polyp is a sessile serrated polyp
Chapter 14 — Gastrointestinal cancers 411

without dysplasia, the hepatic flexure polyp is a The distance between the adenocarcinoma and
tubular adenoma and the sigmoid polyp is hyper- resection margin is a 0.5mm without evidence of
plastic. lymphovascular invasion.

When should the patient return for surveillance What is the next best step in management?
colonoscopy?
A. Colonoscopy in three months
A. One year B. Colonoscopy in one year
B. Three years C. Refer for surgical resection
C. Five years D. PET scan
D. Ten years
CORRECT ANSWER: C
CORRECT ANSWER: B
RATIONALE
RATIONALE A malignant polyp is defined as a polyp with can-
Surveillance intervals after polypectomy depend on cer invading through the muscularis mucosa and
size, pathology and polyp number assuming com- into the submucosa. Unfavorable histologic fea-
plete resection of the polyp and an adequate bowel tures of malignant polyps include: poorly differen-
preparation. A three year interval is recommended tiated, presence of angiolymphatic invasion and/
for the following findings on baseline colonoscopy: or positive margin of resection (defined as tumor
three to 10 tubular adenomas, one or more tubular less than one to two mm from the transected mar-
adenomas 10mm or larger, one or more villous ad- gin or tumor cells present at the margin). In cases
enomas, adenoma with high-grade dysplasia, sessile of a malignant polyp with unfavorable histological
serrated polyps 10mm or larger, sessile serrated features, surgery is recommended. With favorable
polyp with dysplasia or traditional serrated ad- features, endoscopic resection is curative. There is
enoma. In this case, the most advanced finding was a controversy as to whether sessile malignant polyps
sessile serrated polyp greater than 10mm; therefore, should all be treated surgically, or if endoscopic
the surveillance interval should be three years. resection is feasible in the case of favorable histo-
logic features. In the case, there were unfavorable
REFERENCE features, and surgery is the treatment of choice.
Lieberman DA, Rex DK, Winawer SJ, et al. Guide- There is no role for PET scan in the evaluation of
lines for colonoscopy surveillance after screening malignant polyps.
and polypectomy: A consensus update by the US
Multi-Society Task Force on Colorectal Cancer. REFERENCE
Gastroenterology 2012; 143:844. Volk EE, Goldblum JR, Petras RE, Carey WD,
Fazio VW. Management and outcome of patients
with invasive carcinoma arising in colorectal pol-
Question 33 yps. Gastroenterology. 1995 Dec;109(6):1801-7.
A 55-year-old male undergoes his first colonos- ASGE Standards of Practice Committee,
copy for screening. He reports no gastrointestinal Fisher DA, Shergill AK, et al. Role of endoscopy in
symptoms and no family history of colorectal the staging and management of colorectal cancer.
cancer. A 20mm pedunculated polyp is found in Gastrointest Endosc. 2013 Jul;78(1):8-12.
the rectosigmoid colon and resected completely National Comprehensive Cancer Network.
using snare polypectomy with cautery. Pathology Colon Cancer (Version 3.2018). https://www.
of the polyp reveals poorly differentiated adeno- nccn.org/professionals/physician_gls/pdf/colon.
carcinoma with invasion into the submucosa. pdf Accessed September 24, 2018.
412 Digestive Diseases Self-Education Program®

Question 34 having a medullary growth pattern with many


A 45-year-old female presents with dyspepsia for tumor infiltrating lymphocytes. The pathology re-
three months. She reports pain in the epigastrium port has an addendum that states: “immunohisto-
that is not related to eating. She denies nausea, chemistry shows intact MLH1 and PMS2 staining
vomiting, and heartburn. She has not had weight and absence of MSH2 and MSH6 staining”. The
loss. A stool H. pylori antigen test is found to be patient had been recommended to see a genetic
positive and the patient is treated with quadruple counselor, but did not follow up. She reports a
therapy with complete resolution of her symp- family history of a father with colon cancer at age
toms. As part of the evaluation, a right upper 49 years and paternal aunt with endometrial can-
quadrant ultrasound was performed and revealed cer at age 52 years. The patient’s colonoscopy is
a 12mm fixed, hyperechoic shadowing defect in completed to the cecum with an adequate prepa-
the gallbladder. No gallstones were present. ration. A 5mm tubular adenoma at the splenic
flexure is completely excised.
What is the best next step?
What is the best surveillance interval for this
A. Refer for cholecystectomy patient?
B. HIDA scan
C. MRCP A. Six months
D. Repeat ultrasound in six to12 months B. One year
C. Three years
CORRECT ANSWER: A D. Five years

RATIONALE CORRECT ANSWER: B


The prevalence of gallbladder polyps is approxi-
mately four to seven percent and most are found RATIONALE
incidentally on imaging done for another indica- Although the patient has not had genetic test-
tion. Risk of malignancy is associated with age ing, she most likely has Lynch syndrome. The
over 50 years, polyp size larger than one cm, gall- tumor pathology has features typical of mismatch
stones, solitary polyp and symptomatic polyp. In repair deficiency and immunohistochemistry
cases with polyps 10mm or greater or if symptoms shows absent MSH2 and MSH6 staining, most
are present or if gallstones are present, cholecys- likely consistent with a germline MSH2 mutation.
tectomy is advocated. For polyps that are less than The patient’s family history meets Amsterdam II
10mm, surveillance is indicated. criteria with three affected individuals, at least
two generations, one individual is a first degree
REFERENCE relative of the others and one individual was
Gallahan WC, Conway JD. Diagnosis and manage- diagnosed under age 50 years. Current guidelines
ment of gallbladder polyps. Gastroenterol Clin for surveillance in Lynch syndrome recommend
North Am. 2010 Jun;39(2):359-67. colonoscopy every one to two years.

REFERENCES
Question 35 Giardiello FM, Allen JI, Axilbund JE Guidelines
A 51-year-old female diagnosed with colon cancer on genetic evaluation and management of Lynch
in the ascending colon one year ago returns for syndrome: a consensus statement by the US
surveillance colonoscopy. She underwent right Multi-society Task Force on colorectal cancer. Am
hemicolectomy and was found to have no lymph J Gastroenterol. 2014 Aug;109(8):1159-79.
node involvement. The tumor was described as Syngal S, Brand RE, Church JM, et al.
Chapter 14 — Gastrointestinal cancers 413

ACG clinical guideline: Genetic testing and TR, Robertson DJ, Rex DK. Colonoscopy Surveil-
management of hereditary gastrointestinal lance after Colorectal Cancer Resection: Recom-
cancer syndromes. Am J Gastroenterol. 2015 mendations of the US Multi-Society Task Force
Feb;110(2):223-62. on Colorectal Cancer. Am J Gastroenterol. 2016
Mar;111(3):337-46.

Question 36
A 64-year-old man presents for colonoscopy due Question 37
to new iron deficiency anemia and constipation. A A 40-year-old female comes to the office to estab-
circumferential mass is found in the sigmoid colon lish care. She reports a history of freckling on her
that precludes passage of the colonoscope. Biopsies lips and buccal mucosa that has faded since child-
of the mass confirm adenocarcinoma. A computed hood (see image). She reports a history of small
tomography colonography (CTC) is performed and bowel obstruction at age 20 years that required
confirms an almost completely obstructing mass in surgery to remove a large jejunal polyp. Since
the sigmoid, but no other polyps are visualized. The then, she reports about 10 cumulative polyps in
patient undergoes a sigmoid resection and returns the small intestine that have been removed by
for follow-up three months later. double balloon enteroscopy. Pathology of the
small intestinal polyps showed an arborizing
What is the best next step in management? pattern of smooth muscle proliferation. She had
genetic testing that showed a germline mutation
A. Colonoscopy now in the STK11 gene. Video capsule endoscopy, EGD
B. Colonoscopy in nine months and colonoscopy performed six months were nor-
C. CTC now mal without polyps.
D. CTC in nine months
E. fecal immunochemical testing now

CORRECT ANSWER: A

RATIONALE
The patient had an obstructing colon cancer
that precluded complete colonoscopy at diagno-
sis. Therefore, current guidelines recommend a
complete colonoscopy three to six months after
surgical resection to assess for synchronous le-
sions. Even though the patient had a CTC pre-
operatively, colonoscopy three to six months
later is still considered prudent. Small and flat
lesions can be missed or not reported by CTC,
but are highly relevant in a patient with colorec- What is the next best step in her management?
tal cancer. Fecal immunochemical testing is not
recommended for surveillance after colorectal A. CT enterography
cancer resection. B. Colonoscopy
C. MRI/MRCP
REFERENCE D. Small bowel follow through exam
Kahi CJ, Boland CR, Dominitz JA, Giardiello FM,
Johnson DA, Kaltenbach T, Lieberman D, Levin CORRECT ANSWER: C
414 Digestive Diseases Self-Education Program®

RATIONALE performed that shown many sessile polyps in


The patient has Peutz-Jeghers syndrome due the fundus and body (see image). Five polyps are
to a mutation in the STK11 gene. This is a rare sampled and pathology shows fundic gland polyps
cancer predisposition syndrome characterized with low-grade dysplasia.
by mucosal freckling, hamartomas (especially of
the small intestine), with a characteristic pattern
of arborizing smooth muscle, and increased risk
of multiple cancers including colorectal, small
bowel, stomach, pancreas, breast, ovarian, uter-
ine, cervical and testicular. The risk of pancreatic
cancer is estimated to be 132-fold increased over
the general population. Gastrointestinal surveil-
lance including colonoscopy, upper endoscopy
and small bowel imaging are recommended every
three years or more frequently if polyps are identi-
fied. Pancreatic cancer screening is also suggested
in individuals with Peutz-Jeghers syndrome, with
MRI and/or endoscopic ultrasound every one to
two years starting at age 35 years. This should ide-
ally be performed in a center utilizing a multidisci-
plinary approach. In this question, the best answer
is MRI/MRCP for pancreatic cancer screening
since she recently had upper and lower endoscopy What is the best next step in management?
as well as imaging of the small intestine.
A. Stop proton pump inhibitor
REFERENCE B. Repeat EGD and resect more polyps
Syngal S, Brand RE, Church JM, et al. ACG clini- C. Perform colonoscopy
cal guideline: Genetic testing and management of D. Refer for partial gastrectomy
hereditary gastrointestinal cancer syndromes. Am
J Gastroenterol. 2015 Feb;110(2):223-62. CORRECT ANSWER: C

RATIONALE
Question 38 This patient likely has familial adenomatous
A 24-year-old undergoes upper endoscopy for polyposis (FAP) and colonoscopy should be per-
dyspeptic symptoms. She reports six months of formed to evaluate for colorectal adenomas. In
epigastric abdominal discomfort and abdominal addition to colorectal and duodenal adenomas, in-
fullness. She denies nausea, vomiting, heartburn dividuals with FAP can develop numerous fundic
or weight loss. There is no family history of gas- gland polyps in the stomach. Fundic gland polyps
trointestinal malignancies. Laboratory evaluation are not considered to have malignant potential,
reveals normal comprehensive metabolic panel. even with the presence of low-grade dysplasia.
Complete blood count shows mildly decreased he- Gastrectomy is not indicated for management of
moglobin and hematocrit. H. pylori stool antigen fundic gland polyps. However, it should be noted
testing was negative. The patient was treated with that prevalence of gastric cancer appears to be ris-
once daily proton pump inhibitor therapy. She ing in FAP and careful evaluation for lesions with
returns two months later and reports no improve- malignant potential should be undertaken. Fundic
ment in her symptoms. An upper endoscopy is gland polyps are also associated with proton pump
Chapter 14 — Gastrointestinal cancers 415

inhibitor use, though the patient in this question What is the most likely diagnosis?
had only taken this medication for a short period
of time and this is unlikely to explain the large A. Juvenile polyposis syndrome
number of gastric polyps. B. Menetrier’s disease
C. Cronkhite-Canada syndrome
REFERENCES D. Peutz-Jeghers syndrome
Syngal S, Brand RE, Church JM, et al. ACG clini- E. Gastric lymphoma
cal guideline: Genetic testing and management of
hereditary gastrointestinal cancer syndromes. Am CORRECT ANSWER: A
J Gastroenterol. 2015 Feb;110(2):223-62.
Mankaney G, Leone P, Cruise M, LaGuardia L, RATIONALE
O’Malley M, Bhatt A, Church J, Burke CA. Gastric This patient has juvenile polyposis syndrome
cancer in FAP: a concerning rise in incidence. Fam (JPS) with large gastric hamartomatous
Cancer. 2017 Jul;16(3):371-376. polyps. JPS is due to germline mutations
in BMPR1A and SMAD4. Individuals with
SMAD4 mutations are more likely to have pol-
Question 39 yps and cancer in the stomach and are at risk
A 27-year-old man presents for an episode of he- for hereditary hemorrhagic telangiectasias
matemesis. He reports a history of about 20 cu- (HHT). The incidence of upper GI cancers in
mulative colonic polyps starting at age 15 years. JPS is about 21 percent. Hamartomas in JPS
The pathology of these polyps shows glandular have characteristic glandular dilation with an
dilation and inflammatory cell infiltrates. The inflammatory infiltrate. Menetrier’s disease
patient reports no history of skin freckling, hair is a non-hereditary condition that presents
loss, nail changes or weight loss. His family his- with thickened gastric folds and characteristic
tory is significant for a father with gastric cancer foveolar hyperplasia with tortuous (cork-
age 45 years, paternal uncle with gastric cancer screw) and cystically dilated foveolar glands
age 48 years, paternal uncle with colon cancer on histology. Cronkhite-Canada syndrome is
age 46 years. Upper endoscopy is performed and a non-hereditary syndrome characterized by
shows large polypoid masses in the gastric fun- hamartomas in the gastrointestinal tract with
dus (see image). protein-losing enteropathy, hair loss and nail
growth problems. Peutz-Jeghers syndrome
is a hereditary syndrome due to mutations
in the STK11 gene that predisposes to ham-
artomas, especially in the small intestine, as
well as numerous other cancers. Patients with
Peutz-Jeghers also have characteristic freck-
ling on the mouth and buccal mucosa. Gastric
lymphoma can present with thickened folds,
but the pathology is not consistent with this
diagnosis.

REFERENCE
Syngal S, Brand RE, Church JM, et al. ACG clini-
cal guideline: Genetic testing and management
of hereditary gastrointestinal cancer syndromes.
Am J Gastroenterol. 2015 Feb;110(2):223-62.
416 Digestive Diseases Self-Education Program®

Question 40 such as Zollinger-Ellison syndrome and multiple


A 51-year-old female with history of hypothyroid- endocrine neoplasia 1 (MEN1). Types 3 and 4
ism presents for symptoms of epigastric abdomi- neuroendocrine tumors have poor prognosis and
nal discomfort and fatigue. She denies nausea, should be managed with surgical resection. Fundic
vomiting or weight loss. She does not have heart- gland polyps have dilated glands lined by normal
burn. Her sister died from gastric adenocarcinoma oxyntic mucosa. Spindle cells are characteristic of
at age 48 years. Laboratory evaluation shows gastrointestinal stromal tumors (GIST). Glandular
hemoglobin of 11 g/dL, ferritin 6 ng/mL, iron 18 dilation and inflammatory cell infiltrates are com-
mcg/dL and vitamin B12 100 ng/L. Upper endos- monly seen in juvenile hamartomatous polyps.
copy is performed and showed atrophic mucosa in
the fundus with a 3mm polyp (see image arrow). REFERENCES
ASGE Standards of Practice Committee. The role
of endoscopy in management of premalignant and
malignant conditions of the stomach. Gastrointest
Endosc. 2015 82(1): 1-8.
Shaib YH, Rugge M, Graham DY, Genta RM.
Management of gastric polyps: an endoscopy-
based approach. Clin Gastroenterol Hepatol. 2013
Nov;11(11):1374-84.

Question 41
What is the most likely finding on histological A 35-year-old female presented with rectal bleed-
examination of the polyp? ing. Colonoscopy showed approximately 25 pol-
yps throughout the colon (see images). Eighteen
A. Solid nests of monomorphic endocrine cells polyps ranging in size from eight to 12mm were
B. Dilated glands lined by normal oxyntic removed from the ascending colon. One 20mm
mucosa polyp was removed from the transverse colon.
C. Spindle cells Five polyps were removed from the descend-
D. Glandular dilation and inflammatory cell ing colon ranging in size from six to 10mm. One
infiltrates 25mm polyp was removed from the rectosigmoid
colon. The polyps in the ascending and descend-
CORRECT ANSWER: A ing colon were sessile serrated polyps. The polyp
in the transverse colon was a traditional serrated
RATIONALE polyp and the rectosigmoid polyp was a tubulo-
This patient has a small neuroendocrine tumor villous adenoma. (See two images on the follow-
(e.g., carcinoid) of the stomach in the setting ing page.)
of autoimmune atrophic gastritis and perni-
cious anemia. Neuroendocrine tumors appear What is the best next step in management?
as nests of endocrine cells and are derived from
enterochromaffin-like cells. Type 1 neuroendo- A. Refer for colectomy now
crine tumors are well-differentiated, often mul- B. Genetic testing for APC mutation now
tifocal and are associated with chronic atrophic C. Colonoscopy in six to 12 months
gastritis. Small tumors, as found in this patient, D. FIT-DNA stool testing in six to 12 months
can be resected endoscopically. Type 2 neuroen-
docrine tumors are associated with conditions CORRECT ANSWER: C
Chapter 14 — Gastrointestinal cancers 417

RATIONALE dice. He reports weight loss of 10lbs over the last


This patient meets World Health Organization six months. Computed tomography shows a mass
(WHO) criteria for serrated polyposis syndrome, in the head of the pancreas with dilation of the
which include: 1) at least five serrated polyps common bile duct. The patient reports a history of
proximal to the sigmoid, with two or more 10 mm multiple moles on his back for which he has regu-
or more; 2) any serrated polyps proximal to the lar skin examinations. His family history is no-
sigmoid with family history of serrated polyposis table for a mother with melanoma at age 48 years
syndrome; or 3) more than 20 serrated polyps of and a maternal grandfather with melanoma age
any size throughout the colon. A clear genetic basis 55 years. The patient reports no personal or family
for this syndrome has not been identified. The history of pancreatitis. The patient is referred for
majority (80 percent) of sessile serrated polyps genetic testing.
are located in the proximal colon. Serrated polyps
with cytologic dysplasia carry significant malignant In which gene is a mutation likely to be found?
potential. Traditional serrated polyps are more
commonly found in the distal colon and also carry A. BRCA1
significant malignant potential. Patient with ser- B. PRSS1
rated polyposis syndrome can have different types C. MLH1
of serrated lesions as well as adenomatous polyps. D. CDKN2A
Current recommendations are to perform colonos-
copy surveillance with polypectomy every six to 12 CORRECT ANSWER: D
months depending on polyp burden. Colectomy is
reserved for patients in whom endoscopic surveil- RATIONALE
lance is not possible. There is no role for FIT-DNA This patient has familial atypical multiple mela-
stool testing in serrated polyposis syndrome. noma and mole (FAMMM) syndrome due to a
mutation in CDKN2A (also known as p16). FAM-
REFERENCE MM is characterized by multiple atypical moles as
Lieberman DA, Rex DK, Winawer SJ, et al. Guide- well as increased risk of melanoma and pancreatic
lines for colonoscopy surveillance after screening cancer. The risk of pancreatic cancer in FAMMM
and polypectomy: A consensus update by the US is estimated to be 13-39-fold increase over the
Multi-Society Task Force on Colorectal Cancer. general population. Patients with mutations in the
Gastroenterology 2012; 143:844. other genes listed here also have increased risk
of pancreatic cancer, but the personal and family
history is most suggestive of FAMMM. The risk of
Question 42 pancreatic cancer in BRCA1 mutation carriers is
A 57-year-old man presents with painless jaun- approximately two-fold increase over the general
418 Digestive Diseases Self-Education Program®

population. Mutations in PRSS1 are associated with Numerous studies confirm an association between
autosomal hereditary pancreatitis, a syndrome that new-onset non-insulin-dependent diabetes and
also carries increased risk of pancreatic cancer, development of pancreatic cancer, especially in
but will present at an earlier age with recurrent individuals who are elderly, have low BMI, experi-
pancreatitis. Lynch syndrome due to mutations in ence weight loss or do not have a family history
mismatch repair genes carries an estimated nine to of diabetes. The other tests would not be the best
11-fold increased risk of pancreatic cancer. choice for evaluation of the pancreas.

REFERENCE REFERENCE
Syngal S, Brand RE, Church JM, et al. ACG clini- National Comprehensive Cancer Network. Pancre-
cal guideline: Genetic testing and management of atic Adenocarcinoma (Version 2.2018). https://
hereditary gastrointestinal cancer syndromes. Am www.nccn.org/professionals/physician_gls/pdf/
J Gastroenterol. 2015 Feb;110(2):223-62 pancreatic.pdf Accessed September 24, 2018

Question 43 Question 44
A 62-year-old female developed a painful and A 34-year-old male with a history of ulcerative
swollen nodule on her right calf that resolved colitis diagnosed four years ago and maintained
spontaneously. She had recurrence of a similar on 5-ASA presents with right upper quadrant
nodule on her left calf three months later. She pain. Laboratory evaluation shows elevated
reports generalized fatigue, bloating and eight alkaline phosphatase with otherwise normal liver
lbs. weight loss in last three months. Her labora- chemistries. MRCP shows features consistent with
tory tests are normal, except for a fasting glucose primary sclerosing cholangitis. When should the
of 110. She denies jaundice, nausea or vomiting, patient have his next colonoscopy?
dysphagia or change in her bowel habits. She had
a colonoscopy that was normal two years ago. She A. Now
has no family history of cancer. B. One year
C. Four years
What is the next best step in her management? D. Six years

A. Colonoscopy CORRECT ANSWER: A


B. Computed tomography of the abdomen/pelvis
C. ERCP RATIONALE
D. EGD IBD with primary sclerosing cholangitis (PSC) is
associated with a four to five-fold increased risk
CORRECT ANSWER: B of colon cancer compared to IBD patients without
PSC. Colon surveillance is recommended to start
RATIONALE in PSC patients with colitis at the time of the PSC
The patient should have imaging of the pancreas diagnosis. Therefore, the correct answer is that
given the presentation of weight loss, vague gas- this patient should undergo colonoscopy now.
trointestinal symptoms, elevated blood sugar and Colonoscopy should be repeated yearly, preferably
Trousseau’s syndrome, suggestive of pancreatic with chromoendoscopy.
adenocarcinoma. Trousseau’s syndrome is a para-
neoplastic condition characterized by migratory REFERENCE
superficial thrombophlebitis. This syndrome is as- Lindor KD, Kowdley KV, Harrison ME; American
sociated with pancreatic, gastric and lung cancer. College of Gastroenterology. ACG Clinical Guide-
Chapter 14 — Gastrointestinal cancers 419

line: Primary Sclerosing Cholangitis. Am J Gastro- on Colorectal Cancer. Gastroenterology. 2017


enterol. 2015 May;110(5):646-59 Jul;153(1):307-323.
Imperiale TF, Ransohoff DF, Itzkowitz SH,
Levin TR, Lavin P, Lidgard GP, Ahlquist DA,
Question 45 Berger BM. Multitarget stool DNA testing for
Which of the following statements is true regard- colorectal-cancer screening. N Engl J Med. 2014
ing colorectal cancer screening? Apr 3;370(14):1287-97.

A. Colonoscopy every 10 years is recommended


as the only tier 1 option Question 46
B. Fecal immunochemical test (FIT) should be A 54-year-old female undergoes a screening colo-
performed every three years noscopy that shows a 18mm polyp in the ascend-
C. FIT-fecal DNA has higher sensitivity and ing colon (see image). The polyp is removed using
lower specificity compared to FIT alone submucosal saline injection and hot snare. Pathol-
D. Adenoma detection rate should be 15 percent ogy of the polyp shows a saw-tooth appearance of
or more overall. the epithelium with basal crypt dilation and boot-
shaped crypts without cytologic dysplasia.
CORRECT ANSWER: C
Which of the following is true about this polyp?
RATIONALE
The correct statement is that FIT-fecal DNA A. It harbors a KRAS mutation
has higher sensitivity and lower specificity com- B. It has no malignant potential
pared to FIT testing alone. In a large colonoscopy C. It is due to CpG island methylation
screening study, the sensitivity of one time FIT- D. It is most commonly located in the distal colon
fecal DNA for colorectal cancer was 92 percent,
whereas sensitivity of FIT testing alone in this CORRECT ANSWER: C
study was 74 percent. Specificity of FIT-fecal DNA
was 89.8 percent in those with normal colonos- RATIONALE
copy compared with 95 percent for the FIT alone. Serrated colorectal lesions have been more re-
Colonoscopy every 10 years and annual FIT test- cently recognized as precancerous. Serrated lesions
ing every year are recommended as tier 1 screen- are thought to account for 30 percent of colorectal
ing options based on the 2017 Multi-Society Task cancers. There are several types of serrated lesions:
Force recommendations. FIT testing should be hyperplastic, sessile serrated polyps (also known
performed annually, while FIT-fecal DNA is rec- as sessile serrated adenomas) with or without
ommended to be repeated every tjree years. Based cytologic dysplasia, and traditional serrated
on current recommendations, adenoma detection adenomas. Hyperplastic polyps are not consid-
rate in average risk screening patients should be ered premalignant, whereas sessile serrated and
25 percent or more overall or 30 percent or more traditional serrated lesions are premalignant. In
in men and 20 percent or more in women. this case, the patient has a sessile serrated polyp.
Sessile serrated polyps are more commonly found
REFERENCE in the proximal colon, have BRAF mutations and
Rex DK, Boland CR, Dominitz JA, Giardiello CpG island methylation. They do not have KRAS
FM, Johnson DA, Kaltenbach T, Levin TR, Li- mutations. The endoscopic appearance is sessile
eberman D, Robertson DJ. Colorectal Cancer or flat with few or no blood vessels, and sometimes
Screening: Recommendations for Physicians and a mucus cap, features that make these lesions more
Patients From the U.S. Multi-Society Task Force difficult to detect with colonoscopy.
420 Digestive Diseases Self-Education Program®

REFERENCE positive for CD117 and harbor KIT mutations.


Rex DK, Boland CR, Dominitz JA, Giardiello Malignant potential and decreased survival is
FM, Johnson DA, Kaltenbach T, Levin TR, Li- associated with size more than two cm and high
eberman D, Robertson DJ. Colorectal Cancer mitotic index (more than 5/50 high power field).
Screening: Recommendations for Physicians and EUS with tissue sampling is the preferred diagnos-
Patients From the U.S. Multi-Society Task Force tic technique. High-risk features include lobu-
on Colorectal Cancer. Gastroenterology. 2017 lated or irregular borders, invasion into adjacent
Jul;153(1):307-323. structures and heterogeneity. Fine needle aspirate
Rex DK , Ahnen DJ , Baron JA et al. Serrated (FNA) may be suboptimal and core biopsy is an
lesions of the colorectum: review and recommen- acceptable alternative. Resection is indicated for
dations from an expert panel . Am J Gastroenterol lesions that are symptomatic, size more than two
2012; 107: 1315 – 29. cm or high-risk EUS features. Lesions less than
two cm without high risk features can be surveyed
by EUS annually. Endoscopic resection might be
Question 47 possible for small lesions, but should be done in
A 65-year-old man undergoes upper endoscopy specialized centers. Metastatic or recurrent lesions
for epigastric discomfort. The exam is normal, are treated with imatinib.
except for a 3cm submucosal mass in the body
of the stomach. Endoscopic ultrasound shows REFERENCE
that the mass arises from the fourth layer of the ASGE Standards of Practice Committee. The role
stomach wall. Computed tomography of the abdo- of endoscopy in management of premalignant and
men confirms the solid gastric mass with several malignant conditions of the stomach. Gastrointest
small lesions in the liver concerning for metastatic Endosc. 2015 82(1): 1-8.
disease. Biopsy of the mass shows CD117 positive
spindle cells.
Question 48
Which of the following is true about this tumor? Which of the following is an indication for H.
pylori testing?
A. Small intestine is the most common location
B. KIT negative A. Family history of gastric cancer
C. Worse prognosis for tumors located in the B. Endoscopic resection of early gastric cancer
stomach C. Enteropathy-associated t cell lymphoma
D. Treatment for recurrent or metastatic disease D. Familial adenomatous polyposis
is imatinib
CORRECT ANSWER: B
CORRECT ANSWER: D
RATIONALE
RATIONALE H. pylori infection is associated with gastric
This patient has a gastrointestinal stromal tu- adenocarcinoma and gastric mucosal associated
mor (GIST) of the stomach. GISTs are the most lymphoid tissue (MALT) lymphoma (now called
common mesenchymal tumor found most com- “marginal zone B-cell lymphoma of MALT type”).
monly in the stomach. Gastric GISTs have a better Current guidelines recommend testing and treating
prognosis than those found in the small intestine. for H. pylori in patients with low-grade MALT lym-
GISTs are often found incidentally, but can cause phoma or a history of endoscopic resection of early
symptoms such as bleeding due to ulceration. gastric cancer. There is insufficient evidence to
Pathology of a GIST shows spindle cells that stain support screening for H. pylori in family members
Chapter 14 — Gastrointestinal cancers 421

of patients with gastric cancer. Enteropathy-asso- United States and Western Europe, rates of squa-
ciated T cell lymphomas are associated with celiac mous cell carcinoma have declined, while rates of
disease, not H. pylori. Finally, patients with famil- adenocarcinoma are increasing. Tumor length of
ial adenomatous polyposis have an increased risk of less than or equal to two cm has been shown to be
gastric cancer and screening with upper endoscopy a predictor of long-term survival. Endoscopic ul-
is warranted, though there are no recommenda- trasound with fine needle aspirate has been shown
tions to screen for H. pylori in these patients. to be more sensitive for nodal staging of esopha-
geal cancer than computed tomography.
REFERENCE
Chey WD, Leontiadis GI, Howden CW, Moss SF. REFERENCE
ACG Clinical Guideline: Treatment of Helico- National Comprehensive Cancer Network. Esoph-
bacter pylori Infection. Am J Gastroenterol. 2017 ageal Cancer (Version 3.2018). https://www.
Feb;112(2):212-239. nccn.org/professionals/physician_gls/pdf/esoph-
ageal.pdf Accessed September 24, 2018.

Question 49
A 42-year-old man presents with dysphagia for Question 50
three months. He reports progressive symptoms Which of the following conditions is NOT associ-
primarily with solid food. He also reports a 10 lb. ated with an increased risk of small bowel adeno-
weight loss. He has no history of heartburn. He carcinoma?
has a 10 pack-year smoking history but quit 10
years ago. His BMI is 35 kg/m2. Upper endoscopy A. Lynch syndrome
reveals a mass in the distal esophagus. The mass B. Crohn’s disease
is four cm in length with a diameter of 1.2 cm. C. Celiac disease
Biopsies confirm adenocarcinoma. D. Familial adenomatous polyposis
E. Common variable immune deficiency
Which of the following is true about his condition?
CORRECT ANSWER: E
A. Tumor diameter is a predictor of long-term
survival RATIONALE
B. EUS-FNA is more sensitive for nodal staging Adenocarcinoma of the small intestine accounts
than computed tomography for 30-40 percent of small bowel cancers. Other
C. Rates of esophageal adenocarcinoma are de- small bowel malignant tumors include neuro-
creasing endocrine tumors, lymphomas, and sarcomas.
D. Tobacco use is the strongest risk factor Risk factors for small bowel adenocarcinoma
include Lynch syndrome, Crohn’s disease, celiac
CORRECT ANSWER: B disease and familial adenomatous polyposis. The
relative risk for small bowel adenocarcinoma in
RATIONALE patients with Crohn’s disease of the small bowel
Esophageal cancers are classified as squamous cell is about 30. In celiac disease, the relative risk of
carcinoma or adenocarcinoma. Tobacco and alco- small bowel adenocarcinoma is 60-80-fold. Celiac
hol use are strong risk factors for squamous cell disease is also associated with enteropathy-asso-
carcinoma, whereas obesity, high body mass index ciated T cell lymphomas. Patients with inherited
and Barrett’s esophagus are strong risk factors cancer syndromes including Lynch syndrome and
for adenocarcinoma. Tobacco use is considered a FAP are also at risk for small bowel adenocarci-
moderate risk factor for adenocarcinoma. In the noma. There is a 100-fold increased risk in Lynch
422 Digestive Diseases Self-Education Program®

syndrome, and FAP is also associated with a 50-90 indications. It is important to identify lesions
percent lifetime risk of duodenal adenomas, of with malignant potential. In this case, the le-
which 3-10 percent will develop into adenocar- sion is most consistent with a mucinous cystic
cinoma. Common variable immunodeficiency neoplasm (MCN) that has malignant potential.
(CVID) is associated with increased risk of small MCNs are found almost exclusively in middle-
bowel lymphoma, but has not been identified as a aged women. They are most commonly located
risk factor for adenocarcinoma. in the body or tail of the pancreas and typically
do not communicate with the pancreatic duct.
REFERENCES Fluid aspirate on MCNs shows high CEA and
Pan SY, Morrison H. Epidemiology of cancer of variable amylase levels. Pathology shows co-
the small intestine. World J Gastrointest Oncol. lumnar epithelium surrounded by ovarian-type
2011 Mar 15;3(3):33-42. stroma. Resected MCNs without cancer do not
Syngal S, Brand RE, Church JM, et al. require postoperative surveillance based on a
ACG clinical guideline: Genetic testing and large systematic review that showed no syn-
management of hereditary gastrointestinal chronous lesions or recurrence in the absence of
cancer syndromes. Am J Gastroenterol. 2015 invasive cancer. Therefore, there is no role for
Feb;110(2):223-62. imaging or EUS. Serum CEA levels are not used
for diagnosis or surveillance of MCNs.

Question 51 REFERENCES
A 62-year-old female presents with bilateral lower Elta GH, Enestvedt BK, Sauer BG, Lennon AM.
abdominal pain and constipation. Computed ACG Clinical Guideline: Diagnosis and Man-
tomography of the abdomen and pelvis was nor- agement of Pancreatic Cysts. Am J Gastroen-
mal except for a five cm cystic lesion in the tail of terol. 2018 Apr;113(4):464-479.
the pancreas. The patient underwent EUS that Nilsson LN, Keane MG, Shamali A et
confirmed a five cm cystic lesion with septations al. Nature and management of pancreatic
that did not appear to be in communication with mucinous cystic neoplasm (MCN): a system-
the pancreatic duct. Fluid aspiration of the cyst atic review of the literature. Pancreatology
showed CEA of 490ng/mL and amylase of less 2016;16:1028–36.
than250U/L. The patient underwent surgical resec-
tion of the lesion and pathology showed columnar
epithelium surrounded by ovarian-type stroma Question 52
without high-grade dysplasia or pancreatic cancer. Which of the following conditions is NOT
associated with increased risk of
What is the best next step in management? cholangiocarcinoma?

A. MRI/MRCP in 12 months A. Cholelithiasis


B. Endoscopic ultrasound in 12 months B. Lynch syndrome
C. No further surveillance C. Primary sclerosing cholangitis
D. Serum CEA level in six months D. Choledochal cyst
E. Liver fluke infection
CORRECT ANSWER: C
CORRECT ANSWER: A
RATIONALE
Pancreatic cystic lesions are increasingly found RATIONALE
incidentally on imaging studies done for other A number of conditions are associated with in-
Chapter 14 — Gastrointestinal cancers 423

creased risk of cholangiocarcinoma. The lifetime


risk of biliary cancer in Lynch syndrome is esti-
mated to be one to four percent in individuals with
mutations in MLH1 and MSH2. Primary scleros-
ing cholangitis (PSC) carries a six to 36 percent
lifetime risk. Choledochal cysts, especially type I
(solitary, extrahepatic) and type IV (extra- and in-
trahepatic), increase the risk of cholangiocarcino-
ma with a lifetime risk of 6-30 percent. Liver fluke
infections with Opisthorchis viverrini (O. viver-
rini) and Clonorchis sinensis (C. sinensis) are risk
factors for cholangiocarcinoma. These infections
are most common in Southeast Asia. Cholelithiasis
has not been identified as a risk factor for cholan-
giocarcinoma.

REFERENCE
Tyson GL, El-Serag HB. Risk factors for cholan-
giocarcinoma. Hepatology. 2011 Jul;54(1):173-84.
424 Digestive Diseases Self-Education Program®
Answers & critiques

CHAPTER 15

Nutrition, obesity and


eating disorders
Matthew R. Kappus, MD, Matthew Berger, MD,
Christina Ha, MD, Melissa G. Teitelman, MD, AGAF,
and Susan Ramdhaney MD AGAF

Question 1 Syndrome: Making the Most of What the Patient


In adult short bowel patients, a bowel length of Still Has. 2005;72(9)833-38.
less than this is highly predictive of permanent
intestinal failure?
Question 2
A. 30 cm A 59-year-old male presents to the emergency
B. 50 cm room with epigastric abdominal pain and nausea
C. 100 cm for the last 72 hours. Upon presentation, he
D. 125 cm is alert and oriented with a blood pressure of
E. 200 cm 163/88 and a heart rate of 115. He has normal
respiratory rate. Laboratory evaluation reveals
CORRECT ANSWER: C a lactate of 3.5 mmol/L, white blood cell count
of 19.4 g/dL, hemoglobin of 15.1 g/dL, Na 130
RATIONALE meq/l, potassium 3.0 meq/l, AST 83 IU/L, ALT
In adult short bowel patients, a small bowel 34 IU/L, total bilirubin 1.6 mg/dL. He is found
length of less than 100cm is highly predictive to have a lipase 520 U/L. The patient is admitted
of permanent intestinal failure. The presence and resuscitated with intravenous fluids using
of terminal ileum and/or colon in continuity goal directed therapy. You recommend which
enhances both weaning off parenteral nutrition nutritional intervention?
and survival probabilities. After two years of
parenteral nutrition, probability of permanent A. Nothing per oral in order to allow for
intestinal failure is as high as 94 percent. This pancreatic rest
rate may lead to selection of other treatments, B. Enteral feeding prepylorically
especially intestinal transplantation, instead of C. Enteral feeding postpylorically
parenteral nutrition, for permanent intestinal D. Total parenteral nutrition
failure caused by short bowel syndrome.
CORRECT ANSWER: B
REFERENCES
Platell, C, Coster J, et al. The Management of RATIONALE
patients with Short Bowel Syndrome. World This patient has acute pancreatitis, Enteral
Journal of Gastroenterology 2002;8(1)13-20. nutrition is preferred to parenteral nutrition
Parekh, N, Seidner D. Managing Short Bowel because it leads to significantly better glyce-

425
426 Digestive Diseases Self-Education Program®

mic control, decreases infectious complications, through dietary intake. Iron deficiency is an effect
reduces the need for surgery, and reduces mortal- of untreated celiac disease.
ity. As to the site at which to feed, the alternatives
include postpyloric (mainly, nasojejunal) and pre- REFERENCE
pyloric (nasogastric) tube placement. The former Shepherd SJ, Gibson PR. Nutritional Inadequacies
usually requires the assistance of an endoscopist of the Gluten-Free Diet in Both Recently-diag-
or a radiologist, and this may result in a delay in nosed and Long-term Patients with Celiac Disease.
commencing enteral nutrition. This delay may Journal of Human Nutrition and Dietetics. 2012;
have an impact on the clinical outcome because 26, 349-358.
it is now believed that enteral nutrition should
commence as soon as possible after adequate fluid
resuscitation in order to maximize clinical benefit. Question 4
In contrast, a nasogastric feeding tube can usually A 57-year-old Caucasian female presents with
be inserted immediately and with ease, such that stable, decompensated cirrhosis due to primary
prepyloric feeding can be started without delay. biliary cholangitis. Her cirrhosis is complicated
by large ascites managed with 20 mg of furose-
REFERENCE mide and 50 mg of spironolactone. She has small
Crockett S, Wani S, et al. American Gastro- esophageal varices, and hepatic encephalopathy
enterological Association Institute Guideline controlled with rifaximin and lactulose. She also
on Initial Management of Acute Pancreatitis. has a history of osteoporosis and chronic kidney
2018;154(4)1096-1101. disease with a creatinine of 1.6 mg/dL. Aside from
her chronic autoimmune illnesses, she has contin-
ued to try and optimize her health, and has a body
Question 3 mass index (BMI) of 28kg/m2. She wants to know
Patients with celiac disease, subscribing to a strict more about how to optimize what she is eating.
gluten free diet are at particular risk for which The recommended dietary energy and protein
nutrient deficiency? intake for this patient is:

A. Fiber A. 8 to 10 kcal/kg per day and 1.2 to 1.5 g/kg pe


B. Protein day protein
C. Folate B. 25 to 35 kcal/kg per day and 1.2 to 1.5 k/kg
D. Thiamine per day protein
E. Iron C. 25 to 30 kcal/kg per day and 0.8 to 1.2 g/kg
per day protein
CORRECT ANSWER: D D. 25 to 30 kcal/kg per day and 1.2 to 1.5 g/kg
per day protein
RATIONALE E. 35 to 40 kcal/kg per day and 1.2 to 1.5 g/kg
Deficient intake of fiber and folate may originate per day protein
in the food choice of the individual, whereas some
deficiencies of intake, such as thiamine, appear CORRECT ANSWER: E
to be celiac-specific. The provider should encour-
age intake of nutrient-dense foods including RATIONALE
wholegrain foods, enriched if possible, legumes, It is vital to recognize the risk of malnutrition in
fruits, vegetables, lean meat, fish, chicken and patients with advanced stage liver disease, and a
eggs. It is not necessary to prioritize micronutrient primary goal should be to maintain weight and
supplements over achieving nutritional adequacy sustain a balanced diet. Ideally, a diet with an
Chapter 15 — Nutrition, obesity and eating disorders 427

energy intake of 35 to 40 kcal/kg and a protein A. As soon as she wakes up in the morning
intake of 1.2 to 1.5 g/kg per day protein is recom- B. Around lunchtime
mended. Lower calorie requirements, like 8-10 C. Before dinner
kcal/kg per day are believed to be safe in criti- D. Evening snack before bedtime
cally ill, hospitalized patients. Caloric intake for
patients with a body mass index of less than 30 CORRECT ANSWER: D
is recommended to be between 35 and 40 kcal/
kg per day, with opportunity to modify to a lower RATIONALE
caloric goal of 25-35 kcal/kg/day in patients with This question was evaluated in a trial by admin-
a BMI of 30-40, and 20-25 kcal/kg/day in patients istering a high protein nutritional supplementa-
with a BMI greater than 40. The increased protein tion to patients with cirrhosis from either 9 a.m.
goal of 1.2 to 1.5 g/kg per day is recommended to until 7 p.m. or from 9 p.m. to 7 a.m. Patients were
maintain an otherwise negative nitrogen balance administered the same supplement with caloric,
in patients with cirrhosis. This same recommenda- carbohydrate, and protein balance. Total body
tion is made for all patients with cirrhosis, regard- protein was evaluated at three, six and 12 months.
less of body mass index. Dietary protein does not There was not statistically significant improve-
contribute to hepatic encephalopathy, however ment in total body protein in patients adminis-
patients with cirrhosis demonstrate evidence of tered the supplement during the daytime, but
increased lean body mass loss. Healthy people those patients who received the supplementation
require 25-30 kcal/kg/day and 0.8-1.2 g/kg per in the evening experienced a statistically signifi-
day of protein. cant 2kg improvement in muscle mass over the 12
months of the study.
REFERENCE
EASL Clinical Practice Guidelines on Nutrition REFERENCE
in Chronic Liver Disease. European Association Tsuchia M, Sakaida I, et al. The Effect of a Late
for the Study of the Liver. Journal of Hepatology. Evening Snack in Patients with Liver Cirrhosis.
2019. 70, 172-193. Hepatology Research. 2005; 31(2): 95-103.

Question 5 Question 6
A 57-year-old Caucasian female presents with A 64-year-old man with amyotrophic lateral
stable, decompensated cirrhosis due to primary sclerosis had a PEG tube placed 12 hours ago. A
biliary cholangitis. Her cirrhosis is complicated routine KUB film shows evidence of air under the
by large ascites managed with 20 mg of furose- diaphragm. The team wants to call surgery for
mide and 50 mg of spironolactone. She has small further management of a perforation and leak.
esophageal varices, and hepatic encephalopathy The patient has a white blood cell count of 6.7 g/
controlled with rifaximin and lactulose. She also dL, hemoglobin of 11.2 g/dL, and a normal lactate.
has a history of osteoporosis and chronic kidney What is your next step?
disease with a creatinine of 1.6 mg/dL. Aside from
her chronic autoimmune illnesses, she has contin- A. Remove the PEG tube
ued to try and optimize her health, and has a body B. Call the surgical team for urgent exploratory
mass index (BMI) of 28kg/m2. laparotomy
C. Call the gastroenterology team for an urgent
When should you recommend that this patient upper endoscopy
consume a healthy snack such as a complex carbo- D. Perform a CT scan with oral contrast
hydrate or protein-based snack? E. Monitor patient’s clinical status
428 Digestive Diseases Self-Education Program®

CORRECT ANSWER: E hypergranulation tissue is level with the surround-


ing skin or the wound edge begins to open.
RATIONALE
Pneumoperitoneum is common after insertion of REFERENCE
an enterally placed PEG tube, occurring in 40-56 Hucl T, Spicak J. Complications of Percutaneous
percent of cases. Other risks include acute gas- Endoscopic Gastrostomy. Best Pract Res Clin Gas-
tric distension, prolonged ileus in less than three troenterology. 2016; 30(5): 769-781.
percent of cases. This is a common finding that
usually resolves over time. The only intervention
is to continue monitoring your patient. Question 8
You are called to examine the access site for a pa-
REFERENCE tient started on peripheral parenteral nutrition for
Hucl T, Spicak J. Complications of Percutaneous a patient with an enterocutaneous fistula. He has
Endoscopic Gastrostomy. Best Pract Res Clin Gas- a peripherally inserted intravenous catheter for
troenterology. 2016; 30(5): 769-781. access. He is prescribed a 1100 mOsm/L solution.
His IV site develops erythema and pain. An ultra-
sound is performed and has difficulty identifying
Question 7 the vein. You suspect the following has occurred:
A 77-year-old man who lives in a nursing home is
brought to your office by ambulance for administra- A. The patient has an insertion site infection
tion of care to his PEG site. The nursing home staff B. The patient has bacteremia and sepsis
notes that there is moisture with clear yellow dis- C. He has developed vessel thrombosis
charge under the bandage. There is mild macera- D. He has had an allergic reaction to the TPN
tion to the skin underneath a bandage that has been solution
placed over the PEG tube. Upon examination of the
site, there is a pink, edematous mucosal-appearing CORRECT ANSWER: C
tissue originating from within the PEG site. There
is no bleeding, there is no purulent drainage or foul RATIONALE
smell. The external bumper is loose, and there is no The patient has developed vessel thrombosis as
evidence of an enlarged the hole. indicated by pain, erythema, and phlebitis. Fac-
tors such as parenteral solution concentration
How do you treat this man’s problem? can greatly impact the incidence of vessel throm-
bosis in peripheral veins. Peripheral lines are
A. Do nothing and monitor clinically limited to parenteral solutions containing 900
B. Restart tube feeds mOsm/L or less, whereas central venous lines
C. Apply silver nitrate to the edematous tissue. can tolerate a more concentrated solution. Other
D. Remove the PEG tube and replace factors that contribute to phlebitis include inser-
E. Start antibiotics tion site, vein size, duration of insertion, cannula
size, material, and colonization. Peripheral lines
CORRECT ANSWER: C should be rotated every 48 to 72 hours if they are
going to be used.
RATIONALE
The patient has hypergranulation tissue originat- REFERENCE
ing from the orifice of his PEG tube. Silver nitrate Mirtallo J, Canada T, et al. Safe practices for par-
is an effective cautery agent, and the provider can enteral nutrition. JPEN J Parenteral Enteral Nutr.
repeat application every one to two days until the 2004;28(6):S39.
Chapter 15 — Nutrition, obesity and eating disorders 429

Question 9 CORRECT ANSWER: D


A 59-year-old man is admitted to the emergency
department with increased weakness. He is noted RATIONALE
to have a serum sodium of 126 meq/l and explains Individuals with inadequate sun exposure are at
that he drinks between 12 and 18 beers per day. risk for vitamin D deficiency; populations at risk
The first B vitamin deficiency to manifest in pa- include older adults, nursing home residents, dark
tients with alcohol overuse is: skinned individuals, people who wear clothing or
veils that expose little skin, people who are indoors
A. Niacin much of the day or use sunscreen daily and exclu-
B. Pantothenic Acid sively breastfed babies. Patients with extensive skin
C. Vitamin B6 damage from burns, fat-malabsorptive disorders,
D. Thiamin or renal disease are also at risk. In this stem, this
patient has several of these problems. One clini-
CORRECT ANSWER: D cal sign of vitamin D deficiency is tetany. This is
defined as continuous tonic spasm of a muscle with
RATIONALE steady contraction. There is not twitching involved.
Thiamin is stored in the liver, but only in small Vitamin A deficiency may manifest at follicular hy-
quantities. Due to this, when patients have malab- perkeratosis, xeropthalmia, night blindness, excess
sorption or eat a diet deficient in thiamin, this is bone deposition, impaired wound healing. Vita-
usually the first to become deficient. min A toxicity may manifest as pruritus, alopecia,
conjunctivitis, cheilitis, ataxia, bone loss/fracture,
REFERENCE hyperlipidemia. Vitamin E toxicity manifests as
Hahn JS, et al. Wernicke encephalopathy and bruising from decreased vitamin K absorption, in-
beriberi during total parenteral nutrition attribut- clusion bodies in bone marrow, thrombocytopenia,
able to multivitamin infusion shortage. Pediatrics. cerebral hemorrhage. Vitamin E deficiency may
1998;101(1):E10. manifest at ceroid pigmentation (age spots), vision
changes, ophthalmoplegia, ptosis, dysarthria, he-
molytic anemia, increased platelet aggregation.
Question 10
A 78-year-old, African American man is brought REFERENCE
in from his nursing home with severe spasms Bjelakovic G, et al. Antioxidant supplements for
in his lower legs. His medical history includes prevention of mortality in healthy participants and
chronic kidney disease, though not dialysis patients with various diseases. Cochrane Database
dependent, diabetes mellitus, and a previous Syst Rev. 2012.
stroke which has left him hemiplegic and wheel
chair bound. On physical exam, you note continu-
ous extension of both ankles without distinctive Question 11
twitching. Which vitamin or mineral abnormality What additive should be considered routinely for
is currently present? hemodynamically stable ICU patients being given
a standard enteral formula?
A. Vitamin A toxicity
B. Vitamin A deficiency A. Glutamine
C. Vitamin D toxicity B. Fermentable fiber
D. Vitamin D deficiency C. Arginine
E. Vitamin E toxicity D. DHA (docosahexaenoic acid)
F. Vitamin E deficiency E. EPA (eicosapentaenoic acid)
430 Digestive Diseases Self-Education Program®

CORRECT ANSWER: B C. Hypochloremic metabolic alkalosis


D. Hypochloremic metabolic acidosis
RATIONALE
Fermentable fiber has multiple benefits to the CORRECT ANSWER: C
healthy and poorly functioning intestine. Ferment-
able fibers include fructosoligosaccharides, gums, RATIONALE
and pectin. These are metabolized by colonic This patient likely has hyperemesis gravidarum.
bacteria to produce short chain fatty acids, 2- to 5- She is most at risk for hypochloremic metabolic
carbon fatty acids which include acetate, propio- alkalosis. Gastric fluid contains 130 mEq chloride
nate, and butyrate. Once absorbed by intestinal per liter, and is acidic with a pH of 1 to 2. With
mucosal cells, these fatty acids serve as an energy unrelieved emesis, the patient is at risk for both
source. Other effects include increased sodium chloride and acid loss, and the patient is left with
and water absorption, trophic effects, inhibition of an alkalosis.
cholesterol synthesis by the liver, improvement of
colonic and splanchnic circulation, acidification of REFERENCE
the colonic pH which lowers the solubility of bile Perez GO, Oster JR, Rogers A. Acid-base distur-
acids and reduces growth of pathogenic bacteria. bances in gastrointestinal disease. Dig Dis Sci.
Immune modulating enteral formulas contain- 1987;32(9):1033.
ing arginine, DHA, EPA may have a role in the
critically ill patient but the data does not support
routine use at this point. Glutamine exerts a tro- Question 13
phic effect by maintaining gut integrity but has not You are meeting a 36-year-old man with a his-
been shown to have a beneficial effect on mortal- tory of ulcerative colitis, that was diagnosed when
ity, infections or hospital length of stay. he was 12 years old. He has had two prior bowel
resection surgeries for this in his late teens and
REFERENCE early 20’s. His current symptoms include bloody
McClave SA, et al. Guidelines for the provision stools six to eight times per day, and his symptoms
and assessment of nutrition support therapy in the have not been well managed medically. In prior
adult critically ill patient: Society of Critical Care years, he has been treated with several rounds of
Medicine and American Society for Parenteral and steroids, and when not having hematochezia, he
Enteral Nutrition: Executive Summary. JPEN J has described bloating, and foul smelling stools.
Parenter Enteral Nutr. 2016 Feb;40(2):159-211. He has received antibiotics cycled for small in-
testinal bacterial overgrowth in the past and this
has helped. Testing is negative for Clostridium
Question 12 difficile, and biopsies of the colon indicate no
A 27-year-old woman presents with intractable cytopathic effect of cytomegalovirus. The patient is
nausea and vomiting . She is nine weeks preg- evaluated by surgery and is treated with a proc-
nant with her first pregnancy. She is otherwise a tocolectomy with creation of an ileal-pouch anal
healthy young woman, without prior medical or anastomosis with a loop ileostomy. The remaining
surgical history. She notes dizziness and light- small bowel length is greater than 200cm. The pa-
headedness with position change. tient will receive parenteral nutrition due to high
ostomy output, with plan for stomal closure in the
Which acid base disorder is she most at risk for? next three to six months. As the medical gastroen-
terologist, considering this patient’s recent surgery
A. Hyperchloremic metabolic alkalosis and need for parenteral nutrition, the best next
B. Hyperchloremic metabolic acidosis step is which of the following?
Chapter 15 — Nutrition, obesity and eating disorders 431

A. Start probiotics, provide oral rehydration solu- A. Vitamin A


tion, and diet high in starch and low in simple B. Vitamin C
sugars once the diet is advanced C. Vitamin E
B. Begin a diet high in soluble fiber with oral D. Vitamin K
rehydration solution E. Folate
C. Start therapy with immunomodulatory therapy
D. Start therapy with biologic-based therapy CORRECT ANSWER: A

CORRECT ANSWER: A RATIONALE


Vitamin A taken in excess of 10,000 IU/day or
RATIONALE 0.5 to 1.5 mg 12-cis retinoic acid (isotretinoin) per
In this scenario, as the gastroenterologist, your kilogram has been associated with morbidity dur-
responsibility is to rehabilitate this patient’s GI ing pregnancy. This is also true of retinol. Patients
tract. This patient had medically refractory ulcer- pregnant are encouraged to seek a well-balanced
ative colitis, and therefore definitive treatment diet as they are susceptible to deficiencies in Vita-
was performed with proctocolectomy. For this rea- min A, D, K and folic acid deficiencies. Inadequate
son, answers C and D are incorrect. Yes, answer A vitamin A intake is associated as well with growth
is correct, but this is not the best answer. You will restriction, eye abnormalities, and impaired vision
be monitoring for both electrolyte and fluid imbal- in children.
ances when the patient is on parenteral nutri-
tion. Parenteral nutrition is indicated because the REFERENCE
patient has high ostomy output, malabsorption, Academy of Nutrition and Dietetics. Practice
and poor nutritional status. The patient has prior Paper of the Academy of Nutrition and Dietetics
history of SIBO, and altered gut microbial diver- Abstract: Nutrition and Lifestyle for a Healthy
sity. IV nutrition will further compromise the gut Pregnancy Outcome 2014. http://www.eatright.
by reducing gut microbial diversity, which is why a org/members/practicepapers/ (Accessed on
probiotic like VSL 3 should be introduced immedi- March 23, 2016).
ately after surgery. The patient should also receive
information about oral rehydration solutions to
begin conditioning the gut to reabsorb fluid and Question 15
prevent dehydration. Dietary choices such as A 56-year-old man presents to your office and
avoidance of concentrated sugars and carbohy- labs indicate a serum sodium of 127 mEq/L with a
drates will also assist with reducing gas, bloating, blood glucose of 470 mg/dL.
diarrhea/ostomy output, and dehydration.
What is the corrected sodium level?
REFERENCE
Pironi L, et al. Fat-induced ileal brake in humans: A. 125 mEq/L
a dose-dependent phenomenon correlated to the B. 129 mEq/L
plasma levels of peptide YY. Gastroenterology. C. 130 mEq/L
1993;105(3):733-739. D. 133 mEq/L

CORRECT ANSWER: D
Question 14
During pregnancy this micronutrient taken in ex- RATIONALE
cess can lead to adverse effects on the pregnancy Remember, hyperosmolarity related to hypergly-
such as spontaneous abortion and birth defects: cemia shifts fluid from the intracellular space to
432 Digestive Diseases Self-Education Program®

the extracellular space. This compartmental shift Question 17


results in a dilutional decrease in sodium levels by Nutritional support should be considered for
1.6 mEq/L for every 100 mg/dL increase in serum patients who are unable to maintain adequate oral
glucose. The equation once can use is Corrected nutrition for how many days?
Serum Sodium = Measured Serum Sodium +
[0.016 x (Serum Glucose – 100)]. A. Three to five days
B. Seven to 14 days
REFERENCE C. 10-21 days
Daugirdas JT, Kronfol NO, Tzamaloukas AH, Ing D. Longer than 30 days
TS. Hyperosmolar coma: cellular dehydration and
the serum sodium concentration. Ann Intern Med CORRECT ANSWER: B
1989; 110:855.
RATIONALE
Nutritional support can be via the enteral route or
Question 16 parenteral route. The enteral route is preferred.
A 24-year-old woman with ileal Crohn’s disease Patients who particularly benefit from earlier nu-
presents with complaints of loose stools. Her trition therapy include those shown to be critically
Crohn’s disease was well-managed with infliximab ill—burn, liver disease, pancreatic disease, cystic
until two years ago when she developed a stenosis fibrosis, head and neck cancer, etc. Nutritional
in the distal small bowel and underwent a limited support should otherwise be considered for any
resection of her terminal ileum. Recent conserva- patient who is unable to maintain adequate oral
tive management of her diarrhea did not relieve nutrition for seven to 14 days.
her symptoms and she continues to have loose,
frequent stools. You recommend treatment with REFERENCE
the following: Souba WW. Nutrition Support. New England
Journal of Medicine. 1997; 336(1) 41-48.
A. Increased fiber intake
B. Diphenoxylate
C. Loperamide Question 18
D. Cholestyramine A 56-year-old man presents after a recent left
hepatectomy for hepatocellular cancer. His sur-
CORRECT ANSWER: D gery was performed seven days prior, and since
discharge, he notes progressive abdominal disten-
RATIONALE sion. A diagnostic paracentesis is performed which
This patient has ileal Crohn’s disease, where bile reveals a cloudy appearing fluid. The albumin in
salts are absorbed. Bile salt malabsorption usually the fluid is 2.9 g/dL, protein 3.6 g/dL, cell count
does not occur until greater than 100 cm of ileum and differential are 556 and predominantly lym-
is removed. When less than 100 cm of ileum is re- phocytes. Triglycerides are checked and are 430
moved, the excess bile salts can result in choleretic mg/dL. Serum albumin for the patient is 3.6 g/dL.
diarrhea and bile acid binders can be effective in
decreasing diarrhea. You suggest the following to treat this
patient’s ascites:
REFERENCE
Van Citters GW, Lin HC. Ileal brake: neuropepti- A. Serial large volume paracentesis
dergic control of intestinal transit. Curr Gastroen- B. Lasix and spironolactone
terol Rep. 2006;8(5):367. C. Sodium restriction
Chapter 15 — Nutrition, obesity and eating disorders 433

D. Consideration for placement of a trans jugular C. Potassium


intrahepatic portosystemic shunt D. Sodium
E. Treatment with medium chain triglycerides
CORRECT ANSWER: B
CORRECT ANSWER: E
RATIONALE
RATIONALE This patient has an eating disorder and is severely
This patient has chylous ascites based upon the malnourished. She is at risk for refeeding syn-
presence of low SAAG, high protein ascites that is drome. Due to a quick rise in glycolysis and ATP
high in triglycerides. The cell count is predomi- production, there is a dramatic demand for phos-
nantly lymphocytes. If this were ascites related to phorous. Patients at risk may become hypophos-
portal hypertension, the SAAG would be elevated phatemic, hypokalemic, and/or hypomagnesemic
(greater than 1.1), with a low protein (less than as the body is recruiting all of these minerals. Seri-
2.4 g/dL). The reason for the chylous ascites is ous complications of refeeding syndrome include
likely related to his recent surgery. Management pulmonary edema and congestive heart failure, as
of the underlying cause is always the first step well as sudden cardiac death.
in treating ascites whenever feasible. should be
placed on a high protein, low fat diet that is rich REFERENCE
in medium chain triglycerides (MCT). Dietary Friedeli N, et al. Revisiting the Refeeding Syn-
restriction of long-chain triglycerides (LCT) drome: Results of a Systematic Review. Nutrition.
avoids their conversion into monoglycerides and 2017; (35): 151-160.
free fatty acids (FFA), which are transported as
chylomicrons to the intestinal lymph ducts. By
contrast, MCTs are absorbed directly into intes- Question 20
tinal cells and transported as FFA and glycerol You are meeting a 32-year-old woman who carries
directly to the liver via the portal vein. Thus, a a diagnosis of pulmonary atresia with an intact
low-fat diet with MCT supplementation reduces ventricular septum and sinusoids. She underwent
the production and flow of chyle. lateral tunnel Fontan procedure as a child and has a
hypoplastic right ventricle. She has mild congestive
REFERENCE heart failure with NYHA class II symptoms, and
Ohri SK, et al. The Management of Postoperative swelling is treated with torsemide. She is referred
Chylous Ascites. A Case Report and Literature Re- to your office for management of Fontan associated
view. J Clinical Gastroenterology. 1990; (6): 693. liver disease and small volume abdominal ascites,
as well as multiple loose stools per day. She de-
scribes her stools as loose, sometimes greasy. While
Question 19 they are worsened by food intake, her bowels are
A 29-year-old woman is admitted with a body mass loose regardless of whether she has eaten. What do
index of 17kg/m2 and severe electrolyte abnormali- you suspect is the cause of her loose stools?
ties. She is found to have hypochloremic metabolic
alkalosis. She is admitted for fluid resuscitation, elec- A. Infection with clostridium difficile
trolyte monitoring, and enteral nutrition support. B. Crohn’s disease or ulcerative colitis not yet
Which if the following minerals requires close diagnosed
monitoring? C. Celiac disease
D. Small intestinal bacterial overgrowth due to
A. Calcium multiple surgeries
B. Phosphorous E. Protein losing enteropathy
434 Digestive Diseases Self-Education Program®

CORRECT ANSWER: E emesis, hematemesis, or melena. On examina-


tion, it is noted that she is hirsute with parotid
RATIONALE enlargement. Her body mass index is 23kg/m2.
Mucosal diseases without ulcerations, but with Aside from presentation being consistent with a
loss of mucosal integrity and barrier function Mallory-Weiss tear, you believe her underlying
that can result in protein-losing gastroenteropa- diagnosis is the following:
thy, include celiac disease, bacterial overgrowth,
microscopic colitis, viral or parasitic infections, A. Anorexia nervosa
amyloidosis, Menetrier’s disease, and Helico- B. Bulimia nervosa
bacter pylori gastritis. Cardiac diseases caus- C. Binge eating disorder
ing protein-losing gastroenteropathy include d. Acute viral gastroenteritis
constrictive pericarditis, valvular heart diseases, E. Food-borne illness
and cardiomyopathies. Repair of congenital
heart defects with the Fontan procedure has led CORRECT ANSWER: B
to protein-losing gastroenteropathy in about
10 percent of patients. Common presenting RATIONALE
symptoms and signs of PLE include diarrhea, Bulimia nervosa is characterized by not only
peripheral edema, abdominal pain and bloat- binge eating, but inappropriate, compensatory
ing, pleural effusions, pericardial effusions, behaviors to counteract the food intake. Individ-
and ascites. The intestinal protein loss seen in uals with bulimia nervosa typically have a nor-
PLE along with nutrient malabsorption leads to mal weight, as this patient does. Typical physi-
hypoalbuminemia, lymphopenia, hypogamma- cal signs include poor dentition, parotid gland
globulinemia, and failure to thrive. Aside from inflammation, callused knuckles, hematemesis
management with a high protein, low fat diet, with a Mallory-Weiss tear, hemorrhoids and
other treatment includes diuresis and manage- rectal prolapse from laxative abuse.
ment of heart failure, the use of budesonide and/
or octreotide. REFERENCE
Harrington BC, Jimerson M, Haxton C,
REFERENCES Jimerson DC. Initial evaluation, diagnosis,
Fujii T, et al. Fecal alpha1-antitrypsin concentra- and treatment of anorexia nervosa and bu-
tions as a measure of enteric protein loss after limia nervosa. American family physician.
modified fontan operations. J Pediatr Gastroen- 2015;91(1):46-52.
terol Nutr. 2003 Nov;37(5):577-80.
Martens L, et al. Protein-losing enteropathy
after the Fontan operation: an international Question 22
multicenter study. PLE study group. J Thorac In the fasting state, oxyntic cells in the fundus of
Cardiovasc Surg. 1998 May;115(5):1063-73. the stomach produce high levels of which hor-
mone to stimulate hunger?

Question 21 A. Cholecystokinin
A 24-year-old woman is being seen in the B. Leptin
emergency department with an acute episode C. Ghrelin
of hematemesis. She tells the admitting doc- d. Glucose-dependent insulinotropic
tor that her hematemesis occurred after several polypeptide
episodes of emesis. She denies nausea currently,
and she has not had any further episodes of CORRECT ANSWER: C
Chapter 15 — Nutrition, obesity and eating disorders 435

RATIONALE Stern JS, Hirsch J, Blair SN, et al. Weighing


Ghrelin is secreted from the oxyntic cells in the options: criteria for evaluating weight-
the fundus of the stomach during the state of management programs. The Committee to
fasting to stimulate hunger. Once food in in- Develop Criteria for Evaluating the Outcomes of
gested, gastric distension results in a decline Approaches to Prevent and Treat Obesity. Obe-
in ghrelin levels. sity research. 1995;3(6):591-604.

REFERENCE
Date Y, Murakami N, et al. The role of the gastric Question 24
afferent vagal nerve in ghrelin-induced feeding A 37-year-old man from central Tennessee
and growth hormone secretion in rats. Gastroen- presents with progressive worsening of abdom-
terology. 2002; 123 (4):1120. inal pain, nausea, and tingling of his mouth
after a heavy meal. He has had a difficult time
distinguishing which foods in particular give
Question 23 him these symptoms, but symptoms are typi-
What magnitude of weight loss has been shown cally experienced two to five hours after a large
to reduce obesity-related complications? meal. His diet is rich in meats, and he explains
that he is an avid outdoorsman and enjoys pre-
A. Five to 10 percent paring what he hunts. You belive his symptoms
B. 10-15 percent may be explained by exposure to which of the
C. 15-20 percent following ticks?
D. 25 percent or more
A. Deer tick
CORRECT ANSWER: A B. Wood tick
C. Lone star tick
RATIONALE D. Brown dog tick
Behavior modification remains the foundation
of obesity treatment and is crucial for long- CORRECT ANSWER: C
term weight loss maintenance. A modest weight
loss of five to 10 percent body weight has been RATIONALE
repeatedly shown to reduce obesity-related com-
plication. The AGA has published a white paper This patient likely has a recently recognized
detailing an approach to help gastroenterolo- food allergy sydrome known as Alpha-Gal al-
gists incorporate obesity management into their lergy. This is an IgE mediated hypersensitivity
practice that emphasizes support for patient, to the non-primate, mammalian carbohydrate
continuous assessment, intervention, and re- galactose-alpha-1,3-galactose. This is contracted
assessment. after a bite from the Lone star tick, Amblyomma
americanum. The tick introduces the alpah-gal
REFERENCES protein from animals which it has previous fed
Acosta A, Streett S, Kroh MD, et al. White Paper from. The hallmark is a hypersensitivity reaction
AGA: POWER - Practice Guide on Obesity and two to five hours post exposure, and includes
Weight Management, Education, and Resources. reactions to lamb, beef or pork.
Clinical gastroenterology and hepatology : the
official clinical practice journal of the American REFERENCE
Gastroenterological Association. 2017;15(5):631- Platts-Mills TA, Schuyler AJ, Hoyt AE, Com-
649.e610. mins SP. Delayed Anaphylaxis Involving IgE to
436 Digestive Diseases Self-Education Program®

Galactose-alpha-1,3-galactose. Current allergy and Which of the following is the next best step?
asthma reports. 2015;15(4):12.
A. Recommend low fat diet supplemented wit
medium chain triglyceride (MCT) oil
Question 25 B. Total Parenteral Nutrition
Which of the following is one of the best validated C. Obtain liver ultrasound
screening indicators for malnutrition risk? D. Placement of gastric feeding tube and start
isotonic formula
A. Patient reports unintentional weight loss E. Clear liquid diet
B. Patient reports following a low fat,
low carbohydrate diet CORRECT ANSWER: A
C. Patient has advanced liver disease
D. Patient reports a recent flu-like illness RATIONALE
Chylous ascites has been detected in approximate-
CORRECT ANSWER: A ly one to 6.6 percent of patients after colorectal
cancer surgery and up to 11 percent after pancre-
RATIONALE atic surgery. This is diagnosed with a triglyceride
The only well-validated measure of malnutrition in level greater than 110mg/dL, cholesterol level less
the above mentioned list is unintentional weight loss. than 200mg/dL, and presence of chylomicrons in
The other items can certainly be found in the history the peritoneal fluid.
of a patient with malnutrition, but are not in and of Conservative management of chylous ascites
themselves validated measures of malnutrition risk. consists of either high protein, low fat diet that is
rich in MCT oil or total parenteral nutrition. Both
REFERENCE options successfully resolve the chylous ascites in
Detsky AS, et al. What is Subjective Global Assess- a majority of patients. TPN is typically reserved
ment of nutritional status? JPEN Journal Paren- for those that cannot tolerate an oral diet or can-
teral Enteral Nutrition. 1987; 11(1):8-13. not meet their nutritional requirements orally or
enterally.
Medium chain triglycerides as opposed to long
Question 26 chain fats (LCF) reduce the production of chyle.
A 52-year-old man is status post a right hemico- Bile salts create micelles with LCF allowing easier
lectomy for colon cancer three weeks ago. He pres- access for pancreatic enzyme hydrolysis and ab-
ents to his follow up visit complaining of progres- sorption in the intestinal mucosa. Once absorbed,
sive abdominal distention. He does not complain the fatty acids and monoglycerides are re-esterified
of abdominal pain. On physical exam, his abdo- into triglycerides combining with cholesterol,
men is distended and there is a fluid wave and protein and other substances to form chylomicrons,
shifting dullness noted. A diagnostic paracentesis which enter the lymphatic system through lacteals.
is performed. The results are as follows: MCTs are directly absorbed into intestinal cells and
transported as free fatty acids and triglycerides to
Serum lactic dehydrogenase: 356 U/L the liver via the portal vein effectively bypassing the
Serum total protein: 6.0 g/dL lymph system. Parenteral lipids are phospholipids
Pleural fluid glucose: 42 mg/dL designed for intravenous infusion. They are deliv-
Pleural fluid lactic dehydrogenase: 242 U/L ered directly into the blood stream and as a result
Pleural fluid total protein: 2.9 g/dL do not pass through the lymph system as chyle.
Pleural fluid triglycerides: 210 mg/dL Therefore, intravenous lipids are not contraindi-
Pleural fluid cholesterol: 90 mg/dL cated in patients with chyle leaks.
Chapter 15 — Nutrition, obesity and eating disorders 437

REFERENCES RATIONALE
Sriram K, Meguid RA, Meguid MM. Nutritional Overfeeding causes increased CO2 production, in-
support in adults with chyle leaks. Nutrition creases the respiratory drive, and requires higher
2016;32(2):281-6. rates of intermittent mandatory ventilation on
Weniger M, D’Haese JG, Angele MK et al. mechanical ventilation.
Treatment options for chylous ascites after major The patient formula was providing enough
abdominal surgery: a systemic review. The Ameri- energy to meet 41.5kcal/kg (Normal maintenance
can Journal of surgery. 2016;211:206-213. caloric goal 25-35 kcal/kg).
Baek S-J, Kim S-H, Kwak J-M et al. Incidence This is calculated by the following formula:
and risk factors of chylous ascites after colorectal (Formula strength x rate x hours of feeding) /
cancer surgery. The American Journal of surgery. (patients’ weight (kg) (1.5 x 60cc/hr x 24 hours) /
2013;2016:555-559. 52 kg = 41.5 kcal/kg.
McCray S PC. When Chyle Leaks: Nutrition The respiratory quotient (RQ) obtained from indi-
Management Options. Practical Gastroenterology rect calorimetry (IC) is defined by the ratio of carbon
2004;17:60-76. dioxide production (VCO2 ) to oxygen consumption
(VO2 ). The physiologic range for the RQ exists be-
tween 0.67 to 1.3. Overfeeding results in lipogenesis,
Question 27 which increases the VCO2 and the RQ respectively.
A 66-year-old woman with emphysema is admit- A higher RQ leads to increases in minute ventilation,
ted to the medical intensive care for acute hypoxic alveolar ventilation and VO2. Those patients af-
and hypercapnic respiratory failure. She is 5 feet fected most are those who are nutritionally depleted,
4 inches tall and weighs 52 kg. ABG on admission stressed, or hypermetabolic, or those who have lim-
revealed the following: pH 7.24, pCO2 67, pO2 53, ited pulmonary reserve for eliminating CO2.
bicarb = 31. A CT of the chest showed multiple bi- In the absence of fever, the disease-specific
lateral patchy opacities consistent with multifocal variations in energy expenditure may be abolished
pneumonia. The patient was intubated, started on with deep sedation among medical and surgical
empiric vancomycin and cefepime, steroids, nebu- ICU patients.
lizer treatment, and midazolam for sedation. An
orogastric tube is placed and tube feeding is initi- REFERENCES
ated and brought to goal (Osmolite 1.5 @ 60cc/ Guenst JM, Nelson LD. Predictors of total par-
hr) by 72 hours of intubation. On day four, the enteral nutrition-induced lipogenesis. Chest
patient’s respiratory rate increases and starts to 1994;105:553-9.
breath over the ventilator. A chest x-ray shows the Liposky JM, Nelson LD. Ventilatory response
multifocal opacities have improved. Repeat ABG to high caloric loads in critically ill patients. Crit
shows pH 7.27, pCO2 64, pO2 75, bicarb = 34. Care Med 1994;22:796-802.
Haugen HA, Chan LN, Li F. Indirect calorime-
Which of the following is the next best step? try: a practical guide for clinicians. Nutr Clin Pract
2007;22:377-88.
A. Increase sedation A skanazi J, Rosenbaum SH, Hyman AI, et al.
B. Increase tidal volume Respiratory changes induced by the large glu-
C. Change antibiotics to cover for anaerobic bacteria cose loads of total parenteral nutrition. JAMA
D. Decrease feeding rate 1980;243:1444-7.
E. Check for residual tube feeding formula in Raurich JM, Ibanez J, Marse P, et al. Resting
stomach energy expenditure during mechanical ventilation
and its relationship with the type of lesion. JPEN J
CORRECT ANSWER: D Parenter Enteral Nutr 2007;31:58-62.
438 Digestive Diseases Self-Education Program®

Question 28 SMA syndrome is rare and it results from com-


A 21-year old woman, accompanied by her mother pression of the duodenum between the superior
presents to the emergency department complain- mesenteric artery and the aorta. In this patient,
ing of two weeks of nausea, vomiting and epigas- the compression results from the loss of fatty
tric pain. She describes sharp abdominal pain that tissue, which surrounds the superior mesen-
improves with vomiting. She denies fevers, recent teric artery and its neurovascular pedicle. This
travel, alcohol and illicit substance use. Upon can be a result of either weight loss or a rapid
questioning of both patient and mother, it was growth spurt. In the absence of an appropriate
discovered that she is on a specialized diet and fatty scaffolding, the angle at which the SMA
exercise regimen for her college track team. Her branches from the aorta is reduced resulting in
mother has noticed increased time in the bath- compression of the third portion of the duode-
room and frequent pacing around the home. Since num between the SMA and the aorta.
initiation of her regimen, she eventually reached The initial goal of management of SMA
a weight of 36 kg, down from her baseline of 55 syndrome is relief of the obstruction and correc-
kg. On physical examination, the patient appears tion of the underlying etiology. Known causes
cachectic. Her abdomen is mildly distended with are anatomic variants, mesenteric fat reduction
diffuse tenderness and decreased bowel sounds. (as in this patient), anatomic abnormalities due
Laboratory evaluation at admission included to trauma or surgery and consumptive disease.
electrolytes that were significant for a reduced After decompression via nasogastric (NG) tube
chloride (92 mmol/l) and an elevated bicarbonate placement and correction of electrolyte dis-
(32 mmol/l). Her amylase was 141 U/l, and lipase turbances, treatment in this patient consists
was 164 U/l, and her complete blood count and primarily of nutritional therapy. Nutritional
liver function studies were essentially normal. A repletion ranges from conservative attempts to
small bowel follow through with barium swallow increase weight with enteral tube feedings past
is performed that demonstrates a dilated stomach the site of obstruction or parenteral nutrition,
and inability of contrast to flow past the third por- to more aggressive surgical therapy. Surgical
tion of the duodenum. A nasogastric tube is placed therapy may involve duodenojejunoanastomosis
for decompression, which drains 120ml of bilous or gastrojejunoanastomosis aimed at bypass-
fluid and provides immediate relief. ing the area of obstruction. Alternative surgical
management may be removal of the ligament
Of the following recommendations, which is the of Treitz in order to relieve the obstruction and
most appropriate immediate course of action in allow greater intestinal mobility. Surgical man-
this patient? agement is necessary if medical management
does not relieve the obstruction, if a nasojejunal
A. Placement of nasojejunal feeding tube (NJ) tube cannot be passed distal to the com-
B. Surgical consultation pression, or if the compression is not relieved by
C. NPO and high infusion rate of lactated ringers repletion of fat stores.
D. Psychiatry consult The patient likely has a diagnosis of anorexia
E. CT abdomen/Pelvis with contrast nervosa given the very low BMI and excessive
purging. She warrants inpatient psychiatry
CORRECT ANSWER: A management, but only after management of her
acute medical issues.
RATIONALE The patient’s diagnosis was confirmed with the
The excessive weight loss from decreased oral small bowel follow through with barium swallow
intake and excessive purging techniques resulted and does not need additional imaging to proceed
in superior mesenteric artery (SMA) syndrome. with a nasojejunal feeding tube.
Chapter 15 — Nutrition, obesity and eating disorders 439

REFERENCES Four days later, the abdominal cramping has


Wang T, Wang Z-X, Wang H-J. Clinical insights improved, but there are no changes in ostomy
into superior mesenteric artery syndrome with output.
multiple cases: A case report. Dig Dis Sci.2018
Dec 20. doi: 10.1007/s10620-018-5436-2. [Epub Which of the following is the most likely cause of
ahead of print] high ostomy output?
Valdes A, Cardenas O, Espinosa A, et al. Supe-
rior mesenteric artery syndrome. J Am Coll Surg A. Intestinal bacterial overgrowth
2005;201:808. B. Exocrine pancreatic insufficiency
Biank V, Werlin S. Superior mesenteric artery C. Crohn’s disease
syndrome in children: a 20-year experience. J D. Bile acid diarrhea
Pediatr Gastroenterol Nutr 2006;42:522-5. E. Short bowel syndrome
Adson DE, Mitchell JE, Trenkner SW. The
superior mesenteric artery syndrome and acute CORRECT ANSWER: E
gastric dilatation in eating disorders: a report of
two cases and a review of the literature. Int J Eat RATIONALE
Disord 1997;21:103-14. The patient was likely complicated with short
bowel syndrome (SBS) after his second surgery.
The clinical manifestations vary depending on the
Question 29 remaining bowel anatomy and function. In this
A 46-year old man with a history of stricturing example, the patient had significant increase in
Crohn’s disease of the small bowel and colon is output after complete resection of the ileum and
admitted to the medical floor with high ostomy loss of continuity with the colon. Normal small
output and acute kidney injury. He had a right bowel length in adults ranges from 300 to 800 cm,
hemicolectomy and an unspecified small bowel which makes it difficult to determine the length of
resection with an ileocolonic anastomosis after remaining segments after resection. Though, the
complications of stricturing disease 15 years ago. ultimate determining factor is the critical mass of
Four months ago, he developed another stric- functional absorptive epithelia remaining. Three
ture that required an additional 60 cm of bowel bowel anatomies are generally described in SBS
removed and placement of a jejunostomy. He include jejuno-colic, jejuno-ileocolonic and end
reports having four to five liters of liquid stool jejunostomy.
coming from his ostomy daily in the last three In contrast to the ileum and colon, jejunal
months. He denies nausea, vomiting, but admits epithelial cell junctions have considerable space in
to abdomen pain, 10 pound weight loss and oc- between which results in a rapid flux of fluids and
casional blood streaking around the inferior edge nutrients and inefficient fluid absorption. The co-
of his stoma. Basic metabolic panel is notable lon has the greatest efficiency of water and sodium
for K 3.4 mEq/L, BUN 43 mg/dL, Cr 2.1 mg/dL. absorption and has the ability to adapt to absorb
Vitamin B12 190 pg/mL (normal 240 – 900 pg/ up to 6 liters daily. Ileal and colonic resections
mL), CRP 11mg/L (normal less than 5mg/L), lead to loss of intestinal adaptation by several
Fecal elastase 136 ug/g (normal greater than hormones including glucagon-like peptides 1 and
200 ug/g), Fecal Fat 145g (normal 2-7g). Further 2 and peptide YY15. Those with SBS may also be
workup for stool microorganisms, hepatic panel further burdened by gastric hypersecretion as a
and lipase are unremarkable. Patchy areas of result of loss of inhibitory hormones produced in
erythematous bowel are biopsied during jeju- the proximal gut.
noscopy. The patient is started on rifaximin and Bacterial overgrowth is common in patients
prednisone and is given an infusion of infliximab. with SBS and can lead to fat malabsorption and vi-
440 Digestive Diseases Self-Education Program®

tamin B12 deficiency, though this patient had little Question 30


response to empiric treatment with rifaximin. A 44-year-old man is admitted for an ulcerative
The patient does not have risk factors for pancre- colitis flare. He was in his usual state of health
atic insufficiency. A fecal elastase collected from until two weeks ago when he developed abdominal
liquid stool will be diluted and can result in a false cramping and bloody diarrhea. During this time,
positive. his symptoms progressively worsened resulting
He likely has concomitant active Crohn’s dis- in failure to tolerate PO and a five pound weight
ease; however, the immunosuppressive treatment loss. He is started on intravenous steroids and
had no effect on the ostomy output. given an infusion of infliximab. His response is
Resection of the terminal ileum decreases minimal and the decision is made to proceed with
reabsorption of bile acids into the enterohepatic total abdominal colectomy with ileorectal anas-
circulation and ultimately loss of bile acids. This tomosis. Two days after surgery, the patient is
will clinically manifest as steatorrhea and fat- diagnosed with an anastomotic leak. The patient is
soluble vitamin deficiency. In this example, the made NPO and abdominal drains are placed with
small bowel is not in continuity with the colon and an average total output of 650ml daily. He has
therefore will not be affected by the caustic effects reduced strength on exam, but does not have signs
seen in bile acid diarrhea including net fluid secre- of muscle/fat wasting or edema. He agrees to start
tion and accelerated colonic motility. parenteral nutrition.

REFERENCES Which is the optimal choice for starting the formula?


DiBaise JK PC. Short Bowel Syndrome in Adults
– Part 1 Physiological Alterations and Clini- A. Low volume, 50 percent normal saline, high
cal Consequences. Practical Gastroenterology dextrose
2014;132:30-39. B. Standard volume, 50 percent normal saline,
Carbonnel F, Cosnes J, Chevret S, et al. The low dextrose
role of anatomic factors in nutritional autonomy C. Increased volume, 75 percent normal saline,
after extensive small bowel resection. JPEN J low dextrose
Parenter Enteral Nutr 1996;20:275-80. D. Increased volume, 25 percent normal saline,
Fordtran JS, Rector FC, Jr., Carter NW. The low dextrose
mechanisms of sodium absorption in the human E. Low volume, 75 percent normal saline, high
small intestine. J Clin Invest 1968;47:884-900. dextrose
Nightingale JM, Kamm MA, van der Sijp JR, et al.
Gastrointestinal hormones in short bowel syn- CORRECT ANSWER: C
drome. Peptide YY may be the ‘colonic brake’ to
gastric emptying. Gut 1996;39:267-72. RATIONALE
Cortot A, Fleming CR, Malagelada JR. Im- The patient has severe malnutrition based on his
proved nutrient absorption after cimetidine in significant reduction in PO intake, weight loss,
short-bowel syndrome with gastric hypersecre- reduced strength and active inflammatory disease.
tion. N Engl J Med 1979;300:79-80. He requires support with parenteral nutrition
Hoffman AF PJ. Cholestyramine treatment of (PN) to help manage the leaking anastomotic site.
diarrhea associated with ileal resection. N Eng J Titrating the components of a PN formula is vital
Med 1969:397-402. to ensure the patient is corrected for deficiencies
Islam RS DJ. Bile Acids: An Underrecognized or to prevent complications.
and Underappreciated Cause of Chronic Diarrhea. Considering the patient is losing fluids from
Practical Gastroenterology 2012;110:32-44. the anastomotic leak, he needs increased volume
in his formula in order to maintain a euvolemic
Chapter 15 — Nutrition, obesity and eating disorders 441

state. Furthermore, patients with intestinal anas- Which catheter is the optimal choice for replace-
tomotic leakage are at risk for hyponatremia. Vol- ment in this patient?
ume and sodium are lost into the peritoneal cavity
and third space. Therefore, the patient requires A. Tunneled silicone catheter
increased sodium content (75 percent to 100 per- B. Implanted port
cent of normal saline) to correct for sodium losses. C. Peripherally inserted central catheter
This patient is at risk for refeeding syndrome D. Midline
given his state of malnutrition. Refeeding syn- E. Internal jugular triple lumen catheter
drome is characterized by an abrupt decrease in
serum potassium, magnesium and/or phosphorus CORRECT ANSWER: A
after the introduction of a consistent nutrient
source. This results from pancreatic stimulation RATIONALE
with increased insulin and decreased glucagon This patient is at high risk for thrombosis given her
secretion, which causes intracellular shift of the history of a DVT and active malignancy and is now
electrolytes. Parenteral nutrition should be started admitted for a catheter related bloodstream infection
with reduced calories and appropriate supplemen- after five weeks of use. A tunneled silicone catheter is
tation of electrolytes and vitamins. the optimal choice in this patient because they have
the lowest rates of infection and thrombosis. They
REFERENCES are durable for long-term use and commonly used
Kaser SA, Nitsche U, Maak M, et al. Could hy- for delivery of chemotherapy. Tunneled catheters en-
ponatremia be a marker of anastomotic leakage ter through a subcutaneous tissue tract and contain a
after colorectal surgery? A single center analysis bonded extravascular Dacron cuff near the exit site,
of 1,106 patients over 5 years. Langenbecks Arch which provides a barrier to external infection.
Surg 2014;399:783-8. In comparison to tunneled catheters, subcuta-
McCray S WS, Parrish CR. Much ado about neous ports are associated with an increased risk
refeeding. Practical Gastroenterology 2005; of catheter related blood stream infections.
23:26-44. Although a trained physician or nurse-led vascu-
Mehanna HM, Moledina J, Travis J. Refeeding lar access teams can conveniently place peripherally
syndrome: what it is, and how to prevent and treat inserted central catheters at the bedside, they have a
it. BMJ 2008;336:1495-8. significantly increased risk of deep vein thrombosis
when compared to other central venous catheters.
Midline catheter tips do not reach the superior
Question 31 vena cava. Parenteral nutrition formulas often have
A 66-year-old woman with metastatic gastric high osmolarity, which can cause thrombophlebitis
cancer and history of left leg deep vein throm- and thrombosis if delivered to a vessel with low
bosis on warfarin presents with one day of fever, flow rates. The superior vena cava and right atrium
chills and nausea. The patient was started on par- is the optimal location for delivery of a high osmo-
enteral nutrition five weeks ago after discovery of lality formula (greater than900 mOsm/L) which
a high-grade malignant bowel obstruction at her have rapid blood flow of 2-6L/min.
distal jejunum. She has been receiving FOLFOX There is increased risk of catheter thrombosis
through her implanted port. She takes clear and infection with multi-lumen catheters.
liquids for comfort, but is otherwise dependent
on the PN. Peripheral and central blood cultures REFERENCES
are positive for methicillin resistant staphylococ- Micic D, Semrad C, Chopra V. Choosing the Right
cus aureus. She is started on vancomycin and the Central Venous Catheter for Parenteral Nutrition.
port is removed. Am J Gastroenterol 2019;114:4-6.
442 Digestive Diseases Self-Education Program®

Buchman AL, Opilla M, Kwasny M, et al. Risk ing System of Balthazar grade D. The existing
factors for the development of catheter-related evidence supports the superiority of enteral
bloodstream infections in patients receiving home nutrition in mild and severe acute pancreatitis
parenteral nutrition. JPEN J Parenter Enteral if patients cannot tolerate oral feedings. TPN
Nutr 2014;38:744-9. is indicated only when enteral nutrition is not
Herc E, Patel P, Washer LL, et al. A Model to possible, or the patient is not able to meet their
Predict Central-Line-Associated Bloodstream In- nutritional requirements via enteral feedings.
fection Among Patients With Peripherally Inserted There is no difference in tolerance or clinical
Central Catheters: The MPC Score. Infect Control outcomes between patients fed via the nasogastric
Hosp Epidemiol 2017;38:1155-1166. as opposed to the nasojejunal route. Provision
Hammes M, Desai A, Pasupneti S, et al. of enteral nutrition to patients with severe acute
Central venous catheters: incidence and predic- pancreatitis helps maintain intestinal integrity by
tive factors of venous thrombosis. Clin Nephrol preventing permeability, maintaining functional
2015;84:21-8. tight junctions and promoting the role of com-
Madsen H FE. The Hitchhiker’s Guide to Par- mensal bacteria. In studies that compared enteral
enteral Nutrition Management for Adult Patients. versus parenteral nutrition in acute pancreati-
Practical Gastroenterology 2006:46-68. tis, there was faster resolution of SIRS, shorter
duration of time to resolution of the disease
and reduced overall complications in patients
Question 32 who received enteral nutrition. In studies that
A 62-year-old women with history of alcoholic compared enteral versus parenteral nutrition in
chronic pancreatitis is admitted with nausea, acute pancreatitis, there was faster resolution of
vomiting and epigastric discomfort radiating to the SIRS, shorter duration of time to resolution of
back. Laboratory workup is significant for hemo- the disease and reduced overall complications in
globin 10.2 g/dL, WBC 17,200/ml, serum amylase patients who received enteral nutrition. In studies
134 IU/L, serum lipase 88 U/L and serum LDH that compared enteral versus parenteral nutrition
380 IU/L. CT abdomen shows a calcified pancreas in acute pancreatitis, there was faster resolution
with peripancreatic fluid collection. Analgesia and of SIRS, shorter duration of time to resolution of
lactated ringers are started. A nasojejunal tube is the disease and reduced overall complications in
placed after three days due to inability to tolerate patients who received enteral nutrition. In studies
oral diet and standard formula is started. Twenty- that compared enteral versus parenteral nutrition
four hours after advancing the formula, the patient in acute pancreatitis, there was faster resolution
begins to have worsening abdominal cramping and of SIRS, shorter duration of time to resolution of
steatorrhea. Infectious diarrhea workup is negative. the disease and reduced overall complications in
Which of the following is the next best step? patients who received enteral nutrition.
She is not tolerating the standard polymeric
A. Hold tube feeds for 24 hours formula of tube feeds due to pancreatic insuf-
B. Switch to an elemental formula ficiency and therefore needs to switch to either
C. Switch to parenteral nutrition a formula that contains peptides and MCTs and
D. Addition of glutamine to the enteral formula if still intolerant, the next step is a nearly fat-
free elemental formula. Although glutamine has
CORRECT ANSWER: B a trophic effect on gut integrity, it has not been
shown to alter mortality, infections or length of
RATIONALE hospital stay in critically ill patients and therefore
This patient has severe acute pancreatitis ac- supplemental glutamine is not recommended in
cording to a Ransons Criteria of 3 and CT grad- this patient.
Chapter 15 — Nutrition, obesity and eating disorders 443

REFERENCES RATIONALE
Kalfarentzos F, Kehagias J, Mead N, et al. Enteral This patient likely has vitamin A and K deficiency
nutrition is superior to parenteral nutrition in due to improper and longstanding use of chole-
severe acute pancreatitis: results of a randomized styramine.
prospective trial. Br J Surg 1997;84:1665-9. Vitamin A deficiency causes conjunctival
Windsor AC, Kanwar S, Li AG, et al. Com- xerosis, Bitot’s spots and night blindness, which
pared with parenteral nutrition, enteral feeding explains her frequent injuries in the night. The
attenuates the acute phase response and im- easy bruising over the traumatized areas is likely
proves disease severity in acute pancreatitis. Gut caused by vitamin K deficiency which is respon-
1998;42:431-5. sible for posttranslational gamma carboxylation of
Jabbar A, Chang WK, Dryden GW, et al. Gut im- clotting factors (II, VII, IX, X) and anticoagulant
munology and the differential response to feeding proteins C and S.
and starvation. Nutr Clin Pract 2003;18:461-82. Cholesytramine is a sequestrant used to treat
Kagnoff MF. Immunology of the intestinal bile acid diarrhea. These agents are positively
tract. Gastroenterology 1993;105:1275-80. charged indigestible resins that bind bile acids in
DeWitt RC, Kudsk KA. The gut’s role in metabo- the intestine to form an insoluble complex that is
lism, mucosal barrier function, and gut immunol- excreted in the stool, thus preventing their secre-
ogy. Infect Dis Clin North Am 1999;13:465-81, x. tomotor actions on the colon. However, by binding
bile acids, they can inhibit fat emulsification and
micelle formation, which interferes with fat-soluble
Question 33 vitamin absorption. These medications should be
A 34-year-old woman presents with pain in her taken one hour before or four hours after meals. Co-
left foot. Six months ago, she had a cholecystec- lesevalem is another sequestrant that can be taken
tomy for recurrent biliary colic. Shortly after, she that does not interfere with vitamin absorption.
developed diarrhea, which responded well to cho- Hemoglobin a1c is used to evaluate for diabe-
lestyramine. She takes it four times per day with tes mellitus. While the foot x-ray was positive for a
meals. Over the last month, she has stubbed her fracture, other typical findings for charcot ar-
toe on the bedroom and hallway furniture multi- thropathy including osteopenia, loss of joint space,
ple times in the night when getting up to go to the and sclerosis were not present.
bathroom. She lives with her boyfriend who often Domestic abuse can be considered, but her
works late and has put a strain on their relation- areas of bruising are atypical for this presentation.
ship. Her left foot has multiple areas of scabbing Vitamin B12 deficiency can cause paresthesias
and purpura, especially her big toe, which is also of hands and feet, loss of vibration sense and posi-
swollen and painful with passive flexion. She has a tion sense, which could explain the foot injury, but
bruise over her right forehead and dry/thickened does not explain the eye manifestations and bruis-
appearing conjunctiva on eye exam. X-ray of the ing of the head.
left foot shows fracture of her left toe.Which of Referral to an ophthalmologist is not indicated
the following is the next best step? at this time.

A. Obtain fat-soluble vitamin levels REFERENCES


B. Obtain hemoglobin a1c Suttie J. Vitamin K1. Modern Nutrition in Health
C. Evaluate for domestic abuse and Disease. 10th ed. Philadelphia, PA: Lippincott
D. Obtain vitamin B12 level Wiliams & Wilkins, 2006:412-425.
E. Referral to ophthalmology Scaldaferri F, Pizzoferrato M, Ponziani FR, et al.
Use and indications of cholestyramine and bile acid
CORRECT ANSWER: A sequestrants. Intern Emerg Med 2013;8:205-10.
444 Digestive Diseases Self-Education Program®

Slowman-Kovacs SD, Braunstein EM, Brandt mocysteine levels but normal methylmalonic acid
KD. Rapidly progressive Charcot arthropathy fol- levels. Both methylmalonic acid and homocyste-
lowing minor joint trauma in patients with diabetic ine levels decline rapidly after treatment and can
neuropathy. Arthritis Rheum 1990;33:412-7. help with monitoring response to vitamin B12
Boosalis M. Vitamins. Contemporary Nutrition repletion.
Support Practice: A Clinical Guide. Philadelphia, Vitamin B12 is released from proteins ingested
PA: Gottschlich MM, 1998:145-163. by the action of hydrochloric acid and pepsin in
Dukowicz AC, Lacy BE, Levine GM. Small intes- gastric secretions. The free B12 binds R binder
tinal bacterial overgrowth: a comprehensive review. (transcobalamin I), which is secreted by the
Gastroenterol Hepatol (N Y) 2007;3:112-22. salivary glands. This complex travels to the small
intestine where pancreatic proteases hydrolyze
R protein to free B12, which then goes on to bind
Question 34 intrinsic factor (IF). IF is made by the gastric
A 44-year-old man presents to clinic with one parietal cells. The IF-B12 complex moves into the
month of fatigue and painful parasthesias. He ileum where it attaches to a specific IF receptor
underwent a biliopancreatic diversion with and is absorbed. After absorption, the vitamin is
duodenal switch procedure for weight manage- bound to transcobalamin II, a carrier protein that
ment seven months ago. He lost significant transports vitamin B12 to the liver, bone marrow
weight within the first four months, but has since and erythrocytes.
stabilized. His laboratory workup is significant The biliopancreatic diversion with duodenal
for Hgb 8.4 g/dL, MCV 112 fL and serum vitamin switch involves a restrictive and malabsorptive
B12 level 120 pg/mL. component. During this surgery, 60 percent of
the stomach is resected therefore limiting expo-
Which of the following confirms the diagnosis of sure to hydrochloric acid and reducing intrinsic
B12 deficiency? factor production. The duodenal stump is closed.
The main site of vitamin B12 absorption, the
A. Increased methylmalonic acid and homocyste- proximal ileum is transected and anastomosed
ine levels to the distal small bowel about 50cm proximal
B. Increased methylmalonic acid and normal ho- to the ileocecal valve. The distal ileal segment is
mocysteine levels anastomosed to the remaining stomach. This pa-
C. Normal methylmalonic acid and homocysteine tient will likely need lifetime vitamin B12 supple-
levels mentation.
D. Normal methylmalonic acid and increased
homocysteine levels REFERENCES
Stabler SP. Vitamin B12 deficiency. N Engl J Med.
CORRECT ANSWER: E 2013;368(2):149-160.
Carmel R. Cobalamin (Vitamin B12). Modern
RATIONALE Nutrition in Health and Disease. Philadelphia, PA:
Measurements of methylmalonic acid total ho- Lippincott Williams & Wilkins, 2006:482-497.
mocysteine levels or both is helpful in making the Moestrup SK. New insights into carrier binding
diagnosis of vitamin B12 deficiency in patients and epithelial uptake of the erythropoietic nutri-
who have not yet been treated. The levels of both ents cobalamin and folate. Curr Opin Hematol
methylmalonic acid and total homocysteine are 2006;13:119-23.
markedly elevated in almost all patients with Xanthakos SA. Nutritional deficiencies in obe-
vitamin B12 deficiency. In contrast, patients with sity and after bariatric surgery. Pediatr Clin North
isolated folate deficiency will have elevated ho- Am 2009;56:1105-21.
Chapter 15 — Nutrition, obesity and eating disorders 445

Question 35 combat the malabsorption. He has been opti-


A 39-year-old man with fistulizing Crohn’s disease mized from a medical and dietary standpoint and
with multiple small bowel resections and an end continues to be dependent on parenteral nutrition.
ileostomy presents for follow up visit in clinic. Therefore, his next best option to improve his
According to his last small bowel follow through, ability to maintain hydration and limit parenteral
he has less than 100cm of small bowel remaining. nutrition use is teduglutide.
Since his last surgery one-year prior, he has been Teduglutide is a GLP2 agonist that has shown
dependent on parenteral nutrition (PN) and takes to increase absolute weight absorption, small
optimal dosing of loperamide, diphenoxylate/ bowel villus height and crypt depth. Recent
atropine and pantoprazole. His Crohn’s disease studies that evaluated patients with short bowel
is maintained with infliximab and he reports no syndrome on parenteral nutrition showed tedu-
usual flare symptoms. He has frequent, small glutide reduced the required parenteral volume,
meals with low fat and dairy content. He also adds parenteral nutrition free days and even complete
salt to meals and liquids whenever possible and independency from parenteral nutrition.
avoids artificial sweeteners. Despite the above The colon has the greatest efficiency of wa-
measures, he continues to have one to 1.5 liters of ter and sodium absorption and has the ability
ostomy output daily. He had a normal EGD and to adapt to absorb up to six liters daily. Primary
ileoscopy with unremarkable biopsies and a colo- anastomosis would be appropriate when there is
noscopy, which showed a 15cm Hartmann’s pouch sufficient length of healthy colonic tissue. This
with mild amount of fibrotic tissue. The parenteral patient’s colon is too small and fibrotic from previ-
nutrition was titrated to three days per week, but ous surgery.
he remains dependent on PN to maintain weight This patient’s Crohn’s disease is well controlled
and hydration. based on endoscopic and microscopic findings.
Medication changes are not required at this time.
Which of the following is the next best step?
REFERENCES
A. Evaluate for surgical anastomosis with colon Jeppesen PB, Hartmann B, Hansen BS, et al.
B. Start Glucagon-like peptide analogue Impaired meal stimulated glucagon-like peptide
C. Change infliximab to adalimumab 2 response in ileal resected short bowel patients
D. Check infliximab trough level and with intestinal failure. Gut 1999;45:559-63.
antibody level Orskov C, Holst JJ, Poulsen SS, et al. Pancre-
atic and intestinal processing of proglucagon in
CORRECT ANSWER: B man. Diabetologia 1987;30:874-81.
Jeppesen PB, Gabe SM, Seidner DL, et al. Fac-
RATIONALE tors Associated With Response to Teduglutide in
Glucagon-like peptide-2 (GLP-2) is an endog- Patients With Short-Bowel Syndrome and Intesti-
enous hormone secreted by enteroendocrine L nal Failure. Gastroenterology 2018;154:874-885.
cells, mainly in the distal ileum and proximal co- Bremholm L, Hornum M, Andersen UB, et al.
lon. This hormone induces proabsorptive changes The effect of Glucagon-Like Peptide-2 on mes-
including prolonged gastric emptying, increased enteric blood flow and cardiac parameters in
mesenteric blood flow, mucosal absorption and end-jejunostomy short bowel patients. Regul Pept
small intestinal villus hypertrophy. 2011;168:32-8.
This patient had his proximal colon and likely Jeppesen PB, Hartmann B, Thulesen J, et al.
most of his ileum resected due to complications Glucagon-like peptide 2 improves nutrient absorp-
from his Crohn’s disease. Therefore, limiting his tion and nutritional status in short-bowel patients
remaining small bowel to adapt as necessary to with no colon. Gastroenterology 2001;120:806-15.
446 Digestive Diseases Self-Education Program®

Jeppesen PB, Sanguinetti EL, Buchman A, et al. day whereas a high output fistula drains greater
Teduglutide (ALX-0600), a dipeptidyl peptidase than 500ml/day.
IV resistant glucagon-like peptide 2 analogue, The recommended daily allowance (RDA) for
improves intestinal function in short bowel syn- protein in healthy adults is 0.8 g/kg body weight
drome patients. Gut 2005;54:1224-31. per day. High output fistulas usually require
Jeppesen PB, Gilroy R, Pertkiewicz M, et al. 30-35 kcal/kg of energy and 1.5-2.5 g/kg/day
Randomised placebo-controlled trial of teduglu- of protein for achievement of positive nitrogen
tide in reducing parenteral nutrition and/or intra- balance and wound healing. Studies have shown
venous fluid requirements in patients with short decrease in mortality in patients who are provided
bowel syndrome. Gut 2011;60:902-14. 30-35 kcal/kg for management of enterocutane-
Pevny S, Maasberg S, Rieger A, et al. Experi- ous fistulas. Low output fistulas typically require
ence with teduglutide treatment for short bowel 20-30 kcal/kg/day and 1-1.5g/kg/day of protein.
syndrome in clinical practice. Clin Nutr 2018. These patients also often need increased provision
of trace minerals and vitamins such as vitamin
C, copper , selenium and zinc to promote wound
Question 36 healing. Although this patient is starting on TPN
A 59-year-old man is admitted to the surgery because it has been shown to decrease enteral
service after findings of a colocutaneous fistula. fluid secretions by 30-50 percent, the goal will be
Two weeks ago he underwent partial colectomy to transition as soon as possible to enteral nutri-
with primary anastomosis for stage II colon ad- tion. It is recommended that patients receive at
enocarcinoma of his descending colon. Over the least 20 percent of their calories via the enteral
last three days, he noticed leakage of fecal matter route if possible to maintain mucosal integrity,
coming from his skin in the left lower quadrant. hormone signaling and immune function.
He lost seven pounds since his procedure and
currently has a BMI of 21kg/m2. Since admission, REFERENCES
the fistula has put out 400-600ml of fluid daily. Gribovskaja-Rupp I, Melton GB. Enterocutaneous
The primary service wishes to pursue conserva- fistula: proven strategies and updates. Clin.Colon
tive management by making the patient NPO and Rectal Surg. 2016;29(2):130-7.
starting parenteral nutrition. Sheldon GF, Gardiner BN, Way LW, et al.
Management of gastrointestinal fistulas. Surg
Which combination of energy and protein require- Gynecol Obstet 1971;133:385-9.
ments is most appropriate for this patient? Badrasawi M, Shahar S, Sagap I. Nutritional
Management in Enterocutaneous Fistula. What is
A. Total Energy: 20-25 kcal/kg/day, Protein: 2.5g/ the evidence? Malays J Med Sci 2015;22:6-16.
kg/day Council NR. Recommended Dietary Allow-
B. Total Energy: 15-20 kcal/kg/day, Protein: 2.0g/ ances. Washington, DC: National Academy Press,
kg/day 1989.
C. Total Energy: 30-35 kcal/kg/day, Protein: 1.5g/ Dudrick SJ, Maharaj AR, McKelvey AA. Artifi-
kg/day cial nutritional support in patients with gastroin-
D. Total Energy: 30-35 kcal/kg, Protein: 0.8 g/kg testinal fistulas. World J Surg 1999;23:570-6.

CORRECT ANSWER: C
Question 37
RATIONALE A 32-year-old with history of Roux-en-Y gastric
This patient has a high output colocutaneous fis- bypass five months ago presents to clinic with
tula. A low output fistula drains less than200ml/ complaints of a rash. Two weeks ago, she noticed
Chapter 15 — Nutrition, obesity and eating disorders 447

a dry scaly rash over both her legs. She tried bound to albumin and 30 percent bound to alpha-
moisturizing ointments without relief. She also macroglobulins. Physiologic conditions that cause
noticed frequent dandruff in the last few days. hypoalbuminemia may impair hepatic release of
Her appetite has been poor since the surgery zinc, which may cause a falsely low serum level.
and is concerned that she is losing the weight Vitamin C is typically absorbed in the jejunum
too fast. Examination shows contiguous dry and and ileum through sodium, energy dependent
scaly rash over her anterior legs and cracking and active transport. Its absorption is typically not
peeling at her fingertips. Labs are significant for affected by roux-en-y gastric bypass surgery and
copper 1.28 ug/mL(0.75-1.45 ug/mL), zinc 0.62 usually presents with anemia, bleeding gums,
ug/mL (0.66-1.10 ug/mL), selenium 102 ng/mL impaired wound healing, fatigue and depression
(80-142 ng/mL), albumin 3.1 g/dL (3.5-5.0g/dL), rather than causing skin manifestations.
triene:tetraene ratio 0.24 (0.01-0.038).
REFERENCES
A deficiency in which nutrient most likely explains John S, Hoegerl C. Nutritional deficiencies after
this patient’s symptoms? gastric bypass surgery. J Am Osteopath Assoc
2009;109:601-4.
A. Copper Lopes TI, Geloneze B, Pareja JC, et al. Blood
B. Zinc Metabolome Changes Before and After Bariatric
C. Vitamin C Surgery: A (1)H NMR-Based Clinical Investiga-
D. Essential fatty acids tion. OMICS 2015;19:318-27.
Wene JD, Connor WE, DenBesten L. The
CORRECT ANSWER: D development of essential fatty acid deficiency in
healthy men fed fat-free diets intravenously and
RATIONALE orally. J Clin Invest 1975;56:127-34.
This patient has essential fatty acid deficiency Mogensen K. Essential Fatty Acid Deficiency.
(EFAD) after roux-en-y gastric bypass surgery. Practical Gastroenterology 2017;164:37-44.
Monitoring of macro- and micronutrients af- Barr LH, Dunn GD, Brennan MF. Essential fatty
ter this procedure is important as it can lead to acid deficiency during total parenteral nutrition.
multiple deficiencies, including essential fatty Ann Surg 1981;193:304-11.
acids. Patients with EFAD often complain of a dry, Hamilton C, Austin T, Seidner DL. Essential
scaly rash, impaired wound healing and immune fatty acid deficiency in human adults during par-
dysfunction. The minimal amount of fat to pre- enteral nutrition. Nutr Clin Pract 2006;21:387-94.
vent EFAD is 100g per week. In the absence of the MJ D. Zinc. Present Knowledge in Nutrition.
essential fatty acids (Linoleic acid and alpha-lino- 8th ed. Washington DC: ILSI Press, 2001:329-
lenic acid), oleic acid is metabolized to eicosatrie- 343.
onic acid (triene) and there is reduced production Prelack K, Sheridan RL. Micronutrient supple-
of eicosatetraenoic acid (tetraene). This results in mentation in the critically ill patient: strategies for
an elevated triene:tetraene ratio. A ratio greater clinical practice. J Trauma 2001;51:601-20.
than 0.2 is diagnostic. V R. Micronutrient Recommendations for
Although, copper deficiency is possible after Wound Healing. Support Line 2004;24:3-9.
roux-en-y gastric bypass, this patient’s levels are
within normal limits.
Although zinc deficiency can cause dry, scaly Question 38
rashes, it is unclear if the patient has a true zinc A 28-year-old woman presents with symptoms
deficiency given coexistent hypoalbuminemia. The of abdominal pain, distention, nausea without
liver releases zinc into circulation with 70 percent vomiting and early satiety. She reports a weight
448 Digestive Diseases Self-Education Program®

loss of approximately 20 pounds over the past six A. Hold the tube feeds
months. Her physical exam is notable for a BMI B. Check his stools for Clostridium difficile
of 14.6 and a distended abdomen, but otherwise infection
unremarkable. Laboratory evaluation reveals C. Switch to a lower volume hyperosmolar formula
hyponatremia, hypokalemia, hypomagnesemia, D. Start a probiotic
hypophosphatemia, and low serum creatinine. E. Start Imodium therapy
Abdominal imaging reveals a large gastric bezoar.
CORRECT ANSWER: B
Which of the following diagnoses is most likely?
RATIONALE
A. Anorexia nervosa In ICU patients receiving enteral nutrition, diar-
B. Pyloric stenosis rhea is common and can be severe resulting in
C. Celiac disease dehydration, electrolyte disturbances and perianal
D. Jejunal intussusception skin breakdown. Despite that, enteral nutrition
E. Functional dyspepsia should not be reflexively held but instead continued
while evaluating the cause of the diarrhea. Most
CORRECT ANSWER: A nosocomial diarrhea is self-limited. Common eti-
ologies of diarrhea in enterally fed patients include
RATIONALE infections such as C. difficile, medications (proton
The notable features that suggest a diagnosis of pump inhibitors, antibiotics, selective serotonin
anorexia are the BMI, the laboratory findings and reuptake inhibitors etc.) and lack of fiber in the
the gastric bezoar on imaging which suggests de- enteral formula. Hyperosmolar formulas are less
layed gastric emptying. Delayed gastric emptying volume and are more calorically concentrated but
can occur in the setting of restricted oral intake and can also cause an osmotic diarrhea. Although there
weight loss. A potential mechanism for the gastro- is a lack of data that probiotics decrease diarrhea in
paresis may be smooth muscle atrophy due to long- this patient population, the most consistent benefit
standing protein calorie malnutrition. The primary of probiotics has been the decrease in infectious
mode of treatment is nutritional rehabilitation. morbidity in select patient populations (transplan-
tation, trauma, pancreatectomy) where it has been
REFERENCE shown to decrease infectious complications. Soluble
Bern et al. Gastrointestinal manifestations of eat- fiber supplementation is recommended in patients
ing disorders. JPGN. 2016;63:77-85. that have diarrhea with enteral feedings. Therefore,
the management of this patient should consist of
a thorough search for a cause of diarrhea includ-
Question 39 ing checking for C. difficile, assessing for potential
A 66-year-old man is in the ICU after esophagecto- medication side effects and possibly supplementing
my for esophageal adenocarcinoma. He underwent with soluble fiber.
PEG tube placement last week for inadequate oral
intake due to dysphagia. He had been tolerating his REFERENCE
enteral feedings with a standard polymeric formula McClave, SA, Taylor BE, Martindale RG et al.
until last night when the nursing staff noted the Guidelines for the provision and assessment
development of voluminous diarrhea. His perianal of nutrition support therapy in the critically ill
area is erythematous due to diarrhea. patient: society of critical care medicine (SCCM)
and American society for parenteral and enteral
What is your first step in managing this patient’s nutrition (ASPEN). J Parenter Enteral Nutr.
diarrhea? 2016;4(2):159-211.
Chapter 15 — Nutrition, obesity and eating disorders 449

Question 40 you proceed to discuss the risks and benefits of


A patient presents to your office seeking infor- this medication with your patient. Which of the
mation regarding obesity. His BMI is currently following is a contraindication to phentermine/
32.1kg/m2 and he has tried multiple weight loss topiramate therapy?
strategies. Although he is usually initially success-
ful, he eventually regains the weight. Which of A. History of pancreatitis
the following hormones is elevated when patients B. Pregnancy
experience increased hunger and decreased satiety C. Hypothyroidism
after weight loss? D. Tobacco use
E. Obstructive sleep apnea
A. CCK
B. Leptin CORRECT ANSWER: B
C. Ghrelin
D. PYY (peptide YY) RATIONALE
E. GLP-1 Behavioral and dietary therapy combined with
exercise should be offered to all patients with
CORRECT ANSWER: C a BMI greater than 25. These efforts can be
combined with pharmacotherapy for patents
RATIONALE There are multiple hormones with a BMI greater than 30 or BMI greater than
that signal satiety including peptide YY which is 27 with associated comorbidities. The FDA
secreted in the ileum and colon and CCK which approved Phentermine/topiramate therapy in
is secreted in the duodenum. GLP-1 is secreted 2012 for chronic weight management. Phen-
by L cells in the ileum in response to glucose and termine is a norepinephrine –releasing agent
subsequently stimulates insulin release in the pan- and topiramate is a GABA receptor modulator.
creas. High leptin levels and insensitivity to exog- Phentermine/topiramate therapy is teratogenic
enous leptin is almost universally associated with and increases the risk of oral cleft defects. A
human obesity. Increased hunger and decreased negative pregnancy test prior and during ther-
satiety associated with weight loss corresponds to apy is recommended as is two forms of contra-
increases in increased ghrelin and decreased levels ception for the duration of therapy in women of
of PYY, CCK, leptin and insulin. These hormonal childbearing age. Phentermine/topiramate is
changes persist for at least a year following weight also contraindicated in hyperthyroidism, glau-
reduction. coma and in patients taking MAO inhibitors.
Liraglutide has also been approved for chronic
REFERENCE weight management.
Apovian, CM, Aronne LJ, Bessesen DH et al. Phar-
macological management of obesity: An endocrine REFERENCE
society clinical practice guideline. J Clin Endocri- Apovian, CM, Aronne LJ, Bessesen DH et al. Phar-
nol Metab. 2015:100(2):342-362. macological management of obesity: An endocrine
society clinical practice guideline. J Clin Endocri-
nol Metab. 2015:100(2):342-362.
Question 41
A 33-year-old otherwise healthy woman with a
BMI of 31kg/m2 is seen in the clinic to discuss Question 42
obesity management. She is specifically interested A 57-year-old patient underwent Roux en Y gastric
in phenteramine/topiramate therapy. After taking bypass surgery one year ago. She has lost 45 per-
a careful history and performing a physical exam, cent of her baseline weight. She reports that she
450 Digestive Diseases Self-Education Program®

has had difficulty with nausea and vomiting, she as shuffling gait and mental sluggishness. Zinc de-
has numbness and tingling in her extremities and ficiency causes impaired wound healing, immune
she is experiencing ataxia. On physical exam, she dysfunction, night blindness and skin lesions.
has a sensory and motor neuropathy. Her Hgb is Copper deficiency causes a myeloneuropathy, ane-
13.5 g/dl, MCV 93 fL, WBC 5.7x109/L and platelets mia and neutropenia. Selenium deficiency causes
250x109/L. cardiomyopathy and skeletal muscle dysfunction.
Essential fatty acid deficiency is rare in bariatric
What is the most likely cause of this patient’s surgery patients and causes a dry scaly rash, hair
symptoms? loss, hair depigmentation and poor wound heal-
ing. Therefore, the most likely explanation for this
A. Copper deficiency patient’s symptoms of ataxia, peripheral neuropa-
B. Selenium deficiency thy, in the setting of nausea and vomiting without
C. Zinc deficiency anemia is thiamine deficiency.
D. Thiamine deficiency
E. Vitamin B12 deficiency REFERENCES
Mechanick JI,Youdim A, Jones DB. Clinical
CORRECT ANSWER: C Practice Guidelines for the Perioperative Nutri-
tional, Metabolic, and Nonsurgical Support of the
RATIONALE Bariatric Surgery Patient—2013 Update: Cospon-
Vitamin and mineral deficiencies are common after sored by American Association of Clinical Endo-
bariatric surgery. The most common deficiencies are crinologists, The Obesity Society, and American
thiamine, vitamin B12, vitamin D, iron and copper. Society for Metabolic & Bariatric Surgery. Obesity.
There is also more awareness recently of subclinical 2013;21(1):S1-27.
micronutrient deficiencies in obese patients preop- Saltzman E, Karl JP. Nutrient deficien-
eratively. Thiamine deficiency has been reported cies after gastric bypass surgery. Ann Rev Nutr
in 0-29 percent of patients preoperatively and in 2013;33:183-203.
one to 49 percent of patients after bariatric surgery.
Symptomatic thiamin deficiency occurs weeks to
years after surgery and can occur after all bariatric Question 43
procedures. The most commonly reported deficiency A 57-year-old woman presents with a primary
syndrome is Wernicke’s encephalopathy. This is a complaint of intermittent dysphagia complicated by
neuropsychiatric disorder consisting of abnormal occasional food impactions. She undergoes EGD and
eye movements, ataxia and altered mental func- the endoscopist notes linear furrows and stacked
tion. Risk factors for thiamine deficiency in bariatric circular rings in her esophagus. Biopsies from the
patients include persistent vomiting, avoidance of proximal and distal esophagus revealed up to 25
eating and noncompliance with supplementation. eosinophils per hpf. She would like to investigate
Thiamine deficiency or beriberi has two entities. Dry non-pharmacologic treatments. Which of the follow-
beriberi consists of symmetrical peripheral neuropa- ing diets is most effective in treating this disease?
thy with both sensory and motor impairments. Wet
beriberi affects the heart and can result in cardio- A. Six food elimination diet (SFED)
megaly, cardiomyopathy, heart failure and tachycar- B. Elemental diet
dia. Post bariatric surgery patients should take a B C. Allergy testing directed elimination diet
complex vitamin containing 50mg thiamine once to D. Gluten free diet
twice daily in addition to a multivitamin. E. Cow’s milk elimination diet
Vitamin B12 deficiency presents with a macro-
cytic anemia, yellow skin, neurologic findings such CORRECT ANSWER: B
Chapter 15 — Nutrition, obesity and eating disorders 451

RATIONALE cently had hgba1c checked and it was 6.5 percent,


This patient has eosinophilic esophagitis (EoE) and she has a history of hypertension and is other-
treatment consists of medical (corticosteroid) or wise healthy. She does not want to have to start
dietary therapy. The role of food allergy in the etiol- diabetic medications and therefore is looking for
ogy of EoE stems from resolution of symptoms with a way to successfully lose weight and decrease her
avoidance of trigger foods. All of the above diets have cardiovascular risk factors.
been studied and are effective in achieving histologic
remission (reduction in eosinophils to less than 15 Which of the following would you recommend for
per hpf). Elemental diets in which the patient is ex- initial management of her weight?
clusively fed an amino acid based elemental formula
attain histologic remission rates in greater than 90 A. Place a referral to a bariatric surgery clinic
percent of patients. Although it is the most effective B. Start orlistat therapy
therapy, this diet clearly has limitations includ- C. Prescribe phentermine-topiramate therapy
ing palatability, cost and compliance especially in D. Prescribe lorcaserin therapy
social situations. The allergy testing – directed food E. Provide education on a moderate fat, low
elimination diet eliminates foods that have a positive calorie diet along with instructions for keeping
result with skin testing. This diet leads to histologic a food diary and activity log
remission in 32.2 percent of adults and 47.9 percent
of children. The fact that this strategy is success- CORRECT ANSWER: E
ful supports the immunologic/allergic etiology of
this disease. The six-food elimination diet (SFED) RATIONALE
removes the 6 most common IgE mediated food Patients are candidates for drug therapy if they
allergens which are milk, egg, soy, wheat, peanuts/ have a BMI greater than 30 or a BMI 27-29.9 with
tree nuts and fish/shellfish. Although, it should be comorbidities and have failed comprehensive
noted that peanuts/tree nuts and fish/shellfish are lifestyle intervention. This patient has not tried
rare triggers for EoE. SFED induces histologic remis- lifestyle intervention and therefore this should be
sion in 72.8 percent of children and 71.3 percent first line therapy. The goal the lifestyle interven-
of adults. Cow’s milk is the most common trigger tion is loss of five percent of total body weight
for EoE and the basis of the cow’s milk elimination by six months. Patients who undergo intensive
diet which has a remission rate of 66.3 percent in lifestyle intervention can lose five to 10 percent of
children (although the numbers studied are small). their initial weight and reduce cardiovascular risk
Wheat is the second most common trigger for EoE factors and prevent or delay the development of
and the gluten free diet induces remission in 45.5 type 2 diabetes. Orlistat is a gastrointestinal lipase
percent of children and 88.8 percent of adults. inhibitor that leads to excretion of 30 percent of
ingested fat. Lorcaserin is a selective serotonin re-
REFERENCE ceptor agonist. Phentermine-topiramate, Orlistat
Arias A, Gonzalez-Cervera J, Tenias JM et al. and Lorcaserin are all likely to result in clinically
Efficacy of Dietary interventions for inducing meaningful weight loss of at least five percent of
histologic remission in patients with eosinophilic initial weight when combined with lifestyle inter-
esophagitis: a systemic review and meta-analysis. vention. Although drug therapy has been shown to
Gastroenterology 2014;146:1639-1648. be safe and effective, when the maximum thera-
peutic effect is reached, there is no further weight
loss. Once the medication is stopped, the weight
Question 44 will be regained if the patient has not adopted
A 52-year-old female would like to discuss weight lifestyle interventions including behavior modifi-
management. She has a BMI 28.2kg/m2. She re- cations and exercise.
452 Digestive Diseases Self-Education Program®

REFERENCE der. It is a central nervous system stimulant and


Yanovski, SZ, Yanovski JA. Long-term drug treat- dextroamphetamine prodrug. Topiramate is an
ment for obesity: a systemic and clinical review. antiepileptic drug. Cognitive behavioral therapy,
JAMA 2014;311(1):74-86. lisdexamfetamine, second generation antidepres-
sants and tompiramate (not in combination with
phenteramine) have all been shown to reduce
Question 45 binge eating related psychopathology. Lisdex-
A 33-year-old patient comes in to clinic to discuss amfetamine and topiramate decreased weight in
weight loss strategies. She has a BMI of 33kg/ these adult patients as well. Sleeve gastrectomy
m2 and a history of depression but is otherwise and orlistat therapy are both effective obesity ther-
healthy. She states that her weight is causing apies but do not treat the psychopathology related
an increase in her feelings of hopelessness and to binge-eating disorder. Sibutramine is a weight
depression. When asked about her eating habits, loss medication removed from the market in 2010
she states that she usually eats healthy foods but due to increased cardiovascular events and stroke
sometimes when under stress, she will eat too fast in patients taking this drug.
and keeps eating despite feeling full. She finds this
habit embarrassing and overall feels disgusted REFERENCE
with herself and her weight. These episodes occur Brownley KA, Berkman ND, Peat CM et al. Binge-
approximately once per week and have been pres- eating disorder in adults: A systemic review and
ent intermittently for years. She denies laxative meta-analysis. Ann Intern Med. 2016;165(6):
use or purging. Which of the following is the best 409-420.
choice of therapy for this patient?

A. Lisdexamfetamine Question 46
B. Sleeve gastrectomy A 23-year-old woman presents to clinic to discuss
C. Orlistat therapy her diet. She has recently become a vegan and is
D. Sibutramine asking about the safety of this diet and any needed
E Phentermine-topiramate nutritional supplementation.

CORRECT ANSWER: A Which vitamin or mineral listed below will likely


need supplementation?
RATIONALE
This patient has binge-eating disorder. This is the A. Folate
most common eating disorder and affects three B. Vitamin B12
percent of US adults in their lifetime. The DSM V C. Vitamin C
criteria for binge- eating disorder includes eating D. Iron
a large amount of food (larger than most people E. Vitamin E
would in the same amount of time) in a discrete
amount of time (two hours). These episodes occur CORRECT ANSWER: B
at least once per week for at least three months
and are associated with a lack of control and psy- RATIONALE
chological distress related to binging. Patients eat A vegetarian diet is a diet that does not contain
until uncomfortably full, they often feel embar- meat (including fowl) or seafood. A vegan is a
rassed and disgusted with themselves. vegetarian that excludes eggs, dairy and all animal
Lisdexamfetamine was approved by the FDA products from their diet. Vegetarian diets are
in 2015 for the treatment of binge-eating disor- associated with multiple health benefits including
Chapter 15 — Nutrition, obesity and eating disorders 453

decreased serum cholesterol levels, decreased risk ability to see well at night and onset of non bloody
of heart disease and hypertension and a decreased diarrhea. He also noted to have frequent “colds”
risk of diabetes. Vegetarians also have lower BMIs and minor cuts and bruises which take long to
than non-vegetarians and a lower risk of cancer. heal. He has been married but unable to have
The vegetarian diet is lower in saturated fats and children. Physical examination reveals a bald male
higher in fiber, magnesium, potassium, vitamin with erythematous rash on his feet. Neurological
E and C as well as folate. The vegetarian diet and examination reveals normal tone and gait is ob-
more commonly, the vegan diet often contains served normal. Laboratory tests show normal TSH
lower intake of vitamin B12, calcium, vitamin D, and T4, normal CBC but low alkaline phosphatase.
zinc and long chain fatty acids. Lacto-ovo-vege- Stool studies are negative.
tarians obtain adequate vitamin B12 from dairy
foods or eggs. Vegans will need to obtain vitamin What supplementation is appropriate?
B12 from foods fortified with B12 or vitamin B12
supplements since no unfortified plant foods con- A. Biotin
tain significant amounts of vitamin B12. B. Zinc
The iron in plant food is nonheme iron, which is C. Thiamine
more susceptible to inhibitors and enhancers of D. Vitamin A
iron absorption. Organic acids such as vitamin
C can increase the absorption of iron. In addi- CORRECT ANSWER: B
tion, avoiding inhibitors of iron such as phytates,
calcium and polyphenolics (found in tea, coffee, RATIONALE
cocoa) will also enhance absorption. Incidence of This patient has zinc deficiency. Zinc deficiency
iron deficiency in vegetarians is similar to non- is one of the major factors contributing to burden
vegetarians and most experts do not recommend of disease in developing countries. Populations
supplementation unless a patient is found to be in South Asia, South East Asia, and sub-Saharan
iron deficient. Africa are at greatest risk of zinc deficiency. Low
serum zinc levels can be found in patients with
REFERENCES Crohn’s disease and subsequent small bowel
Position of the American Dietetic Association: veg- malabsorption, short bowel syndrome, hookworm
etarian diets. J Am Diet Assoc. 2009;109:1266-1282. infestation, pancreatic insufficiency, exclusive
Agnoli C, Baroni L, Bertini I et al. Position pa- parental nutrition, strict vegetarian diets, and an-
per on vegetarian diets from the working group of orexia nervosa. Common features include alope-
the Italian society of human nutrition. Nutr Metab cia, dysgeusia, acrodermatitis or bullous pustular
Cardiovasc Dis 2017;27(12):1037-1052. dermatitis(erythematous, pustular, vesiculobul-
barlesions), and low alkaline phosphatase. Zinc
deficiency is also associated with an increased
Question 47 incidence of night blindness, diarrhea, delayed
A 65-year-old male who recently migrated from wound healing and upper respiratory tract infec-
South Africa was referred to you by the primary tions. Delayed sexual maturation, oligospermia,
care physician for evaluation of new onset diar- hypogonadism and impotence are not uncommon.
rhea and weight loss. He has a history of alco- Medications including penicillamine, various di-
holism and known pancreatic insufficiency. He uretics, and sodium valproate can inhibit absorp-
reports that for many years he has decreased tion of zinc.
appetite because of difficulty with smell and taste. Biotin deficiency can present with similar hair
As a teenager he remembers having occasional and skin findings but additionally patients have
red blisters on his face. As he aged, he reports in- hypotonia, ataxia, seizures, conjunctivitis and
454 Digestive Diseases Self-Education Program®

hearing loss. Patients with thiamine deficiency comparison, roux-en-Y gastric bypass has 60-75
have nystagmus, peripheral neuropathy and percent expected loss, vertical sleeve gastrectomy
ataxia/ Vitamin A deficiency presents with night a 50-70 percent weight loss (3 year data) and
blindness and skin manifestations but in the ab- laproscopic adjustable gastric banding the lowest
sence of diarrhea. with 30-50 percent weight loss. It is important to
note that although biliopancreatic diversion with
REFERENCES duodenal switch provides the greatest weight loss
Ogawa Y,Kinoshita M,Shimada S,Kawamura T, and remission of diabetes, it will also cause more
Zinc and Skin Disorders. Nutrients. 2018 Feb 11 metabolic complications compared with the other
Assessment of marginal zinc status in humans.J surgeries. Also noted the roux-en-Y gastric bypass,
Nutr. 2000;130(5S Suppl):1350S. vertical sleeve gastrectomy and biliopancreatic
Prasad AS Zinc and immunity.Mol Cell Bio- diversion with duodenal switch all improves meta-
chem. 1998;188(1-2):63. bolic disease with vertical sleeve gastrectomy often
US Department of Health and Human Ser- used in a first step of a staged approach for weight
vices (HHS) and US Department of Agriculture loss which can later be converted to biliopancre-
(USDA): Dietary guidelines for Americans – 2015- atic diversion with duodenal switch procedure.
2020, eight edition(2015).
REFERENCES
Bays HE, Seger JC, Primack C, McCarthy W, Long
Question 48 J, Schmidt SL, Daniel S, Wendt J, Horn DB, West-
A 43-year-old female with a BMI 50kg/m2 and man EC: Obesity algorithm, presented by the Obe-
a history of hypertension and diabetes comes to sity Medical Association. www.obesityalgorithm.
you for advice on weight loss procedures. She has org. 2016-2017.
spent years trying to lose weight by diet and exer- Rubino F, Nathan DM, Eckel RH et al. Meta-
cise with no success. You review her labs which are bolic surgery in the treatment for type 2 diabetes:
unremarkable except HgA1C of 8.3 percent. She a joint statement by international diabetes organi-
has a referral for the bariatric surgeon but comes zations. Diabetes care. 2016;39:861-877.
to you investigating other options but wants to
know which bariatric procedure will provide the
most weight loss with greatest improvement in her Question 49
diabetes. Based on her question, what procedure Leptin secretion from adipocytes is an important
do you recommend? hormone in regulating obesity. Which of the fol-
lowing mechanisms is true of leptin?
A. Roux-en-Y gastric bypass
B. Sleeve gastrectomy A. Leptin secretion is decreased in overfeeding
C. Biliopancreatic diversion with duodenal switch B. As insulin levels rise, leptin levels decreases.
D. Gastric banding. C. Leptin upregulates food intake by signals to
the hypothalamus
CORRECT ANSWER: C D. Leptin is secreted in direct proportion to the
amount of fat present.
RATIONALE
According to the OMA Obesity algorithm (pg148) CORRECT ANSWER: D
expected loss in percent excess body weight at two
years for biliopancreatic diversion with duodenal RATIONALE
switch is 70-80 percent with greatest amount of Leptin is secreted by adipocytes in direct propor-
weight loss and resolution of metabolic disease. In tion to the amount of fat present. The levels fall
Chapter 15 — Nutrition, obesity and eating disorders 455

in response to fasting, not overeating. As insulin D. She has developed a gastro-gastric fistula and
levels fall, leptin secretion also decreases. Leptin need immediate endoscopy.
forms a negative feedback loop that down regu- E. She has developed dumping syndrome
lates food intakes at the level of the hypothalamus.
Hence obesity is thought to be leptin resistant and CORRECT ANSWER: A
insulin resistant state.
RATIONALE
REFERENCES Internal hernias are common with Roux-en-Y and
Perry B, Wang Y. Appetite regulation and weight biliopancreatic diversion with duodenal switch pro-
control: The role of gut hormones. Nutrition and cedures. They are usually accompanied by intermit-
Diabetes. tent, post prandial pain and emesis but sometimes
Dardeno TA, Chou SH, Moon HS, Chamber- patients present only with pain. It occurs because
land JP, Fiorenza CG, Mantzoros CS. Leptin in of herniation through a defect in the mesentery
human physiology and therapeutics.Front Neuro- created during the surgery with herniation through
endocrinol. 2010;31(3):377. Epub 2010 Jun 17. the transverse mesocolon defect the most com-
mon. Internal hernias have been described in up
to five percent of patients undergoing laparoscopic
Question 50 bariatric surgery. Diagnosis can be challenging both
A 35-year-old female comes to your office for clinically and radiographically since episodes are
evaluation of intermittent abdominal pain. About intermittent. Internal hernias can be difficult to de-
two years ago she had a roux-en-Y gastric bypass tect radiographically because they are intermittent.
for weight loss which has been effective: her BMI Several studies have shown that the “mesenteric
is now 27kg/m2 which is reduced from an initial swirl” sign on CT scan is the best indicator of an in-
BMI of 48kg/m2 and previously uncontrolled ternal hernia following gastric bypass. If suspected,
diabetes has now resolved. She complains that emergent laparoscopy is needed if attack is acute.
recently after eating, she experiences extreme Although gallstones and gallbladder disease
crampy pain and sometimes may vomit digested can be an early or late complication following bar-
contents. She denies loose bowel movements. She iatric surgery, and although her WBC is border-
has had a recent CT scan which is read as “unre- line elevated, her transaminases are normal. This
markable”. She also had an EGD remarkable only patient also does not have right upper quadrant or
for gastritis and evidence reflux. Today however epigastric post prandial or nocturnal pain com-
she has had increased abdominal pain. Vital signs mon with gallbladder or gallstone disease. Pa-
show BP 128/82 mm Hg and HR 103 bpm. Physi- tients with incisional hernia experience pain at the
cal examination produces mid abdominal tender- incision site and often can be palpated depending
ness; however she has no tenderness at the site of on the body habitus. Gastro-gastric fistula is sus-
the surgical scar. pected when there is suboptimal weight loss and
often weight regain and recurrence of metabolic
Labs show WBC 11,600 per mL, AST 43 IU/L, disease. These patients may also have non healing
ALT 46 IU/L, alkaline phosphatase 136 IU/L. gastric ulcers. This is not the case in this patient
What is your concern? who has optimal weight loss with no regain,
resolution of metabolic disease and recent normal
A. She has an internal hernia and need immediate EGD. Dumping syndrome is more common before
surgical consult two years post op with common symptoms of
B. She has an attack of acute cholecystitis facial flushing, light headedness, fatigue, reactive
C. She has developed an incisional hernia which hypoglycemia and post prandial diarrhea. This
may need repair patient had none of these symptoms.
456 Digestive Diseases Self-Education Program®

REFERENCES CORRECT ANSWER: D


Bays HE, Seger JC, Primack C, McCarthy W, Long
J, Schmidt SL, Daniel S, Wendt J, Horn DB, West- RATIONALE
man EC: Obesity algorithm, presented by the Obe- Lorcaserin may cause valvulopathy, attention or
sity Medical Association. www.obesityalgorithm. memory disturbance. This patient has normal
org. 2016-2017. ECHO and does not work with heavy machinery.
American Society of Metabolic and Bariatric Given his other history, may be the best choice for
Surgeons Standards Manual version 2.0. Resourc- him. Naltrexone HCl / Buproprion Hcl extended
es for optimal Care of the Metabolic and Bariatric release is contraindicated in patients with seizure
Surgery Patient 2016. https://facs.org. disorder, chronic opioid use, anorexia nervosa,
Higa KD, Ho T, Boone KB. Internal hernias bulimia, undergoing abrupt discontinuation of
after laparoscopic Roux-en-Y gastric bypass: alcohol, benzodiazepines, barbiturates or antiepi-
incidence, treatment and prevention. Obes Surg. leptic drugs because buproprion lowers the seizure
2003;13(3):350. threshold. Liraglutide contraindicated with
Iannuccilli JD, Grand D, Murphy BL, Evange- personal or family history of medullary thyroid
lista P, Roye GD, Mayo-Smith W .Sensitivity and carcinoma or MENII. In addition,GLP1 agonists
specificity of eight CT signs in the preoperative can increase the risk of pancreatitis in patients
diagnosis of internal mesenteric hernia following with a history of pancreatitis. Phentiramine/
Roux-en-Y gastric bypass surgery. Clin Radiol. topiramate can increase the risk of nephrolithia-
2009;64(4):373. Epub 2008 Dec 16. sis. All of these medications are contraindicated in
pregnancy and in patients with hypersensitivity to
the drug and drug class.
Question 51
A 54-year-old male is referred to you for advice REFERENCES
on weight loss management. His BMI is currently Steelman, M and Westman, E. Obesity Evaluation and
37kg/m2 and exercises regularly and is interested Treatment Essentials. Boca Raton: CRC press, 2016.
in starting medications for weight loss. He is a Bays HE, Seger JC, Primack C, McCarthy W,
chronic alcoholic which history of pancreatitis Long J, Schmidt SL, Daniel S, Wendt J, Horn DB,
in the past and few admissions for manage- Westman EC: Obesity algorithm, presented by the
ment of alcohol withdrawal including seizures. Obesity Medical Association. www.obesityalgo-
However he has maintained his job as a cook at rithm.org. 2016-2017.
the local diner. Then only other past history is Liraglutide Prescribing Information(Saxenda)
kidney stones as a teenager. He recently visited http://novo-pi.nnittest,com/saxenda.pdf
his primary care physician who “cleared” him. Naltrexone HCL/Bupripropion HCL Extended
He remember going for a sonogram of the heart Release Prescribing Information (CONTRAVE)
which was normal. He claims that he has been http://general.takedapharm.com/content/file.aspx
depressed about his brother’s recent diagnosis of Phentiramine HCl/Topiramate Extended
thyroid cancer and vowed to stop drinking and Release Prescribing Information (Qsymia) http://
lose weight. Which of the following medications www.vivus.com/docs/qsymia.pdf Locaserin (Bel-
is the best option for him? viq) Prescribing Information. http.belviq.com/pdf/
belviq_prescribing_information.pdf
A. Naltrexone HCL/Buproprion HCl ER (Con-
trave)
B. Liraglutide (Saxenda) Question 52
C. Phentiramine HCl/ Topiramate ER (Qsymia) A 21-year-old African American male with no
D. Lorcaserin (Belviq) significant medical problems except for borderline
Chapter 15 — Nutrition, obesity and eating disorders 457

hypertension controlled on lisinopril comes to you recommended to increase aerobic activity to 300
for advice on weight loss. He has always main- minutes a week (five hours) of moderate intensity
tained a BMI of 24.5kg/m2 until about one year or 150 minutes a week of vigorous intensity exer-
ago when gradually gained weight as he started cises plus resistance training two days a week.
university where he reports to high level of stress
and poor eating habits. BMI is now 30kg/m2, his REFERENCES
body fat is 25 percent and his waist circumfer- U.S. Department of Health and Human Services.
ence is 40 inches. His current exercise program Physical Activity Guidelines for Americans. Office
includes 30 minutes of brisk walking three days a of Disease prevention and Health Promotion,
week and he lifts weights twice a week. Recently 2008. https://health.gov/paguidelines/2008/
he decided to join and gym and has a trainer to summary.aspx
help him lose weight. Slentz CA, Duscha BD, Johnson JL, Ketchum
K, Aiken LB, Samsa GP, Houmard JA, Bales CW,
How would you advise him to develop a good exer- Kraus WE Effects of the amount of exercise on
cise program for weight loss? body weight, body composition, and measures of
central obesity: STRRIDE--a randomized con-
A. Moderate intensity exercise 75min/week plus trolled study. Arch Intern Med. 2004;164(1):31.
two days a week muscle strengthening activi-
ties
B. Moderate intensity exercise 150min/week
plus two days a week muscle strengthening
activities
C. Moderate intensity exercise 300min/week
plus no muscles strengthening activities
D. Vigorous intensity exercise 150min/week
plus two days a week muscle strengthening
activities

CORRECT ANSWER: D

RATIONALE
This patient went from normal weight to class 1
obesity (BMI 30-34.9). By measurements of body
fat and waist circumference he is now classified as
obese (greater than or equal to 25 percent and 40
inches or more respectively).
USDHHS recommends for substantial health
benefits adults do at least 150min (two hours
and 30 minutes) a week of moderate intensity or
75 minutes (one hour and 15 minutes) a week of
vigorous intensity or an equivalent combination of
moderate and vigorous intensity aerobic activity.
Adults should do muscle strengthening activi-
ties of moderate or greater intensity involving all
major muscle groups on two or more days a week.
For extensive benefits such as weight loss, it is
458 Digestive Diseases Self-Education Program®
Answers & critiques

CHAPTER 16

Issues in pediatric
gastroenterology
Nancy McGreal, MD and Mary Kathleen Rogers Boruta, MD

Question 1 Turner syndrome, William’s syndrome, selective


Celiac disease is a form of autoimmune enter- IgA deficiency and in individuals with Sjogren’s
opathy associated with the ingestion of gluten in disease as well as thyroiditis. There is no known
wheat, rye and barley. The prevalence of celiac association between a risk of celiac disease with
disease in the United States is estimated to be neurofibromatosis, Grover’s disease, multiple
1:133 of Caucasians of European descent. There sclerosis of neurofibromatosis. The frequency
are medical conditions with an increased risk with which high risk populations need to be
of celiac disease which warrant screening in as- screened depends upon the individual condition,
ymptomatic individuals. Which of the following symptoms and family history of celiac disease.
conditions is associated with an increased risk of
celiac disease? REFERENCE
Guandalini, S. (2005). Chapter 18: Celiac Dis-
A. Type 1 diabetes ease. In Essential Pediatric Gastroenterology,
B. Grover’s disease Hepatology and Nutrition (pp221-239). New
C. Multiple sclerosis York, NY: McGraw-Hill.
D. Wegner’s granulomatosis
E. Neurofibromatosis Question 2
A 10-year-old boy presents to the office with
CORRECT ANSWER: A abrupt onset of watery, non-bloody diarrhea,
flatulence and decreased appetite. His mother
RATIONALE reports that these symptoms evolved after he
Celiac disease has a strong genetic underpin- attended a Boy Scout camp where he swam in a
ning. In order for an individual to develop celiac fresh water lake and food was prepared on camp-
disease, they must possess either an HLA DQ fire grills. He denies associated fevers, chills,
2 or DQ 8 allele. There is a known association nausea, vomiting or significant abdominal pain.
between the development of celiac disease and There is no other history of international travel,
other autoimmune conditions. Studies have antibiotic exposure or prior gastrointestinal ill-
demonstrated that approximately 10 percent of nesses. In the office, he is nontoxic appearing
patients with type 1 diabetes have typical fea- and mildly dehydrated. He is afebrile with stable
tures of celiac disease on duodenal biopsy. The vital signs and abdominal exam is benign. At the
prevalence of Down syndrome and celiac disease end of the interview, his mother also offers that
is approximately five to 12 percent. There is also she has heard from other parents some of their
a known increased risk of celiac disease with children are experiencing similar symptoms.

459
460 Digestive Diseases Self-Education Program®

Which of the following conditions would be high- perianal pain. His primary care provider per-
est on your differential diagnosis? formed laboratory studies which ruled out enteric
pathogens, but did show evidence of anemia with
A. Inflammatory bowel disease hemoglobin 9.7g/dL, thrombocytosis with plate-
B. Celiac disease lets 535x109/L, ESR 27 and CRP 5.76. His physi-
C. Giardiasis cal examination demonstrates a palpable mass in
D. E. Coli 0157:H7 the right lower quadrant as well as one draining
E. Irritable bowel syndrome perianal fistula without evidence of abscess. Up-
per endoscopy is endoscopically and histologically
CORRECT ANSWER: C normal. Colonoscopy demonstrates diffuse, scat-
tered inflammation with aphthous ulcers, ery-
RATIONALE thema and contact bleeding throughout the colon.
The abrupt onset of symptoms, exposure to The ileocecal valve appears to be mildly stenosed
potential pathogens in a freshwater lake as well but is able to be intubated and shows moderate
as other afflicted individuals strongly suggests inflammatory changes with erythema, edema and
an infectious etiology rather than chronic condi- aphthous ulceration. MR enterography reveals a
tions such as inflammatory bowel disease, celiac right lower quadrant phlegmon with mild inflam-
disease or irritable bowel syndrome. Given the matory stenosis. Based upon this information,
mild severity and lack of rectal bleeding, E. coli you diagnose stricturing ileocolonic and perianal
0157:H7 is less likely, making giardiasis of greater fistulizing Crohn’s disease. What is your first line
concern. Giardiasis is a protozoal infection spread of medical treatment for this young gentleman?
through the fecal oral route most commonly via
contaminated food or water. The incubation A. Oral and rectal mesalamine
period is typically one to two weeks after ingestion B. Thiopurine
of a contaminated substance. Stool testing can be C. Methotrexate
performed to evaluate for ova and parasites and D. Ciprofloxacin and metronidazole
microscopy. More sensitive testing using nucleic E. Biologic therapy
acid amplification testing (NAAT) for Giardia can
be performed for quicker analysis. The first-line CORRECT ANSWER: E
treatment is metronidazole adjusted to the child’s
weight. Alternate therapies may include tinida- RATIONALE
zole, nitazoxanide, mebendazole, albendazole, and The patient in the above scenario presents with
paromomycin. a phenotype of moderate to severe stricturing
ileocolonic and perianal fistulizing Crohn’s disease
REFERENCES with associated phlegmon which warrants treat-
Dunn et al. StatPearls [Internet]. Treasure Island ment with biologic therapy. Studies have demon-
(FL): StatPearls Publishing; 2018 Jun 26. strated that biologic therapy initiated early in the
Bagarazzi M, Bell L (1998). Chapter 35: Para- course of complicated Crohn’s disease portends
sites. In Clinical Pediatric Gastroenterology (pp247- the best prognosis. Young gentleman and indi-
253). Philadelphia, PA: Churchill Livingstone. viduals of African-American descent have been
documented to have more aggressive courses in
Crohn’s disease. Cochrane meta-analyses have not
Question 2 shown benefit of mesalamine agents in patients
A 17-year-old African-American male presents to with Crohn’s disease. Thiopurines, 6-mercapto-
the office with a two-month history of abdomi- purine and as azathioprine, have been associated
nal pain, bloody diarrhea, anorexia, fatigue and with the development of hepatosplenic T-cell
Chapter 16 — Issues in pediatric gastroenterology 461

lymphoma in young gentleman with IBD and are You decide to admit him to the hospital for further
preferred to be avoided if possible. Methotrexate, evaluation. Which of the following is an appropri-
while useful for the treatment of luminal Crohn’s ate consideration of management of acute severe
disease, is not as efficacious as biologic therapy in colitis?
the setting of fistulizing Crohn’s disease. It also
has risks of hepatotoxicity and teratogenicity in A. Add a probiotic
young individuals of childbearing age which need B. Treat for C. difficile in the setting of negative
to be considered in decision making. Antibiotics, testing because it is a common complicating
such as ciprofloxacin and metronidazole, can be factor in IBD
utilized as adjuvant therapy in the setting of mild C. Consider flexible sigmoidoscopy if there is a
colonic Crohn’s disease but would not be effica- lack of response to IV corticosteroids to rule
cious to mitigate the inflammation of moderate to out complicating cytomegalovirus infection
severe Crohn’s disease as monotherapy. (CMV)
D. Start exclusive enteral nutrition therapy
REFERENCES E. If there is a lack of response to IV corticoste-
Dotson J et al. Inflamm Bowel Dis 2017 roids within five to seven days consider rescue
May;23(5):767-774. therapy with vedolizumab
Ford, AC et al. Am J Gastroenterol. 2011
Apr;106(4):617-29. CORRECT ANSWER: C
De Zoeten EF, Pasternak BA, Mattei P, Kramer
RE, Kader HA. J Pediatr Gastroenterol Nutr. 2013 RATIONALE
Sep;57(3):401-12bpm. The medical management of pediatric and adoles-
cent acute severe colitis is similar to adult man-
agement algorithms. In a multicenter study of
Question 3 children with ulcerative colitis, 15 percent de-
An 18-year-old male diagnosed with mild left- veloped acute severe colitis within three months
sided ulcerative colitis two weeks ago presents of diagnosis. Further population-based studies
to the office with worsening diarrhea, abdominal have demonstrated that nearly 30 percent of
pain, hematochezia, urgency and tenesmus. After children diagnosed with ulcerative colitis may
his biopsy results were available he was started on require hospitalization within the first three years
treatment with dose optimized oral mesalamine. of disease onset. It is always appropriate to rule
When his symptoms worsened one week later, out complicating enteric infections especially
mesalamine enemas were added to his regimen C. difficile in the setting of worsening ulcerative
for synergistic topical and oral rectal therapy. On colitis. In an individual who was recently started
physical examination, he is afebrile but tachy- on mesalamine preparations, either oral or rectal,
cardic with a heart rate of 115 bpm and borderline it is appropriate to consider whether or not they
hypotensive with a blood pressure of 100/85 mm are experiencing a hypersensitivity reaction to the
Hg. Clinically, he appears to be dehydrated with mesalamine agent which worsens their disease.
dry mucous membranes. Abdominal exam is In this setting, discontinuation of the mesalamine
noteworthy for tenderness in the bilateral lower therapy is appropriate. IV corticosteroids in the
quadrants. Laboratory studies are concerning for form of solumedrol or hydrocortisone are the
a white blood cell count 15.4K/mL, hemoglobin mainstay of rescue therapy. Studies indicate that
8.0 g/dL (previously 11.0 two weeks ago), platelets patient should demonstrate an improvement with-
554K/mL, ESR 59 and CRP 2.43. Stool studies in three to five days. In the pediatric population,
are negative for complicating enteric pathogens it is recommended that practitioners monitor the
including bacterial stool culture and C. difficile. Pediatric Ulcerative Colitis Activity Index (PUCAI)
462 Digestive Diseases Self-Education Program®

which measures abdominal pain, rectal bleeding, ents report that she has had symptoms of reflux
stool consistency, number of bowel movements since birth. She was started on a proton pump
per day, nocturnal stooling and impact on activity inhibitor by her primary care doctor which has im-
level. A flexible sigmoidoscopy is recommended if proved her fussiness but not resolved the volume
there is no improvement with IV corticosteroids of emesis. Her appetite remains stable and there
by day three to rule out CMV infection. A PUCAI is no report of growth failure. On physical ex-
score greater than 45 points after the third day amination, there is notation of a small, palpable
of intravenous corticosteroids indicates the need mass in her abdomen. Her parents report that
for consideration of rescue therapy. Second line she is stooling normally and deny any diarrhea or
salvage therapy should not be delayed beyond the constipation. Laboratory studies demonstrate hy-
fifth day of IV corticosteroid treatment. Rescue pochloremic and hypokalemic metabolic alkalosis.
options include infliximab as a recommended She is not anemic, nor are there any abnormalities
therapy for both induction and maintenance of of her liver or kidney function.
remission. Due to hypoalbuminemia, it is sug-
gested that an induction dose of 10 mg/kg may be Which diagnosis which would you be most con-
preferred in the setting of acute severe ulcerative cerned about at this juncture?
colitis. Calcineurin inhibitors including cyclo-
sporine and tacrolimus can also be considered in A. Typical infantile gastroesophageal reflux
individuals who are not previously exposed to bio- B. Pyloric stenosis
logics or immune modulators. The data regard- C. Cyclic vomiting syndrome
ing accelerated infliximab dosing in the setting D. Celiac disease
of acute severe ulcerative colitis remains mixed. E. Inflammatory bowel disease
In the adult literature, the short-term colectomy
rate is decreased, however, at two years colectomy CORRECT ANSWER: B
rates have been no different between individuals
who received accelerated infliximab dosing in the RATIONALE
setting of acute severe ulcerative colitis than those Infantile hypertrophic pyloric stenosis results in
who received standard dosing or 10 mg/kg at 0, obstruction of the pyloric channel. The etiology of
two and four weeks. Vedolizumab is an anti-inte- the condition is unknown but may be associated
grin therapy which is gut specific but slow acting. with failure of nitric oxide synthetase. It is typi-
At the current time, it is not recommended as an cally diagnosed in early infancy most commonly
induction agent in the setting of either children between two to eight weeks of age. While many
or adults hospitalized with acute severe ulcerative infants have symptoms of acid reflux, infants with
colitis. Preliminary studies have suggested that pyloric stenosis have more profuse and projectile
individuals may be able to be induced with cyclo- emesis sometimes with evidence of bile. There
sporine followed by vedolizumab only as a mainte- can be a palpable mass on examination which has
nance agent. been described as an “olive” in the mid abdomen.
Classic laboratory studies indicate hypochloremic
REFERENCE and hypokalemic metabolic acidosis. Ultrasonog-
Turner D et al. JPGN 2018; 67(2): 292-310. raphy is regarded as the diagnostic gold stan-
dard measure which demonstrates evidence of a
hypertrophied pylorus showing a muscle thickness
Question 4 of 4 mm or more in a pyloric channel of 16 mm or
A six-week old female, term infant presents to the more. While typical infantile GERD can present
emergency department with symptoms of recur- with symptoms of recurrent regurgitation, it is
rent, non-bloody, non-bilious vomiting. Her par- not usually associated with laboratory abnormali-
Chapter 16 — Issues in pediatric gastroenterology 463

ties. Cyclic vomiting syndrome and inflamma- RATIONALE


tory bowel disease are very rare in this age group. The differential diagnosis of hematochezia in
Similarly, celiac disease is also uncommon in this infants is relatively small. The most likely con-
age group as most infants have not been exposed siderations are anal fissures, vascular malforma-
to significant amounts of gluten to warrant evalu- tions, cow milk protein soy intolerance, bleeding
ation for this diagnosis. In an infant with recur- diatheses, swallowed maternal blood in the first
rent vomiting, small bowel malrotation should one to two days of life, and necrotizing enterocolitis
be considered and a barium upper GI series is an in preterm infants. In the setting of an otherwise
appropriate portion of such an evaluation. healthy term infant who presents with hematoche-
zia without anorectal malformations the most likely
REFERENCE etiology is cow milk protein soy intolerance. This
Hollands C, Hoffman M (1998). Chapter 21: is an IgG mediated disorder that does not neces-
Congenital Anomalies of the Intestine. In Clinical sarily construe other predilections to food allergies.
Pediatric Gastroenterology (pp155-25168). Phila- Most infants outgrow this by one year of life or
delphia, PA: Churchill Livingstone. thereafter. In mother’s who are breast-feeding, it is
recommended that they eliminate both cow and soy
milk proteins from their diet. There is a 70 percent
Question 5 cross reactivity between cow and soy milk proteins.
A six-week-old otherwise healthy, female term In infants who are formula feeding or those who
infant presents to the office for evaluation of do not respond to maternal elimination diets, it is
hematochezia. Her pre-and perinatal course was recommended that they consume partially hydro-
uncomplicated. Her mother has been breast- lyzed or fully hydrolyzed formula. Such infants are
feeding her and noted evidence of small streaks usually able to tolerate cow and soy proteins later
of blood in her diaper with some mucus over the in life.
last one to two weeks. There have been no asso-
ciated fevers, chills, nausea, vomiting or abdomi- REFERENCE
nal pain. She is otherwise breast-feeding well Mäkinen OE, Wanhalinna V, Zannini E, Arendt
and her mother has not introduced any formulas. EK. Crit Rev Food Sci Nutr. 2016;56(3):339-49.
There is no report of bleeding diatheses. She has
no bruising or other abnormalities. Her mother
is very concerned. At this juncture what is your Question 6
next recommendation? A 10-year-old boy has complained of symptoms of
chronic recurrent abdominal pain for two months.
A. Reassurance and consideration of cow milk His primary care doctor started him on a proton
protein soy intolerance with elimination of pump inhibitor for concern of possible gastritis or
these antigens in mother’s diet GERD. Both his mother and grandmother who
B. Cross-sectional imaging with CT scan of reside in their residence with him were recently
abdomen and pelvis diagnosed with helicobacter pylori infection. His
C. Consideration of celiac disease with testing and mother would like him to be tested for H. pylori.
recommendation for gluten-free diet Which of the following would be your recommen-
D. Consideration of lactose intolerance and dation regarding testing for H. pylori infection in
elimination of lactose from diet the pediatric population?
E. Consideration of a LOW FODMAP diet
A. You can reassure his mother that H. pylori
CORRECT ANSWER: A infection is rare in the pediatric population and
does not require testing
464 Digestive Diseases Self-Education Program®

B. Draw a serum H. pylori IgG utilizing a urea breath test or stool antigen test.
C. Request the child submit a stool specimen for Prior to performing noninvasive studies to confirm
H. pylori stool antigen now eradication children should be off of proton pump
D. Advise the mother to discontinue PPI therapy inhibitors for at least two weeks and antibiotics for
for two weeks and return for a urea breath test at least four weeks.
E. Schedule an upper endoscopy with biopsy with
immunohistochemistry or culture REFERENCE
Jones N et al. JPGN 2017; 64(6): 991-1003
CORRECT ANSWER: E Kotilea K et al. Pediatr Drugs 2018; 20: 337-351.

RATIONALE
H. pylori infection is not uncommon in the pediat- Question 7
ric population. Seroprevalence rates from stud- Which of the following is TRUE regarding Clos-
ies have suggested it that affects 33-37 percent of tridioides difficile (C. difficile) infection in infants
children. This varies depending upon worldwide and children?
location as well as socioeconomic status. Higher
rates of infection are typically associated with low A. The incidence of C. difficile infection in
to low-middle class populations and common in children in the United States has declined over
areas or countries with a severe income inequality. the last few decades
Epidemiologic studies have demonstrated that the B. Antibiotics are not an important risk factor of
prevalence of H. pylori infection in both children initial C. difficile infection in children
and adults appears to be declining and developed C. C. difficile testing in infants less than one year
countries. This may be related to changes in gastric of age is not recommended due to high rates of
microbiota, economic development and/or the non-pathogenic colonization
increase in incidence of immunological disorders D. C. difficile in children usually only occurs once
in children. Therefore, it would not be appropriate and recurrent C. difficile infection is not seen
to dismiss this as a possible diagnosis especially E. Fecal microbiota transplant cannot be utilized
in light of the pertinent family history. A variety in children with recurrent C. difficile
of noninvasive tests are available including stool
antigen testing as urea breath testing. Studies have CORRECT ANSWER: C
demonstrated that H pylori infection is not associ-
ated with symptoms in the absence of peptic ulcer RATIONALE
disease in the pediatric population. For this reason, Clostridioides (formerly Clostridium) difficile (C.
a test and treat strategy utilizing noninvasive difficile) is a spore forming, anaerobic gram-pos-
methods is not recommended. Current pediatric itive bacillus. Affect humans either from contact
guidelines recommend that the diagnosis of H. in the environment or through fecal oral transmis-
pylori infection be based upon histopathology con- sion. The bacteria produces two toxins, toxin A
sistent with H. pylori positive gastritis plus at least and toxin B, which are believed to be a primary
one other type test and/or culture. Therefore, the component and causing infection. Studies have
recommended diagnostic testing would be an upper demonstrated that children can be infected with
endoscopy examination. Serologic testing for H. C. difficile and in fact the incidence has increased
pylori IgG would not necessarily construe current in the United States in the pediatric population.
infection and may represent prior resolved infec- Exposure to antibiotics appears to be a significant
tion and is not recommended as a diagnostic test. risk factor for initial C. difficile infection. Those
It is recommended that eradication of H. pylori be most commonly implicated are third and fourth
assessed four weeks after completion of treatment generation cephalosporins, clindamycin and fluo-
Chapter 16 — Issues in pediatric gastroenterology 465

roquinolones. When considering testing a child past medical history noteworthy for anxiety and
for C. difficile diarrhea it is important to recognize depression. She has been seen by two previous
that testing and infants less than one year of age gastroenterologists with normal laboratory test-
is not recommended due to high rates of coloniza- ing, endoscopic evaluation and cross-sectional
tion. In the current literature, it is estimated that imaging in the form of ultrasound, CT scan and
37 percent of infants were less than one month of MRI of the abdomen and pelvis. Her family feels
age are colonized with C. difficile and are asymp- frustrated and desperate that no one has identi-
tomatic. Young infants do not possess receptors fied an etiology of her symptoms. You perform
for the toxins generating C. difficile diarrhea and additional testing which reveals normal fecal
are therefore asymptomatic. By two years of age, calprotectin, studies for porphyria and have ruled
toddlers possess similar microbiota profiles to out median arcuate ligament syndrome. She has
adults and therefore may be at risk. The clinical missed 20 days of school and her parents request
presentation is similar to adults with diarrhea homebound teaching and narcotic medication.
which may or may not be associated with hema- Which of the following is the most appropriate
tochezia or abdominal tenderness. American management strategy?
Academy of Pediatrics guidelines recommend oral
metronidazole for initial treatment and first recur- A. Obtain an EKG and consider a trial of a
rence of mild to moderate disease. If a child has tricyclic antidepressant in the form of
severe C. difficile infection or a second recurrence amitriptyline or nortriptyline
of C. difficile, oral vancomycin is the preferred B. Prescribe a long acting narcotic to manage pain
agent. For recurrent C. difficile pulse tapered C. Recommend indefinite homebound schooling
vancomycin is an option as well as consideration D. Suggest surgical laparoscopy for an
of fidaxomicin in children over the age of six years. unidentified source of abdominal pain
Fecal microbiota transplant has been studied in E. Repeat upper endoscopy, colonoscopy,
children with recurrent C. difficile who have not CT and MRI to reassure parents
responded to standard antibiotic regimens and
found to be efficacious. CORRECT ANSWER: A

REFERENCES RATIONALE
D’Ostroph A et al. Infection and Drug Resistance Chronic recurrent abdominal pain that is functional
2017: 365-375. in nature is an exceedingly common condition in
Crews J and Kaplan S (2018). Clostridioi- pediatrics. It is a diagnosis of exclusion and due
des (formerly Clostridium) difficile infection in diligence should occur with additional testing espe-
children: Clinical features and diagnosis. In M cially in the presence of alarm symptoms of overt
Torchia (Ed.) UptoDate. Retrieved October 12, gastrointestinal symptoms, growth failure or other
2018 from https://www.uptodate.com/contents/ extraintestinal manifestations. When children
clostridioides-formerly-clostridium-difficile-infec- present with symptoms of recurrent abdominal
tion-in-children-treatment-and-outcome pain, it is appropriate to evaluate for conditions
such as inflammatory bowel disease, celiac disease,
H. pylori infection, chronic pancreatitis, forms of
Question 8 carbohydrate malabsorption, small bowel bacte-
A 14-year-old girl presents with symptoms of rial overgrowth syndrome and unusual and rare
chronic abdominal pain for four years. It is peri- etiologies as described in the scenario above if
umbilical in nature and not associated with fevers, appropriate. When other medical etiologies have
chills, nausea, vomiting, diarrhea or abdominal been ruled out, it is appropriate to review with
pain. Her growth is stable. She has a significant families the management of functional, chronic
466 Digestive Diseases Self-Education Program®

recurrent abdominal pain which may include medi- ogies may include nonsteroidal anti-inflammatory
cal therapeutics in the form of antispasmodics, use, corticosteroid use as well as individuals being
tricyclic antidepressants, gabapentin, pregabalin or critically ill in the pediatric intensive care unit are
topiramate. Some patients may require multidisci- all risk factors for the development of gastritis.
plinary therapy including psychological counseling, Psychological stress has not been associated with
biofeedback or other multimodality therapies. In the development of physiologic changes of gastri-
general, the psychosocial isolation resulting from tis and there have been no documented studies
remaining out of school is not in the best interest to indicate in association with serotonin reuptake
of most children with these conditions. Narcotic inhibitors. While children with excess psychosocial
medications are also not recommended due to stress may experience abdominal pain in the form
issues related to dependence and the possibility of of visceral hypersensitivity it does not necessarily
the development of narcotic bowel syndrome. correlate with histological changes of gastritis. An-
tibiotic use in children has not been associated with
REFERENCES the development of gastritis.
Rajindrajith S, Zeevenhooven J, Devanarayana
NM, Perera BJC, Benninga MA. REFERENCE
Expert Reviews in Gastroenterolology and Sierra D et al. Pediatrics in Review 2018; 39(11):
Hepatology. 2018 Apr;12(4):369-390. 542 – 547.

Question 9 Question 10
A common cause of gastritis in the pediatric A five-year-old boy is admitted to the hospital with
population is: a three-day history of abdominal pain, arthralgias
and a purpuric rash which developed over the but-
A. Helicobacter pylori infection tocks and lower extremities. His parents report
B. Anxiety that prior to the onset of his symptoms he had
C. ADHD medications what appeared to be a viral illness. Laboratory
D. Serotonin reuptake inhibitors (SSRIs) studies disclose mild leukocytosis with a normal
E. Antibiotics hemoglobin and platelet count. ESR is signifi-
cantly elevated to 54. Urinalysis shows evidence
CORRECT ANSWER: A of both proteinuria and hematuria. ANA, rheu-
matoid factor and complement levels are normal.
RATIONALE He is diagnosed with Henoch-Schönlein Purpura
Gastritis, inflammation of the gastric intestinal mu- (HSP). Despite nonsteroidal anti-inflammatory
cosa can occur in children. The incidence of peptic medications (NSAIDS), his abdominal pain is
ulcer disease in the pediatric population is estimat- not improving and nephrology would like to limit
ed to range from two to eight percent. The most exposure to NSAIDs due to concern for renal in-
common etiologies of gastritis in children are due volvement. What is the next appropriate step for
to H. pylori infection, corticosteroid use and immu- managing his abdominal pain?
nosuppressive medications. Studies have demon-
strated that there can be a predilection towards the A. Narcotic pain medication
development of gastritis based upon Race ethnicity. B. Corticosteroids
Non-Hispanic black and Mexican American popu- C. Infliximab
lations have been shown to have a higher rate of D. Cyclophosphamide
gastritis. Among the answers presented, H. pylori E. He does not require further medical therapy as
infection is the most common etiology. Other etiol- HSP usually resolves on its own
Chapter 16 — Issues in pediatric gastroenterology 467

CORRECT ANSWER: B REFERENCE


Keenan G (1998). Chapter 28: Henoch-Schonlein
RATIONALE Purpura. In Clinical Pediatric Gastroenterology
HSP is an immune complex mediated disease. (pp 197-204). Philadelphia, PA: Churchill
Circulating IgA and activated complement im- Livingstone Song Y et al. World J Gastroenterol
mune complexes deposit in organs such as the 2015; 21(19): 6082-6087.
skin, kidney and GI tract resulting in symptoms
of palpable purpura, renal involvement and
abdominal pain. It typically occurs in individuals Question 11
between two to 12 years of age with a peak inci- A two-year-old male presents to the emergency
dence between four to seven years of age. Males department with the abrupt onset of painless,
are almost twice as commonly affected as females. rectal bleeding. His parents report that he has no
By history there is often report of an antecedent other medical issues and he was in his usual state
infectious illness which likely stimulates the im- of health until earlier in the afternoon when he be-
mune response. Skin manifestations of palpable gan passing a profuse amount of bright red blood
purpura are the most common clinical symptom. and clots of stool in his diaper. He has not mani-
Some children, however, will complain of abdomi- fested symptoms of abdominal pain, fevers, chills,
nal pain that may be severe in nature prior to the nausea, vomiting or abdominal pain. He has no
development of cutaneous lesions. The arthralgias history of bleeding diathesis. There is no report
associated with HSP tend to be periarticular and of epistaxis, gum bleeding, easy bruising or liver
usually without effusion. While renal complica- disease. He has not been taking routine NSAIDs.
tions are a concern, the glomerulonephritis of HSP Laboratory studies performed in the emergency
often resolves within a few months. The develop- department are largely normal with the exception
ment of chronic kidney disease is rare. of mild anemia with a hemoglobin of 10.1 g/dL.
The mainstay of treatment is symptom relief He has a normal comprehensive metabolic panel,
and prevention of complications of HSP. NSAIDs ESR, CRP, PT, PTT and INR.
can be used such as ibuprofen to manage both
joint and abdominal pain but may need to be used What is the most likely diagnosis?
with caution in children who are demonstrating
renal involvement. When children do not respond A. Inflammatory bowel disease
to first-line therapy with NSAIDs, corticosteroids B. H. pylori infection
at a dose of prednisone 0.5 mg/kg per day may C. Intussusception
be efficacious to mitigate both abdominal and D. Meckel’s diverticulum
joint symptoms. Often these can be used in short E. Hemorrhoidal bleeding
courses of one to three weeks with the ability to ta-
per and improvement in symptoms. Narcotic pain CORRECT ANSWER: D
medications are generally avoided. This is par-
ticularly so given the potential for gastrointestinal RATIONALE
complications such as ileus and intussusception The abrupt onset of painless rectal bleeding is
which could be masked by narcotic pain medi- uncommon in children. In children with profuse,
cation. Infliximab is not utilized as a standard painless rectal bleeding the most likely consider-
treatment for HSP and in fact there have been case ation is Meckel’s diverticulum as well as arteriove-
reports of the development of HSP in children nous malformations and duplication cysts. Minor
with IBD utilizing anti-TNF therapy. Cyclophos- painless rectal bleeding can be caused by polyps
phamide would only be indicated for severe renal and lymphoid nodular hyperplasia. Meckel’s di-
involvement. verticulum has an incidence of one to four percent
468 Digestive Diseases Self-Education Program®

with a male to female ratio of 2:1. The most for how she should best be followed. Which of
common presentation is in children two years of the following do you recommend?
age and location in the distal 100 cm of the ter-
minal ileum. Approximately 50 percent of diver- A. Because she is a known APC gene mutation
ticulae contain heterotopic gastric tissue which carrier, you begin annual surveillance
results in acid secretion and subsequent bleed- colonoscopies and upper endoscopy now
ing. Meckel’s diverticulum can be diagnosed B. Perform small bowel follow-through to
with 85-90 percent sensitivity with a radionucle- evaluate for the possibility of small bowel
otide 99 m TC pertechnetate scan demonstrating polyps
evidence of ectopic gastric mucosa in the right C. Recommend the child undergo abdominal
lower quadrant. Treatment require surgical re- ultrasound and AFP testing every six months
section. While inflammatory bowel disease can until the age of five due to risk of
contribute to lower GI bleeding, the abrupt onset hepatoblastoma in the setting of APC
and acute nature makes this less likely. H pylori mutations
and intussusception are typically associated with D. Schedule a thyroid ultrasound because there
abdominal pain. Hemorrhoids are an uncom- is an associated risk of thyroid cancer in the
mon occurrence in young individuals and are setting of FAP
generally not the cause of painless rectal bleed- E. Hemoccult her stools annually to decide
ing until at least adolescence. when you will begin endoscopic screening
and surveillance.
REFERENCES
Turk D, Michaud L (1998). Chapter 346.3b. CORRECT ANSWER: C
Lower Gastrointestinal Bleeding. In Walker’s
Pediatric Gastrointestinal Disease (pp 1314- RATIONALE
1315). Hamilton, Ontario: BC Decker. FAP is the most common inherited polyposis
Lirio R et al. Gastrointestinal Endoscopy syndrome in the pediatric population. The ge-
Clinics of North America 2016; 26: 63-73. netic underpinnings of FAP are due to a trun-
cated mutation in the adenomatous polyposis
coli (APC) gene. It is inherited in an autosomal
Question 12 dominant fashion, however, de novo mutations
A two-year-old otherwise healthy girl presents may occur in 20-30 percent of cases. In classic
to your office with her parents. Her father was FAP, innumerable colon polyps carpet the colon.
recently diagnosed with familial adenomatous Individuals with attenuated FAP may have fewer
polyposis syndrome (FAP) after he presented colon polyps. In families with history of FAP
with rectal bleeding and underwent ileocolonos- and known genetic mutation, related family
copy. He reports that his colonoscopy showed members can be referred for mutation testing.
innumerable polyps throughout his colon but no If no genetic mutation is known, it is recom-
evidence of dysplasia or colon cancer. Her father mended that children begin endoscopic surveil-
subsequently underwent genetic testing which lance between the ages of 10-12 years. Biannual
revealed a positive APC gene mutation variant endoscopic screening is recommended after the
associated with FAP. He has been referred for age of 10-12 years with intensification of screen-
colectomy. Their daughter was seen by a geneti- ing if the number of polyps significantly increas-
cist and underwent genetic testing which showed es or demonstrates worrisome histopathology.
that she possesses a similar APC gene mutation While gastric and duodenal lesions can occur in
as her father. Her parents are exceedingly wor- children with FAP, current guidelines recom-
ried about her prognosis and seek your advice mend beginning upper endoscopic surveillance
Chapter 16 — Issues in pediatric gastroenterology 469

between the ages of 20-25 utilizing a side-view- RATIONALE


ing scope to evaluate for periampullary tumors. Refeeding syndrome connotes a constellation
Performing an annual Hemoccult would not be of metabolic, cardiopulmonary and neurologic
indicated and not necessarily diagnostic for the complications which can be seen in severely mal-
presence of colon polyps. Similarly perform- nourished individuals after abrupt reinstitution
ing a small bowel follow-through to evaluate for of nutritional resources. Attention to hydration
small bowel polyps is not indicated. There is an and electrolyte abnormalities is critical before
800-fold increased risk of hepatoblastoma in nutritional restitution. Complications of refeed-
young children before the age of five years who ing syndrome may occur in the setting of admin-
possess APC mutations. Therefore, it is recom- istration of glucose, total parenteral nutrition as
mended that such children undergo liver ultra- well as enteral nutrition. Refeeding syndrome
sonography and AFP screening every six months is associated with a variety of electrolyte abnor-
until the age of five. While there is an increased malities including hypophosphatemia, hypokale-
risk of thyroid cancer associated with FAP, mia, hypomagnesemia and hypocalcemia. Low
this risk does not appear to increase until late levels of phosphorus in refeeding syndrome may
adolescence or adulthood and therefore there is be due to shifts of phosphorus from the extra-
no indication to start this form of screening in a cellular to intercellular spaces resulting in an
two-year-old. increase need for adenosine triphosphate (ATP)
metabolism. Due to its ubiquitous nature as an
REFERENCE electrolyte in multiple body systems, hypocalce-
Barnard J. JPGN 2009; 48 (Suppl 2); S72-S75 mia can result in cardiac, neuromuscular, respi-
Syngal S et al. Am J Gastroenterol 2015; ratory, hematologic and immunologic complica-
110:223–262. tions. Low levels of magnesium and potassium
are also believed to be related to intracellular
shifts. Magnesium is a critical cofactor in many
Question 13 enzymatic processes. Common symptoms of
A 14-year-old girl with a history of anorexia both hypokalemia and hypomagnesemia include
nervosa is admitted to the hospital for medical weakness, neuromuscular dysfunction and car-
stabilization prior to inpatient treatment in an diac arrhythmias. Hypocalcemia in the setting
eating disorders unit. She has manifested re- of refeeding syndrome is often associated with
strictive eating behaviors. Review of her growth hypomagnesemia. Many patients with refeeding
chart demonstrates that her BMI declined from syndrome are also hypoalbuminemic contribut-
21 to 13 kg/m2 over the last year. As part of her ing to hypocalcemia. Hyperglycemia may in-
medical evaluation you are concerned about the volve in malnourished individuals after glucose
possibility of refeeding syndrome. administration due to the body’s inability to
maintain normoglycemia. When reinitiating
Which of the following electrolyte abnormalities nutrition, it is appropriate to consider starting
is associated with refeeding syndrome? at 25-50 kcals/g or at 75 percent of estimated
resting energy expenditure. Careful monitoring
A. Hypermagnesemia of fluid balance, frequent electrolyte checks, as
B. Hyperphosphatemia well as monitoring cardiac and neurologic status
C. Hypokalemia is indicated.
D. Hyperglycemia
E. Hypercalcemia REFERENCES
Shamir R, Wilschanski M (1998). Chapter 11:
CORRECT ANSWER: C Malnutrition. In Clinical Pediatric Gastroen-
470 Digestive Diseases Self-Education Program®

terology (pp 81-87). Philadelphia, PA: Churchill distal tracheoesophageal fistula is the most com-
Livingstone. mon comprising 8.5 percent of cases. Most chil-
McCray S et al. Practical Gastroenterology dren with such malformations undergo surgical
2016 pp 56-66. repair in early infancy. Depending upon the type
and length of atresia, as well as fistula location,
there can be varied surgical outcomes. Long-term
Question 14 studies demonstrate that GERD is experienced by
A 20-year-old male with a history of congenital many patients. One study revealed ongoing GERD
type C tracheoesophageal fistula diagnosed at in 35-50 percent of children with associated
birth status post repair presents to your office to repairs. Heartburn remains present in up to 46
transition his care from his pediatric gastroen- percent of adults. There does appear to be an in-
terology provider to your office. Which of the fol- creased risk of Barrett’s esophagus, and therefore
lowing is a common long-term complication after long-term antacid therapy may be indicated both
repair of a tracheoesophageal fistulae that both for symptom relief and suppression of Barrett’s
adult and pediatric gastroenterologists should be esophagus if it is identified endoscopically. Up to
aware of? 75 percent of patients with congenital esophageal
atresia and tracheoesophageal fistula experience
A. GERD and future risk of Barrett’s esophagus esophageal dysmotility as documented by manom-
B. Eosinophilic esophagitis etry. Many children have difficulty with feeding
C. Schatzki’s ring disorders and need to learn how to eat slowly and
D. Zenker’s diverticulum swallow appropriately. Esophageal strictures
E. Esophageal inlet patch may occur postoperatively in six to 40 percent of
patients. Many patients require serial dilations
CORRECT ANSWER: A postoperatively. Respiratory complications are
also common after such surgical repairs. These
RATIONALE include tracheomalacia, recurrent pulmonary in-
Congenital esophageal atresia (EA) and/or tra- fections in childhood, bronchial reactivity as well
cheoesophageal fistula (TEF) are common con- as persistent pulmonary function abnormalities
genital abnormalities of the aerodigestive tract. particularly in some individuals where there can
The incidence of congenital esophageal atresia be a long-standing history of aspiration. There is
is estimated to be one in 3000-4500 live births. no definitive increased risk of eosinophilic esopha-
There are five different types of congenital esoph- gitis, Schatzki’s ring, Zenker’s diverticulum or
ageal atresia with associated tracheoesophageal esophageal inlet patch with EA or TEF.
fistula. See diagram below. As illustrated in this
vignette, type C proximal esophageal atresia with REFERENCES
Kovesi T et al. Chest 2004; 126(3): 915-925
Burjonrappa SC et al. E J Pediatr Surg 2011; 21(1):
25-29 Long AM et al. J Pediatr Surg 2017; 52(2):
226-220.
Burge K et al. Brit J Surg 2013; 100: 515-52.

Question 15
A 14-year-old girl presents to the office with symp-
toms of gas, bloating, abdominal discomfort and
intermittent diarrhea. She has no other antecedent
Chapter 16 — Issues in pediatric gastroenterology 471

medical problems and her growth is adequate. She bowel intestinal brush border over time. This
denies unexplained fevers, chills, nausea or vomit- is rare below the age of five years and more
ing. During dietary history review, you identify a common in teenagers and young adults. It
link between ingestion of dairy and her symptom- is also more common in individuals of non-
atology, prompting the possible diagnosis of lactose European descent.
intolerance. 4. Secondary lactose intolerance -this may be a
sequelae of small bowel injury due to etiology
Which of the following testing would you perform such as giardiasis, Crohn’s disease or celiac
to confirm her diagnosis? disease. This typically resolves with restitu-
tion of the small bowel epithelial border.
A. Milk skin prick testing
B. Lactose breath hydrogen testing The most common symptoms are as described
C. Stool eosinophil count above with a constellation of abdominal pain,
D. Increase volume of lactose in diet and evaluate diarrhea and bloating. At times there can be as-
if symptoms worsen. sociated nausea and vomiting with the ingestion of
E. Milk IgE RAST testing dairy-containing products.
A variety of diagnostic modalities are avail-
CORRECT ANSWER: B able including measurement of stool reducing
substances and stool pH. Stool reducing sub-
RATIONALE stances will be increased in the setting of lactase
Lactase non-persistence (ie; lactose intolerance) deficiency in the stool pH reduced below a level
is a common gastrointestinal condition affecting of 5.5 to six. Lactose breath testing is also avail-
up to 70 percent of the world’s population due to able. Breath samples are measured every 15-30
decline in lactase expression over time. Lactose minutes for up to three hours. The diagnosis of
is the main carbohydrate in human and mamma- lactose intolerance is substantiated by an increase
lian milk. It is the primary source of nutrition for in exhaled hydrogen greater than 20 parts/mm
mammalian infants. Lactose is metabolized by the from baseline. False negatives can occur in the
small bowel enzyme called lactase which resides setting of recent antibiotic use. The gold standard
on the small bowel brush border within entero- for diagnosing lactase deficiency is measurement
cytes. Lactase deficiency can be divided into four of duodenal disaccharidases during upper en-
main classifications: doscopy examination. Due to the nature of this
invasive testing, it is not necessarily performed as
1. Developmental lactase deficiency - Due to a first-line test unless there are other symptoms
maturational delay, premature infants born that need to be evaluated. It is very common in
before 34 weeks of life may have a deficiency clinical practice to recommend a lactose-free diet
of lactase to digest lactose and may have a and assess response; increasing lactose in the diet
transient form of lactose intolerance until they would not be appropriate. If symptoms resolve
reached an appropriate chronological age. this is both diagnostic and therapeutic at the
2. Congenital lactase deficiency (alactasia) - This same time. Cow’s milk allergy is different from
is an incredibly rare autosomal recessive dis- lactose intolerant pathophysiologically. Cow’s
order in which infants are unable to tolerate milk allergy is mediated by the IgE portion of the
lactose presenting with loose watery diarrhea, immune system and is a true allergy rather than
failure to thrive and electrolyte imbalances. an intolerance. Individuals diagnosed with true
3. Lactase non-persistence - The most com- cow’s milk allergy in infancy and childhood need
mon of lactose intolerance is associated with to avoid proteins and calcium milk including
decrement of lactase enzymes across the small albumin, lactate albumin and over albumin. Some
472 Digestive Diseases Self-Education Program®

children may outgrow this in childhood but others cal remission with normal laboratory, stool and
may have persisting issues related to a true cow’s radiographic studies. After small bowel surgi-
milk allergy. As such they should be followed cal resection, small bowel bacterial overgrowth
by an allergist in the long-term. Milk skin prick syndrome is a common complication in the setting
testing, stool eosinophil count and milk IgE RAST of Crohn’s disease and other chronic gastrointes-
testing are not indicated as there are tests for milk tinal conditions. It is always prudent to consider
allergy or cow milk protein soy intolerance. the possibility of recrudescence of Crohn’s disease
contributing to active symptoms after surgical
REFERENCE resection. When inflammatory disease develops
Heine et al. World Allergy Organization Journal after surgical resection it is most likely to occur at
2017; 10: 41-49. the site of surgery and anastomosis. In this set-
ting it is appropriate to rule out infectious etiolo-
gies contributing to flare symptoms and consider
Question 16 colonoscopy to reevaluate the colonic mucosa as
A 17-year-old young woman with a 10-year his- well as the surgical anastomosis. It is prudent to
tory of stricturing ileal Crohn’s disease status post inquire about NSAID use in all IBD patients due
ileocecal resection five years ago presents to your to the risk of flare. In this patient, enteric infection
office for evaluation and management of bloating, as already ruled out making C. difficile unlikely.
abdominal discomfort and diarrhea. She denies Lactose intolerance is less likely given her report
unexplained fevers, chills, nausea, vomiting, joint of tolerance to dairy.
pain, skin rashes or oral ulcers. Her growth is
adequate and she has normal menses. Stool studies REFERENCE
are negative for enteric pathogens. Since her sur- Quigley E et al. Infect Dis Clin N Am 2010; 24:
gery she has been maintained on adalimumab 40 943-959.
mg subcutaneously every two weeks with labora-
tory testing indicating a normal CBC, CMP, ESR,
CRP, fecal calprotectin, and therapeutic drug levels Question 17
without antibody formation. MR enterography is A two-week-old ex-23-week premature infant
normal. Re-staging upper endoscopy and colonos- being cared for her in the neonatal intensive care
copy are endoscopically and histologically normal. unit receiving infant formula develops symptoms
She denies NSAID use and sensitivity to dairy. of feeding intolerance, abdominal distention, loose
stool and hematochezia. The infant has manifest-
What is your most likely diagnosis? ed some regurgitation but no projectile vomiting.
There is no report of bilious emesis. Plain film
A. Small bowel bacterial overgrowth syndrome of the abdomen demonstrates evidence of ileus
B. NSAID enteritis with pneumatosis. The infant is overall clinically
C. C diff infection stable. Laboratory studies demonstrate anemia
D. Lactose intolerance and thrombocytosis.
E. Active Crohn’s disease
What is the most likely diagnosis?
CORRECT ANSWER: A
A. Pyloric stenosis
RATIONALE B. Inflammatory bowel disease
The correct answer is small bowel bacterial C. Juvenile polyp
overgrowth syndrome. Active Crohn’s disease D. Necrotizing enterocolitis
is unlikely diagnosis given evidence of surgi- E. C. difficile infection
Chapter 16 — Issues in pediatric gastroenterology 473

CORRECT ANSWER: D Toxigenic C. difficile infection is uncommon in


infants because they do not possess the receptor
RATIONALE for the toxins to result in pathologic C. difficile
The correct diagnosis is necrotizing enterocolitis. infection.
This is an inflammatory condition of the bowel
most common in premature infants, although REFERENCE
10 percent of term infants can also experience Kim, J, Abrams S. (2018). Necrotizing enteroco-
necrotizing enteric colitis. The etiology is not litis: Clinical features and diagnosis. In M.S. Kim
entirely defined but could be related to the devel- (Ed) UptoDate. Retrieved November 14, 2018
oping infant microbiome and impaired immune from https://www.uptodate.com/contents/neona-
system. Necrotizing enterocolitis is most common tal-necrotizing-enterocolitis-clinical-features-and-
in premature infants, particularly those less than diagnosis
26 weeks of gestation. There is an inverse relation
between age and the development of necrotizing
enterocolitis. Fortunately, with developments Question 18
in science there has been a decrease in necrotiz- Which of the following is a critical element of ini-
ing enterocolitis in the United States over the last tial management of acetaminophen toxicity?
decade. It has been associated with the use of
non-breast milk formulas as well as illnesses such A. Initial measurement of serum electrolytes,
as congenital heart disease, other GI disorders and acetaminophen, aspirin levels, blood and urine
sepsis. Clinically, there are three stages of nec- toxicology screens
rotizing enterocolitis. Stage I is characterized by B. Immediate referral for liver transplant
temperature instability, apnea, lethargy, abdomi- C. Initiation of IV N-acetylcysteine in cases 48-72
nal distention, vomiting and heme-positive stools. hours after ingestion
Stage II manifests with a diminishment of bowel D. Advise family to make patient do not resuscitate
sounds, progressive clinical illness and abdominal E. Disregard acetaminophen poisoning
distention. Grade 3 necrotizing enterocolitis is nomogram to determine risk of toxicity based
characterized by hypotension, bradycardia, sleep upon acetaminophen level and timing of
apnea as well as worsening pneumatosis and ingestion
pneumoperitoneum. It is diagnosed by clinical
features as well as radiographic findings. The vast CORRECT ANSWER: A
majority of infants with early stage disease will
resolve with withholding feedings and treatment RATIONALE
with broad-spectrum gram-negative antibiotics. Acetaminophen is commonly utilized for pain con-
Infants manifesting the symptoms should be ruled trol as well as an antipyretic. It is metabolized by
out for sepsis. The role of probiotics in the setting the cytochrome P450 system with acetaminophen
of necrotizing enterocolitis remains controversial being metabolized to metabolites of sulfate and
and are not necessarily recommended at this time. glucuronidate conjugates. The active metabolite
As for the other diagnoses listed, pyloric stenosis N- acetyl benzoquinone imine (NAPQI) is hepato-
may present with feeding intolerance but would toxic. Based upon nomograms delineating serum
not necessarily result in hematochezia. Inflam- concentration of acetaminophen and time of acute
matory bowel disease would be incredibly unusual ingestion, risk of toxicity can be delineated. At the
in this age group and less there was an early onset initial evaluation it is prudent to check serum elec-
IBD genetic syndrome. Juvenile polyps can pres- trolytes, acetaminophen level, aspirin level, urinal-
ent with rectal bleeding in childhood but typically ysis and urine toxicology screen. Liver enzymes
later in age and the toddler or grade school years. and coagulation factors should be evaluated.
474 Digestive Diseases Self-Education Program®

In the setting of acetaminophen toxicity is criti- brae. Abdominal ultrasound does not indicate
cal to measure serial electrolytes, liver enzymes hepatosplenomegaly. Based upon these findings
and acetaminophen levels at least every 12 hours. what is the most likely diagnosis of cholestatic
Mental status and physical examination should liver disease in an infant?
be assessed for evolution of acute liver failure. Of
the treatment options offered, if there has been A. Biliary atresia
documented ingestion of greater than 150 mg/ B. Alagille’s syndrome
kg within less than four hours of initial presenta- C. Rotor syndrome
tion, treatment with activated charcoal can be D. Dubin-Johnson syndrome
considered unless there are symptoms of gastro- E. Congenital CMV infection
intestinal obstruction, altered mental status or
airway issues. Individuals who present four to 24 CORRECT ANSWER: B
hours after ingestion should have an acetamino-
phen level checked and be treated with empiric RATIONALE
N-acetylcysteine. In patients who present more The vignette describes a child with Alagille
than 24 hours post ingestion or are at risk of liver syndrome characterized by triangular facies,
injury and failure, consideration should be given cholestatic jaundice, heart murmur and butterfly
to chronic acetaminophen poisoning. Treatment vertebrae. In the differential diagnosis it is critical
protocols typically utilize N-acetylcysteine for 21- to rule out other causes of cholestatic liver dis-
72 hours. Liver transplantation is rarely required ease including TORCH syndromes, biliary atresia,
in children or adolescents with acetaminophen alpha-1 antitrypsin deficiency, drug induced liver
poisoning compared to adults. Children should disease and Alagille disease. The presence of con-
have close monitoring of liver enzymes, coagula- genital malformations raises concern for a genetic
tion factors and mental status. anomaly. In the setting of abnormal facies, chole-
static jaundice and heart murmur, biliary atresia
REFERENCE as well as Alagille’s syndrome are considerations.
Heard K, Burns M. Management of acetaminophen Vertebral abnormalities are more closely associat-
poisoning in children and adolescents. In J.F. Wiley ed with Alagille’s syndrome making this the most
(Ed). UptoDate. Retrieved November 14, 2018 from likely diagnosis.
https://www.uptodate.com/contents/manage- Alagille syndrome is an autosomal dominant
ment-of-acetaminophen-paracetamol-poisoning- disorder caused by heterozygous mutations in
in-children-and-adolescents JAG1 and NOTCH2 genes in the Notch signaling
pathway. Histologically it results in bile duct pau-
city. The hallmark features of Alagille’s syndrome
Question 19 include characteristic triangle facies, congenital
A two-month-old term infant presents with cardiac disease, vascular anomalies, renal anoma-
symptoms of difficulty gaining weight. Physical lies and skeletal malformations. Ninety-six percent
examination demonstrates an alert infant with of individuals diagnosed with Alagille’s syndrome
mildly dysmorphic triangular facies and scleral will have an identified mutation in the JAG1 or
icterus. A 2/6 systolic ejection murmur is auscul- NOTCH2 gene. They will manifest with cholestatic
tated. No evidence of hepatosplenomegaly. The liver disease and often symptoms of pruritus. Dur-
child appears mildly jaundiced. Laboratory studies ing their evaluation it is critical that in addition to
demonstrate ALT 80 U/L, AST 70 U/L, alkaline laboratory studies, they undergo cardiology evalu-
phosphatase 200 U/L, total bilirubin 2.6 mg/dL, ation, renal ultrasound, radiographic assessment
direct bilirubin 1.7 mg/dL, GGT 258 U/L. Chest of the spine, nutritional assessment and genetic
x-ray shows evidence of possible butterfly verte- testing as detailed above. Twenty to 30 percent of
Chapter 16 — Issues in pediatric gastroenterology 475

patients with Alagille’s syndrome will ultimately the intrahepatic bile ducts. The etiology remains
require liver transplantation. unknown but it is suspected that it may be related
While biliary atresia is in the differential to a form of perinatal infection. Affected infants
diagnosis the clinical features described are more typically present with jaundice in the first months
characteristic of Alagille’s syndrome. Rotor syn- of life and direct hyperbilirubinemia. Charac-
drome and Dubin-Johnson syndrome are condi- teristic hallmarks on liver biopsy include portal
tions of indirect hyperbilirubinemia rather than tract edema, proliferation of bile ducts, bile duct
direct hyperbilirubinemia. Congenital CMV infec- plugs and fibrosis. Surgical management consists
tion, as part of the TORCH syndromes, should of hepatoportoenterostomy known as the Kasai
be considered but is less likely in the absence of procedure. This involves dissection of the biliary
microcephaly and petechiae. remnant proximal to the level of the porta hepa-
tis anastomosed to a Roux-en-Y loop of jejunum
REFERENCE to reinstate bile duct flow. The most common
Mitchell E at al. Clin Liver Dis 22(2018); complication after hepatoportoenterostomy is as-
625-641. cending cholangitis. This most commonly occurs
within two years of surgery and may affect more
than half of children who have undergone Kasai
Question 20 procedure. It is characterized by fever and eleva-
A two-year-old infant with a history of biliary tion of hepatic transaminases as well as alkaline
atresia status post Kasai procedure presents to phosphatase. Young children are prone to many
the emergency department with the abrupt onset infectious illnesses, however, in febrile children
of fussiness, irritability and tactile temperature. who have undergone a Kasai procedure, ascend-
Evaluation reveals temperature 102.7°F, tachycar- ing cholangitis should always be a critical concern.
dia and borderline hypotension. Laboratory stud- Most commonly this is caused by gram-negative
ies indicate white blood cell count 15.4 x 109/L , organisms such as Pseudomonas. It is appropri-
ALT 245 U/L, AST 238 U/L, alkaline phosphatase ate to provide broad-spectrum antibiotic coverage
490 U/L, total bilirubin 4.6 mg/dL, direct biliru- while blood cultures are pending.
bin 4 mg/dL, and indirect bilirubin 0.6 mg/dL. While portal hypertension and liver failure are
Liver ultrasound demonstrates evidence of prior possible sequelae of biliary atresia, the acute presen-
surgical hepatoportoenterostomy but no other tation with fever more likely supports a diagnosis of
acute changes. Blood cultures are pending. ascending cholangitis. The presence of choledocho-
Which of the following is the most likely lithiasis is uncommon in children including those
diagnosis? with biliary atresia making this unlikely. Pancreati-
tis is not necessarily associated with biliary atresia
A. Cholangitis and also less common in children.
B. Portal hypertension
C. Liver failure REFERENCE
D. Choledocholithiasis Murray K and Jonas M (1998). Chapter 48: Biliary
E. Pancreatitis Atresia. In Clinical Pediatric Gastroenterology (pp
331-347). Philadelphia, PA: Churchill Livingstone.
CORRECT ANSWER: A

RATIONALE Question 21
Extrahepatic biliary atresia is a disorder that A 17-year-old girl presented to her primary care
results in progressive destruction of the extrahe- physician with symptoms of lethargy, malaise and
patic biliary system with variable involvement of fatigue. She has no significant past medical histo-
476 Digestive Diseases Self-Education Program®

ry and takes no medications. On physical exami- between boys and girls, however, there is a higher
nation she was not jaundiced and her sclerae were incidence of females in adolescence. It may pres-
anicteric, however, hepatomegaly was present ent subacutely with vague symptoms of malaise,
with a liver edge palpable 3 cm below the costal lethargy and sometimes joint pain and rash.
margin. Screening laboratory tests performed by Children with subacute disease may not have
her primary care provider disclosed AST 285 U/L, clinical stigmata of liver disease with the exception
ALT 363 U/L, alkaline phosphatase 82 U/L, Total of hepatosplenomegaly. Approximately one half
bilirubin 1.0 mg/dL, direct bilirubin 0.4 mg/dL, of children with autoimmune hepatitis present
albumin 3.2 g/dL. Laboratory testing for hepatitis with fulminant acute disease which can progress
A, B, C, Epstein-Barr virus and Cytomegalovirus to liver failure. Autoimmune hepatitis is divided
are negative. Abdominal ultrasound shows mild into type I and type II depending upon circulating
hepatomegaly with normal liver echogenicity. She autoantibodies. Type I autoimmune hepatitis is
is referred to your office for further hepatology more common and associated with positive anti-
evaluation. Your laboratory testing shows: ceru- smooth muscle antibodies as well as antinuclear
loplasmin 38 mg/dL, total serum IgA 80 mg/dL, antibodies. Type II autoimmune hepatitis is less
tissue transglutaminase IgA 8, alpha 1 antitrypsin common and associated with the presence of anti-
level 118 mg/dl, ANA positive 1:640, anti-smooth LKM antibodies.
muscle antibody positive and total serum IgG Histologically, the classic findings of autoim-
2184 mg/dL. You determine the need to perform mune hepatitis include a portal inflammatory in-
a liver biopsy. Based upon the serologic findings filtrate consisting of lymphocytes and plasma cells
in this workup what will liver histopathology most extending beyond the limiting plate and around
likely demonstrate? adjacent periportal hepatocytes. This is referred
to as piecemeal necrosis or interface hepatitis. In
A. PAS-positive, diastase resistant granules in the severe cases necrosis may be present. PAS posi-
hepatocytes tive, diastase resistant granules in the hepatocytes
B. Concentric fibrosis around the interlobular would be indicative of alpha-1 antitrypsin deficien-
bile ducts cy which has been ruled out based upon serologic
C. Microvesicular and macrovesicular steatosis level and genotype testing. Concentric periportal
with hepatic copper content greater than fibrosis otherwise known as onion skinning would
250 µg dry weight be a classic finding in primary sclerosing cholan-
D. Portal inflammatory infiltrate consisting of gitis. While there can be overlap syndromes of
lymphocytes and plasma cells extending across autoimmune hepatitis and sclerosing cholangitis,
the limiting place into the periportal hepatocytes the adolescent in the vignette above does not have
E. Bile duct proliferation, portal tract fibrosis and features of cholestasis. A finding of bile duct pro-
bile duct plugs. liferation, portal tract fibrosis and bile duct plugs
would be suggestive of a cholestatic disorder and
CORRECT ANSWER: D is a classic finding of biliary atresia. The patient’s
laboratory testing is not indicative of cholestasis
RATIONALE and it would be unusual for such a diagnosis to be
The adolescent and the above scenario is most made in adolescence.
likely presenting with autoimmune hepatitis
based upon her serologic evaluation disclosing a REFERENCES
positive ANA, anti-smooth muscle antibody and Roberts E. Liver International 2011 31(10):
elevated total serum IgG. Autoimmune hepatitis 1424-1431.
can present in childhood at any age. In younger Le Bail B. Clinics and Research in Hepatology
children the incidence appears to be equivalent and Gastroenterology 2012; 36(3): 248-252.
Chapter 16 — Issues in pediatric gastroenterology 477

Question 22 physiologic standpoint, while there does appear


A 15-year-old adolescent is referred to the office to be association of PSC with HLA haplotypes as
for evaluation of abnormal liver enzymes which well as other genes associated with IBD, there are
were noted incidentally during a lab draw at his no current guidelines which recommend screen-
primary care office. He reported feeling well ing of family members of those afflicted with the
without any significant symptoms with the excep- condition. Patients may appear asymptomatic
tion of fatigue and mild intermittent diarrhea. until there is further progression of liver disease
Review of his hepatic function panel disclosed resulting in symptoms of pruritus due to cholesta-
albumin 3.5 mg/dL, AST 286 U/L, ALT 432 U/L, sis. There is a strong association between PSC
alkaline phosphatase 640 U/L, total bilirubin 0.7 and IBD. Among patients diagnosed with PSC
mg/dL, and direct bilirubin 0.3 mg/dL. CBC was approximately 70 percent will have concomitant
normal with the exception of mild anemia with a IBD. There is a greater propensity for ulcerative
hemoglobin of 13 g/dL. You perform additional colitis however up to 15 percent of cases may
laboratory testing which shows a normal PT, PTT have Crohn’s colitis. Conversely, in the setting
and INR. GGT is elevated at 269 U/L. Abdominal of known IBD, the prevalence of PSC is only two
ultrasound reveals geographic regions of intrahe- to eight percent. Due to the strong association
patic biliary ductal dilatation and abnormal liver between PSC and IBD it is recommended that
parenchymal echogenicity, with additional large individuals diagnosed with PSC be screened im-
areas of sparing. Similar findings are confirmed mediately with an ileocolonoscopy. There is an
on MRCP and he is referred for a liver biopsy. increased risk of colonic dysplasia and malignancy
This shows evidence of small duct PSC. in the setting of IBD and PSC. Such individuals
require annual colonoscopy for dysplasia screen-
What is the next appropriate step in management? ing. With regard to diagnostic studies, a choles-
tatic pattern of liver injury with elevated hepatic
A. Refer for ileocolonoscopy given the strong as transaminases alkaline phosphatase and GGT
sociation between PSC and IBD are most commonly found. Bilirubin levels may
B. Start ursodeoxycholic acid 30 mg/kg daily remain normal until the disease has progressed
C. Immediately referred for liver transplant more significantly. While a positive P-ANCA
evaluation as he will likely need a liver serology is associated with a diagnosis of PSC, in
transplant in the future the vignette above the diagnosis has been suf-
D. Send a p-ANCA serology for further confirma- ficiently confirmed and it is not a necessary part
tion that you have the correct diagnosis of the evaluation. Classic findings of intra-and/
E. Screen all family members for possible PSC or extrahepatic bile duct dilatation and strictur-
as there can be a genetic predisposition ing can be found on ultrasound and MRCP. Liver
biopsy may show characteristic findings of peri-
CORRECT ANSWER: A ductal concentric fibrosis of small interlobular bile
ducts also known as onion skinning. Liver biopsy
RATIONALE can also help delineate between PSC and autoim-
Primary sclerosing cholangitis is a chronic chole- mune hepatitis overlap syndromes. With regard
static liver disease characterized by inflammation to medical therapy, there are no definite effective
and fibrosis of the intra-and or extrahepatic bile medical treatments for PSC. Previous literature
ducts. It is a progressive condition for which an suggested treatment with ursodeoxycholic acid.
etiology is unknown but can lead to cirrhosis and More recent data, however, has suggested that
end-stage liver disease. Epidemiologically, there ursodeoxycholic acid may increase severe adverse
appears to be a predilection for male sex with a events when compared to placebo. It is not uni-
2:1 male to female predominance. From a patho- versally recommended.
478 Digestive Diseases Self-Education Program®

REFERENCE and C, cytomegalovirus, enterovirus, adenovirus,


Vo H et al. JPGN 2014; 59(3): 288-299. human herpes virus 6, herpes simplex virus 1 and
2, parvovirus, metabolic disorders, autoimmune
hepatitis, Wilson’s disease and drug toxicity such
Question 23 as acetaminophen ingestion. Children suspected
A five-year-old girl presents to the hospital with a of having PALF should be cared for in a moni-
one-week history of lethargy, weakness and pro- tored intensive care setting. Acute liver failure
gressive jaundice. History reveals she was a previ- is associated with a high inflammatory state and
ously healthy child without any known medical hyperdynamic circulatory failure. Close attention
issues or underlying genetic syndromes. Her fam- needs to be paid to respiratory and volume status.
ily denies recent travel, medications or witnessed Liver failure and hypoalbuminemia can be as-
ingestions. On evaluation she is noted to appear sociated with third spacing as well as hepatorenal
sleepy. Physical examination demonstrates mild syndrome. For this reason, it is critical to monitor
hepatomegaly but no splenomegaly. Her labora- patient’s creatinine and urine output.
tory studies are concerning for ALT 684 U/L, AST Hepatic encephalopathy, cerebral edema and
590 U/L, total bilirubin 2.3 mg/dL, direct biliru- seizures can occur in the setting of acute liver fail-
bin 1.8 mg/dL, INR 2.0, ammonia 90 mcg/dL. ure. It is important to monitor mental status as
Acute hepatitis and toxicology panel are negative. well as ammonia levels. Administration of lactu-
Liver ultrasound demonstrates mild hepatomegaly lose, rifaximin or neomycin is appropriate in the
but no intrinsic masses or other abnormalities. setting of evidence of hepatic encephalopathy.
There is concern for pediatric acute liver failure. Children suspected of having pediatric acute
What is an appropriate consideration in the man- liver failure should be referred to medical centers
agement of pediatric acute liver failure? with transplant capabilities. In the setting of co-
agulopathy, a transjugular liver biopsy is appro-
A. Close monitoring of mental status and priate given risks of percutaneous liver biopsy.
ammonia levels for hepatic encephalopathy. Randomized studies in adults and children have
B. Hospice referral not shown a benefit of the empiric administration
C. Administration of N-acetylcysteine in the of N-acetylcysteine in non-acetaminophen related
absence of witnessed acetaminophen ingestion to acute liver failure. Specifically, in a pediatric
D. Milk thistle as a primary treatment randomized controlled trial of 184 patients with
E. Start a probiotic non-acetaminophen related acute liver failure, the
one year transplant free survival was lower in those
CORRECT ANSWER: A who received N-acetylcysteine (35 percent) com-
pared to those who received placebo (53 percent),
RATIONALE particularly for children under two years of age. As
Pediatric acute liver failure (PALF) is defined such, the empiric administration of N-acetylcyste-
as: 1) biochemical evidence of liver injury and a ine in cases of liver failure unrelated to acetamino-
child without evidence of chronic liver disease, 2) phen ingestion is not recommended.
coagulopathy not corrected by vitamin K ad-
ministration and 3) INR greater than 1.5 if the REFERENCE
patient has encephalopathy or greater than 2.0 Lutfi R et al. JPGN 2017; 64(5): 660-670
if encephalopathy is absent. The epidemiologic Squires RH, Rand RB (2018). Acute liver failure
occurrence of PALF is unknown but accounts for in children: etiology and evaluation. In AG. Hop-
10-15 percent of pediatric liver transplants per- pin (Ed) UptoDate. Retrieved November 14, 2018
formed in the United States annually. The etiolo- from https://www.uptodate.com/contents/acute-
gies are variable including forms of hepatitis A, B, liver-failure-in-children-etiology-and-evaluation
Chapter 16 — Issues in pediatric gastroenterology 479

Question 24 Rectal prolapse in children should warrant


Pediatric cystic fibrosis may present with gastro- consideration of testing for cystic fibrosis. Rectal
intestinal manifestations. Which of the following prolapse is uncommon in the pediatric popula-
is common gastrointestinal manifestation of cystic tion. It is estimated that three percent of children
fibrosis? with rectal prolapse have cystic fibrosis. This is
propagated by constipation and similarly a missed
A. H. Pylori infection diagnosis of cystic fibrosis should be considered in
B. Meconium ileus young children struggling with constipation unre-
C. Crohn’s disease sponsive to laxative therapy and no other findings
D. Irritable bowel syndrome on anorectal manometry.
E. Neuroblastoma Pancreatic insufficiency is the most common
gastrointestinal complication of cystic fibrosis
CORRECT ANSWER: B affecting approximately 85 percent of patients at
some point in their course. This results in steat-
RATIONALE orrhea as well as fat soluble vitamin deficiency
Cystic fibrosis is a genetic condition related to ab- including vitamins A, E, D and K. Patient will
normal chloride and bicarbonate transport caused require long-term pancreatic enzyme replacement
by mutations in the cystic fibrosis transmembrane and careful monitoring of fat-soluble vitamins.
conductance regulator gene on chromosome 7. While studies have demonstrated that individ-
Mutations result in abnormal secretions in the uals with cystic fibrosis can have an increased risk
lungs, gastrointestinal tract, pancreas and hepato- of digestive tract cancers including colon polyps
biliary system. Individuals with cystic fibrosis are and colon cancer there has not been an associated
prone to a variety of gastrointestinal conditions risk with hepatoblastoma. There is an increased
including GERD, meconium ileus, rectal prolapse risk of hepatoblastoma with familial adenomatous
and pancreatic insufficiency. Studies have demon- polyposis and children with that condition should
strated that GERD is present in 30-40 percent of undergo routine ultrasound and alpha-fetoprotein
individuals with cystic fibrosis. This may be relat- testing every six months until the age of five years.
ed to increased intra-abdominal pressure second-
ary to coughing, decreased basal tone of the lower REFERENCE
esophageal sphincter muscle, gastric hyperacidity Sabharwal S and Mallory G (2018). Cystic fibrosis:
as well as delayed gastric emptying from a high fat Overview of gastrointestinal disease. In AG. Hop-
diet or neuropathy caused by cystic fibrosis related pin (Ed) UptoDate. Retrieved November 14, 2018
diabetes. GERD may have a significant impact on from https://www.uptodate.com/contents/cystic-
pulmonary disease. Patients with cystic fibrosis fibrosis-overview-of-gastrointestinal-disease
typically require proton pump inhibitor therapy
both for GERD and administration of pancreatic
enzyme replacement. Question 25
Meconium ileus is a condition of neonates in A 17-year-old girl with a past medical history notewor-
which inspissated meconium results and obstruc- thy for stricturing ileal Crohn’s disease requiring ileo-
tion of the terminal ileum. Infants with this con- cecal resection five years prior presents to the office
dition will fail to pass meconium in the first three for routine follow-up. She indicates that her Crohn’s
days of life. There may be associated vomiting or disease remains in remission on infliximab infu-
abdominal distention. This is typically identified sions 5 mg/kg every eight weeks but she has noted
via radiographic imaging. It can be managed via progressive fatigue over the last four to six months.
enema therapy or may require surgical manage- Her physical exam is unrevealing. Laboratory studies
ment if enemas fail. disclose evidence of megaloblastic anemia.
480 Digestive Diseases Self-Education Program®

Her iron levels are normal. Which nutrient defi- that the patient had passage of meconium within
ciency would you be most concerned about? the first 24 hours of life. Around 2.5 years of age,
he successfully toilet trained. However, after
A. Vitamin B12 insufficiency starting kindergarten around six months ago, the
B. Niacin deficiency patient developed decreased bowel movement
C. Ascorbic acid deficiency frequency and developed fecal incontinence. He
D. Riboflavin deficiency is currently having one hard, large volume bowel
E. Vitamin A deficiency movement every week with daily fecal inconti-
nence. He does report discomfort with bowel
CORRECT ANSWER: A movements. His examination demonstrated a
large rectal stool burden.
RATIONALE
Vitamin B12 insufficiency is a common complica- What would be the recommended next step in
tion of ileal resection after Crohn’s disease due management?
to loss of absorptive surface area in the ileum.
There appears to be a higher risk with ileal resec- A. Initiation of an osmotic laxative
tions greater than 20 cm. Individuals with mild B. Initiation of a stimulant laxative
or relatively recent vitamin B12 deficiency may C. Bowel cleanout
be asymptomatic or simply report mild fatigue. D. Increased dietary fiber
In more chronic cases, vitamin B12 deficiency E. Behavioral therapy
can affect demyelination of nerves and results
in paresthesias, depression and poor memory. CORRECT ANSWER: C
Niacin deficiency, also known as pellagra, is a rare
nutritional deficiency. Clinical symptoms consist RATIONALE
of dermatitis, diarrhea and dementia. Ascorbic Functional constipation is found in approximately
acid deficiency (Vitamin C) results in scurvy. Indi- 14 percent of children. The Rome IV criteria
viduals present with bony abnormalities, petechial for functional constipation in children is two or
hemorrhage of the skin, myalgias gingivitis and more of the following occurring at least one per
other bone abnormalities. Riboflavin deficiency week for a minimal of one month: 1. Two or fewer
presents with photophobia, glossitis, angular defecations in the toilet per week for a child at a
stomatitis and growth abnormalities. Vitamin developmental age of four-years-old, 2. At least
A deficiency would also be less likely given her one episode of fecal incontinence per week, 3.
overall well appearance and stably managed History of retentive posturing, 4. History of hard
Crohn’s disease. Clinical symptoms of vitamin A or painful bowel movements, 5. Large fecal mass
deficiency would include xeropthalmia as well as in rectum, 6. History of large diameter bowel
night blindness. movements. This patient meets criteria for func-
tional constipation. He does not have alarming
REFERENCE features to suggest another etiology leading to his
Tershakovec (1998). Chapter 13. Vitamin Defi- constipation symptoms. Functional constipation
ciency: In Clinical Pediatric Gastroenterology (pp commonly starts in children at the time of toilet
95-102). Philadelphia, PA: Churchill Livingstone. training or starting school with either pain with
defecation, fear of toileting or social pressures
leading to retention. Treatment for this typically
Question 26 consists of disimpaction with either oral agents or
A six-year-old male is seen in clinic due to enemas followed by establishing a maintenance
chronic fecal incontinence. His father reports regimen. Osmotic laxatives like polyethylene
Chapter 16 — Issues in pediatric gastroenterology 481

glycol 3350 are typically used prior to stimulant along the intestine during early gestation, result-
laxatives. ing in a distal portion of the intestine being agan-
glionic. Rectal and distal sigmoid involvement is
REFERENCES seen in around 85 percent of cases, with the other
Borowitz, et al. Treatment of childhood consti- 15 percent involving more proximal intestine. It
pation by primary care physicians: efficacy and can rarely involve the entire colon and small intes-
predictors of outcomes. Pediatrics. 2005; 115: tine. Ganglion cells inhibit local smooth muscles,
873-877. resulting in the characteristic inability for agangli-
Hyams, J, et al. Childhood functional gastroin- onic bowel to relax. This lack of inhibition gives
testinal disorders: Child/adolescent. Gastroenter- rise to the absence of rectoanal inhibitory reflex
ology. 2016; 150: 1456-1468. (RAIRs) during anorectal manometry. The lack
Mugie, S. et al. Epidemiology of constipation of inhibition also produces a transition zone on
in children and adults: a systematic review. Best contrast enema, with the distal aganglionic bowel
Pract Res Clin Gastroenterol. 2011; 25: 3-18. being narrow and the more proximal bowel con-
taining ganglia being dilated.
Lack of meconium passage in the first 48 hours
Question 27 of life raises concern for Hirschsprung’s disease.
A two-month-old male presents with abdominal Other causes for possible failure to pass meco-
distention and poor appetite. His family notes nium include cystic fibrosis, anorectal malforma-
that the patient has chronic difficulties with con- tion, small left colon syndrome, meconium plug
stipation, reporting that they have to use a glyc- syndrome and megacystis-microcolon-intestinal
erin suppository to help him have a bowel move- hypoperistalsis syndrome.
ment every two to three days. The family reports
that he even needed a suppository in the newborn REFERENCE
nursey at day of life three due to lack of passage of Kenny, S et al. Hirschsprung’s disease. Semin
meconium. Pediatr Surg. 2010; 19 (3): 194-200.
Wyllie R. et al. Pediatric Gastrointestinal and
What finding would you expect to during evalua- Liver Disease. 4th edition. Elsevier Saunders,
tion to explain the patient’s symptoms? Philadelphia, 2011.

A. Presence of rectoanal inhibitory reflex (RAIRs)


during anorectal manometry Question 28
B. Absence of ganglion cells on rectal biopsy A three-year-old male with a history of peanut
C. Absence of transition zone on unprepped allergy presents to clinic due to chronic feeding
contrast enema intolerance. His mother reports that he eats more
D. Normal sweat chloride study slowly than other toddler aged children in his
E. A&B daycare. She reports that he will also refuse to eat
meat and tends to drink a large amount of juice
CORRECT ANSWER: B with meals. On several occasions, the family notes
that the patient will appear uncomfortable while
RATIONALE eating and then subsequently have emesis. What
Hirschsprung’s disease occurs in approximately is the most appropriate next step in management?
one out of 5000 live births and is caused by ab-
sence of ganglion cells in the myenteric plexus of A. Feeding therapy referral
the intestine. The condition arises from failure B. pH/Impedance probe
of the neural crest cells to fully migrate caudally C. Initiation of PPI alone
482 Digestive Diseases Self-Education Program®

D. Esophageal manometry the family has recently tried adding rice cereal
E. Upper intestinal endoscopy to the milk. What is the most likely cause for the
patient’s presentation?
CORRECT ANSWER: E
A. Cow’s milk protein allergy
RATIONALE B. Rotavirus infection
Eosinophilic esophagitis (EoE) is an allergy C. Food protein induced enterocolitis syndrome
mediated disease with eosinophilic infiltration of (FPIES)
the esophageal mucosa with resulting symptoms D. Metabolic disease
of esophageal dysfunction. EoE has a prevalence E. Intestinal malrotation
of 29.5 cases per 100,000 in children and 43.4
cases per 1000,000 in adults, with the disease CORRECT ANSWER: C
being common in patients with another under-
lying atopic condition (asthma, food allergies, RATIONALE
eczema, allergic rhinitis). Younger children Food protein inducted enterocolitis syndrome
commonly present with vomiting, oral aversion (FPIES) is a non-IgE cell mediated food allergy
or abdominal discomfort. Older children com- that presents at a young age commonly with
monly present with dysphagia, food impaction vomiting, diarrhea and dehydration after exposure
and abdominal discomfort. EoE is diagnosed by to the food allergen. In severe cases, exposure to
upper intestinal endoscopy, with greater than 15 the allergen can lead to shock. The prevalence of
eosinophils per 40x field found in the esophagus FPIES is relatively low, with one series demon-
while on proton pump inhibitors (PPIs) being di- strating three cases per 1000 births. The most
agnostic for the condition. This patient is noted common allergens for FPIES include cow’s milk,
to have a history of food allergy with oral aver- soy, rice, oats and fish. Symptoms for cow’s milk
sion and possible episodes of food impactions, or soy based FPIES typically present less than
raising concern for EoE. six months of age with infant formulas, with
solid food based symptoms starting five to seven
REFERENCE months of age. The diagnosis of the condition
Dellon, E. and Hirano, I. Epidemiology and natu- is typically made by withdrawing the suspected
ral history of eosinophilic esophagitis. Gastroen- allergen and seeing if symptoms resolve. Allergy
terology. 2018; 154(2): 319-332. testing is of limited diagnostic value. Treatment
consists of managing hemodynamic instability
following exposure, with maintenance focusing
Question 29 on strict avoidance of triggers. FPIES is typically
A five-month-old unvaccinated female presents to a self-limited process, with most children hav-
the emergency department due to vomiting and ing resolution of reactions between one to five
diarrhea. On evaluation, the patient has promi- years after symptom onset. For this patient, she
nent dehydration with hypotension requiring IV has characteristic episodes of vomiting, diarrhea
rehydration. The family reports that this is their and hypotension following feedings of expressed
second visit to the emergency room in the last breastmilk mixed with rice. With this, rice should
two weeks for similar symptoms. With the last be eliminated from the patient’s diet.
episode, the patient had resolution of symptoms
within 24 hours of presentation. They deny any REFERENCE
significant vomiting or diarrhea at baseline. The Nowak-Wegrzyn, A. et al. International consensus
patient normally fed with expressed breastmilk. guidelines for the diagnosis and management of
To try to help the infant sleep through the night, food protein-induced enterocolitis syndrome: Ex-
Chapter 16 — Issues in pediatric gastroenterology 483

ecutive summary-workgroup report of the adverse cross over the vertebral column. Treatment typi-
reactions to foods committee, American Academy cally consists of a Ladd’s procedure to reduce any
of Allergy, Asthma & Immunology. J Allergy Clin volvulus, divide Ladd’s bands, broaden the base
Immunol. 2017; 139(4): 1111-1126. of the mesenteric vascular pedicle and perform an
appendectomy.

Question 30 REFERENCES
You are asked to evaluate a two week old female Adams, S. and Stanton, M. Malrotation and
in the clinic due to recurrent vomiting. The family intestinal atresias. Early Hum Dev. 2014; 90(12):
notes that the patient has intermittent episodes of 921-925.
vomiting throughout the day. They describe the Wyllie R. et al. Pediatric Gastrointestinal and
emesis as having a “green” color with the occa- Liver Disease. 4th edition. Elsevier Saunders,
sional appearance of formula. During some of the Philadelphia, 2011.
episode, the infant is noted to appear uncomfort-
able with abdominal distention. In clinic today,
the patient is feeding without distress. What is Question 31
the most likely diagnosis for this patient? A seven-year-old female presents to clinic due to
recurrent vomiting. She is noted to have a one-
A. Pyloric stenosis year history of recurrent vomiting. On further
B. Gastroesophageal reflux questioning, she reports that the vomiting typi-
C. Cow’s milk protein allergy cally occurs every three to four weeks and lasts for
D. Intestinal malrotation around one day. The family describes her waking
E. Food protein induced enterocolitis syndrome up in the morning vomiting, with symptoms spon-
(FPIES) taneously resolving later in the day. During the
episodes, she will have greater than 10 episodes of
CORRECT ANSWER: D non-bloody and non-bilious emesis. What treat-
ment options might be offered to this patient?
RATIONALE
This patient presents with intermittent bilious A. Amitriptyline
emesis with associated abdominal pain and dis- B. Cyproheptadine
tention, raising concern for an obstructive process. C. Propranolol
Since the symptoms are intermittent, with the D. L-carnitine and Coenzyme Q10
patient able to easily feed in between episodes, E. All of the above
pyloric stenosis is unlikely. During the fifth week
of gestation, the midgut herniates into the yolk CORRECT ANSWER: E
sac and undergoes sequential growth, elongation
and rotation. With normal rotation, the small RATIONALE
bowel mesentery is fixed to the posterior body Cyclic vomiting syndrome occurs is around 0.2-1.0
wall resulting in minimal opportunity for torsion. percent of the population. The condition typically
With intestinal malrotation, the lack of intestinal starts between 3.5 to seven years of age but can
fixation increases the risk for volvulus. In patients occur in adulthood. The ROME IV criteria for the
with malrotation, volvulus is more common in condition is 1. Occurrence of two or more periods
the neonatal age range, with untreated volvulus of intense, unremitting nausea and paroxysmal
potentially leading to ischemia and bowel necro- vomiting lasting hours to days within a six month
sis. The diagnosis is typically made by an upper period, 2. Episodes are stereotypical, 3. Episodes
GI series demonstrating the duodenum failing to are separated by weeks to months with return to
484 Digestive Diseases Self-Education Program®

baseline health between episodes, 4. The symp- C. Rectal biopsy


toms cannot be attributed to another cause. This D. Screening for Celiac disease and
patient meets all of the diagnostic criteria for hypothyroidism
cyclic vomiting syndrome. Treatment consists of E. Lumbar spine MRI
symptomatic management during episodes with
baseline prophylaxis to try to avoid episodes. For CORRECT ANSWER: E
children less than five years old, cyproheptadine is
the recommended first-line therapy. In children RATIONALE
older than five years old, amitriptyline is the rec- Tethered cord syndrome is a condition where
ommended first-line therapy. Propranolol is the there is abnormal fixation of the spinal cord lead-
typical second-line therapy with L-carnitine and ing to a stretch induced injury of the conus and
Coenzyme Q10 used as an adjunct agents. caudal spinal cord. The incidence of the syndrome
is 0.05 to 0.25 per 1000 births, with there being
REFERENCES a female predominance. Tethered cord is also
Fitzpatrick, E. et al. The incidence of cyclic vomit- seen more commonly in patients with a history
ing syndrome in children: population-based study. of anorectal malformation. Infants may present
Am J Gastroenterol. 2008; 103: 991-995. with dimpling or a hair tuft on the lower back.
Hyams, J, et al. Childhood functional gastroin- In young children, typical symptoms consist of
testinal disorders: Child/adolescent. Gastroenter- progressive motor and sensory abnormalities,
ology. 2016; 150: 1456-1468. with patients having difficulties with walking after
Li, B. et al. North American Society for Pediat- previous normal motor function as well as urinary
ric Gastroenterology, Hepatology, and Nutrition incontinence and more rarely bowel dysfunction.
consensus statement on the diagnosis and man- Older patients may also report back pain, scoliosis
agement of cyclic vomiting syndrome. J Pediatr and leg pain. In neonates, a lumbar spine ultra-
Gastroenterol Nutr. 2008; 47: 379-393. sound can be used to screen for tethered cord. In
older children and adults, a lumbar spine MRI is
needed to detect the condition. Treatment is in-
Question 32 stitutionally dependent with either monitoring of
A 5-year-female is referred due to fecal inconti- relatively asymptomatic patients or tethered cord
nence. The parents report that the patient toilet release. For this patient, she has urinary inconti-
trained at 2.5 years old but has had progressive nence, bowel dysfunction and a change in gait to
difficulties with fecal and urinary incontinence. suggest possible tethered cord. Her physical ex-
Her parents report that she is also acting more amination findings of a sacral dimple also support
“clumsy” that normal, with her now frequently the concern for tethered cord.
falling with walking. She has tried no medications
or dietary management for her symptoms. On REFERENCES
examination, the patient has a dimple over the Drake, J. Occult tethered cord syndrome: not an
sacral region. On rectal examination, there was indication for surgery. J Neurosurg. 2006; 104(5
not significant stool in the rectal vault and an anal suppl): 305-308.
wink was difficult to obtain. Granada, C. et al. Tethered cord syndrome in
the pediatric-adolescent gynecologic patient. J
What is the most appropriate next step in Pediatr Adolesc Gynecol. 2015; 28(5): 309-312.
management?

A. Initiation of an osmotic laxative Question 33


B. Anorectal manometry A two-year-old female presents to urgent care
Chapter 16 — Issues in pediatric gastroenterology 485

following swallowing an object around two hours pediatric foreign body ingestions. Gastrointest
ago. The patient is asymptomatic with no drool- Endosc Clin N AM. 2016; 26(1): 99-118.
ing or apparent discomfort. On X-ray, a 20mm
radiopaque foreign body with a double halo sign is
see in the region of the proximal esophagus. What Question 34
management is needed at this time? A 22-month-old male presents to urgent care for
evaluation after being found in his parents’ garage
A. Observation in urgent care with repeat drinking from an unlabeled bottle. The patient is
X-ray in two hours crying throughout the examination and has a rash
B. Emergent endoscopic removal of the around his mouth. He does not appear to be able
foreign body to handle his secretions, with the patient drool-
C. Inpatient admission for observation ing throughout the examination. What long-term
D. Surgery consultation complication is the patient at highest risk for
E. Upper GI series with water soluble contrast developing?

CORRECT ANSWER: B A. Esophageal carcinoma


B. Esophageal perforation
RATIONALE C. Esophageal stricture
The most critical indication for endoscopic inter- D. Behavioral oral aversion
vention in children is a retained esophageal button E. Esophageal dysmotility
battery. Button batteries are used in many com-
mon household goods, making them available for CORRECT ANSWER: C
ingestion. On PA imaging, button batteries have
a classic “double halo” sign. The mechanism of RATIONALE
esophageal injury in patients is from the genera- Caustic ingestions primarily occurs in pediatric
tion of hydroxide radicals in the mucosa leading in children less than five years of age. Common
to a pH rise and coagulative necrosis. Necrosis of agents include household cleaners (detergents
the lamina propria can occur within 15 minutes of and bleach), hair relaxers and pesticides. Caustic
contact with a button battery, with outer muscular agents can be divided into acidic or alkaline agents.
layer injury occurring within 30 minutes. Compli- Acid agents typically have a stronger taste, limiting
cations from esophageal button batteries include their intake. They cause a coagulation necrosis,
aortoenteric fistula, tracheoesophageal fistula, and which limits injury to superficial mucosa in most
esophageal strictures. Aortoenteric fistula and cases. Alkaline agents cause liquefaction necrosis,
resulting death by exsanguination has occurs up which can result in submucosal injury and perfora-
to two weeks after foreign body removal. With tion. Management consists of documenting the
the high mortality and morbidity associated with ingested substance and monitoring for symptoms
esophageal button batteries, their removal is con- of oral or esophageal injury. If the patient is symp-
sidered emergent. tomatic or ingested a severely caustic agent, upper
intestinal endoscopy should be performed within
REFERENCES the first 24 hours to assess the degree of mucosal
Kramer, R. et al. Management of ingested foreign injury. If the patient has evidence of necrosis or
bodies in children: a special clinical report of the deep circumferential injury, nasogastric tube place-
NASPGHAN Endoscopy Committee. J Pediatr ment is indicated. This should be followed up with
Gastroenterol Nutr. 2015; 60(4): 562-574. an upper GI series in two to three weeks to assess
Leinwand, K. et al. Button battery ingestion in for an esophageal stricture, the most common long-
children: a paradigm for management of severe term complication of caustic ingestion.
486 Digestive Diseases Self-Education Program®

REFERENCES The high risk for injury with ingestion led to the
Hyams, J, et al. Childhood functional gastrointes- mandatory recall of the common Buckyballs (rare
tinal disorders: Child/adolescent. Gastroenterol- earth magnets) in 2014. If two or more magnets
ogy. 2016; 150: 1456-1468. are in the esophagus or stomach, urgent endo-
Wilsey, M. et al. The role of upper gastroin- scopic removal is indicated. If the foreign body is
testinal endoscopy in the diagnosis and treatment distal to the stomach, serial monitoring to ensure
of caustic ingestion, esophageal strictures and progression of the magnets is recommended in an
achalasia in children. Gastrointest Endosc Clin N asymptomatic patient. In a symptomatic individ-
Am. 2001; 11(4): 767-787. ual, like the patient in this clinical vignette, urgent
surgical evaluation is needed.

Question 35 REFERENCE
Gastroenterology is consulted about a three-year- Kramer, R. et al. Management of ingested foreign
old male who presented to the emergency depart- bodies in children: a special clinical report of the
ment due to abdominal pain and distention. On NASPGHAN Endoscopy Committee. J Pediatr
examination, the patient is febrile with notable Gastroenterol Nutr. 2015; 60(4): 562-574.
discomfort to palpation over the abdomen. On
abdominal X-ray, four circular, radiopaque objects
that are in a linear arrangement are seen in the re- Question 36
gion of the stomach without evidence of intestinal A three-month-old male presents in clinic with
obstruction or free air. recurrent spitting. The mother notes that the in-
fant is currently on a standard cow’s milk formula.
What is the next step in management? With approximately half of his feedings, he is hav-
ing effortless regurgitation of a small amount of
A. Inpatient observation formula without a bilious appearance. She reports
B. Endoscopic retrieval of foreign body intermittent crying when she places him in his
C. Urgent surgical evaluation crib after feedings. His pediatrician has had the
D. Outpatient evaluation with serial abdominal family to thicken the formula with rice cereal and
X-rays try positional changes without improvement. The
E. Proceed with abdominal CT patient is growing well with no abnormal find-
ings on examination. His stool sample is guaiac
CORRECT ANSWER: C negative. What is the most likely etiology for the
infant’s symptoms?
RATIONALE
When ingested, magnets can attract between dif- A. Eosinophilic esophagitis
ferent layers of bowel. This attraction can lead to B. Pyloric stenosis
localized ischemia with risk for necrosis, perfora- C. Intestinal malrotation
tion and enteroenteric fistula formation. Many D. Physiologic gastroesophageal reflux
toys contain rare earth magnets like neodymium E. Delayed gastric emptying
that have increased attractive force compared to
conventional magnets. These magnets appear in CORRECT ANSWER: D
X-rays as circular radiopaque objects typically in
a linear distribution in small numbers and circu- RATIONALE
lar in larger numbers. The risk for bowel injury Regurgitation is common in infancy with 51 per-
from magnets is significantly higher with rare cent of infants having daily emesis at 0 to three
earth magnets compared to conventional ones. months of age and 67 percent having daily em-
Chapter 16 — Issues in pediatric gastroenterology 487

esis at four months of age. For infants 0 to three a two year history of a burning sensation in the
months of age, 14 percent are perceived as having epigastric region that occurs several times a week.
a “problem” with gastroesophageal reflux due to The patient also reports having abdominal full-
discomfort, frequent emesis and back arching. ness and early satiety associated with the episodes
For individuals in the age range of the clini- of discomfort. Due to the discomfort, the patient
cal vignette, 41 percent of infants have spitting will rarely avoid eating full meals and reports in-
with most feedings. Infants have a higher rate of creased belching. She denies any difficulties with
reflux symptoms due to a weaker lower esopha- bowel movements, reporting one Bristol 4 stool
geal sphincter as baseline, shorter intra-thoracic daily. The patient also denies vomiting or chest
esophagus, higher rate of supine positioning, and discomfort. What is the most appropriate diagno-
large fluid volume when compared to adults. The sis for this patient?
most common causes of gastroesophageal reflux
symptoms in children are physiologic gastro- A. Irritable bowel syndrome
esophageal reflux and cow’s milk protein allergy. B. Abdominal migraines
This patient does not have red flags to suggest a C. Functional dyspepsia
different etiology for his symptoms. Management D. Gastroesophageal reflux
of reflux in this age group starts with parental re- E. Peptic ulcer disease
assurance, upright positioning after feedings and
thickening feedings with oatmeal or rice cereal. CORRECT ANSWER: C
For refractory symptoms, a trial of a partially hy-
drolyzed or amino acid based formula for possible RATIONALE
cow’s milk protein allergy is recommended prior Chronic abdominal pain is common in the pedi-
to initiation of a proton pump inhibitor or H2 atric age range, with symptoms occurring in up
blocker. to 19 percent of children greater than four years
of age. Functional abdominal pain disorders are
REFERENCES the primary etiologies for the discomfort in the
Martin, A. et al. Natural history and familial majority of children. Based on ROME IV guide-
relationships of infant spilling to 9 years of age. lines, functional abdominal pain disorders consist
Pediatrics. 2002; 109(6): 1061-1067. of functional dyspepsia, irritable bowel syndrome,
Nelson, S. et al. Prevalence of symptoms of gas- abdominal migraines and functional abdominal
troesophageal reflux during infancy. A pediatric pain not otherwise specified (NOS). This patient
practice-based survey. Pediatric Practice Research reported epigastric burning discomfort, raising
Group. Arch Pediatr Adolesc Med. 1997; 151(6): concern for functional dyspepsia. The criteria
569-572. for functional dyspepsia is one or more symp-
Rosen, R. et al. Pediatric gastroesophageal toms for at least four days per month for greater
reflux clinical practice guidelines: joint recom- than two months of: 1. Postprandial fullness, 2.
mendations of the North American Society for Early satiety, 3. Epigastric pain or burning not
Pediatric Gastroenterology, Hepatology, and associated with defecation, 4. Symptoms cannot
Nutrition and the European Society for Pediatric be explained by another condition. This patient
Gastroenterology, Hepatology and Nutrition. J meets the above criteria. In addition, she also
Pediatr Gastroenterol Nutr. 2018; 66(3): 516-554. reports difficulties with finishing meals and post-
prandial belching suggesting postprandial distress
syndrome subtype of functional dyspepsia. She
Question 37 denies stooling difficulties, making irritable bowel
A 14-year-old female seeks evaluation due to syndrome unlikely. She does not have episodic
chronic abdominal discomfort. She reports having symptoms to suggest abdominal migraines.
488 Digestive Diseases Self-Education Program®

REFERENCE REFERENCES
Hyams, J, et al. Childhood functional gastrointes- Krishnasamy, S. and Abell, T. Diabetic gastropare-
tinal disorders: Child/adolescent. Gastroenterol- sis: principles and current trends in management.
ogy. 2016; 150: 1456-1468. Diabetes Ther. 2018; 9(suppl 1): S1-S42.
Saliakellis, E. and Fotoulakim M. Gastroparesis in
children. Ann Gastroenterol. 2013; 26(3): 204-211.
Question 38 Waseem, S. et al. Spectrun of gastroparesis
A 17-year-old female with type 1 diabetes mellitus in children. J Pediatr Gastroenterol Nutr. 2012;
presents to clinic due to abdominal discomfort. 55(2): 166-172.
She describes epigastric fullness that worsens with
eating. Associated with eating, the patient reports
nausea with intermittent non-bilious emesis. She Question 39
describes the emesis as having the appearance An 18-month-old male infant presents to an ur-
of previous eaten food. She does report a five gent care with vomiting and intermittent episodes
pounds weight loss. Her recent endocrinology of abdominal pain. The abdominal pain episodes
appointment indicated that she had a hemoglobin are intermittent, with the toddler appearing to
A1c of 12.4 percent. have severe pain and drawing his legs up towards
his abdomen followed by relative calm. During
What is the most appropriate long-term the episodes, he is having prominent bilious em-
treatment option? esis. On examination, the patient has a palpable
mass in the right upper quadrant but is otherwise
A. Erythromycin stable. A rectal examination was guaiac positive
B. Improved glycemic control without visible blood. Abdominal ultrasound
C. Ondansetron describes a “target-sign” mass in the right side of
D. Metoclopramide the abdomen. The family reports that this is the
E. Cognitive behavioral therapy longest ongoing episode of abdominal pain that
the patient has experienced. What is the most ap-
CORRECT ANSWER: B propriate next step in management?

RATIONALE A. Abdominal CT
The prevalence of gastroparesis is unclear in the B. Diagnostic laparoscopy
pediatric age range. For adults, the prevalence for C. Observation
definite gastroparesis is 37.8 for women and 9.6 D. Air contrast enema
for men per 100,000 population. Gastroparesis in E. Colonoscopy
adults is most commonly caused by diabetes mel-
litus (around 1/3), post-surgical complications, CORRECT ANSWER: D
neurologic conditions or idiopathic. For pediat-
rics, 70 percent of cases are idiopathic in natures RATIONALE
with just 4 percent being associated with a compli- Intussusception is the most common cause of me-
cation from diabetes mellitus. Diabetic gastropa- chanical intestinal obstruction in children ages 6 to
resis is thought to be caused by visceral autonomic 36 months, seen in up to 50 percent of cases. This
neuropathy. Initial treatment involves improving condition is caused by invagination of one portion
glycemic control with prokinetics and antiemetic of the gastrointestinal system into the lumen of an
for ongoing symptoms. This patient has classic adjacent segment. With this, the proximal intes-
symptoms of gastroparesis with reported post- tine is the intussusceptum, with the distal recipient
prandial fullness, abdominal pain and vomiting. intestine is the intussuscipiens. The invagination
Chapter 16 — Issues in pediatric gastroenterology 489

typically occurs in an antegrade order, with ileoco- been having multiple episodes of emesis starting
lic involvement seen in 80 percent of cases. This today. On examination, the patient has promi-
invagination is caused by the unbalanced forces nent discomfort with palpation or percussion over
created when a normal peristaltic wave encounters the periumbilical region. Screening labs demon-
a focal deformity in the bowel wall (lead point) strate a white blood cell count of 14,000 cells/mL
or a region with different mechanical properties. with an elevated C-reactive protein. A plain film
Focal lead points are found in only 10 percent of abdominal X-ray did not demonstrate evidence for
children, with the most common lead point being obstruction. While waiting for results, the patient
a Meckel’s diverticulum. While adults are found now reports more intense right lower quadrant
to have lead points in over 90 percent of cases. discomfort. What is the most likely etiology for
Ultrasound is the gold standard modality in detect- the patient’s symptoms?
ing intussusception. Non-operative management
with hydrostatic or pneumatic enemas under imag- A. Functional abdominal pain
ing guidance can resolve the condition in 80 to 95 B. Crohn’s disease
percent of physiologically stable cases. Surgical re- C. Intussusception
duction is recommended in patients with evidence D. Acute appendicitis
of perforation, pneumoperitoneum, physiologic E. Volvulus
instability or failure to fully reduce after three to
four non-operative attempts. Recurrence of intus- CORRECT ANSWER: D
susception is seen in five to 15 percent of pediatric
patients after non-surgical reduction. RATIONALE
This patient has a high clinical probability for Acute appendicitis is the most common surgi-
intussusception based on his age and intermittent cal condition found in the population, with the
symptoms of likely intestinal obstruction. His lifetime risk for diagnosis being 9 percent. Chil-
abdominal ultrasound highlighted the character- dren ages 10-14 have the highest incidence at
istic “target-sign” lesion for the condition. Since 169.6 cases/100,000 for diagnosis of appendicitis.
the patient is physiologically stable, non-operative Typical symptoms for the diagnosis include peri-
reduction with an air contrast enema would be the umbilical pain followed by right lower quadrant
most appropriate initial treatment. pain, nausea and vomiting. Other factors sug-
gesting the condition include a positive Rovsing
REFERENCES sign, psoas sign, or obturator sign on examination
Applegate K.E. Intussusception in children: as well as laboratory findings of white blood cell
evidence-based diagnosis and treatment. Pediatr count greater than 10,000 cells/mL, sterile pyuria
Radiol. 2009; 39(Suppl 2):S140-S143. and an elevated c-reactive protein. The Pediatric
Wyllie R. et al. Pediatric Gastrointestinal and Appendicitis Score (PAS) seen below is one of the
Liver Disease. 4th edition. Elsevier Saunders,
Philadelphia, 2011. Diagnostic Finding Value
Cough or Percussion tenderness 2
Fever 1
Question 40
RLQ tenderness 2
A 9-year-old male with a history of chronic ab-
Anorexia 1
dominal pain presents to the emergency depart-
Nausea and vomiting 1
ment due to acute worsening of periumbilical
Migration of pain 1
abdominal discomfort. His father reports new
onset of fever to 102F this afternoon. He also Leukocytosis (WBC >10,000) 1

notes that the patient refused to eat lunch and has Polymorphonuclear neutrophilia 1
490 Digestive Diseases Self-Education Program®

scoring systems used in pediatrics to assess risk of months of 1. Repeated regurgitation and rechew-
appendicitis. With this system, a score of five or ing or explusion of food that begins soon after
less is not suggestive of appendicitis. A score six or eating and stops with sleeping, 2. Not proceeded
more suggests appendicitis, with a score seven to 10 by retching, 3. No other clear etiology for symp-
indicating a high probability of the condition toms. This patient is at higher risk for rumination
(P 0.001). In this case, the patient has a PAS of syndrome with her developmental differences.
nine, making appendicitis the most likely diagnosis. Her painless regurgitation after eating meets cri-
teria for the condition. Prolonged high resolution
REFERENCES esophageal manometry can identify specific sub-
Anderson, J. et al. Examining a common disease groups of rumination. Antroduodenal manom-
with unknown etiology: trends in epidemiol- etry can detect simultaneous contractions called
ogy and surgical management of appendicitis in R-waves that can be seen in some patients with
California, 1995-2009. Worl J Surg. 2012; 36 (12): rumination syndrome. Since regurgitation stops
2787-2794. with sleeping, pH/Impedance probes demonstrate
Samuel, M. Pediatric appendicitis score. J Pe- resolution of symptoms with sleep. The condition
diatr Surg. 2002; 37(6): 877-881. is primarily diagnosed clinically, with other stud-
ies performed as clinically indicated. Treatment
typically consists of behavioral management.
Question 41
A 14-year-old female with a history of cerebral REFERENCE
palsy presents for evaluation due to recurrent Hyams, J, et al. Childhood functional gastrointes-
regurgitation. By report, she is regurgitating tinal disorders: Child/adolescent. Gastroenterol-
food into her mouth several times daily following ogy. 2016; 150: 1456-1468.
meals. Her parents report that the regurgitation
does not appear to be painful. What diagnostic
finding could be seen in this patient? Question 42
A 8-year-old previously healthy female presents
A. Normal pH/Impedance probe findings following an isolated episode of rectal prolapse.
during sleeping The family notes that the prolapse spontane-
B. Esophageal dysmotility on upper GI series ously reduced within one hour of onset with the
C. Decrease in gastric emptying time patient not having any perceived discomfort. On
D. Esophageal eosinophilia on upper intestinal examination, the patient has a dilated rectum with
endoscopy increased stool burden but no other clear abnor-
E. Absence of “R-waves” on antroduodenal malities. Which condition is most likely contrib-
manometry uting to the patient’s symptoms?

CORRECT ANSWER: A A. Perianal trauma


B. Chronic constipation
RATIONALE C. Cystic fibrosis
Rumination syndrome is a functional gastrointes- D. Infectious diarrhea
tinal disorder that can present in all age groups. E. Rectal polyp
The true prevalence of the disorder is unknown,
but the condition can be seen more commonly in CORRECT ANSWER: B
patients with developmental disorders and other
high risk groups like teenage females. The ROME RATIONALE
IV criteria for the condition include at least two Rectal prolapse is most frequently seen in children
Chapter 16 — Issues in pediatric gastroenterology 491

less than four years of age. It occurs with one or A. JAG1


more layers of rectal tissue pass through the anus. B. ATP8B1
The first type of prolapse is a mucosal prolapse C. RET
(false procidentia), which occurs when only the D. NOTCH
rectal mucosa protrudes through the anus produc- E. ATP7B
ing characteristic radial folds with the junction
at the anal skin. The second type is a complete CORRECT ANSWER: E
prolapse (true procidentia) that involves full thick-
ness protruding of the rectal wall through the anus. RATIONALE
Complete prolapses are further subdivided into The patient in this vignette is noted to have
first degree with protrusion greater than or equal chronically elevated aminotransferase levels. On
to 5cm and involving the mucocutaneous junc- evaluation, she is found to have a low ceruloplas-
tion, second degree with protrusion 2-5cm and not min and elevated urine copper, raising concern
involving the mucocutaneous junction, and third for Wilson’s disease. Wilson’s disease is an auto-
degree with fullness thickness involvement but the somal recessive disorder of copper metabolism
prolapse being internal and not passing through the involving gene ATP7B on chromosome 13 with
anal verge. In children, the most common causes of an estimated prevalence rate in children of one
rectal prolapse include constipation (28 percent), in 37,000. With this condition, there is exces-
diarrheal disease (20 percent), cystic fibrosis (11 sive accumulation of copper in numerous tissues
percent) and polyps. Less common contributing including the liver, brain, and cornea. This occurs
conditions include infectious diarrhea, ulcerative by failure to export copper into bile leading to
colitis, Hirschsprung’s disease, malnutrition, and failure to incorporate copper into ceruloplasmin.
anorectal malformations. Based on the physical ex- The excess copper typically leads to liver mani-
amination, the most likely etiology for the patient’s festations (chronic hepatitis, cirrhosis, acute liver
isolated prolapse would be constipation. Initial failure) in children. Overtime, neurologic and
management would consist of osmotic laxatives to psychiatric symptoms with characteristic Kayser-
address underlying constipation. For refractory Fleischer rings can develop. Diagnosis of Wil-
symptoms, evaluating for cystic fibrosis and intesti- son’s disease is suspected in the setting of a serum
nal polyps would be recommended. ceruloplasmin less than 20mg/dL and 24 hour
urine copper over 100µg. The diagnosis is further
REFERENCES supported by a liver biopsy showing a copper con-
Rentea, R. and St Peter, S. Pediatric rectal prolapse. tent over 250µg/g dry weight of tissue or the pres-
Clin Colon Rectal Surg. 2018; 31(2): 108-116. ence of Kayser-Fleischer rings. Since this patient
Siafakas, C. et al. Rectal prolapse in pediatrics. likely has Wilson’s disease, a defect in ATP7B is
Clin Pediatr (Phila). 1999; 38(2): 63-72. suspected. Defects in JAG1 and NOTCH are seen
with Alagille syndrome. Patients with progres-
sive familial intrahepatic cholestasis (PFIC) have
Question 43 defects in ATP8B1. Finally, RET defects are seen
A 15-year-old female was referred due to elevated in Hirschsprung’s disease.
aminotransferases over the last first year. The pa-
tient is asymptomatic but is noted to have an AST of REFERENCE
110 U/L and ALT of 47 U/L. Initial screening labs Seo, J. Diagnosis of Wilson disease in young chil-
demonstrated a low ceruloplasmin of 12 mg/dL. A dren: molecular genetic testing and a paradigm
confirmatory 24-hour urine copper determination shift from the laboratory diagnosis. Pediatr Gas-
came back at 190 µg/24 hours. What gene defect is troenterol Hepatol Nutr. 2012; 15(4): 197-209.
likely responsible for the patient’s presentation?
492 Digestive Diseases Self-Education Program®

Question 44 liver dysfunction. Decreased GALT activity in red


You are called to the neonatal intensive care unit blood cells confirms the diagnosis of galactosemia.
to assist with a 15-day old male who presented Treatment for the condition consists of elimina-
with profound hypoglycemia, hypotonia and sei- tion of all dietary galactose. For infants, this in-
zure activity. His family reported significant vom- volves eliminating all lactose (breastmilk or cow’s
iting following breastfeeding. Screening labs show milk formula) and switching to a soy or partially
unconjugated hyperbilirubinemia, positive urinary hydrolyzed formula. For other individuals, treat-
reducing substances and a blood culture positive ment consists of strict avoids of dairy containing
for E. coli. Physical examination demonstrates a products as well as other galactose rich foods like
liver edge 3cm below the right costal margin and garbanzo and black beans.
absent red reflexes. Confirmatory testing showed
decreased galactose-1-phosphate uridyltransfer- REFERENCE
ase (GALT) activity in red blood cells. What is the Burton, B. Inborn errors of metabolism in infancy:
most appropriate next step in management? a guide to diagnosis. Pediatrics. 1998; 102(6): E69
Demirbas, D. et al. Hereditary galactosemia. Me-
A. Change to soy based infant formula tabolism. 2018; 83: 188-196.
B. Initiation of continuous feedings with
breastmilk
C. Adding cornstarch to feedings Question 45
D. Change to tyrosine restricted formula A 20-day old female presents for evaluation due
E. Elimination of dietary fructose to jaundice. On examination, the patient is noted
to have pale appearing stool in her diaper. Her
CORRECT ANSWER: A screening labs demonstrate AST 204 IU/L, ALT
107 IU/L, GGT 275 IU/L, total bilirubin 6.3mg/
RATIONALE dL and direct bilirubin 3.0 mg/dL. Her initial ab-
This infant is noted to have neonatal onset hypo- dominal ultrasound showed a positive triangular
glycemia, raising concern for an inborn error of cord sign. Her liver biopsy showed bile duct pro-
metabolism. The positive urinary reducing sub- liferation and hepatocanalicular cholestasis. What
stances indicates that the patient has a carbohy- is the most appropriate next step in management?
drate other than glucose in their urine, indicating
an inborn error in carbohydrate metabolism. The A. Magnetic resonance cholangiopancreatography
most common cause of a positive urinary reducing (MRCP)
substances is galactosemia, which is also support- B. Intraoperative cholangiogram
ed by the reported early onset liver disease. Neo- C. Kasai procedure
nates with galactosemia also have a high rate of D. Observation
E. coli sepsis and absent red reflexes due to early E. Initiation of ursodiol
cataract formation. Classic galactosemia is caused
by a deficiency of galactose-1-phosphate uridyl- CORRECT ANSWER: B
transferase (GALT). It is an autosomal recessive
condition with a prevalence of 1:40,000-60,000. RATIONALE
This deficiency results in a buildup of galactose- Biliary atresia is the common surgery managed
1-phosphate after intake of galactose and leads to liver condition in infants, occurring in one in
direct toxic effects on numerous organ systems 8,000-16,000 depending on location. It is char-
including the brain, eyes and liver. The toxic ef- acterized by progressive ascending obliteration of
fect results in the characteristic long-term com- the extrahepatic biliary tree, resulting in promi-
plications of cataracts, mental retardation and nent cholestasis and early onset cirrhosis. Pa-
Chapter 16 — Issues in pediatric gastroenterology 493

tients typically present within the first one to two Question 46


months of life with jaundice, acholic stools, and A 12-year-old Hispanic male presented in clinic
dark urine. Laboratory findings with this condi- for evaluation for elevated aminotransferases for
tion demonstrate direct hyperbilirubinemia along over six months. On examination, the patient is
with aminotransferases, alkaline phosphatase and obese with a BMI of 37 kg/m2 and does not have
ɣ-glutamyltranspeptidase (GGT). Ultrasound is appreciable hepatosplenomegaly. His liver ultra-
typically performed next to screen for a possible sound demonstrated increased echogenicity with-
choledochal cyst. The ultrasound can almost out focal abnormalities. Screening labs showed
demonstrate an atrophic gallbladder (normal AST 90 U/L, ALT 113 U/L, total bilirubin 0.7mg/
gallbladder seen in 20 percent of cases) as well dL with GGT, albumin and alkaline phosphatase
as the “triangular cord sign”. Liver biopsy has a all being within normal limits. Additional screen-
diagnostic accuracy of around 90 percent, demon- ing labs for alpha-1-antitrypsin deficiency, autoim-
strating preserved hepatic architecture, bile duct mune hepatitis, Wilson disease and viral hepatitis
proliferation, bile plugs and periportal fibrosis. An were all negative. What is the most likely finding
intraoperative cholangiogram is finally performed on liver biopsy?
to assess patency of the biliary system. Without
surgical correction, cirrhosis and portal hyper- A. Ballooning degeneration of hepatocytes with
tension arise around two months of age with few macrosteatosis
children surviving past 18 months of age. To try B. Mixed portal tract inflammation with portal
to reestablish bile flow, patients undergo a Kasai venule endothelialitis (ACR)
portoenterostomy. Early identification of patients C. PAS-positive, diastase-resistant globules in the
with biliary atresia is critical because the sooner endoplasmic reticulum of hepatocytes (A1AT)
a Kasai procedure is performed, the better the D. Mononuclear and plasma cell infiltration of the
chance to achieve bile flow. Survival with native portal areas (AIH)
liver appears to be highest if a Kasai is performed E. Bile ductular proliferation with canalicular and
within the first 60 days of life. Despite undergoing cellular bile stasis (BA)
a Kasai, 1/3 of patients will not achieve bile flow
and will require liver transplantation as an infant. CORRECT ANSWER: A
Around 50 percent of those who achieve bile flow
will have ongoing inflammation leading to devel- RATIONALE
opment of cirrhosis and need for liver transplanta- This patient is noted to have a long history of obe-
tion at a mean age of 5.4 years. This patient was sity with elevated aminotransferases. Screening
noted to have concern for biliary atresia based on labs for other causes of chronically elevated amino-
cholestasis, a positive triangle sign on liver ultra- transferases were negative, suggesting underlying
sound and bile duct proliferation and bile stasis on nonalcoholic fatty liver disease (NAFLD). NAFLD
liver biopsy. A cholangiogram is the most appro- is a chronic liver disease caused by excess fat ac-
priate test to assess biliary system patency prior to cumulation in the liver. It is most commonly found
a Kasai portoenterostomy. in the setting of obesity and can result in fibrosis
and eventual cirrhosis. Other causes for steatosis
REFERENCES in children include metabolic disease, medication
Sanchez-Valle, A. et al. Biliary atresia: epidemiol- use and malnutrition. The prevalence of NAFLD is
ogy, genetics, clinical update, and public health 0.7 percent in children ages two to four years of age
perspective. Adv Pediatr. 2017; 64(1): 285-305. and increases up to 29-38 percent in obese chil-
Wyllie R. et al. Pediatric Gastrointestinal and dren. Hispanic children appear to have the highest
Liver Disease. 4th edition. Elsevier Saunders, risk of the condition, having a four-fold higher risk
Philadelphia, 2011. compared to non-Hispanics, with African-American
494 Digestive Diseases Self-Education Program®

children having the lowest risk. Conditions like A. Liver ultrasound


obstructive sleep apnea, diabetes mellitus type 2 B. Liver biopsy
and panhypopituitarism also increases the risk for C. Discontinuation of breastfeeding
NAFLD in children. NAFLD should be considered D. Formula supplementation
in obese children age 10 and older who chroni- E. Fractionate bilirubin
cally have an ALT 50 or greater in males and 44 or
greater in females. Routine liver ultrasound has not CORRECT ANSWER: E
been shown to be accurate for detecting or quanti-
fying steatosis in children. Diagnosis is confirmed RATIONALE
by liver biopsy demonstrating steatosis involving Most infants have a degree of jaundice soon after
more than five percent of hepatocytes or liver tissue birth that is called physiological jaundice. This
by weight. Nonalcoholic steatohepatitis (NASH) unconjugated hyperbilirubinemia condition is
occurs when steatosis is accompanied by hepatocel- caused by increased red blood cell breakdown,
lular injury and possible fibrosis. For this patient, decreased bilirubin uridine 5’-diphospho-
he would be expected to have NASH with biopsy glucuronsyltransferase (UGT) activity at birth
findings of balloon degeneration of hepatocytes with and lack of intestinal flora to metabolize biliru-
macrosteatosis. Acute cellular allograft rejection bin. Physiological jaundice typically resolves
commonly has mixed portal tract inflammation with at two weeks of life. Breastfeeding jaundice is
portal venule endothelialitis. Alpha-1-antitrypsin an unconjugated hyperbilirubinemia condition
deficiency can present with PAS-positive, diastase- that occurs in the first one to two weeks of life
resistant globules in the endoplasmic reticulum and is caused by impaired breast milk intake
of hepatocytes. Autoimmune hepatitis commonly and relative dehydration. It is treated by supple-
has mononuclear and plasma cell infiltration of the mentation with expressed breastmilk or formula.
portal area. Finally, biliary atresia is characterized Breast milk jaundice is also unconjugated and
by bile duct proliferation with bile stasis. starts after the first week of life. It is thought to
be caused by breast milk compounds inhibiting
REFERENCE UGT activity and increasing enterohepatic circu-
Vos, M. et al. NASPGHAN clinical practice guide- lation. The patient in this vignette is noted to be
line for the diagnosis and treatment of nonalco- breastfed and have visible jaundice. Her stool is
holic fatty liver disease in children: recommen- reported to be yellow, suggesting that she does
dations from the Expert Committee on NAFLD not have disruption of bile flow in a condition
(ECON) and the North American Society of like biliary atresia. The next step in management
Pediatric Gastroenterology, Hepatology and Nutri- would be to determine if her hyperbilirubinemia
tion (NASPGHAN). J Pediatr Gastroenterol Nutr. is conjugated or unconjugated in nature by differ-
2017; 64(2): 319-334. entiating the total bilirubin. Conjugated hyper-
bilirubinemia in neonates is defined as a direct
bilirubin greater than 2 mg/dL or greater than 20
Question 47 percent of total bilirubin. If the hyperbilirubine-
A 3-week old female presents to clinic with per- mia was conjugated in nature, prompt diagnostic
sistent jaundice. The patient is exclusively breast- evaluation would be required even in a clinically
fed and, she is gaining weight appropriately. On well appearing infant.
examination, the patient does not appear to have
hepatosplenomegaly. A yellow and seedy stool REFERENCE
was seen in clinic. Screening labs demonstrated Pan, D. and Rivas, Y. Jaundice: Newborn to
a total bilirubin of 6.2mg/dL. What would be the Age 2 months. Pediatr Rev. 2017; 38(11):
recommended next step in management? 499-510.
Chapter 16 — Issues in pediatric gastroenterology 495

Question 48 Raphael, K. and Willingham, F. Hereditary


A 12-year-old female presents with her fourth pancreatitis: current perspectives. Clin Exp Gas-
episode of pancreatitis, with her symptoms start- troenterol. 2016; 9: 197-207.
ing around 10 years of age. She is asymptomatic
in between her episodes of pancreatitis. Her father
notes that he and his father have also required Question 49
numerous admissions due to pancreatitis, with his A 5-year-old male with a history of biliary atresia
father having a history of pancreatic cancer. status post Kasai procedure presents for routine
follow-up. His mother reports that the patient is
Which genetic defect may occur in this patient? having increased difficulties with finding his way
to the restroom at night. Previously, the patient
A. CTRC was independent but now is needing assistance
B. PRSS1 at night.
C. CFTR
D. SPINK1 What vitamin deficiency may be playing a role in
E. All of the above the patient’s clinical change?

CORRECT ANSWER: E A. Vitamin A


B. Vitamin C
RATIONALE C. Vitamin D
This patient is presenting with multiple episodes D. Vitamin E
of pancreatitis consist with a diagnosis of acute E. Vitamin K
recurrent pancreatitis. Possible etiologies for
acute recurrent pancreatitis include hereditary CORRECT ANSWER: A
pancreatitis, anatomic abnormalities (pancreatic
divisum, sphincter of Oddi dysfunction), gallstone RATIONALE
disease, autoimmune pancreatitis and cystic fibro- Patients with cholestasis like this patient with
sis. With the history of multiple family members biliary atresia are at increased risk for fat-soluble
with recurrent pancreatitis, hereditary pancreati- vitamin deficiency. The fat-soluble vitamins in-
tis is of primarily concern. Hereditary pancreatitis clude vitamin A, D, E and K. Vitamin C is primar-
commonly involves the genes PRSS1, SPINK1, ily absorbed in the ileum, with this patient not
CFTR, and CTRC. PRSS1 causes hereditary pan- being at high risk for deficiency with a regular fruit
creatitis in an autosomal dominant pattern due to containing diet. Vitamin A is absorbed primarily
premature activation of trypsin. SPINK1 is inher- in the upper portion of the small intestine. The
ited in an autosomal recessive pattern and causes retinal form of vitamin A is needed to form rho-
pancreatitis through lowering the levels of trypsin dopsin in the eye to allow for low light absorption.
inhibitor. Dysfunction of CFTR results in failure Deficiency of vitamin A can lead to night blind-
of alkalization of the acinar cells with resulting ness, complete blindness, xerophthalmia forma-
retention of zymogens in the pancreatic duct and tion and keratomalacia. This patient is having
digestion of pancreatic tissue. Finally, CTRC al- progressive worsening of night vision, suggesting
terations interfere with trypsin destruction. vitamin A deficiency. Treatment consists of dedi-
cated vitamin A supplementation.
REFERENCE
Hasan, A. et al. The role of genetics in pancreati- REFERENCE
tis. Gastrointest Endosc Clin N Am. 2018; 28(4): Wolf, G. A history of vitamin A and retinoids.
587-603. FASEB J. 1996; 10(9): 1102-1107.
496 Digestive Diseases Self-Education Program®

Question 50
An exclusively breastfed 15-month-old female
presents for evaluation due to an elevated alkaline
phosphatase. On examination, the patient is not-
ed to have a persistently open fontanelle, beading
on the costochondral junctions, and progressive
lateral bowing of the femur and tibia. What vita-
min is most likely to be deficient in this patient?

A. Vitamin A
B. Vitamin C
C. Vitamin D
D. Vitamin E
E. Vitamin K

CORRECT ANSWER: C

RATIONALE
This patient is noted to have a persistently open
fontanelle, beading on the costochondral junc-
tions and bowing of the femur and tibia. These
findings are commonly seen with rickets. Other
possible physical exam can include decreased
growth velocity, frontal bossing, decreased muscle
tone and delayed motor milestones. Rickets is
classified as poor mineralization of the growth
plate due to abnormal serum calcium or phospho-
rus. Calcipenic rickets is caused by insufficient
vitamin D or insufficient intake/absorption of
calcium. Phosphopenic rickets is typically related
to renal phosphate wasting. To assess for rickets,
parathyroid hormone, calcium and phosphorus
levels should be obtained. If the parathyroid level
is elevated, then the vitamin D level should be as-
sessed. Treatment for vitamin D deficiency rickets
focuses on vitamin D supplementation. This
patient is at increased risk for rickets due to her
continuing to exclusively breastfeed.

REFERENCE
Lambert, A. and Linglart, A. Hypocalcaemic and
hypophosphatemic rickets. Best Pract Clin Endo-
crinol Metab. 2018; 32(4): 455-476.
Answers & critiques

CHAPTER 17

Small bowel disease


Matthew Whitson, MD, MSEd, Parakkal Deepak, MBBS, MS and
Christina Ha, MD, FACG, AGAF

Question 1 REFERENCE
Which of the following patients is the most Leffler DA, Schuppan D. Update on serologic
appropriate candidate for TTG-IgA testing for testing in celiac disease. Am J Gastroenterol.
celiac disease? 2010. 105(12):2520-4.

A. An 18-month-old child whose mother has


recently confirmed celiac disease Question 2
B. A 33-year-old female patient with CVID Which of the following disease states can cause
whose brother was recently diagnosed with villous blunting to be seen on duodenal biopsies?
celiac disease
C. A 24-year-old male patient with unexplained A. Autoimmune gastritis
iron deficiency who maintains a strict low- B. Proton pump inhibitor therapy
FODMAP diet C. Gluten intolerance
D. A 26-year-old female patient with elevated D. Giardiasis infection
AST and ALT who maintains a diet with no E. Pancreatic insufficiency
restrictions
CORRECT ANSWER: D
CORRECT ANSWER: D
RATIONALE
RATIONALE Villous blunting is not specific to celiac disease,
TTG-IGA is the primary diagnostic test for celiac though this may be the most frequent associa-
disease. In patients with an indication for celiac tion with villous blunting. Villous blunting can
testing who currently include gluten in their also be seen in autoimmune enteropathy, medi-
diet (D), this is the appropriate first test. Pa- cations induced enteropathy with NSAIDS or
tients who should undergo celiac testing include olmesartan, small intestine bacterial overgrowth
patients with unexplained iron deficiency, (SIBO), collagenous sprue, tropical sprue, com-
unexplained gastrointestinal complaints includ- mon variable immunodeficiency (CVID), eosino-
ing bloating and diarrhea, patients with a first- philic enteritis, whipples disease, and Giardiasis
degree relative with celiac, and patients with infection (D).
elevated liver enzymes. Testing should only be While autoimmune gastritis (A), PPI therapy
performed after the age of two years old (A) as (B), and pancreatic insufficiency (E) are poten-
the sensitivity for the test is significantly less in tial causes of SIBO, they are not directly associ-
young children. Patients with low IGA levels (B) ated with villous blunting in the duodenum.
or not including gluten in their diet (C) should While gluten intolerance (C) may have similar
not undergo TTG-IGA testing. clinical findings as celiac disease, gluten intoler-

497
498 Digestive Diseases Self-Education Program®

ance has not been shown to be associated with of IV therapy with a cephalosporin (C) or car-
villous blunting on endoscopic biopsy. bapenem in order to reach the CNS, which can be
affected by T. whipplei with no symptoms. There
REFERENCE is no role for Immunosuppressive therapy (E),
Pallav K, Leffler DA, Tariq S et al. Noncoeliac anti-parasitic therapy (B), or dietary therapy (A)
enteropathy: The differential diagnosis of villous in the treatment of Whipple’s Disease.
atrophy in contemporary clinical practice. Aliment
Pharmacol Ther. 2012;35(3):380-90. REFERENCES
Moos et al. Changing paradigms in Whipple’s
disease and infection with T. whipplei. Eur J Clin
Question 3 Microbiol Infect Dis. 2011; 30:1151-1158.
A 52-year-old male with past medical history of Feurle GE, Junga NS, Marth T. Efficacy
hypothyroidism presents with two months of of ceftriaxone or meropenem as initial thera-
watery diarrhea and weight loss. In addition, he pies in Whipple’s disease. Gastroenterology.
notes that his knee has been hurting more signifi- 2010;138(2):478-486.
cantly and he is having more regular headaches.
He undergoes upper endoscopy with duodenal
biopsy which reveals villous atrophy with PAS- Question 4
positive macrophages. Which of the following is Which of the following clinical situations has the
the most appropriate treatment for this patient? best indication for oral steroid therapy?

A. Gluten Free Diet A. A 23-year-old female with a past medical


B. Mebendazole history of fistulizing small bowel Crohn’s
C. Ceftriaxone followed by Bactrim disease with new onset fevers and chills
D. Tetracycline B. A 34-year-old female with celiac disease
E. Infliximab therapy found to have a persistent TTG-IgA elevation
after two months of gluten free diet
CORRECT ANSWER: C C. A 50-year-old male presenting with weight loss
and diarrhea found to have PAS+ Macrophages
RATIONALE on duodenal biopsy
This patient has Whipples Disease, a chronic, D. A 37-year-old male presenting with eight weeks
multi system infection of the Tropheryma whip- of diarrhea with a positive anti-enterocyte
plei species. Classic symptoms include arthralgia antibody
and diarrhea, but common symptoms also include
weight loss, abdominal pain, lymphadenopathy, CORRECT ANSWER: D
skin changes, and neurologic symptoms. Diagno-
sis is made by demonstrating the classic periodic RATIONALE
acid–Schiff (PAS)–positive macrophages in the Steroid therapy, either in systemic form (i.e. sol-
lamina propria of small intestinal biopsies. This umedrol, prednisone) or topical (i.e. Budesonide)
may be associated with villous blunting and lym- plays a role in many gastrointestinal diseases of
phatic dilatation. the small bowel. These may include collagenous
Initially tetracycline (D) was the mainstay of enteritis, eosinophilic enteritis, Crohn’s disease,
therapy, but resistance has significantly increased bechet’s disease, refractory celiac disease, and
leading to decrease usage as first line therapy. autoimmune enteropathy.
Bactrim is now first line therapy and in patients Answer D represents a patient with autoim-
more significantly ill, it is preceded by two weeks mune enteropathy. Autoimmune enteropathy is
Chapter 17 — Small bowel disease 499

a rare disorder characterized by severe diarrhea, D. Small intestinal biopsies positive for Congo
weight loss secondary to malabsorption, villous Red Staining with birefringence under
atrophy of the small intestinal mucosa and the polarized light
presence of autoantibodies, primarily anti-entero- E. Small intestinal biopsies demonstrating
cyte and anti-goblet cell antibodies. Oral steroids, granulomas
either prednisone or budesonide, is considered
first-line therapy. Second-line therapies may CORRECT ANSWER: D
include mycophenolate mofetil, azathioprine, or
cyclosporine RATIONALE
In Answer A, the patient does have small bowel This patient has amyloidosis of the small bowel.
crohn’s disease, however new onset fevers and Amyloidosis of the small intestine can present
chills in a patient with fistulizing disease is con- with malabsorption, diarrhea, abdominal pain, or
cerning for an abscess formation or other infec- ulceration resulting in gastrointestinal bleeding.
tion. Steroids would not have an indication at this Rarely it presents with perforation of the bowel.
time. In Answer B, the patient has celiac disease Non-GI involvement of primary amyloidosis can
but has not met criteria for refractory celiac disease include restrictive cardiomyopathy, chronic kid-
as she has not undergone 6 months of a gluten free ney disease, peripheral neuropathy, weight loss, or
diet. Lastly, there is no role for steroid therapy in periorbital purpura.
Whipples disease, an infection with Tropheryma Amyloidosis of the small bowel is diagnosed
whipplei bacteria, as seen in answer C. by biopsy of the affected area. The gold standard
for diagnosing amyloidosis is Congo red staining
REFERENCE of biopsies typically obtained from the rectum or
Akram S, Murray JA, Pardi D et al. Adult au- duodenum that characteristically shows apple-
toimmune enteropathy: Mayo Clinic Roches- green birefringence under polarized light.
ter experience. Clin Gastroenterol Hepatol. Celiac disease is diagnosed by TTG-IGA (A),
2007;5(11):1282-1290. Autoimmune enteropathy can be associated with
auto-enterocyte antibodies (B), and Crohn’s dis-
ease and multiple infections including tuberculo-
Question 5 sis can be associated with granulomas (E). Gastric
A 70-year-old African-American male with signifi- hyperkinesis (C) is not associated with ulceration
cant past medical history of type 1 diabetes and of the small bowel.
restrictive cardiomyopathy presents to your office
with six weeks of abdominal discomfort and diar- REFERENCE
rhea. This has been associated with a 15-lb. weight Ebert et al. Gastrointestinal manifestations
loss. Most recently, he has some dark tarry stools of amyloidosis. Am J Gastroenterol. 2008;
for the past three days. An upper endoscopy and 103:776-787.
colonoscopy are grossly normal. You perform a
video capsule endoscopy which demonstrates an
extended area of ulcerated, edematous, friable mu- Question 6
cosa in the mid small bowel. Which of the follow- A 67-year-old man with a history of chronic
ing test results is most likely for this patient: kidney disease on dialysis presents with diarrhea
and a 20 pound weight loss. On endoscopy, the
A. Elevated TTG-IgA mucosa is granular with polypoid protrusions.
B. Positive Auto-Enterocyte Antibody Histologic examination is positive for Congo read
C. Accelerated transit time in the stomach on staining. Which of the following treatment options
SmartPill testing (Wireless Motility Capsule) is the most appropriate for this patient?
500 Digestive Diseases Self-Education Program®

A. Gluten Free Diet heart failure, sepsis, and recent myocardial infarc-
B. Immunosuppressive therapy tion. Traditional presentation includes severe,
C. Supportive Care and Anti-diarrheal therapy acute abdominal pain out of proportion to the
D. Proton Pump Inhibitor therapy exam. CT Angiography or MR Angiography are
E. Anti-Tumor Necrosis Factor Therapy the major diagnostic tools, as lab testing includ-
ing lactate may be delayed. Treatment is usually
CORRECT ANSWER: C emergent surgical revascularization. Exploratory
laparotomy may be required if peritonitis is pres-
RATIONALE ent on exam. There is a significant urgency to
The mainstay of therapy for small bowel amy- treatment as the mortality rate is over 50 percent
loidosis is supportive care. There is no role for and studies have shown that the greater that time
gluten free diet like there is for celiac disease from symptom presentation to intervention, the
or gluten intolerance (A), immunosuppressive intestinal viability decreases significantly to nearly
therapy (B), Acid Suppression (D), or Anti-TNF 18 percent after 24 hours (E).
therapy (E) On the other hand, chronic mesenteric isch-
emia can present as sitophobia (C), can be caused
REFERENCE by artherosclerosis (A), and can be tested using
Ebert et al. Gastrointestinal manifestations of Balloon Tonometry (B). Interventional treat-
amyloidosis. Am J Gastroenterol. 2008; ment, when necessary, includes percutaneous
103:776-787. transluminal mesenteric angioplasty on an elec-
tive basis (D).

Question 7 REFERENCES
Which of the following is true regarding acute American Gastroenterological Association Medical
mesenteric ischemia? Position Statement: Guidelines on Intestinal Isch-
emia. Gastroenterology. 2000;118(5):951-953.
A. Mesenteric atherosclerosis is the major cause. LoboMartinez E, Carvajosa E, Sacco O et al.
B. Balloon Tonometry testing can be used to Embolectomy in mesenteric ischemia. Rev Esp
distinguish the clinical significance of a Enferm Dig. 1993; 83:351-354.
radiologist finding of mesenteric ischemia.
C. Sitophobia is an associated symptom.
D. Elective percutaneous transluminal mesenteric Question 8
angioplasty is an effective treatment. At 36-year-old Caucasian female with past medi-
E. After 24 hours of symptoms, the ability to cal history of hypothyroidism presents to the
salvage healthy intestinal tissue is less than gastroenterology office for ongoing bloating. This
50 percent. bloating is associated with altered bowel habits
resulting in two to three bowel movements a day.
CORRECT ANSWER: E She has tried a Histamine-2 Antagonist agent and
Simethicone with limited results.
RATIONALE
Acute mesenteric ischemia is a serious, potentially The gastroenterologist sends blood testing with
fatal disease in which blood vessel is acutely oc- results indicated below:
cluded via emboli, thrombi, or potentially vaso-
constriction resulting in damage to the bowel. WBC 4.6K/mL
Risk factors include older age, hypercoagulable Hgb 13.4 gm/dL
states (i.e. Protein C deficiency), long standing Hct 40.2%
Chapter 17 — Small bowel disease 501

Plt 178K/mL REFERENCES


Na 137 mEq/L Leffler DA, Schuppan D. Update on serologic test-
K 4.9 mEq/L ing in celiac disease. Am J Gastroenterol. 2010.
Cl 116 mEq/L 105(12):2520-4.
HCO3 24 mEq/L Chow MA, Lebwohl B, Green PH et al. Immu-
BUN 7 mg/dL noglobulin A deficiency in celiac disease. J Clin
Cr 0.7 mg/dL Gastroenterol. 2012. 46(10): 850-4.
Glucose 87 mg/dL

AST 24 U/L Question 9


ALT 18 U/L A 40-year-old Asian female with a history of
Alk Phos 58 U/L chronic HBV currently off therapy presents with
T Bili 0.7 mg/dL abdominal pain for three days. She has been hav-
ing nausea, vomiting, and some decrease in bowel
TTG IGA < 5 U/mL habits. She had a similar episode approximately
IGA 4.5 mg/dl six months ago when she was found to have acute
IGG normal cholecystitis that was treated with antibiotics
IGM normal and then by cholecystectomy. She presents to
the Emergency Room where she is found to be
Which of the following is the next most afebrile, tachycardic to 110 bpm, and with a blood
appropriate test in this patient’s workup? pressure of 100/60 mm Hg. While awaiting lab
results, she undergoes an abdominal CT Scan that
A. EGD with duodenal biopsies demonstrates pneumatosis of the small intestine.
B. TTG - Deaminated gliadin peptide The most likely cause of this patient’s finding is:
C. Empiric trial of gluten free diet
D. Lactulose breath test A. Polyarteritis Nodosa (PAN)
E. Wireless motility capsule (SmartPill) B. Chronic Mesenteric Ischemia
testing C. Gallstone Ileus
D. Bechet’s Disease
CORRECT ANSWER: B E. Rheumatoid Vasculitis

RATIONALE CORRECT ANSWER: A


This is a patient with IGA deficiency undergo-
ing celiac testing. A TTG-IGA is an ineffective RATIONALE
test for this patient. To complete celiac testing, Polyarteritis Nodosa (PAN) affects both small and
a TTG – Deaminated gliadin peptide is the next medium sized arteries. It is associated with Hepa-
appropriate test (B). If sero-negative celiac is titis B. GI involvement often affects the small in-
suspected, then EGD with duodenal biopsies may testine and gallbladder resulting in pneumatosis,
be a reasonable next step (A). A trial of gluten free infarction, or perforation. The patient presented
diet (C) would not diagnosis celiac disease as it appears to have had possible gallbladder involve-
can be a treatment for many other diseases includ- ment previously and now pneumatosis of the
ing glucose intolerance. Lactulose breath test is small bowel. PAN is the most likely diagnosis.
for small intestinal bacterial overgrowth (D) and Gallstone ileus is the entry of a gallstone into
Wireless Motility Capsule (E) testing is for intesti- the bowel via a biliary-enteric fistula (C) which
nal dysmotility. Both may be appropriate testing does not apply to this case. Acute mesenteric
if the celiac workup is negative. ischemia may present with sudden symptoms and
502 Digestive Diseases Self-Education Program®

pneumatosis on imaging, however chronic mesen- CORRECT ANSWER: A


teric ischemia (B) does not typically present with
this acuity or imaging. Rheumatoid vasculitis (E) RATIONALE
is an inflammatory condition of small and medium This patient has a protein losing enteropathy as
sized vessels which presents many years after the indicated by his diarrhea, peripheral edema, and
onset of rheumatoid arthritis. While it may have positive stool alpha-1 antitrypsin test. Multiple
the same presentation, this is the wrong clinical diseases, particularly in their later stages, can
scenario. Bechet’s disease (D), a systemic vasculi- be associated with a protein losing enteropathy
tis, tends to affect young male patients and involve including primary intestinal lymphangectasia,
other sites including the oral mucosa. The clinical Crohn’s disease of the small intestine, small intes-
scenario does match any of these scenarios, mak- tinal bacterial overgrowth (SIBO), and amyloido-
ing PAN the best option. sis of the small intestine (A).
Celiac disease (B) is not associated with protein
REFERENCE losing enteropathy. While Crohn’s disease can
Ebert EC, Hagspiel KD, Nagar M et al. Gastroin- be associated with protein losing enteropathy,
testinal involvement in polyarteritis nodosa. Clin ulcerative colitis (C) is not usually associated with
Gastroenterol Hepatol. 2008;6(9):960-6. it. Small bowel dysmotility (D) does not impact
absorption or secretion unless associated with
SIBO, making this a wrong answer.
Question 10
A 65-year-old male no significant past medical
history presents with significant diarrhea. He Question 11
reports that for the past three months, he has had Which of the following clinical scenarios is MOST
four to five bowel movements a day. He charac- appropriate to send for small intestinal bacterial
terizes them as greasy and foul smelling, but not overgrowth (SIBO) lactulose breath testing?
entirely watery. He notices no blood or mucous in
the stool. Over the same time period, he has also A. A 55-year-old female with a past medical
noticed increased swelling in both of his ankles. history of heartburn on chronic proton pump
The physician sends a broad workup up. inhibitor therapy who has been having three
weeks of bloating and loose stools
Stool testing is seen as below: B. A 23-year-old female with excessive flatulence
and loose stools who is currently being treated
Clostridium difficile – Negative for a urinary tract infection with ciprofloxacin
Stool Ova and Parasite – Negative C. A 30-year-old male with gastroparesis
Stool Culture – Negative controlled on daily metoclopramide therapy
Stool Elastase – within normal limits who has developed two weeks of bloating and
Fecal Fat (spot test) – within normal limits loose stools
Stool Alpha-1 Antitrypsin – Elevated D. A 52-year-old female with a history of obesity
status-post roux-en-y procedure ten years ago
Which of the following diagnoses best explains complicated by iron deficiency who has
the patient’s symptoms? worsening bloating and loose stools for the
past two weeks
A. Amyloidosis involving the small intestine
B. Celiac disease CORRECT ANSWER: D
C. Ulcerative Colitis
D. Small bowel dysmotility
Chapter 17 — Small bowel disease 503

RATIONALE A. Lactulose breath test


Small intestinal bacterial overgrowth (D) is patho- B. Helicobacter pylori breath test
logic overgrowth of bacteria in the upper small C. Stool giardia
intestine. This is often caused by altered anatomy D. Fecal calprotectin
(i.e. gastric bypass, small diverticulosis) or altered E. Clostridium difficile stool test
motility (i.e. gastroparesis, small bowel dysmotil-
ity). It is often associated with watery diarrhea, CORRECT ANSWER: A
bloating, and increased flatulence. It can also lead
to malabsorption of vitamins including B12 and RATIONALE
fat-soluble vitamins. Small intestinal bacterial overgrowth is pathologic
Testing for SIBO is typically completed by a overgrowth of bacteria in the upper small intes-
lactulose or glucose breath test in which patients’ tine. This is often caused by altered anatomy (i.e.
breaths are testing for hydrogen and methane gastric bypass, small diverticulosis) or altered
after ingestion of a substrate. The accuracy of motility (i.e. gastroparesis, small bowel dysmotil-
breath tests is quite variable and can be affected ity). It can also be triggered after antibiotic use
by multiple medications including: acid-suppres- that leads to an imbalance of the bacterial flora of
sion agents, promotility drugs, laxatives, antibiot- the gut. Symptoms can include watery diarrhea,
ics, and probiotics. Answer D is correct as it is the bloating, and increased flatulence. It can also lead
only patient who is not currently taking a medi- to malabsorption of vitamins including B12 and
cation that can impact the quality of the study. fat-soluble vitamins. Patients with celiac disease
Empiric treatment would be an appropriate option have a higher incidence of SIBO. A lactulose
in these patients. breath test is the first line test for SIBO.
H. Pylori infection (B) may cause ulcers, ma-
REFERENCE lignancies, or dyspepsia. While giardia infection
Rezaie A, Buresi M, Lembo A, et al. et al. Hydro- (C) can cause bloating and loose stools, the patient
gen and Methane-Based Breath Testing in Gastro- has not had any recent travel to endemic areas.
intestinal Disorders: The North American Consen- Clostridium difficile (E) can cause a diarrhea after
sus. Am J Gastroenterol. 2017. 112; 775-84. antibiotics but tends to present with a more severe
clinical course and have a leukocytosis. The dura-
tion of symptoms also makes this less likely. Fecal
Question 12 calprotectin (D) is a non-invasive test to look for
A 23-year-old female with a past medical history bowel inflammation and is often in positive in zz
of celiac disease, well controlled on gluten free patients.
diet, presents for worsening GI symptoms over
the past six weeks. She has noticed that she now REFERENCE
has up to six loose bowel movements a day. They Khoshini R, Dai SC, Lezcano S et al. A systematic
have been foul smelling and associated with bloat- review of diagnostic tests for small intestinal bacte-
ing. She denies any weight loss, blood in stool, rial over-growth. Dig Dis Sci. 2008; 53:1443-1454.
or nausea/vomiting. She has not had any recent
travel. The only change to her health has been a
recent treatment of a urinary tract infection with Question 13
antibiotics about eight weeks ago. Repeat TTG- A 50-year-old Caucasian male presents to discuss
IgA testing and her WBC were normal. new GI symptoms. He has been having looser
stools for the past two to three months that he takes
Which of the following tests will most likely be Loperamide for as needed. His fatigue and joint
positive? aches are now prohibiting him from completing
504 Digestive Diseases Self-Education Program®

some tasks on the farm, which he attributes to the Question 14


diarrhea. He has had no history of travel in the Which of the following patients would be the most
past year and has started no new medications. appropriate candidate for small intestinal transplant?
Reviewing the chart, you notice he has lost 10 lbs.
His only lab abnormality is an albumin of 1.8 g/dL. A. A 24-year-old female patient with short gut
Which infection is the most likely cause of syndrome on chronic TPN and a history of
his symptoms? superficial vein thrombosis
B. A 35-year-old male patient with protein losing
A. Giardiasis enteropathy due to HIV who has frequent
B. Salmonella electrolyte derangements despite adjustments
C. Clostridium difficile to TPN
D. Tropheryma Whipplei C. A 64-year-old female patient with past medical
E. Tropical sprue history of breast cancer status post resection
and collagenous sprue requiring TPN who has
CORRECT ANSWER: D multiple admissions per year for dehydration
D. An 18-year-old female with history of acute
RATIONALE mesenteric ischemia episode resulting in lifezzz
This patient has Whipples disease (D), a chronic, long TPN who has her first case of bacteremia.
multi system infection of the Tropheryma whip-
plei species. Classic symptoms include arthralgia CORRECT ANSWER: C
and diarrhea, but common symptoms also include
weight loss, abdominal pain, lymphadenopathy, skin RATIONALE
changes, and neurologic symptoms. This patient Small Bowel Transplants are used in patients with
also has hypoalbuminemia which is often seen in intestinal failure and resulting nutritional and
Whipples disease. The disease is more common in metabolic complications. Indications for small
Caucasian males with a mean age of approximately bowel transplant include TPN failure for the fol-
50 years old. It is suspected to be agricultural in ori- lowing reasons:
gin. Diagnosis is made by demonstrating the classic - Impending or overt liver failure
periodic acid–Schiff (PAS)–positive macrophages in - Thrombosis of the major central venous chan-
the lamina propria of small intestinal biopsies. nels (two thromboses in subclavian, jugular, or
Giardiasis (A) may cause loose stools and femoral veins)
bloating, but does not cause many extra luminal - Frequent line infections (two episodes of sys-
ailments. Salmonella (B) is associated with acute, temic sepsis due to line infection OR one episode
potentially bloody, diarrhea. Clostridium diffi- of line related fungemia OR septic shock or ARDS
cile (C) is a bacterial infection of the GI tract that due to line infection)
is typically seen after antibiotic therapy or after - Frequent severe dehydration
nosocomial exposures. Tropical sprue (E) may
present with many of the above symptoms includ- Reasons for exclusion of small bowel transplant
ing chronic diarrhea, weight loss, bloating, and include metastatic malignancy, chronic systemic
hypoalbuminemia. However, it is present largely infections, or HIV infection.
in the Caribbean and southern Asia.
REFERENCE
REFERENCE American Gastroenterological Association Medi-
Moos et al. Changing paradigms in Whipple’s cal Position Statement: Short Bowel Syndrome
disease and infection with T. whipplei. Eur J Clin and Intestinal Transplantation. Gastroenterology.
Microbiol Infect Dis. 2011; 30:1151-1158. 2003; 124:1105-1110.
Chapter 17 — Small bowel disease 505

Question 15 REFERENCE
A 22-year-old male with a past medical history Khoshini R, Dai SC, Lezcano S et al. A systematic
of eosinophilic esophagitis on fluticasone inhaler review of diagnostic tests for small intestinal
440 mcg daily and chronic nausea controlled bacterial overgrowth. Dig Dis Sci. 2008; 53:
with Metoclopramide presents to the GI office for 1443-1454.
new onset bloating and loose stools. Blood tests
including a TTG-IgA and IgA level are within
normal limits. An upper endoscopy demonstrates Question 16
atrophic gastritis on biopsy while a colonoscopy A 62-year-old female with past medical history of
reveals diverticulosis of the transverse colon. He Non Hodgkins Lymphoma treated with chemo-
undergoes a hydrogen breath test which confirms therapy and radiation eight years ago presents to
the diagnosis of small intestinal bacterial your office with increased watery diarrhea. The
overgrowth. patient complains of increased bloating, disten-
sion, and some increased flatulence as well. Infec-
Which aspect of his past medical history tious stool studies have been negative. Endo-
most likely contributed to his development scopic evaluation and biopsies are negative. She
of SIBO: undergoes a CT Enterography demonstrating a 5
cm stricture in her ileum. She is treated with 10
A. Metoclopramide days of rifaximin which resolves the symptoms,
B. Diverticulosis of the large intestine but they return a few weeks after treatment.
C. Eosinophilic esophagitis
D. Atrophic gastritis Which of the following is the next best step in
therapy?
CORRECT ANSWER: D
A. Put patient on regimen of cycling antibiotics
RATIONALE B. Loperamide PRN
Small intestinal bacterial overgrowth is a patho- C. Referral to surgeon for resection of stricture
logic overgrowth of bacteria in the upper small D. Wireless motility capsule (SmartPill) testing
intestine. Risk factors for SIBO include the E. Cholestyramine
following: dysmotility of the gut (I.e. gastropare-
sis), primary systemic scleroderma, small bowel CORRECT ANSWER: C
diverticulosis, altered anatomy (i.e. small bowel
strictures, gastric bypass surgery), celiac disease, RATIONALE
Immunodeficiency (i.e. IgA deficiency), pancreatic This patient has recurrent small intestinal bacte-
insufficiency, and Gastric hypochlorhydria from rial overgrowth (SIBO), a pathologic overgrowth
either atrophy (D) or from medications such as of bacteria in the upper small intestine. Risk fac-
proton pump inhibitors. tors for SIBO include altered anatomy of the small
While medications that slow the bowel may be bowel: small bowel diverticulosis, strictures of
associated with SIBO, pro-motility agents such as the small bowel, gastric bypass, or blind limbs. If
metoclopramide (A) do not have a known associa- altered anatomy are the cause of recurrent SIBO,
tion. Diverticulosis of the large intestine (B) is not treatment should be geared towards reversal of
associated with SIBO, only diverticulosis of the these changes when possible. Resection of a 5 cm
small intestine. And while eosinophilic enteritis stricture is a reasonable next step.
or other inflammatory diseases of the small intes- Cycling antibiotics can be used when the
tine can contribute to SIBO, there is no associa- underlying cause of SIBO is non reversible (A).
tion with eosinophilic esophagitis (C). Loperamide (B) and cholestyramine (E) poten-
506 Digestive Diseases Self-Education Program®

tially might help manage symptoms, but will pleural and pericardial effusions, hypoalbumin-
not fix the underlying problem. There is no role emia hypogammaglobinemia, and potentially
for wireless motility capsule testing (D) in this fat-soluble vitamin deficiencies.
patient, particularly given the ileal stricture. Diagnosis is made by endoscopy where you
may see a creamy yellow-white appearance
of the jejunal villi. Endoscopic biopsy reveals
Question 17 dilated mucosal and submucosal lymphatic ves-
A 28-year-old female with a history of Turner sels. Stool testing may show an elevated alpha 1
syndrome presents and complaints of progres- antitrypsin.
sively worsening odiferous diarrhea that appears The cornerstone of treatment is a low-fat diet
to have float atop the water in the toilet. On supplemented with medium chain triglycerides.
review of symptoms she also reports worsening The goal is to minimize the engorgement of the
lower extremity edema. Infectious stool studies lymphatics. As medium chain triglycerides are
are negative, but she has a positive alpha-1 anti- directly absorbed, they do not cause further lym-
trypsin stool study. On endoscopy, she is found phatic dilation.
to have dilated lymphatics in the second part of
the duodenum. Biopsy results confirm dilated REFERENCE
lymphatics in the mucosal and submucosal lay- Alfano V, Tritto G, Alfonsi L et al. Stable reversal
ers. The next step in treatment is: of pathologic signs of primitive intestinal lym-
phangiectasia with a hypolipidic, MCT-enriched
A. Rifaximin x 10 days diet. Nutrition. 2000;16(4):303- 304.
B. Gluten free diet
C. Short bowel transplant
D. Total parenteral nutrition Question 18
E. Low fat diet supplemented with medium A 20-year-old male presents to the emergency
chain triglycerides room with dark stools for two days. He reports
that he has had ongoing abdominal discomfort
CORRECT ANSWER: E intermittently for the past month, but never
noticed a change in color of his stools until now.
RATIONALE He denies taking any medications. His weight
This patient has primary intestinal lymphangiec- has been stable. Physical exam reveals diffuse
tasia (PIL), also known as Waldmann’s disease. ulcerations in his mouth and on his scrotum.
PIL is a rare condition characterized by dilata- His hemoglobin is 9.6 g/dL on admission. Up-
tion of the intestinal lymphatics which blocks per endoscopy and colonoscopy are normal. A
effective fluid drainage leading to a subsequent video capsule endoscopy reveals deep, cratered
loss of lymph fluid into the GI tract. PIL is as- ulcerations in the jejunum.
sociated with von Recklinghausean syndrome,
Turner syndrome, and Hennekam syndrome. The most likely cause of this patients’ findings is:
Secondary intestinal lymphangiectasia can
present with similar symptoms, but is caused A. Crohn’s disease
by impaired lymphatic flow from lymphoma, B. Bechet’s disease
right sided heart failure, and thoracic duct outlet C. Surreptitious NSAID use
obstruction. D. Mast cell disorder
Major symptoms include a protein losing E. Takasayu arteritis
enteropathy that may result in lymph edema,
peripheral edema, serous effusion - particularly CORRECT ANSWER: B
Chapter 17 — Small bowel disease 507

RATIONALE to the esophagus or duodenum. Which is the


This patient has Bechet’s disease. Bechet’s most likely diagnosis?
disease is a necrotizing, systemic vasculitis. It is
most predominant in young males and is char- A. Gastric adenocarcinoma
acterized largely ulcerations of the oral mucosa B. H. pylori associated ulcer
and the genitalia. It can also involve joints, CNS, C. Mast cell disorder
eyes, and the GI tract – predominantly the small D. Gastrinoma
bowel. Often it is characterized by deep, cra- E. Gastrointestinal stromal tumor (GIST)
tered ulcerations which may result in bleeding or
perforation. Treatment can involve steroids or CORRECT ANSWER: C
immunosuppressive therapy.
An acute bleed from small bowel Crohn’s dis- RATIONALE
ease (A) is uncommon and usually characterized This patient has mast cell disorder (C), a sys-
by punctate ulcerations. Surreptitious NSAID temic disorder characterized by the expansion
(C) use is a common cause of gastrointestinal of mast cells into various organs. A significant
bleeding, but would not be associated with extra portion of patients with mast cell disorder have
luminal ulcerations seen on this patient. Mast gastrointestinal symptoms including abdominal
cell disorder (D) can be associated with dif- pain, diarrhea, nausea, and vomiting. Extralu-
fuse ulcerations, but typically is associated with minal symptoms including itching and flush-
abdominal pain and/or diarrhea. Cutaneous ing. The increase in histamine from mast cell
lesions in mast cell disorder typically are macu- activation results in an activated parietal cell,
lopapular cutaneous lesions, not ulcerations of increased acid production, and ultimately gastric
the oral mucosa. Takayasu Arteritis (E) is a fe- and duodenal ulcers. This can be seen frequent-
male predominant disease, particularly of Asian ly in patients with mast cell disorder. Diagnosis
descent, making this diagnosis less likely. is confirmed on biopsy showing KIT/CD117 and
potentially increased tryptase.
REFERENCE Gastric adenocarcinoma (A) usually is a
Griffin JW Jr, Harrison HB, Tedesco FJ et single discreet ulceration rather than multiple
al. Bechet’s disease with multiple sites of superficial. H. pylori (B) is certainly a potential
gastrointestinal involvement. South Med J. cause, but would not explain the extra luminal
1982;75(11):1405-8. symptoms. The lack of esophagitis, duodenal
ulcers, or diarrhea make gastrinoma (D) less
likely. There is not report of a submucosal lesion
Question 19 to suggest GIST (E).
A 60-year-old Caucasian female presents to your
office for ongoing abdominal discomfort. For the REFERENCE
past five years she has had intermittent abdomi- Jensen RT. Gastrointestinal abnormalities and
nal pain, particularly triggered by alcohol and involvement in systemic mastocytosis. Hematol
non-steroidal anti-inflammatory drugs. This is Oncol Clin N Am. 2000;14(3):579-623.
occasionally associated with nausea and vomit-
ing, and on full review of symptoms she reports
some flushing and diffuse itching intermittently Question 20
over the past few years. WBC and liver enzymes A 43-year-old female with primary systemic
are normal on blood testing. She undergoes an scleroderma presents to the gastroenterologist
upper endoscopy and you find scattered ulcer- for multiple gastrointestinal complaints. For the
ations throughout the stomach, but no changes past year, she has had bloating, increased gas
508 Digestive Diseases Self-Education Program®

discomfort, and loose stools. Infectious workup A. Small intestinal bacterial overgrowth
is negative. Endoscopic evaluation with duode- B. Small bowel lymphoma
nal biopsies is negative. A lactulose breath test is C. Refractory celiac disease
positive. Which of the following lab abnormalities D. Inadvertent gluten exposure
is not associated with this entity:
CORRECT ANSWER: D
A. Folate deficiency
B. Vitamin A deficiency RATIONALE
C. B12 deficiency This patient is a classic demographic patient for
D. Vitamin D deficiency celiac disease and had both positive serologic and
E. Low albumin histologic testing. In addition, they responded well
to initial treatment with a normalization of the
CORRECT ANSWER: A TTG-IgA. The most common cause of an elevated
TTG-IgA in a patient with celiac disease is dietary
RATIONALE indiscretion – either inadvertently or on purpose.
Small intestinal bacterial overgrowth (SIBO) is Small bowel lymphoma (A) is a known com-
a pathologic overgrowth of bacteria in the upper plication of long term celiac disease and would be
small intestine. A lactulose or glucose breath test unlikely at this time. Refractory celiac disease (C)
is the diagnostic test of choice for SIBO. SIBO can refers to persistent TTG-IgA elevation despite ad-
result in significant vitamin deficiencies including herence to a gluten free diet for six or more months.
fat soluble vitamins (A, D, E, K and B12). It may This patient previously responded to a gluten free
also be associated with protein losing enteropathy, diet making this diagnosis less likely. While SIBO
resulting in a low albumin. Folate (A) tends to be (A) does have a higher prevalence in patients with
increased due to bacterial production of folate, celiac, it will not elevate the TTG-IgA.
rather than a deficiency.
REFERENCES
REFERENCE Abdallah H, Leffler DA, Dennis M et al. Refrac-
Khoshini R, Dai SC, Lezcano S et al. A systematic tory celiac disease. Curr Gastroenterol Rep.
review of diagnostic tests for small intestinal bacte- 2007;9(5):401-5.
rial overgrowth. Dig Dis Sci. 2008; 53:1443-1454. Di Sabatino A, Corazza GR. Coeliac disease.
Lancet 2009;373(9673):1480-93.

Question 21
A 23-year-old female from the Middle East pres- Question 22
ents to your office to establish care for her celiac A 45-year-old female with past medical history of
disease. Upon reviewing her records, you see that anti-phospholipid syndrome and subsequent acute
she had TTG-IgA testing with a normal IgA level mesenteric ischemia presents to establish GI care.
and subsequent duodenal biopsies confirming the One year ago, she had an episode of acute mes-
diagnosis. Since her diagnosis two years ago, the enteric ischemia requiring resection of 140 cm of
patient reports maintaining a strict gluten free small bowel. She currently has an end ileostomy.
diet. Subsequent testing after starting this diet For the past three months, she has had significant
showed a normal TTG-IgA level. For the past ostomy output of over three liters per day despite
three months, she has increasing bloating and trying loperamide, atropine/diphenoxylate and a
loose stools. You repeat her TTG-IgA and find tincture of opium. She has been hospitalized twice
that it is elevated. The most likely cause of the for dehydration in this time period. She has had
patient’s increased TTG-IgA level is: negative infectious workup and negative endos-
Chapter 17 — Small bowel disease 509

copy and ileoscopy with biopsies. Which of the (baseline 13.8) with an MCV of 78 fl. Iron studies
following medications is most indicated for this demonstrate a low total iron and low ferritin. An
patient: endoscopy and colonoscopy are normal. On video
capsule endoscopy, you see an approximately
A. Tyrosine kinase inhibitor 3 cm, non-mucosal mass in the jejunum. The
B. Glucagon-like peptide-2 agonist lesion is surgically excised and no spread to the
C. Anti IGE humanized monoclonal antibody lymph nodes or metastasis to the liver is seen. On
D. Cromolyn sodium pathology review, the lesion tests positive for KIT
E. Anti-tumor necrosis factor therapy (CD 117 leukocyte antigen). What is his diagnosis:

CORRECT ANSWER: B A. Carcinoid tumor


B. Small bowel adenocarcinoma
RATIONALE C. Melanoma of the small bowel
This patient has short gut syndrome with in- D. Kaposi sarcoma
creased output that is not controlled with con- E. Gastrointestinal stromal tumor (GIST)
servative measures. The next step in therapy
is a glucagon-like peptide-2 agonist (B) such CORRECT ANSWER: E
as teduglutide. These agents are indicated for
patient with short bowel syndrome with intestinal RATIONALE
failure. Teduglutide is thought to stimulate muco- This patient most likely has a GIST (E). GIST are
sal growth and to promote intestinal absorption, tumors of the mesenchymal origin which are often
and has been shown to decrease stool output and found in individuals during their sixth decade of
improve quality of life life. GISTS predominantly are found in the stom-
Tyrosine kinase inhibitors (A), such as ima- ach, small intestine, or rectum usually presenting
tinib, can be used for GIST. Both Anti IgE human- with bleeding, intussusception, or abdominal pain.
ized monoclonal antibody therapy (C) such as Diagnosis is made by staining for KIT/CD117 on
omalizumab and cromlyn (D) are used to control histology. Treatment is predominantly surgical
the diarrhea associated with mast cell disorder. resection, potentially followed by Imatinib, a tyro-
Anti TNF therapy (E) such as Infliximab is used in sine kinase inhibitor.
Crohn’s disease among other diseases, but has no Carcinoid tumor (A) can be found in the distal
role in short bowel syndrome. small bowel and is often associated with chronic
flushing and/or diarrhea. Small bowel adeno-
REFERENCE carcinoma (B) are mucosal lesions, not submu-
Jeppesen PB, Pertkiewicz M, Messing B et al. cosal, and occur more frequently in patients with
Teduglutide reduces need for parenteral sup- Crohn’s disease or other chronic inflammatory
port among patients with short bowel syndrome states. Melanoma (C) of the GI tract can often be
with intestinal failure. Gastroenterol. 2012 Dec; found in the small bowel or the rectum, but test
143(6):1473-81. positive for PS-100 and HMB-45. Kaposi sarcoma
(D) usually presents in patients with CD 4 count
less than 200 and appears as a hyper pigmented,
Question 23 mucosal lesion on endoscopy.
A 58-year-old male with past medical history of
well controlled HIV (CD4 greater than 500, Nega- REFERENCE
tive Viral Load) is referred to your office for iron Miettinen M, Lasota J. Gastrointestinal stromal
deficiency anemia by his primary care physician. tumors. Gastroenterol Clin North Am 2013;
He is found to have a hemoglobin of 10.4 g/dL 42:399-415.
510 Digestive Diseases Self-Education Program®

Question 24 managed her short gut syndrome with TPN for


A 24-year-old female with past medical history of three years. However, over the past year, she has
protein C deficiency on coumadin who presents had three separate bacterial infections from her
with severe abdominal pain for six hours. On PICC line, the most recent of which required a
presentation, she is afebrile with a heart rate of prolonged ICU stay for septic shock. The patient
130 bpm and blood pressure of 100/60 mm Hg. asks you about small bowel transplantation.
On exam, she has a diffusely tender abdomen to When counselling the patient on potential small
mild palpation and appears very uncomfortable bowel transplant, which of the following is infor-
squirming on the bed. On labs, her WBC is nor- mation is appropriate to tell the patient:
mal, her INR is 1.1, and her lactate is 4.6 mmol/L.
On cross sectional imaging, she has nonspecific A. The patient does not have an appropriate
changes in the small bowel. The next step in her indication for small bowel transplant
therapy is: B. Acute rejection is an uncommon experience in
patients with small bowel transplant
A. Exploratory laparoscopy C. Common post-transplant complications might
B. Start IV heparin drip include infections (i.e. CMV) and post-
C. Percutaneous angioplasty transplant lymphoproliferative disorder
D. Total parenteral nutrition (TPN) D. The life expectancy the first year after
transplant is less than 25 percent
CORRECT ANSWER: A
CORRECT ANSWER: C
RATIONALE
This patient has acute mesenteric ischemia, which RATIONALE
is a medical emergency. On exam she has peritone- Small bowel transplant is indicated for patients
al signs and an elevated lactate, which is suggestive with intestinal failure and resulting nutritional
of some component of bowel ischemia. The next and metabolic complications. This patient has
step in management is exploratory laparoscopy and intestinal failure secondary to short gut syndrome
potentially surgical revascularization if needed. with failure of total parenteral nutrition due to
Anticoagulation (B) is not used routinely in the complications of TPN. She appears to be a suit-
urgent management of acute mesenteric ischemia. able candidate for short bowel transplant.
Percutaneous angioplasty (C) is used in chronic When counselling patients about small bowel
mesenteric ischemia, but has limited role in acute transplant post-operative complications and the
mesenteric ischemia. While this patient may importance of immunosuppression need to be
ultimately require TPN (D), surgical intervention emphasized.
is necessary at this time. Post-transplant, patients are placed on im-
munosuppression often with tacrolimus and
REFERENCE prednisone. If no anastomotic leak is present on
American Gastroenterological Association Medical imaging, enteral feeding is introduced about one
Position Statement: Guidelines on Intestinal Isch- week after surgery. The major complications post-
emia. Gastroenterology. 2000;118(5):951-953. transplant are acute rejection (79 percent of pa-
tients), chronic rejection (13 percent of patients),
infectious complications such as CMV enteritis,
Question 25 and post-transplant lymphoproliferative disor-
A 52-year-old female patient with past medical der. Life expectancy at one year post transplant is
history of short gut syndrome presents to your between 75-85 percent, while long term survival at
office to discuss small bowel transplant. She had five to ten years is approximately 60 percent.
Chapter 17 — Small bowel disease 511

REFERENCE celiac disease (NRCD). NRCD is a clinical diag-


Sudan D. Long-term outcomes and quality of life nosis defined by the persistence or recurrence of
alter intestine transplantation. Curr Opin Organ signs, symptoms and/or laboratory abnormali-
Transplant. 2010;15(3):357-60. ties consistent with active celiac disease despite
treatment with a gluten-free diet for at least six
months. There are multiple causes of NRCD. The
Question 26 most common include small intestinal bacterial
A 36-year-old woman was diagnosed with ce- overgrowth (SIBO), microscopic colitis (diagnosed
liac disease 24 months ago after presenting with by ileocolonoscopy), refractory celiac disease
symptoms of abdominal bloating and intermittent, (RCD) and food intolerances. More rare causes for
non-bloody diarrhea. At diagnosis, anti-tissue NRCD include tropical sprue, inflammatory bowel
transglutaminase (anti-tTG IgA) levels were mark- disease and malignancy.
edly elevated, and confirmatory testing with upper Microscopic colitis has a known association
endoscopy with biopsies revealed complete villous with celiac disease and colonoscopy with biopsies
atrophy with increased intraepithelial lympho- would be recommended if celiac serologies and
cytes. She reports adherence to a gluten-free diet. upper endoscopic findings were normal in the set-
However, on follow-up she reports persistence of ting of persistent diarrhea.
initial symptoms with minimal improvement and RCD is defined as persistent clinical symptoms
a persistently elevated IgA anti-tTG levels at 50 U/ and villous atrophy on histology despite strict
mL (normal less than 4 U/ mL). A repeat endos- adherence to a gluten-free diet for at least 12
copy reveals mucosal fissuring and scalloping with months. Refractory celiac disease (RCD) tends to
histology shows Marsh 3b subtotal villous atrophy. affect older adults (particularly those diagnosed
after the age of 50) and is classified as type 1 with
Which of the following is the most appropriate normal intraepithelial T lymphocytes or as type 2
next step in the evaluation? with aberrant intraepithelial T lymphocytes. Dif-
ferentiation of type 1 versus type 2 can be made by
A. Ileocolonoscopy with biopsies performing flow cytometry to look for the preva-
B. Careful evaluation of gluten-free diet with a lence of abnormal intraepithelial T lymphocytes
registered dietician with greater than 20 percent aberrant lympho-
C. Wireless capsule endoscopy cytes present in type 2 RCD. MR enterography and
D. Flow cytometry of small bowel biopsies capsule endoscopy are imaging modalities which
E. Magnetic resonance enterography may be useful to evaluate the remainder of the
small bowel for etiologies if celiac disease was well
CORRECT ANSWER: B controlled, or if there was suspicion of malignancy
or more distal small bowel lesions in RCD.
RATIONALE
It is important to remember that if a patient does REFERENCES
not respond to the GFD clinically or continues to Tack GJ, Verbeek WHM, Schreurs MWJ, et al. The
have elevated IgA anti-TTG, the most likely cause spectrum of celiac disease: epidemiology, clini-
is lack of adherence to the diet and/or inadver- cal aspects and treatment. Nat Rev Gastroenterol
tent exposure to gluten. Once gluten ingestion is Hepatol. 2010;7(4):204-13.
ruled out, up to 30 percent of patients with celiac Rostom A, Murray JA, Kagnoff MF. American
disease either fail to fully improve on a gluten- Gastroenterology Association (AGA) Institute
free diet or have recurrent signs and symptoms Technical Review on the Diagnosis and Man-
of celiac disease after initial improvement. These agement of celiac disease. Gastroenterology.
patients are categorized as having nonresponsive 2006;131(6):1981-2002.
512 Digestive Diseases Self-Education Program®

Question 27 Question 28
A 25-year-old woman presents with a five-year A 30-year-old woman was diagnosed with celiac
history of watery, non-bloody diarrhea and recur- disease 36 months ago. She has remained compli-
rent bacterial pneumonias. Laboratory studies ant with a strict gluten-free diet including regular
show iron-deficiency anemia with low 25-OH Vi- visits with a dietician. She is currently asymptom-
tamin D. celiac serologies are negative, and small atic with normalization of tissue transglutaminase
bowel biopsies reveal near total villous atrophy, IgA antibody and duodenal biopsies.
increased intraepithelial lymphocytes and crypt
hyperplasia with absent plasma cells. What is the In addition to advising her to remain on a strict
most appropriate initial treatment strategy to ad- gluten-free diet, her management should include
dress the gastrointestinal symptoms? which of the following?

A. Gamma globulin A. Video capsule Endoscopy


B. Infliximab B. Colonoscopy
C. Budesonide C. Lactulose breath testing
D. Gluten-free diet D. Pneumococcal vaccination
E. Rifaximin E. Anti-endomysial antibody testing

CORRECT ANSWER: C CORRECT ANSWER: D

RATIONALE RATIONALE
This patient has gastrointestinal manifestations This patient’s celiac disease is well-controlled on
of common variable immune deficiency (CVID), a strict gluten-free diet. With normalization of her
which can present similarly to celiac disease. tissue transglutaminase antibody levels and duo-
Histologically, intestinal biopsies will reveal villous denal biopsies have normalized. Given this, there
atrophy, crypt hyperplasia and intraepithelial lym- is no role for surveying her small bowel with video
phocytosis similar to celiac disease. However, while capsule endoscopy to look for ulcerative jejunitis
plasma cells are increased in celiac disease, they are or lymphoma. In the absence of current diarrhea
absent in common variable immune deficiency. to suspect concomitant microscopic colitis, there
The initial treatment strategy for CVID typi- is no indication for colonoscopy. Although small
cally includes oral corticosteroids, either pred- intestinal bacterial overgrowth can be seen with
nisone or budesonide, while infliximab or other celiac disease, she has no current symptoms to
anti-tumor necrosis factor agents are reserved for suggest this diagnosis. With an established diag-
steroid-dependent or refractory disease. Other nosis of celiac and a normalized tissue transglu-
options including gluten-free diet and intravenous taminase antibody, there is no role for additional
immunoglobulin therapy do not improve the GI serologic testing. Patients with celiac disease,
symptoms associated with CVID. Rifaximin may even when well-controlled, are at higher risk for
treat any associated SIBO in the setting of CVID pneumococcal pneumonia due to underlying hy-
but is unlikely to cease the diarrhea as bacterial posplenism and hence should undergo vaccination
overgrowth is typically the consequence of the against Streptococcus pneumoniae.
luminal changes not the cause.
REFERENCE
REFERENCE Rubio-Tapia A, Hill ID, Kelly CP, et al. ACG
Agarwal S, Mayer L. Gastrointestinal manifesta- clinical guidelines: diagnosis and management
tions in primary immune disorders. Inflamm of celiac disease. Am J Gastroenterol. 2013 May;
Bowel Dis. 2010;16:703-711. 108(5): 656–677.
Chapter 17 — Small bowel disease 513

Question 29 B. Evaluation of patients on a GFD for more than


Testing for celiac disease (CD) starting with an IgA two months in whom no testing for CD was
tissue transglutaminase antibody is indicated in done before GFD
which of the following scenarios? C. Patients with dermatitis herpetiformis (DH)
D. Patients with suspicion of refractory CD in a
A. Asymptomatic patient with a second-degree patient with previously confirmed celiac
family member with a confirmed diagnosis disease
of CD E. Patients with Turner’s syndrome
B. Elevated serum aminotransferase levels when
no other etiology is found CORRECT ANSWER: B
C. Patients with type 2 diabetes mellitus
D. Parkinson’s disease RATIONALE
E. Adrenal adenoma TTG-IgA may be negative in a patient with
biopsy findings typical of CD if they have IgA
CORRECT ANSWER: B deficiency. Here follow-up testing with IgG
based serological tests would be recommended.
RATIONALE In a patient on a GFD for more than one month,
Patients with unexplained elevation of liver enzymes serology may turn negative. While a gluten chal-
should be assessed for CD. Gluten-dependent hy- lenge is the gold standard for the diagnosis of
pertransaminemia will normalize in most patients CD in a patient on a GFD, HLA DQ 2/8 test-
(greater than95 percent) on a GFD while rarely, CD ing if negative can help to rule out CD before a
can be associated with severe liver disease. gluten challenge. Most patients with dermatitis
Testing of truly symptomless first-degree rela- herpetiformis have evidence of villous atrophy
tives is reasonable but controversial while it is although not all patients manifest gastrointesti-
definitely not supported for asymptomatic second nal symptoms. Therefore, any patient diagnosed
degree relatives. Finally, testing for celiac disease with DH should be evaluated for celiac disease
is recommended for patients with Type 1 diabetes by serologic testing. T-cell receptor rearrange-
mellitus (not type 2), epilepsy (not Parkinson’s ment studies on duodenal biopsies would be the
disease) and autoimmune thyroid diseases (not next step in a patient with suspected refractory
adrenal adenoma). CD with previously confirmed CD. There is a role
for HLA DQ2/8 testing patients with Down’s
REFERENCE syndrome. The prevalence of CD among persons
Rubio-Tapia A, Hill ID, Kelly CP, et al. ACG affected by Down’s syndrome has been reported
clinical guidelines: diagnosis and management to be around 10 percent in the United States
of celiac disease. Am J Gastroenterol. 2013 May; with most children with Down’s syndrome and
108(5): 656–677. CD testing positive for either HLA-DQ2 or DQ8.
Hence, testing negative for both HLA-DQ2 and
DQ8 can reassure most parents of children with
Question 30 Down’s syndrome about the absence of genetic
In which one of the following clinical scenarios is risk for CD development.
human leukocyte antigen DQ2/DQ8 genotyping
testing recommended to effectively rule out REFERENCE
celiac disease: Rubio-Tapia A, Hill ID, Kelly CP, et al. ACG
clinical guidelines: diagnosis and management
A. Patients with Marsh III lesions with negative of celiac disease. Am J Gastroenterol. 2013 May;
IgA anti-TTG 108(5): 656–677.
514 Digestive Diseases Self-Education Program®

Question 31 ter experience. Clin Gastroenterol Hepatol.


A 55-year-old woman presents with a one-year his- 2007;5(11):1282-90.
tory of large volume foul-smelling stools that float Montalto M, D’Onofrio F, Santoro L, et al.
in water associated with 40-pound weight loss. Autoimmune enteropathy in children and adults.
Laboratory evaluation reveals low vitamin A and D Scan J Gastroenterol. 2009;44(9):1029-36.
levels. An upper endoscopy with duodenal biopsies
reveals complete villous blunting with decreased
goblet and Paneth cells, absence of surface intraepi- Question 32
thelial lymphocytes and increased crypt apopto- A 42-year-old woman presents with diarrhea,
sis. She denies non-steroidal anti-inflammatory which she has had for eight months with five
drug use, celiac serologies were not elevated and a loose, non-bloody bowel movements per day
glucose hydrogen breath test was negative. She also without weight loss. Her past medical history is
has coexisting rheumatoid arthritis and multiple significant for hypertension, gastroesophageal
sclerosis. Which of the following tests can help reflux disease and hyperlipidemia treated with
establish the diagnosis? metoprolol, olmesartan, pantoprazole, aspirin and
atorvastatin for the past two years. Labs includ-
A. Anti-nuclear antibodies ing anti-endomysial and IgA anti-TTG antibodies
B. Anti-smooth muscle antibodies were both negative, and serum immunoglobulin
C. Anti-gastric parietal cell antibodies levels were normal. An upper endoscopy with
D. Anti-enterocyte antibodies duodenal biopsies demonstrated villous blunting
E. Anti-thyroglobulin antibodies and a mild increase in intraepithelial lymphocytes
while a colonoscopy with terminal ileum inspec-
CORRECT ANSWER: D tion was grossly normal, as were random colon
biopsies. Fecal fat and electrolytes were normal as
RATIONALE was an MR enterography of the abdomen and pel-
Autoimmune enteropathy (AIE) is characterized by vis. A glucose hydrogen breath test was negative.
a severe malabsorption and secretory diarrhea, and Which of the following is the most appropriate
is differentiated from celiac disease on small bowel next step in his management?
biopsy by the decreased numbers or absence of sur-
face intraepithelial lymphocytes, apoptotic bodies A. Stop her olmesartan
present in the intestinal crypts and absent goblet B. Initiate a gluten-free diet
and Paneth cells. Patients with AIE may also carry C. Prescribe a 10-day course of rifaximin
other autoimmune conditions such as rheumatoid D. Stop pantoprazole
arthritis and multiple sclerosis. Mayo group has E. Prescribe a six-week course of budesonide
published a set of diagnostic criteria based on their
case series of adult AIE which requires a rule out of CORRECT ANSWER: A
other causes of chronic diarrhea in adults, specific
histology supportive of AIE, presence of malabsorp- RATIONALE
tion and ruling out other causes of villous atrophy. This patient’s enteropathy is most likely due to
The presence of anti-enterocyte or anti-goblet cell olmesartan, a known cause of enteropathy. Given
antibodies are supportive of a diagnosis of AIE, extensive testing that has ruled out celiac disease,
their absence does not exclude the diagnosis. inflammatory bowel disease, SIBO and micro-
scopic colitis, initiating a gluten free diet, a course
REFERENCES of rifaximin, stopping pantoprazole and a six-week
Akram S, Murray JA, Pardi D, et al. Adult au- course of budesonide would not be the most ap-
toimmune enteropathy: Mayo Clinic Roches- propriate next step in her management.
Chapter 17 — Small bowel disease 515

REFERENCE can lead to villous atrophy. The findings of a stric-


Rubio-Tapia A, Herman ML, Ludvigsson JF, et ture suggest against a primary cause such as SIBO
al. Severe sprue-like enteropathy associated with to support the use of Rifaximin although SIBO
olmesartan. May Clin Proc. 2012;87(8):732-8. may coexist in this setting.

REFERENCE
Question 33 Frye JM1, Hansel SL, Dolan SG, Fidler JL, Song
A 34-year-old woman presents for consultation LM, Barlow JM, Smyrk TC, Flicek KT, Hara AK,
for bloating, abdominal pain and diarrhea for the Bruining DH, Fletcher JG. NSAID enteropathy:
past three months after a recent hip fracture. Labs appearance at CT and MR enterography in the age
demonstrate iron deficiency anemia with nega- of multi-modality imaging and treatment. Abdom
tive celiac serologies. Endoscopy reveals normal Imaging. 2015 Jun;40(5):1011-25. doi: 10.1007/
appearance with biopsies revealing partial villous s00261-015-0367-2.
atrophy with increased intraepithelial lympho-
cytes. A CT enterography reveals a diaphragm
like stricture in the proximal ileum. A subsequent Question 34
colonoscopy reveals normal appearing ileum and A 36-year-old woman from Texas presents to the
colon with biopsies from ileum revealing acute office for further evaluation of chronic diarrhea.
ileitis without architectural distortion. She reports a five-month history of post-prandial
diarrhea with foul-smelling stool associated with
Which of the following is the next best step in an unintentional 15-pound weight loss due to lack
management? of appetite, increased abdominal cramping and
bloating. History notes a trip to the Dominican
A. Start gluten-free diet Republic 11 years ago. Notable features on physi-
B. Six-week trial of budesonide cal exam include glossitis, pitting pedal edema,
C. Stop NSAIDs and a distended abdomen with hyperactive bowel
D. Trial of rifaximin sounds. Laboratory testing reveals hemoglobin of
E. Start infliximab 10.2 g/dL, hematocrit 33 percent with an MCV of
110 fL (elevated), folate 2.0 ng/ml and a slightly
CORRECT ANSWER: C decreased albumin of 3.1 g/dL with negative celiac
serologies, stool studies negative for infection, but
RATIONALE positive for fecal fat testing with 20 grams of fat
In a person with negative celiac serologies with- in 24-hours. An upper endoscopy was performed
out conclusive histology to support a diagnosis of with duodenal biopsies showing mild villous
celiac disease, trial of GFD is not recommended. blunting with increased numbers of plasma cells,
Similarly, a trial of budesonide and infliximab is eosinophils, and lymphocytes in the lamina pro-
not appropriate in this setting without findings pria. Which of the following is the most appropri-
to support a diagnosis of Crohn’s disease. The ate treatment?
findings of a diaphragm like stricture is specific to
NSAID enteropathy which is a likely cause given A. Metronidazole
the history of a recent hip fracture. The adverse ef- B. Tetracycline
fects of NSAIDs on the upper gastrointestinal tract C. Folic acid plus B12 plus tetracycline
have been well documented. Data suggest that D. Amoxicillin + clarithromycin
NSAIDs can also be harmful to the small intestine. E. Rifaximin
In addition to causing diaphragm-like strictures,
ulcerations, perforations and diarrhea, NSAID use CORRECT ANSWER: C
516 Digestive Diseases Self-Education Program®

RATIONALE C. Colonoscopy with random biopsies


This represents a case of latent tropical sprue D. Lactulose hydrogen breath testing
presents as it has presented after more than 10 E. Magnetic resonance enterography
years from exposure with symptoms of chronic
malabsorptive diarrhea with associated folic acid CORRECT ANSWER: D
deficiency. Tropical sprue is endemic to the Carib-
bean, southern and Southeast Asia, less common RATIONALE
in Middle east and Africa, with clinical symptoms This patient likely has small intestinal bacterial
similar to small intestinal bacterial overgrowth. overgrowth (SIBO) based on his symptoms, the
Endoscopic features can be similar to celiac dis- slight anemia with an elevated MCV with high B12
ease, but the degree of villous atrophy is often less and normal or high folate levels and risk factors
severe. A three to six-month course of folic acid for SIBO including type 1 diabetes with likely au-
plus tetracycline 250 mg four times daily has been tonomic neuropathy affecting small bowel motil-
shown to completely reverse the endoscopic, his- ity as well as a prior history of radiation therapy
tologic and laboratory abnormalities in addition that may also affect small bowel motility. While
to resolution of the steatorrhea. The other options hydrogen breath testing may help establish the
listed here including metronidazole, amoxicillin diagnosis of SIBO, there is variable sensitivity and
plus clarithromycin and rifaximin are not appro- specificity of the testing with false positive and
priate for tropical sprue as first line agents. false negative test results frequently occurring. An
alternative strategy is to treat empirically with an
REFERENCES accepted antibiotic regimen and assessing re-
Brown IS, Bettington M, Rosty C. Tropical sprue: sponse after the course is completed.
revisiting an under recognized disease. Am J Surg
Pathol. 2014;38:666-72. REFERENCES
Westergaard, H. Tropical sprue. Curr Treat Bures J, Cyrany J, Kohoutova D, et al. Small
Options Gastroenterol. 2004;7(1):7-11. intestinal bacterial overgrowth syndrome. World J
Gastroenterol. 2010;16(24):2978-90.
Abu-Shanab A, Quigley EM. Diagnosis of
Question 35 small intestinal bacterial overgrowth: The chal-
A 58-year-old man with a history of type-1 diabe- lenges persist! Expert Rev Gastroenterol Hepatol.
tes with associated retinopathy and nephropathy 2009;3(1):77-87.
and prior radiation therapy for prostate cancer Khoshini R, Dai SC, Lezcano S, Pimentel M.
presents for evaluation of recurrent episodes of A systematic review of diagnostic tests for small
bloating, burping and excessive flatulence with intestinal bacterial overgrowth. Dig Dis Sci.
three to four bowel movements a day with large, 2008;53(6):1443-54.
foul-smelling stool for the past five months without Jain G, Scolapio J, Wasserman E, et al. Chronic
associated weight loss, night sweats, gastrointes- radiation enteritis: a ten-year follow-up. J Clin
tinal bleeding or fevers. Laboratory testing reveals Gastroenterol. 2002; 35(3):214-7.
hemoglobin of 10.8 g/dL with an MCV 106 fl with
high B12 and high normal folate levels. An upper
endoscopy is unrevealing including biopsies from Question 36
the duodenum. Stool studies are negative for occult A 26-year-old-man with a history of a testicular
blood. Which of the following is the best next step? malignancy with a history of surgery and radia-
tion therapy for this presents for the evaluation
A. Helicobacter pylori stool antigen testing of chronic diarrhea that he has had starting two
B. Tissue transglutaminase antibody testing years after the radiation therapy for several years,
Chapter 17 — Small bowel disease 517

along with progressive pedal edema, shortness of after she returned from a year-long service mis-
breath and ascites. On labs, his albumin is 1.5 g/ sion in Haiti. She has up to five bowel movements
dL and fecal alpha-1-antitrypsin clearance is 400 a day, and she describes her stool as malodor-
mg per 10ml (normal, less than 27mg per 100ml). ous and “greasy,” but not bloody. Her stool has
tested negative for bacterial pathogens, ova and
Which of the following findings would be expected parasites and Clostridium difficile toxin assay on
on upper endoscopy? two separate occasions in the month prior to her
clinic visit. Despite this, her primary physician
A. Diffusely ulcerated, friable mucosa treated her with a 10-day course of ciprofloxacin
B. Scalloped duodenal folds and metronidazole which improved her symptoms
C. Normal small intestinal mucosa but symptoms recurred two months after comple-
D. Multiple jejunal angioectasias tion of antibiotic therapy. Subsequent lab testing
E. Creamy yellow-white jejunal villi revealed negative tissue transglutaminase and
anti- endomysial antibodies. Her hemoglobin was
CORRECT ANSWER: E 10.2 g/ dL with an MCV of 105 fL. Her vitamin
B12 level was 1000 ng/L and folate was 3 ng/mL.
RATIONALE Her liver chemistries and serum creatinine were
This patient has protein-losing enteropathy due normal. An upper endoscopy was grossly normal,
his previous testicular malignancy that required but duodenal biopsies revealed normal villi with
radiation therapy to lymph nodes in his abdo- increased lymphocytes in the lamina propria. A
men leading to secondary lymphangiectasia from colonoscopy performed at the same time including
obstruction of lymphatic ducts in the abdomen. random colonic biopsies and ileal inspection, was
Protein losing enteropathy is characterized by normal.
hypoalbuminemia, which can lead to edema
and ascites due to decreased oncotic pressures, Which of the following is the most appropriate
hypogammaglobulinemia, and fat malabsorp- next step in her management?
tion. Luminal protein loss can be measured by
checking the fecal alpha- 1-antitrypsin clearance A. Initiate a gluten-free diet
as well as identification of the underlying cause. B. Perform HLA haplotyping
Classic endoscopic findings in lymphangiectasia C. Treat with tetracycline and folic acid
include creamy yellow-white jejunal villi indicat- D. Treat with albendazole
ing the dilated lymphatic vessels without evidence E. Perform lactulose hydrogen breath testing
of duodenal scalloping, ulceration, friability or
angioectasias. CORRECT ANSWER: E

REFERENCE RATIONALE
Umar SB, DiBaise JK. Protein-losing Enteropa- While the travel history would suggest a risk
thy: Case Illustrations and Clinical Review. Am J of tropical sprue, the high B12 and response
Gastroenterol. 2010;105:43-49. to ciprofloxacin and metronidazole with re-
currence of symptoms in a patient with a risk
factor for SIBO (long standing poorly controlled
Question 37 type 1 diabetes mellitus) and a relatively normal
A 34-year-old woman with a 20-year history of duodenal biopsies except for increased IELs
poorly controlled type 1 diabetes mellitus comes to would suggest underlying SIBO as the cause of
clinic complaining of several months of bloating, her symptoms. Breath testing would therefore
diarrhea and 10 lb weight loss that began shortly be the next best step.
518 Digestive Diseases Self-Education Program®

In this case, serologically-negative celiac dis- CORRECT ANSWER: D


ease is less likely than tropical sprue, so starting
a gluten- free diet or evaluating further with HLA RATIONALE
haplotyping is not indicated. Though helminthic Current criteria for consideration of small bowel
infection is a consideration, this is less likely transplantation due to complications from TPN
given her negative stool testing, duodenal biopsy include: impending liver failure, two or more
findings and folate deficiencies, so treating with episodes of line-related sepsis in a year, one
albendazole would not be the best choice. episode of line-related fungemia, frequent severe
dehydration, and thrombosis of a major central
REFERENCES venous vein including the subclavian, jugular or
Van Citters GW, Lin HC. Management of small in- femoral. Fatty liver disease from TPN, thrombosis
testinal bacterial overgrowth. Curr Gastroenterol of superficial veins, a single episode of non-fungal
Rep. 2005;7(4):317-20. line sepsis and rare episodes of severe dehydra-
Stotzer PO, Blomberg L, Conway PL, et al. tion do not warrant consideration for small bowel
Probiotic treatment of small intestinal bacterial transplantation which still carries high rates of
overgrowth by Lactobacillus fermentum. Scand J morbidity and mortality.
Infect Dis. 1996;28(6):615-9.
REFERENCE
American Gastroenterological Association Medi-
Question 38 cal Position Statement: Short Bowel Syndrome
A 48-year-old man with short bowel syndrome, and Intestinal Transplantation. Gastroenterology.
as the result of repeated resections for Crohn’s 2003;124:1105-10.
disease, is TPN dependent and is frequently
admitted to the hospital for complications.
Over the past year, he has been in the ICU twice Question 39
for line-related sepsis, one episode related to A 60-year-old man with a 35-year history of
fungemia with septic shock needing ICU-level ileal Crohn’s disease and ankylosing spondylitis
care. He has had recurrent thrombosis in his presents with recurrent diarrhea, increased bloat-
cephalic and basilic veins. He has had one visit ing, cramping, progressive weight loss and lower
to the Emergency Department in the past year extremity edema. Labs are notable for an iron-
with tachycardia, dizziness and azotemia that deficiency anemia, albumin of 1.8 g/dL, serum
correct with IV fluids. Recent labs are notable creatinine 0.8 mg/dL and hypergammaglobu-
for an albumin of 2.1 g/dL, AST 60 U/L, ALT 78 linemia. His serum B12 levels are high while folate
U/L with normal bilirubin and INR of 1.7 and an levels are low.
ultrasound of the liver showing increased echo- He has been treated with several courses of
genicity without cirrhosis. antibiotics for presumed bacterial overgrowth
with only partial relief of symptoms. He undergoes
Which of his complications from TPN warrants an MR enterography that did not show any active
consideration for small bowel transplantation? Crohn’s disease in the small bowel.

A. Fatty liver disease from TPN Which of the following will most likely establish
B. Repeated cephalic and basilic vein thrombosis the underlying diagnosis?
C. One episode of line-related sepsis from non-
fungal organisms A. Anti-enterocyte antibody testing
D. One episode of line-related fungemia B. Duodenal biopsy with Periodic Acid Schiff
E. One episode of severe dehydration staining
Chapter 17 — Small bowel disease 519

C. Rectal biopsy with Congo red staining Ebert E, Nagar M. Gastrointestinal manifestations
D. Fecal calprotectin testing of amyloidosis. Am J Gastroenterol. 2008;103(3):
E. Glucose hydrogen breath test 776-87.

CORRECT ANSWER: C
Question 40
RATIONALE An 81-year-old man is admitted for a post-polyp-
This patient most likely has gastrointestinal (AA) ectomy bleeding after removal of 5 mm polyp with
amyloidosis associated with chronic inflamma- a cold-snare during a surveillance colonoscopy
tory conditions of Crohn’s disease and ankylos- which demonstrated normal appearing mucosa
ing spondylitis. Gastrointestinal amyloidosis can except for the single polyp. He details a history
present with features of dysmotility, including of another episode of post-polypectomy bleed-
gastroparesis, dysphagia, gastroesophageal reflux ing with a prior procedure five years ago. He is
and intestinal pseudo-obstruction or diarrhea otherwise healthy and is on no medications except
with intestinal inflammation and concomitant Aspirin 81 mg which was stopped five days prior
bacterial overgrowth. Symptoms suggestive of a to the procedure. He does not drink alcohol and
protein-losing enteropathy with hypoalbumin- is a non-smoker. Labs demonstrate a hemoglobin
emia, edema, malabsorptive diarrhea, edema, of 10.2 g/dL, with albumin 3.4 g/dl and serum
and pleural/pericardial effusions can also occur. creatinine 1.1 mg/dl.
Diagnosis is made through biopsies ideally from
the duodenum or from the rectum with Congo red Which of the following test is the most appropriate
staining of mucosal biopsies showing the charac- next step to determine the reason for the post-
teristic apple-green birefringence under polarized polypectomy bleeding?
light. In AA amyloidosis – gamma globulins tend
to be elevated due to chronic inflammation from A. Bleeding time
associated conditions such as Crohn’s disease. B. Activated partial thromboplastin time (APTT)
Treatment is usually supportive with anti-diarrhe- C. Fibrinogen level
als, nutritional supplementation and addressing D. Rectal biopsy at the time of relook colonoscopy
the underlying condition. with Congo-red staining
Other options such as anti-enterocyte antibody E. Ristocetin cofactor activity
for autoimmune enteropathy and PAS staining
(for Whipple’s disease) are unlikely to establish CORRECT ANSWER: D
the diagnosis. There is no role for checking a fecal
calprotectin in a Crohn’s patient with a normal RATIONALE
MR enterography due to poor sensitivity for small This patient most likely has senile amyloidosis
bowel inflammation. Finally, the patient may have which is found in 10-36 percent of patients over
underlying small bowel intestinal overgrowth with 80 years old and mainly involves the heart and to a
Crohn’s disease and AA amyloidosis which may be lesser extent the GI tract. In one study, 41-44 per-
diagnosed by the breath test but would not yield cent of elderly patients had amyloid deposits in the
the underlying diagnosis of AA amyloidosis. sub-serosal veins localized to the small and large
intestine. This may present as recurrent gastroin-
REFERENCES testinal bleeding after a routine polypectomy.
Petre S, Sha A, Gilani N. Review article: gastro- Hence option D is the best option. Other options
intestinal amyloidosis – clinical features, di- cover testing for platelet function, von-Willebrand
agnosis and therapy. Aliment Pharmacol Ther. disease (Ristocetin cofactor activity), DIC (fibrino-
2008;27:1006-1016. gen levels) and coagulation cascade defects (APTT).
520 Digestive Diseases Self-Education Program®

REFERENCES the dietary option is not working. There is no evi-


Ebert E, Nagar M. Gastrointestinal manifestations dence that this patient has celiac disease, so there
of amyloidosis. Am J Gastroenterol. 2008;103(3): is no role for a gluten-free diet. A low FODMAP
776-787. containing diet is advised for diarrhea predomi-
Rocken C, Saeger W, Linke RP. Gastroin- nant irritable bowel syndrome and bloating, but
testinal amyloid deposits in old age. Report on not for this disease. Elemental diets effectively
110 consecutive autopsical patients and 98 ret- treat children with eosinophilic esophagitis but
rospective bioptic specimens. Pathol Res Pract. would not benefit patients with lymphangiectasia.
1994;190(7):641-9.
REFERENCES
Freeman HJ, Nimmo M. Intestinal lymphangiec-
Question 41 tasia in adults. World J Gastrointest Oncol. 2011.
A 60-year-old woman with a history of ovarian 3(2):19-23.
carcinoma and a history of multiple abdominal Alfano V, Tritto G, Alfonsi L, et al. Stable rever-
surgeries is referred for evaluation of a one year of sal of pathologic signs of primitive intestinal lym-
progressive peripheral edema with normal car- phangiectasia with a hypolipidic, MCT-enriched
diac and renal function. She has fatigue and non- diet. Nutrition. 2000;16(4):303-4.
bloody diarrhea with pitting peripheral edema.
Her serum albumin is 1.2 g/dl with a fecal alpha-
1-antitrypsin clearance of 100 mg/dl. A recent Question 42
upper endoscopy revealed creamy white-appear- A 21-year-old man presents to the outpatient GI
ing duodenal mucosa, and biopsies show dilated clinic with a six-month history of nausea and vom-
lymphatic vessels. What is the most appropriate iting accompanied by abdominal pain and non-
next step in the management for this patient? bloody diarrhea. He reports episodes of flushing of
the skin and maculopapular rash and worsening
A. Low-fat diet with medium-chain triglyceride of symptoms with consumption of alcohol. Which
supplementation of the following will most likely to establish the
B. Gluten-free diet with vitamin supplementation diagnosis?
C. LeVeen shunt
D. Low FODMAP diet A. Skin biopsy of rash
E. Elemental diet B. Bone marrow biopsy with tryptase staining and
detection of aberrant CD25 expression
CORRECT ANSWER: A C. Octreotide scan
D. Serum tryptase level
RATIONALE E. Serum C1 esterase inhibitor level
This patient’s history and findings are consistent
with secondary intestinal lymphangiectasia likely CORRECT ANSWER: B
caused by impaired lymphatic flow. Peripheral
edema, hypoalbuminemia and an elevated stool RATIONALE
alpha-1-anti- trypsin clearance are hallmarks of Systemic mastocytosis is a rare disorder of excess
this disease, and her endoscopic findings sup- mast cell accumulation and activation resulting
port the diagnosis. The primary management of in excess release of vasoactive mediators leading
this condition is a low-fat diet supplemented with to presentation similar to anaphylaxis. Cutane-
medium-chain triglycerides. A peritoneovenous ous manifestations of mastocytosis are common
(LeVeen) shunt is another alternative to decom- including urticarial pigmentosa, which is a diffuse
press the blockage of lymphatic fluid but only if macular or maculopapular rash beginning at the
Chapter 17 — Small bowel disease 521

extremities but progressing to involve the abdo- loss with large, foul-smelling four to five bowel
men and chest. The bone marrow and gastroin- movements per day. Physical examination is no-
testinal tract are common extracutaneous sites table for a febrile, cachectic-appearing male with
for mast cell accumulation – common GI tract numerous patches of hyperpigmentation of the
symptoms include nausea, vomiting and diarrhea skin, multi-site lymphadenopathy and a protuber-
often times related to gastric acid hypersecre- ant abdomen with shifting dullness on examina-
tion and may be increased due to excess circu- tion. He undergoes and EGD with small bowel bi-
lating histamine, which can be associated with opsies where PAS positive foamy macrophages are
gastrointestinal bleeding. Diagnosis of systemic observed. Which of the following additional stains
mastocytosis requires the presence of one major should be requested with the biopsy specimen?
criterion (presence of greater than 15 mast cells as
detected by tryptase staining) in the bone marrow A. Congo-red staining
or extracutaneous organs and one minor criterion B. Quantitation of eosinophils
or three minor criteria. Minor criteria for diagno- C. CD-117 staining
sis include: 1) atypical morphology in greater than D. Acid-fast bacillus staining
25 percent of mast cells, 2) codon 816 mutation of E. Trichrome staining
KIT (the gene for c-kit), 3) mast cells expressing
CD2 and/or CD25 in the bone marrow or extra- CORRECT ANSWER: D
cutaneous organs and 4) serum tryptase levels
greater than20ng/mL. RATIONALE
A skin biopsy alone will not make the diagnosis The patient is presenting with classic signs and
of systemic mastocytosis, while serum tryptase can symptoms of Whipple’s disease.
also be elevated in cases of acute myeloid leuke- Whipple’s disease is due to Tropheryma whip-
mia (AML), chronic myeloid leukemia (CML) and plei, a gram-positive bacillus, more commonly
myelo- dysplastic syndrome (MDS) so its diagnos- affecting Caucasian men between the ages of
tic utility is limited on its own. An octreotide scan 49-55 years of age exposed to the bacteria, which
is used to detect extra-pancreatic neuroendocrine is typically found in sewage, soil or fecal matter
tumors while a serum C1 esterase levels are useful in rural communities. This involves primarily the
in the work up of angioedema. gastrointestinal tract, but has numerous extrain-
testinal manifestations including neurologic,
REFERENCES cardiovascular, pulmonary, ophthalmologic and
Pardanani A. Systemic mastocytosis in adults: musculoskeletal sequelae. Small bowel biopsies
2012 update on diagnosis, risk stratification and should show characteristic foamy macrophages
management. Am J Hematol. 2012;87(4):401-11. with positive periodic acid-Schiff staining. How-
Horny HP, Metcalf DD, Bennett JM, et al. ever, these histologic findings need to be differ-
Mastocytosis. In: WHO classification of tumors entiated, in the appropriate clinical setting, from
of hematopoietic and lymphoid tissues. Swerdlow histologic findings of M. avium complex disease
SH, Campo E, Harris NL, et al. (Eds). Lyon IARC in which PAS- positive macrophages are also
Press, Lyon 2008; 54-63. found. Staining for acid-fast bacilli should differ-
entiate between the two diseases. Additionally, a
positive Tropheryma whipplei polymerase chain
Question 43 reaction testing can also confirm the diagnosis.
A 50-year-old Caucasian man who is a farmer by Trimethoprim-sulfamethoxazole for at least one
occupation, presents with eight-month history of year is the standard treatment regimen, however,
migratory arthralgias of the large joints followed doxycycline and hydroxychloroquine can be used
by more recent onset of headaches, 20 lb. weight for resistant cases.
522 Digestive Diseases Self-Education Program®

REFERENCE The current regimen includes TMP-SMX (one


Bai JC, Mazure RM, Vazquez H, et al. Whipple’s double-strength tablet b.i.d.) for at least one year.
disease. Clin Gastroenterol Hepatol. 2004;2: However, only the SMX component of TMP-SMX
849-60. is effective since T. whipplei lacks the target of
TMP. As a result, alternative regimens have been
proposed for patients with SMX resistance. These
Question 44 include doxycycline (200 mg/day) with hydroxy-
A 50-year-old Caucasian man who works as a chloroquine (600 mg/day).
rural veterinary technician is referred because of It is critical to check the CSF for T. whipplei in
diarrhea, with up to six loose foul-smelling bowel cases of CNS involvement since oral TMP-SMX has
movements per day for the past seven months. to be added to the regimen until the CSF-PCR turns
His stool has been non-bloody. He denies rashes negative owing to doxycycline’s poor CSF penetra-
or eye problems, but he has had significant ar- tion. One of the major concerns of treatment is
thralgias. He has lost 15 pounds and also reports CNS relapse on antibiotic treatment. In fact, up to
having some trouble with memory over this 50 percent of patients may contain the character-
time. Initial lab tests including celiac serologies istic macrophages of Whipple’s disease or PCR-
were negative. Stool cultures, ova and parasite positive material in their CSF in the absence of CNS
evaluation and Clostridium difficile toxin assay symptoms; the positive PCR may indicate remnants
were all negative. of bacterial DNA or colonization.
An upper endoscopy reveals grossly unremark-
able mucosa throughout and duodenal biopsies REFERENCES
are performed. The biopsies demonstrate atrophic Moos V, Schneider T. Changing paradigms in
villi filled with the foamy, PAS-positive macro- Whipple’s disease and infection with Trophy-
phages which are acid fast stain negative along erma whipplei. Eur J Clin Microbiol Infect Dis.
with dilated lymphatics. 2011;30(10):1151-8.
von Herbay A, Ditton HJ, Maiwald M et al.
What additional testing is recommended in the Diagnostic application of a polymerase chain
management of this condition? reaction assay for the Whipple’s disease bacte-
rium to intestinal biopsies. Gas- troenterology.
A. Duodenal biopsy T. whipplei PCR 1996;110(6):1735-1743.
B. Salivary T. whipplei PCR Ramzan NN, Loftus E Jr, Burgart LJ, et al.
C. Stool T. whipplei PCR Diagnosis and monitoring of Whipple disease
D. CSF T. whipplei PCR by polymerase chain reaction. Ann Intern Med
E. Duodenal biopsy fluorescence in-situ 1997;126(7):520-7.
hybridization (FISH)

CORRECT ANSWER: D Question 45


A 20-year-old woman presents to the emergency
RATIONALE department (ED) with a 12-month history of
This patient’s symptoms and histological find- dysphagia, post-prandial epigastric pain, cramp-
ings are consistent with Whipple’s Disease. This ing abdominal pain and bloody diarrhea, with
diagnosis requires identification of periodic acid- frequent ED visits for pain medications and
Schiff (PAS) staining macrophages in the duode- intravenous fluids. Her complete blood count is
nal lamina propria which are AFB stain negative. normal, including white blood cell count, but the
Further PCR analysis can also be used to identify percentage of eosinophils is elevated at 18 percent
RNA of the causative pathogen, T. whipplei. with an elevated absolute eosinophil count. Upper
Chapter 17 — Small bowel disease 523

endoscopy reveals a furrowed esophagus with anecdotal success. There are now case reports and
concentric ring like appearance, mild erythema in abstracts reporting the possible efficacy of anti-
the antrum and a nodular, erythematous duode- alpha4-beta7 Integrin (vedolizumab) therapy in
num. The colonoscopy reveals ulcerated, inflamed ameliorating eosinophilic esophagitis. However,
mucosa in the recto-sigmoid. Antral biopsies are more data in large patient groups is necessary be-
negative for Helicobacter pylori and duodenal bi- fore advocating vedolizumab for the management
opsies reveal villous blunting with greater than 50 of eosinophilic gastroenteritis.
eosinophils per high-powered field. Additionally,
biopsies from the esophagus and recto-sigmoid REFERENCES
also demonstrate greater than 30 eosinophils per Sampson HA, Sicherer SH, Birnbaum AH. AGA
high-powered field. technical review on the evaluation of food allergy
in gastrointestinal disorders. Gastroenterology.
Which of the following would you recommend 2001;120(4):1026-40.
next for this patient? Nhu QM, Chiao H, Moawad FJ, Bao F, Konijeti
GG. The Anti-α4β7 Integrin Therapeutic Antibody
A. Cromolyn sodium for Inflammatory Bowel Disease, Vedolizumab,
B. Diphenhydramine Ameliorates Eosinophilic Esophagitis: a Novel
C. Vedolizumab Clinical Observation. Am J Gastroenterol. 2018
D. Elemental diet Aug;113(8):1261-1263.
E. Corticosteroids

CORRECT ANSWER: E Question 46


A 47-year-old woman with an 18-year history of
RATIONALE ileal Crohn’s disease presents for post-operative
The patient likely has eosinophilic gastroenteri- follow- up. Approximately two years ago, she
tis, which can present with esophageal, gastric, had a second laparoscopic ileocecal resection for
duodenal and colonic disease with dysphagia, fistulizing Crohn’s disease refractory to inflix-
abdominal pain, diarrhea, malabsorption and imab therapy with therapeutic infliximab levels
abdominal pain or obstruction. However, other and absence of anti-infliximab antibodies. At
conditions may be associated with an eosinophilic the time of surgery, she had multiple areas of
infiltrate on mucosal biopsies such as a parasite strictured distal and terminal ileum with inter-
infection, medication-related mucosal injury (e.g. loop fistulae. Approximately 25 cm of ileum were
NSAIDs), malignancies such as lymphoma and resected while 30 cm were resected at the first
gastric cancer, and inflammatory bowel disease. surgery along with the cecum. Post-operatively,
Peripheral eosinophilia and elevated serum IgE she was started on vedolizumab without adverse
levels are typically associated with the disease but reactions. Her post-operative colonoscopy is six
are not always present. Biopsies are required to months after surgery was unremarkable. Cur-
establish the diagnosis; however, the eosinophilic rently, she notes five to six watery, post-prandial
infiltration can be patchy and multiple biopsies bowel movements daily including one to two
throughout the suspected area are needed. With nocturnal bowel movements. She also has near
the luminal form of the disease, an eosinophil daily abdominal pain and bloating after meals
count greater than 20-25 per high power field in that resolves after bowel movements. Her he-
mucosal biopsies establishes the diagnosis. moglobin is 10.6 g/ dL and C-reactive protein
Corticosteroids are the most successful ap- 10 mg/dL. An MR enterography shows 20 cm
proach to therapy as elimination diets, cromo- of active inflammation consistent with Crohn’s
lyn, and leukotriene inhibitors have had limited disease in the neo-terminal ileum.
524 Digestive Diseases Self-Education Program®

Which of the following would you recommend Question 47


next in the management? A 24-year-old male of Turkish descent presents
for evaluation of new-onset diarrhea in the setting
A. Switch to ustekinumab of recurrent oral and genital ulcers. The lesions
B. Add medium-chain triglycerides take anywhere from one to two weeks to resolve,
C. Add cholestyramine and he has multiple areas of genital scarring from
D. Add nortriptyline healed ulcers. He also reports recurrent episodes
E. Add eluxadoline of redness and pain in the eyes with migratory
arthritis of his knees, ankles and wrists, as well as
CORRECT ANSWER: A tender violaceous nodules on the extensor aspects
of his lower extremities. Over the past few weeks,
RATIONALE he additionally reports four to five bloody, diar-
In this patient with prior resections for ileal rheal bowel movements daily. His labs reveal iron
Crohn’s disease, current symptoms, CRP and MR deficiency anemia along with a C-reactive protein
enterography are consistent with active disease at 10 mg/dL. Colonoscopy demonstrates terminal
despite current therapy with vedolizumab. She ileal and ascending colon ulcerations, approxi-
has also been previously treated with infliximab mately 0.5-1 cm in size, with normal intervening
where the disease progressed despite therapeutic mucosa. A pathergy test is positive. He responded
drug levels. In this situation, the next best step in well to an induction regimen of corticosteroids
management is to switch therapy to ustekinumab with immediate response.Which of the following
which is an IL 12/23 antagonist. In patients with agents would you recommend next?
ileal resection of less than 100 cm where the ileum
is connected to an intact colon, this is associ- A. Sulfasalazine
ated with bile acid wasting, leading to a secretory B. Azathioprine
diarrhea called choleretic enteropathy which can C. Methotrexate
be treated with bile acid binding agents such as D. Tacrolimus
cholestyramine. However, in this case, that would E. Vedolizumab
not be the next best step in management. Resec-
tions of greater than 100 cm of ileum leads to CORRECT ANSWER: B
insufficient bile acid concentrations, fat malab-
sorption and fat-soluble vitamin deficiency where RATIONALE
cholestyramine may worsen the malabsorption. Behcet’s disease (BD) is a necrotizing vasculitis
Agents such as nortriptyline maybe useful to treat of unknown etiology that predominantly affects
abdominal pain in a Crohn’s disease patient with young males. It is characterized by oral and genital
IBS like symptoms in the absence of active dis- ulcerations, uveitis and skin lesions. GI ulceration
ease while eluxadoline may similarly be useful to with resultant bleeding is common and can affect
manage diarrhea in a Crohn’s disease patient with the small intestine. Other sites of involvement in-
intact anatomy and gallbladder in the absence of clude the terminal ileum, cecum, ascending colon
active disease. and esophagus. Pathergy test is positive in some
patients: 8.6 percent (in India) to 70.7 percent
REFERENCES (in China). As underlined in the guidelines of the
Lichtenstein GR, Loftus EV, Isaacs KL, Regueiro European League Against Rheumatism (EULAR)
MD, Gerson LB, Sands BE. for the management of BD, evidence-based rec-
ACG Clinical Guideline: Management of ommendations regarding GIBD are not provided
Crohn’s Disease in Adults. Am J Gastroenterol. due to the limited number of published clinical
2018 Apr;113(4):481-517. trials. Medical treatment such as corticosteroids,
Chapter 17 — Small bowel disease 525

sulfasalazine, and azathioprine are capable of autoimmune complex, small vessel vasculitis, with
inducing remission without the need for surgery multiorgan involvement. It is characterized by a
in many patients, whereas TNF-a antagonists and classic tetrad of nonthrombocytopenic palpable
thalidomide have proven useful in resistant and purpura, arthritis or arthralgias, gastrointestinal
complicated cases. and renal involvement. GI involvement may mimic
Crohn’s disease although the biopsies are usually
REFERENCES diagnostic. Most cases are self-limiting although
Hatemi G, Christensen R, Bang D, et al. 2018 oral steroids are indicated in patients with severe
update of the EULAR recommendations for the colicky abdominal pain usually started as predni-
management of Behçet’s syndrome. Ann Rheum sone or methylprednisolone at 1 to 2 mg/kg per day
Dis. 2018 Jun;77(6):808-818. for one to two weeks and then tapering to a stop in
Lopalco G, Rigante D, Venerito V, et al. Up- next one to two weeks. Steroids may prevent major
date on the Medical Management of Gastroin- complications such as gastrointestinal bleeding
testinal Behçet’s Disease. Mediators Inflamm. or intussusception. Immunosuppressive drugs
2017;2017:1460491. (cyclophosphamide, azathioprine, cyclosporine A,
and mycophenolate mofetil) in combination with
high-dose IV pulse steroids are recommended if
Question 48 there is no benefit from steroids alone.
A 26-year-old Caucasian male presented with
fever and sore throat for five days along with ery- REFERENCE
thematous, nonpruritic rash involving the extrem- Sohagia AB, Gunturu SG, Tong TR, Hertan HI.
ities and arthralgias. He subsequently develops Henoch-schonlein purpura-a case report and
right lower quadrant pain, aggravated with meals, review of the literature. Gastroenterol Res Pract.
and associated with watery diarrhea. Labs show 2010;2010:597648.
showed WBC: 14,900/microL, CRP: 12.6 mg/dL.
An MR enterography showed 20 cm of thickened
ileum. An upper endoscopy showed multiple ero- Question 49
sions in the duodenum and antrum while a colo- A 50-year-old woman with short bowel syndrome,
noscopy showed erythema and inflammation in as the result of repeated resections for Crohn’s dis-
the terminal ileum and cecum. Biopsies from both ease, is TPN-dependent and is frequently admit-
areas demonstrate evidence of leukocytoclastic ted to the hospital for line-sepsis. Despite dietary
vasculitis. Skin biopsy also showed leukocytoclas- alterations, acid-reducing medications and use of
tic vasculitis. Which of the following agents would anti-diarrheal medications such as Imodium and
you recommend next? Lomotil, she continues to have admissions for
severe dehydration.
A. Sulfasalazine
B. Prednisone What would be the next best step in the
C. Azathioprine management?
D. Infliximab
E. Vedolizumab A. Eluxadoline
B. Cholestyramine
CORRECT ANSWER: B C. Teduglutide
D. Octreotide
RATIONALE E. Prednisone
This is a case of Henoch-Schonlein purpura which
is a self-limited, systemic, non-granulomatous, CORRECT ANSWER: C
526 Digestive Diseases Self-Education Program®

RATIONALE unrevealing. A CT enterography reveals a mural


Teduglutide is a novel therapeutic agent which is a based mass in mid-jejunum without evidence of
glucagon-like 2 peptide (GLP-2) analog approved metastatic disease in the abdomen. An antegrade
for use in patients with the short bowel syndrome balloon-assisted enteroscopy reveals a submucosal
(SBS) with intestinal failure which reduces their mass in mid-jejunum with central ulceration with
need for parenteral support. The conventional biopsies revealing KIT (CD117 leukocyte antigen)
management of patients with SBS includes dietary positive spindle cell histology.
manipulation, oral rehydration solutions, an-
tidiarrheal and anti-secretory treatments. When Which of the following is the next best step in the
these are not enough, then use of Teduglutide management for his symptoms?
might improve intestinal structure and functional
integrity by promoting mucosal growth and pos- A. Surgical resection of jejunal mass.
sibly reducing gastric emptying and secretion B. Imatinib mesylate
thereby reducing intestinal losses and promoting C. Rituximab based chemotherapy
intestinal absorption. Treatment with this agent D. Lanreotide
could promote intestinal rehabilitation through E. Prednisone
its intestinotrophic and pro-absorptive effects,
leading to a reduction in diarrhea and reduction CORRECT ANSWER: A
in the need for parenteral support. Somatostatin
and its analog octreotide have been used to mini- RATIONALE
mize refractory high-output stomal fluid losses Gastrointestinal stromal tumors (GIST) are the
by inhibiting gastric, pancreatic, and intestinal most common gastrointestinal tumors of mesen-
secretions. However, somatostatin usage has been chymal origin. These tumors are typically found
found to affect enteral utilization of amino acids in older individuals with the mean age of diagno-
for splanchnic protein synthesis and interfere with sis in the 6th decade. GISTs can occur anywhere
physiologic postresection adaptation. along the gastrointestinal tract, most commonly
found in the stomach, followed by the jejunum/
REFERENCES ileum. GISTs are believed to arise from interstitial
Vipperla K, O’Keefe SJ. Targeted therapy of short- cells of Cajal cells and nearly universally express
bowel syndrome with teduglutide: the new kid mutations in KIT receptor tyrosine kinase (TK)
on the block. Clin Exp Gastroenterol. 2014 Dec or platelet derived growth factor receptor (PDG-
10;7:489-95. FRA) that allow for constitutive activation. These
Jeppesen PB. Teduglutide, a novel glucagon- tumors may be found incidentally or present
like peptide two analog, in the treatment of with abdominal pain, gastrointestinal bleeding
patients with short bowel syndrome. Ther Adv or obstruction. Standard treatment for primary
Gastroenterol. 2012. 5(3): 159-171. and localized GISTs is complete surgical resection
Jeppesen PB. Pharmacologic options for intes- avoiding tumor rupture or injuries to the pseudo-
tinal rehabilitation in patients with short bowel capsule. Lymph node metastasis is rare in GIST
syndrome. J Parenter Enteral Nutr. 2014;38(1 making prophylactic lymphadenopathy unneces-
Suppl):45S-52S. sary in most cases. Nearly 50-60 percent of pa-
tients are cured with complete surgical resection
alone. The remaining 40-50 percent of patients
Question 50 have relapses that require additional targeted
A 65-year-old man is admitted with iron defi- therapy. Imatinib mesylate is a tyrosine kinase
ciency anemia and recurrent episodes of melena. (TK) inhibitor which is standard first-line therapy
An initial upper endoscopy and colonoscopy are for recurrent, metastatic or inoperable GIST. In
Chapter 17 — Small bowel disease 527

the case of progressive disease despite imatinib


therapy, the recommendation is to treat with the
multi-target inhibitor, sunitinib maleate. Third
line therapy for GISTs progressing under both
imatinib and sunitinib treatment is regorafenib,
another multi-target inhibitor. An alternative op-
tion is to rechallenge with imatinib after progres-
sion under sunitinib treatment.

REFERENCES
Miettinen M, Lasota J. Gastrointestinal stro-
mal tumors. Gastroenterol Clin North Am
2013;42:399-415.
Poveda A, del Muro XG, Lopez-Guerrero JA,
et al. GEIS 2013 guidelines forgastrointestinal
sarcomas (GIST). Cancer Chemother Pharmacol
2014;74:883-98.
528 Digestive Diseases Self-Education Program®

You might also like